Sunteți pe pagina 1din 1306

Great Golden Guide6 Th

**
**





$$
:
$$

@4@ !!! @ 4@

OUR GROUPS ON FACEBOOK


PACES Exam Cases [PEC] ... By Dr. Ahmed Maher Eliwa
https://www.facebook.com/groups/695652397155163/11915
23304234734/?notif_t=like&notif_id=1477770924622389
Guidance To PACES Success[GPS] .. By Dr. Ahmed Maher Eliwa
/https://www.facebook.com/groups/285352564831987
The Great Golden Guide
why this specific name???
As we know ,,there are many & many sources for PACES
ALL of them are great & so helpful
Also there are many & different points of veiws regarding:
how to prepare??
how to study???
how to examin???
BUT
But we need the source & the advice that paint to us the right , honest
& the nearest ROAD
At the same time we need the SHORTEST one
So ,, we give you the ( GPS ) & THE ( GGG ) The Great Golden Guide
in your hand in order to shorten your ROAD & JOURNEY to PASS PACES
PLEASE ,,read it carefully
In particular befor your FINAL REVISION
Also you can Browse it whilst you are preparing for PACES
Best wishes for ALL
good luck
Just Pray 4 me
**BEBO BEBO**
The Team Work of GGG
My exam experiences in 4th of July, 2017 at
Mandalay General Hospital.
Station 1
Respiration - Left lung collapse, I dun know
what I missed. 17/20
Abdomen - Hepatomegaly with huge
splenomegaly 20/20
Station 2
20 years old female dance instructor, persistent
diarrhoea for 4 months, LOW (+), Travelling history
to Thailand before diarrhoea episode. Full blood
count, ESR, CRP, Stool culture all normal. Family
history of CA Colon in grandma. GP think of IBS. Dx
- IBS
DDX- Thyrotoxicosis, Coeliac disease, Malabsorption
syndrome, IBD, Giardiasis.
19/20
Station 3
Neuro - Look at the eye and proceed.
Dx Occular myasthenia 20/20
CVS - Mitral stenosis, Aortic regurgitation with Afib
no features of IE, no features of heart failure. 20/20
Station 4
Talk to the daughter about his father condition.
Recently got stroke and now got MRSA infection at
pressure sores of both heels, no signs of clinical
infection.
13/16
Station 5
1) 55 years old female constipation for 3 years
without red flag signs, but mild hypertension on
Thiazides diuretic. No other features of
hypercalcaemia. On examination, no features of
hypothyroid and hypopit, lower midline scar for child
delivery 17 years ago(+) no features of intestinal
obstruction. Family history of CA colon in mother.
Dx hypercalcaemia due to thiazide or primary
hyperparathyroidism
I gave general advice for constipation and refer for
colonoscopy for CA colon screening.
28/28
2) 63 years old lady Uncontrolled diabestes and
hypertension, underlying bronchial asthma on
inhalers, develop white skin patches in lower limbs
and she is taking OTC medications for two weeks for
skin problem. On examination, no physical features
of Cushing's Syndrome, widespread rhonchi (+) on
auscultation of the chest. White patches looks like
Vitiligo.
Dx Uncontrolled diabetes is multifactorial but most
likely because of? Steroid medication
Mx I said Multidisplinary team approach
28/28
Thanks for all in this group. I passed with 165/172.

UK EXPERIENCE
,Glasgow college.
Exam was on 16th july, and fortunately I passed
it.It was excruciating experience to go through
the tormenting phase of PACES prep however in
the end it is very rewarding.
My cases were
St 1
VSD AND PARKINSONISM
14/16
I don't know why I have not received good score in
these two case coz both were straightforward cases
St2
Confusion in a known prostate carcinoma patient; I
gave a differential of
Hypercalcemia,metastasis,delerium
Resp and abd st were ILD and liver transplant
respectively 16/15
St4 convincing a young female for steroid inhaler
St 5 Raynauds phenomena and transient ischemic
attack 27 /27
The key of the exam is to be fluent,must not fumble
and be confident.they often try to check that whether
you are stick with your findings or not and if you are
keep changing your opinion ,that may be proven
detrimental.
I am not going to tell you anything extraordinary,the
most of the things you already know.
during exam we do feel nervous but just trust your
instincts.you have invested so much time and finances
to reach in this state .you can not just screw up
everything by silly mistakes.
accept your limitations,when diagnosis is not obvious
just state it loud and give best possible differentials.
Exam doesn't want you to be sherlock holmes ,just act
as Med registrar,you dont have to be a consultant for
this exam
Thanks this group.
My experiance , mandalay, first day last round
Start with history
25 years old man with recurrent headache,
hypertension, this morning he got glycosuria++.
So came to consult, while waiting , he argued
with nurse n got palpitation, nurse took ECG-
normal. He has history of anxiety n taking dz.
Take history.
In the room, serogate didn't tell more than that
except recurrent sweating. Drug- taking high
dose vit C.
Explain- gland problem . Need to confirm.
Is it curable?-depends on cause. DM?- no.
Examiner- dx- pheo, hyperthyroid, anxiety
Points for pheo?- not sure. Any asso: problem-
no. DM?- no. Why glucosuria?- vit C high dose.
May be anxiety but need to exclufe pheo
I got 20/20

Diet 2 Penang Hospital Malaysia


This group has given much help for the
preparation of PACES. These are my cases
My advice is FOR HISTORY AND COM SKILLS
-try to share out to our fellow frens the
important points in the scenario as
cases are repeating (easier to prepare with
concrete info)
As for short cases, identifying physical signs is
important and comes to a unifying
diagnosis based on findings u get
Station 5
scenario 1- middle aged lady with long standing
rheumatoid arthritis comes with
foot weakness
Scenario 2- 48 year old lady presented with
tremors
Scenario 1
One quick glance: gross deformity of hands, left
foot drop
Hx: weakness over left foot-2-3
months,worsening, loss of sensation over
lateral side of leg
RA for 30 years, no recent
history of trauma
on T methotrexate 15mg/weekly, T prednisolone
10mg OD (meds taken for 2 years)
no osmotic symptoms
staying with daughter, quickly assess home
environment
Physical signs
-walking frame by bedside
-wasting of distal muscles of bilateral lower
limbs
-wasting of small muscles of hand with
Boutoneirre,swan neck,Z deformity,ulnar
deviation
-left lower motor neuron foot drop
-claw toes
-right ankle:Charcot joint (all movements of
ankle restricted,deformed)
-no scars
-unable to invert and evert left foot
-hip flexors and hip adductors power 5/5
-sensation intact
-proprioception loss
-mention want for gait )-stopped by examiners
concern: what is wrong with me?
discussion: how to determine level of lesion in
foot drop, differential diagnosis
I gave: mononeuritis multiplex (due to RA, can
be due to DM-steroid induced, vasculitis),
CIDP, mx-multidisciplinary
Scenario 2
Hx- tremor of hand, left side only, usually at
rest, disappears during action,
no symptoms of hyperthyroid, no improved with
alcohol
no frequent falls
no giddiness upon postural change
no constipation
drug history : antihypertensives only
no family history
working as clerk
Physical signs
-mask liked facies
-resting tremor over left hand
-cogwheel rigidity
-signs more prominent upon distraction
-bradykinesia
-shuffling gait,absence of arm swing
-writing-micrographia
-no cerebellar signs, no pyrimidal signs,no gaze
palsy
-mention to check BP for postural drop
concern: what am i having
impression: idiopathic parkinson disease,Yahrs
grade 1, in view of asymmetrical findings
lack of other signs
* but also want to investigate for other causes of
Parkinsonism in view of her age still
young
questions from examiner- ix for Parkinsons
disease,mx
Abdomen
young lady
inverted J shaped scar over abdomen -->likely
to be renal transplant
non functioning AVF over right arm
multiple previous puncture marks at neck
pink
no Cushingnoid appearance,no gum
hypertrophy
mass about 5x5 cm beneath the scar at the right
iliac fossa,firm,mobile,non tender
able to get above,no bruit
no other organomegaly
no ascites
no pedal oedema
lungs clear
no dextrostix marks, no alopecia,no malar
rashes
requested for BP during end of examination
impression
end stage renal disease,current mode of renal
replacement therapy is right renal
transplant which is functioning well, previous
mode of RRT is hemodialysis
clinically not overloaded and pink.Likely cause
for ESRD would be glomerulonephritis
in view of the age
Questions from examiner- ix and mx
Respiratory
young gentleman
Respiratory rate 20 under nasal prong oxygen
clubbing of fingers
tattoo all over arms and body
thoracotomy scar and previous chest tube scar
over right hemithorax
trachea central
reduced chest expansion right mid and lower
zone
dull percussion right mid and lower zone
bronchial breathing right upper zone with
increased vocal resonance
coarse crepts left lower zone
impression
features of consolidation right upper
zone,bronchiectasis left lower zone,
and right lower lobe lobectomy/decortication
unifying diagnosis considering the geographical
area would be pulmonary TB
questions: ix and mx
History taking(i only write out important points)
30 years old gentleman presented with sudden
loss of vision over right eye
lasted for few seconds only
no other neurological symptoms
no symptoms of connective tissue disease
history of having 'blood clot in the vessel of left
eye' few years back-presented with
loss of vision also
hypertension diagnosed at few years back ( so
rule out all causes of young HPT)
significant family history of stroke
no one in family having recurrent
miscarriages,thrombosis
drug history- started on aspirin few years back-
compliant
smoker-stopped few years ago
driving
Impression
amourosis fugax
Ix
CT brain TRO stroke,ECG for AF, ECHO for LA
size,thrombus, as well as patent foramen ovale
USG doppler carotid
fasting glucose and lipids
Hb to look for polycythemia
thrombophilia screening
Mx
extended release dypridamole/ clopidogrel
warfarin if AF or thrombophilia
advise for driving precaution
CVS
middle age gentleman with SOB
slow rising pulse,low volume
apex beat not deviated
ESM radiating to carotid and apex
no other murmurs
no signs of IE or failure
mention to check for BP
impression -severe AS with Gallavardin
phenomenon
questions on ix and mx
Neuro
middle age gentleman with blurring of vision
left complete ptosis
3rd nerve palsy
pupils dilated
other CN intact
no pyrimidal signs
no dextrostix marks
mention to check BP and ask for any headache
Impression - left surgical third nerve
palsy,lesion -PCOM aneurysm
questions on ix and mx
Com skills
56 years old gentleman came for UGIB and
endoscopy done showed malignant looking
ulcer. Task is to explain findings to patient and
counsel for CT
questions asked by surrogate
Is it cancer?
Am I going to die? how long do I have to live?
straightforward scenario, cant find any hidden
agenda

2/2017 Mandalay General Hospital, Myanmar.


5 July 2nd round
Station 4
24 years old lady history of recent
thyroidectomy for medullary carcinoma of
thyroid and hoarseness of voice after post-op
but recovered completely. Tested MEN 2a gene
which is positive. It's autosomal dominant &
chance of phaeochromocytoma for 50% and
hyperparathyroidism for 30%. No family history
of cancer. Planning to marry soon.
Task- counsel about genetic and family planning.
Concerns - how should she tell her husband?
What about her children future?
How about her parents? Do they need to screen?
Examiner Q:
What does this disease like?
How will the genetic counsellor explain this disease to
this patient?
Does she need to tell her husband?
Station 5 - 1
Old-aged gentleman, arthritis for 3 days.
Inside - Big toe inflammation, tophi present on big
toe, fingers, no fever. History of similar attack last
year but no consultation and no treatment. History
hypertension and Diabetes.
Dx- acute on chronic gout most likely due to
metabolic syndrome.
Station 5 - 2
Young lady, polyarthralgia and low grade fever for 1
month.
Inside- no fever now, history of butterfly rash, oral
ulcers and painful rash on palms last 3 months. No
consultation and treatment taken. Now pregnant and
5-month. No significant signs apart from systolic
murmur.
Station 1
Chest - left lung collapse
Abdo - hepatosplenomegaly with ascites
Station 2
24 years old gentleman with fever and cough for 3
days. On examination BBS is heard in rt lower zone.
History of repeated chest infection.
Inside - features of pneumonia, repeated chest
infection last 3 years, history of tonsillectomy at the
age of ten. History diarrhoea last year diagnosed as
giardiasis. No history of pancreatic insufficiency . No
family history.
Dx - lower respiratory tract infection most propably
due to primary immune deficiency like
hypogammaglobulinaemia and CVID.
DDx - cystic fibrosis, immotile ciliary syndrome,HIV,
bronchiectasis
Station 3
CVS - ?valvular heart disease
CNS - mixed motor and sensory peripheral
neuropathy.
Thanks a lot to this group for sharing experiences. It
does give a great help for me to pass PACES.

Copied from Dr ZAIN group


Thanks GOD for passing paces ,,,, don't loose hope if
delayed accepetance and continous rejection it will
come to be true one day Inshallah.
One advise I will give what I was doing is studying my
colleagues experience , no time for reading books in
and out with heavy duties
Egypt 2017/May 23 Almaadi Military Hospital
I started with station 2#
While waiting i asked myself why they put station 2
my first ststion , if it went bad every thing will go bad
,,, ok i will try to forget about it however it finishes
A 35 yrs old male presented with cough , past medical
history he has multiple sclerosis ,, vitals are stable
I started with opening and history of present
complaint ; cough for the last 1 month ,dry , diurnal
variation? all the day but mostly at night , no wheeze ,
sob ,chest pain ,orthopneae, pnd,leg swelling or
haemoptysis,
any thing that increase it , i gave all allergy risks
negative including pets , relives it ?only partialy
relived by gavascon.no improvement at weekends or
vaccation. May be GORD? So i went to GI questions
all negative except mild heart burn.
I was asking my self why they brought it directly
GORD ,, it came around my mind GORD ass with
weight gain zI directly asked did u gsin any weight?
Surprisingly he said yes ,, non intentional ,no specific
distribution, no Hypothyroid or cushing symptoms,
no proximal myopthy , so why weight gain , i said
leave it time will finish.
I screened other systems , when reached
muscloskletal any weakness right now ( he has MS)
he said no but 2 month back he had weakness , I said
was it MS attack ? he said yes and voulantry told he
was admitted there and apon discharge the doc
changed his medicine to Methylpredinsone tabs 5
days a week
Aha ,,,, and did he give u any medicine for stomach , i
dont remember his answer ,,
,I finished reaching drug history nothing ,, affecting
him badly in job but job not related to cough. 2 min
remaining i was still taking history they remind me
again
CONCERN : do l have aspiration pneumonia? I read
in internet MS causes it am so afraid and soo much
worried he was talkative ,,, i paniced I cant stop him
now
I remebered dont answer concern with managment
plan so I answered NO its not aspiration pneumonia ,
good you dont have sob, fever, etc ,,, I told him breifly
the tabs given for MS caused u weight gain and it
relaxed the stomach opening this why stomach acid
goes up irritate you ,,,,didnt give him mang plan time
finished
1 min reflection : i read wat i wrote in my paper
Examiner questions :
whats his problem : ended with GORd , how? Relax
of sphincter
Why you think its not pneumonia
Investigstion : basic ,,, any thing specific ,,,i was
blocked he said like invasive I said you mean
endoscopy? he said something to confirm , i said
esoph manometry , for what? To check acidity , Then
he asked if this man came to you in ER will you not
do for him CXR ,,,, i realized i forget I said yes but
really i said why CXR! After exam i remembered
chronic cough
DD i listed all starting with GORD , other dd and
what is against
No time for treatment
I got 20
Station 3 # CVS :
Double aortic ,,, with possible mitral regurgitation
Questions:
Findings
Investigations
managment including indications for replacment
1 got 19
#Station 4
From outside long scenario : A young lady with long
standing history bronchail asthma well controlled on
brochodilator had an attack of srver astha wranted
ICU admission and intubation 4 days discharged on
steroid inhaler but her symptoms are uncontrolled
since disharge. The GP sent you the patient because
he feels she is not compliant to inhaler ,,, she also has
eczema since childhood
I started by the opeing as Dr zein taught us , she is a
teacher , no one with her , how she feels now, she has
all symptoms day and night ? And how much she
knows about her condition , she knows everything
,she left nothing to add, , so told you are right and am
sorry for the unfortinate event(icu intubation) i know
it was a hard experience but happy
that you are out of it snd I hope it want happen
again.she said thanks.
So what about the tabs given to you to control your
symptoms , she said no one gave me tabs , i asked
mo one gave you steriod tabs ? she said no ,,, i
looked at the paper coz I prepared my self as
councling for steroid tabs ,, they gave her inhaler ,,,,
so i said the doc prescribed steroid inhaler , she said
yes again I repeated no one wrote for you tabs coz i
was preoccupied by tabs ,, so every thing is changed
now ....
So do you have any problem with the inhaler you take
, any difficulty with technique ,,,I didnt want to say
GP told you are uncomplient ,,, she said am not
taking it ? Any thing nothering you with it? Then she
talked too much about worriness of side effects she
said all side effects ,,, I said i appreciate your
worriness but i want to calm you that inhalers are not
like tabs and this side effects dont happen unless with
long time use , the most thing that happens is throat
infection for which you need to wash mouth after
each use , and the inhelr is important to controll
symptoms and not having a bed experience like
before. I went far that without controlling asthma an
attack may lead to DEATH.
She asked repeatedly of side effects and i told the
same.
Then she said any other ttt thsn this? I said in asthma
we follow a step up method we give simple ttt then
add more potent ,,,, i said if not controlled you may
need to be given higher dose inhaler or steroid tablets
which will have more side effects ,,,,
I asked how do you think about using the inhaler?
She said i will think because you told no side effects
with it ,, i said it can happen minimally with long time
use but they are important to controll symptoms.
She is a teacher lives with her husband who is
supporting her financially ,many sickleaves and her
husband leaves the room at night coz of her coughing
,,, I appreciated this and empathized for that ,,, then i
asked sny other concern : she said no you answered
all concerns ( she asked many questions)
Then I summerized check understanding and closed
,,,, and again said you agreed to take them she said
yes . and advised sgsin on compliance and regular
follow.
They told one & half minute remaining
Ok i will talk about any thing , i told do you have
kids? She said no ,, i told i hope you will get kids
soon
This is non sense , Then asked you have symptoms
right now , she said all the time , i said I will asses you
and check your oxygen and wether you need
emergency ttt right now . She agreed .and we kept
silent waiting for bell.
Examiner question :
Whats her problem
What are ethical issue : autonomy,councling,
empathy
What are her concerns
Did you convince her , i said she told she will use it
If she came to you next time and didnt use inhaler
what you will do? I said I will council her again , if
not?I said I will send her to gp and talk to her
husband!! He said you will talk to her husband? I
said after taikng her permission , and do you think he
will help her? I said yes I forgot he leaves the room at
night,, he asked and how he will help her? I said
he will tell she is important for him I forgot she
told me that he leaves the room for her at night.
Then he said what else? I said GP , do you think GP
will convince her? I said yes gp is the family doctor
and their relation is strong with patients ,,,( i worked
as family doc before) what else? Bell rang ,,, he
wanted to hear consultant and specialised nurse I
think
got 14/16
# BCC1 : A 35 yrs old lady presented with SOB for 1
month or more ,,, All her vitals are given normal
when I entered I great examiners , patient and
surrogate ,, looked at the pt to see if spot diagnosis in
face nothing ,,,
I asked all questions regard SOB it was exertional
now at rest , No symptoms of anaemia, H failure or
other chest symptoms ,,, i was taking a look at the pt
every time when I realised her skin on dorsum of
hands is shiny ? may be scleroderma ,,, fingers were
in slight flexion position ,,, I said i would like to
exsmine ,they said proceed , ehen standing b
eside her I rememberd risk for Pulmonary embolism
all negative
Hands skin was tight dorsum only , fingers were blue
, slightly flexed but not deformed,function good, I
asked about Raynaud , other connective tissues (
mixed, overlap) all negative ,proximal weakness, in
the face normal skin normal mouth opening she can
introduce 3 fingers , i examined the back crackles on
right side only dont change with cough , the examiner
put his stethoscope at the same place I did and said
continue, I checked pulmonary area for PHTN
couldnt hear loud P2 , i finished history, dhe is a
house wife , sob affecting her soo much , I said that is
hard with empathy
CONCERN : will she get back to her normal status
like before? I said am afraid to tell that she will not
return to normal as before , because the SOB is due to
scarring of her lungs ,,, and this related to the change
in her hand skin and also causing the( raynaud) all a
condition called scleroderma ,,, but some medicine
can help to improve her symptoms .... I completed she
needs some test and images and more assesment
better as inpatient
Ex questions : patient problem
Diagnosis , limited SS complicated
What did u find in exam: i said limited skin changes
to dorsum hand , mild sclerodactly , calcinosis ( no
cslcinosis actually) he said look at her fingers they
were cyanosed
Do you want to examine her legs i said no , I should
have checked for skin and pitting edema
Investigations : all
Bell rang got 26/28
BCC2 :
Skin rash macular scaly itchy on the arms, it started
in abdomen i took a look red scally , no joint,nail
affection , no back pain, no ppreceeding sore throat,,
I asked for every thing negative, , I jumped to PMH,
drug history ,HIV risks, social all negative ,then silent
for around a minute , i didnt know what to ask , then
I
Asked any loss of wt blindly,
Surprisingly Pt had weight loss 5 kgs non Intentional,
fever and night sweats
Others negative( searched quickly for infection and
malignancy source as no time),,,I asked to examine
chest they said proceed nothing in chest , I want to
examine lymph node I examined right submandibular
, they said no lymphadenopthy ,,bell rang , 2 min
remaining , I said I would like to examine,abdomen
for hepatospleenomegly ,they said again 2 min
remaining .
Concern : will this rash disappear and improve?
I said am afraid it may not disappear bacause am
thinking of serious cause like growth , i hope its not
the case , we csn give something to help with itching
and need to do some blood test and may go for some
scans to rule out growth.
Examiner q:
Whats his problem?
What is your d or dd ? I said in presence of wt loss ,
fever sweats and this rash I put infection , malignancy
he asked what type of malignancy answered (
haematological & solid) what infection I said like TB
although not sure .
Investigations : no time
I got 28/28
got28/28
Station 1 # Chest : pneumonectomy 20/20
Diagnosis
Causes
Investigations
Treatment , they discussed LOTOT indications
# Abdomen : hepatosplenomegaly , ascites ,signs of
CLD
CLD for differential 20/20
BCC2 raelly i couldnt reach diagnosis coz if not
following the previous feedback from colleagues it
came before as icthyosis with underlying malignancy
,,,
I would like to thank dr.zein Zein Mahgoub for his
great ,unlimited support and the group he created
where all previous feedbacks are shared beside
continuos disccusion ,,, Thanks for all colleagues and
good luck.

Copied from Dr. Zain group


Experience in paces in maadi hospital in
May(EGYPT)2017
Station 5..
1_from out side pt 50 yrs complain of loss of
vision.. Inside she is diabetic and hypertensive.
History going with TIA.. I examined the pulse
and the carotids. I asked to do fundus exam and
cardiovascular the examiner said normal. I
asked about driving not driving..concerned
about is it stroke..I got 21
2_about 60 years male complain of loss of wt.
Inside there is significant wt loss no any other
GIT symptoms..pH of bowel resection and on
replacement for vit B12. On examination midline
laprotomy scar.. Concerned is it cancer igot 18
..
Station 1..
Chest.. Lt sided pleural effusion..
Discussion about causes differential
investigation then bell rang..
I got 20/20
Abdomen.. hepatosplenomegally for
differential diagnosis
18/20
Station3..
CVS .. mitral regurgitation with pulmonary
hypertension and atrial fibrillation.. Discussion
about investigation and management
20/20
Neurology.. Amiddle age male.. Instruction
to examine the lower limbs and tell the positive
findings..there was monoparesis with hyper
reflxia and the planter was equivocal for me.. I
put ms and they ask about the investigations
and the rang..
20/20
Communication.
A young male.. Admitted to the hospital
because of pneumonia. And started to improve
but suddenly developed convulsions.. When
you revised his notes,he is a known epileptic
but not given his medications since the
admission...
Inside.. I introduced my self. I ask how is he
feeling today.. He said IAM not fine.. I said IAM
sorry for that and directly I asked him what he
knows about his condition so far.. He replied
that I was fine.. And admitted to your hospital
because of cough but today I developed
convulsions l don't know why.. Because IAM
compliant to my medicine even in the day of the
admission in the morning I took my medicine
and for the last 7years it didn't happen... I
listened to his story then I replied yes u are right
and directly I said lam sorry to tell that that
happened because it seems that u didn't receive
your medicine because the medical team have
to give you but that didn't happen..he start to
shout how is that.. IAM coming here to be cured
and u r causing me more problem..this is not
acceptable at all..I tried to stay calm and make
him to show his anger.. I said you are right in
every thing u r telling.. And this is our mistake
and I will be thankfull if u accept my apologies
and on the behalf of all the team I repeat the
apology and I hope to accept that.. and I
told him already we wrote something called
incident report and we inform our senior and we
will discuss this in the doctors meeting.. After
repeating more apology he started to show me
his concern which was that he want to make
acomplain.. I replied that your right and will
direct u to the authorized place to make your
complain.. The other concern was the driving
and. When he can drive and that we make the
mistake and at the moment he wants acar with
the driver to to be able to go to his work place
as ( I can not remember but seems something
important). I replied IAM sorry to tell you that u
are adviced to stop driving and to inform the
DVLA.. he asked for how long l said it is months
rather than days I was not sure for how long
exactly.. And regarding the private driver lsaid I
will inform the social worker he will do his best
to help you.. He asked he will give me the car
and the driver l told IAM not sure exactly if
possible to do so but I am sure that he will do
his best to help..Finally he looked convinced.. I
ask him what he get from our talk he replied I
want to complain ...I thank him and
apologized again.. The examiners asked about
the ethical issues and if it is mistake or not..I
replied yes it is.. They ask who is responsible..
Isaid the admitting team of doctors.. And what u
want to do to prevent such event.i repeated
same what I said to the pt...then the rang
I got16/16
History station..
A young female known diabetic present with wt
loss..her Dm is well controlled.. Inside history
going with Addison disease.. Discussion about
differential and investigations and management
of Addisons
And alhamdolellah before and after.. And thanks
again to Dr. Zein and everybody in the group

UK EXPERIENCE
experience Fairfield Hospital, Buay, Manchester:
station 5 - 1: 35 YO F c/o palpitaion and
dizziness with history of dislocated hip joint
inside: symptoms in attacks and occurs during
standing or any exertion: no chest pain no sob,
palpitation is regular. +ve all hyperextended jts.
my diagnosis EDS with anaemia for DD. qs:
criteria to diagnose EDS and causes of anaemia
how to investigate.
Station 5-2: 72 YO Male, c/o SOB and x-ray shows
pleural plaques. inside midsternotomy scar, decrease
air entry bibasal posteriorly. qs: why this scar, I said
it is tissue valve replacement vs CABG as no click no
vien harvesting which may from internal mammary
artery. causes of SOB I said heart failure but no LL
oedema or increased JVP, pleural effusion, lung
fibrosis. causes of lung fibrosis and effusion.
Abdomen: hemeplegic bed ridden 75 YO man, with
upper midline scar, has SOB. no HSM, no ascites, no
Heart failure. Qs.: why this scar, why he is
hemeplegic, why SOB. I mentioned general causes
like cancer stomach or bleeding gut, one of the
examiners insiste about what is the simplist
investigation if he has acute abdomen.
unfortunately I forget to mention brown frecklings
under his tongue which may be the key diagnosis as
Peutz J disease.
Chest: Lung fibrosis bilateral bibasal, causes of
fibrosis investigations how to treat.
Neurology: Parkinsonism, what is the DD of this
tremors.
CVS: AVR with sob, causes of SOB the examiner was
happy when I told him warfarin complication.
History: attacks of anexiety and palpitation MEN
syndrome i forgot the smoking and alcohol but I
covered every thing else, the problem is that the
serrogate keeps asking about his job what is the
causes is it stroke and so on not give me any space to
talk.
Communication: patient with asthma admitted with
excerpation treated well and on the day of discharge
given wronge treatment to other patient shared her
last name, asprin, steroid, BB and ACEI. very angry
and annoying surrogate. she didnt give me any space
to talk keeping shoting and didnt want to listen to me
even I tried to apologize, aknowledge her feeling,
asking about social problem, but without response
from her. finally, I stopped her and informed her that
this is only one dose we will keep you for 24 hours for
observation. the examiner accept my behaviour and
he said that she is a real actress dont worry.
PACES EXAM 3 July 2017 Mandalay General
Hospital
Station 3 CVS
Pt come to OPD for breathlessness
Middle age lady with submammillary
scar?Previous mitral valvotomy scar.
Pulse AF present.
Apex beat 5th ICS over mid axillary line and
diastolic thrill.
MA loud 1st sound MDM grade 4/6 loudest at the
end of expiration with the bell of stethoscope
Other NAD
DX This lady has MS with AF no sign of pul
hypertension ,no sign of IE and no sign of heart
failure.Possibly from restenosis of previous
mitral valvotomy and aeitiology is possible
rheumatic origin.
How will u manage?Firstly ECG for AF.Echo for
severity of MS,valve area,gradient and EF.Also
thrombus in LA.
She may benefic from anticoagulation treatment
with warfarin,and cardiologist referral.
Which investigation would you like to do?
I answered cardiac cath.
I got 19/20
Copied ,,,
Firstly I want to share that after a long journey I
have cleared PACES from Chennai. My score is
159/172. I am thankful to all teachers, mentors,
members of this wonderful group, my family,
friends and well wishers who have encouraged
me during my dark days of failure. It was my
third attempt and making mind for this was not
easy.
Anyway, coming to this diet,
Abdomen- Large Liver with fullness of flanks. I
was not confident of PKD and hesitated a bit.
11/20
Respi- Middle aged lady with diffuse polyphonic
wheeze. Trachea was deviated to right and there
was supraclavicular hollowing at right side.
When asked about the diagnosis I said
obstructive airway disease with possible
fibrosis or fibrocavitary lesion. I was forbidden
to exam the front , so I said I would expect a
bronchial BS or Crackels at right side. Then
about investigation and management was very
smooth. 20/20
History: 35yr old male with recurrent chest
infection. Infertility and malabsorption. I did
explore all social and familial issues including
economic. D/D -Cystic fibrosis, cartegeners
Syndrome. Questions came regarding
investigation and management. 20/20
Cardio- MS with PAH in sinus rhythm 20/20
Neuro- Command was middle aged man with
difficulty walking. Examine the neurological
system. Initially I thought but parkinsons. When
asked to exam gait, the patient took 1min to
stand and adjust his dresss. I got panicked due
to ongoing time loss. Anyway when I saw a
circumduction gait, I got relieved. Hemiplegia ,
questions on investigation , localization of
stroke, management etc.20/20.
Communication: 26 yr old Advocate, diagnosed
with ESRD. 5 yr back he was seen a blood
donation program when his BP was high. No
follow up. Now task is to discuss the diagnosis
and treatment options. I started in BBN style
then focused on the disease and effect of ESRD
on different systems of our body. Then focused
on management options- general and specific.
Transplant, Hemodialysis, Two varieties of
Peritoneal dialysis. The surrogate repeatedly
asked whether the doctor who saw him initially
is negligent? Questions came more on
treatment than ethical issues. 16/16.
BCC1: TIA, a middle aged man with active AV
fistula. I forgot to ask history of smiking and
alcohol and did a sketchy neuro exam which the
examiner did not like. He was expecting a more
detailed exam. 25/28
BCC2: Middle aged lady with fatigue. Large
goiter, clinically hypothyroid. I forgot to exam
ankle jerk. Questions came on investigation and
management. 27/28.
My first attempt (Kochi February 2016) was
underprepared one but surprisingly I scored
very high 153/172 and lost in skill B by 1 mark.
In second attempt ( Kolkata Nov 2016) I again
started 2.5month before the exam. I tried hard to
make it through by concentrating on station 5,
but was not confident in clinical stations. I lost
the diet 132/172 , 4 mark short in skill B. While
preparing I applied for the third attempt in
Chennai ( April 2017). After the second failure I
started rapidly and this time I tried to form a
group, so I was shifted to rented house and
some of my other exam gong friends actively
participated in group discussion and seeing
cases together. This actually helped a lot and I
regained my confidence.
I have noticed in my previous attempts that
examiners ask only few questions in clinical
stations and they expect a quick systematic
answer. I have prepared timed answers ( 30 -40
sec) for investigations and managements for
most of the common cases and practiced it
repeatedly with friends, juniors and even mirror.
In reality these have made a difference which I
felt this time. I did not have to think when I was
answering the common questions.
I suggest all fellow comrades to prepare their
own notes and rehearsal beforehand so that
everything becomes smooth during the exams.
For understanding I shall share my notes after
few corrections soon.
Lastly, I can surely say that even if it took me 3
attempts to clear PACES, but it enhanced my
clinical skill significantly and made me a better
doctor. Thanks you all again.
PACES 3 July 2017 Mandalay General Hospital
Station 4 Communication Skill
Patient record full paper about joint
pain,morning stiffness,lab results.
25 yr old lady with symmetrical joints pain
,morning stiffness duration about 3months.CP
auto Hb 10.5,RA positive,anti CCP titre
raised.U&C normal,LFTs normal.
Your tasks explains results,your consultant
want to start methotrexate and explains patient
concerns.Patient wants to start family planning
comming soon.
Firstly,introduce to patient.
Do you come alone.Do you want any coffee or
tea.How about your joint pain.
Patient said well and fine.I said Good.What
wrong my result dr.I makes like BBN style.I had
your blood results in my hands,I sorry the result
is not good.What?I sorry,you had serious
disease.I makes like PACES CS idol in PACES
video like multiple sclerosis explanation.
RA.Do you heared about it.Joint problem?Yes it
is long standing and deforming joint problem
without treatment.Not only joint but also it
damages your lung & kidney.I carry patient to
the darkness.But I give hope.But there is a
special drug which delay these problem.Called
methotrexate .My consultant want to give you
these special drug.
Pt ,now I am good,may I need these drug.
For long term,it is beneficial for you.
Pt,it has many side effect?It is true dr?
You are right,methotrexate has many side effect
but it is manageble and we can detect early and
we can stop timing.
What are the SE dr?
MTX has effect on your blood count,lung and
liver.Some inflammation of your lung and
liver.But if the drug stopped they recovery to
normal.
Pt,I am planning for family,it is ok?
Oh,it is very important.At least 4 months we
may stop methotrexate before pregnancy.
Why dr?It had bad effect on your baby.
What means dr?Your baby has some body
structural and organ dysfunction.
So what can I do?
Don`t worry.We may give you joint care with
lady O&G dr and joint specialist.
If mtx stopped my joint problem will be serious?
Don`t worry.Jt specialist will give some
alternative.Ofcourse your baby is precious not
only for you but also for me.
By the way,do you smoke?
May be 2 cigarettes ocasionally.
You must stop smoking to improve your
health.If you have some problem,I will refer you
smoking cessation clinic.
Do you drink?No.It is good.
What are you doing for living?School teacher.
Great your job is important.Any impact on your
jog.Because of joint pain some difficulty .I will
refer you to social worker to solve this problem.
How about your family?
Fine but my father has also RA.I sorry to hear
that.
Bell ring
Examiner asked me why you said serious
problem to patient.I shocked.Actually I expectes
what are ethical issues in this cases.
I explain 1,for compliance 2,for long term
complication of RA.
Examiner asked me why you didn`t make
another follow up with patient and her husband
with your consultant.
I shocked.
It is very important,I sorry,sir.
Finished....
After CS ,I believe I will preare for another
attempt.I depressed .I think this time is not for
me:-)
I prayed to the Buddha .
I read MAHAPATHAN Bible and I makes due
respect and pray to all examiners every night.
After 20 days,the result come out.
I pass.155/170.My station 4 is 8/
16.I happy.
Special thanks and due respect to Prof Col Khin
Phyu Pyar.Very thanks and respect to AP Dr U
Sein Win,AP Dr U Moe Naing and Dr U Phyo Thi
Ha.Thanks to Lt Col Aung Moe Myint,Maj Hlawn
Moe Han,Capt Nyi Min Han,Maj Pyae Nyein
Maung,Maj Thet Aung Zaw Myint,Maj Nyein
Chan Aung,Capt Han Myint Oo,Lt Col Aung
Myo.Finally but not the least ,Dr AhMed Maher
Aliwa,Dr Bal S Jhar. for very good and effective
web site and information.Thanks to all PACES
members.Never surrender .Fighting.

PACES Mandalay 3 July 2017 Day 1


Station2
35yr old female with joint pain
On analysis of joint pain,non specific
polyarthralgia,moring stiffness is minimal and
nonspecific,involing small and large joints,no
functional deficit,pain is increased by
movement and reduced by rest and
para.Associated with cough.I reviewed about
cough,mostly dry cough,associated with TOC
and ocasionally noisy breathing.Weight loss
present.Reviewing of JOSEPH joint ,occular,
skin lesion ,esophagus,photosensitivity and
hair loss.I explore skin lesion on leg.I
happy.Skin lesion suggestive of E
nodosum.Patient has low grade fever off and on
present.Past medical history of cough and CXR
done by GP showed some swelling of gland?I
so happy.
No urinary,bowel problem and no private part
sores.No relevant drug history.
I explains to pt,You had problem called
sarcoidosis.It is due to your overactive defense
mechanism,attacked on your jt,skin,lymph
gland and your lungs.
Patient concerns it is serious?
No,it is manageable and you must take regular
medication and follow up.
May I need admission?
No,we can do some blood test and imaging to
your chest as out patient.I will refer you to gland
specialist,joint specialist and lung doctor.Now I
will give you some pain killer.Do you have any
concerns? No.
Examiner asked me ,what is your
dx?Sarcoidosis
What is your DDx?
RA SLE
How will you manage this pt?Firstly,CP for
anaemia of chronic disease.ESR may
raised.Serum Ca and ACE level.auto immune
screening and rheumatology profile for
DDx.CXR and CT chest for lymphadenopathy
and pul fibrosis.Lung function test for restricted
lung disease.
Symptomatic Tx NSAID with GI cover PPI for
joint pain,mucolytic agent for chest
symptoms.Specific Tx she may benefic from
immunosuppressive Tx.I will consult with
rheumatologist and chest physician.What for?
Bronchoscopy and lymph node biopsy.
Which infection similar to this presentation?
I answered TB.
I got 18/20.

PACES Exam 3 July 2017


Station 3 CNS
Examine this patient and proceed neurological
exanmination
Young lady was sitting and smiling.Her left arm
was moving.At first,I think chorea but not sure.I
decided to walk and examine her gait.I so
happy,there is mild hemiplegic gait.I examine
upper limb and lower limb motor ,sensory and
cerebellar thoroughly and quickly.After that I
request pulse for AF,Heart sound for
MDM,Carotid bruit and fundus for OA.
My PDx is UMNL left hemiparesis most probably
young stroke with cerebral infarction or
vasculitis or MS.
Examiner asked me causes of young stroke.
I got 18/20

Station3
CVS- sinus rhythm, apex -not shift, PSM at apex,
ESM at aortic area, radiation+,
Dx- MR,AS- examiner not satisfy- AS murmur
can heard at apex. Which is more likely? Why?
AS- normal apex , sinus rhythm
I got17/20
C
Neuro
I'm not sure whethet scitica or CPN
Only9/20
Station 4
35 year old man with recurrent palpitation. His
father died at 40 . His brothet dx cardiac
disease, now on ICD. His ECG n Echo- HOCM
Task- ivx results, dx, concerns n
electrophysiological study.
Serogate- consult due to his brother's doctor
advice, he refuse to seek medical attention
when his dad died. IT expert, doing gym,
strenguous exercises, he has a son.
Concern- sure? Am i die like my dad?-sorry,
progressive disease, but will treat any symptom,
is it late dx?-no. Not change tx. Explain all tx
avaliables up to heart transplant. How about his
son,2 year old, - take him, ivx n regular follow
up. Exercise- warned not to do strenguous
exercises, sings to seek attention.
Examiner- ethical? Inheretant of HOCM, chance
of the son? Is it late dx? How to mx his son?
Regular follow up. Not satisfy- genetic
screening-ok
I got16/16

PACES EXAM 3 July 2017 Mandalay General


Hospital
I started with BCC 1 Station 5
40 yr old male with headache and BP 150/90.On
quick analysis of headache,chronic headache
with suggestive of raised ICP,early morning and
vomiting.On reviewing patient,all features of
neurofibromatosis.I examine CN 3 4 6 7 8and
pronator drip.Fundus NAD.I explained to
patient,headache may be associated with skin
growth,there may be nusty growth in brain.Need
special imaging CT Head.May be needed
to referral to brain dr and brain surgeon.I will
give some pain killer and some injection to
relieve pain.At present,importance of stop
driving.
Examiner`s questions
What is your diagnosis?
Why you say neurofibromatosis to these patient
?
What are possible causes of brain tumour in
these patient?
How will u manage?
I got 28/28
BCC 2 25 yr old lady with secondary
amenorrhoea for 6 yr
Positive symptom ,amenorrhoea and weight
gain.On quick examination ,obese lady,no
hirsutism,cn 2346 normal,no goitre nor
hypothyroid sign.
Mood normal.Libido normal.
I explained to patient,there may be female gland
problem and cystic formation.Need imaging
study ,blood test for hormone study,referral to
lady doctor,O&G,gland doctor,need hormone
treatment.Weight reduction.
Examiner`s Questions
What is your Dx?
What is your DDx?
How will u manage this pt?
I got 24/28
Station 1
Respiration
35 yr old man with breathlessness
Trachea shift to right,dullness on right upper
zone,VR increase and some crept
Dx Collapse Consolidation RUL/Fibrocavitating
lesion TB
Ddx Malignancy
I got 20/20

Station5 BCC1
C/o back pain - patient is a mook, got clue from
measuring talpe on the bed. Found ? posture
only when walk, did all measurements. Explain
dx, tx n occupational n physiotherapy.
Got28/28
BCC 2
C/o-fatigue. History of blindness+
Cushing patient. Recurrent blind on pregnancy
period, got headache on preg n blind , brain doctor
gave tx n releived. Now on tx. No drug chart. Lt eye
blind totally. Fundus- found nothing.
Dx- drug induced cushing. Ivx- no need blood test. To
do slow drug reduction
Got 26/28
Station1
Respi- stridor-with lt upper lobe collapse
Got-20/20
Abd- hepatosplenomagly with palmar erythema
Ddx-
Got- 16/20

Sat for the paces exam on 7th july, 2017 in


Aberdeen Royal Infirmary under Royal College
of Physicians, Edinburgh.
Station 1:
Respiratory: Left Lobectomy(probably lower
lobectomy). findings: thoracotomy scar on left,
and reduced breath sound, reduced vocal
resonance in left lower lung field. cant
remember
if percussion note was reduced in lower left
lung field as well. No other findings.
Was asked about the causes of lobectomy.
What inv to do in obstructive lung disease?
What
to expect in spirometry in obstructive lung
disease. What to do if the patient with
lobectomy
comes with breathlessness.
Got 20/20
Abdomen: CLD with hepatomegaly. Stigmata of
CLD like spider naevi, palmar erythema and
gynaecomastia
was present. Clubbing +/- so decided to ignore.
There was tender hepatomegaly, abdominal
flanks was full
but shifting dullness was not present so sed
there was no ascites. But people around me told
there was ascites.
Forgot to assess fluid thrill. It was written that
patient was suffering from weight loss.
Was asked about the causes: i told HCC on top
of CLD. Also sed CLD due to alcoholic liver
disease/non alcoholic fatty liver disease.
Was asked what else could cause the CLD. Then
sed this could be HCC on top of CLD due to all
the other causes. Was asked how to investigate
the case.
Got 20/20
Station 2:
Prompt was that patient has been suffering from
shortness of breath and wheezing for last 3
months. Previously he was completely fine. No
h/o allergy, chest
clear, x ray clear.
While interviewing the patient, it became evident
that the patient has been suffering from
exertional breathlessness, waking him up at
night, some degree
of orthopnoea, some dry cough and
breathlessness more in the weekends. Patient
works as a spray painter. Does not use the
mask regularly. During this period, went on a
holiday to spain
during which he was completely fine. Was also
a heavy smoker but gave up 10 years ago after
smoking for around maybe 25 years. Daughter
had h/o eczema.
Suggested to him that it might be occupational
asthma so made a plan of measuring his peak
flow regularly both in the
week days and in the weekends and arranged a
follow up. Told him to inform the occupational
health department but he sed there was none.
So told he might
have to give up his job, but he was shocked at
this.
Examiner asked diagnosis. I gave the
differentials of occupational asthma, late onset
asthma, copd and also wanted to rule out
ccf(but there was no h/o
chest pain, claudication, past h/o stroke or mi,
no dm, no htn, no high cholesterol but still told
abt ruling ccf out bcos of orhopnoea and
nocturnal dyspnoea)
How to investigate. How to treat. what other can
be the trigger? the exercise itself(this was
suggested by the examiner himself to which i
completely agreed)
Examiner picked up my mistake on bringing up
the issue of giving up job too early and
suggested that i cud have simply told him to
use the mask more regularly
and wait for the peak flow readings. What was
against COPD in this case?
Got 20/20
Station 3:
Cardiology:
A man with Atrial Fibrillation(rate controlled)
with signs of over anticoagulation. Patient had
kyphoscoliosis, a midsternotomy scar, no
corresponding vein
harvest scars and a very loud second heart
sound(seemed metallic to me).
Gave the cause as avr(metallic) with af with over
anticoagulation. the fact that he has
kyphoscoliosis might mean he has an
underlying connective tissue
disorder. Was asked if this patient comes with
breathlessness, what to do? Criteria for
anticoagulation
Got 19/20
Neurology:
It was a disaster. Was asked to examine arms.
Patient was lying down. Started with gait but
was denied by examiners. Power was reduced
in the left side, with
diminished reflexes and some wasting over left
deltoid and sensory loss in dermatomal pattern.
Examined the patient with him lying down trying
to maintain
the welfare but that turned into a disaster cos it
was so difficult to examine upper limbs with him
lying down and hence wasted a lot of time,
messed up the
examination technique in panic and could not
complete properly. Did not notice properly if
power in left side was lost proximally or distally.
Did not try
to reinforce the reduced reflexes. Did not find
out properly which dermatomes the sensations
were lost, just the distal dermatomes.
presented as radiculopathy/brachial plexopathy.
was asked what else could cause. I told
mononeuritis multiplex. what else. Sed Stroke
with spinal shock. Examiner
dint seem to be satisfied and asked if i noticed
any pattern in his power loss. Asked about the
reflexes, asked if it is a lower motor type of
lesion.
Asked how to investigate?
Got 18/20.
Station 4:
Breaking Bad news. Old Lady had h/o ca lung
which was operated two years back. felt bit bad
for the last 2 weeks and had sudden spastic
paraparesis from yesterday.
presented to the emergency department and mri
showed spinal mets. now on catheter.
Asked permission to drag the chair near her.
Then slowly broke the bad news cautiously after
assessing understanding of current and past
conditions and giving
warning shot. She cried, offered her tissues.
Gave her time. She was first in denial but then
started to accept. She asked what now from
here? told her abt
involving the oncologists, the macmillan
nurses, and maybe the physiotherapists for her
paralysis. Tole her that further investigations
will
be needed to assess how far it has spread.
Focussed on doing all these urgently. Asked
about home support. Her husband takes care of
her
but he goes to office in the morning so offereed
social workers. She was uncomfortable about
the catheter so gave her hope that with proper
medicines(steroids)
and radiotherapy things will improve so at one
point catheter wont be necessary. Woman was
very nice and cooperative.
Examiners were nice too. Summarised the case
to them. they asked what stages of grief the
patient went through. Asked what to do abt the
catheter, i also
suggested intermittent self catheter to them.
They liked the idea. How to treat? I Brought up
the palliative care team in the discussion with
examiners.
The examiner asked what will i do if patient
refuses treatment due to side effects of anti
cancer treatment which she experienced 2 yrs
back. TOld that will
counsel her abt newer better meds. What if she
still refuses. Respect autonomy. What are the
ethical principals involved in any case?
Got 16/16
Station 5:
BCC1: Abdominal bloating and discomfort in
young lady with BMI 19.
Entering the room found a cheerful relatively
healthy young lady whose BMI was certainly not
19 so immediatly understood this is a dummy
case.
history also revealed loose stool. no alarm
symptoms, not much wt. loss, no family h/o
bowel cancer, no symptoms of thyroid
problems.
Gave differentials mainly of coeliac, ibs and
infections like giardiasis. How to investigate?
when told esr, crp, examiner asked whY? I told
bcos of
ruling out IBD so examaminer asked if it was
one of the d/d. I sed it can be but less likely due
to no reasons given above.
Got 26/28
BCC2: Shortness of breath and painful fingers.
Middle aged man with raynaud's, h/o finger
ulcer. no joint pain, no other signs of sys.
sclerosis. had past h/o lung cancer and bowel
cancer resection.
Examination revealed no signs of sys. sclerosis
except telengiectasia on the face which the
patient commented that these are usual for him
from before.
Chest had bilateral thoracotomy scars which the
patient sed was due to lobectomy on one side
and removal of a lump on the other. There was
also probly
abdominal scar. Fine creps in right lower lung
field.
Gave diagnosis of sys sclerosis with pulmonary
fibrosis. What findings? Told the findings found
including telengiectasias and scars. How to
investigate
and treat?
Got 22/28.
In total got 161/172. Alhamdulillah
Did the Ealing course. Found it okayish as a
whole but for me it was also kinda life saving
bcos it wud have been probly suicidal to sit for
uk exam
without doing any course at all. Scottish accent
is impossible to understand at times but
thankfully, this was not a problem in the exam
cos almost all of the
surrogates and patients had very
understandable accent. Examiners were largely
very kind and well behaved.

I would like to share my exam experience with


you. Special thanks to all this group admins and
their members... it has been so benificial to me.
The exam was in Grantham and district hospital
in UK on june 2017.
First was station 4: 30 years patient dx with
testicular cancer 3m ago complained of
dysphagia ct done showed advanced
esophageal metastisis... your task is to explain
the management that the consultants reached
which is either esophageal stent or
radiotherapy... went inside introduced my self and
task... asked about patient previous knowladge of
condition... explained the result to ptn. and available
managements and their quick side effects... ptn
concerns were I want to live my life with no
constrains... and am I going to die... examiners q.
were if ptn refused both managements what to do, I
said tpn, peg tube, ngt., they said if he refused those??
I said inform my consultant... they said have u heard
about cancer teams... I got (16/16).
Next was st5.
Ptn after thyroidectomy complaining from tingling
and fatigue... inside he said he had carpo-pedal
spasms, those symptoms happenex only after
surgery... he had collar scar... I checked thyroid
function quickly then did chovestic and Traussaue
signs which were negative.... q. were what is dx?
Iatrogenic hypocalcemia... management? Ca. Level,
mg. Pth. Level.... ttt. Give Ca and vitD. (28/28).
Other st5 .young Ptn complaining of deminution of
vision for 2 years... had +fh his sister has simillar
symptoms however milder, did visual acuity...the
visual field testing which was constiricted then
fundus... it was retinitis pigmentosa... questions...
about management... occupational therapist,
opthalmologist... what else? They wanted refferal to
genetisist... mode of inheritance.. syndromatic Rp.
Also were asked about... got (28/28).
St1. Abd. Patient with Rif scar and mass also below
umbellicus scar.... Q. Dx? Renal
transplant...Management? Got (20/20).
Chest. This one I didnt do well in she was a female
with a small scar in left axillary reigon and basal left
crackles... they kept asking do u notice any thing in
her eyes?.... my colleagues said she had left horner
syndrome... got (9/20).
St2. Ptn has type 1 dm. Complaining of fatigue and
dizziness.... dx. Addisons disease... Q. Dx.?
Investigations? Management? (20/20).
Stn 3. Cardio... bilateral mitral anf Aortic valve
replacement... Dx? Investigations? What if he had
fever what u will be worried about? Prophylaxsis for
infective endocarditis?(20/20)
Neuro. Old lady has bilateral gloves and stalkings LL.
Sensory loss and left sided LL weakness. Dx. P.
Neuropathy with possible left sided weakness CVA?
wasnt sure about this station... however thank God
got (20/20).
Total 161/172... it was only with Gods bless.... it
wasn't my first experience.
A big thank you for the admins of this page it was so
benificial to all of us.
Advice: dont give up continue. Cheers.

UK EXPERIENCE
Exam Experience was in grantham and district
hospital glasgow n my exam cases were
I started with respiratory station command was
this patient has presented with recurrent
pleuritic chest pain
A middle age Lady with a scar on back of left
lung field n left horner's rest of examination was
normal
Scar was thoracotomy scar n examiner asked
about possible cause of horner syndrome
investigations n treatment 11/20
Next was abdomen
Faint scar in right illiac fossa n no other scar i
can
Found so i gave my diagnosia as functioning
renal transplant n gave differentials for renal
failure as in goutam mehta it is given as causes
of renal failure viva was about the most possible
cause i said diabetes n then how ll u investigate
if this pt comes to u in emergency with
abdominal pain n about treatment with
immunosuppressants 20/20
Then my next station was history that was
young patient with history of weight loss n
diabetes mellitus
In the start of history he told about postural
hypotension then weight loss for about 1 year n
recurrent hypoglycaemia for the last few weeks
even no change in his current diabetic regime n
family history of thyroid problem so my most
likely diagnosis was addisons n differentials
was autonomic dysfunction due to DM he asked
me about else for weight loss in young patients
i said i would like to assess for coeliac n thyroid
then viva about addisons investigations n
treatment 20/20
Then cardio
A man with midline sternotomy scar n audible
metallic click coincides with second heart
sound n ejection syatolic murmur so that was
AVR with AS
Viva was about investigation n management
how ll u follow up this patient
He also asked me either valve is functioning
well or not if well then what r the signs of not
functioning well then signs of cardiac failure
In neurology a lady with vericose veins charcoat
joints at ankle bilaterally on inspection then i
asked for walk in start her romberg sign was
positive n there was reduced power 4/5 with
hyporeflexia but hypertonia in left knee
surprisingly n sensory loss in stocking
distribution so i gave my most likely diagnosis
as peripheral sensori motor neuropathy viva
was about differentials which i told about
causes of sensorimotor neuropathy as given in
goutam mehta n investigation n treatment
17/20
Station 4 was about
A young patient with dysphagia due to
metastatic testicular carcinoma got multiple
treatments in pasr n now offer him about
palliative care n for dysphagia give him 2
options
Stent placement
Radiotherapy
I followed the scheme n as he already knew
about his disease n treament failure so his main
concern was about to live an independent life so
i told him about that options their pros n cons n
that we respect his wishes n 2 points were imp
in this scenerio
1.advance directive or LPA 2. Palliative care
team referral at the end he agreed upon stent
placement 16/16
Now the last station was station 5
First scenerio was a young patients
complaining of abnormal sensations in legs
after thyroidectomy n all vitals r normal i
thought about tetany n peripheral neuropathy
due to hypothyroid or it might be any other
diagnosis by keeping my mind open for all
options i went inside the room in the start he
told me its actually carpopedal spasm when i
asked what exactly happens n then i asked
about recent surgery cause n current thyroid
status n rule out other causes of hypocalcaemia
diet,renal,malabsorption
Examination was about chvostek n trosseu sign
n thyroid scar with palpation n looking for
tremors n sensations even these signs were
negative but still history was typically about
hypocalcaemia so i gave my diagnosis with
cause is parathyroidectomy with thyroidectomy
Then investigation n treatment
27/28
The next station was this patient has gradual
loss of vision...young patient who gave me
history of nyctalopia on exploring typical
history of retinitis pigmentosa n no associations
with other syndromes
N family history positive I examined visual
acuity perimetry n fundoscopy n looking for
general appearance he was thin lean so no
association with lawrence moon
Then explained to patient
Viva was about diagnosis fundoscopy findings
n what r the syndromes associated with RP n
what referrals r imp i said 2
Low vision clinic n genetic counseller 27/28

UK EXPERIENCE
Experience in oxford centre
Station 1
Respiratory patient with dilated veins on chest
with clubbing bilateral lobectomy scar and has
bronchiectasis
Abdomen renal transplantation with old
peritoneal dialysis with ascitis
Station 2
History of left knee and right ankle swelling and
pain with last month food poisoning it was
reactive arthritis reiter syndrom
Station 3
Cardio aortic stenosis with mitral incompetence
Neuro
Eye examination only i find visual acuity
diminished in both sides with POF in left
Visual field on right revealed temporal
hemianopia with movement revealed
internuclear ophthalmoloplegia with fundus
bilateral optic atrophy more at left
My diagnosis was MS
Station 4 uncertainty for cancer stomach with
further management plan
Station 5
1-Graves eye with thyroidectomy scar with
acropathy and peritipial myxedema patient has
typically hypothyroidism and non complaint to
medication
2- patient with collapse with no witnesses he
start amilodipin recently 3 weeks and collapsed
on driving and there ejection systolic murmur i
tell aortic stenosis appears when he takes
vasodilator and stop driving is mandatory
Pray for me plzzzzz

My experience in Victoria hospital, kirckadly


Edinbrugh, UK
July 2017
Station one
Chest : about 60 y old pt co SOB
Pt had clubbing , insp crepitations all over the
chest
ILF
Examiner asked about inv &ttt
Abdomen
Pt had mild hepatomegally
Huge spleen
Midline labarotomy scar
RIF scar
No signs of CLD, or anaemia
I talked about hepatosplenomegaly DD
Examiner was not happy
Station 2
Young lady , k/c IDDM
History of wt loss &dizziness
No skin pigmentation or GIT symptoms
Few hypoglycaemic attacks
D. Addisson's dis
DD hyperthyroidism
Celiac dis
Examiner asked about inv for addisson &ttt
Station three
Cardio
Pt had mid sternotomy scar & venous harvest
scar
No metallic clicks
Waterhammer pulse
Murmurs of AS &AR
D. Mixed AoVD
With CABG
Discussion about predominant lesion AR or AS
Indications of valve replacement
Neuro
Examine upper limb
Pt is sitting on chair
When I asked about pain he said he had pain in
rt shoulder & elbow
When I tried to examine tone I could not then
examiner told me to take care as pt has pain in
rt arm
I examined tone and power around wrist only
Reflexes normal
Sensation normal except vibration
Pt could do test for coordination I asked the
examiner to test for nystagmus to assess
cerebellar function but he refused
Presentation I said i could not examen him dt
pain but he had weakness more in flexors and
loss of vibration
DD Cervical myelopathy
MS
He said if pt had hyper extension trauma to
shoulder what do you suspect
Discussion was about brachial plexus injury
&frozen shoulder
Station 4
Communication
Pt admitted with wt loss, CT abdomen results
shows suspected cancer colon with liver
metastases.radiologist suggested meeting with
treating consultant &oncologist to confirm D.
Her daughter is angry as she is in the ward for 3
hours & nobody informed her about her
mother's condition. She requested to see the
doctor. You did not see the pt before but you
checked the file before talking to the daughter.
Assume that pt gave permission to discuss her
condition with the daughter.
After greeting and confirming the relation I
apologized for the delay, then agreed the
agenda
Breaking bad news
Mostly she had nasty growth but we need to
wait for MDT meeting to confirm D &put
management plan
She did not show any feelings
Concern Staying in hosp for long time
Living alone: options
Her father died with cancer :sympathy
Guilty about delay in D: I will discuss the history
with her mother, usually difficult for old people
to pick symptoms early , but I will check if there
is delay
I finished early and examiner was not happy &
told there is 2 minutes
I offered a drive &
A near aptt to discuss more
Discussion about autonomy
Why u did not discuss management?
D not confirmed, she did not ask
He said u should volunteer
I told it is better not to discuss it in same
Meeting after BBN & I gave near aptt in same
day
Then he accepted
Station 5
BCC1
Young lady with back pain
Pain is more in the morning & improve with
exercise
PH anterior uveitis
I examined neck movements
Then lumbar
I asked to do shoeber test surprisingly he said
yes
So I did it without putting marks
The wall was far so i did not do occiput wall to
save time
I examined lung apices & ht
Then concern & plan
Ankylosing Sp
Discussion about ass condition , management
BCC2
Pt admitted with MI 4 days ago & co chest pain
Long history to exclude relation with effort and
food
Compare with pain if Previous MI
No LL pain
One attack fever
No cough
I ex LL & chest quickly
Concern is it another stroke
Plan of management
Examiner asked about positive findings
No positive findings
DD dressler sy
PE
P eumonia
Gerd
Inv , ECG in pericarditis
In general all examiners donot allow u to
present ur finding or complete ur sentences. All
the time asking rapid question &distracting
It was my First trial
Alhumdullelah I passed

UK EXPERIENCE JULY 2017


Oxford diet 2 UK .
st 1 renal transplant n hepatospleenomegaly
with laprotomy scar n incisional hernia , fistula
not active,astrexis present
St 2 bronchial asthma
St 3 cardio median sternotomy n syncope
Neuro peripheral motor sensory neuropathy with b/l
pescavus , spasticity+, power decreased in left lower
limb, reflexes diminished .heel shin test intact.high
stepping gate.
St 5 bronchiactasis
St5 frozen shoulder
St 4 MRSA positive
St 3 resp pul fibrosis secondary to scleroderma
I would like to share with you what I have learnt
from my PACES experiences. This group has
been very helpful in my preparations and I
thought I would contribute in terms of the
lessons I have learnt. Here are a few of them
1) Understand that PACES is a entry exam into
Specialty training in the UK. So it requires you
to be able to make sound judgement and also
be competent in managing patients welfare and
concerns. You will rarely get something out of
the ordinary. Common things are common. This
is not an exam to test only your knowledge. It
has been tested in Part 1 and 2. This exam is to
test your showmanship. Show that you know
your stuff! Do not be overconfident . Do not
fumble and look unsure either.
2) Pertaining Station 1 and 3. You should aim to
get full marks for your Abdo station. The types
of cases that you can get is very limited! The
key to abdo stations are your peripheral
findings. The other station that you should do
well in is your respiratory station. The list of
cases that may turn up is standard.
3) Always..always take a step back and look at
your patient in Stations 1/3/5. Take about 10-20
sec just to do that. In previous attempts I
realised I was so focussed on the diagnosis that
I forgot to look at the patient in entirety. The
diagnosis would have been more evident. That
20 seconds you spend may give you an obvious
finding that may help you spot a diagnosis!
4) Be VERY polished in your physical exam. It
has to be mint. It has to be second nature. I
know you have read this like a thousand times.
But the logic here is, you dont want to be
thinking of what to examine after tone in a neuro
exam. What you want to be doing is interpreting
the signs to come up with a diagnosis.
5) When you are in a communication station, try
to establish - What kind of station is this? Is this
a breaking bad news station? Is this an angry
patient station? That will help you set out a
template on what to say exactly. Be familiar with
concepts like - capacity, DVLA rulings, Mental
Health Act, Human Tissue Transplant Act and
the four principles of ethics. When you discuss
with your examiner, you must tell them how
does
beneficience/maleficience/autonomy/justice
apply in this particular scenario.
6) It really doesnt make sense to work on skills
that you are already good at. Work on things
that you dont like. I hated fundoscopy! So when
I was at my shifts I ended up doing almost 5-7
fundoscopies a day. At the end of the day,
eventhough I had a lazy left eye, fundoscopy
wasnt a difficult thing.
7) Station 5 carries the MOST marks in the
exam. On top of the usual cases that you may
find in the books, station 5 are becoming more
relevant to Acute Medicine these days. One of
the stations I had was a patient with lung cancer
. And another was a cluster headache.
Remember ...common things are common.
8) Sometimes it is rather tough to get people to
watch you examine. Use your smartphone! I
dressed up my pillow with my shirt and
recorded myself examining it. You will be
surprised how much you can learn and improve
with just doing this!!
Hope this helps!! All the best!!
MRCP - An exam that teaches more than tests!

UK EXPERIENCE
Exam EXPERIENCE was in grantham and
district hospital glasgow n my exam cases were
I started with respiratory station command was
this patient has presented with recurrent
pleuritic chest pain
A middle age Lady with a scar on back of left
lung field n left horner's rest of examination was
normal
Scar was thoracotomy scar n examiner asked
about possible cause of horner syndrome
investigations n treatment 11/20
Next was abdomen
Faint scar in right illiac fossa n no other scar i
can
Found so i gave my diagnosia as functioning
renal transplant n gave differentials for renal
failure as in goutam mehta it is given as causes
of renal failure viva was about the most possible
cause i said diabetes n then how ll u investigate
if this pt comes to u in emergency with
abdominal pain n about treatment with
immunosuppressants 20/20
Then my next station was history that was
young patient with history of weight loss n
diabetes mellitus
In the start of history he told about postural
hypotension then weight loss for about 1 year n
recurrent hypoglycaemia for the last few weeks
even no change in his current diabetic regime n
family history of thyroid problem so my most
likely diagnosis was addisons n differentials
was autonomic dysfunction due to DM he asked
me about else for weight loss in young patients
i said i would like to assess for coeliac n thyroid
then viva about addisons investigations n
treatment 20/20
Then cardio
A man with midline sternotomy scar n audible
metallic click coincides with second heart
sound n ejection syatolic murmur so that was
AVR with AS
Viva was about investigation n management
how ll u follow up this patient
He also asked me either valve is functioning
well or not if well then what r the signs of not
functioning well then signs of cardiac failure
In neurology a lady with vericose veins charcoat
joints at ankle bilaterally on inspection then i
asked for walk in start her romberg sign was
positive n there was reduced power 4/5 with
hyporeflexia but hypertonia in left knee
surprisingly n sensory loss in stocking
distribution so i gave my most likely diagnosis
as peripheral sensori motor neuropathy viva
was about differentials which i told about
causes of sensorimotor neuropathy as given in
goutam mehta n investigation n treatment
17/20
Station 4 was about
A young patient with dysphagia due to
metastatic testicular carcinoma got multiple
treatments in pasr n now offer him about
palliative care n for dysphagia give him 2
options
Stent placement
Radiotherapy
I followed the scheme n as he already knew
about his disease n treament failure so his main
concern was about to live an independent life so
i told him about that options their pros n cons n
that we respect his wishes n 2 points were imp
in this scenerio
1.advance directive or LPA 2. Palliative care
team referral at the end he agreed upon stent
placement 16/16
Now the last station was station 5
First scenerio was a young patients
complaining of abnormal sensations in legs
after thyroidectomy n all vitals r normal i
thought about tetany n peripheral neuropathy
due to hypothyroid or it might be any other
diagnosis by keeping my mind open for all
options i went inside the room in the start he
told me its actually carpopedal spasm when i
asked what exactly happens n then i asked
about recent surgery cause n current thyroid
status n rule out other causes of hypocalcaemia
diet,renal,malabsorption
Examination was about chvostek n trosseu sign
n thyroid scar with palpation n looking for
tremors n sensations even these signs were
negative but still history was typically about
hypocalcaemia so i gave my diagnosis with
cause is parathyroidectomy with thyroidectomy
Then investigation n treatment
27/28
The next station was this patient has gradual
loss of vision...young patient who gave me
history of nyctalopia on exploring typical
history of retinitis pigmentosa n no associations
with other syndromes
N family history positive I examined visual
acuity perimetry n fundoscopy n looking for
general appearance he was thin lean so no
association with lawrence moon
Then explained to patient
Viva was about diagnosis fundoscopy findings
n what r the syndromes associated with RP n
what referrals r imp i said 2
Low vision clinic n genetic counseller 27/28
UK EXPERIENCE
Experience of Pilgrim hospital Boston
Station 1: Abdomen:
Very elderly female, on couch wearing full
sleeves shirt, fully dressed
O/E hepatosplenomegaly with ascites, palmar
erythema, unstable and can not roll for shifting
dullness so examiner said it's okay, pedal
odema with very bad superficial big ulcers on
both legs with oozing of transparent secretions.
She took one minute to make her arms above to
have a look at arms and don't like helping her
for exposing.
I request complete exposure to examiners
before starting examination and they said talk to
patient. Discussion on myeloproliferative and
lymphoproliferative disorders and I said I will
put differential of CLD with portal hypertension.
After finishing examiner asked me patient can't
roll for shifting dullness so y u did not do fluid
thrill. I hardly finished in time coz of this me
already wasted by patient. There were no
abdominal scars at all
Respiratory station
45 yrs old male with marked clubbing and
pursing of lips, reduced cricosternal distance
and crepitations in upper lobes bilaterally, with
normal vocal resonance
I give ILD, COPD as differentials but did not
mention bronchiectasis with COPD
In discussion examiner ask y u would like to see
sputum pot, no sputum in pot, then.
I said I will proceed further, asking findings of
CXR in COPD and then ILD and when I tell him
one by one he said u will find in all cases these
findings, I said not in all pts
History station
was 40 yrs old male with backache after lifting
something heavy and he lives alone at home,
information outside the room
On history he gave positive history of weight
loss 5 kgs in last 2 months and have pain in ribs
2 sites on right side too, and fell down at home
3 times in last 2 months jus standing coz of
sudden weakness of lower limbs which later
recover and no sensory symptoms and no
urinary or bowel incontinence
In diffrentials list Malignancy was at the top and
spinal cord compression to be ruled out by
urgent MRI and involvement of Orthopaeds
team , primary Malignancy symptoms not
evident in history with an system
Examiner was satisfied and happy and also the
patient, I said I will not discharge u and will do
urgent MRI spine and will see u again to make
further plan. Coz u are alone at home also so we
will discuss about it further in more detail and
will try to help u at home maximum by involving
ur GP and occupational health therapist. But
currently urgently thing is to rule out
compression of ur back
Cvs: young female with pansystolic parasternal
murmur not affected with inspiration or
expiration so I told VSD
It was very clear, and apex beat was displaced.
Visa went on NICE guidelines of prophylaxis
before dental extraction and then before
colonoscopy
Went the best station
CNS: lower limb examination
After inspection started with gait, broad based
ataxic gait, Romberg test not done as pt cannot
stand with feet together
Motor, sensory normal, he managed to do heel
chin test but badly, went to upper limb for
cerebellar signs and time finished
Patient also had clear nystagmus
Viva on causes and how to investigate
cerebellar disease causes, examiner said well
done at the end
Communication was young 26 yrs female
diagnosed as RA and started on methotrexate
and she did not start treatment as she is going
to get married in 2 months time and had heard
about side effects
Patient was alcoholic too, and planning not to
be pregnant for 2 yrs after marriage
Went really good with explanation for RA, then
how methotrexate works, what are benefits and
what are side effects and patient agreed.
Station 5 1st: hyperthyroidism with all
symptoms and small neck swelling and diplopia
in multiple directions
So Graves' disease, not on any treatment
2nd: young male with h/o ischemic gut removed
1 year ago and had complain of chronic diarrhea
for last 1 year not improved with anti-diarrheal
And on rivaxoxaban 20 mg OD since 1 year
Inside the room he said his last part of small
intestine was removed and has clot in his heart.
No family history of any illness or clots
anywhere in other organs.
I did abdominal examination and laparotomy
scar only, nothing else.
And this went really bad and scored badly too
I discuss about malabsorption and b12
deficiency
Patient asked should I keep taking rivarxaban, I
said yes to keep taking as he has clot in heart
and we will consult heart drs for it too.

Central Middlesex hospital 8/July /2017


St5
Chronic cough(GORD/EAA/drug induced)
TIA
Abdomen ascites/leuconechia/+_ telengiectasia for
differential
Chest basal crackles with clubbing
Neuro Parkinson with PSP
Cardio AVR( not sure if it's mechanical or
biological)/pacemaker
St 2
Back pain metastatic
St4
Daughter angry why her father not on dialysis, her
father capacity was intact to refuse after explanation

India( Bengaluru)
Station 4 , 26yrs old lady physiotherapy as st.
Working in stroke unit .admitted with flaccid
limb weakness , CT and MRI normal explain
about functional weakness . Pt was reluctant to
accept the diagnosis and wish to see
neurologist urgently don't want to see
psychiatrist .social issue about job , and
grandmother died 3 months back with
stroke.discussed about psychiatric referral and
physiotherapy . Station 5 - 1 st case 30 yrs old
lady with high prolactin levels and normal TSH c/o
scanty and irregular menstruation. 2nd case- 26 yrs
old lady with SLE since 6 yrs presented with right
sided pleuritic chest pain ,with fever. Discussion
about DD of chest pain. Station 1 -- Respiratory --
lung fibrosis Abdomen = ascites with chronic liver
disease , jaundice, parotid swelling,flapping tremor,
spider nevi , examiner asked about if there is fever
what can be the cause and how to treat .management
of ascites . Station 2 - 30 yrs old lady with facial and
neck swelling sudden onset ,adopted child , no other
positive history , concern about allergy .DD-
hereditary angioedema . Investigation and treatment .
Station 3 Neuro - right sided weakness , with
proximal wasting hypertonia, hyperreflexia
,dyddiadokokinesia ,sensory normal . Cardio -- young
lady with MS - tapping apex sinus rhythm ,loud S1
diastolic murmur , phtn and raised jvp .
bangalore. Manipal ..7/4/17
Started with st2...Anaemia (MHA.) H/o ibs 6 yrs on
peppermint oil,occasion leg swelling,ho rta and nsaids
,father died of cancer colon,mother taking inj.3
monthly for anaemia,so many issues, ,DD was so
many issues..coeliac..nsaids, ,ulcer
St3,cardio MS,,neuro..patient was very un
cooperative, not following me,,actually language
barrier was prob. I didn't understand what he told
about sensation, ,planter was very confusing, ,Indian
examiner didn't tell anything! !
St.4 esrd bbn and plan of mx
St5,,copd with sudden breathing difficulty and rt
sided chest pain.. o/e Dec breath sound.rt mid.dd was
ptx,pul embolism, ,pneumonia. It was good
St5 female was increase weight,,increase bp,,,and
proximal myopathy, ,o e no thyroid abnormalities,
,some rash,,and proximal myopathy,,features of
osteoarthritis, ,knee jt,leg oedema, ,dd,was cushing,
,hypothyroidism, metabolic syndrome, ,discuss was
on cushing. .

UK EXPERIENCE
Royal preston hospital lancashire
06-07-17
Station 1
Resp.. left lobectomy
Abd...renal and liver transplant
Station 2
Pt diagnosed as asthmatic since childhood well
controlled with sos basis salbutamol now for 3 months
his cough,wheez,sob is worsening
During history he told that he had dry cough and 6
weeks ago he developed rash on legs
Diagnosis was churg strauss syndrom
Station 3
CVS...AVR,pt was marfan syndrom
CNS...sensory motor neuropathy
Viva about causes management
Station 4
Pt known case of psoriatic arthropathy on
methotrexate
She had UTI and GP started her on trimethoprin and
as a result she developed pancytopenia
Presented in hospital with nose bleed and bruises
Explain the medical error to her
Her concerns
Am I going to die
Complain about GP
What about methotrexate I dont want to discontinue
as iam well controled with it
Station 5
Evalaute pt with abdominal pain and fever
Inside lady with left wrist av fistula and she told she
has renal transplant for 12 years doing well
3 days history of lower abdominal pain and smell in
urine
No vomiting haematuria on examination she has
tenderness on suprapubic area
She was taking tacrolimus,steroids,
Questions
Daignosis
Cystitis,uti
Management
Station5
2nd pt
Young guy presented with fatigue out side given
TSH,T3,t4 low
Inside an actor
Gave hiatory of fatigue all time had severe headache
1 month ago and now also developed visual problem
Asked about dizziness when stand he tell yes every
time and when asked he tell he is feeling loss of libido
also
On exam
He had bitemporal hemianopia(pretending)
No blackening of skin
Concern was what is happening
He ask me to give me thyroxin as my thyroid profile
is deranged
I advised labs and trestment after
Advise about stop deriving,some medicine for erectile
dysfunction etc

Copied from telegram group


Glasgow, today cases
1) Respi - right lobectomy , some said right
pneumonectomy because in btw right lung
there's an area of bronchial breathing. However
, I still think right lung lobectomy coz trachea
Nor deviated.
Examiner asked about if suspected recurrent mitotic
lesion of lungs ... and CT- date pending ... what
investigation to get other than cxr
2)Abdomen- everyone said normal abdomen but I felt
minimal enlarge hepatomegaly.
She is an obese lady with palmar erythema and 3
spider naevi.
I gave diff diagnosis of PBC/ autoimmune hepatitis /
Hep B / hep C / NASH
Examiner questioned about PBC clinical finding ... I
said pruritus / scratching mark / jaundice ... he wants
more then bell rang
3) Neuro - a 50 yo gentleman with normal finding ...
he was given L-dopa kaw kaw ... then I only managed
based on the clue from the stem ... there written
frequent fall. Noted subtle sign of Bradykinesia, then
divert him to tap his lap then noted very subtle
cogwheel rigidity ( not sure whether I created sign
onot ). Checked for psp and request to do mmse/ Bp/
mocrographia.
Examiner asked about Parkinson treatment / ix/
causes
4) Cardio- MR+ AF + HF + grade 2 finger clubbing +
peripheral cyanosis
Some heard purely AF
1 said MS
Examiner asked about MR , how to manage , how to
diagnose , warfarin therapy
History
Back pain , 50 yo , sudden onset
Managed to get history of LOW 6kg, constipation ,
urinary retention, nocturia, no red flag
Given whole bunch of DD but examiner further
discussed bout mets to bone 2 prostate Ca/ lung CA/
GIT Ca
When I came out only realised that it's actually
multiple myeloma. Didn't get a chance to discus with
examiner
Comm skills
40yo gentleman u/l epilepsy 10 years fits free 7 years
admitted to ward for pneumonia. Forgot to prescribe
AEM for him in ward. So in ward 36h developed
seizure
BCC
1) 45yo gentleman c/o headache for 4/52
Generalised headache
BP 145/90
Normal pt
Given DD - migraine / Raised intracranial pressure /
tro SOL/ tension headache / cluster headache and
mentioned also want to rule out young hpt-
phaechromo
BCc 2
Pt came with anemia
Further h/o renal transplant + Dual valve
replacement + latest INR 4
Given
1) UGIB
2) valve hemolysis
3) kidney graft failure

UK EXPERIENCE
Colchester today Under London College
I stared first with station 5
BCC1: 5 days h/o diarrhoea with signs of
dehydration not on patient but mention the
scenario
BCC2: Goitre with weight loss and hyperthyroid
status
Resp: Brochiectasis typical case
Abdomen: mass in the lumber region with scar on it
viva was about Polycystic kidney and differentials
CVS: Aortic stenosis n viva wa related to it
CNS: examine the lower limb and do relevant and it
was typical case of myotonic dystrophy
Communication:
Breast cancer with mets to hilar lymph nodes and
pressing on esophagus and MDT team offered
palliative radiotherapy and esophageal stenting
Talk to the patient
History station:
DM Type1 weight loss and dizzy spell and very good
control of diabetes
I made diagnosis of Addison disease and forgot to ask
about autonomic neuropathy and viva was on
Addison disease
Good luck every one

UK exam experience (2017/2) diet


Royal infirmary Glasgow
22-06-17
History was unusual so going to share
Young girl has presented with Recurrent
abdominal Pain and recurrent vomiting for last 9
Months.
She was operated for Adenomyosis 3 years
back ...
Diagnosis is subacute intestinal obstruction
New lister building
Glasgow
22 june 2016
Station 2.
40 years old man
smoker
had advanced Lung cancer and mass in left side
but not metastasized diagnosed 3 months back
and was treated with chemotherapy
has now presented with Pain
( 3 types of pain , Backache,left sided chest pain
( severe not responding to strong analgesics) ,
Right sided chest pain .
Station 4:
Old man
known hypertensive on medicine and aspirin
Had a fracture of femur and admitted in
Orthopaedic ward and on investigations was
found to have UTI as patient was unconscious .
Then consulted with doctors of medical floor
and despite treatment did not respond so brain
scan done which showed Frontal lobe
hematoma.
patient was also given Heparin in orthopaedic
department for prophylaxis of DVT .
Talk to son about the situation
GOLDEN JUBILEE NATIONAL HOSPITAL
15th June 2017
Station 2:
History
Middle aged lady having Multiple sclerosis and
on regular follow ups has presented with cough.
it was dry cough more on lying flat with burning
in the chest
No other chest or heart symptoms
patient concern was
is that Aspiration pneumonia ???
diagnosis : GERD
STATION 4:
83 year old man has End Stage Renal Disease
with underlying DM and HTN .
was admitted with Pneumonia and herat failure
1 month back and recovered.
was advised Dialysis which he refused at that
time .
Now talk to the son who is insisting for Dialysis
as he says his father says yes to him but
refuses when doctors talk to him about dialysis.
Assume the permission was given by the father
to talk to the son about his condition.
( Father concern was that he lives alone and can
not manage to go to hospital regularly for the
Dialysis )
STATION 5:
BCC 1:
Man presented with weight loss and palpitations
.
concern was why i get lot of sweating .
BCC2:
Man with previous history of Aortic valve
replacement now presented with joint pains
including hands and Knee !!
history was suggestive of RA with acut flare.
viva was also about Hyperuricrmia?
Concern was will it affect golf which i play since
long?
Can you settle my Deformities?
Wolverhampton Hospital
8th june 2017
station 2:
Middle aged female
with Type 1 DM
well controlled ( on history)
Now presented with Fatigue and Weight Loss
Station 4:
Old Lady with advanced CA breast
encroaching upon esophagus causing
dysphagia
Break bad news
Discuss about treatment options
including Stenting and Radiotherapy
Station 5:
BCC1
Old male
presented with Collapse
inside Parkinson Disease
concern : will i get better
BCC2:
Man with Rheumatoid arthritis
presented with Fever
inside told about night sweat
weight loss and
cough
visited india few months back
taking new medicine for RA with much relief (
Infliximab)
concer: Will you stop my medicine as i am
feeling much better after this medicine
irrespective of TB or else ( Lymphoma)
9Th june
Wolverhampton Hospital
Here are the cases :
Station 1 :
Abdomen
Splenomegaly with jaundice and Vitiligo
Respiratory :
Cushingoid patient with midline Sternotomy
scar otherwise normal examination
Station 2:
Middle aged female with previous history of
headache is referred with continuous headache
despite paracetamol and tramodol intake .
station 3:
Cardiology:
AVR
Neurology :
Stroke
Station 4:
Middle aged female with Uncontrolled DM
diabetic retinopathy has been diagnosed with
Nephropathy as noticed by protrinurea and were
given ACE inhibitors .
Task is to check compliance and counsel about
ACE as kidney protective medicines .
Station 5:
BCC1:
SOB and CHEST PAIN :
Inside scleroderma
ILD and GERD
BCC2:
Vision problem
inside she told about diplopoda
Turned out to be Myasthenia Gravis
Queen Elizabeth Hospital
Glasgow college
7th june 2017
Station 2
Patient with history of knee pains presented
with fatigue and melena !
Pain associated with stiffness and improves
with walking .
Family history of RA is positive.
patient concern was do i have RA like other
family members ...!!
Station 4
Communication with a young girl who is
married and planning for kids who has been
diagnosed with RA and now on follow up visit
as she was told to start Methotrexate in her last
visit and now she is back again to discuss the
plan after reading about the therapy on internet
station 5:
BCC 1.
Man with multiple problems of
Psoriasis
Athropathy
Had shoulder dislocation few days back and
was operated
now presented again with shoulder pain with
fever.
BCC 2 :
Young male with history of crohn's disease
presented now with back and stiffness along
with pain in the neck ..
Exam from Queen Elizabeth Hospital
Glasgow centre
6th June 2017
Statio. 2
History of Rash on hands .
History of intake of Tetracycline .
Patient concern was will i get scarring after the
rash is over .??
station 4:
A chef had anaphylactic reaction to eating salad
sandwich. He was allergic to peanuts. Had
minor reactions to them before twice. Was also
asthmatic and on inhalers. As a result of
anaphylaxis was admitted into icu and
ventilated. Task was to explain seriousness of
his allergy and implication on his job
Station 5 :
BCC 1 :
Patient with RA presented with Backache
BCC 2:
Patient with dermatmyositis presented with
sudden hip pain
She was on steroids .
Barnsley Hospital , Glasgow on 3rd of June
Station 1
Chest ILD with cushing
Abdomen Left Lumbar Mass(Renal ?)
Station 3
Neurology. Bilateral cerebellar Syndrome
Heart. Dextrocardia ( No clues , Notmal pulse,
apex beat not detectable , muffled heart sounds
.... must check other side to rule out
dextrocardia).
station 2. Joint pains and Colour changes of
fingers
Station 4.
Talk to daughter about the condition of her
father who is confuse after injury and fracture of
tibia and mild hematoma without midline shift (
on CT Brain ) .
He had atrial fibrillation and was on Warfarin
with Normal INR currently .
Daughter's concern were:
she was it being updated about the progress of
her father?
Will he become alright ?
Station 5 :
BCC1
Old lady presented with collapse after episode
of melena.
Pulse 110/minute
Postural Drop evident ( Given)
In history she told about NSAIDs use because of
OA
She was also on Warfarin because of previous
pulmonary embolism with Normal INR currently
BCC 2 :
Old aged male with stiffness and weakness of
muscles ( proximal as patient told inside)
ESR 85
Station 1
Resp.Cryptogenic fibrosing alveolits
Abd, massive spleenomegaly
Station2
Paroxysmal asthma, not taking inhalers
properly,
Was on MTX for RA for 8 years.
Station 3
Cardio aortic stenosis
Neuro
Wasting of small muscles of hand
Station 4
Presented with harmatemesis, malignant
looking ulcer on OGD, breaking bad news and
inform reg CT TAP
Station 5
Stroke 3yrs ago, now on/off involuntary jerky
movements left arm for last 3months
2nd, type 1diabetic presented with crampy abd
pain 6 months and weight loss 1 stone.
Yesterday exam at Reigmore center

UK Experience ,,,
The Exam in West general hospital in
Edinburgh,,, 6 / 2017..
{ copied from Dr. Zain group}
Station 2:
Outside information :
Fatigue pt with high createnin level discovered
accidentally during follow up.
Inside there is skin rash, joint pain, and sinusitis (it
will not be given unless you asked direct)
DD: I give wegner granum, goodpature , and SLE
examiners were so happy.
Qs: Investigation and ttt
Station 4:
Pt is a known parkinsonism , has UTI which result in
deterioration of her symptoms, but her baseline were
not that good , plan to talk about prognosis ..
Concerns Inside:
Prognosis? I said prognosis is variable, but your
condition will become worse (progressive disease)
-how many years until she will die: I said it difficult to
predict,
- dementia : she have a risk of dementia (mood,
memory,...etc)
-ttt: MDT: neurologist and memory clinic, refered
her for social worker, environmental and
occupational therapy.
Main concern : when she will die? I said
unpredictable she repeat it several time and I was
calm as Dr. Zain teach us and said no one can know,
difficult to predict surely the disease will shorten her
live , bad lifestyle but no one knows when she will die
Cardio:
Aortic and mitral metallic valve replacement
Neuro:
Cranial nerve examination (5th, 7th and 8th) cranial
nerve palsy .. I asked to do cerebellum.. one of the
examiner said ok .. I started to examine , the other
examiner said is it part of cranial nerve.. I said no , he
said please follow the instruction .. however at this
stage I already got cerebellum impairment,
Abdomen:
hepatosplenomegaly
Chest:
End inspiratory cracks with hyper inflation, and
clubbing, I said creptogenic fibrosis and
bronchoectasis
Station 5 :
1- pt has palpitation, inside the station pt has feature
of thyroid
2- pt present with blurred vision, in hx she has DM, I
took quick hx of DM, then V acuity and spent all the
remaining time in fundus examination: there was
laser scars and preproliferative changes
It seems very fair exam.
Please join me wishing our colleague a good luck &
success

UK Experience ,,,
Royal infirmary hospital abeerdeen ,,, 20 - 6 -
2017
Station 1
Hepatomegaly in female
Chest female with clubbing
Telangectasia
crepitation
Fibrosis vs bronchiactesis
I missed scar ?? Bikini scar
Station 2
Knee pain
Ankle pain
By asking
Loose motion
Red eye
His brother have psoriasis
Reactive arhritis
Septic
Station 3
Cardio
Midsternotomy scar in female
MVR
AF
murmur but I didn't mention it
Neuro
LL exam in male
UMNL with cerebellar manifestation
Nystagmus
Not sure because at the end the patient looks like
myotonia dystrophy
???
St4
Highly suspected cancer stomach by endoscopy after
haematemisis
Need to do CT chest and abdomen for staging
Task explain endoscopy result and the need to do CT
Chest and abdomen
I missed this scenario because I went more with that
diagnosis is cancer not highly suspected cancer
This was the trick ..
St5
Headache for 6 month
History of brain surgery before with the same
headache
She is taking analgesic for 6 month twice daily
No feature of acromegaly
No blurred of vision
Examine visual field and movement and field
Didn't complete examination for acromegaly feature
..
I think examiner wanted me to look for large toungue
..
St5
Palpitations in female
No CHEST or cardiac or stress no excessive smoking
no sweating no gland problem
Examination I found irregular plus
I said maybe arrhythmia AF
Examined heart and carotid ..

UK Experience *** Glasgow***Queen Elizabith


Hozpital 7th june 2017
St_2
pt w hx of knee pains presented w fatigue &
melena
pain associated w stiffness & improves w
walking
family hx of RA is positive
concern ::do i have RA like my family members???
St_4
communication w young girl who is married &
planning 4 kids who has been diag. w RA &now on
follow up visit as she was told to start Methotrexate in
her last visit & now she is back again to discuss the
plan after reading about the therapy on internet
St_5
BCC1
Man w multiple problems of Psoriasis Arthropathy
he had shoulder dislocation few dayes back & was
operated
Now presented again w shoulder pain & fever
BCC2
Young male w hx of crohns disease preasented now w
back & stiffness along w pain in the neck
################
Unfortunately that is ALL ??!!

UK Experience 6 / 2017
Hx
Known pt with DM, HTN, IHD,
C/o - lethargy & wkness for 6/12 , CBC - normal
On questioning -
Not following regularly
HbA1c 6m back 9 ,
MI 2 yrs back
on BB, statin, aspirin, ACEi, metformin, Isophane
insulin
Importance
Problem @ work deu to poor concentration
Mouth becomes dry frequently
Stopped smoking 2yrs back
Those were the only positive symptoms he gave & all
other questions were answered as no no no....
I was unable to come to a unified diagnosis
D/D
Uncontrolled DM ,
Renal failure,
psychological impact of importance
Questions
D/Ds
How you will investigate?
How you will manage?
20/20
3)
CVS -
Young male pt with VSD
Que
Positive findings
Investigations
14/20
CNS -
Ulnar nerve palsy - trauma scar present
Positive findings
Investigations
Management
17/20
4)
Parkinson's disease
Diagnosed 3y back
Not started Rx at diagnosis
This time came for UTI started on trimithopim &
responded well
Now the patient started on carbidopa before her
discharge from the hospital.
Explain to daughter regarding diagnosis, prognosis,
management , future plans
16/16
Hidden agendas
Hobby - drawing
Husband CVA bed ridden & only carer - do not
willing to give institutional care for him.
5)
BCC 1
Collapsed while walking with his wife
On questioning
Had a MI & fitted with pacemaker 15y back
Not on any medicine / regular follow up
Diag - Pacemaker dysfunction
How you will investigate ?
17/28
BCC 2
KT
On regular medicine & follow up
Derangement of renal function
On questioning
Recent use of NSAIDs for 1wk for mechanical
backpain after lifting a wt
28/28
St 1
RS
ILD / Bronchiectasis
Positive findings
How you will differentiate clinically?
Investigations
Management
19/20
Abd
Thalassemia with splenectomy scar & Hepatomegaly
Positive findings
Investigations
Complications
Causes of abd pain in this patient
18/20

UK Experience 6 / 2017
Hull Royal Infirmary Hospital
Cardio
valve replacement ? Mvr vs Avr with LL edema
viva about pro bnp and negative predictive
value
Andomin Renal trasplant
Neurology examine upper limb myotonia
dystrophica
Chest left lower pleural effusion
Station 5
1 Reumatoid artharitis
2 acromegally with carpal tunnel
History
reactive artharitis
Comunication
a 55 yr f with hematamesis found to have ulcer on
endoscopy looks like malignant and biopsy a waiting
ur consultant want to do ct scan abdomin and chest to
r/o metastesis please to to her and explain the need of
ct scan

UK Experience 6 /2017
Station 2
Wagner granulamatosis
Renal impairment given out side .. and pt present
with fatigue
Inside I got sinusitis
Joint pain
Skin rash
Urine frothy and red
Station 5 :
1st: palpitation
In the analysis I realized it is thyroid
I asked in detail about thyroid
I asked in detail about thyroid
I examined thyroid gland
Eye
Tibial mexodema
Other station 5 was diabetic
Present with blurred vision
I took he in quick, visual acuity and the remain of
time I spent in fundus
Preproliferative with laser therapy marks
Cardiovascular
was easy also
I had dual valve replacement
Neuro:
cranial nurve ex
7, 8, and 5
Cerebropontine angle lesion
Abdomen : HSM
Respiratory there was end inspiratory crack and also
hyper inflation
St 4:
Parkinson's with uti
To explain the prognosis
UK Experience
GOLDEN JUBILEE NATIONAL HOSPITAL
15th June 2017
Station 2:
History
Middle aged lady having Multiple sclerosis and on
regular follow ups has presented with cough.
it was dry cough more on lying flat with burning in
the chest
No other chest or heart symptoms
patient concern was
is that Aspiration pneumonia ???
diagnosis : GERD
STATION 4:
83 year old man has End Stage Renal Disease with
underlying DM and HTN .
was admitted with Pneumonia and herat failure 1
month back and recovered.
was advised Dialysis which he refused at that time .
Now talk to the son who is insisting for Dialysis as he
says his father says yes to him but refuses when
doctors talk to him about dialysis.
Assume the permission was given by the father to talk
to the son about his condition.
( Father concern was that he lives alone and can not
manage to go to hospital regularly for the Dialysis )
STATION 5:
BCC 1:
Man presented with weight loss and palpitations .
concern was why i get lot of sweating .
BCC2:
Man with previous history of Aortic valve
replacement now presented with joint pains including
hands and Knee !!
history was suggestive of RA with acut flare.
viva was also about Hyperuricrmia?
Concern was will it affect golf which i play since long?
Can you settle my Deformities?

copied from Dr. Zain group


sharjah experience on may 17th , al qasmi
hospital, sharjah,3rd cycle
I started by station 4 , 38 years old lady , she is
diabetic for 22 years, on insulin, speak to her
and explain the importance of proteinuria ,
discovered
by her Gp,and the importance of adhenence to
control ,that is the task. she has diabetic
retinopathy also, she born tow pre mature baby
before.long scenario
I introduce my self to her, confirm identity,
agreed agenda, I asked if she want any one to
attend, shae said no.I asked her tell me more,
she told I have aprotein in urine and i am afrid
am I going to dialysis? i told her I am her to help
you, let us to think how to control your sugar,
she told how doctor, every doctor tell me do
that and that, and go there and there ,and i am
busy with my kids and my sick mother
here I showed empathy , i told her i will help in
this issue , i will refer you to social worker.
about your sugar , you need to follow some
restrictions regarding your diet, exercise, and
follwo up,
and I will refer you to dietician , he will help
more
also I will refer you to gland doctor, and eye
doctor for regular check up , here i asked her do
you have other problems, she told like what?
I told nerve problems, she told no, I forgot to
ask about macro complication
her concerns:
end in dialysis? I will be blind ? I told her this
will depend on control of your sugar, if the
sugar controlled and you follow diabetic diet ,
with regular follow up we can prevent
further deterioratin in kidney function, and
vision problem, and if the sugar not controlled ,
this will lead to more deterioration in kidney
function, and might you go to dialysis,
I hope this will not happened to you.I
summarized to her, check understanding,
agrred plan : referral to specialities as above,
offer help , contact number , leaflets , web sites
she told me I conviced doctor, then thank her
Examiner questions: did you think you
convinced her? confidently I said yes. he asked
why , I answered , because at the end she told
me I conviced
he asked me , did you bleive her, I replied yes.
he asked what important thing confirm that she
convinced and she can follow your plan? he
told me , if she agreed to come to follow up, and
you did not ask her for follow up
what ethical issue in this scenario? I told
autonomy
what is the cause of complication in diabetes?
he told I mean pathophysiology , I answered
microvasular complication.
why you refer her to social worker? I answered
to help her , because of her mother is sick.I got
11/16
station five:
BCC1 : 38 years old lady has neck swelling for
five years, all vital sings not avilable N/A.this is
the scenario. I think about thyroid problems.
BCC 2: 18 years old has spasm and shaking of
her right side of body. this only in scenario,
from outside I put d/d of myotonic epilepsy,
wilson disease, hemiballismus , because these
are common in station 5 in sharjah centre.
when I entered the room they told me your first
case on left, I saw young lady on the rigt side
with clear adenoma sebaceum on her face, so I
confirmed my second case is tuberus sclerosis
BCC 1: after introduction and greeting , can you
tell me more about this swelling, I asked her
about symptoms of hypo hper and pressure
symptoms, she has only constipation, family
history of similar problem in her sister, I asked
her about other auto immne diseases, are
negative, durg history , she is taking thyroxine,
her concerns is it curable? it will affect my kids?
i told her yes it is curable,regarding to your
kids, you have some we call it auto immune
thyroiditis , it is not inhereted , but your kids
have more chance to get a disease,
examination: smooth diffuse goitre, no eye
signs, no retrosternal extention, no proximal
myopathy, no pretibial myxedema, no lower
limb edema, I asked to do ankle reflex ( not
allowed )
examiner questions:
what is your diagnosis? hashimoto thyroditis.
whatis your findings ? diffuse smooth goitre.
clinically what her thyroid status? euthyroid
what other d/d than hashimoto ? It could be
simple goitre. what investigations you want to
do ? A: thyroid funtion tests, thyroid ultrasound,
thyroid Abs. what test use for follow up ? A ;
TSH. also asked about frequency of follow up.
what treatment? I will refer her to
endocrinologist and to continue on thyroxin
time finished, I got 28/28
BCC2 :
as scenario above, there is young lady and
surrogate beside her, after greeting and
introduction, I asked her tell me about that
shaking , she gave description of tonic clonic
seizure, I asked about performance at school,
she told me low performance, I asked about
rash in face since when ? she told since
childhood. any other rash , she told no
i asked about respiratory symptoms, GIT
symptoms, eye symptoms all are negative. I
asked about similar problem in family , No. drug
history : she is taking two anticonvulsants , one
is topiramate , other I dont remember
no significant past medical history.I examined
the rash on face , rash in distribution of butterfly
, adenoma sebaceum, I asked to examine the
back for shagreen patch, not there, I asked to
examine the trunk for other rash , they told
leave it, I examined the eyes by torch light,
nothing
concerns: is it curable ? I explained to her she
has some we call tuberus sclerosis , it run in
family , it is not curable but we can control it , as
she taking this medication to prevent fits.other
concern : that medication is correct medications
and if she can continue on them ? I told her I
will refer her to nerve doctor , who will check
that medicines and who will decide to continue
or to change.
I told her disease can affect respiratory , gastro
or eyes , IF any symptoms we will refer her to
specific specialty.
examiner questionS
what is your diagnosis? A : tuberus sclerosis .
do you think the rash related to her fits ? A : yes
, both are features of tuberus sclerosis. do you
think she taking correct medicatins ? A: yes but
I think not correct medications, because
examiner told me you did not ask about
frequency of fit , she has two attacks last week,
so I think she need to change anticonvulsants.
what investigations? A : ct brain. what you will
find in CT? A: tuber and calcification time
finished I got 23/28
station one : abdomen : scenario , examine the
abdomen of this pt , who came for follow up
young boy, pale , jaundiced, no signs of CLD ,
palpable spleen about 5 cm below costal
margin, liver: right lobe 4cm bellow costal
margin,liver span is 12 cm, left lobe about 5 cm ,
no ascites, there is tanned face
examiner questions
what is your positive findings? A: as above
what is your diagnosis? chronic hemolytic
anemia . what exactly ? A: thalssemia
what is D/D? A: I told myeloproliferative , she
told me myeloproliferative in this young patient
??, then I told infiltrative disease, she is not
happy also, I told chronic liver disease,
accepted, what aganist it ? no stigmata of CLD,
what investigations ? A : u/s abdomen,
complete blood count, HB electrophoresis,
blood film, and investigations for complications.
what complicatin? secondary hemochromatosis
from repeated transfusin, how you treat this
complication ? A; iron chelating agent like
desferoxamine .
what is cause of death in those patients ? A :
cardiomyopathy I got 20/20
chest : scenario is examine chest of this patient
who has shortness of breath
elderly male with cannula on his left arm,
anterior chest examination , for me normal at
this point
back examination: wide spread ronchi and
creps , I recalled from anterior examination,
percussion seem like hyper-resonant
examiner questions: what are positive findings?
A : as above
what is your diagnosis ? obstructive airway
disease most probably is COPD with acute
exacerbation.
what is clue in his hand for COPD ? A: I did not
find, I think he mean tar staining .
what are investigations? cxr , pulmonary
function test , CBC .
what is treatment for acute exacerbation ? A :
admission , i.v steroids , B2 neublization ,
antibiotics , oxygen.
what is treatment for COPD ? none
pharmacological and pharmacological in
details.
what are the indications for long term oxygen
therapy ?
I got 19/20
station 2 :
scenario: 35 years old , tody discovered at Gp
clinic , he has glucosuria , he has history of
uncontrolled hypertension , anxiety , chest pain.
in the history : no syptoms of DM, other positive
findings are sweating , headache , palpitation ,
family history of death of his father due to
kidney problems, no symptoms of MEN 2, no
symptoms of other causes of seconadry
hypertension, I put diagnosis of
phaeochromocytoma, I explained to patient the
diagnosis and treatment
his concerns is about diagnosis and is it
curable , I told him it is curable.
examiner questions ? what do you think about
cause of glucosuria ? A: related to his
phaeochromocytoma
what are the causes of false positive glucosuria
?
what is your diagnosis & D/D? A:
phaeochromocytoma on top, D/D , other causes
of seconadry hypertensin, anxiety
what are the investigations?
why you did not assure the patient?
I got 10/20
station 3:
cardio: scenario, examine the heart of this lady
who has shortness of breath.
pakistani lady, look in thirties, with other young
lady for translation
low volume pulse, midsternotomy scar, no
harvesting scar, palpable second heat sound,
loud second heartsound.I did not hear any click
sounds
what are your finndings ? as above
what is the diagnosis ? A: mitral vave
replacement , I guess it because pt is young, I
am not sure but I mentioned It.
do you think she is in failure ? A: no
what investigatios ? A: echo, ecg. cxr, INR
what target ranges for MVR?
I got 7/20. for me the case is not clear and I
know other two candidates got 7 , one put AVR,
and other DVR.
neuro:
examine the lower limbs of this patient
middle age male , he has hemiplegic flexion of
his left upper arm
normal examination on right lower limb
hypertonia, weakness , hyperreflexia , up going
planter on left lower limb.
examiner questions:
what are the finding? A: as above
what is your diagnosis ? left sided hemiplegia
given his flexion of his left upper limb.
where is the lesion ? A: internal capsule. what is
the side? A: right side
what investigations ? A: CT brain in acute stage
to exclude haemorrhage
what is mangement ? A: in acute stage , I will
refer him to stroke unit, what is treatment right
now ? A : seconadry preventions: control of BP
if hypertensive, mangement of DM, and
hyperlipidemia and other risks.
I got 20/20
thanks god I passed the exam

Am one of the silent observer here, just want to


share my recent exam experience in brunei.
Alhamdulillah, i've passed the exam.
PACES Brunei May 2017
St 2- 48 yo lady with abdo pain n constipation.
Concern - father died of colon cancer. Further hx,
back pain, polyuria, polydypsia, reduced
concentration, hx of kidney stone. Previous hx of
breast ca. DD - hypercalcaemia, hypothyroid
St 3
CVS - pulmonary stenosis
CNS - MG (bilateral ptosis, bilateral 7th CN LMN,
nasal speech, fatiguability of eye n UL)
St 4 - elderly lady with moderate alzheimer n OA
with fall at home, recurrent hospital admission.
Admitted again for UTI n become more confused in
hospital. Speak to daughter. Daughter concern why
more confused n demand ct brain. Ethical issue,
autonomy, do good, do no harm. Hidden agenda, need
to discuss on DNR and advanced directive, referral to
palliative team.
St 5
BCC 1 - pt came with fall - peripehral neuropathy
BCC 2 - young lady with right calf pain - need tro dvt
coz hx of miscarrige n recent long haul flight.
St 1
Respi - right pneumonectomy
Abdo - thalasaemia with splenectomy (pallor,
jaundice, hepatomegaly, splenectomy scar)

Exam Experience {My sister} ... Egypt May 2017


metalic mitral valve
neuro : ulnar n palsy
Resp not sure
abdomen hepatosplenomegaly
Station 4: medical error, administered trimethoprim
for a patient on methotrexate fir psoriasis so she had
bone marrow suppression, discussion about the
mechanism by which they have caused it
Hx taking : poorly controlled asthma, likely
occupational Vs churg Strauss
Station 5: proximal myopathy due to
hyperthyroidism and goiter ,
frequent seizure in known previously controlled
epileptic y likely drug interactions as he was given
clarthomycin recently

I passed PACES from dubai. Thanks to this


group for the support!
Cases to my concern
Station 4. Task was talk to the daughter( NOK)
about the diagnosis of her father who is 70 yrs
Known AF and HTN and was on warfarin. Now
got brain haemorrhage, confirmed on CT brain.
INR was below threshold level. Bp well
controlled
Q why this happened to my father?
Q was it due to medicine ?
Q should he continue that medicine
Q what will happen now
Q I want to complain
Concern. I am the only daughter , my husband
supports me
I asked the cardiologist not to start this
medicine but....
My answers ... medicine is not the cause.
... team will take care of him and will decide
what is better for him . Meanwhile we will stop
that medicine
Examiners Qs
1) what are the ethical issue here?
I told autonomy .. examiner not happy
Then I told BBN .....examiner was totally not
agreed
Then he asked me what is malfience ?
I answered it
Q what do u think that was right decision to
start with warfarin for this patient? I said yes as
he was with AF. But he was not happy
Q what will u do now ? U will start warfarin or
not? When u will restart?
I said sorry I will ask for opinion from senior and
examiner smiled and said what is yours
opinion? I said at the moment warfarin will be
hold.
I missed to guide her to make complaint as she
made bombardment of another concern as she
is alone .... I told examiner there was no time but
if I had time will guide her.....score 11/16
Statition 2 : 45 yrs old with transient loss of
unilateral vision .
Past h/o blindness in other eye 5 yrs back due
to retinal vein thrombosis. Family h/0 stroke and
heart Attack.
Medications list was there.
Diagnosis. Transient ischemic attack(
amarousis fugax)
Examiner Qs
1) what is the cause ?
2) investigations
3) management
Concern:
Driving
Would I need admission? Got 20/20 ( amazing)
Station 1 : respiratory case
Examiner command : this lady has some
shortness of breath . Examine her back of
chest....
it broke my scheme as Pt was total blind when I
told her please can you make yours hands like
this
I did not do proper general examination and
findings at the back were scattered all over
chest but more over left lower zone.... confused
crackles... not clear about diagnosis ..not clear
work up and same management ....11/20
CVS:; very interesting case
When I entered the room , Pt was angry to the
examiner as he wants to go home. I tried to
make him relax that I am the last dr to check you
as that was last station of the cycle...he became
relaxed . Thanks to him. Then I read the case.
"THIS PATIENT IS POST CARDIAC CATH......"
Findings. Unequal radial pulses ....left radial
pulse was not palpable.
There was thrill over RT radial pulse and I told
irregular....
Precordium .... nothing found even heart sounds
were muffled...
Examiner asked wat is diagnosis??
I said unequal pulses and iatrogenic AV fistula
post cardiac cath
Examiner asked on which side cardiac cath was
done..... I was lost because RT and left radial
pulses both were with findings.... I said may be
repeated both sides....not happy. Investigation
and management.... examiner asked be
specific..... I felt this case also misssed as
respiratory case but I got 17/20 ( awaiting for the
correct diagnosis)
Good luck

Dubai Paces exam. 15/5/2017


Station 4:
A pt. with COPD with frequent exacerbations
and hospital admission on Theophylline and
monitoring serum level regularly admitted by
chest infection and clarithromycin given for him
causing Fits. He is a taxi driver.
I address the problem, apologize and ask him to
tell DVLA.
Examiner asked is this medical error I said yes.
Station5:
1st:Neck swelling:Throidectomy scar.
I took a brieve history and DD symptoms of
hypo and hyperthyroidism and concern.i did
examionation for thyroid,hand and visual field
of eyes,lymph nodes.
Nothing except thyroidectomy scar. Examiner
ask about investigations.
2nd: Young Pt. complaining of dizziness and
lightheadness when standing from sitting
position. Type 2 DM on insulin and metformin
uncontrolled.She has abdominal fullness .Data
given BP supine 135/90 and standing 115/70 like
that.
I took history excluding the causes,address the
concern.No physical findings, I asked to
examine the heart he told me no needbecause
she is normal.
Asked me what is your diagnosis:Postural
hypotension,autonomic neuropathy.
Other causes:Fits,hypoglycemia,cardiac
arrhythmia.
Asked me about investigations and
management,,,the bill ringing.
Station 1: Male with abdominal discomfort.
Abdomen:Renal tx,no functioning,tender,many
AV fistulas one of them is working and recent
punctures there.
She asked me what is your
diagnosis,nonfunctioning renal tx,how did you
know.Complications of medications
physically:striae ,cushingoid face.
She asked me what is in mouth, I told her my
gingival hyperplasia,she did you check I told her
I am not sure really
Investigations :urine, C&S,cbc,ca ph,PTH.
Chest:Pt. with dyspnealt. pleural effusion.
Asked about finding,investigations.
Station 2: History taking
Femal pt. around 45 y complaining of recurrent
central abdominal pain,diarrhea,no bloody
diarrhea,no loss of wt. no NSAIDS.
History systematic,concern about the
diagnosis,cancer,admission or not I said less
likely to be cancer,no need for hospital
admission.
Examiner for DD and analysis.
Station 3:
Neurology:Young male about 35 y. with
difficulty waling.
Neurological ex:Hpotonia,loss of joint
reflexes,power normal.Sensation fine touch and
pinprick impaired,heel shine test intact,no time
for deep sensation.
UK examiner:I was hesitated about the
weakness at first then I told him there is
hypotonia,abscent joint reflexes ,impaired
sensation,the he told me whatis your diagnosis I
told peripheral neuropathyDM,vit. B12 def he
repeated vit. B12 are you sure I told yes.
Investigations:NCS,EMG,Blood
sugar,HbA1c.ringing bell thanks God.
Cardiology:
Pt with neurological insult for cardiology
exam.Pt. has hemiparesis.
I did general exam pulse irregular.
Local inspection no thing,palpation no
thing,Auscultation no murmur..i am surprised
no murmur???
I forgot to check carotid and auscultate the pt.
sitting for AR??
Final minut:I checked bach chest bilateral fine
crep. Basally.
LL edema ++,sacral edema.
what is your diagnosisI told her no signs of IE
,pulse irregular she asked which irregularity I
told irregular irregularity AF.
Really I couldnt detect any murmur?? The
looked for me I said yes.
What is your diagnosis I tod them congestive
heart failure.what is the cause I told may be the
same causing cerebral stroke,IHD,mycordial
infarction.
Investigations:I told searching for risk factors
ECG,ECHO,lipids,blood sugar,BP.
Management:For AF formal anticoagulation ,she
asked how you decide I told CHAD VASC score
,diuretics.
UK doctor ask me you did not check carotids
???
I told him yes I have to check and doing carotid
doupler for internal carotid stenosis,he said
GOODI feel bad to forget carotid and sitting
the pt. and hear aortic area but it is very
distressing during exam. Short time.
Your comments are highly appreciated.

Firstly, Thanks Allah, for his inspiration and


Tawfiq. I passed from the FIRST ATTEMPT. Till
now I'm surprised by this happy news and I
hope all success to all of you.
Thanks to all my teachers, mentors, members of
this wonderful group, my family and my friends.
I will tell you my PACES story in details hoping
that it will be beneficial to you.
Station 1:
I started with Station 1, I was very afraid about
the exam at that time. I remembered some
prayers at the time and read it. Then, I prepared
myself as I will start with the Chest Station.
The bell rang. "Okay this is your abdomen case,
start doctor" the examiner said. I got stressed at
this moment because I'm preparing as I'm
entering a Chest case. On the board patient is
asymptomatic. I started. There is fistula in the
left forearm, which is not working. Patient is
mildly pale. There is right iliac fossa scar with
mass under it. I concluded that, this gentleman
has renal transplant which is working well, with
mild pallor. Examiner kept asking about pallor
"Are you sure ???" more than 3 times. I'm afraid
about inventing signs. He said: "what else ?" I
said: "maybe the patient has gum hypertrophy I
didn't concentrate on it, I would like to examine
if you give me a chance ". He said: "if the
patient has RIF pain, what did you think about
?" I said maybe graft failure". "How you will
follow ?" "Renal function, vital signs and urine
outpt". "What other differential diagnosis ?"
"Maybe infection". "If the patient develop
glomerulonephritis, what is the best
investigation to confirm the diagnosis ?" "Renal
biopsy". Bell rang. My impression is that I did
very bad in this station. I got 20/20.
Next case, Chest. Young male with SOB on
Oxygen with nasal cannula. I started my
examinaton. Cachexic patient with Clubbing.
Patient is tachypnic. There is left thoracotomy
scar. Chest move better in the other side !!!
Variable percussion note. Bronchial breathing at
left upper lobe. Crepitations all over, change
with coughing, with scattered wheeze. Trachea
is central.
1 minute left and even I didn't start examination
from the back. Time finished. I concluded that,
patient has bronchiectesis evidence by the
previous signs. He kept asking "what is the
cause of thoracotomy scar ?" I gave a
differential diagnosis of upper lobectomy and
other types and what with and what aganist. He
kept asking me what else, what else about the
differential diagnosis and the causes of
thoracotomy scar until the end of the station. I
felt completely lost . I got 19/20
Station 2:
I moved to station 2, with a feeling of
depression. History of young man, 28 year old
who lost his consciousness for 2 minutes while
he was playing football after feeling some
palpitation. This is the first time to him to lose
his consciousness. He got tonic clonic
convulsions, without eye rooling up, without
tongue pitting and without loss of sphincter
control. He waked up completely well, and went
home. He had previous history of palpitations
13 years back. He is adopted. Nothing more.
I gave a differential diagnosis of Vasovagal
attack, Epilepsy, Cardiac causes
(Arrhythmias/HOCM) and Hypoglycemia. He
discussed with me to put it again in order. After
that I put cardiac causes firstly. He asked me
about each one' investigations. Then the bell
rang. I got 13/20.
Station 3.
At this time, I remembered what Dr. Zein said to
us: "Take every station as an independent
exam. Don't be bother about your performance
in one station. Forget about it and move on".
Cardiac case. It was straightforward. MVR. He
asked me about investigations and the causes
with some discussion about the prophylaxis. I
got 19/20
Neuro case. Middle age man with difficulty in
walking. Examine the lower limbs. Examiner
adviced me to start with the gait. Mmmmm I
think for a while and then I start with it. Wide
base gait. Patient has cerebellar signs in both
legs with loss of superfecial sensation in the left
leg up to the inguinal region. Examiner didn
't allow me to move more. I concluded that this
paK A'Rahman Ageeb:
tient has cerebellar signs with superfrcial
sensation loss for Differential diagnosis. I said
MS, and then I had brain block . I couldn't
remember any Differential diagnosis and he
was waiting. Then he asked me about the
investigations and the treatment. I got 14/20
Station 4 about 60 year old man, hypertensive
for 8 years on treatment. He got DVT before 6
weeks, and started warfarin. He came yesterday
to the hospital with loss of consciousness. His
Blood Pressure at admission was 190/110 and
INR was 4.0. CT brain showed brain
haemorrhage (I think they didn't tell more than
this). One day after, the patient deteriorated
more, desaturated, intubated and admitted to
ICU. Neurosurgery opinion, they said they can't
interfer because of the condition of the patient.
Your task is to speak to the wife to tell her about
the condition, the prognosis and the plan.
After opening and After taking her idea about
her husband' condition, I explained to her
everything about his condition. She asked:
"what is the cause of the haemorhage ?" I said:
"maybe his high blood pressure, maybe from
the warfarin that he takes, but his INR at
admission was 4, and maybe other causes. She
said: "Is this the fault of the GP that prescribed
the Warfarin ?" I said: "it is unlikely, because
your husband take the warfarin now for almost 6
weeks and before he didn't have any problem
with it. She said: "Then any thing to be done to
him ?" I said: "we will do other best, we will give
him his feeding and antibiotics that he need and
close observing his vital signs but unfortunately
his condition is not good". She said "Any
surgery ?" "The opinion of our neurosurgery is
that, he can't not interfer with your husband
case because of his condition". "Will he die
soon and when?" "I can't say to you when, but
unfortunately he may die at any time". "Will I
call my son to see him ?" "Yes, please as early
as possible". Then, she kept to ask me, any
more help, any surgery to my husband, and
transfere to another hospital until the end of the
station. I forgot to give her another chance to sit
with my consultant if she want.
Examiner asked me "Did you think she is happy
about the plan ?" I said "no". "What you will do,
then ?" "I will give her chance to sit with my
consultant if she want". "Okay, nice, what is the
cause of this brain haemorrhage" I said: "may
be his high blood pressure, maybe from the
warfarin that he takes but his INR at admission
is 4, and maybe other causes like berry
aneurysm associated with polycystic kidney
although she denied any previous history of
renal problem or family history". "Is these
Differential diagnosis written FRANKLY in the
letter ?" No. "Then, don't say what is not
written" ..... "What is the percentage of
survival from his condition" "What is the
percentage of survival from cerebral
haemorrhage in general" and questions like
this. I got 5/16
Station 5
1st one is 28 year old female presented with
headache for the last 6 months, increased in the
4 weeks. She said it is mainly unilateral,
throbbing in nature, relieved by ibuprofen,
associated with nausea, no vomiting, more at
morning, with positive family history of same
condition. All other history are clear (no history
of clots, no skin rash, no joint pain, no
weakness, no ulcers, no symptoms of anemia,
no eye symptoms, no local symptoms, no
medicines or OCP). All vital signs are normal.
Brain block happened . I don't know what to
examine. I didn't do fundoscopy or offer it.
"Doctor do I need to do CT for my brain ? I'm
afraid It maybe cancer" "No need for the
moment, it is most probably Migraine. We will
refer you to the neurologist and after medication
if no improvement may be we will need CT
brain". Examiner questions: "What is your
Differential diagnosis ?" "Most probably it will
go with migraine, I can put medication over use
headache, although I forget to ask about it".
"Will you do CT brain ?" No need for the
moment, it is most probably Migraine. I will refer
her to the neurologist and if
needed we will do CT later". I got 18/28
2nd case: around 60 year old gentleman
presented with loss of vision. His all vital signs
are normal. He has history of brain surgery
(Hypophysectomy) 2 years back, and he usually
takes his medications. He has gradual loss of
vision over weeks to months. No eye pain or
headache. No other symptoms at all. He has
acromegalic feartures. He has bitemporal
haemonepia. When I start to do fundoscopy, 2
minutes remaining and another brain block
happened . I think there is papilledema but
I'm not sure. Examiner asked "what is your
diagnosis ?" "Most probably recurrence" "What
did you see in the fundus" By mistake I said
optic atrophy, and I didn't want to return to say
papilledema because I'm not sure. His face
changed. Then he asked: "what else ?" I said
"silver wiring". He said "What is the best
investigation that you will do ?" I said "MRI
brain". I got 24/28.
My total score was 132/172.
GOOD LUCK.

Egypt / May /2017


Regarding exam today 3rd carousel
St 5 first case is inability to walk
nd one is inability to lactate post delivery2
Chest pneumonectomy
Abdomen thalasemia
Neuro paraplegia with level
Cardio mitral valve replacement
Hist back pain in a women post menauposal with
mastictomy 15 years back
Communication suspected cancer stomach waiting
investigation
Sharjah exam today =17 / 5 / 2017
Cvs;
Cabg scar , av fistula, systolic murmer on apex
with radiation
Cns;
Motor neuropathy
Resp;
Right side pleural effusion in ckd patient and
bibasal crepts
Abd
Renal translant
St 5;
1.excessive hair growth
2.gradual visual loss 5 yrs now blind
St 2;
Female with ankle swelling, joint pains , dysnea
and cough
St 4
Anaphylaxis in chef for discussion with
seriousness of disease and job implication and
treatment
Exam Experience Egypt May 2017
Cairo today
St 1
CLD with ascitis
COaD with possibility of Rt up lobe fibrosis
St 2"
Bronchial asthma
Ca bronch
St 3
Mixed mitral valve
AR
Spastic paraplegia
St 4
Unawareness of hypoglycemia
St 5
Behcet disease. V vasculitis
Polyglandular failure

Anaphylaxis scenario.. {Copied from Dr .Zain


group}
Communication station:
Middle age man work as a chef in a restaurant.
He's known allergic to peach and peanuts.
Brought to the casualty after collapsing at work
just after eating a pear
He doesn't have known allergy to it before. He is also
asthmatic. In ER he was in anaphylatic shock for
which given ttt then admitted to icu. After 2 days
become more stable
Patient concerns: 1.This my only job what
I will do, advice to change his job in the same
resturant but not in the area that he will expose to
substance that patient is allergic to like casher and we
can talk to occupation therapy
2. What abou my kids? This allergy can run in
family ,we need to involve specialist DR to test your
kids and to see if they allergic to common substance
3. Can I die: Death is one of possibilities, we need
to do some protective/ preventive measure like we will
give you small pen contain substance called
epinephrine in case of allergy you can use it
4. Can this allergy occur again: Yes if your are not
avoid the substance that you are allergic to.
From the discussion patient: he changed his house, I
asked him whether the allergy start when he transfer
to new house he said no
Reagarding concern 3: I have advised him that
we will give a medical alert card in your poket
Examiner's Qs
Q1: What if patient collapsed at home alone: We
can arrange teaching scessions for his wife and if his
alone he can use EPI pen & call the ambulance.
Q2: What you will write on the card that you will
give to patient? The substance that he is allergic to,
treatment that patient should recive, take me quickly
to hospital
Q3. Any other possibility of something else that can
lead to this patieny allergy?
The new house but unlikely because the allergy
started after he ate the pear
He got 16/16

Sharjah cycle 1 & 2 May / 2017


History : pain in wrist and knee joint for month ,
sob, pain in shins for 3 wks, ,female 52
,sarcoidisis
communication: anaphylaxis in a chef ,allergic
to salad, talk to him regarding to diagnosis , job
, treatment
Sta 5 : hirsutism for dd , cushing ,pcos
bcc 2 : deterioration of vision , esstential intracranial
htn, optic atrophy

Dubai - May 2017


The first cycle 15/5
History: sudden blindness ( amurosis Fugaus )
Comm:- pt elderly with AF on warfarin.
Develop left sided weakness & INR = 1.5 .. possible
hypert hage bleed ::: discuss with his daughter.
Station5
1- post-MI came with lethargy
On lisinooril . Statin , bisoprol & aspirin
?? Postural hypotension , statin se,
Anemia
2- elderly lady 60yr came c/o back pain, k/c
osteoporosis on treat
Temp 39
DD: fracture discitis

Alhamdulillah i have passed paces


First attempt in Royal Hospital Oman held on
8th April 2017
Scored 146
Well before i share my experience I would like to
thank dr Ali Hameed
Dr Ali Raza
And all Mypaces team for their generous
support and a very special thank to Dr Abdul
Fateh
I learnt alot from his course and it really helped
me all through preparation especially
Neurology which was the big elephant for
me since final year MBBS
I got my result a little later then my other
colleagues which made me think Mera
Faisala bhi Mehfooz ho gaya hai (my Pakistani
colleagues can relate to this joke )
Down below is my experience (i was very sure i
failed because I didn't follow my scheme during
exam that i had made while preparing for the
Exam but they say miracles do happen )
Started with station 4 delayed diagnosis of
pheochromocytoma
Surrogate was not aware of the diagnosis which
i had to break and give him reasons for delay in
diagnosis
Dont know what i did wrong got 3/16
Station 5 BCC1 frozen left shoulder in Diabetic
pt
In the d/d I didn't mention Frozen joint at all and
kept beating around the bush got 25/28
BCC2 was Toxic Goiter staright forward got
26/28
Station 1 unsure of the actual diagnosis
of Resp case but i diagnosed as COPD with Bi
Basal Fibrosis/bronchiectesis (i know doesn't
make sense ) pt didn't have tht wet type of
cough but crepts were course and didn't change
with cough
Viva was usual on management of the pt and
investgations got 18/20
Abdomen was Renal Transplant due to Apkd
,well here also the examiner pulled diagnosis
out of stomach as i was again not telling the
actual diagnosis in my presentation ,and i also
failed to identify the transplanted kidney(i said
may be he had some abdominal surgery for
some infection ,what was I thinking still
got 17/20
Station 2 coelic disease (lady with fatigue ankle
swelling and Microcytic Anemia) Viva was about
the investigations specifically asked me the
Antibodies and management of coelic got 18/20
Station 3 (my most feared but the best station
)
Cardio was young male around 25 yrs with AVR
no murmur at all,though he had Afib,viva on
management of replaced valve pts ,I mentioned
all the usual with diet and medicine care, he
asked me which is the most common medicine
interfering with INR ,i said Antibiotics got
20/20
Neurology was young 25 to 30 yrs probably with
involvement of post columns and PYrmidal type
weakness in L/L left then right
Reflexes exaggerated in right diminished in
left(appeared with reinforcement) i gave D/D of
MS ,syringomelia,Friedrch with MS on top due
to patchy involvement ( remembered a line by
my hubby when nothing makes sense its MS )
Viva was usual on investigation and
management of MS got 19/20
So now my advice to all preparing
Have a study partner
Think simple in exam
Do loads of prayers (as i did )
Allah make things easier for all of you
Ameen
Thanks again to all my teachers who taught me
my parents my in laws they were huge support
to all my whining and self speculations of
failure

Dubai 17/May 2017


{Copied from Dr .Zain group}
station 4
Conunseelling about proteniuria, adherent to
medication
Station 5
BCC1: Hypothyroidism, goitre
BCC2:tubeus sclerosis
Abdomen, thalassemia
Copd chest
Station 2
phaeochromocytoma
Cardio: valve replacemet
Neuro, hemiplegia
Dubai - May 2017
History :
recurrent chest infection
Communication:
Hypoglycemia unawareness
Examiner asked about the cause of Unawareness,
B blocker is written in the scenario
#####################

Dubai - May 2017


First day 15/5 third cycle :-
History:-Diarhoea with FH of Cancer.
Communication :-
Theophylline toxicity after give clarthomycin &
developed seizure
Jobe: taxi driver
ClNS :-Caw hand
Chest :- Pleural effusion
Abdomin:- chronic liver disease
Cvs:- CABG with murmur & AS
Dubai /May 2017
{Copied from Dr .Zain group}
St_3
metalic aortic valve,
neuro : ulnar n palsy
St_1
Resp not sure,
abdomen hepatosplenomegaly
St_4
Station 4: medical error, administered trimethoprim
for a patient on methotrexate so she had bone
marrow suppression, discussion about the mechanism
by which they have caused it
St_2
Hx: poorly controlled asthma, likely occupational Vs
churg Strauss
St_5
Station 5: goiter, frequent seizure in known epilepsy
likely drug interactions as he was given clarthomycin
recently

Exam experience in Glasgow in Golden Jubilee


Hospital 23/3/2017
I passed alhomdellah with 148/172
This is my first trial in PACES but i had the
sense that i know what to expect in the exam
from the experience that had been shared from
other colleges so i would like to thank
everybody who share his experience.
I started with station5 and when i knew that i
panicked because i thought if i messed it up i
will fail but then i said to my self just do what
you can and forget about it in that room don not
take it to other stations.
The first case was a lady with weight loss and
type 1 DM so i put from outside grave' s disease
and coeliac disease and when i entered the
room it is obvious that it is Grave' s so i started
by shaking hands with the patient and then she
gave history of gritty eyes and weight loss with
good appetite and diarrhea and then i started
the examination by checking the eyes she has
led lag and retraction and ophthalmoplegia and
proptosis and then i asked her to take a sip of
water she does not have goiter but i examined
the neck and there is no finding and in the
beginning i checked the pulse and it is regular.
She has positive family history of type 1 dm and
she is on b blocker and no smoking history and
not drinking alcohol and her concern was what
is my problem so i explained the grave, s and i
refer her to endocrinologist.
The examiner questions were what is the
diagnosis and what you find in the eyes and
what is the investigations and the bell rang and i
did not finish the discussion and i forgot to see
the legs for pretibial myxedema and the
proximal myopathy and i knew that i will lose
marks and i got 23/28
The second case was systemic sclerosis
I became calm and confident in this case
because i found that there is nothing to worry
about it went fine the first case. The scenario
was a man came with discoloration in his
fingers and pain so i went inside the room and
the patient has obvious talengectasia. I started
taking the history and the patient has rynaude '
s with dysphagia and shortness of breath and
he is a builder so i examined him i saw the
talengectasia and i saw his hands he has ulcer
in his finger and the skin also he has lung
fibrosis with fine crackles and then i asked the
examiner about patient BP to exclude the renal
problem and the examiner was happy. The
patient concern was what i have and then i
explained tge disease and i told him that i will
refer him to rheumatologist and i advised him to
keep his hands warm and to avoid vibrating
tools.the discussion was on diagnosis, finding
and investigation and treatment and i knew that
i will have full mark and i have it 28/28
Station 1 :
The abdomen case was liver transplantation,
patient with Mercedes Benz scar so i started the
examination and the secret is to be fluent do not
think about what you want to do next, the
examiner wil Will have good impression
The scenario was patient cane with abdominal
discomfort.the discussion was about what is
your finding ? The patient has tinge of jaundice
and the scar and the liver is palpable six cm
below the costal margin and no signs of chronic
liver disease so my diagnosis is that the patient
has liver transplantation and the discomfort
could be due to rejection or portal vein
thrombosis or hepatic vein thrombosis and i
should say biliary stenosis but i forgot.
Then he asked about what might be the cause of
transplantation.
What the think you will do before you send the
patient to the liver unit for transplantation and
what are the side effects of immunosuppression
medications.
I got 17/20
The respiratory case was an easy one but i
messed it up. Patient came with shortness of
breath. On examination he has right side
thoracotomy scar and crackles so it is
bronchiectasis. I presented my finding and then
i said fibrosis i don not know why i said it even
he said is it bronchiectasis or fibrosis and then i
said bronchiectasis but i became confused so i
forget to lung function test in investigation and i
forgot postural drainage in treatment. I knew i
will have 10 and it is i got 10/20
Station2:
The scenario was patient 36 years old male with
type 1 DM came with recurrent hypoglycemia
and weight loss and anemia he has renal
impairment the GFR was 38 and he has
retinopathy.
So i started by asking open questions and the
patient answered by himself all my questions
from the first 5 minutes.
He has recurrent episodes but he has
awareness of hypoglycemia only two times he
needed help from his wife and he is an IT and he
is driving his car but now he stopped so i told
him that is good and i appreciate that you
stopped driving.
He also has abdominal pain and recently
discovered that he has renal problem.
Not smoker.
His concern was what i have.
So i explained that he has an autoimmune
disease which is type 1DM and there are others
and now he may have Addison and i explained
what is it and it could be due to the renal
problem and he may need to the
endocrinologist to decrease his insulin dose but
idid not say to the patient coeliac disease. The
examiner question what is your differential
So i put addison, coeliac and due to renal
problem and then he asked why he has anemia i
said it could be due to the chronic kidney
disease so he asked what GFR can cause
anemia i said 30 and then he said why he has
anemia i said pernicious.
Second questions was what investigations and
then he asked the patient is on ramipril what
you will do i said he will continue on it because t
is protective he said excellent answer i got 16/20
Actually it was nice experience and the
examiners was good
Station 3:
The cardio was an old patient with regular pulse
and on examination he has pansystolic murmur
radiating to the axilla
Examiner asked what is your finding
And he was so happy about my presentation
I said that the patient has regular pulse 70 per
minute and he has pansystolic murmur
radiating to axilla and my impression is that the
patient has mitral regurgitation and the
differential is Tricuspid regurgitation but the
things that against are no hepatomegaly and
raised JVP and the other differential is VSD and
the age of the patient against that.
The examiner agreed
And then what are the causes of mitral
regurgitation
What investigations
What are the treatment
I said replacement and then he asked about the
indication for it
Then he asked me about the medical treatment
and i said vasodilators and diuretics
I got 20/20
The neuro was old patient with mask face
The instruction was to examine this patient
I started doing the parkinson disease
examination
I asked the patient a question how he came to
the hospital
And then i examined him for tremor
And then i did the tone in hands
And the examination for bradykinesia
In upper and lower limbs
Then i asked him to walk
And the finger nose test
Just i forget to test the supra nuclear palsy
But the examiners were so happy and he said
you still have time but i said i don not have
anything to do else and then he said what about
the eyes then i remembered and i told him how i
will do it and he said ok no need to do it
In discussion
What is your diagnosis
What is the differential
I said parkinson plus and the drugs induced
What investigation
I said it is a clinical diagnosis but if in doubt we
can do CT and MRI and SPECT scan
What treatment
I said all the medication with their side effects
He asked what this patient at risk of
I said fall
He said what you will do
I said multidisciplinary with physio and
occupational to adjust the house and also the
social worker for benefits i got 20/20
Station 4
The scenario was 54 old patient with congestive
heart failure and he is at maximum treatment
and the cardiologist said no more added
treatment then he complained of lump in his
neck and biopsy was taken and CT abdomen
revealed that the patient has primary kidney
cancer which is spread all over his body
The task was to break that news and to tell the
patient that he is for palliative treatment and the
role of specialist nurse.
I started by asking him if he wants some one to
attend with him and he said no body
Then i asked him what he knows about his
condition and he said everything about his
cardiac condition and he knows what the
cardiologist said
And he said that he has a lump and a biopsy
was taken and he wants to know the result
So i asked what do you think the cause and he
said am afraid of the worst so i told him am
afraid you are right and then i told him the result
if the biopsy and the CT
Then i kept silent and i let him speak and he
took his time then i said can you tell me what is
going in your mind and then he said he is
shocked so i show empathy and i gave him time
again then i asked him do you want me to
proceed
He said yes and then i told him about the
palliative treatment and i told him that we will
provide support to him and his family and he is
wondering how to tell his family so i offered to
him to bring them in the upcoming meeting and
i will tell them
Then he asked about how long he will live and
empathetically i said i don not know then i
proceed and told him about the specialist nurse
and i asked him again for more questions he
said he can not think now so i told him to write
every question come to his mind and we will
discuss in the upcoming meeting and then he
repeat the question again about how long tine
he has
And again i said i don not know and i explained
that no body can tell
Time finished and the surrogate while he is
going out he told me verygood
I got 14/16
The discussion was about why you did not
specify time
the examiner wants me to tell the patient that he
has months to live so as to prepare his living
will
And then she asked me about the ethical issues
I said breaking bad news
And beneficence vs malfecience
Both examiners were happy
And the. She asked what websites you want him
to search on and what you don not want to
search
I told her that i don not want him to search for
hope while there is no and treatment will harm
him more and his functional level already
impaired
By the cardiac problem
And during my discussion with the patient i
advised him to go to support group and that is
why she asked about the benefits if the support
group so o told her the benefits are for the
patient and his carers
I finished the exam with score 148/172
Sorry i forgot also i told him about the role of
the specialist nurse and how she will help him
and his family
UK Exam Experience ,,,wirral university
teaching hospital
May 2017
St_1
chest ,,,,COPD
Abdomen ,,,Liver + kidney Transplant
with left hypochondrial mass
St_2
History
abnormal creepy sensation since 4 month
DD. RESTLESSLEG SYNDROME
hypocalcemia
neuropathy
St_3
cadio..... can NOT diagnosed
I am NOT sure
Neuro,,,,spastic paraparesis=MS
St_4
COMM
acase of open TB
start on ttt 1 week back
specialist nurse informed that he hadnot pick up his
ttt
yr task :: manage concern & implications of the dis
St_5
1= frequent falls
come out due to frequent joint dislocation
it is Ehler Danlos synd
2=pins & needles
in hands
it was Raynauds phenom. with scleroderma

Thanks everyone here sharing cases . I benefit


much from this group.
Finally I passed in Malaysia , selayang
diet2017/01.
Station 1
Resp:right upper zone collapse consolidation
Ddx: mitotic, tb
Abdomen:
Hepatospleenomegaly , pallor
Ddx: myeloproliferative , lymphoproliferative
Station 2: recurrent chest infection , infertile , sx of
malabsorption underlying DM on insulin
Ddx: cystic fibrosis, chronic lung disease,
hypogammaglobulinamia
Station 3:
Cns : right 5,67,8,12 cranial n palsy, left 3,6 nerve
palsy, big occipital scar hidden underneath hair . No
pydramidal involvement
Ddx : cns lymphoma , NPC
Cvs: 2 big scar . Midline sternotomy scar , no harvest
vein , another big scar from below left nipple extend
till upset border left scapula( I x sure wat the scar for
n correlation with midline scar), metallic click with
first heart sound , TR murmur with raise JVP ,
bilateral pedal ordered ascites . Loud P2 ,AF
Communication skill:
Angry relative . Daughter of a old lady with
underlying advanced heart failure on digoxin .
Currently kinday failure . Digoxin accidentally serve
and pt developed Brady and died .
? Drug error causing death .. explain
Bcc1:
Stem : young lady with joint pain
Hx: typical SLE sx
Sign: RA type involvement joint deformity
Ddx SLE with jaccoub arthopathy , MCTD
Bcc2:
stem:heart murmur detected while applying job
Hx : udenerlying thyrotoxicosis treated , then
Recently postpartum having ? Toxic sx
Clinically euthyroid plus Psm loud at LSE
Ddx: thyrotoxicosis dilated cardiomyopathy , VSD
All the best to everyone !

I want to share them the success ; happiness; and the


the exam cases
St 1
Chest: young male clupped , changing crepts with
cough (cystic fibrosis )
Abd : middle aged female with RIF scar (kidney
transplant dt ADPKD)
ST 2
young female with weight gain 15 month post partum
hit of post partum hemorrhage (hashimoto ; post
partum thyroiditis ;Sheehan synd)
St3
Cardiology
Middle aged male with AR and MR
Neuro
Young male with cereballar signs PC
Hypotonia in patchy distribution (MS)
ST 4 Communication
Young male with CRF
Break the bad news and explain the options of
mangement
St5
BCC1 young male with Neurofibromatosis with
Pheochromocytoma
BCC2 young male with weight loss and exophthalos
(grave's)
I passed with score of 156 out of 172
Thanks a lot

Exam Experiences(of my Dear Brother) -Kolkata


- India April 2017
St2 changle in bowel habbit for 6 months gp
prescribed mebeverine no improvement.
Analysis of complaint large volume diarrhea
500ml for 5 times(small intestine pathology) no
blood no slim no relation to food no
improvement after fasting awake him at night
with urge to move bowel vague abdminal pain
,wt loss not significant.
Pmh of joint surgery,
Travel history last year.
What is your dx?ibd
Your dd ibs , caeliac,tropical sprue,cancer,hiv .
Invs?basic,stool,antibodies screen,faecal
calprotectin,endoscopy ,cancer markers?
Which antibodies to order?iga,ttg,ema
St3
Cardio
Mid sternotomy scar ,harvest graft ,no metallic
,systolic murmur at apex.
Dx MR,PHT
Invs,ttt
Neurology examine cranial nerves
Right Homonymous Hemianopia
Causes?leasion beyond chiasma tract
,radiation,occipital lobe,pisterior cerebral artery
occlusion,internal capsule.
Causes?invs?
St4 SAH on warfarin inr 4 with large onfarcts
specialist see no benefit from surgery talk to wife
BBN?
Qs what percentage can recover?why not ask about
organ donation?
BCC1 female bilateral arm pain
My dd was muscular vs rheumatological vs vasculitis
Postove finding pain with exertion absent pulse no
systemic symptoms other criteria of takayasau
absent
First dd was takayasu
What is your dd?other vasculitis, GCA,PMR,FM
One onvestigation to order?angiography
BCC2 hematemsis
Positive data nsaids tds last week ,pmh of colitis and
on pentasa and moderate alcohol no finding no
organomegaly no persistent vomiting or liver
cirrhosus ridk or hepatitis
DD? PU vs Crohns VS MWS
Others varices ,malignancy
St1
Abdomen
Dark coloured patient with chevron scar no stigmata
of cld no spleen with normal span liver, maxillary
bossing ,no LNs, No ascites,
DD? Thalassemia major, HHC ,aih,psc,infiltration.
If this HHC what operation for this scar?
Hepatecomy,transplant
Chest
Patient with rheumatoid hand
COPD with bilateral basal crackles
Dx?OLD with ILD
Forget the rheumatoid what is other dx ? COPD
invs?
TTT?.
Muscat,Oman hoping my brothers will have a
benefit from my experience .
I started with station 4:
This lady diagnosed with SLE 3 days ago. And
her urine showed protein. Please explain to the
need for renal biopsy and manage her concern
I started with do u want anyone of ur family to
attend our meeting. She Saied no.
how much u know about ur health, surprisingly
she doesnt know anything
so I explained to her
then unfortunately ur kidneys were affected and
in order to know to witch level ur kidneys
affected we need to do one more test wish is
renal biopsy. Then I explained to her renal
biopsy. and I draw for her the kidneys and how
we will do the procedure
her concern is it by general anesthesia . I said
no but with local and sometimes we might just
let u sleep but it is not general.
She asked me will it affect my university. I told
her we are here to help u, we will do our best ,
we need to involve MDT to give u a proper
management and if it so u will live a nearly
normal life but us should under regular follow
up
then she asked me will it affect me if I want to
get pregnancy. I told her it should be planned
whenever It is planned and under close monitor
, it will not affect u
Then I told her there is a consent she has to
sign
she asked me If she doesnt like to do the
biopsy. Will it affect me.
I told her we do not know to witch extend ur
kidneys affected so we can not give u a proper
management.
Then she agreed and accepted to sign the
consent
examiner q :
do u think u convinced her.
I Saied yes
what is the ethical issues :
do u think renal biopsy is important for her.
I told absolutely, why , I answered the same
answer above
at what stage of lupus nephritis u think this lady
is :
I Saied wt least stage 3
What is the modalities of manager. He asked
about the drugs
I Saied im not sure but methotroxate is one of
drugs plus other immunosuppressant like
azathio.
did u answered her concern about pregnancy > I
Saied yes
then he told me did u speak about methotroxate
during pregnancy
I told him no but I have to
ok thank u
Station 5:
1.Young male with presented with diarrhea for
days after using antibiotics
for ur kind care :
when I entered the room 50 years male
I started with tell me about ur health
he said diarrhea now settled down . so whats ur
problem now> he said this skin rash with hand
pain
I asked to see the rash It was not looks like
psoriasis . so I stunted show me ur hand. There
were in rt hand only
ulnar deviation at metacarbophalengial joint
the left hand is normal
then I analyzed the pain and he said morning
stiffness for more than one hour
I asked all CTD anakysis from hair till symptoms
of myathenai qravis
then I asked him to see his elbow no rash
his back no rash
then finally where else u suffered he told me in
my scalp
yes this is psoriasis although no typical rash
but I Saied this is psoriatic arthropathy
I explained to him . and asked him how this
affect his daily activity and job. He answered a
lot.
then I managed his concern and I will refer him
to multidisciplinary team including joint and
skin doctor and they will provide agood plan of
management for him
is it treatable. Unfortunately but controllable
2. this lady co difficulty in swallowing . for ur
kind care.
I thought I will find systemic sclerosis but when
I went to room I found no evidence of SS. But
she has peaked nose strangely
I analyzed the symptoms which was toward
solid
no loss of weight , lymphadenopathy , sweating
nor alarm signs
I asked her to see her neck
the I found smaaaaaal goiter
then I started to analyze the thyroid symptoms
which was negative
social drug and past medical history non
significant.
I examined her thyroid and no retro sterna
extension
I asked her what is ur concern
she Saied what is going on with me
I explained to her every thing
is it treatable
I told her thankfully no alarm signs but we need
to exclude serious condition but from her
history no alarm symptoms
we need to do upper GI endoscopy
examiner q:
how u will investigate
how u will mange:
do u think this thyroid is the cause for her
symptoms
I Saied no thats why we need to do upped GI
endoscopy
Clinical stations :they asked about whats ur DD,
investigation and how u will manage this
patient.
Station 1:
1. Abdomen:
Please examine this gentleman :
When I saw the patient he is a young male
I do not know if he is overpigmented or not but
he looks so
then I took inspection then after finishing
examination the examiner asked me to present
my findings
then I was shocked but I saied I will present
what I found
This is my pleasure to examine this gentleman
who is lying comfortable with an average build
the patient is not pale jaundices or cyanosed .he
has gum hypertrophy with good oral hyagine
there is no evidence of CLD
this gentleman has mild fine termer
this gentleman has mid laboratomy scan with
some drainage scars
he has hepato-splenomegally
liver span is 10 cm and spleen is 10 cm below
the costal margin
No evidence of ascites and there is no evidence
of lymphadenopathy
I would like to complete my examination by
doing DRE and examination for gentalia.
Examiner q :
What is ur diagnosis?
I said in the presence of gum hyperplasia and
mild fine termer , hepatosplenomegally and mid
laboratomy scan I will put liver transplant at the
top of my DD
although there is no evidence of CLD but I
cannot exclude CLD with portal hypertension
How do u this the cause of CLD in the man?
I saied alcohol , infective like hepatitis,
infiltrative and autoimmune like autoimmune
hepatitis, PBC and PSC
hemochromatosis.
How u will invewstigate this pt ?
I will start with baseline investigation in form of
CBC , urea and electrolyte.
LFT
autoimmune antibody and viral screening
Iron study and copper study
Then I will go for abdominal ultrasound and I
maight need liver biopsy
How u will manage this pt?
Non pharmacological and
pharmacological
2. Chest: bronchiactasis with left lower
lobectomy
Station 2:
a 38 years old male presented with recurrent
chest infection since long time. All labs were
normal. for you kind care.
During analysis he has recurrent ear pain(Otitis
media) and sometimes loose motion. sometimes
he has burning in micturition.
He is smoker and alcohol consumer.
No other symptoms
I have to exclude HIV( sexual history) in a
sensitive way.
Social and family history
it affect his job and has recurrent absence from
work.
no significant drug history apart from recent
antibiotic usage
So examiner q:
what is ur dd
common variable immunodeficiency vs cystic
fibrosis vs HIV vs hypogamaglobulinemia
how u will investigate this gentleman:
baseline investigation in form of CBC
chemistry.
xray
immunoglobulin
sputum for culture and sensitivity . for gram
staining and acid fast bacilli
Na sweat test
how u will manage this pt:
pharmacological and non pharmacological
last q : do u think this pt has CF> I said I ahave
to exclude but for me unlikely
so this gentleman diagnosed with CF> what do
u think. I said wrong diagnosis
Station 3:
CNS:
Neuro case :
Please examine this gentleman:
It is my pleasure to examine this gentleman who
is lying comfortable with an average build
by inspection there is no deformity , scars or
wasting
This gentleman has weakness in a pyramidal
distribution in witch flexar is weaker than
extensor, abductor is weaker than adductor and
distal is weaker than proximal.
The weakness is more in left lower limb than RT
There is loss of sensation up to amblicus to pin
prick (strange finding)
posterior column modalities of sensation in
form of joint position and vibration are intact
He has an evidence of cerebellar signs evident
by impairment of heel shin test
I would like to complete by examining the upper
limb, eye looking for nystagmus , gait looking
for wide ataxic gait
then the eaxaminer told me just forget about
sensiation.
What is ur diagnosis:
I told him this gentelman has cerebellar
syndrome
Whats ur DD?
I told him I couldnt get ur Q
He said what are the causes of cerebellar
syndrome?
I said demylination like MS, degenerative and
dierty , infective, vascular like stroke,
inflammatory , neaoplasm, vit b12 dif and
hypothyroidism.
Whisch cause can be reversible after
treatment?(I thought he mean reversible
completely)
I said Iam not sure
How u will investigate this pt :
I will start with baseline investigation in form of
CBC , urea and electrolyte.
MRI of spinal cord and posterior fossa
CSF analysis, VEP
Vit b12 level, THT
How u will manage this pt? Pharmacological
and non-pharmacological
Cardio: Double valve replacement
Wish you all the very best of luck
UK exam exerience, Hull (18 March, 2017).
..I started with station 5, my first case was an
old lady with a skin rash. As soon as I set my
eyes on her, I quickly figured out that she had
extensive scaly maculopapular rash affecting
her head, face and trunk. Hx>>>Long standing
for almost 20 years, Worse on exposure to
sunlight, no itching or pain, now getting worse,
drinks alcohol. I examined her elbows and back
of the years as well as examined for
arthropathy. Viva questions were about
diagnosis, differential diagnosis and
management. I got 15/28 Next ST5 second
case was a lady in her fifties who had elevated
ALT on routine examination and had some
painful periods. Nothing else of note. No
findings on examination. Questions were about
diagnosis, differential and management. I told
them that it's most likely Non-alcoholic fatty
liver disease, other differentials were alcoholic
liver disease but I told the examiner that in that
case, I expect AST to be higher than ALT as well
as high GGT. Other differential I said could be
autoimmune hepatitis. She asked what
investigations....? ASMA....I got 14/28
Next I went for respiratory st. An old man,
actually there were no significant findings on
auscultation. But thanks to Dr. Magdy, I was
thorough in clinical examination, therefore I
missed the diagnosis but still got 9/20. It was
COPD but don't know how, (perhaps in exam
pressure) I said Interstitial lung disease. My viva
was about causes and managment of ILD. Next I
went to Abdomen. Patient had gum
hypertrophy, Tremors, PD catheter mark and as
scar in Rt. iliac fossa with a nontender
underlying mass. My diagnosis was a kidney
transplant. Viva questions were about what
modality of dialysis was he on, I said PD. Then
what meds he could be taking, I said
Cyclosporin because he has tremors and gum
hypertrophy. He asked are these side effects
only because of cyclosporin. I said they are a
common side effects of any calcineurin
inhibitors. Then, he asked me if this pt came
with fever and abdominal pain, what could be
the possibilities? I said Infections., UTI etc etc.
He asked me could he be rejecting? I said yes,
but in that case the graft will be tender. How will
you investigate rejection? again, thanks to Dr.
Magdy, I said blood works, including routine
CBC, renal profile etc, special such as
cyclosporine levels and imaging Renal US and
possibly a kidney Bx...I got 19/20 Next was
History, a man in his 40s, on multiple meds, had
A.Fib on warfarin and a number of other meds
including simvastatin, IHD, c/o difficulty
walking. Initially I thought, it was ?Stroke, but
he had difficulty climbing up stairs as well as
coming down, generalized body pains,
shoulders, legs etc. I thought of Polymyalgia
rheumatica but his age was against it. I thought
of proximal myopathy, Cushings,
hypothyroidism but no features to suggest
them. I could not tell a unifying diagnosis, and
forgot to stop his statin. But I told the examiner
that he needs physio, stop statin and
investigate for the above causes. I got 10/20
Next I had neuro: Again messed up. A
freightening case, a lady sitting in a wheelchair
with short hands and lax skin, having right
sided weakness of all muscle groups and
depressed reflexes but no sensory affection. I
said MND But when he asked me what's
against MND, I said unilateral signs. He said
where is the lesion, by that time I had realized
what mistake I had made. I said cervical
spine...Examiner was looking at me, then he
asked me if I had seen the back of the neck, I
said no. He asked me to do so. AND......
There was a scar at the back of the neck.....
Got 10/20 Next cardio, Here I would advise
candidates who think in UK, an old white man
>>>thnink of aortic valve, young female
>>>mitral ....THIS CONCEPT IS WRONG and
Decieving....It was an old white male, but his
first heart sound was metallic with central
sternotomy scan. No murmurs. Thanks to Dr.
Magdy for giving us an excellent opportunity of
cardiology practice. Exam questions were:
Diagnosis, MVR. Valve functioning well? Yes,
management, anticoagulation. Indications? Got
19/20 Last, I had ST4. Task was to talk to the
son of a patient who was started on Amiodarone
for V. Tach and now admitted with pulmonary
fibosis. He was insisting why it was started if it
could harm him. I showed empathy, told him
that I can completely understand his feelings,
and he is feeling so because of his love and
care for his father. Let me tell you that when the
drug was started, it was given to him in his best
interest as the other alternates are not as
effective as this drug is. He said my GP never
informed us of the S/E. I said I was not part of
the team when it was started, so don't know
exactly what happened. Will look at the charts
and get back to you. Let's look forward now and
see how we can help your father. I asked his
whether his father's bedroom is on ground floor
or he has to climb up the stairs, who lives with
him, how he used to manage his activities of
daily living prior to admission, and offered him
all the social support. I got 16/16 My overall
score was 112.
Didn't pass this time and I think my actual
problem was station 5.

my pleasure to share my experience in


Muscat,Oman hoping my brothers will have a
benefit from my experience .
I started with station 4:
This lady diagnosed with SLE 3 days ago. And
her urine showed protein. Please explain to the
need for renal biopsy and manage her concern
I started with do u want anyone of ur family to
attend our meeting. She Saied no.
how much u know about ur health, surprisingly
she doesnt know anything
so I explained to her
then unfortunately ur kidneys were affected and
in order to know to witch level ur kidneys
affected we need to do one more test wish is
renal biopsy. Then I explained to her renal
biopsy. and I draw for her the kidneys and how
we will do the procedure
her concern is it by general anesthesia . I said
no but with local and sometimes we might just
let u sleep but it is not general.
She asked me will it affect my university. I told
her we are here to help u, we will do our best ,
we need to involve MDT to give u a proper
management and if it so u will live a nearly
normal life but us should under regular follow
up
then she asked me will it affect me if I want to
get pregnancy. I told her it should be planned
whenever It is planned and under close monitor
, it will not affect u
Then I told her there is a consent she has to
sign
she asked me If she doesnt like to do the
biopsy. Will it affect me.
I told her we do not know to witch extend ur
kidneys affected so we can not give u a proper
management.
Then she agreed and accepted to sign the
consent
examiner q :
do u think u convinced her.
I Saied yes
what is the ethical issues :
do u think renal biopsy is important for her.
I told absolutely, why , I answered the same
answer above
at what stage of lupus nephritis u think this lady
is :
I Saied wt least stage 3
What is the modalities of manager. He asked
about the drugs
I Saied im not sure but methotroxate is one of
drugs plus other immunosuppressant like
azathio.
did u answered her concern about pregnancy > I
Saied yes
then he told me did u speak about methotroxate
during pregnancy
I told him no but I have to
ok thank u
Station 5:
1.Young male with presented with diarrhea for
days after using antibiotics
for ur kind care :
when I entered the room 50 years male
I started with tell me about ur health
he said diarrhea now settled down . so whats ur
problem now> he said this skin rash with hand
pain
I asked to see the rash It was not looks like
psoriasis . so I stunted show me ur hand. There
were in rt hand only
ulnar deviation at metacarbophalengial joint
the left hand is normal
then I analyzed the pain and he said morning
stiffness for more than one hour
I asked all CTD anakysis from hair till symptoms
of myathenai qravis
then I asked him to see his elbow no rash
his back no rash
then finally where else u suffered he told me in
my scalp
yes this is psoriasis although no typical rash
but I Saied this is psoriatic arthropathy
I explained to him . and asked him how this
affect his daily activity and job. He answered a
lot.
then I managed his concern and I will refer him
to multidisciplinary team including joint and
skin doctor and they will provide agood plan of
management for him
is it treatable. Unfortunately but controllable
2. this lady co difficulty in swallowing . for ur
kind care.
I thought I will find systemic sclerosis but when
I went to room I found no evidence of SS. But
she has peaked nose strangely
I analyzed the symptoms which was toward
solid
no loss of weight , lymphadenopathy , sweating
nor alarm signs
I asked her to see her neck
the I found smaaaaaal goiter
then I started to analyze the thyroid symptoms
which was negative
social drug and past medical history non
significant.
I examined her thyroid and no retro sterna
extension
I asked her what is ur concern
she Saied what is going on with me
I explained to her every thing
is it treatable
I told her thankfully no alarm signs but we need
to exclude serious condition but from her
history no alarm symptoms
we need to do upper GI endoscopy
examiner q:
how u will investigate
how u will mange:
do u think this thyroid is the cause for her
symptoms
I Saied no thats why we need to do upped GI
endoscopy
Clinical stations :they asked about whats ur DD,
investigation and how u will manage this
patient.
Station 1:
1. Abdomen:
Please examine this gentleman :
When I saw the patient he is a young male
I do not know if he is overpigmented or not but
he looks so
then I took inspection then after finishing
examination the examiner asked me to present
my findings
then I was shocked but I saied I will present
what I found
This is my pleasure to examine this gentleman
who is lying comfortable with an average build
the patient is not pale jaundices or cyanosed .he
has gum hypertrophy with good oral hyagine
there is no evidence of CLD
this gentleman has mild fine termer
this gentleman has mid laboratomy scan with
some drainage scars
he has hepato-splenomegally
liver span is 10 cm and spleen is 10 cm below
the costal margin
No evidence of ascites and there is no evidence
of lymphadenopathy
I would like to complete my examination by
doing DRE and examination for gentalia.
Examiner q :
What is ur diagnosis?
I said in the presence of gum hyperplasia and
mild fine termer , hepatosplenomegally and mid
laboratomy scan I will put liver transplant at the
top of my DD
although there is no evidence of CLD but I
cannot exclude CLD with portal hypertension
How do u this the cause of CLD in the man?
I saied alcohol , infective like hepatitis,
infiltrative and autoimmune like autoimmune
hepatitis, PBC and PSC
hemochromatosis.
How u will invewstigate this pt ?
I will start with baseline investigation in form of
CBC , urea and electrolyte.
LFT
autoimmune antibody and viral screening
Iron study and copper study
Then I will go for abdominal ultrasound and I
maight need liver biopsy
How u will manage this pt?
Non pharmacological and
pharmacological
2. Chest: bronchiactasis with left lower
lobectomy
Station 2:
a 38 years old male presented with recurrent
chest infection since long time. All labs were
normal. for you kind care.
During analysis he has recurrent ear pain(Otitis
media) and sometimes loose motion. sometimes
he has burning in micturition.
He is smoker and alcohol consumer.
No other symptoms
I have to exclude HIV( sexual history) in a
sensitive way.
Social and family history
it affect his job and has recurrent absence from
work.
no significant drug history apart from recent
antibiotic usage
So examiner q:
what is ur dd
common variable immunodeficiency vs cystic
fibrosis vs HIV vs hypogamaglobulinemia
how u will investigate this gentleman:
baseline investigation in form of CBC
chemistry.
xray
immunoglobulin
sputum for culture and sensitivity . for gram
staining and acid fast bacilli
Na sweat test
how u will manage this pt:
pharmacological and non pharmacological
last q : do u think this pt has CF> I said I ahave
to exclude but for me unlikely
so this gentleman diagnosed with CF> what do
u think. I said wrong diagnosis
Station 3:
CNS:
Neuro case :
Please examine this gentleman:
It is my pleasure to examine this gentleman who
is lying comfortable with an average build
by inspection there is no deformity , scars or
wasting
This gentleman has weakness in a pyramidal
distribution in witch flexar is weaker than
extensor, abductor is weaker than adductor and
distal is weaker than proximal.
The weakness is more in left lower limb than RT
There is loss of sensation up to amblicus to pin
prick (strange finding)
posterior column modalities of sensation in
form of joint position and vibration are intact
He has an evidence of cerebellar signs evident
by impairment of heel shin test
I would like to complete by examining the upper
limb, eye looking for nystagmus , gait looking
for wide ataxic gait
then the eaxaminer told me just forget about
sensiation.
What is ur diagnosis:
I told him this gentelman has cerebellar
syndrome
Whats ur DD?
I told him I couldnt get ur Q
He said what are the causes of cerebellar
syndrome?
I said demylination like MS, degenerative and
dierty , infective, vascular like stroke,
inflammatory , neaoplasm, vit b12 dif and
hypothyroidism.
Whisch cause can be reversible after
treatment?(I thought he mean reversible
completely)
I said Iam not sure
How u will investigate this pt :
I will start with baseline investigation in form of
CBC , urea and electrolyte.
MRI of spinal cord and posterior fossa
CSF analysis, VEP
Vit b12 level, THT
How u will manage this pt? Pharmacological
and non-pharmacological
Cardio: Double valve replacement
Wish you all the very best of luck

my experience in after passing the PACES exam


at MALTA center 1/2017, excuse me if little long
but just trying to benefit all my dear colleagues.
-
Starting by telling about the examiners all of
them are British and local examiners from Malta
really all of them are very supporting,
encouraging meaning that it was wonderfull
exam environment
- The sources in terms of books I used Ryder for
history and communication and station 5 But
take care books are only good guide ultimately
you have to have your own approach after
acquiring the experience
In the beginning I like to mention about those
people that are leaders from whom I got benefit.
I attended the first clinical course long time ago
with Prof Dr Abdulfattah Arafa. And Prof Dr
Magdy Mohamed Abbas as a team .
The second course with Prof Dr Ramadan Zaki
All od them are wonderful team in teaching
PACES very sincere and encouraging with
marvellous experience, doing the maximum
efforts for their colleagues.
. THERE are great people you can tell Angeles
that I didn't meet till the moment. from their
videos and publications and online Internet
activities were wonderful source and motivating
materials. Even it was free help reflecting how
much it was for the sake of humanity only ,
cuurently a number of them doing training
coursed Thanksa lot for all of them those great
people are profs Drs Sadek Al-Rokh Ahmed
Maher Eliwa Nashwa Aboamera Shiny Moon
The cases :
- Communication ; to discuss with the wife
whose husband is 45 years old gentleman who
suffered from headache and rapid deterioration
of the general condition within few hours (while
being at work) bought to the hospital and final
diagnosis is meningococcal meningitis ,GCS
ONLY 7 and the plan to shift him to ICU .
After the usual introduction almost she knew
nothing about his condition, I explained
everything about meningococcal meningitis
with the help of a paper to draw something
about the brain and surrounding meninges,
I explained about the expected outcomes
considering the GCS ONLY 7 and the plan to
shift him to ICU and the prognosis is guarded,
Of course contact tracing and related issues.
She was to much concerned about her heath
and her son are there any risk they might got
the infection. I explained about infection control
department in contact tracing and MDT, offered
all forms of support.
Finally I asked if there is anybody to drop her
home
-She was understanding appreciatiating
everything. I got 16/16
-
- BCC1: Elderly PT with Ankylosing spondylitis.
Has low back pain. With history of treated
breast Ca. Chrons dis. Multiple abd scars. DD.
1_Active AS for optimisation of treatment
2_recurrence of breast Ca with metastases 3-
osteoporotic frature, what are the
investigations. I got 28/28
- BCC2; YOUNG LADY known to have bronchial
asthma has worsening cough for the last three
months. Examining her she has expiratory
wheeze otherwise normal concern again about
the cause DD Exacerbation of bronchial asthma.
When I mentioned people as she is using oops
the examiners didnot agree (no tachycardia or
leg swelling) I think I missed asthma mimics as I
went deeply in thinking Unnecessarily I got
19/28
-
-Station 1 Chest
:bilateral basal pulmonary fibrosis. Discussion
about the causes and investigations and
management. I answered all only forget to
mention drug induced among the causes.
Scored 19/20.
Excuse me all , So sorry I will comlete later
because of time constraints,
Many thanks and all the best.
MALTA Center 1/2017
Communication ; to discuss with the wife whose
husband is 45 years old gentleman who
suffered from headache and rapid deterioration
of the general condition within few hours (while
being at work) bought to the hospital and final
diagnosis is meningococcal meningitis ,GCS
ONLY 7 and the plan to shift him to ICU .
After the usual introduction almost she knew
nothing about his condition, I explained
everything about meningococcal meningitis
with the help of a paper to draw something
about the brain and surrounding meninges,
I explained about the expected outcomes
considering the GCS ONLY 7 and the plan to
shift him to ICU and the prognosis is guarded,
Of course contact tracing and related issues.
She was to much concerned about her heath
and her son are there any risk they might got
the infection. I explained about infection control
department in contact tracing and MDT, offered
all forms of support.
Finally I asked if there is anybody to drop her
home
-She was understanding appreciatiating
everything. I got 16/16
-
- BCC1: Elderly PT with Ankylosing spondylitis.
Has low back pain. With history of treated
breast Ca. Chrons dis. Multiple abd scars. DD.
1_Active AS for optimisation of treatment
2_recurrence of breast Ca with metastases 3-
osteoporotic frature, what are the
investigations. I got 28/28
- BCC2; YOUNG LADY known to have bronchial
asthma has worsening cough for the last three
months. Examining her she has expiratory
wheeze otherwise normal concern again about
the cause DD Exacerbation of bronchial asthma.
When I mentioned people as she is using oops
the examiners didnot agree (no tachycardia or
leg swelling) I think I missed asthma mimics as I
went deeply in thinking Unnecessarily I got
19/28
-
-Station 1 Chest
:bilateral basal pulmonary fibrosis. Discussion
about the causes and investigations and
management. I answered all only forget to
mention drug induced among the causes.
Scored 19/20.
NEUROLOGY ;SPASTIC PARAPLEGIA
WITHOUT SENSORY LEVEL, discussion as
usual around DD, investigations (20/20)
Cardiology; an adult pt, with PSM over the apex
mostly MR ,discussion around DD,
investigations,echo FINIDING (15/20)
Abdomen ; left hypochondrial mass for DD
mostly spleen , DD, plan, no features of CLD ,no
lymphadenopathy , no facial plethora, 12/20,
WAITING FOR THE FEEDBACK TO SEE WHY
THIS MARK
HISTORY; adult pt. with migraine developed
sever headache (7/10) at the occipital area with
gait unsteadiness since three days ,
unsteadiness improving partially ,no wakness ,
no sensory abnormality, no visual problem, no
fits, I did not ask about vertigo , NO FEVER , NO
NECK STIFFNESS
DD I mentioned storke , SAH (THAT THE
EXAMINERS DID NOT LIKE) WORSENING OF
MIRAINE
.DISCUSSION ABOUT workup , why not
meningitis , I got 14/20
FINALLY PASS 143/172
Wishing all the best for all of you, the exam
needs reasonable preparation, good practice
Malta centr 2 April 2017
Started with abdomen
Left hypochondrial mass said spleen
Pneumonectomy straightforward.
S2
Young gentleman 24 year while running a marathoon
(after 8 klm) lost consciousness with jerking , brief
concerned is it epilepsy.
PHx similar episodes but no loss of consciousness.
No DM NO epilepsy or other illness no trauma .
Drug Hx
eczema on antihistamine the examiner said it was
significant and i should have taken more details ' I
think they wanted you to think about prolonged QT .
FHx adopted
Social negative
Concern is it epilepsy .
S3 Don't know other candidate said aortic
regurgitation.
Examine lower limb upper motor findings in one leg
gait hemipligic with walking aid
S4
Speak to Mr ...son of mrs ....80 year old lady admitted
3 days with pneumonia and developed delerium task
explain mangement and answer concern.
Discussion about treatment and prognosis of delerium
.
S5
Repeated lady after back surgery came with pain and
fever
Discussion how to cover staph aureus .
...........
Other case
12 years post CABG presented with chest symptoms
and faint concern is it heart attack .
Discussion about beta blockers side effects

Sudan 2017
1.RT
1.Left.lower lobectomy
2.cystic fibrosis.
3.MVR. FLASID PP
4.Renal biopsy in SLE.
5.BCC1Peripheral neuropathy due to anti TB.
BBC2.Familial hypercholestrolemia

Exam experience in bradford royal infirmary on


27 march 2017.
BCC1:
lady comes with tiredness fatigue and sob, with
previous hx of some brain surgery
(hypophysectomy and she was on steroids,
levothyroxine and GH) hx looks like OSAS. It
didn't clicked to my mind that she is acromegaly
and OSAS is due to acromegaly itself. I gave
differential of hypothyroidism and cushing
being cause of her OSAS. Exam findings were
nil for anything :-( she repeatedly gave me clue
why she is sleepless and i kept on beating
about the bush, same as viva.
BCC2: RA with sob and GP mentioned nodules
in xray.
I couldn't find any abnormalities on chest
auscultation:-( and gave d/d if rhenatoid lung
and they asked about any d/d of pulmonary
nodules, i said neoplastic. Next qs about invx
and mx.
4:
Deal with annoyed son who is NIK and his
mother was admitted yesterday with acute
confusional state due to UTI and AKI (brought
by ex husband who claims to be taking care for
her) son seems to be away and less caring but
he was annoyed why his father (who left his
mother when she needed) is around and he
demanded that his father shouldn't be around
and why the son was not called by the hospital.
And he semanded that when she will be
discharged he doesn't want his father to be
around. (Being next of kin he demanded that he
has the right to decide about his mother) i
inquired about power of attorneyband advance
directive which son said he has no idea and
repeated that he is NIK.
Questions were about rights of next of kin.
and if son claims that his father is using mother
to get financial benefits and he is responsible
for her ill health then what should be your
stance. Will u ignore it, where to report it.(I had
no idea)
In case there is no legal report where you will
get guidance (i said hospital legal advisor,
examiner asked who else, i said ex husband. He
asked who else..then he told my by himself that
GP can be contacted)
then he asked if she gets improved then how
will u manage the issue. I said if she is proven
competent upon recovery then she should
decide (autonomy).
3:
Cvs:
lady with palpitations,,,
Midline sternotomy, audible click with 1st heart
sound. Pacemaker scar. I gave MVR and
functioning well. Other candidates told there
was ejection systolic murmur of aortic which i
didn't find:-(
Questions were typical.
Neuro : examine lower limb of lady who has
weakness.
Examination showed power 3/5 both lower
limbs. absent ankle jerks, downgoing plantars.
Stocking sensory loss of all modalities. I gave
d/d of peripheral sensorimotor neuropathy but
examiner was interested in spinal causes and
spinal level.
Asked me to give specific investigations only.
2:
40 Yrs lady with multiple visits with sob and
wheeze, smoker. GP found wheeze at
exanination. And no DVT Further hx
Cat at home. Hx of sob at cold exposure.
CONCERN : is it cardiac.
( No suggestions of cardiac). I gave dd of
asthma copd churg strauss etc. I am still
confused whether there was any trick in case.
1:
Abd:
male with night sweats.
HSM with no lymph nodes, gave dd of
lymphoproliferative and myeloprolifetive
disorders viva about invx and management.
Respiratory : subtle findings. I found wheeze,
examiner was interested in clubbing (though not
apparent, he wanted clear answer whether there
is clubbing or not. Viva about invx and
management.

Common scenario=Station 4
Young male with ulcerative colitis un controlled
with mesalazine
With inc diarrhea and weight loss
Council pt regarding steroid
Started with open questions
Then pt afraid. From steroid as he read about
side effects
Then I filled the gaps
Then every complication how we might mange
Involved the GP
Conserns
1) for how long you will use steroid
Ans
Until we control the disease the dec dose till we
reach remission with minimal dose
2) I want to use herbs
Ans
Sorry but I can't be sure what will be the effect
on on the disease course or how it may
interact with steroid
If u choose to use herbs plz inform ur GP
( DR.zain advice never say no to stupid thing pt
want to do just smile and give all options to him
and then till him ultimately you may hurt your
self)
Discussion
What is ethical issues
What are the complications of ulcerative colitis
1)anemia
Which type
All type
Norm normo (of chronic disease)
Megaloblastis ( fe + b12 malabsorption)
Iron deficiency ( due to blood loss)
2) colorectal Ca
This what he want to hear
3) what kind of diet you will give him
I tried to be smart I will refers to
dietitian
Then agin what diet
I told him high fiber diet
He asked me ru sure
I said with smile of ignorance
Yes sure
I got 20/20
My exam experience in kolkata
st 4 pt non compliant to take steroid, kc
Addison. Husband pt of MS
St 5 (1)RA
LOC(2)
St (1)hepatosplenomegaly wth jaundice
ILD wth systemic sclerosis
St 2)return traveller bloody diarrhoea grandfather ca
colon dx at age 78, 2nd cousin having UC.
St 3 1)mixed mvd with af with hf, 2)spastic paraplegia
traumatic.

Exam experience of PACES=MALTA= 01.04.2017


I started with station 2 : 24yrs old lady c/o
fatigue and headaches found to have high BP
160/98 on different occasions. No family hx , no
complications.....examiners asked would you
start treatment...i said not now if secondary
causes r/o including white coat htn then yes.
Discussion about secondary causes of htn and
investigation.
Got 20/20
Station 3: cardio VSD/TR pansystolic murmur in
tricuspid area. Discussion about investigations and
management.
Got 16/20
Neuro; middle age with walker reflexes with re
inforcement rigid limbs throughout....planters
equivocal no cerebellar signs ...out of time could not
check sensory system....made dx of spastic
paraparesis....asked of causes got 11/20
Station 4: talk to the wife of 50years man known
MND recurrent admissions for pneumonia. This time
admitted with aspiration pneumonia not responding
to anti biotics and confused. Discuss with her his
management and condition. I explained to her that he
is in advance stage....and any invasive procedure like
intubation is futile. We will give him palliative
treatment only and keep him comfortable. Examiner
told that i did not told about NIV when the wife
asked. Got 15/16
Station 5: first case young lady with joint pain...i was
happy will be from common causes. When i entered
took hx all joints pain including hand joints.
Surrogate told she was told by GP that she has double
joints. I was more confused...asked what u mean
double joint she said i dont know gp told me that and
he was not sure.i examined no sign of inflammation.
Raised skin dorsum of hand elastic . Checked
functions. Found high arched palate. Joints
hyperextensible.arm lenght normal. Gave diff of
marfans pseudoxanthoma elasticum forgot to tell
ehlers danlos syndrome. Examiner asked were there
marfanoid features i said no. I wanted to examine
chest they stopped me asked what u want to see i said
AR....they said normal. Asked how you investigate
joints in general...started with inflam marker RA
factor ACCP...time finished got 28/28
2nd cast st.5: believe me i totally forgot....got 23/28
Station 1: abdomen; lady in 50 has pruritis and abd
pain.....i neglected pruritis....no signs of CLD or
immuno suppression...distended abdomen tender all
over mid line laprotomy scar below umbilicus.
Shifting dullness+ could not appreciate any
organomegaly or mass due to tenderness. Differential
...causes of ascites asked about causes of transudative
ascitic fluid....got 8/20
Respiratory: c/o of Sob pneumonoctomy scar left side
trachea deviated to left no breath sounds left
......asked about investigation ....told so many still
asking was happy when told sputum analysis and c/s
and AFB ....infection as cause of Sob
Got 20/20
Wish success for all you going for PACES......my
advice is to perform during exam in relaxed way,
smile and forget about any station where you did not
perform well.

Oman 9/4/2017
Day 4 Cycle 2
I started with station 2
My case a lady of 35 years old present
complaining of fatigue and weight gain 3 Kh in
the last 6 months , she gave history of typical
hypothyroidism
I ask about other hypos no other symptoms
apart from amenorrhea ( hypogonadism ) .
She had family history of thyroid problem ( her
mother )and her any is using regular vitamin
injection but she is not sure what is it .
Her last delivery about
18 months ago and was eventful, she had been
transfused much amount of blood , but against
Sheehan's syndrome she is lactating for more
than one year.
No PMH and not on any medications.
Examiner question what is your DD :
Postpartum thyroditis ( he ask why you think
about it ) I told him because her symptoms
started after delivery )
Autoimmune poly glandular syndrome (family
history of endocrine disease)
Sheehan's syndrome .( against it no symptoms
suggesting hypoadrenalism and she is lactating
for one year .
Other questions :
Investigations and treatment.
At the end he asked about Addison disease
How do will treat patient with Addison disease ?
I stared if he came in emergency I will stared
with IV fluid normal saline
Before I answer hydrocortisone bell rang
Station 3
*Cardiology*
Examine this patient and present your findings .
First and second heart sound are metallic clic
MVR + AVR
Questions:
Investigations
Echo , What is single blood test you want to do
(*He want INR*)
Other question can you prescribe for him new
oral anticoagulant ? I told him up to date it's not
license in patient with metallic valve
(*Neurology*)
Strange case :
Young patient with lower limbs weakness.
On examination LMN weakness (proximal
myopathy)+ loss of vibration sense with intact
joint position in the left side ).
Most probably the patient had Becker ( I could
not match the things together )
I told him proximal myopathy for DD but against
that the loss of vibration sense in the left side
He ask me forget about vibration sense , what
could be the cause of his weakness ? How are
you going to investigate him
Bell rang
*Station 4*
29 years old Omani male he is complianing of
fatigue and dizziness for the last 6 months ,
today he had been call by his GP because he
has abnormal renal function , your role to
explain to him his condition.
BP : 160/105
K : 5.2
High creatinine
High Urea .
High creatinine .
During discussion he told me I have cut wound
in my hand 5 years before could it be related ?
My BP was high since that time and the Dr.at
that time did not bother him self .
What is final treatment ?
I explain to him what chronic renal failure , what
is complications and what are option of
treatment including renal replacement therapy (
dialysis).
*Examiners Questions*:
You mention to this gentleman that you want to
admit him , why ? During the discussion the
patient mentioned that he has shortness of
breathing , consequently I told him you might
need dialysis after I full examined you and doing
CXR ( fluid overload ).
As he is lower what mode of dialysis you will
over him ? I told him haemodialysis is better (
less chance of complications).
Then he asked me what is problem of high
serum K ? Do you think 5.2 need dialysis?
*Station 5*
*BCC 1*
35 years old male complaining of attacks of
headache, sweating and palpitations .
When I went inside I'm asking about course,
onset and duration of symptoms.
I asked also about aggravating and decreasing
factors (none)
Loss of weight , fever night sweating , analysis
of headache , analysis of palpitations
Mean while am asking I told the surrogate I will
examine the patient meanwhile I will ask some
questions
It was clear neurofibromatosis
After that I ask about PMH , family history ( he
had family history of similar condition).
Medications history .
I ask about the concerns. She ask me what the
is cause for his problem ?
How he can be treated ?
How skin lesions be treated ?
I ask I want to examine BP , they told me no
need
I examine the back of the patient looking for
cafe auilt spots
Auscultation of heart sound .
Bell rang.
Examiner Questions:
What is your diagnosis?
DD
Management.
Time finished .
*BCC2*
38 years old male with history of loss of weight
and fatigue .
Hyperthyroidism
Graves' disease.

Dear all,
Thank you so much for sharing your knowledge.
I finally passed paces!
First attempt at Bangalore India 2017/1
Cvs
Very loud MR with displaced apex and thrill.
Otherwise not in failure. But also noted ar and tr.
Collapsing pulse
Asked about how to ascertain cause of mr- echo and
angiography
Respi
Left pleural effusion
Asked about lights criteria
History
Young woman overtly hypothyroid post partum. No
other features of hypopituitarism. History of pph with
massive transfusion. Still able to breastfeed. Ddx
Sheehan , postpartum thyroiditis
Neuro
Old CVA with left hemiparesis, left umn facial nerve
palsy
Asked about mx for acute and chronic stroke ie rehab
Abdo
Cld, cachexia with ascites
Asked about peritoneal tap
Communication
Elderly lady with obstructive jaundice suspecting ca.
All investigations negative. To convey the uncertainty.
BBN. Advise next step. Concern is what if daughter
wants to keep the news from mother.
Bcc1 ank spon patient on biologics got fever and
cough. Clinically no signs of ank spon or pneumonia.
Bcc 2 chronic headache ? Due to sinusitis. No red flag
sign

I had my exam in Brunei on the last day in


second schedule. Exam was tough with some
atypical cases, but ALHAMDULILLAH (All praise
to Allah), I passed it. It was my first attempt. My
sincere thanks to PACES EXAM CASES and all
it's contributors, esp. Bebo bebo and
Mahiuddin. I had been a silent observer here. Dr
Mahiuddin gave a lot of useful tips here which
really helped me. I also thank to my all teachers
esp Dr Abdulfattah, who taught me the basics of
this exam in a very simple way. I would like to
share my cases here.
1. Respiration: Young short lady, with SOB.
Patient could not lie down, so all examination in
sitting position. No clubbing, central trachea,
B/L basal crackles not fine but doesn't change
with cough as well. My diagnosis Pulmonary
fibrosis, Other DD Brochiectasis. Examiner
asked about diagnosis and different causes.
British lady examiner was very cooperative and
she sensed my nervousness as it was my first
ever PACES station, that also respiratory (time
taking) and plus young lady.
I got full marks.
2. Abdomen: Obese man, round face, and
abdominal striae; with active fistula at left wrist.
Few scars in the neck, left subcostal scar with
few scars beside it. No hepatosplenomegaly. I
felt some fluid hitting my hand when patient
turned his body. It was a very difficult palpation.
I got shifting dullness as well (??). My
diagnosis- Patient with end stage renal disease
on haemodialysis, most probably on steroids,
cause could be due to Glomerulonephritis.
Examiner asked me why he had ascites. I said
due to volume overload (uraemic). Then why not
pedal edema? I told may be partially treated. He
asked for any other reason for this ascites in
renal patient. I told he might have peritoneal
dialysis, which could be reason for fluid. He
asked me for any proof? I showed him the scars
on abdomen. He said it could be due to surgical
drainage. I said it could be. Then he repeated
the question, any other reason for ascites in
renal patient. I was very nervous and couldn't
answer further and the bell rang.
3.History: Middle aged man with SOB and leg
swelling and past history of recurrent chest
infection. I finished before time. Examiner asked
me about diagnosis. My diagnosis
Bronchiectasis with cor pulmonale (right heart
failure). He asked me of any other possibility. I
could not get it. He asked me about
complications of bronchiectasis, I said local and
systemic. He asked further about systemic.
When I told amyloidosis, he asked, "could it
affect kidney" . I told yes, it can cause Nephrotic
syndrome and that is one of the possibility in
this case. He was very happy to hear this from
me and he gave me thumbs up.
4. Nervous system: Middle aged lady lying down
with her right hand near body and wrist looks
dropped. I asked her to put her hands in front
and turn the hands up. Initially the right wrist
was dropped but slowly she raised it. That
added to my confusion. I immediately started
typical upper limb examination. Power 4/5 in the
right upper limb. Tone - normal, reflexes -
absent bilaterally with negative Hoffmann.
Sensations - I checked pain and vibration only,
due to shortage of time. And both were reduced
on the right side. There was no obvious facial
deviation. I was fully confused. I went for
common thing first and said it could be stroke
in spinal shock. British examiner asked me the
proof to support my diagnosis. I told it is
difficult to say without examining the lower
limbs and cranial nerves. But the typical
pyramidal pattern of weakness with unilateral
sensation loss of all modalities could be the
clue. She asked what did it mean by pyramidal
weakness, I said "even though it is more typical
in lower limb here I can see that abductors of
shoulder and extensors of elbow and wrist are
weaker, giving the typical posture."
I got full marks ( I can't believe, I am still not
sure about diagnosis).
5. CVS : Middle aged man, with midline
sternotomy scar. Dual valve replacement with
MR, AR and AS, with chest congestion but no
pedal edema. I forgot to check thrills. British
examiner did not agree with my apex finding,
which I immediately accepted. He asked me
about diagnosis and complication. It was a
typical station.
6. Communication skills: Young man from
military was referred by GP for further check up
as his brother died of HOCM last year. His ECG
done by GP was normal. He had appointment
for Echo after 2 weeks but still couldn't get
appointment for genetic studies. He was not
eager for further tests and had concern that his
life would be disturbed and he might lose job if
it came out to be positive. He started
aggressively, Alhamdulillah, I tamed him and
convinced him. My MRCGP skill helped me.
Examiner asked some typical questions and
also what would I do if he didn't turn up for
further investigation. I told I would take the help
of GP or employer to trace him back. Chief
examination coordinator was present during
this consultation.
I got full marks.
7. BCC1: The coordinator confused me with
other case. I lost some time in confirmation.
Young lady with decreased vision of sudden
onset in both eyes for 2 days. Diabetic for 6
months, not following up, not controlled. Father
had glaucoma. Past history of gestational DM.
She could only read the top line of chart. Field
normal. Before I started fundoscopy, examiner
informed that two minutes were left. I looked in
the right eye, there were black pigments
suggesting retinitis pigmentosa. I had no time to
look at optic disc or macula. I told I would like to
refer her urgently to Ophthalmologist and also
check her blood sugar. Examiner asked me
about diagnosis. I said it could be due to
osmotic changes in the eye due her
uncontrolled sugar. She asked me about
anterior chamber. I said I could not examine due
to shortage of time. As there is no pain the
chances of glaucoma is less. As it is acute and
bilateral, Retinitis pigmentosa can't explain this.
She asked me about complications of DM, I
answered everything except Retinopathy
(funny? I felt very depressed that how I forgot
this... Exam tension). I am still not sure about
diagnosis.
8. BCC2: Young lady with hand deformity. She
had pain in hand joints and backache. Fingers
were deformed just like rheumatoid arthritis.
Nails were normal. On asking I got to know she
had rashes over elbows which were well hidden
with clothes. Alhamdulillah I got it. I examined
her properly. I managed the time very well here.
Examiner asked me about diagnosis I said
Psoriatic arthritis. Then he asked about type of
deformities, signs of activity of disease, chest
findings and management.
I got full marks.
Alhamdulillah, I passed the examination
comfortably. All praise to Allah.
Mohamed Khalid Albasha


Ahmed Maher Eliwa...




Mahade Hassan
By the Grace of Almighty Allah..Passed PACES..in
1st attempt
My gratitude to my sir.. Ahmed Maher Eliwa...
kuwait 1/5/2017
1st day 3rd cycle kuwait centre
Mobarak alkabeer hospital
started with station 5 bcc1 young lady c/o
difficult swallowing and wt loss goitre (signs of
hyperthyroidism)
bcc2 old lady c/o SOB with exertion it was
systemic sclerosis

station 1 abd. rt iliac scar with mass(renal


transplant)
resp. confused not sure about the case

station 2 40yrs lady c/o SOB and cough for


6months
positive thing in history contact with bird the
examiner want extrinsic allergic alviolitis,

station 3 cardio mitral valve replacement and


early diastolic murmur seem to be aortic
regurgitation
CNS flaccid paraparesis with normal sensation
I put differential of MND and I think examiner
want gullian barre
station 4 discuss about side effect of
amiodarone pt developed SOB CT confirmed
lung fibrosis

discuss with the daughter the finding and she


concerned about why the cardiologist not
mention this SE and also concerned about
other option if amiodarone stopped.

Actually the communication scenario was long I


didn't concentrate what is exact cardiac
problem for which the cardiologist start
amiodarone. ISaid to surrogate that her father
need admition with follow up with chest and
heart physician.
The examiner ask me again what is substitute
for amiodarone I answered with the same that
we need cardiologist and chest physician then
he asked me and what is your role
some candidate said outside ICD

the indication mentioned in the scenario and I


miss it.
My advice that every one should read the
scenario well
paces exam experience at indian centre for
2017/01 diet.....
my second attempt. first attempt at 2016/03 diet
at golden jubilee glasgow in nov 2016. failed.
though scored 136....failed in physical signs
skill by 2 marks. desperately wants to finish
mrcp so applied for international centre in oct
2016 for 2017/01 diet. got indian centre for exam
in first week of april 2017. ..
started with station 3
Cardio: 60 year lady, left al thoracotomy scar
indicative of previous closed mitral valvotomy,
severe mitral restenosis, sinus rhythm, in
congestive cardiac failure, no evidence of ie or
thromboembolic manifestations...routine
standard questions...missed oral
anticoagulation in management .....got 18/20
Neuro:60 year lady...right lower motor neuron
facial palsy...passive...routine standard
questions.....got 19/20
station 4: 25 lady..working in .it sector....came
yesterday to er with cough
...fever..hemoptysis....left against medical
advice...now diagnosed with sputum positive
pulmonary tb showing multiple infiltrates in
bilateral lungs on chest x ray..called back to opd
today...discusss...explore and manage
concerns....went well ...got 14/16.
BCC1:50 year gentle man...bilateral joint
pains......diagnosed to have...psoriasis...skin
changes...hand changes...nail
changes....checked spine movement....on
methotrexate since 2 years... so examined
hands...checked for anemia...oral
ulcers..auscultated bilateral lung
bases...palpated abdomen for
liver..then...standard routine questions.....got
27/28
BCC2;27 year man...acute onset of chest pain
and sob since morning....explored....known
lupus on steroids since 1 year...had knee joint
injury 6 weeks back...cast removed
recently...examined...eyes..oral
cavity..hands..pulse...complete respiratory
system...neck vein...auscultated heart
sounds..checked knees...legs...given
differential.. as...pulmonary
thromboembolism.....sle pleuritis....atypical
pneumonia....and management
accordingly.....got 27/28..
Resp: 70 year man....median sternotomy
scar..left posterolatral and anterolateral
thoracotomy scars, bilateral leg vein harvesting
scars....decreased movements on left
hemithorax...trachea deviated to left...dull ness
left bases...bronchial breath in apical area.....so
my diagnosis...post cabg...post left lobectomy
or left pneumonectomy....then standard
questioning and answers...got 18/20..
Abdomen: 50 year man....functioning right
radiocephalic fistula....grossly anemic...bilateral
pitting pedal edema...distended neck
veins....bilateral ballotable kidneys,....palpable
hepatomegaly....ascites....my
diagnosis....APKD...with anemia...with fluid
overload...with multiple cystic liver...with
functioning fistula....standard questionnaire and
answering.....got 20/20....
History: 30 year lady....fever...abdominal
pain...bloody diarrhoea...since 1 week ..after
returning from a trip from kenya..gone well...got
19/20.
finally got ..162/172...goodscore and pass ..
my experience:
1. focus...focus...focus...last one week...more on
physical examination...should be thoroughly
rehearsed....rest ...we have time in
examination...we can get back to them even if
miss....but physical examination....should be
flawless...and is in our hands..
2. never ever create physical signs that are not
there. we may miss physical signs that are
there...but never ever create physical signs that
are not there..if we do...sure shot of failure.
3. come exactly at the reporting time mentioned
on admission usually one hour before start of
actual exam..relieves lot of stress..
4. i have written just my initials rather than my
full name on answer sheets.
5. no verbal roughness with examiners or with
patients.
Edinburgh - centre =April 2017
Glan Clwyd Hospital in Rhyl Wales

1.a) Abdo- multiple abdominal scars which


looked like renal transplant scars but no kidney
underneath palpable. AV fistula which was
buzzing and looked like had been recently
needled. Could feel a retro peritoneal mass- said
it might be a polycystic kidney. Also had
parathyroidectomy scar. Was asked for Mx. Said
U&E , Doppler US of abdomen. Then bell rang. I
thought this station was a disaster and would
fail. 16/20
1 b) Resp- 65 yr old man reading newspaper.
Only finding was some crackles R>L and
perhaps a dull left lung base. My DD's were ILD,
Bronchiectasis and pleural effusion. Said would
start with CXR then HRCT. Examiner directed
discussion towards dull lung base. Reasons,
lights criteria and cut of of pH for empyema.
Thought I had failed this as well as was not sure
about the dull lung base and this obviously was
the diagnosis. 18/20
3 a) Neuro. Elderly lady in wheelchair. Examine
LL. Asked if I could see her walk. Examiner said
not needed. B/L increased tone L>R, B/L
Proximal weakness in pyramidal distribution.
Knee jerk increased. Ankles N. plantars
equivocal, coordination N. Sensation N. asked
to check coordination in upper limb as pt was
having problems lifting legs. Examiner said its
LL examination. Anyway when i was presenting
my findings he started asking me questions
about reasons for cerebelar dysfunction. Did
not give me chance to give my diagnosis which
most likely was spastic paraparesis. Was not
sure how i had done. Got 13/20
3.b) metallic MVR with AF. Pt tachycardic. Was
expecting atleast 18/20. Questions- if this pt
came With SOB- what would your dd's be. Got
14/20
2. Hx. 30 yr old female with palpitations.
Recently had a baby. Fx hx of some auto
immune condition. Dx- post partum thyroiditis.
Was expecting 20/20. Got 16/20
4) communication skills: elderly lady with end
stage heart failure and new renal failure who
has just died. While going through notes you
notice that she was earlier on digoxin which
was stopped due to renal failure and new
bradycardia. However this was inadvertently
started by the on call team. Tell this to pts
daughter. I told her what had happened.
Apologised for mistake. Told her would do
incident report and RCA. Try to ensure this
doesn't happen again. She asked if the digoxin
could have killed her mother. I told her that this
might have been the immediate cause although
her heart was already in a bad way. Told her
would inform coroner about this. Key point -
apologise, be honest about drug error possibly
being the cause of death. Got 14/16
5 a) 60 yr man with weakness and pain in L hand
. Worried if it could be TIA. I did not have a clue
till I started examining. By fluke, I felt pulse in
both hands and noted absent radial pulse in L
hand. Went up and absent brachial as well. Told
DD as Takayasu's and examiner frowned and
asked what else. Said subclavian steal
syndrome and he gave me the thumbs up. 28/28
5 b) Gp referral for pt with dizziness. BP 130/80.
Hx of diet controlled diabetes. On asking pt
gives hx of postural hypotension. And if you ask
about standing BP , examiner tells you its 90/60.
If you check medications, then on 4 anti
hypertensives. Acei, bisoprolol, alpha blocker
and calcium channel blocker as well. So told pt
will stop both alpha and calcium channel
blocker as pt also had ankle swelling. 28/28

Thank you so much for sharing your knowledge.


I finally passed paces!
First attempt at Bangalore India 2017/1
Cvs
Very loud MR with displaced apex and thrill.
Otherwise not in failure. But also noted ar and tr.
Collapsing pulse
Asked about how to ascertain cause of mr- echo and
angiography
Respi
Left pleural effusion
Asked about lights criteria
History
Young woman overtly hypothyroid post partum. No
other features of hypopituitarism. History of pph with
massive transfusion. Still able to breastfeed. Ddx
Sheehan , postpartum thyroiditis
Neuro
Old CVA with left hemiparesis, left umn facial nerve
palsy
Asked about mx for acute and chronic stroke ie rehab
Abdo
Cld, cachexia with ascites
Asked about peritoneal tap
Communication
Elderly lady with obstructive jaundice suspecting ca.
All investigations negative. To convey the uncertainty.
BBN. Advise next step. Concern is what if daughter
wants to keep the news from mother.
Bcc1 ank spon patient on biologics got fever and
cough. Clinically no signs of ank spon or pneumonia.
Bcc 2 chronic headache ? Due to sinusitis. No red flag
sign
Hi all,I would like to share my station
Day 5 round 2 Yangon
Station 1
Resp - Rt.sided pleural effusion with Lt crepts
and rhonchi
Abd- Hepatosplenomegaly (Thalassaemia)
Station 2
20yrs old lady with hypertension
GP did urine dipstick show proteinuria and
haematuria. Pt also c/o fatigue for 6mths and fever
for 3weeks ago.
Station 3
CNS- MND
CVS- MVR with vulvotomy scar
Station 4
Talk with granddaughter about 89yrs old rt. sided
weakness. PMH - TIA and AF with wafarin ,fail to
follow up at wafarin clinic ,
INR 1.2,CT - infarct stroke and no bleed
Task - explain about CT result and refer to stroke
unit
Station 5
BCC1 - H&M due to NSAID and prednisolone
overdose with low back pain and ankylosing
spondylitis
BCC2 - Peripheral neuropathy with DM and also
taking anti -TB and diet -vegetatrian
My one and only PACES experience
Alhamdulillah .
Appeared from Glasgow College.
STATION 2: (Both examiners British)
40-year-old female, presented with complaint of
fatigue, Hb 8 and MCV 70. She also has
complaint of mild ankle edema. Her father had
colon cancer. Abdominal examination was
normal.
Both the examiners were smiling and seemed
nice when I went inside which boosted my
confidence a bit since it was my first station. .
When I started with history, the patient told me
about this fatigue which she had from the last 3
months. Along with that she said she had
complaint of palpitations and shortness of
breath. She was a fitness instructor. She said
she had no other complaints. I went on with my
questioning. When I asked her about her bowel
works, she said more or less my bowel habits
are okay. I then specifically asked if she had
any diarrhea or constipation? She said she had
complaint of diarrhea and bloating from the last
9 years!!!!!!! Next I asked her if she felt it was
associated with any specific kind of food, and
she said yes to that. She had injured her back
and was taking Diclofenac off and on for it.
Her concerns:
1. Do I have colon cancer like my Dad?
2. Can this be due to the peppermint
supplement that I am taking? (I said I am not
sure what you are taking but I can look it up and
get back to you with the information)
3. Why the mild ankle swelling? (I replied that it
might be due to the Diclofenac that you are
taking as it can cause fluid retention but we will
get the necessary tests done today and
hopefully find out)
My Diagnosis: Coeliac Disease
Examiner Questions:
DD? Diagnosis of Coeliac disease? Why not
colon cancer? (I said that I had inquired about
all the alarm symptoms which were negative).
They seemed happy, one of the examiners even
said Well done
Score: 15/20
STATION 3: (Both examiners British)
CVS - Please examine the cardiovascular
system of this patient
Elderly female with mid-sternotomy scar. No
harvesting scar and no clicks audible. I could
appreciate and ejection systolic murmur which
radiated to the carotids.
My Diagnosis: AVR with bi-prosthetic valve/
AVR with CABG with internal mammary graft
(Examiner said fair enough)
Examiner Questions:
Please tell me ONLY your positive findings?
Why ejection systolic murmur still there after
AVR? (I said that the murmur does sometimes
pre-exist despite of valve replacement)
Criteria for diagnosis of AS?
Criteria for Aortic valve replacement?
If patient has bi-prosthetic valve, how will you
manage further?
Score: 18/20
CNS - This patient had difficulty in walking.
Please examine
As soon as I introduced my self to the patient
who was an elderly male, I asked him to walk. I
expected him to have difficulty doing that, but
her was up and running in a second. No
problem at all. I actually asked him to walk twice
. On inspection there was a slight twitch in just
his right foot, which I initially could not
interpret. It seemed more like voluntary
movement!
No other abnormality in lower limbs. Anyway, I
was always advised by seniors to proceed
methodically and to stick to my scheme, which I
did. Examined the power, tone, reflexes,
planters and even sensation in the lower limb
(NOTHING). I was lost. Just then the examiner
said, one-minute left. I resigned to the fact
that I had totally lost this case Then out of
nowhere a thought occurred to me that the case
was Parkinsonism. I immediately asked him to
sit facing me and asked him to close his eyes
and there it was blepharoclonus. Then I
immediately checked the tone of his limbs first
without and then with synkinesis. Then time
was up
My Diagnosis:
Idiopathic Parkinsonism
Examiner Questions:
Diagnosis?
Differential diagnosis of tremors? Types of
tremors? Why not cerebellar? (I said gait was
fine and no ataxia)
Drugs causing parkinsonism?
How would you help this patient besides
medical management? (I said physiotherapist,
occupational therapist)
If he has a problem opening jars, who would
you said him to? Occupational or
Physiotherapist? Choose one? (I said
occupational. Examiner seemed happy)
Score: 19/20
STATION 4: (Both examiners were Asian)
You have to talk to Mr. Glass who is the son of
Mrs. Glass. She was admitted in the ward 3 days
back with community acquired pneumonia and
was being treated with IV antibiotics. No other
medication is being given to her. Today, she has
gone into a state of delirium, and is abusing the
nurses in the ward. She did not get her self
checked by you when you went to see her
today. She is refusing to take any medications
and is not eating anything thinking that the staff
is going to poison her. Your task is to talk to her
son and explain the delirium and further
management plan.
After reading that scenario, I though definitely
this is going down. Anyway, I went in and
started talking to the patients son. Explained
that his mother was in delirium, cause of which
might be the recent illness which she is going
through. Elderly people do sometimes behave
this way.
Concerns:
1. What is the cause? (I said it can be just due to
the illness, or something else like salt
imbalance in the body)
2. Can she go home? (I said it would be best if
she stayed at the hospital till she is all well as
her health is our first priority)
3. Is this Alzheimers Dementia?
Well. Guess what? I was done with the
scenario, the son seemed very satisfied. I asked
him twice if he had any more concerns or any
thoughts? He said, no doctor, I am happy that
she is being looked after well by you. and the
examiner told me I had 4 more minutes!!!!!!! We
sat in silence!
I finished this scenario 4 minutes
early!!!!!!!!!!!!!!!
Examiner questions:
Explain briefly what the scenario was and how it
went?
What is delirium? Please explain to me in
layman terms as if I not am not a doctor?
What one test you would like to do? (I said
Serum Electrolytes. He was happy)
Will you put the patient to sleep? I said
definitely not, I will just give her something to
calm her down.
The patient is mentally incapable now, what will
be your plan regarding this patient? Will the son
decide? (I said, no, it is the doctors
responsibility to decide but it is very very
important to get the relatives on board)
How will you judge mental incapacity?
What ethical laws apply to this scenario? (I said
beneficence, and non maleficence. He smiled)
Score: 16/16
STATION 5 (The one I was most scared of!!!)
Both examiners were British
BCC 1:
This 66 year old female has presented with
complaint of swelling of fingers.
Went inside and it was a spot diagnosis of
Acromegaly.
Examiner questions:
Diagnosis? Investigations? Management?
Score:28/28
BCC 2:
This 50-year-old male presented with painful
and cold fingers.
My Diagnosis: Limited systemic sclerosis with
ILD.
On history there were all features positive of
CREST syndrome, in addition he had bilateral
velcro crackles. Again, it was a very obvious
diagnosis. Sclerodactyly, Teleangectasis,
Pinched nose, puckered mouth.
Examiner questions:
Diagnosis? Investigations? Management?
Score: 28/28
STATION 1 (One examiner was Asian and one
was British)
RESP: This gentleman presented with
complaint of exertional breathlessness. Please
examine
There was a nebulizer next to the patients bed
which I didnt see till the end that too when the
examiner pointed out. He was breathless at
rest.
The patient appeared pink with bilateral ronchi
and left sided basal crepts which changed on
coughing.
My Diagnosis: COPD with Bronchiactasis
Examiner Questions:
Diagnosis? Investigations? Management of
COPD? Causes of Bronchiactasis?
Score: 19/20
GIT: This gentleman has had multiple
admissions in the hospital in the past. Please
examine his abdomen
The patient was all covered up, I thought that as
soon as he would remove his sheet there would
be multiple scars on it. But it was all clean. No
marks, no ascites. I saw nicotine staining on his
hands and just a single spider angioma on his
chest. Then on palpation of his abdomen I could
not feel anything massive. I told the examiner
hepatomegaly, because I felt it was very slightly
enlarged. I finished one minute early!!!!!
I forgot to auscultate the abdomen
My Diagnosis: CLD
Examiner questions:
Diagnosis? Differential? He had multiple
admissions for ascites. If he comes back with
massive ascites what would be your
management plan.
Score: 19/20
Total score:162/172
Alhamdulillah, I cannot thank Allah enough. I
have tried to write down my experience as
detailed as possible so that it can help
everyone. I did not attend at courses in UK. I
attended the two courses available in Lahore
where I live. I have been following this group
regularly and has been of immense help to
me. :). Thank you and Good Luck!

My exam experience in bradford royal infirmary on 27


march 2017
:BCC1
lady comes with tiredness fatigue and sob, with
previous hx of some brain surgery (hypophysectomy
and she was on steroids, levothyroxine and GH) hx
looks like OSAS. It didn't clicked to my mind that she is
acromegaly and OSAS is due to acromegaly itself. I
gave differential of hypothyroidism and cushing being
cause of her OSAS. Exam findings were nil for anything
:-( she repeatedly gave me clue why she is sleepless
.and i kept on beating about the bush, same as viva
.BCC2: RA with sob and GP mentioned nodules in xray
I couldn't find any abnormalities on chest
auscultation:-( and gave d/d if rhenatoid lung and they
asked about any d/d of pulmonary nodules, i said
.neoplastic. Next qs about invx and mx
:4
Deal with annoyed son who is NIK and his mother was
admitted yesterday with acute confusional state due to
UTI and AKI (brought by ex husband who claims to be
taking care for her) son seems to be away and less
caring but he was annoyed why his father (who left his
mother when she needed) is around and he demanded
that his father shouldn't be around and why the son
was not called by the hospital. And he semanded that
when she will be discharged he doesn't want his father
to be around. (Being next of kin he demanded that he
has the right to decide about his mother) i inquired
about power of attorneyband advance directive which
.son said he has no idea and repeated that he is NIK
.Questions were about rights of next of kin
and if son claims that his father is using mother to get
financial benefits and he is responsible for her ill health
then what should be your stance. Will u ignore it,
)where to report it.(I had no idea
In case there is no legal report where you will get
guidance (i said hospital legal advisor, examiner asked
who else, i said ex husband. He asked who else..then
)he told my by himself that GP can be contacted
then he asked if she gets improved then how will u
manage the issue. I said if she is proven competent
.)upon recovery then she should decide (autonomy
:3
:Cvs
lady with palpitations
Midline sternotomy, audible click with 1st heart sound.
Pacemaker scar. I gave MVR and functioning well.
Other candidates told there was ejection systolic
(-:murmur of aortic which i didn't find
.Questions were typical
.Neuro : examine lower limb of lady who has weakness
Examination showed power 3/5 both lower limbs.
.absent ankle jerks, downgoing plantars
Stocking sensory loss of all modalities. I gave d/d of
peripheral sensorimotor neuropathy but examiner was
.interested in spinal causes and spinal level
.Asked me to give specific investigations only
:2
Yrs lady with multiple visits with sob and wheeze, 40
smoker. GP found wheeze at exanination. And no DVT
Further hx
Cat at home. Hx of sob at cold exposure. CONCERN : is
.it cardiac
No suggestions of cardiac). I gave dd of asthma copd (
churg strauss etc. I am still confused whether there
.was any trick in case
:1
:Abd
.male with night sweats
HSM with no lymph nodes, gave dd of
lymphoproliferative and myeloprolifetive disorders
.viva about invx and management
Respiratory : subtle findings. I found wheeze, examiner
was interested in clubbing (though not apparent, he
wanted clear answer whether there is clubbing or not.
.Viva about invx and management

My exam experience in chennai today 3/4/17


Station 2
History taking of cough and shortness of breath for 6
months. She has fever on and off but no night sweating
weight loss of 6 kg within 6
Months. No wheezing she works in printing company
and her colleagues have also cough which she is
thinking due to printing materials they use all
cardiovascular history is negative. No history of TB
contact no HIV risk factors. she receive many
.antibiotics without improvement
I put differential of TB and lymphoma and asthma.
During childhood she has asthma which improved. Not
smoker or drinker. But unfortunately the diagnosis was
.extrinsic allergies alveolitis. I miss birds at home
Station 3
.Mitral stenosis with AF it was clear
Neuro
Is parkinson plus CVA. Rigidity only on distraction so I
put parkinsonism plus CVA reflexes was exaggerated
.on the right upper limb only no tremor
Stations 4
.Easy case explain renal biopsy for SLE patient
Station 5
Difficult young on phenytoin developed seizures
yesterday after history of vomiting once and loose
.bowel four times
.I don't know the case
Second case history of bilateral knee pain with stiffness
less than 10 minutes. It was osteoarthritis no other
significant history of skin rashes or other joint problem
.except back pain occasionally
Stations one
Polycystic kidney and clear function fistula on dialysis.
Polycystic kidney is common in chennai take care it
..mimics hepatosplenomegaly
.Chest I don't know it
pray for all candidates in chennai please
Firstly I want to share that after a long journey I
have cleared PACES from Chennai. My score is
159/172. I am thankful to all teachers, mentors,
members of this wonderful group, my family,
friends and well wishers who have encouraged
me during my dark days of failure. It was my
third attempt and making mind for this was not
easy.
Anyway, coming to this diet,
Abdomen- Large Liver with fullness of flanks. I
was not confident of PKD and hesitated a bit.
11/20
Respi- Middle aged lady with diffuse polyphonic
wheeze. Trachea was deviated to right and there
was supraclavicular hollowing at right side.
When asked about the diagnosis I said
obstructive airway disease with possible
fibrosis or fibrocavitary lesion. I was forbidden
to exam the front , so I said I would expect a
bronchial BS or Crackels at right side. Then
about investigation and management was very
smooth. 20/20
History: 35yr old male with recurrent chest
infection. Infertility and malabsorption. I did
explore all social and familial issues including
economic. D/D -Cystic fibrosis, cartegeners
Syndrome. Questions came regarding
investigation and management. 20/20
Cardio- MS with PAH in sinus rhythm 20/20
Neuro- Command was middle aged man with
difficulty walking. Examine the neurological
system. Initially I thought but parkinsons. When
asked to exam gait, the patient took 1min to
stand and adjust his dresss. I got panicked due
to ongoing time loss. Anyway when I saw a
circumduction gait, I got relieved. Hemiplegia ,
questions on investigation , localization of
stroke, management etc.20/20.
Communication: 26 yr old Advocate, diagnosed
with ESRD. 5 yr back he was seen a blood
donation program when his BP was high. No
follow up. Now task is to discuss the diagnosis
and treatment options. I started in BBN style
then focused on the disease and effect of ESRD
on different systems of our body. Then focused
on management options- general and specific.
Transplant, Hemodialysis, Two varieties of
Peritoneal dialysis. The surrogate repeatedly
asked whether the doctor who saw him initially
is negligent? Questions came more on
treatment than ethical issues. 16/16.
BCC1: TIA, a middle aged man with active AV
fistula. I forgot to ask history of smiking and
alcohol and did a sketchy neuro exam which the
examiner did not like. He was expecting a more
detailed exam. 25/28
BCC2: Middle aged lady with fatigue. Large
goiter, clinically hypothyroid. I forgot to exam
ankle jerk. Questions came on investigation and
management. 27/28.
My first attempt (Kochi February 2016) was
underprepared one but surprisingly I scored
very high 153/172 and lost in skill B by 1 mark.
In second attempt ( Kolkata Nov 2016) I again
started 2.5month before the exam. I tried hard to
make it through by concentrating on station 5,
but was not confident in clinical stations. I lost
the diet 132/172 , 4 mark short in skill B. While
preparing I applied for the third attempt in
Chennai ( April 2017). After the second failure I
started rapidly and this time I tried to form a
group, so I was shifted to rented house and
some of my other exam gong friends actively
participated in group discussion and seeing
cases together. This actually helped a lot and I
regained my confidence.
I have noticed in my previous attempts that
examiners ask only few questions in clinical
stations and they expect a quick systematic
answer. I have prepared timed answers ( 30 -40
sec) for investigations and managements for
most of the common cases and practiced it
repeatedly with friends, juniors and even mirror.
In reality these have made a difference which I
felt this time. I did not have to think when I was
answering the common questions.
I suggest all fellow comrades to prepare their
own notes and rehearsal beforehand so that
everything becomes smooth during the exams.
For understanding I shall share my notes after
few corrections soon.
Lastly, I can surely say that even if it took me 3
attempts to clear PACES, but it enhanced my
clinical skill significantly and made me a better
doctor. Thanks you all again.

My exam first cycle dubai 20/2


I start with history Yong patient complain of chest pain
with family history father died at 45 years IHD and they
gave that his cholesterol is high upper normal started
by grating the patient introduce my self as Dr told us
then I asked about the pain dull aching not radiated no
any other symptoms
He is smoker
his work as something inthe bulding carrying object
that what I understand and he was seen by other
doctor and stress done for him 9 month back
Then his concerns what is the cause of the pain I said
we need to rule out ischemic cause first as he had
strong family history and smocker and his cholesterol is
high but he is not happy then I said we need to do echo
and possible coranery angio then
I closed by checking understanding
Discussion with examiner's what could be the cause
I said ischaemia. And mention many differential but he
is not happy at the end I said muscular
Then he told me what will be your first diagnosis then
I said ischemic then muscular time finshed i am not
happy they gave me 12

Station 3 first
Cardilogy aortic stenosis with classic murmur but large
volume pulse collapsing and neck vein pulsation
Pulse regular
Discussion what is your finding
Diagnosis
AS and possibly AR but I could not heard the murmur
who you confirm ur diagnosis
echo looking for 1234
Management
Education and counselling avoid exercise
Valve replacement then at end what other i said TAVI
I got 20
Neur examin hands of the patient
No obvious deformity I start with inspection then I
examin ulnar median and redial she had lt hand
median palsy
Discussion what could be the cause investigation
I got 20
station 4 Communication
Bdn of uncertain malignancy
Long scenario female 80 ys presented e
vomiting,obstructive jaundice loss 20 kg of body
weight but still obese for 2 m u/s and CT no mass no
lymph node Stent inserted in common bile duct no
malignant cells in biliary fluid but still malignancy is
suspected
Task inform her daughter and answer her queries l
started as Dr zain taught us identify myself my role,
check relative identity, ask if she want any body to
attend meeting said no I inform her about reason of
meeting, then I ask her about what she knows about
her mother condition she started to talk for about 3
minutes telling the story of her mother so I explain to
her the result of investigation and told her that we are
still suspecting that her mother bad growth she said
what do mean I told her mean malignancy she get the
phases bad news reaction l left her to express herself,
and after becoming calm asked what happened after I
told her we need to take tissue sinp from suspected
area, she told please don't tell mama I answer as in
doctor zain course I handle thus issue gently if your
mam wants to know we will inform her we will not
enforce information to her and this will help her to talk
decision about management plan and if hide
information she might know and then she loose trust
in medical team. and accepted. Another concern want
to take her mama I said now we need to do some test
and I need to consult my senior and oncologist then if
she remain stable after she can be discharged then
discussion was about ethical issues, why you will
inform the pt I answered as said to daughter and the
autonomy of pt as she is competent. as about elment
of competence he asked about involving the senior
I got 16
.St 1
Resp. Middle aged lady with COPD and -
.fibrocavitatory lesion
Qs: causes of COPD, what are the possible cause of the
lesion (old TB, ABPS), what investigations to do and
how would you manage. When I was presenting my
findings I got confused and forgot to mention the
bronchial breathing and VR over the cavitatory lesion.
15/20

Abdomen. Young male with functioning AV fistula -


and HSP, no signs of CLD. I wasn't sure exactly how to
link all findings together when asked. I suggested 2ry
amyloidosis or CTD leading to ESRD. Also asked what
investigations would u do for him. 19/20

.St 2
Young male known to have asthma with worsening
symptoms over 4 months. The key point in history was
a new pet cat he purchased 3 months ago. His concern
.was losing his job because of recurrent absence
Examiner asked me about differential diagnosis, tests
to be done. I said skin allergen test then he asked
about the latest test -> RAST. he asked about the
method of RAST test which I didn't know then he went
on to ask about the difference between atopy and
anaphylaxis but thankfully time was up. 19/20

.St 3
Neuro. Young lady with cerebellar syndrome and pes
.cavus
Qs: what is pes cavus associated with? Differential
diagnosis for this case (FA, MS, vascular, tumor) What
sensory findings to expect if she had friedrich's ataxia -
> peripheral neuropathy. What Investigations? 20/20

Cardio. Confusing case. Lady with midsternotomy scar,


palpable S2, LPH. I couldn't hear any prosthetic valve
clicks or murmurs. S2 was split. I gave differentail
diagnosis of ASD with previous corrective surgery or
pulmonic valve disease and pulmonary HTN. Examiner
asked about investigations only. And surprisingly I got
20/20

.St 4
yr old lady with diabetes was admitted to the 25
hospital with pneumonia and while she was admitted
ahe received the wrong type of insulin when compared
to her GP notes and developed only 1 episode of mild
hypoglycemia. Task was to explain the error to a
."somewhat" angry patient
Concern was having another hypoglycemic attacks at
home. Examiner asked who was responsible for the
error I said it was a medical team responsibility as
there are multiple factors leading to it. While the
doctor was overwhelmed in the emergency room his
senior or the nurse could have contacted the GP for the
medication list. Examiner seemed happy with that
response and asked what could have been done to
avoid it. I suggested a double signature system for
medications and a pharmacy policy to review GP
records of long term medications before prescribing.
16/16

.St 5
Case 1. Middle aged female with worsening exertional
dyspnea and ankle swelling. From history she said she
was hypertensive for 2 years but didn't take any
medications for it. On examination she had an
inframammary scar, raised jvp and bibasal crackles as
well as lower limb edema up to the knees. I couldn't
hear any murmur probably because she was a little
obese and I was rushing to address the concerns. I said
the pt was in heart failure, the scar suggests mitral
valve disease that was repaired and probably recured.
The other possibility is untreated HTN. Examiner asked
about management of heart failure and hypertension.
23/28

Case 2. 30 yr old male with uncontrolled high BP


.170/100
History was only positive for similar problem in his
father who had high BP and developed an intracranial
hemorrhage. Examination was negative but I forgot to
check for radiofemoral delay so of course the examiner
asked about coarctation of the aorta, what were the
other diffentials (APKD, phaeochromocytoma) and if I
would admit him. 24/28

Overall it was a tough exam but I passed with the


.praise of Allah

Chennai-Nov 16
Communication station in cochin
years lady diagnosed with obstructive jaundice 75
relieved by stenting and now the pt well ,the team not
found the cause despite full investigations
But the team think that the cause either small
pancreatic cA or cholangiocarcinoma
The pt give you a consent to discuss with her daughter
her condition
Task to explain mother condition and inform relative
about uncertainity of diagnosis
:Daughter cocerns
What could be the cause
May she go to die
Why the team not find the cause yet
What can we do for her
Can we hide this information from her

The examiner appeared not satisfied when i told her


that if she asked to know about her health i will inform
her as she is competent and has all the right to know
,and he asked about prons and cons of telling the
patient the posdibility of CA

Also he asked:Do you think this patient need palliative


care and still the diagnosis not reached ? I told him
palliativr care isnot only for cancer and obstructive
jaundice in this age mean advance stage of spreading
of growth
Also he was not happy about term of stent coz hr
thought this is jargon
Oman 6/4/2017
S2
Postpartum thyroiditis-
Female 55 yrs complains of intermittent early -
morning headache+vomiting having pmhx of
hemiplegia CT done was normal and depression
.+anxiety on FU with psych
Her headache sounds to be due to SOL
Tension headache less likely
I think CT needs to explore whether done with contrast
.or not
St 4
.Pt with UC to be convinced for steroid recently came-
Pt with nsemi underwent ptca started on dual -
antiplatelet and other medication told by her gp that
she is allergic to aspirin but that wasn't in her record
.Given aspirin and developed epigastric pain
Your role is to speak to her husband to persuade her
.for compliance

I would like to convey my experience for paces April


2017
Station 3
Cardio aortic valve replacement
Neuro paraplagia
Station 4
Late diagnosis of pheochromocytoma
Breaking bad news plus negligence
Station 5
Shoulder pain in diabetic patient - 1
Adhesive capsulitis
Weight loss -2
Thyrotoxicosis plus goiter
Abd
Right iliac fossa scar without transplanted kidney
Left iliac fossa big scar with a mass under it
Possible kidney transplant with possible poly cystic
kidney

Chest
Copd with clubbing
.Possible interstitial lung disease or bronchiectasis

History
Anaemia in young female using diclofenac for back
pain with history of bilateral ankle swelling
Family history of colon cancer in her father

My colleague exam today


Oman 10/4, 3rd cycle
:St 1
.)Chest: Bronchiectesis + Thoracotomy (? Bullectomy
.Abd: Renal transplant
:St 2
.)Patient came with collapse (most probably HOCM
:St 3
Cardio: MVR
Neuro: Cerebeller syndrome + unilateral hypothesia
.with persevered dorsal column for DD
:St 4
BBN about patient known HTN on treatment, not
controlled, and he has DVT on warfarin with INR 4 and
developed Large SAH who is ventilated and admitted
.to ICU
St 5
.)BCC 1: Young female with headache (Migrain
BCC 2: Patient with previous hypophysectomy on
treatment developed gradual loss of vision (?
.)Recurrence

Oman exam 8/4


St 5
lt shoulder pain ?? Frozen shoulder 1
toxic goiter 2
St 1
Transplanted polycystic kid d having fistula
Chest: bronchiatisis
St 2
Fatigue with family history of colon cancer
?? Her mother taking iv drug every 3 months
She was taking also diclofinac for 6 months
St 3
All candidates said AVR
St 4
Delayed phychromocytoma diagnosis
Oman 3rd cycle 8/4

Station 1 seems polycystic kidney


Bronchiectasis due to cystic fibrosis
Cardiology AS. MS
Neuro. Charcot marry tooth
Station 4 headache and fatique
all investigations are normal they asked you to tell her
about the results
Conversion disorder
Station 2 back pain inflammatory type
With family history of rash
Station 5 essential tremors and psoriatic arthropathy

oman 4/7
Cycle 3
St 1
Congenital bronchiectasis kartegeners syndrome
Renal Tx
St 2
Cough and SOB for 6 months
St 3
Spastic paraparesis with intact sensation for DD
AS
St 4
Elderly known advanced bladder cancer and bed ridden
admitted with obstructive uropathy and deterioration
of consciousness for discussion with his son about need
for nephrostomy and future treatment
St 5
Reduced visual acuity in a young (33 yrs) with
background HTN and DM
valve replacement came for follow up and c/o dry eye
for few month

Cycle 1&2
Kartegners syndrome with lobectomy
.DVR. AR
Transplant kidney. Chronic h anemia with splenectomy
Charcot marytooth. Spastic paraplregia without
sensory level
Lung fibrosis due to amiodarone
History. Pt with familial hyperlipidemai presented for
chest pain

Oman exam 9/4 cycle 1-2


History postpartum thyroiditis
Communication CKD
Chest bronchiatisis + copd
CVS DVR
Abdomen Thalassemia with HSM
Neuro Charcot marry tooth
Ct5 Neurofibromatosis. Thyroid
Oman exam 10/4
Cycle 1&2
??St 1 bronchiatisis + liver transplant
St 2 common variable immunodeficiency vs hiv vs CF
vs hypogamaglobulinemia
St 3 DVR. Cerebellar syn
St 4 SLE for reanal biopsy
St5 simmple goiter + psoriatic arthropathy

.Oman 10/4- 2nd group


information:following in medical clinic for abd -1-5
problem, dirrhore few days. Vitals normal. Some nail
changes treated by GP as fungal inf, no benefit
Inside: it was AB induced dirrhore,lasted 3 days,
settled now without treatment, Actual problem was lt
wrist, ltMPJ Arthritis,Rt normal, brown rash macules all
over body , lower half of legs are all brown, said no
treatment taken ever, long lasting - deliberately hided.
Nail only flat, one nail only slight onycholysis, only one
side. Later same patient gave us lift and told us he have
.psoriasis, treated, only one hand arthritis
f, Dysphagia to solids for one month, no other 50 -2
positive Hx, ex only small central thyroid nodule, no
.other finding or Hx
Abd: bilateral IF scars, RIF mass, Rt chest and neck -1
.CVLine scars, some gum hypertrophy
Resp: Yong male, Lt Lower half coarse crackles,
..rhonchi
Hx: recurrent chest infection in a Yong male, is DM on
.insulin, Asthma on Symbicort
Recurrent chest infection since child hood, poorly
controlled DM, constipation, infertility - immobile cilia,
.CF and others as you said
Resp: localised bronchiectasis, I mentioned all
.posibilities
CVS- long midsternotomy scar, extending to -3
umbilical, collapsing pulse, Double metallic clicks, AR
murmur, apex 5th ICS, lateral to mid MCL. Double valve
.with AR
CNS: LL-- Examiners didn't allow to check gate, back,
said normal. Bilateral cerebellar signs, lt LL lmn
weakness with decreased sensations, dorsal column
.ok- it was confusing, I said MS
SLE and Renal biopsy -4

I would like to share my experience which was in 23/3


in Glasgow in Golden Jubilee Hospital
I passed alhomdellah with 148/172
This is my first trial in PACES but i had the sense that i
know what to expect in the exam from the experience
that had been shared from other colleges so i would
.like to thank everybody who share his experience
I started with station5 and when i knew that i panicked
because i thought if i messed it up i will fail but then i
said to my self just do what you can and forget about it
.in that room don not take it to other stations
The first case was a lady with weight loss and type 1
DM so i put from outside grave' s disease and coeliac
disease and when i entered the room it is obvious that
it is Grave' s so i started by shaking hands with the
patient and then she gave history of gritty eyes and
weight loss with good appetite and diarrhea and then i
started the examination by checking the eyes she has
led lag and retraction and ophthalmoplegia and
proptosis and then i asked her to take a sip of water
she does not have goiter but i examined the neck and
there is no finding and in the beginning i checked the
.pulse and it is regular
She has positive family history of type 1 dm and she is
on b blocker and no smoking history and not drinking
alcohol and her concern was what is my problem so i
explained the grave, s and i refer her to
.endocrinologist
The examiner questions were what is the diagnosis and
what you find in the eyes and what is the
investigations and the bell rang and i did not finish the
discussion and i forgot to see the legs for pretibial
myxedema and the proximal myopathy and i knew that
i will lose marks and i got 23/28
The second case was systemic sclerosis
I became calm and confident in this case because i
found that there is nothing to worry about it went fine
the first case. The scenario was a man came with
discoloration in his fingers and pain so i went inside the
room and the patient has obvious talengectasia. I
started taking the history and the patient has rynaude '
s with dysphagia and shortness of breath and he is a
builder so i examined him i saw the talengectasia and i
saw his hands he has ulcer in his finger and the skin
also he has lung fibrosis with fine crackles and then i
asked the examiner about patient BP to exclude the
renal problem and the examiner was happy. The
patient concern was what i have and then i explained
tge disease and i told him that i will refer him to
rheumatologist and i advised him to keep his hands
warm and to avoid vibrating tools.the discussion was
on diagnosis, finding and investigation and treatment
and i knew that i will have full mark and i have it 28/28
: Station 1
The abdomen case was liver transplantation, patient
with Mercedes Benz scar so i started the examination
and the secret is to be fluent do not think about what
you want to do next, the examiner wil Will have good
impression
The scenario was patient cane with abdominal
discomfort.the discussion was about what is your
finding ? The patient has tinge of jaundice and the scar
and the liver is palpable six cm below the costal margin
and no signs of chronic liver disease so my diagnosis is
that the patient has liver transplantation and the
discomfort could be due to rejection or portal vein
thrombosis or hepatic vein thrombosis and i should say
.biliary stenosis but i forgot
Then he asked about what might be the cause of
.transplantation
What the think you will do before you send the patient
to the liver unit for transplantation and what are the
.side effects of immunosuppression medications
I got 17/20
The respiratory case was an easy one but i messed it
up. Patient came with shortness of breath. On
examination he has right side thoracotomy scar and
crackles so it is bronchiectasis. I presented my finding
and then i said fibrosis i don not know why i said it
even he said is it bronchiectasis or fibrosis and then i
said bronchiectasis but i became confused so i forget to
lung function test in investigation and i forgot postural
drainage in treatment. I knew i will have 10 and it is i
got 10/20
:Station2
The scenario was patient 36 years old male with type 1
DM came with recurrent hypoglycemia and weight loss
and anemia he has renal impairment the GFR was 38
.and he has retinopathy
So i started by asking open questions and the patient
answered by himself all my questions from the first 5
.minutes
He has recurrent episodes but he has awareness of
hypoglycemia only two times he needed help from his
wife and he is an IT and he is driving his car but now he
stopped so i told him that is good and i appreciate that
.you stopped driving
He also has abdominal pain and recently discovered
.that he has renal problem
.Not smoker
.His concern was what i have
So i explained that he has an autoimmune disease
which is type 1DM and there are others and now he
may have Addison and i explained what is it and it
could be due to the renal problem and he may need to
the endocrinologist to decrease his insulin dose but
idid not say to the patient coeliac disease. The
examiner question what is your differential
So i put addison, coeliac and due to renal problem and
then he asked why he has anemia i said it could be due
to the chronic kidney disease so he asked what GFR can
cause anemia i said 30 and then he said why he has
.anemia i said pernicious
Second questions was what investigations and then he
asked the patient is on ramipril what you will do i said
he will continue on it because t is protective he said
excellent answer i got 16/20
Actually it was nice experience and the examiners was
good
:Station 3
The cardio was an old patient with regular pulse and
on examination he has pansystolic murmur radiating to
the axilla
Examiner asked what is your finding
And he was so happy about my presentation
I said that the patient has regular pulse 70 per minute
and he has pansystolic murmur radiating to axilla and
my impression is that the patient has mitral
regurgitation and the differential is Tricuspid
regurgitation but the things that against are no
hepatomegaly and raised JVP and the other differential
.is VSD and the age of the patient against that
The examiner agreed
And then what are the causes of mitral regurgitation
What investigations
What are the treatment
I said replacement and then he asked about the
indication for it
Then he asked me about the medical treatment and i
said vasodilators and diuretics
I got 20/20
The neuro was old patient with mask face
The instruction was to examine this patient
I started doing the parkinson disease examination
I asked the patient a question how he came to the
hospital
And then i examined him for tremor
And then i did the tone in hands
And the examination for bradykinesia
In upper and lower limbs
Then i asked him to walk
And the finger nose test
Just i forget to test the supra nuclear palsy
But the examiners were so happy and he said you still
have time but i said i don not have anything to do else
and then he said what about the eyes then i
remembered and i told him how i will do it and he said
ok no need to do it
In discussion
What is your diagnosis
What is the differential
I said parkinson plus and the drugs induced
What investigation
I said it is a clinical diagnosis but if in doubt we can do
CT and MRI and SPECT scan
What treatment
I said all the medication with their side effects
He asked what this patient at risk of
I said fall
He said what you will do
I said multidisciplinary with physio and occupational to
adjust the house and also the social worker for benefits
i got 20/20

Station 4
The scenario was 54 old patient with congestive heart
failure and he is at maximum treatment and the
cardiologist said no more added treatment then he
complained of lump in his neck and biopsy was taken
and CT abdomen revealed that the patient has primary
kidney cancer which is spread all over his body
The task was to break that news and to tell the patient
that he is for palliative treatment and the role of
.specialist nurse
I started by asking him if he wants some one to attend
with him and he said no body
Then i asked him what he knows about his condition
and he said everything about his cardiac condition and
he knows what the cardiologist said

And he said that he has a lump and a biopsy was taken


and he wants to know the result

So i asked what do you think the cause and he said am


afraid of the worst so i told him am afraid you are right
and then i told him the result if the biopsy and the CT
Then i kept silent and i let him speak and he took his
time then i said can you tell me what is going in your
mind and then he said he is shocked so i show empathy
and i gave him time again then i asked him do you
want me to proceed
He said yes and then i told him about the palliative
treatment and i told him that we will provide support
to him and his family and he is wondering how to tell
his family so i offered to him to bring them in the
upcoming meeting and i will tell them
Then he asked about how long he will live and
empathetically i said i don not know then i proceed
and told him about the specialist nurse and i asked him
again for more questions he said he can not think now
so i told him to write every question come to his mind
and we will discuss in the upcoming meeting and then
he repeat the question again about how long tine he
has
And again i said i don not know and i explained that no
body can tell
Time finished and the surrogate while he is going out
he told me verygood
I got 14/16
The discussion was about why you did not specify time
the examiner wants me to tell the patient that he has
months to live so as to prepare his living will
And then she asked me about the ethical issues
I said breaking bad news
And beneficence vs malfecience
Both examiners were happy
And the. She asked what websites you want him to
search on and what you don not want to search
I told her that i don not want him to search for hope
while there is no and treatment will harm him more
and his functional level already impaired
By the cardiac problem

And during my discussion with the patient i advised


him to go to support group and that is why she asked
about the benefits if the support group so o told her
the benefits are for the patient and his carers
I finished the exam with score 148/172
Sorry i forgot also i told him about the role of the
specialist nurse and how she will help him and his
family

Oman 9/4/2017
Day 4 Cycle 2
I started with station 2
My case a lady of 35 years old present complaining of
fatigue and weight gain 3 Kh in the last 6 months , she
gave history of typical hypothyroidism
I ask about other hypos no other symptoms apart from
. ) amenorrhea ( hypogonadism
She had family history of thyroid problem ( her mother
)and her any is using regular vitamin injection but she
. is not sure what is it
Her last delivery about
months ago and was eventful, she had been 18
transfused much amount of blood , but against
Sheehan's syndrome she is lactating for more than one
.year
.No PMH and not on any medications
: Examiner question what is your DD
Postpartum thyroditis ( he ask why you think about it )
I told him because her symptoms started after delivery
)
Autoimmune poly glandular syndrome (family history
)of endocrine disease
Sheehan's syndrome .( against it no symptoms
suggesting hypoadrenalism and she is lactating for one
. year
: Other questions
.Investigations and treatment
At the end he asked about Addison disease
? How do will treat patient with Addison disease
I stared if he came in emergency I will stared with IV
fluid normal saline
Before I answer hydrocortisone bell rang
Station 3
*Cardiology*
. Examine this patient and present your findings
First and second heart sound are metallic clic
MVR + AVR
:Questions
Investigations
Echo , What is single blood test you want to do (*He
)*want INR
Other question can you prescribe for him new oral
anticoagulant ? I told him up to date it's not license in
patient with metallic valve
)*Neurology*(
: Strange case
.Young patient with lower limbs weakness
On examination LMN weakness (proximal myopathy)+
loss of vibration sense with intact joint position in the
.) left side
Most probably the patient had Becker ( I could not
) match the things together
I told him proximal myopathy for DD but against that
the loss of vibration sense in the left side
He ask me forget about vibration sense , what could be
the cause of his weakness ? How are you going to
investigate him
Bell rang

*Station 4*
years old Omani male he is complianing of fatigue 29
and dizziness for the last 6 months , today he had been
call by his GP because he has abnormal renal function ,
.your role to explain to him his condition
BP : 160/105
K : 5.2
High creatinine
. High Urea
. High creatinine
During discussion he told me I have cut wound in my
? hand 5 years before could it be related
My BP was high since that time and the Dr.at that time
. did not bother him self
? What is final treatment
I explain to him what chronic renal failure , what is
complications and what are option of treatment
.)including renal replacement therapy ( dialysis
:*Examiners Questions*
You mention to this gentleman that you want to admit
him , why ? During the discussion the patient
mentioned that he has shortness of breathing ,
consequently I told him you might need dialysis after I
.) full examined you and doing CXR ( fluid overload
As he is lower what mode of dialysis you will over him
? I told him haemodialysis is better ( less chance of
.)complications
Then he asked me what is problem of high serum K ?
?Do you think 5.2 need dialysis
*Station 5*
*BCC 1*
years old male complaining of attacks of 35
. headache, sweating and palpitations
When I went inside I'm asking about course, onset and
.duration of symptoms
I asked also about aggravating and decreasing factors
)(none
Loss of weight , fever night sweating , analysis of
headache , analysis of palpitations
Mean while am asking I told the surrogate I will
examine the patient meanwhile I will ask some
questions
It was clear neurofibromatosis
After that I ask about PMH , family history ( he had
family history of similar condition). Medications history
.

I ask about the concerns. She ask me what the is cause


? for his problem
? How he can be treated
? How skin lesions be treated
I ask I want to examine BP , they told me no need
I examine the back of the patient looking for cafe auilt
spots
. Auscultation of heart sound
.Bell rang
:Examiner Questions
?What is your diagnosis
DD
.Management
. Time finished
*BCC2*
years old male with history of loss of weight and 38
. fatigue
Hyperthyroidism
.Graves' disease

)Alhamdulilah passed PACES in Malta ( 2/4/17


Neuro: lower limb umn monoperesis
Cardio: AR
Abdomen: ?CLD
Respiratory: ILD
BCC1: IHD with stent complaining of chest pain one
month, one episode faintaing ( bradycardia on
atenelol), stent thrombosis
BBC2: spine surgery 6 wks back complaining of fever,
lethargy n persistent back pain one week? epidural
abcess
Communication: 80 yrs old female admitted with CAP
.developed delirium 3 days after admisdion. Talk to son
Son concerned if his mother will have permanant
.psychiatric problem
History taking: young marathon ...chest pain
.palpitation and syncope
Concern,,,,if its an epilepsy

My experience of PACES 01.04.2017 MALTA


I started with station 2 : 24yrs old lady c/o fatigue and
headaches found to have high BP 160/98 on different
occasions. No family hx , no
complications.....examiners asked would you start
treatment...i said not now if secondary causes r/o
including white coat htn then yes. Discussion about
.secondary causes of htn and investigation
Got 20/20
Station 3: cardio VSD/TR pansystolic murmur in
tricuspid area. Discussion about investigations and
.management
Got 16/20
Neuro; middle age with walker reflexes with re
inforcement rigid limbs throughout....planters
equivocal no cerebellar signs ...out of time could not
check sensory system....made dx of spastic
paraparesis....asked of causes got 11/20
Station 4: talk to the wife of 50years man known MND
recurrent admissions for pneumonia. This time
admitted with aspiration pneumonia not responding to
anti biotics and confused. Discuss with her his
management and condition. I explained to her that he
is in advance stage....and any invasive procedure like
intubation is futile. We will give him palliative
treatment only and keep him comfortable. Examiner
told that i did not told about NIV when the wife asked.
Got 15/16
Station 5: first case young lady with joint pain...i was
happy will be from common causes. When i entered
took hx all joints pain including hand joints. Surrogate
told she was told by GP that she has double joints. I
was more confused...asked what u mean double joint
she said i dont know gp told me that and he was not
sure.i examined no sign of inflammation. Raised skin
dorsum of hand elastic . Checked functions. Found
high arched palate. Joints hyperextensible.arm lenght
normal. Gave diff of marfans pseudoxanthoma
elasticum forgot to tell ehlers danlos syndrome.
Examiner asked were there marfanoid features i said
no. I wanted to examine chest they stopped me asked
what u want to see i said AR....they said normal. Asked
how you investigate joints in general...started with
inflam marker RA factor ACCP...time finished got 28/28

nd cast st.5: believe me i totally forgot....got 23/282

Station 1: abdomen; lady in 50 has pruritis and abd


pain.....i neglected pruritis....no signs of CLD or
immuno suppression...distended abdomen tender all
over mid line laprotomy scar below umbilicus. Shifting
dullness+ could not appreciate any organomegaly or
mass due to tenderness. Differential ...causes of ascites
asked about causes of transudative ascitic fluid....got
8/20

Respiratory: c/o of Sob pneumonoctomy scar left side


trachea deviated to left no breath sounds left
......asked about investigation ....told so many still
asking was happy when told sputum analysis and c/s
and AFB ....infection as cause of Sob
Got 20/20
Wish success for all you going for PACES......my advice
is to perform during exam in relaxed way, smile and
forget about any station where you did not perform
.well

My experience
I did my paces exam in malta
Mater dei hospital on 2 April 2017
At 9 am
St 4
ys woman admitted 2 days agowih chest infection 80
started antibiotic IV and today became aggressive
refuse treatment and she said doctor and nurse will kill
her diagnosed delirium
Note the pt at time of admission
OK conscious and oriented develop this at hospital
Please took to her son had many concern
And discuss future management
I entered the room
I started same as Dr Zein told us because this the first
time to do delirium
I great the son agreed agenda
?What you know about your mother condition
He tell me what written up
So I told him you are right
He all time disturbed me
Why she developed this
I said I answer your questions
But let me ask you about your mother before is she ok
He said she is completely ok
So I asked is she developed same condition like this
before
He said yes I remembered now 2 year2ago had same
like this but short period and less aggressive I know
from him
Not associated with admittion or treatment or
changing home and resolve spontaneous without
seeking medical advice
So I ask what do you think might be wrong with your
mother
Said I don't know
I asked about concern he what to know why she
developed this
And what treatment
And if she will be OK
What about future
I try to remember all this
Concern
So I said your mother had conditions called in medical
term delirium do your heard about it said no
I will give you leaflet and about it is acute confusion
state
Means disturbed in her brain fuction so change her
way of thinking not knowing time place and person
because of that think that doctor and nurse want to kill
her
And some time the cause can be infection itself
He stop me and said she is OK when came here
I said is she complain of any
Water problem he said no
I said infection of water can come without any illness
and can cause this and her infection of chest and some
people when changing Their place can have this and
also some sort of treatment can cause this I will call my
senior to revise her medication
Is your mother in general ward means with other pt
He said yes
I told first step in tr
Treatment to shift your mother in single room and one
nurse or medical aid who is available (because I don't
know who will sit with her)to be wih her to be sure not
hurt her self
And I will call my senior
And also I will call one doctor in our team we work
closely with him called psychiatric to give you mother
medicine through blood tube to be calm to start again
her medication is that OK
And about becoming ok I am afraid I wouldn't gruntee
about that I am sorry to say that but might become
completely ok or might not but I am sorry to that your
mother had previous problem like this and her age 80
So this also factors(show empathy all through the
station l understand your feeling l know you are
worried about your mother) and right now I am
thinking about dementia do you heard about it here
said yes and he started explain it
I said yes it is aging process
But cannot come suddenly
I will give you leaflet about future who is with your
mother at home he said alone
Where are you said living away for work coming only
weekend I said any possibility to shift with your
mother town or to shift your mother with you he said
no
I asked about financial problems no if your mother
become ok we will talk to social worker and
occupational therapist to do home modifications but if
not we should discuss future career nursing home
He said yes you mean nursing home care I said yes
He started to say oh oh
At this time I told him I know I gave you so many
information so I will give near appointment in this
afternoon with me myself and my consultant to discuss
nursing home care and all you queries and worried he
)said thanks it ok(really I said that to be in safe side
I summarize and check understand
Qs by examiner he ask everything that I told to son
And then he said you said psychiatric some time not at
hospital and pt fighting
I said I will discuss to my senior to give pt haloeridol
What about leave son to sit with his mother at single
room
I said according to hospital policy the examiner laugh
and said you are doctor who putting policy I said if
possible better to stay with his mother familiar face
can help her I got 15/16
My exam experience in kolkata
st 4 pt non compliant to take steroid, kc Addison.
Husband pt of MS

St 5 (1)RA
)LOC(2

St (1)hepatosplenomegaly wth jaundice


ILD wth systemic sclerosis

St 2)return traveller bloody diarrhoea grandfather ca


.colon dx at age 78, 2nd cousin having UC
St 3 1)mixed mvd with af with hf, 2)spastic paraplegia
.traumatic

I just finished the exam b4 1 hour. Please pray for me


I pass my exam in chennia India-- Thank good
It was tough one with very atypical clinical stations
I started with station 1
Abd : middle age male with active fistula + LL odema
No signs of fluid over load
Not on anti rejection Med
Ploysystic kidney disease
With cystic liver
Q.1 what is ur diagnosis
Q.2 did u see the rash now clue if there was any
Q.3 what can cause liver cirrhosis in this pt
)worst station got 14/20 (
I think the rash is lichen plans with HCV I do not know
:Resp
Middle age man again active fistula
With hyperinflated chest
Tar stain in fingers and teeth
With no wheezes only fine Basel crip
Not changing with coughs
Q.1 what is ur diagnosis
Copd with lung fibrosis
Q.2 how would you like to investigate Q.3 any
relationship between his lung problems and dialysis
First thing may be Med then saved by the bell
one of the examiners was sitting on his chair at the (
foot of the pt
I asked him to move politely and even moved his chair
to sit far away
Station 5
I was happy
st case 1
Unilateral limb swelling
All vitals are stable
I put diff of cellulits + rupture beacker test
And I entered
To very young male
Again on dialysis with permicath
With chronic limb swelling over 5 years
Pain less with strange knee joint
?? No history of insect bite
I examined the limb
Slightly pitting
Not hot or tender I examined inguinal
LN
I asked pt to walk then sensation
Examiners escip and normal respectively
There was parathyroid scar
Concern by relative
is it related to dialysis should we increase the the )1
dialysis frequency
I answered not related and best to talk to your kidney
specialist if you have any concerns regarding dialysis
What is the cause)2
It most likely due to lymphatic obstruction
We need to start to do some test to know exact cause
and he do not need admission
Examiners
Q 1.what is cause of swelling
Lymphodema
Q 2. How would you like to investigate him
Stupid answer lymphogram
What might be the cause in this country
With more stupid smile
Chagass disease ( how stupid you may become in exam
Totally forget flaria) where Chagas come from
Saved by bell
Got 18/26
nd case2
Totally straight forward
Bilateral limb swelling in HTN
I put differential
Then went to role out serious complication of HTN
There is decrease frequency of urination
With drug history on
Nefedipine for one year
Complain also from headache
Plan of management
Admit the pt
Switch nefedipine to other Med
Doing some test and scan to ur kidneys
Those were the answer of the concerns
Examiners
Q1. How to mange pt
Education, stop medications switch to other Med,
elevation of the foot
Q2. What Med you will give pt
I said ACI
Why not diuretic
I said not recommend as pt having dependent odema +
there no evidence of fluid over load
Station 3
Cardio
Middle age female again active fistula in right arm
Exam
Obvious Lowe limb edema
Raises JVP with v wave
Obvious apical pulsation
No thrill ,LPH
Irregular pulse
Loud s1
Could not ass the 2 heat sound coz of the fistula thrill (
)which irritates the examiner
Diastolic murmur in apex
And hypothetical tricusp reg
Q1.what is ur diagnosis
Q.2 what r the causes of MS
Any congenital MS syndrome you know
I answered lutinbaker
Got 13/20
CNS
Straight forward
Pt presented with dysphasia
Examine LL
Young male with CVA posture
Q.1 what is the lesion
)Q.2 where is the lesion (subcortical
Q.3 how to investigate and mage
Indian male not understanding English lot of time (
missed in translations, I couldn't not ass the pulse
heart or cranial nerve or ask pt to walk but I told
) examainr I want to do so
Luckily I got 20/20

Station 2
History
Middle age female newly diagnosed HTN
With persistent high reading
Approach with systemic manners
No symptoms suggestive of end organ damage
No symptoms of all secondary causes of HTN
Reached gynecology history
Pt was having irregular menses with prescribed OCP for
1 year
No other important history
after I reached concern I remembered to ask (
biological family if the pt was adopted (APkD) as in
) DR.zain scenario
Concerns
Do I need to take Med for life .1
Ans as your HTN probably due to Med there is a good
chance it might be temporary
will it affect me having babies.2
Ans. High Blood pressure may serious with pregnancy
multi desplinary team involving pregnancy specialist
and your physician would keep a good eye over it
Plan of management
Stop the OCP , seek alternative with the help of ur
pregnancy specialist
We will do some test and may be scan to ur kidneys to
see the extend of ur disease
Examiners questions
Q.1 what is ur diagnosis and differential
Q.2 what is common-cause in this country if it is renal
cause
I answered post streptococcus GN
I got 20/20
Common scenario=Station 4
Young male with ulcerative colitis un controlled with
mesalazine
With inc diarrhea and weight loss
Council pt regarding steroid
Started with open questions
Then pt afraid. From steroid as he read about side
effects
Then I filled the gaps
Then every complication how we might mange
Involved the GP
Conserns
for how long you will use steroid )1
Ans
Until we control the disease the dec dose till we reach
remission with minimal dose
I want to use herbs )2
Ans
Sorry but I can't be sure what will be the effect on on
the disease course or how it may interact with steroid
If u choose to use herbs plz inform ur GP
DR.zain advice never say no to stupid thing pt want (
to do just smile and give all options to him and then till
)him ultimately you may hurt your self
Discussion
What is ethical issues
What are the complications of ulcerative colitis
anemia)1
Which type
All type
)Norm normo (of chronic disease
)Megaloblastis ( fe + b12 malabsorption
)Iron deficiency ( due to blood loss
colorectal Ca )2
This what he want to hear
what kind of diet you will give him )3
I tried to be smart I will refers to dietitian
Then agin what diet
I told him high fiber diet
He asked me ru sure
I said with smile of ignorance
Yes sure
I got 20/20
My exam experience in bradford royal infirmary on 27
.march 17
:BCC1
lady comes with tiredness fatigue and sob, with
previous hx of some brain surgery (hypophysectomy
and she was on steroids, levothyroxine and GH) hx
looks like OSAS. It didn't clicked to my mind that she is
acromegaly and OSAS is due to acromegaly itself. I
gave differential of hypothyroidism and cushing being
cause of her OSAS. Exam findings were nil for anything
:-( she repeatedly gave me clue why she is sleepless
.and i kept on beating about the bush, same as viva
.BCC2: RA with sob and GP mentioned nodules in xray
I couldn't find any abnormalities on chest
auscultation:-( and gave d/d if rhenatoid lung and they
asked about any d/d of pulmonary nodules, i said
.neoplastic. Next qs about invx and mx

:4
Deal with annoyed son who is NIK and his mother was
admitted yesterday with acute confusional state due to
UTI and AKI (brought by ex husband who claims to be
taking care for her) son seems to be away and less
caring but he was annoyed why his father (who left his
mother when she needed) is around and he demanded
that his father shouldn't be around and why the son
was not called by the hospital. And he semanded that
when she will be discharged he doesn't want his father
to be around. (Being next of kin he demanded that he
has the right to decide about his mother) i inquired
about power of attorneyband advance directive which
.son said he has no idea and repeated that he is NIK
.Questions were about rights of next of kin
and if son claims that his father is using mother to get
financial benefits and he is responsible for her ill health
then what should be your stance. Will u ignore it,
)where to report it.(I had no idea
In case there is no legal report where you will get
guidance (i said hospital legal advisor, examiner asked
who else, i said ex husband. He asked who else..then
)he told my by himself that GP can be contacted
then he asked if she gets improved then how will u
manage the issue. I said if she is proven competent
.)upon recovery then she should decide (autonomy

:3
:Cvs
lady with palpitations
Midline sternotomy, audible click with 1st heart sound.
Pacemaker scar. I gave MVR and functioning well.
Other candidates told there was ejection systolic
(-:murmur of aortic which i didn't find
.Questions were typical
.Neuro : examine lower limb of lady who has weakness
Examination showed power 3/5 both lower limbs.
.absent ankle jerks, downgoing plantars
Stocking sensory loss of all modalities. I gave d/d of
peripheral sensorimotor neuropathy but examiner was
.interested in spinal causes and spinal level
.Asked me to give specific investigations only

:2
Yrs lady with multiple visits with sob and wheeze, 40
smoker. GP found wheeze at exanination. And no DVT
Further hx
Cat at home. Hx of sob at cold exposure. CONCERN : is
.it cardiac
No suggestions of cardiac). I gave dd of asthma copd (
churg strauss etc. I am still confused whether there
.was any trick in case
:1
:Abd
.male with night sweats
HSM with no lymph nodes, gave dd of
lymphoproliferative and myeloprolifetive disorders
.viva about invx and management
Respiratory : subtle findings. I found wheeze, examiner
was interested in clubbing (though not apparent, he
wanted clear answer whether there is clubbing or not.
.Viva about invx and management

Khartoum exam today


History IBD
Communication counselling regarding warfarin
Speaking with son
Station five AF for dd
Station bcc 2scleroderma
Abdomen transplanted kidney
Chest double pathology???fibrosis and effusion
wheezey
CNS proximal weakness
CVS valve replacement
another cycle in sudan
Hx : palpitation
post partum thyroiditis
Station 5: RA no clear deformity pt clubbed chest
wheeses more prominent with possible crackles on
methotrexate the scenario from outside only dry
cough & short ness of breath
history panhypopiturism sheehan synd
communication counselling newly discovered renal
failure pt eith shrunken kideny

another cycle in sudan


RT.1
Left.lower lobectomy.1
.cystic fibrosis.2
MVR. FLASID PP.3
.Renal biopsy in SLE.4
.BCC1Peripheral neuropathy due to anti TB.5
BBC2.Familial hypercholestrolemia

Alhamdulilah I have passed PACES; got 159/172


..my exam experience Glasgow Feb 2017
I started by st 3
Cardio
Pt e SOB; O/E; mid sternotomy scar, metallic click , no
murmur , my D; MVR, viva inv & manag + valve types!!,
why no saphenous scar?! got 20/20

Neuro; pt with falls; examine his neurological system; I


started as regular greeted the pt and asked to walk;
Parkinsonian gait, I examined tone then tried
Parkinson approach as Ealing vedio but no tremor
evident, then I was stuck and dont know what to do
more, I tried power, reflexes and bulbar ex and time
out, viva about Parkinson, I thought I performed bad,
20/20
st 4
syncope due to OHG overdose in depressed nurse, who
denies the act but confirmed inv and previously told
about insulinoma, task to communicate D and manage
;concerns
concern: confidentiality, financial troubles & support, I
missed in scenario from where she got the OHG,
thought she was D then corrected myself (looked not
)good
viva; what ethics here, from where she got the OHG
and if from hospital any implications, what kind of
support! And how about psychiatry assessment
I got 11/16

BBC1
Systemic sclerosis with swallowing problems, straight
forward, concern; is it curable?! I dont know what I
missed, 26/28
BBC2
young man with Visible Haematuria with normal Ex
and history, concern is it cancer, I missed a good DD
and plan of management , EX were upset , got 22/28
st 1
Abdomen
Scar of liver transplant + drum stick clubbing, viva abt
possible D and Inv , transplant medications and SEs,
got 20/20
chest
COPD with bronchiectasis, viva causes and inv and
management, got 20/20
st 2
Dizzy spells , postural hypotention and tachy, in AF +
DM+IDH+HTN+DVT, gave DD uncontrolled AF / Drug /
Autonomic neuropathy, viva inv and management, got
20/20

I hope this helps


Paces is a very tough but amusing experience, I passed
from 3rd trial all in UK, done courses in Ealing,
Hammersmith in London and paces 4 u in Manch, got
110 in 1st trial and 117 in 2nd , now 159 praise and
favour to Allah
Try to study hard 50% and try to practice in your real
life 150% and in shaa Allah you will Pass
Hope the best for you all, much thanks to this group
and all colleagues, your experiences were so helpful
Have a nice day

Malta centr 2 April 2017


Started with abdomen
Left hypochondrial mass said spleen
.Pneumonectomy straightforward
S2
Young gentleman 24 year while running a marathoon
(after 8 klm) lost consciousness with jerking , brief
.concerned is it epilepsy
.PHx similar episodes but no loss of consciousness
. No DM NO epilepsy or other illness no trauma
Drug Hx
eczema on antihistamine the examiner said it was
significant and i should have taken more details ' I
. think they wanted you to think about prolonged QT
FHx adopted
Social negative
. Concern is it epilepsy
S3 Don't know other candidate said aortic
.regurgitation
Examine lower limb upper motor findings in one leg
gait hemipligic with walking aid

S4
Speak to Mr ...son of mrs ....80 year old lady admitted
3 days with pneumonia and developed delerium task
.explain mangement and answer concern
Discussion about treatment and prognosis of delerium
.
S5
Repeated lady after back surgery came with pain and
fever
. Discussion how to cover staph aureus
...........
Other case
years post CABG presented with chest symptoms 12
. and faint concern is it heart attack
Discussion about beta blockers side effects

I have passed PACES 168/172 on 7th March 2017, 1st


diet , my exam center is new YGH, Yangon, Myanmar
I started with St 2
St 2 wt gain fatigue , amenorrhea, 18 mths post partum
ho of PPH blood 4 units given
Dx hypopit due to shee han
DDx Hypothyroid most probably due to post partum
thyroiditis
I got 18/20
St 3
A middle age lady presenting with SOB
MS with AF
Examiner asked what's your findings, Dx, DDx, how
would you manage, if the pt has vegetation in echo,
?can it affect the INR target
I got 20/20
Neuro station
A young lady presented with difficulty in walking, plz
examine her neurological system
Pt has ryles tube and urinary cathether inserted
Dx- Lt sided hemiplegia due to cardio embolic stroke
I got 20/20
St 4 angry pt with esrd
Scenario given was the pt had history of high blood
pressure since 5 yrs ago which was found out when he
got accident. He didn't take any medication nor any
follow up since then. Now he suffered SOB and saw his
GP , done blood test showing eGFR < 15, Hb 6.5,
Potassium 5.3, USG showing bilateral contracted
.kidney. He is now seeing you what happens to him
Task - discuss his current condition and future
.management plan as appropriate
I got 16/16
St 5 BCC1 a young lady presented with SOB SpO2 88%
dx diffuse systemic sclerosis with pul fibrosis
Examiner ask Dx, DDx for SOB in this pt, Management
I got 26/28
BCC 2
A 54 yr old lady presented with chest pain
Dx Unstable angina(ACS) with hypothoidism
Hidden agenda here was to refer to hormone specialist
to reduce her thyroxine dose
I got 28/28
St 1
Abdominal examination
Hepatomegaly
Dx Thalassaemia
Examiner asked Dx, points for Dx, DDx, infectious
causes of HSM, treatment of Thalassemia
I got 20/20
Respiratory system examination
Rt sided moderate pleural effusion
Examiner asked Dx, points for Dx, DDx of dullness at
base of lung, Ix, Mx, what would you consider if this pt
?is working in shipyard
I got 20/20
Thanks a lot PEC group! I may not get this achievement
.without your help

Great thanks

My experience
I did my paces exam in malta
Mater dei hospital on 2 April
At 9 am
St 4
ys woman admitted 2 days agowih chest infection 80
started antibiotic IV and today became aggressive
refuse treatment and she said doctor and nurse will kill
her diagnosed delirium
Note the pt at time of admission
OK conscious and oriented develop this at hospital
Please took to her son had many concern
And discuss future management
I entered the room
I started same as Dr Zein told us because this the first
time to do delirium
I great the son agreed agenda
?What you know about your mother condition
He tell me what written up
So I told him you are right
He all time disturbed me
Why she developed this
I said I answer your questions
But let me ask you about your mother before is she ok
He said she is completely ok
So I asked is she developed same condition like this
before
He said yes I remembered now 2 year2ago had same
like this but short period and less aggressive I know
from him
Not associated with admittion or treatment or
changing home and resolve spontaneous without
seeking medical advice
So I ask what do you think might be wrong with your
mother
Said I don't know
I asked about concern he what to know why she
developed this
And what treatment
And if she will be OK
What about future
I try to remember all this
Concern
So I said your mother had conditions called in medical
term delirium do your heard about it said no
I will give you leaflet and about it is acute confusion
state
Means disturbed in her brain fuction so change her
way of thinking not knowing time place and person
because of that think that doctor and nurse want to kill
her
And some time the cause can be infection itself
He stop me and said she is OK when came here
I said is she complain of any
Water problem he said no
I said infection of water can come without any illness
and can cause this and her infection of chest and some
people when changing Their place can have this and
also some sort of treatment can cause this I will call my
senior to revise her medication

Is your mother in general ward means with other pt


He said yes
I told first step in tr
Treatment to shift your mother in single room and one
nurse or medical aid who is available (because I don't
know who will sit with her)to be wih her to be sure not
hurt her self
And I will call my senior
And also I will call one doctor in our team we work
closely with him called psychiatric to give you mother
medicine through blood tube to be calm to start again
her medication is that OK
And about becoming ok I am afraid I wouldn't gruntee
about that I am sorry to say that but might become
completely ok or might not but I am sorry to that your
mother had previous problem like this and her age 80
So this also factors(show empathy all through the
station l understand your feeling l know you are
worried about your mother) and right now I am
thinking about dementia do you heard about it here
said yes and he started explain it
I said yes it is aging process
But cannot come suddenly
I will give you leaflet about future who is with your
mother at home he said alone
Where are you said living away for work coming only
weekend I said any possibility to shift with your
mother town or to shift your mother with you he said
no
I asked about financial problems no if your mother
become ok we will talk to social worker and
occupational therapist to do home modifications but if
not we should discuss future career nursing home
He said yes you mean nursing home care I said yes
He started to say oh oh
At this time I told him I know I gave you so many
information so I will give near appointment in this
afternoon with me myself and my consultant to discuss
nursing home care and all you queries and worried he
)said thanks it ok(really I said that to be in safe side

I summarize and check understand


Qs by examiner he ask everything that I told to son
And then he said you said psychiatric some time not at
hospital and pt fighting
I said I will discuss to my senior to give pt haloeridol
What about leave son to sit with his mother at single
room
I said according to hospital policy the examiner laugh
and said you are doctor who putting policy I said if
possible better to stay with his mother familiar face
can help her I got 15/16

My exam experience in chennai today 3/4/17


Station 2
History taking of cough and shortness of breath for 6
months. She has fever on and off but no night sweating
weight loss of 6 kg within 6
Months. No wheezing she works in printing company
and her colleagues have also cough which she is
thinking due to printing materials they use all
cardiovascular history is negative. No history of TB
contact no HIV risk factors. she receive many
.antibiotics without improvement
I put differential of TB and lymphoma and asthma.
During childhood she has asthma which improved. Not
smoker or drinker. But unfortunately the diagnosis was
.extrinsic allergies alveolitis. I miss birds at home
Station 3
.Mitral stenosis with AF it was clear
Neuro
Is parkinson plus CVA. Rigidity only on distraction so I
put parkinsonism plus CVA reflexes was exaggerated
.on the right upper limb only no tremor
Stations 4
.Easy case explain renal biopsy for SLE patient
Station 5
Difficult young on phenytoin developed seizures
yesterday after history of vomiting once and loose
.bowel four times
.I don't know the case
Second case history of bilateral knee pain with stiffness
less than 10 minutes. It was osteoarthritis no other
significant history of skin rashes or other joint problem
.except back pain occasionally
Stations one
Polycystic kidney and clear function fistula on dialysis.
Polycystic kidney is common in chennai take care it
..mimics hepatosplenomegaly
.Chest I don't know it
pray for all candidates in chennai please
i hv passed paces this diet..i hv already shared my
experience before..i just want to share my notes i hv
prepared by myself.. i prepared all that notes by help
of books, Dr. Ayesha audios and most importantly by
guidance of Dr. Rashmi kushal ( Owner of paces ahead
and examiner of paces exam ) during attending the
course... all of these regarding preasentation of st 1
and 3
*******************
hopefully these will help u
*******************
here all imp topics for abdomen station and their
presentation
its My own notes about presentation of imp topics ....
of abd..hopefully al these will help u taking
****** ...preparation...best wishes
ADPKD
After examining this gentleman my impression is he
has ADPKD as evidenced by

Bilateral flank masses which are ballotable


it is possible to to palpate above the masses and
.The overlying percussion note is resonant
However presence of plethoric face indicating
polycythemia and rt sided nephrectomy scar is also
strongly suggestive of my dx
There is no sign of fluid overload, uremia &
encephalopathy
There is no evidence of active renal replacement
.therapy
Id like to complete my examination by checking
BP
Examining
Cardiovascular
neurological system &
Doing
& Fundoscopy
Bedside urine
..take history about
Abd pain
recurrent urinary infection
High colored urine
Headache
Chest pain/palpitation
I ll request for
Urine microscopy
Renal function test creatinine, urea,electrolyte
USG of whole abdomen
ECG and Echo
MRA or DSA

:Liver transplant
After examining this gentleman my impression is he
has Cirrhosis of liver with liver transplant functionally
active as evidenced by

.A Mercedes benz scar on upper Abdomen


There is another scar on rt iliac fossa with underlying [
smooth mass represents concurrently transplanted
]renal allograft
There are scar marks associated with postoperative
intensive care,such as tracheotomy, central venous line
.for vascular access,and abdominal drains
Functional status) the transplant is functionally active (
at the moment Given there is no sign of fluid overload,
portal HTN, coagulopathy & no sign of hepatic
encephalopathy
There is no sign of transplant dysfunction, rejection or
.recurrence of ds
complication of immunosuppression) presence of skin (
thinning with purpura and cushingoid appearance
indicates steroid therapy/ presence of fine tremor &
dm (tacrolimus)/ presence of hirsutism,gum
hypertrophy, coarse tremor dm htn indicates
.cyclosporine therapy
However Presence of some residual stigmata of CLD
like finger clubbing, leukonychia, palmar erythema,
multiple spider naevi on trunk and back ,
gynecomastia, loss of body hair indicates previous
.chronic liver disease
Furthermore there are some pin prick marks on finger
tips,diabetic dermopathy, increased BMI is suggestive
.of NASH as underlying aetiology of cirrhosis
Presence of Dupuytren contracture and parotid
enlargement are suggestive of Alcohol intake as
.underlying aetiology of cirrhosis
Presence of tattoo and needle marks (i/v drug use)
photosensitive rash -porphyria cutanea tarda(hep c)
palpable purpura and livido
reticularis(Cryoglobulinemia) are suggestive of chronic
hep b and c infection as underlying aetiology of
.cirrhosis
Presence of excoriation mark, xanthelasma,tendon
xanthoma, and hyperpigmentation are suggestive of
.PBC as underlying aetiology of cirrhosis
Presence of bronze pigmentation,
arthropathy,fingertip skin prick suggestive of
.haemochromatosis as underlying aetiology of cirrhosis
Presence of KF ring, abnormal involuntary movements
suggestive of wilson's ds as underlying aetiology of
.cirrhosis

Id like to complete my examination by checking


BP
doing
& Bedside urine
Fundoscopy
looking the observation chart
..take history about
Alcohol intake
Hep infection ,jaundice, i/v drug use or blood
tranfusion
pruritus
Joint pain and pigmentation of skin
Involuntary movements
Drugs
I ll request for
Liver function test, including PT
USG of W/A
Ciclosporin and tacrolimus blood level

Renal Transplant
After examining this gentleman my impression is he
has ESRD with Renal Transplant which appears to be
functioning well as evidenced by

Current mode of RRT) A scar in the rt iliac fossa which (


overlies a smooth firm and nontender mass with dull
.percussion note over it
Previous mode of RRT)There are scar marks on the (
chest wall for previous sites of vascular access for
hemodialysis. And also there is a non active non
.functioning arteriovenous fistula In left forearm
Functional status) the transplant is functionally active (
at the moment Given there is no sign of fluid overload ,
no stigmata of uremia and absence of other signs of
.active renal replacement therapy
complication of immunosuppression) presence of skin (
thinning with purpura and cushingoid appearance
indicates steroid therapy/ presence of fine tremor &
dm (tacrolimus)/ presence of hirsutism,gum
hypertrophy, coarse tremor dm htn indicates
.cyclosporine therapy
However there are some pin prick marks on finger
tips,lipodystrophy,diabetic retinopathy is suggestive of
DM as underlying aetiology
Palpable kidney / nephrectomy scar indicating ADPKD
Hearing aid- Alports
Adenoma sebaceum-tuberous sclerosis
Skin rash joint prob-vasculitis
Lipoatrophy-membranoproliferative gn
Id like to complete my examination by checking
BP
doing
& Bedside urine
Fundoscopy
looking the observation chart
..take history about
Abd pain
recurrent urinary infection
Headache
Drugs
Hep infection,joint prob skin rashs
I ll request for
Renal function test, creatinine urea,electrolyte
Ciclosporin and tacrolimus blood level

:Ascites
After examining this gentleman my impression is he
has ascites as evidenced by

Presence of some stigmata of CLD that includes finger


clubbing, leukonychia, palmar erythema, multiple
spider naevi on trunk and back there is gynecomastia,
loss of body hair & multiple purpura & ecchymosis as
.consequence of coagulopathy
distended abdomen with fullness of flanks .it is soft
. and non tender
The abd is dull to percussion on both flanks with
.shifting dullness positive
.Fluid thrill couldn't be demonstrated
However I couldn't appreciate any organomegaly &
.lymphadenopathy
There is no sign of fluid overload , no sign of CLD or
.hepatic encephalopathy
Id like to complete my examination by checking
BP
cvs
doing
Bedside urine
looking the observation chart
..take history about
Alcohol
jaundice
Altered bowel habit
Weight loss
Abd pain
I ll request for
CBC
Liver function test, including PT
Diagnostic paracentesis
USG of W/A

:CLD
After examining this gentleman my impression is he
has cirrhosis of liver with portal HTN as evidenced by
Presence of some stigmata of CLD that includes finger
clubbing, leukonychia, palmar erythema, multiple
spider naevi on trunk and back there is gynecomastia,
loss of body hair & multiple purpura & ecchymosis as
.consequence of coagulopathy
On exam of abd the is distended with fullness of flanks
& some visible dilated veins around umbilicus i
the spleen is enlarged 6-8 cm from left ant axillary line
..towards rt iliac fossa.splenic notch is palpable
Percussion note is over the splenic area with shifting
dullness positive indicating indicating portal HTN
There is no sign of fluid overload , no sign of hepatic
.encephalopathy

However there are some pin prick marks on finger


tips,diabetic dermopathy, increased BMI is suggestive
.of NASH as underlying aetiology of cirrhosis
Presence of Dupuytren contracture and parotid
enlargement are suggestive of Alcohol intake as
.underlying aetiology of cirrhosis
Presence of tattoo and needle marks (i/v drug use)
photosensitive rash -porphyria cutanea tarda(hep c)
palpable purpura and livido
reticularis(Cryoglobulinemia) are suggestive of chronic
hep b and c infection as underlying aetiology of
.cirrhosis
Presence of excoriation mark, xanthelasma,tendon
xanthoma, and hyperpigmentation are suggestive of
.PBC as underlying aetiology of cirrhosis
Presence of bronze pigmentation,
arthropathy,fingertip skin prick suggestive of
.haemochromatosis as underlying aetiology of cirrhosis
Presence of KF ring, abnormal involuntery movements
suggestive of wilsons ds as underlying aetiology of
.cirrhosis
Id like to complete my examination by checking
BP
doing
& Bedside urine
Fundoscopy
looking the observation chart
..take history about
Alcohol intake
i/v drug or blood tranfusion
Sleep cycle
pruritus
Joint pain and pigmentation of skin
Involuntery movements
I ll request for
Liver function test, including alt ast alp PT,serum alb
Usg of W/A
Upeer GI endoscopy

:Hepatomegaly
Id like to complete my examination by checking
BP
Ext genitalia
Hernial orifices
doing
& Bedside urine
looking the observation chart

...Examination of this gentleman revealed

He is cachexic and icteric.there is evidence of anemia


.as well
On exam of Abd there is hepatomegaly Which is 4 cm
from rt costal margin at midclavicular line,tender, firm
to hard in consistency, with sharp margin & nodular
surface,upper border of dullness at 6th ICS with
.hepatic bruit
.No other organomegaly or ascites is there
There is no sign of encephalopathy or coagulopathy to
.suggest hep failure
He is euvolemic
So my dx is he has hepatomegaly
However presence of cachexia,anemia and jaundice
.strongly suggests towards malignancy as most likely cz
:Splenomegaly
Id like to complete my examination by checking.1
BP
Ext genitalia
Hernial orifices
doing
& Bedside urine
looking the observation chart

Examination of this gentleman revealed

Presence of some stigmata of CLD that includes finger


clubbing, leukonychia, palmar erythema, multiple
spider naevi on trunk and back there is gynecomastia,
loss of body hair & multiple purpura & ecchymosis as
.consequence of coagulopathy
On exam of abd the spleen is enlarged 6-8 cm from left
ant axillary line towards rt iliac fossa.splenic notch is
..palpable
Percussion note is over the splenic area with shifting
dullness positive indicating indicating portal HTN
There is no sign of fluid overload , no sign of hepatic
.encephalopathy
He is cachexic and icteric.there is evidence of anemia
.as well
However presence of Hepatosplenomegaly with
cachexia ,anemia, and lymphadenopathy suggests
towards Lymphoproliferative disorder as most
possible dx

Examination of this gentleman revealed .2


. He is anemic
There are widespread lymphadenopathy involving
.cervical and axillary groups
There are also some excoriation marks suggesting
.pruritus
On exm of abd there is hepatosplenomegaly with the
liver edge palpable 3 cm below the rt costal margin at
midclavicular line, non tender,soft to firm in
consistency, with sharp margin & smooth
surface,upper border of dullness at 6th ICS without any
hepatic bruit and the splenic edge 4 cm from left ant
axillary line towards rt iliac fossa.splenic notch is
palpable with dull Percussion note over the splenic
area
There is no Ascites or peripheral edema
However presence of Hepatosplenomegaly with
cachexia ,anemia, and lymphadenopathy suggests
towards Lymphoproliferative disorder as most
.possible dx

Id like to complete my examination by checking


BP
Ext genitalia
Hernial orifices
And inguinal LN
doing
& Bedside urine
looking the observation chart including temp chart
Taking history about
Fluctuating fever
wet loss
night sweat
Pruritus
Id like to request for
CBC with ESR
USG of W/A
LFTs
PBF
Lymph node biopsy
Immunophenotyping
Examination of this gentleman revealed .3
. He is anemic
There are a symmetrical deforming arthropathy
affecting the small joints of both hands with ulnar
.deviation and nodules in forearms
On exam of abd there is splenomegaly with the splenic
edge 4 cm from left ant axillary line towards rt iliac
fossa.splenic notch is palpable with dull Percussion
note over the splenic area
There is no Ascites or peripheral edema or sign of
.hepatic encephalopathy
However presence of rheumatoid hand deformity with
rheumatoid nodules and splenomegaly makes Feltys
.Syndrome most likely Dx

Id like to complete my examination by checking


BP
Complete Rheumatological exam
doing
& Bedside urine
looking the observation chart
Taking history
Recurrent infection
Weakness
Joint problems
Id like to request for
CBC with looking for neutropenia to confirm my dx
*****************
History: Intro
Hi there... My name is Dr. Imam. I m the a specialist
. register working in this clinic today
Can i start by checking ur detail? U r Mr. Paul
Coling?and u r 62 yrs old..which one u would u prefer
.me to call u?mr. Coling or paul
So paul I m here to hv a brief discussion about the
helth problem u r experiencing recently .well also
discus the management plan towards the end on of the
?session.is that alright
So ive come to know u hv had fever ... wd u like to tell
me a lot about what has been going on & why u r here
.today / what made u come here & see me today
Right...i do appreciate your concern.we'll discuss all urs
symptoms one by one..however there is no need to b
alarmed at this stage...we need to take detail history
and do further testing to eliminate certain dx like
meningitis.(should there be any concern regarding
meningitis diagnosis we have excellent treatment plans
that we will implement to get u completely cured). u r
assured that ull be receiving effective treatment for
any shorts of diagnosis.all our doctors here will make
.sure u feel comfortable.we'll be doing our best
however..id like to go through the history again
.clarifying some details
What i understand u hv had fever for some couple of
days. And u r so concern whether it is something
..serious like meningitis or not
Is there anything else that u wanna address today in ur
clinical appointment? Okay
Now i m coming to ur sympoms

History : H/O present illness

I just wanna go through the history again clarifying


.some detail
What do u mean by funny turn or
.And about the cough can u describe it for me
?How long the cough has been there for
?Has it started suddenly or gradually
?IS it with u all the time or comes on and off
?Is it getting better or worse than before
?Is it there any particular time of the day or night
How about on a scale 1 to 10 is that cough to u 10
?being worst possible
?Does it keep u off work
?Does it wake u up at night
?What makes it worse
?Does anything make it better
Hv u had a pain/ cough like this before?what it exactly
.like this one?what did u do then
Hv u seen ur doc or done any tests for that?taken any
?medication?did it helped
Hv u got any other symptoms like...is that been there
?for same amount of time

:History
D/D: Im going ask u some more specific questions
.now
:Systemic Inquiry
I just want to go through any keys changes of ur body
.system, just to find out about ur overall health
Can u tell me, hv u had any unexplained weight
?loss?unusual weight gain
?Is there any fever or any shivering
hv u noticed any lumps or bumps
anywhere?where?how did u notice it?any other lumps
?and bumps elsewhere in body

Can u tell me do u hv a headache?hv u seen ur doc for


?the headache
?Any problem with memory or concentration
Hv u ever hd weakness or unusual sensation of any
?part of your body?any walking difficulty

Ok so how about ur vision?Did u experience Watery


eyes? Do u think u had any redness of eyes?so do u
?wear glasses

What about ur nasal passage Do u hv nasal blockage?


Any sinus problems? nothing like that? Ok
.Any issues of the chest
any pain in ur chest-
do u hv a cough-
?difficulties in breathing-
?do u ever feel ur heart is racing or thumping-
?ok so can u remember if Hv u ever had blackouts-
?are ur feets swollen-
.And about ur tummy
?is there any tummy pain at all -
?difficulty in swallowing food or water-
.What about ur taste?do u enjoy ur meal-
?do u feel sick? Do u throw up-
hv u ever noticed yellow discoloration of eyes or -
?urine
ok may sound awkward but have to ask u about ur -
?bowels, working alright? Is there any recent alteration
any unusual bleeding from any of ur orifices??great-
what about ur water works?are u passing more /less -
water than u used to? high colored urine or frothy
.urine?excellent

Now tell me about ur joints movements, do they move


freely?do u feel any joint pain
?or stiffness
?ever felt difficulties in combing hair/climbing hair-
hv u had mouth ulcers-
what about ur skin?any recent color changes or -
..rashes or nail changes?great
Do u hv preference of hot or cold? which one do u
?prefer
:Medical Hx
.I want to talk about your health issues
Hv u had any long term illness?like
?High BP
?so u hv high BP?ok...when was it dxed-
?is it well controlled-
when was it last checked?everything was -
.great?perfect
Any other long term illness?high blood sugar?u
?do?alright
?when was it dxed -
do u know what type u hv?(one with tabs or insulin) -
.ok so u hv ur tabs
so when hv u had ur last check up?were they -
.happy?ok
?Lipid prob or heart prob which one do u hv
?Hv u had any gland prob or hormonal prob
:Family Hx
Do any of ur family members ur mom, dad or siblings,
has any of them had any long term illness? Had anyone
had cancer?perfect

:Surgical Hx
Now I'd like to review your surgical hostory.hv u had
any operation in ur past?Alright.could ur recall?Any
?blood transfusion

:Hospitalization
?Ever been Hospitalized before

:Drug Hx
.Lets talk about your medication
Do u take any prescribed medications?i know u hv high
BP and high BS.what r the medications u r on right
.now
R u taking all of these medications regularly?excellent
?Any side effect of them
.Any recent changes in dose of medications
Any over the counter medications or herbal
medication hv been taken? Great
I hope u wont mind me asking, hv u ever taken any
?recreational drugs
?Hv u got any drug allergies
:Allergy Hx
Allergic to anything other things like dust, any food like
nuts or any other env agents at all?brilliant
:Travelling Hx
?Any recent travels abroad
?Where?for how long did u stay there
?What was the reason for your travel
?How frequently were u required to travel
?Was it a rural or urban area?where did u stay
?From where did u manage food and drinking water
?Did u swim in local pool
?Did u hv any insect bite at that area
Was there any endemic ds prevailed at that time?Did u
?taken ur anti malarial medications before travelling
hope u wouldnt mind me asking u some personal
?ques?hv u had sex with local girls

:Sexual Hx
Would u mind me asking u some personal questions
?Are u in a relationship
?May i know is it a male or female
?How long have you been living together
?Do u use protection
?Have u or partner ever beee. tested for HIV
Any other relationship or casual sex apart from ur
?regular partner

:Social History
Soooo if u don't mind i just want to ask u few personal
ques.and a bit about ur social background is that
?alright
Do u smoke at all?can i ask u what do u smoke?how
many ciggs u go through a day.and for how long you
have been a smoker?alright.hv u ever thought about
giving up?u dont want to consider that at the
moment.when u are ready, we r always here for u. we
hv got support strategies to help you. so whenever u
feel ud like to talk about quitting smoking we are here,
.just to let u know
Going to talk about alcohol intake....do u drink?what
do u normally drink.ok.and how much wine do u drink
?a day
Tell me about ur diet.wonderfull.do u exercise?If u find
some time it wd b really relaxing and it suppose to help
.u.its good for ur blood sugar as well
Can i ask u about ur mood.how are u feeling today?no
low mood?no sleep prob?everything good?excellent
What about ur occupation.what do u do for living?hv u
ever been exposed heavily to dust or any other
substances at ur work? Coping well?u happy with ur
.work?no stress?everything good?perfect
U said u hv GF.u live with her?/ who do u live with at
.home?right and u r living happily?no stress?excellent
Do u live in a house or flat?how many flights of stairs
?do u need to climb
.Any pets at home?alright

Let me sum up all this info if u dont mind and plz


.correct me if i got anything wrong
Hv i missed anything imp?wd u like to add anything
?else

History: discussion

D/D: From what you have told me today, there could


be a few possible reasons for the backache that u have
...been experiencing...including
However ,A thorough examination is required along
with some further tests to reach our dx and to make
sure that there is no serious issues/something else is
not going on.well do it as outpatient. U do not need to
.be admitted to hospital

PLAN: today I will examine you and well send off some
blood tests before u leave the clinic and also to do a
xray to hv a look at ur back bones.../ull get a call
letting u know the next date of appointment/ date for
the telescope test...and then well arrange to see u
back in clinic with the results...depending on the test
results well b able to advise on treatment at our clinic
or by referring u to a respective specialist..is that ok
?with u?is there anything ud like to ask me

Check:Hv we covered everything that u r concerned


about? Are there any other problems that uhv been
.having that we hvnt covered today

Further plan:if u hv any problem in the meantime just


give a call to the clinic they'll b able to notify one of our
.doctors to discuss any issues with you
.Goodbye. See u soon with the results
Hi all 3rd carousel blue selayang 17.4 2017
Respi lung fibrosis sec to scleroderma
Abdo ballotable both kidney, mild splenomegaly, a bit
jaundice and pale
Neuro bilateral ptosis MG
Cardio DVR
Hx talking pt with t1dm presented with weight loss and
recurrent hypo. Also got dyspesis symptom
Blood test egfr 34,hb10,hba1c a bit high
Comm skill breaking bad news pt cxr have mass,
counsel for bronch, ct thorax etc
Bcc1 pt ihd dm got unilateral ptosis and double vision 3
days
Bcc2 dermatomyositis

My experience of PACES 01.04.2017 MALTA


I started with station 2 : 24yrs old lady c/o fatigue and
headaches found to have high BP 160/98 on different
occasions. No family hx , no
complications.....examiners asked would you start
treatment...i said not now if secondary causes r/o
including white coat htn then yes. Discussion about
.secondary causes of htn and investigation
Got 20/20
Station 3: cardio VSD/TR pansystolic murmur in
tricuspid area. Discussion about investigations and
.management
Got 16/20
Neuro; middle age with walker reflexes with re
inforcement rigid limbs throughout....planters
equivocal no cerebellar signs ...out of time could not
check sensory system....made dx of spastic
paraparesis....asked of causes got 11/20
Station 4: talk to the wife of 50years man known MND
recurrent admissions for pneumonia. This time
admitted with aspiration pneumonia not responding to
anti biotics and confused. Discuss with her his
management and condition. I explained to her that he
is in advance stage....and any invasive procedure like
intubation is futile. We will give him palliative
treatment only and keep him comfortable. Examiner
told that i did not told about NIV when the wife asked.
Got 15/16
Station 5: first case young lady with joint pain...i was
happy will be from common causes. When i entered
took hx all joints pain including hand joints. Surrogate
told she was told by GP that she has double joints. I
was more confused...asked what u mean double joint
she said i dont know gp told me that and he was not
sure.i examined no sign of inflammation. Raised skin
dorsum of hand elastic . Checked functions. Found
high arched palate. Joints hyperextensible.arm lenght
normal. Gave diff of marfans pseudoxanthoma
elasticum forgot to tell ehlers danlos syndrome.
Examiner asked were there marfanoid features i said
no. I wanted to examine chest they stopped me asked
what u want to see i said AR....they said normal. Asked
how you investigate joints in general...started with
inflam marker RA factor ACCP...time finished got 28/28

nd cast st.5: believe me i totally forgot....got 23/282

Station 1: abdomen; lady in 50 has pruritis and abd


pain.....i neglected pruritis....no signs of CLD or
immuno suppression...distended abdomen tender all
over mid line laprotomy scar below umbilicus. Shifting
dullness+ could not appreciate any organomegaly or
mass due to tenderness. Differential ...causes of ascites
asked about causes of transudative ascitic fluid....got
8/20

Respiratory: c/o of Sob pneumonoctomy scar left side


trachea deviated to left no breath sounds left
......asked about investigation ....told so many still
asking was happy when told sputum analysis and c/s
and AFB ....infection as cause of Sob
Got 20/20
Wish success for all you going for PACES......my advice
is to perform during exam in relaxed way, smile and
forget about any station where you did not perform
.well
My experience
PACES 1/2017 Yangon center
Started with station 4
Delayed Dx of phaeochromocytoma
Examiner asked acceptable delay or not
She wanted the following reasons
1. Different presentations of the disease
2. Episodic in nature
3. Rare disease
I explained surrogate only no 1
14/16
Station 5 BCC 1
Common case Graves' disease presented with
palpitations and grittiness of the eyes
BCC 2
Pt at rheumato clinic for knee pain , UL hypertension
presented with reduced urine output
Higher diclo dose by herself
also taking ACEi
I couldnt organize the case very well
Gave dx of AKI due to NSAID
Forgot to tell abt Interstitial nephritis
25/28
Staion 1
Resp Collaspe with effusion Rt
Ddx TB vs Malignancy
Abd Polycystic kidneys with no evidence of RRT
Q. Ddx of abd pain in PCKD
Infection , cyst haemorrhage and also Traumatic
rupture
Station 2
Yg lady with IBS presented with worsening pain and
diarrhoea
not respond to antispasmodics
Stress factors (+)
Alarm features absent except Noctural diarrhoea
So gave DDX Coeliac . Crohn s . Other malabsorptive
disorder . Infection. Endocrine
Invx Examiner ask abt inflammatory vs
noninflammatory
how to exclude ddx infection
Station3
Neoro
Yg man with bilateral foot drop and minimal sensory
disturbance
Motor dominant PN
Ddx GB syndrome
Forgot to do LP
19/20
CVS
MR AR
No peripheral sign of AR except
Collalpsing pulse
Possible cause of AR in Middle age man
What do u want to look for in Echo
Examiner wanted LV size assessment
Passed with a total score of 166/172
I think practice and patient centered approach is very
important.
Hope my experience would be some helpful tips for u.
Thank u all who ve shared their experiences which
indeed helped me during preparartion for PACES.

Myanmar. PACES 1/2017


I started with St1.
St1 Resp: COPD with bronchiectasis
Examiner Q: DDx. Investigations. 18/20
Abd: Massive hepatosplenomegaly with
anaemia and tinge jaundice (Haemolytic
anaemia - Thalassaemia, Thalassaemic face+)
Examiner Q: Complications of Thalassaemia,
Findings in blood film, Managemant 20/20
St2 young hypertension - conn's $
Examiner Q: DDx, Investigations for Each
Differential 18/20
GP refer the Young female patient with
hypertension for furthur invetigations and
managemnt. Serum electrolytes, urea, creatine
all normal. Urine REME shows proteinuria and
hematuria.
The only symptom patient has is fatigue,
headache, h/o of fever (?viral illness) few weeks
ago.
No urinary symptoms like frothy urine,
hematuria, or color changes, difficulty in
urination, frequency etc. no leg oedema, no
puffy face. So I asked how about in the past, still
no.
As she has headache, I excluded
phaeochromocytoma. No palpitation and
sweating.
For APKD, no family history, not an adopted
child, No abd distension or pain, not notice any
mass, as she has fever asked about symptoms
of UTI which which is absent.
Symptoms of glomerulonephritis and causes -
Nil
No rash. No hepatitis h/o B,C infection. Slight
loss of weight (but Pt dun know exactly how
many kg) but no loss of appetite. No lumps or
bumps anywhere.
Exclde systemic sclerosis, Cushing,
acromegaly, thyroid problems etc.
No past medical history.
Nothing got from system review.
No family history of note.
Drug history -Not on any medication and no
allergy to medication.
So with no significant symptoms, I gave Renal
artery stenosis and Conn first.
Glomerulonephritis (Because of asymptomatic
proteinuria and hematuria, I gave it second
defferential). Vasculitis, Takayasu one of the
possibilities as she has headache.
Examiner discussed mainly about Conn like
how will u diagnose Conn. I answered I'll chek
renin which will be low and aldosterone will be
high. K can be normal in >20% (As the senario
gave normal potassium).
And then Differentials n Investigations related to
DDx.
St3 Cardiac: Combined AS and AR
Examiner Q: Which one is dominant? Finding in
Echo, Severity, Causes of AS and AR in this
patient, Surgical role in this patient 20/20
Neuro: Flaccid paraparesis with Motor and
sensory involvement
Examiner Q: DDx, Causes, Management 20/20
St4 80yrs old male with right sideded weakness.
CT - cerebral infarct. INR 1.2. Past h/o of
recurrent TIA and AF. Missed follow up for AF.
Task - Tell the CT result, management plan,
admission to stroke unit and answer the pt
concern. (You are doctor at emergency
depeartment) 13/16
St5 BCC1 Upper GI bleeding d/t NSAID (pt has
back pain and on examinatin signs of
Ankylosing spondylitis +). He is also alcoholic.
Abd exam: mild hepatomegaly. Also gave
alcoholic hepatitis as differential. 22/28
BCC2 peripheral sensory and moter neuropathy
with DM and its complications, Taking
metformin and currently on treatment for TB
(HRZE). Patient is vegetarian ( I missed to asked
about it ) 20/28
Passed with total score of 151/172. Hope my
experience helps u all.
And also thankful for this group as it is so much
helpful while preparing my PACES

Thanku to every member of this Group


,AulhumduliAllah ,Allah has passed my Paces MRCP
(UK) exam in first attempt. I learnt alot from every
people here really.
I want to share my experience which I took in
Birmingham Sandwell General Hospital.
Abdomen; PCKD .
Respiratory; rt pneumonectomy
CVS; MVR with AF.
Neuro; Charcot marrie tooth Disease
Hx; young lady with peristent reg. palpitations ( DDX
clinically hyperthyroidism, sinus tachycardia, AF,
SVT, Functional ).
CCS; Ptn with PE ,counselling him regarding start of
warfarin. Addressing his concerns huge list.
BCC 1 Obese young male with acanthosis nigrican in
arm pits having uncontrolled BP .He was non
compliant to Rx.(DDX rule out pri and sec cause of
Htn most probably Cushing synd).
BCC 2 Young ptn with CD having colostomy bag in
RIF Having chest pain (DDX ACS, MSK pain,
PE,GERD).

.
My experience
I did my paces exam in malta
Mater dei hospital on 2 April
At 9 am
St 4
ys woman admitted 2 days agowih chest infection 80
started antibiotic IV and today became aggressive
refuse treatment and she said doctor and nurse will kill
her diagnosed delirium
Note the pt at time of admission
OK conscious and oriented develop this at hospital
Please took to her son had many concern
And discuss future management
I entered the room
I started same as Dr Zein told us because this the first
time to do delirium
I great the son agreed agenda
?What you know about your mother condition
He tell me what written up
So I told him you are right
He all time disturbed me
Why she developed this
I said I answer your questions
But let me ask you about your mother before is she ok
He said she is completely ok
So I asked is she developed same condition like this
before
He said yes I remembered now 2 year2ago had same
like this but short period and less aggressive I know
from him
Not associated with admittion or treatment or
changing home and resolve spontaneous without
seeking medical advice
So I ask what do you think might be wrong with your
mother
Said I don't know
I asked about concern he what to know why she
developed this
And what treatment
And if she will be OK
What about future
I try to remember all this
Concern
So I said your mother had conditions called in medical
term delirium do your heard about it said no
I will give you leaflet and about it is acute confusion
state
Means disturbed in her brain fuction so change her
way of thinking not knowing time place and person
because of that think that doctor and nurse want to kill
her
And some time the cause can be infection itself
He stop me and said she is OK when came here
I said is she complain of any
Water problem he said no
I said infection of water can come without any illness
and can cause this and her infection of chest and some
people when changing Their place can have this and
also some sort of treatment can cause this I will call my
senior to revise her medication

Is your mother in general ward means with other pt


He said yes
I told first step in tr
Treatment to shift your mother in single room and one
nurse or medical aid who is available (because I don't
know who will sit with her)to be wih her to be sure not
hurt her self
And I will call my senior
And also I will call one doctor in our team we work
closely with him called psychiatric to give you mother
medicine through blood tube to be calm to start again
her medication is that OK
And about becoming ok I am afraid I wouldn't gruntee
about that I am sorry to say that but might become
completely ok or might not but I am sorry to that your
mother had previous problem like this and her age 80
So this also factors(show empathy all through the
station l understand your feeling l know you are
worried about your mother) and right now I am
thinking about dementia do you heard about it here
said yes and he started explain it
I said yes it is aging process
But cannot come suddenly
I will give you leaflet about future who is with your
mother at home he said alone
Where are you said living away for work coming only
weekend I said any possibility to shift with your
mother town or to shift your mother with you he said
no
I asked about financial problems no if your mother
become ok we will talk to social worker and
occupational therapist to do home modifications but if
not we should discuss future career nursing home
He said yes you mean nursing home care I said yes
He started to say oh oh
At this time I told him I know I gave you so many
information so I will give near appointment in this
afternoon with me myself and my consultant to discuss
nursing home care and all you queries and worried he
)said thanks it ok(really I said that to be in safe side

I summarize and check understand


Qs by examiner he ask everything that I told to son
And then he said you said psychiatric some time not at
hospital and pt fighting
I said I will discuss to my senior to give pt haloeridol
What about leave son to sit with his mother at single
room
I said according to hospital policy the examiner laugh
and said you are doctor who putting policy I said if
possible better to stay with his mother familiar face
can help her I got 15/16
Treatment to shift your mother in single room and one
nurse or medical aid who is available (because I don't
know who will sit with her)to be wih her to be sure not
hurt her self
And I will call my senior
And also I will call one doctor in our team we work
closely with him called psychiatric to give you mother
medicine through blood tube to be calm to start again
her medication is that OK
And about becoming ok I am afraid I wouldn't gruntee
about that I am sorry to say that but might become
completely ok or might not but I am sorry to that your
mother had previous problem like this and her age 80
So this also factors(show empathy all through the
station l understand your feeling l know you are
worried about your mother) and right now I am
thinking about dementia do you heard about it here
said yes and he started explain it
I said yes it is aging process
But cannot come suddenly
I will give you leaflet about future who is with your
mother at home he said alone
Where are you said living away for work coming only
weekend I said any possibility to shift with your
mother town or to shift your mother with you he said
no
I asked about financial problems no if your mother
become ok we will talk to social worker and
occupational therapist to do home modifications but if
not we should discuss future career nursing home
He said yes you mean nursing home care I said yes
He started to say oh oh
At this time I told him I know I gave you so many
information so I will give near appointment in this
afternoon with me myself and my consultant to discuss
nursing home care and all you queries and worried he
)said thanks it ok(really I said that to be in safe side
I summarize and check understand
Qs by examiner he ask everything that I told to son
And then he said you said psychiatric some time not at
hospital and pt fighting
I said I will discuss to my senior to give pt haloeridol
What about leave son to sit with his mother at single
room
I said according to hospital policy the examiner laugh
and said you are doctor who putting policy I said if
possible better to stay with his mother familiar face
can help her I got 15/16

!!!Hi guys
I recently passed my MRCP PACES exam, thanks for
God, and Id like to share my experience of this exam
.with you

My duration of the preparation was about 5 months,


on average of 2 to 6 hours of study per day. I took
about one to two days off every week. I read few
books, subscribed to online pastest videos and
.studying materials and attended three courses
Regarding books, my personal advice is as the
:following

Station 1&3, read it from Cases For Paces, with back


up support from Manual of MRCP PACES (by pastest)
and from oxford speciality training, along with revising
.the videos on pastest website
Station 5, read it from Manual of MRCP PACES, with
back up support from oxford speciality training. Do not
forget to revise station 5 videos on the pastest
.website. They are really useful
Station 2, mainly from Ryder plus revising differential
diagnosis from oxford speciality training and videos on
pastest website. In addition, it is a good idea to have a
.look at station 2 samples on the MRCPUK website
Station 4, mainly from Cases For Paces. The ethics
and the communication principles are written in a very
nice way in this book. The other thing to do to prepare
for this station is to practice station 4 scenarios
available on the MRCPUK website along with the
.videos on pastest website

In addition to reading books, attending courses and


practicing with a study partner is a must. I did practice
with a colleague in the last one month and a half
before the exam. Also, it is a very good idea to
dedicate about one hour a day in the last month before
the exam to practice the examination techniques
either on your friend or on your partner. By doing this,
in the exam you will look professional, confident,
.thorough and sleek in your examination skills

The other useful thing that you can do is to download


the PACES marking sheet from the MRCPUK website
and to try to fulfil all the criteria required during your
.revision and your practice

:The courses I did, which I found worthy and useful, are


Pastest course: 4-day course in London
PACES4U course: the first day was station 4 and 5
.teaching, and the second day was a mock exam
Ealing course: 2-day intensive course in London

As far as station 1 and 3 are concerned, there is an


important tip to follow. Sometimes in PACES exam
spotting the diagnosis is not enough to pass. In other
words, it is not sufficient at the level of MRCP
candidate as a fifth year medical student can spot the
diagnosis as well. I am going to give you an example
just to make it a bit clearer. If in the respiratory station
you find a case of pulmonary fibrosis, it is not enough
to say to the examiner this is a case of pulmonary
fibrosis. You have to show to the examiner that you
look for the underlying cause, for instance, any signs of
connective tissue disease in the hands. Also you need
to look for any possible complication that you can pick
up by the clinical examination, such as signs of
pulmonary hypertension (loud P2 and right parasternal
heave) and signs of cor pulmonale (raised JVP and
pedal oedema). Thus, when you present your findings
to the examiner, you mention the positive findings
which lead you to the diagnosis of pulmonary fibrosis
and to mention the important negatives. So, you can
say this is a case of idiopathic pulmonary fibrosis.
There are no signs of pulmonary hypertension or cor
.pulmonale

I hope that this information is useful and good luck for


.you all

I had my exam there cases were very straightforward


station 5 diabetic retinopathy he had all the
complication of diabetes like peripheral neuropathy
ischemic heart disease etc so I ran out of time a
bit,bcc2 was thyrotoxicosis with a solitary nodule it
was very easy. Abdomen was kidney and pancreas
transplant with lots of scars he was blind of one eye
with vitiligo so diabetes was a reason for the tx.
Respiration was bronchiectasis. History was PUO. Cx I
couldn't understand it was bbn with lung mass but she
didn't know that so I messed it up.CNS was cerebellar
syndrome. I cannot recall CVS good luck

My UK exam exerience, Hull (18 March, 2017)...I


started with station 5, my first case was an old lady
with a skin rash. As soon as I set my eyes on her, I
quickly figured out that she had extensive scaly
maculopapular rash affecting her head, face and trunk.
Hx>>>Long standing for almost 20 years, Worse on
exposure to sunlight, no itching or pain, now getting
worse, drinks alcohol. I examined her elbows and back
of the years as well as examined for arthropathy. Viva
questions were about diagnosis, differential diagnosis
and management. I got 15/28
Next ST5 second case was a lady in her fifties who had
elevated ALT on routine examination and had some
painful periods. Nothing else of note. No findings on
examination. Questions were about diagnosis,
differential and management. I told them that it's
most likely Non-alcoholic fatty liver disease, other
differentials were alcoholic liver disease but I told the
examiner that in that case, I expect AST to be higher
than ALT as well as high GGT. Other differential I said
could be autoimmune hepatitis. She asked what
investigations....? ASMA....I got 14/28
Next I went for respiratory st. An old man, actually
there were no significant findings on auscultation. But
thanks to Dr. Magdy, I was thorough in clinical
examination, therefore I missed the diagnosis but still
got 9/20. It was COPD but don't know how, (perhaps
in exam pressure) I said Interstitial lung disease. My
viva was about causes and managment of ILD.
Next I went to Abdomen. Patient had gum
hypertrophy, Tremors, PD catheter mark and as scar in
Rt. iliac fossa with a nontender underlying mass. My
diagnosis was a kidney transplant. Viva questions
were about what modality of dialysis was he on, I said
PD. Then what meds he could be taking, I said
Cyclosporin because he has tremors and gum
hypertrophy. He asked are these side effects only
because of cyclosporin. I said they are a common side
effects of any calcineurin inhibitors. Then, he asked
me if this pt came with fever and abdominal pain, what
could be the possibilities? I said Infections., UTI etc etc.
He asked me could he be rejecting? I said yes, but in
that case the graft will be tender. How will you
investigate rejection? again, thanks to Dr. Magdy, I said
blood works, including routine CBC, renal profile etc,
special such as cyclosporine levels and imaging Renal
US and possibly a kidney Bx...I got 19/20
Next was History, a man in his 40s, on multiple meds,
had A.Fib on warfarin and a number of other meds
including simvastatin, IHD, c/o difficulty walking.
Initially I thought, it was ?Stroke, but he had difficulty
climbing up stairs as well as coming down, generalized
body pains, shoulders, legs etc. I thought of
Polymyalgia rheumatica but his age was against it. I
thought of proximal myopathy, Cushings,
hypothyroidism but no features to suggest them. I
could not tell a unifying diagnosis, and forgot to stop
his statin. But I told the examiner that he needs
physio, stop statin and investigate for the above
causes. I got 10/20
Next I had neuro: Again messed up. A freightening
case, a lady sitting in a wheelchair with short hands
and lax skin, having right sided weakness of all muscle
groups and depressed reflexes but no sensory
affection. I said MND But when he asked me what's
against MND, I said unilateral signs. He said where is
the lesion, by that time I had realized what mistake I
had made. I said cervical spine...Examiner was looking
at me, then he asked me if I had seen the back of the
......neck, I said no. He asked me to do so. AND
There was a scar at the back of the neck.....Got
10/20 Next cardio,
Here I would advise candidates who think in UK, an old
white man >>>thnink of aortic valve, young female
>>>mitral ....THIS CONCEPT IS WRONG and
Decieving....It was an old white male, but his first heart
sound was metallic with central sternotomy scan. No
murmurs. Thanks to Dr. Magdy for giving us an
excellent opportunity of cardiology practice. Exam
questions were: Diagnosis, MVR. Valve functioning
well? Yes, management, anticoagulation. Indications?
Got 19/20 Last, I
had ST4. Task was to talk to the son of a patient who
was started on Amiodarone for V. Tach and now
admitted with pulmonary fibosis. He was insisting why
it was started if it could harm him. I showed empathy,
told him that I can completely understand his feelings,
and he is feeling so because of his love and care for his
father. Let me tell you that when the drug was started,
it was given to him in his best interest as the other
alternates are not as effective as this drug is. He said
my GP never informed us of the S/E. I said I was not
part of the team when it was started, so don't know
exactly what happened. Will look at the charts and get
back to you. Let's look forward now and see how we
can help your father. I asked his whether his father's
bedroom is on ground floor or he has to climb up the
stairs, who lives with him, how he used to manage his
activities of daily living prior to admission, and offered
him all the social support. I got 16/16
My overall score was 112. Didn't pass this time and I
.think my actual problem was station 5
calcutta , India -
sta1 . res - rt. pl effusion
abd - hepatomegaly with splenectomy
(heamolytic anemia)
sta2 . HB- 9.8 , c/o fatigue ?nsaid induced
sta3 . cvs - MS
cns - spastic paraplegia
sta4 . open TB , wants to go DAMA
sta5 . a) TIA
b) goiter with hyperthyroidism

Calcutta , India -
st1 liver transplant.
Chesy old with ILD in RA
Sts changle bowel habbit
St3 hemonymous hemianopia examine cranial n
Cardio biological valve with mr ,phtn
St5 bbn SAH with INR 4
St 5 takayasu
sta 2 - Hemtatemsis nsaids vs crohns
3rd carousel blue selayang 17.4 2017
Respi lung fibrosis sec to scleroderma
Abdo ballotable both kidney, mild splenomegaly, a bit
jaundice and pale
Neuro bilateral ptosis MG
Cardio DVR
Hx talking pt with t1dm presented with weight loss
and recurrent hypo. Also got dyspesis symptom
Blood test egfr 34,hb10,hba1c a bit high
Comm skill breaking bad news pt cxr have mass,
counsel for bronch, ct thorax etc
Bcc1 pt ihd dm got unilateral ptosis and double vision
3 days
Bcc2 dermatomyositis
Station 2 very tough. I just put pt hypos possibly
1.MEN-insulinoma and gastrinoma
2.advance CKD
3.overdose
Bcc1
Complaint of diplopia 3 and double vision
No sign stroke
DM well controlled.
HPt controlled
Meds antiplt, OHA, antihpt
Possible dx
1.cva
2.mononeurtis multiplex
3.MG
4.TIA
Oman center on 10/4/2017...
Chest case Bronchectesis
Abdomen case Renal transplant
History
Cystic fibrosis
Cardiology MVR
Neurology HSM PN
Communication
Renal biopsy from SLE
Station 5
BCC1 Diarrhea in Rhumatoid and psoriasis
BCC2 Goiter
Sudan- Khartoum-- 9-4-2017-
Station five AF for dd
Station bcc 2scleroderma
Abdomen transplanted kidney
Chest double pathology???fibrosis and effusion
wheezey
CNS proximal weakness
CVS valve replacement
History IBD
Communication counselling regarding warfarin

Edinburgh - centre Glan Clwyd Hospital in Rhyl-


Wales
1.a) Abdo- multiple abdominal scars which
looked like renal transplant scars but no kidney
underneath palpable. AV fistula which was
buzzing and looked like had been recently
needled. Could feel a retro peritoneal mass- said
it might be a polycystic kidney. Also had
parathyroidectomy scar. Was asked for Mx. Said
U&E , Doppler US of abdomen. Then bell rang. I
thought this station was a disaster and would
fail. 16/20
1 b) Resp- 65 yr old man reading newspaper.
Only finding was some crackles R>L and
perhaps a dull left lung base. My DD's were ILD,
Bronchiectasis and pleural effusion. Said would
start with CXR then HRCT. Examiner directed
discussion towards dull lung base. Reasons,
lights criteria and cut of of pH for empyema.
Thought I had failed this as well as was not sure
about the dull lung base and this obviously was
the diagnosis. 18/20
3 a) Neuro. Elderly lady in wheelchair. Examine
LL. Asked if I could see her walk. Examiner said
not needed. B/L increased tone L>R, B/L
Proximal weakness in pyramidal distribution.
Knee jerk increased. Ankles N. plantars
equivocal, coordination N. Sensation N. asked
to check coordination in upper limb as pt was
having problems lifting legs. Examiner said its
LL examination. Anyway when i was presenting
my findings he started asking me questions
about reasons for cerebelar dysfunction. Did
not give me chance to give my diagnosis which
most likely was spastic paraparesis. Was not
sure how i had done. Got 13/20
3.b) metallic MVR with AF. Pt tachycardic. Was
expecting atleast 18/20. Questions- if this pt
came With SOB- what would your dd's be. Got
14/20
2. Hx. 30 yr old female with palpitations.
Recently had a baby. Fx hx of some auto
immune condition. Dx- post partum thyroiditis.
Was expecting 20/20. Got 16/20
4) communication skills: elderly lady with end
stage heart failure and new renal failure who
has just died. While going through notes you
notice that she was earlier on digoxin which
was stopped due to renal failure and new
bradycardia. However this was inadvertently
started by the on call team. Tell this to pts
daughter. I told her what had happened.
Apologised for mistake. Told her would do
incident report and RCA. Try to ensure this
doesn't happen again. She asked if the digoxin
could have killed her mother. I told her that this
might have been the immediate cause although
her heart was already in a bad way. Told her
would inform coroner about this. Key point -
apologise, be honest about drug error possibly
being the cause of death. Got 14/16
5 a) 60 yr man with weakness and pain in L hand
. Worried if it could be TIA. I did not have a clue
till I started examining. By fluke, I felt pulse in
both hands and noted absent radial pulse in L
hand. Went up and absent brachial as well. Told
DD as Takayasu's and examiner frowned and
asked what else. Said subclavian steal
syndrome and he gave me the thumbs up. 28/28
5 b) Gp referral for pt with dizziness. BP 130/80.
Hx of diet controlled diabetes. On asking pt
gives hx of postural hypotension. And if you ask
about standing BP , examiner tells you its 90/60.
If you check medications, then on 4 anti
hypertensives. Acei, bisoprolol, alpha blocker
and calcium channel blocker as well. So told pt
will stop both alpha and calcium channel
blocker as pt also had ankle swelling. 28/28

Bangalore. Manipal ..7/4/17


st2...Anaemia (MHA.) H/o ibs 6 yrs on
peppermint oil,occasion leg swelling,ho rta and
nsaids ,father died of cancer colon,mother
taking inj.3 monthly for anaemia,so many
issues, ,DD was so many
issues..coeliac..nsaids, ,ulcer
St3,cardio MS,,neuro..patient was very un
cooperative, not following me,,actually language
barrier was prob. I didn't understand what he
told about sensation, ,planter was very
confusing, ,Indian examiner didn't tell anything!
!
St.4 esrd bbn and plan of mx
St5,,copd with sudden breathing difficulty and rt
sided chest pain.. o/e Dec breath sound.rt
mid.dd was ptx,pul embolism, ,pneumonia. It
was good
St5 female was increase weight,,increase
bp,,,and proximal myopathy, ,o e no thyroid
abnormalities, ,some rash,,and proximal
myopathy,,features of osteoarthritis, ,knee jt,leg
oedema, ,dd,was cushing, ,hypothyroidism,
metabolic syndrome, ,discuss was on cushing. .

Malta center 2April2017


Started with abdomen
Left hypochondrial mass said spleen
Pneumonectomy straightforward.
S2
Young gentleman 24 year while running a marathoon
(after 8 klm) lost consciousness with jerking , brief
concerned is it epilepsy.
PHx similar episodes but no loss of consciousness.
No DM NO epilepsy or other illness no trauma .
Drug Hx
eczema on antihistamine the examiner said it was
significant and i should have taken more details ' I
think they wanted you to think about prolonged QT .
FHx adopted
Social negative
Concern is it epilepsy .
S3 Don't know other candidate said aortic
regurgitation.
Examine lower limb upper motor findings in one leg
gait hemipligic with walking aid
S4
Speak to Mr ...son of mrs ....80 year old lady admitted
3 days with pneumonia and developed delerium task
explain mangement and answer concern.
Discussion about treatment and prognosis of delerium
.
S5
Repeated lady after back surgery came with pain and
fever
Discussion how to cover staph aureus .
...........
Other case
12 years post CABG presented with chest symptoms
and faint concern is it heart attack .
Discussion about beta blockers side effects

Chennai (3.4.2017) 1st round


History loose motion 6 months GP IBS.
Communication delay Dx phaeochromocytoma.
Bcc.Abdominal pain for 2 days, fever and oligiuria
known DM. ddx
Visual impairment in DM pt... dx: cataract
CNS: Lt hemiplegia
CVS: systolic murmur with AVF machinery mur mur
Resp: LL BBS+crep ddx think lower zone fibrosis or
consolidation
Abd: Ascities+AVF think ESRD with (RRT) fluid
overload.
Resp: bronchiectasis/pneumonia.
CVS: Systolic murmer: PS/ASD/ WITH MS.
CNS:STROKE WITH PERIPHERAL
NEUROPATHY.
ABD: ASCITES WITH FISTULA
Chennai Day 1 -11.45 am cases
Pheochromocytoma telling the diagnosis to patient
station 4.
Bcc 1 diabetes and UTI, bcc2 diabetes and loss of
vision - funds examination.
Abdomen ckd with HSM. Resp left upper lobe
fibrosis. History 6 months altered bowels and
abdominal pain. Cardioloy AS / HOCM ? Associated
MR also. Neuro b/l cerebellar signs with absent
reflexes .. best of luck to all of you
Chennai (3.4.2017) 3rd round
history. Cough for 6 mts SOB. Bird fancier lungs.
Com. SLE renal biopsy.
BCC1 seizures.
BCC 2. knee joint pain OA!
(Copied) My exam experience in chennai today
3/4/17
Station 2
History taking of cough and shortness of breath for 6
months. She has fever on and off but no night
sweating weight loss of 6 kg within 6
Months. No wheezing she works in printing company
and her colleagues have also cough which she is
thinking due to printing materials they use all
cardiovascular history is negative. No history of TB
contact no HIV risk factors. she receive many
antibiotics without improvement.
I put differential of TB and lymphoma and asthma.
During childhood she has asthma which improved.
Not smoker or drinker. But unfortunately the
diagnosis was extrinsic allergies alveolitis. I miss birds
at home.
Station 3
Mitral stenosis with AF it was clear.
Neuro
Is parkinson plus CVA. Rigidity only on distraction
so I put parkinsonism plus CVA reflexes was
exaggerated on the right upper limb only no tremor.
Stations 4
Easy case explain renal biopsy for SLE patient.
Station 5
Difficult young on phenytoin developed seizures
yesterday after history of vomiting once and loose
bowel four times.
I don't know the case.
Second case history of bilateral knee pain with
stiffness less than 10 minutes. It was osteoarthritis no
other significant history of skin rashes or other joint
problem except back pain occasionally.
Stations one
Polycystic kidney and clear function fistula on
dialysis. Polycystic kidney is common in chennai take
care it mimics hepatosplenomegaly..
Mohamed Fadel
Thank good
I pass my exam in chennia India
It was tough one with very atypical clinical stations
I started with station 1
Abd : middle age male with active fistula + LL odema
No signs of fluid over load
Not on anti rejection Med
Ploysystic kidney disease
With cystic liver
Q.1 what is ur diagnosis
Q.2 did u see the rash now clue if there was
any
Q.3 what can cause liver cirrhosis in this pt
( worst station got 14/20)
I think the rash is lichen plans with HCV I do not
know
Resp:
Middle age man again active fistula
With hyperinflated chest
Tar stain in fingers and teeth
With no wheezes only fine Basel crip
Not changing with coughs
Q.1 what is ur diagnosis
Copd with lung fibrosis
Q.2 how would you like to investigate Q.3 any
relationship between his lung problems and dialysis
First thing may be Med then saved by the
bell
( one of the examiners was sitting on his chair at the
foot of the pt
I asked him to move politely and even moved his
chair to sit far away got 17/20

Mohamed Fadel
Station 3
Cardio
Middle age female again active fistula in right arm
Exam
Obvious Lowe limb edema
Raises JVP with v wave
Obvious apical pulsation
No thrill ,LPH
Irregular pulse
Loud s1
Could not ass the 2 heat sound coz of the fistula thrill
( which irritates the examiner )
Diastolic murmur in apex
And hypothetical tricusp reg
Q1.what is ur diagnosis
Q.2 what r the causes of MS
Any congenital MS syndrome you know
I answered lutinbaker
Got 13/20
CNS
Straight forward
Pt presented with dysphasia
Examine LL
Young male with CVA posture
Q.1 what is the lesion
Q.2 where is the lesion (subcortical)
Q.3 how to investigate and mage
( Indian male not understanding English lot of time
missed in translations, I couldn't not ass the pulse
heart or cranial nerve or ask pt to walk but I told
examainr I want to do so )
Luckily I got 20/20

Mohamed Fadel
Station 4
Common scenario
Young male with ulcerative colitis un controlled with
mesalazine
With inc diarrhea and weight loss
Council pt regarding steroid
Started with open questions
Then pt afraid. From steroid as he read about side
effects
Then I filled the gaps
Then every complication how we might mange
Involved the GP
Conserns
1) for how long you will use steroid
Ans
Until we control the disease the dec dose till we reach
remission with minimal dose
2) I want to use herbs
Ans
Sorry but I can't be sure what will be the effect on
on the disease course or how it may interact with
steroid
If u choose to use herbs plz inform ur GP
( DR.zain advice never say no to stupid thing pt want
to do just smile and give all options to him and then
till him ultimately you may hurt your self)
Discussion
What is ethical issues
What are the complications of ulcerative colitis
1)anemia
Which type
All type
Norm normo (of chronic disease)
Megaloblastis ( fe + b12 malabsorption)
Iron deficiency ( due to blood loss)
2) colorectal Ca
This what he want to hear
3) what kind of diet you will give him
I tried to be smart I will refers to dietitian
Then agin what diet
I told him high fiber diet
He asked me ru sure
I said with smile of ignorance
Yes sure
I got 20/20

Mohamed Fadel
Station 5
I was then
1 st case
Unilateral limb swelling
All vitals are stable
I put diff of cellulits + rupture beacker test
And I entered
To very young male
Again on dialysis with permicath
With chronic limb swelling over 5 years
Pain less with strange knee joint
No history of insect bite ??
I examined the limb
Slightly pitting
Not hot or tender I examined inguinal
LN
I asked pt to walk then sensation
Examiners escip and normal respectively
There was parathyroid scar
Concern by relative
1) is it related to dialysis should we increase the the
dialysis frequency
I answered not related and best to talk to your kidney
specialist if you have any concerns regarding dialysis
2)What is the cause
It most likely due to lymphatic obstruction
We need to start to do some test to know exact
cause and he do not need admission
Examiners
Q 1.what is cause of swelling
Lymphodema
Q 2. How would you like to investigate him
Stupid answer lymphogram
What might be the cause in this country
With more stupid smile
Chagass disease ( how stupid you may become in
exam
Totally forget flaria) where Chagas come from
Saved by bell
Got 18/26
2nd case
Totally straight forward
Bilateral limb swelling in HTN
I put differential
Then went to role out serious complication of HTN
There is decrease frequency of urination
With drug history on
Nefedipine for one year
Complain also from headache
Plan of management
Admit the pt
Switch nefedipine to other Med
Doing some test and scan to ur kidneys
Those were the answer of the concerns
Examiners
Q1. How to mange pt
Education, stop medications switch to other Med,
elevation of the foot
Q2. What Med you will give pt
I said ACI
Why not diuretic
I said not recommend as pt having dependent odema
+ there no evidence of fluid over load
I got 24/26
Mohamed Fadel
Station 2
History
Middle age female newly diagnosed HTN
With persistent high reading
Approach with systemic manners
No symptoms suggestive of end organ damage
No symptoms of all secondary causes of HTN
Reached gynecology history
Pt was having irregular menses with prescribed OCP
for 1 year
No other important history
( after I reached concern I remembered to ask
biological family if the pt was adopted (APkD) as in
DR.zain scenario )
Concerns
1. Do I need to take Med for life
Ans as your HTN probably due to Med there is a
good chance it might be temporary
2.will it affect me having babies
Ans. High Blood pressure may serious with
pregnancy multi desplinary team involving
pregnancy specialist and your physician would keep a
good eye over it
Plan of management
Stop the OCP , seek alternative with the help of ur
pregnancy specialist
We will do some test and may be scan to ur kidneys to
see the extend of ur disease
Examiners questions
Q.1 what is ur diagnosis and differential
Q.2 what is common-cause in this country if it is renal
cause
I answered post streptococcus GN
I got 20/20

PACES result
1st attempt
Royal Hospital Muscat
Bronchiectasis 14/20
Transplanted kidney 15/20
History ....young lady fatigue ,chronic diarrhea 9
years ...Coeliac disease 11/20...don't know why
CVS ..AVR ,20/20
CNS . MS 18/20
Communication ....delayed Diagnosis of
Pheochromocytoma .... 01/16 can't believe ..but ..
BCC1 ...Frozen shoulder 25/28
BCC2 ...young male weight loss 27/28
Total 131
Failed in clinical communication .
Sorry couldn't write in detail
Good luck to you all


My experience in MALTA Center 1/2017
Communication ; to discuss with the wife whose
husband is 45 years old gentleman who suffered from
headache and rapid deterioration of the general
condition within few hours (while being at work)
bought to the hospital and final diagnosis is
meningococcal meningitis ,GCS ONLY 7 and the plan
to shift him to ICU .
After the usual introduction almost she knew nothing
about his condition, I explained everything about
meningococcal meningitis with the help of a paper to
draw something about the brain and surrounding
meninges,
I explained about the expected outcomes considering
the GCS ONLY 7 and the plan to shift him to ICU
and the prognosis is guarded,
Of course contact tracing and related issues.
She was to much concerned about her heath and her
son are there any risk they might got the infection. I
explained about infection control department in
contact tracing and MDT, offered all forms of
support.
Finally I asked if there is anybody to drop her home
-She was understanding appreciatiating everything. I
got 16/16
-
- BCC1: Elderly PT with Ankylosing spondylitis. Has
low back pain. With history of treated breast Ca.
Chrons dis. Multiple abd scars. DD. 1_Active AS for
optimisation of treatment 2_recurrence of breast Ca
with metastases 3- osteoporotic frature, what are the
investigations. I got 28/28
- BCC2; YOUNG LADY known to have bronchial
asthma has worsening cough for the last three
months. Examining her she has expiratory wheeze
otherwise normal concern again about the cause DD
Exacerbation of bronchial asthma. When I mentioned
people as she is using oops the examiners didnot agree
(no tachycardia or leg swelling) I think I missed
asthma mimics as I went deeply in thinking
Unnecessarily I got 19/28
-
-Station 1 Chest
:bilateral basal pulmonary fibrosis. Discussion about
the causes and investigations and management. I
answered all only forget to mention drug induced
among the causes. Scored 19/20.
NEUROLOGY ;SPASTIC PARAPLEGIA
WITHOUT SENSORY LEVEL, discussion as usual
around DD, investigations (20/20)
Cardiology; an adult pt, with PSM over the apex
mostly MR ,discussion around DD,
investigations,echo FINIDING (15/20)
Abdomen ; left hypochondrial mass for DD mostly
spleen , DD, plan, no features of CLD ,no
lymphadenopathy , no facial plethora, 12/20,
WAITING FOR THE FEEDBACK TO SEE WHY
THIS MARK
HISTORY; adult pt. with migraine developed sever
headache (7/10) at the occipital area with gait
unsteadiness since three days , unsteadiness
improving partially ,no wakness , no sensory
abnormality, no visual problem, no fits, I did not ask
about vertigo , NO FEVER , NO NECK STIFFNESS
DD I mentioned storke , SAH (THAT THE
EXAMINERS DID NOT LIKE) WORSENING OF
MIRAINE
.DISCUSSION ABOUT workup , why not meningitis
, I got 14/20
FINALLY PASS 143/172
Wishing all the best for all of you, the exam needs
reasonable preparation, good practi

Alhamdulillah i have passed paces


First attempt in Royal Hospital Oman held on 8th
April 2017
Scored 146
Well before i share my experience I would like to
thank dr Ali Hameed
Dr Ali Raza
And all Mypaces team for their generous support and
a very special thank to Dr Abdul Fateh
I learnt alot from his course and it really helped me
all through preparation especially Neurology
which was the big elephant for me since final year
MBBS
I got my result a little later then my other colleagues
which made me think Mera Faisala bhi Mehfooz
ho gaya hai (my Pakistani colleagues can relate to this
joke )
Down below is my experience (i was very sure i failed
because I didn't follow my scheme during exam that i
had made while preparing for the Exam but they say
miracles do happen )
Started with station 4 delayed diagnosis of
pheochromocytoma
Surrogate was not aware of the diagnosis which i had
to break and give him reasons for delay in diagnosis
Dont know what i did wrong got 3/16
Station 5 BCC1 frozen left shoulder in Diabetic pt
In the d/d I didn't mention Frozen joint at all and
kept beating around the bush got 25/28
BCC2 was Toxic Goiter staright forward got 26/28
Station 1 unsure of the actual diagnosis of
Resp case but i diagnosed as COPD with Bi Basal
Fibrosis/bronchiectesis (i know doesn't make sense )
pt didn't have tht wet type of cough but crepts were
course and didn't change with cough
Viva was usual on management of the pt and
investgations got 18/20
Abdomen was Renal Transplant due to Apkd ,well
here also the examiner pulled diagnosis out of
stomach as i was again not telling the actual
diagnosis in my presentation ,and i also failed to
identify the transplanted kidney(i said may be he had
some abdominal surgery for some infection ,what
was I thinking still got 17/20
Station 2 coelic disease (lady with fatigue ankle
swelling and Microcytic Anemia) Viva was about the
investigations specifically asked me the Antibodies
and management of coelic got 18/20
Station 3 (my most feared but the best station )
Cardio was young male around 25 yrs with AVR no
murmur at all,though he had Afib,viva on
management of replaced valve pts ,I mentioned all the
usual with diet and medicine care, he asked me which
is the most common medicine interfering with INR ,i
said Antibiotics got 20/20
Neurology was young 25 to 30 yrs probably with
involvement of post columns and PYrmidal type
weakness in L/L left then right
Reflexes exaggerated in right diminished in
left(appeared with reinforcement) i gave D/D of MS
,syringomelia,Friedrch with MS on top due to patchy
involvement ( remembered a line by my hubby when
nothing makes sense its MS )
Viva was usual on investigation and management of
MS got 19/20
So now my advice to all preparing
Have a study partner
Think simple in exam
Do loads of prayers (as i did )
Allah make things easier for all of you
Ameen
Thanks again to all my teachers who taught me my
parents my in laws they were huge support to all my
whining and self speculations of failure

KOLKATA.
St 1 liver transplant.
Chesy old with ILD in RA
Sts changle bowel habbit
St3 hemonymous hemianopia examine cranial n
Cardio biological valve with mr ,phtn
St5 bbn SAH with INR 4
St 5 takayasu
Hemtatemsis nsaids vs crohns

I am from Myanmar. I have passed PACES


168/172 on 7th March 2017, 1st diet , my exam
center is new YGH, Yangon, Myanmar
I started with St 2
St 2 wt gain fatigue , amenorrhea, 18 mths post
partum ho of PPH blood 4 units given
Dx hypopit due to shee han
DDx Hypothyroid most probably due to post partum
thyroiditis
I got 18/20
St 3
A middle age lady presenting with SOB
MS with AF
Examiner asked what's your findings, Dx, DDx, how
would you manage, if the pt has vegetation in echo,
can it affect the INR target?
I got 20/20
Neuro station
A young lady presented with difficulty in walking, plz
examine her neurological system
Pt has ryles tube and urinary cathether inserted
Dx- Lt sided hemiplegia due to cardio embolic stroke
I got 20/20
St 4 angry pt with esrd
Scenario given was the pt had history of high blood
pressure since 5 yrs ago which was found out when he
got accident. He didn't take any medication nor any
follow up since then. Now he suffered SOB and saw
his GP , done blood test showing eGFR < 15, Hb 6.5,
Potassium 5.3, USG showing bilateral contracted
kidney. He is now seeing you what happens to him.
Task - discuss his current condition and further
management plan as appropriate.
I got 16/16
St 5 BCC1 a young lady presented with SOB SpO2
88%
dx diffuse systemic sclerosis with pul fibrosis
Examiner ask Dx, DDx for SOB in this pt,
Management
I got 26/28
BCC 2
A 54 yr old lady presented with chest pain
Dx Unstable angina(ACS) with hypothyroidism
Hidden agenda here was to refer to hormone
specialist to reduce her thyroxine dose
I got 28/28
St 1
Abdominal examination
Hepatomegaly
Dx Thalassaemia
Examiner asked Dx, points for Dx, DDx, infectious
causes of HSM, treatment of Thalassemia
I got 20/20
Respiratory system examination
Rt sided moderate pleural effusion
Examiner asked Dx, points for Dx, DDx of dullness at
base of lung, Ix, Mx, what would you consider if this
pt is working in shipyard?
I got 20/20
Thanks a lot PEC group! I may not get this
achievement without your help.
Malta center 2April2017
Started with abdomen
Left hypochondrial mass said spleen score 8
Pneumonectomy straightforward.
score 20/20
S2
Young gentleman 24 year while running a marathoon
(after 8 klm) lost consciousness with jerking , brief
concerned is it epilepsy.
PHx similar episodes but no loss of consciousness.
No DM NO epilepsy or other illness no trauma .
Drug Hx
eczema on antihistamine the examiner said it was
significant and i should have taken more details ' I
think they wanted you to think about prolonged QT .
FHx adopted
Social negative
Concern is it epilepsy . Score 13/20
S3 Don't know other candidate said aortic
regurgitation I mentioned to them the diastolic
murmer 12/20
Examine lower limb upper motor findings in one leg
gait hemipligic with walking aid
20/20
S4
Speak to Mr ...son of mrs ....80 year old lady admitted
3 days with pneumonia and developed delerium task
explain mangement and answer concern.
Discussion about treatment and prognosis of delerium
.16/16
S5
lady after back surgery came with pain and fever
Discussion how to cover staph aureus .
...........
Other case
12 years post CABG presented with chest symptoms
and faint concern is it heart attack .
Discussion about beta blockers side effects
28/28 &26/28 not sure which is which

Malaysia paces - Kuala Lumpur hospital first


carousel @ 16/4/17
Blue team
Station one : left thoracotomy scar with previous
chest tube scar -
Abd : adpkd
Station two : post partum thyroiditis
Station three : neuro pure motor neuropathy
Cvs mr n ar
Station four : explain meningococcal septicemia
Station five : bcc 1 left surgical 3rd nerve palsy
Bcc 2 hypercalcemia due to possible
hyperparathyroidism...
Red team
Station one : respi scleroderma with pulm fibrosis
Abd cld
Station two same
Ststion 3 cvs vsd, neuro cmt
Station 4 same
Station 5 bcc 1 optic atrophy
Bcc 2 ugib secondary to nsaids...
Thank you... n hv a nice day...

Alhamdulilah I have passed PACES; got 159/172


my exam experience Glasgow Feb 2017..
I started by st 3
Cardio
Pt e SOB; O/E; mid sternotomy scar, metallic click ,
no murmur , my D; MVR, viva inv & manag + valve
types!!, why no saphenous scar?! got 20/20
Neuro; pt with falls; examine his neurological system;
I started as regular greeted the pt and asked to walk;
Parkinsonian gait, I examined tone then tried
Parkinson approach as Ealing vedio but no tremor
evident, then I was stuck and dont know what to do
more, I tried power, reflexes and bulbar ex and time
out, viva about Parkinson, I thought I performed bad,
20/20
st 4
syncope due to OHG overdose in depressed nurse,
who denies the act but confirmed inv and previously
told about insulinoma, task to communicate D and
manage concerns;
concern: confidentiality, financial troubles & support,
I missed in scenario from where she got the OHG,
thought she was D then corrected myself (looked not
good)
viva; what ethics here, from where she got the OHG
and if from hospital any implications, what kind of
support! And how about psychiatry assessment
I got 11/16
BBC1
Systemic sclerosis with swallowing problems, straight
forward, concern; is it curable?! I dont know what I
missed, 26/28
BBC2
young man with Visible Haematuria with normal Ex
and history, concern is it cancer, I missed a good DD
and plan of management , EX were upset , got 22/28
st 1
Abdomen
Scar of liver transplant + drum stick clubbing, viva
abt possible D and Inv , transplant medications and
SEs, got 20/20
chest
COPD with bronchiectasis, viva causes and inv and
management, got 20/20
st 2
Dizzy spells , postural hypotention and tachy, in AF +
DM+IDH+HTN+DVT, gave DD uncontrolled AF /
Drug / Autonomic neuropathy, viva inv and
management, got 20/20
I hope this helps
Paces is a very tough but amusing experience, I
passed from 3rd trial all in UK, done courses in
Ealing, Hammersmith in London and paces 4 u in
Manch, got 110 in 1st trial and 117 in 2nd , now 159
praise and favour to Allah
Try to study hard 50% and try to practice in your
real life 150% and in shaa Allah you will Pass
Hope the best for you all, much thanks to this group
and all colleagues, your experiences were so helpful
Have a nice day

My Cases in bangalore. Manipal ..7/4/17


Started with st2...Anaemia (MHA.) H/o ibs 6 yrs
on peppermint oil,occasion leg swelling,ho rta
and nsaids ,father died of cancer colon,mother
taking inj.3 monthly for anaemia,so many
issues, ,DD was so many
issues..coeliac..nsaids, ,ulcer
St3,cardio MS,,neuro..patient was very un
cooperative, not following me,,actually language
barrier was prob. I didn't understand what he
told about sensation, ,planter was very
confusing, ,Indian examiner didn't tell anything!
!
St.4 esrd bbn and plan of mx
St5,,copd with sudden breathing difficulty and rt
sided chest pain.. o/e Dec breath sound.rt
mid.dd was ptx,pul embolism, ,pneumonia. It
was good
St5 female was increase weight,,increase
bp,,,and proximal myopathy, ,o e no thyroid
abnormalities, ,some rash,,and proximal
myopathy,,features of osteoarthritis, ,knee jt,leg
oedema, ,dd,was cushing, ,hypothyroidism,
metabolic syndrome, ,discuss was on cushing. .
Thnx and pray for me...
Oman center today
Chest case Bronchectesis
Abdomen case Renal transplant
History
Cystic fibrosis
Cardiology MVR
Neurology HSM PN
Communication
Renal biopsy from SLE
Station 5
BCC1 Diarrhea in Rhumatoid and psoriasis
BCC2 Goiter

Chennai (3.4.2017) 1st round


History loose motion 6 months GP IBS.
Communication delay Dx phaeochromocytoma.
Bcc.Abdominal pain for 2 days, fever and oligiuria
known DM. ddx
Visual impairment in DM pt... dx: cataract
CNS: Lt hemiplegia
CVS: systolic murmur with AVF machinery mur mur
Resp: LL BBS+crep ddx think lower zone fibrosis or
consolidation
Abd: Ascities+AVF think ESRD with (RRT) fluid
overload.
Resp: bronchiectasis/pneumonia.
CVS: Systolic murmer: PS/ASD/ WITH MS.
CNS:STROKE WITH PERIPHERAL
NEUROPATHY.
ABD: ASCITES WITH FISTULA
Chennai Day 1 11.45 am cases
Pheochromocytoma telling the diagnosis to patient
station 4.
Bcc 1 diabetes and UTI, bcc2 diabetes and loss of
vision - funds examination.
Abdomen ckd with HSM. Resp left upper lobe
fibrosis. History 6 months altered bowels and
abdominal pain. Cardioloy AS / HOCM ? Associated
MR also. Neuro b/l cerebellar signs with absent
reflexes .. best of luck to all of you
Chennai (3.4.2017) 3rd round
history. Cough for 6 mts SOB. Bird fancier lungs.
Com. SLE renal biopsy.
BCC1 seizures.
BCC 2. knee joint pain OA!
(Copied) My exam experience in chennai today
3/4/17
Station 2
History taking of cough and shortness of breath for 6
months. She has fever on and off but no night
sweating weight loss of 6 kg within 6
Months. No wheezing she works in printing company
and her colleagues have also cough which she is
thinking due to printing materials they use all
cardiovascular history is negative. No history of TB
contact no HIV risk factors. she receive many
antibiotics without improvement.
I put differential of TB and lymphoma and asthma.
During childhood she has asthma which improved.
Not smoker or drinker. But unfortunately the
diagnosis was extrinsic allergies alveolitis. I miss birds
at home.
Station 3
Mitral stenosis with AF it was clear.
Neuro
Is parkinson plus CVA. Rigidity only on distraction
so I put parkinsonism plus CVA reflexes was
exaggerated on the right upper limb only no tremor.
Stations 4
Easy case explain renal biopsy for SLE patient.
Station 5
Difficult young on phenytoin developed seizures
yesterday after history of vomiting once and loose
bowel four times.
I don't know the case.
Second case history of bilateral knee pain with
stiffness less than 10 minutes. It was osteoarthritis no
other significant history of skin rashes or other joint
problem except back pain occasionally.
Stations one
Polycystic kidney and clear function fistula on
dialysis. Polycystic kidney is common in chennai take
care it mimics hepatosplenomegaly..
Chest I don't know it.

Barnet hospital March 2017


Exam experience :
S2: 40 yrs old lady with a headache...past
history of CVA CT was done small infarct ..from
the history she has symptoms of increase
intracranial pressure..she afraid she may have
the same condition again discussion was about
dd :SOL ,IIH its possible causes and risk factors
investigations basic and CT brain ..if itis SOL
what its types..he want solitary lesion or mets
..if mets from where ? : from breast ca? ..if CT
normal? :for LP what you will see in LP ..the
pressure cells etc 18/20
S3: CVS: pt in 50s with collapsing pulse and
clear systolic murmur in aortic area radiate to
the neck i said Mixed aortic valve what you will
do for him ..in failure or not ..i think i missed the
AF as they asked me to check the pulse again if
regular or not ..not sure of this case 12/20
Neuro: cranial nerve examination in a lady of
50s ..when i asked her if she has any problem
with the sense of smelling she said yes..but i
forgot it in in the discussion ..other affected
cranial nerves ; Rt 5 , lower 7,
8 and 12 ..the discussion about DD why the 12th
is involved. it's pathway?
What you will do. .MRI why MRI and not CT ..to
see the post fossa. .18/20
S4: explain SLE and the need for biopsy for
young lady: i explained the
disease for her, the biopsy and its importance
for treatment , its complications,
She concerned about the effect of the disease
on her life ..i told her itis not curable but
controllable by good follow up also asked about
pregnancy: planned pregnancy then follow up
with woman doctor and the joint
doctor..multidisciplinary team..she was student
i told her also we will give her sick report in the
day of her biopsy and during her current
admission..i offer also a meeting with my
consultant when she wasnt convinced with the
biopsy..but finally she accept it.. I asked her
about her concerns and any thoughts in her
mind many times, she mentioned the point of
pregnancy at the last..i summarized and
checked her understanding ..when i finished
still i have time..so again i asked about her
social history more and summarize again...they
asked if i convince her or not ..what if she
refuse : i will give her a chance to think about it
again and arrange a meeting with my
consultant..what if she still refuse ..i will respect
her autonomy..asked again about
prgnancy...16/16
S5 : 50 yrs old man post MI a few weeks ago..
came with lethargy and tiredness : i didnt know
the diagnosis ..i thought about anemia and
hypothyroidism both was negative from the
history and examination ..so i just tried to be
diplomatic took good history rapid examination
offer investigations to know the cause..the
examiner asked me what you want to do ..as
from the history he has dizzi spell i told him
sitting and standing BP basic investigations
including the ECG ..i dont remember if he was
diabetic or not...i thought i will fail this station
because i didnt know what is the diagnosis
25/28
The 2nd one was easier but i did badly..young
lady came with headach and convulsions.. I
took rapid history about the convulsions and
first time or not ? She was epileptic and stopped
her ttt due to pregancy ..i asked about her social
history..i did rapid examination for the pt power
ask about her concerns advice her to follow
with the neurologist and not to stop the
medication by her self ..yes most of the
antiepileptics has bad effects on the baby but
the neurologist will choose the safest one and
will outweigh between the risks and benefits..in
the history i asked about the symptoms of
increase intracranial pressure but when i knew
that she had epilepsy and stopped the treatment
bcoz of pregnancy i didnt think about it in the
examination ..they asked me what you missed
and when i see the ophthalmoscope beside the
pt i said fundoscopy..they asked what i will look
for? i forgot also..but after a while i remembered
the papilledema! They also asked me why i
didnt ask specifically about the drug name
..which was valporate they said she was
intelligent enough to stop it.(i didnt agree but
but i didnt argue with him (this is the
rule ^^) ...anyhow he was generous enough to
give me the 15/28
S1: COPD ..the discussion was straight forward
investigations,management including ttt for
respiratory failure ..difference between
emphysema and bronchitis 20/20
Abd: i forgot many things in this station ..was
also direct: Renal transplant..i forget to cheque
for the fistulae if functioning or not and when he
asked me if the transplanted kidney functioning
or not i told him i should cheq for the fistula he
asked me to go back and see the pt..it was
fibrosed ..the pt had skin rash in his Rt forearm
also i didnt know it just said may be cellulitis
..he asked about the causes i said DM,HTN
,polycystic and mentioned some other causes
he asked me for a one cause ..i said DM
..why?most common ...how can you know it ?
by finger pricks (which i forgot to look for!)..the
first question after i finishe examination was
what ttt is he taking : Tacrolimus ..Why? Due to
the tremor..mention other drugs:
Azathioprine...what its side effects ...what test
we do before starting it : we measure enzyme
but i dont remeber its name! ...at the end he
smile and said you didnt check and find the
polycystic kidney!(at this poit i thought i missed
the diagnosis and failed) i told him i checked for
the kidney but didnt find it...20/20
Thats all...of course i want to thank both dr Zain
and Dr Ramadan for their v hepfull courses and
advices ..

My exam was in wythenshawe Manchester


: In station 5 my first case was on ANGINA.
instruction was like that::: middle aged lady
present with chest pain. All finding are normal
on observations. On taking history, chest
tightness for couples of months on exertion.
HTN, HYPERLIPIDEMIA, ON TREATMENT.
2nd case . Young lady present with collapse
while listening music
It was a 2nd episode, first 3month back without
warning, no chest pain or palpitation. I took history
from cardiac as well neurology point of view. Social
history was significant as she was a bus driver. I
follow the history, exam, and ask for concern. And
advise not to drive.

Thursday 16/3/2017
Dear friends, I am going to share my painful and
tragic experience with you. I have passed the
paces (overall score) on two occasions
achieved 147/172 and 138/172 , but failed in one
SKILL, first time by one mark and second time
by 2 marks only. ONE CAN SAY IT A HARD
LUCK, but my struggle will continue and
INSHAALLAH with the never given up approach,
success will be on the way some day.
Please pray for me. Good luck for everyone.

My PACES experience,
28th February 2017 at Aberdeen Royal Infirmary.
RCP Edinburgh.
While I was in the elevator to 1st floor where the
exam was to be held, I was accompanied by an old
gentleman who asked me why I was here...I told him I
have exam here...he said 'I might be one of ur guinea
pigs'....I felt an urge to ask him what was wrong with
him (bcz we 2 were alone in the elevator anyway) but
didn't do so for fear of getting disqualified from the
exam!!
Station 1:
Respiratory : left thoracotomy scar.
Chest expansion was equal on both sides, nd
percussion was resonant bilaterally so I excluded
pneumonectomy. No abnormal findings in
contralateral lung. So dx i made was lobectomy.
Examiner asked about causes of lobectomy. When I
mentioned TB as one of the causes, he didn't like that
but then I carried on to mention others with which he
was happy. He then asked me how I would
investigate,,,I mentioned baselines and pulse oximetry
and pulmonary function tests and then THE BELL
RANG.
Score 11/20
ABDOMINAL:
White patient who had extensive spider naevi over
upper torso and upper limbs. It took me time to
differentiate naevi from telangiectasia and I kept on
pressing on them to check how they refilled. Patient
had clubbing and slight tremor. He also had
hepatomegaly, no splenomegaly. He had marked
flank fullness and when I asked him to lean forward a
little, the fullness became even more prominent. I
thought these are polycystic kidneys. I ran out of time
and couldn't check for ascites, pedal edema and the
back of the patient!!! And I made a dx of polycystic
kidney disease. It could very well have been CLD with
ascites!!!! Examiner didn't ask many questions bcz it
took more time for me to justify the dx I had made.
Score 9/20
Station 2:
50 yr old Patient who had a witnessed collapse. Seen
to be jerking his left upper and lower limbs. GP
concerned if it is epilepsy.
I took a detailed and thorough hx and addressed the
patients concerns and gave a good list of d.dx.
Patient had a hx of lymphoma nd hx of radiations to
the chest. He also had a hx of non resolving chest
infection.
Score : 14/20 which was quite unnvelievable bcz I
didn't miss anything in this one - or at least I thought
so
Station 3:
CVS:
patient had AS murmur, collapsing pulse but pulse
was good volume. So I told him that patient has AS
murmur and pulse is good volume which does not fit
well so he may be having mixed aortic valve disease.
He didn't seem to understand my point (this was
taught to us in Ealing and paces ahead courses). He
then asked me how to investigate the case and then
the bell rang.
I had forgotten to check for radiation to the carotid
and examiner asked me about that too.
Score: 7/20
CNS:
Here I met the old patient who I had net b4 in the
elevator. Command was to examine his left hand.
This was a technically very difficult case bcz
examining a big gentleman with left hand weakness
from the right side is very difficult. And his hand
couldn't move or bend at all so it took me a while to
position him for the different parts of motor
examination. In this chaos I forgot to ask him to
clench his teeth so I could reinforce the absent
reflexes which I got. And I was left with sensory
examination when examiner said it have 2 min left. So
in hurry I just ran the sharp pin over a straight line
on the patient's hand instead of checking in
dermatomal fashion. I asked to walk the patient but
examiner said not to check that. When in was asked
to explain my findings, I started off asked nd
examiner told me in between that I didn't reinforce
the reflexes and I didn't check for pain in
dermatomal fashion. I gave dx of MND whereas it
was monoplegic stroke :( I knew I had flunk the
station!!!!!
Score 5/20
Station 4:
60 yr old lady with iron Def anemia. Gets blood
transfusions nd feels better. Task was to counsel
against risks of blood transfusions nd guide about
investigations. Scenario also mentioned that cause of
anemia has been localised to bleeding from the gut.
The patient told me sge had taken iron supplements
for anemia but they made her sick so she is not on any
more now. I told her about rare infections
transmitted through transfusions, transfusion
reactions, risk of fluid overload etc. I told her she wl
need an endoscopy at which she said aloud that she is
never going to have it bcz she had it in the past nd
had a terrible experience and so they had to cancel it
and wl never have it again. I told her she wl be given
a numbing agent sprayed at her throat so she doesn't
feel irritated when the scope is passed through her
mouth,,,at which she cried out aloud: 'u never told me
it's going to be through the mouth' and I felt shocked
bcz she had just told me she had it in the past!!! So I
presumed she would be knowing it's through the
mouth.
Anyhow I counselled her....
But examiner told me i should have told her that she
has a suspicion of cancer which needs to be ruled out
and investigations are necessary for that.
He also told me i should have offered to reduce the
dose of iron supplements to avoid nausea.
Score: 12/16
Station 5:
BCC1 :
Patient with hemoptysis and family hx of malignancy.
I forgot to ask about occupation. And I didn't percuss
on the back bcz I had done complete examination
from the front which was unremarkable and time was
limited and so ended up missing on pleural effusion.
Rest went okey.
Score: 23/28
BCC2:
Patient with dryness of skin and constipation. BP
158/90. Having severe headache.
While taking hx I got to know patient has
hypothyroidism. For headache, I offered to do
fundoscopy but was told that it is normal. Then i did
visual fields...they were also normal. That confused
me bcz this ruled out pituitary adenoma as a cause of
the hypothyroidism and HTN. But I should have said
that it looks like a small non compressing pituitary
adenoma. And patient kept on talking about
headache and what it could have been but I was
clueless....and so missed even offering her some opioid
analgesics for the headache that was not responding
to paracetamol, nsaids etc.
Score: 15/28
So i couldn't clear the exam. I had studied cases for
paces, hx and communication skills from ryder mir,
stations 1 and 3 from gautam mehta and had watched
all the pastest videos. Still I guess i messed it up in the
exam. I tried to be over gentle with patients and so I
ran short of time :(
I also complicated simple things like CLD or
monoplegic stroke.
And last but not the least, I must say the exam is
stricter than i had expected bcz i lost marks even in
the stations that I had done perfect.
Hope this helps

i have passed alhamdulillah


Sharing my experience at London hospital
Station 1...obese patient with tattoo marks and
multiple scars on abdomen...signs of CLD and gum
hypertrophy with livervtransplant...i didnt get the scar
as it wasnt the typical mercedes benz..examiner asked
if he has CLD what would u expect..i said LT then
questions about othet causes of end stage liver and
immunosuppression
Respiratory was broncheictasis with COPD..questions
about D...

Egypt 6 of February, New Kasr Alainy Hospital ..first


carousel
Station 4
A lady admitted to hospital with UTI found to have
... PCKD complicated by ESRD
Task: to peak to her daughter.. I started by greeting her
daughter
Asked her about the previous health of the mother
that led to her being admitted.. BROKE the bad news
with empathy,asked about her knowledge...explained
the disease..it's manifestation..complication..talked to
her specifically about ESRD at that her mother is in
need for dialysis..she was depressed because of her
mother condition explained that this disease runs in
family that's why we need to do certain tests to see
if.she is having the disease as she is at the proper
age....she had many questions which I respond to if she
will end like her mom..explained that there is a
possibility at 60but we will follow her closely..so that
we can control the complications..I asked if she had a
partner she said no.. Siblings she had 2 their ages they
were over 20
Which she did not have asked about any more concern
she had told her that it's the proper age for counselling
about the disease... Summarized what we talked
about.. Checked her understanding Checked if there is
any one looking after the mother with whom she was
... living offered help
The examiner asked what is the issues in this
...scenario
I told him BBN and counselling the daughter
He was smiling all through from the start so that gave
me some relief thank god
First didn't get his point why will she not be ok
Second question was do you think if even she had done
the investigation and turned not to have the disease
she will be ok
Then he explained imean for not having the disease
and carrying the gene..Yes of course she needs to be
counselled about the disease before having the tests
done
Thank god I got 16/16
Station 5
It seemed like a nightmare
Even before entering the room..I was shaking so bad
anyway I managed to get to the room
BCC1 was a man 30 yrs weight gain
I took the history it was going with hypothyroidism
which caused him fatigue.. prox.weaknessthen I
proceeded to examine him..finished the past history
d.h...f.h..social..offered some help group as his
prob.was affecting him socially and his work..his
concern was what is my problem..is it
treatable..examiner asked what's his problem..what
did you find when you examined him..you found him
to have prox.myopthy why not examine lower limbsI
answered examined for prox.myopthy..in his upper
limbs..how.are you going to investigate him..he was
smiling all throughI was still shaking
I got 24
BCC2
A lady about 30 diagnosed one year with skin lesion
and has DM
I asked about the lesion 1 year ago started as vesicles
itchy affecting her mouth..she was put on steroid
devoped cm 6months...I went to examine her started
with the lesion ..asked for torch they won't offer it
unless you askasked about other side effects of
steroid she was not any prophylaxis..offered help
groups in social history..her concern was was the
diabetes related to my treatment any other options I
told her it's an essential ttt for your condition on
weighing the risk and benefits can go back to the skin
doctor to revise the dose and give you some
prophylactic ttt..regarding other options yes there are
other options but also with side effects so let's refer
you to skin doctor the gland doctor etc
Examiner asked what's your diagnosis I said pemphigus
vulgaris...what is the cause of her d.m..I told him her
.steroid..other options I said yes azatiopurine
I got 28
Station one
Chestworst
Positive findings bilateral dullness.trachea deviated to
rt..deceased air entrance bilat..increased tactile vocal
fremitus...examiner what's yr diagnosis I said bilateral
fibrosis more on rt side..then he asked about the
causes of fibrosis
I got 8
..Abdomen
I tried to forget about the last case..patient was
jaundiced..pale..Abdomen left subcondrial scar..liver
palpable tender 6cm below coastal margins 14cm
span..No signs of ascites...examiner asked what's your
positive findings?!like I said above..what's yr diagnosis
appendectomy..plus hepatomegally possible cause
)haemolytic anemia(thalassemia
How can you explain the hepatomegally I said
secondary to repeated blood transfusion developed
hepatitis or coincided with the anaemia
What will you find in blood film of this patient I said hj
bodies
I got 18
Station 2
years old male with history of rt side chest pain 48
occurred when he is at work mainly of short duration
no aggravating or reliefing factors associated with
sweating. palpitaion.dizziness ..known Hypertensive
last reading was was 150/90 on ttt.not known
diabetic..No history of high cholestrol..clots.positive FH
of sudden death his brother..otherwise not known to
smoke drink alcohol
CONCERN what is my problem as it is recurrent..can
you give me some painkiller.. at that point I had put on
my mind differential to be honest no difinit
cause...so I said to him for right now to be able
to.answer your concern.I have to do some tests and
examine you...as there are many possible causes like
..unstable angina..HOCM..arrhythmia
Examiners so what's your diagnosis I said My
differential diagnosis is unstable
angina..HOCM...Arrhythmia
Why don't.you.give him just pain killer...me he is HTN
plus history of sudden death his brother I would have
to role.out serious.causes then I can give him the pain
killer..so you will admit him...yes to investigate..so
what investigation general..then.ECG..cxr..cardiac
enzymes..echo..
I got 17
Station 3
Cardio...old man lying in cardiac bed dyspnic..pulse
small volume irregular..AF...apex difficult to
detect...1st heart sound loud..2nd heart sound
accentuated at pulmonary area
MDM at the mitral area..so my diagnosis was
M.S..AF..PHTN
...EXAMINER what's your diagnosis
I got 20
..Neurology
.Instructions are to examine lower limbs
Female unable to walk for long time
On examining her she had increased tone.clasp knife
rigiditiy..power of umn pattern..increased
reflex..normal sensation in lower limbs..I asked to
proceed and examine upper limbs which was
normal...Examiner what's your diagnosis I said spastic
paraplegia e normal sensation
What are the possible causes I said hereditary spastic
para...CP..then he said what else would help you to
confirm your diagnosis I told him FH...then he said time
.up
I got 20

UK Experience
Glasgow ,, Febriwary 2017
Aberdeen hospital
St_1
Abdomen = HSM
cirrhosis
Respir= rt. lower lobe LOBECTOMY secondery to
telangectasia
St_2
elderly male e type 2 DM
presented e lethergy for 6 months
having HBA1C 5.5 & Chr.kid.dis stage 3
St_3
Cardio = Aortic valve rep
metallic=marfan
Neuro = examin upper limb
pt. has stroke
St_4
BBN to the daughter regard her father who has
advanced bladder cancer .. diag. 3ys back & now
presened to hospital w drowsiness
St_5
BBC1:: ELDER LADY
known RA presened w lt. wrist & forearm pain
take focused history &proceed
::BBC2
young male w sudden loss of vision
in rt. eye
diagnosed to have central branch obstruction

my experience in paces UK , station 1.....copd


+bronchiectasis20/20 .renal transplant in diabetic
pt15/20
St2 .attacks of diarrhoea&vomiting in diabetic pt dd
(autonomic neuropathy, Addison, bacterial
overgrowth) 20/20
St 3...cardiovascular MR+Af 18/20......neuro pt 50 yr
complain, diplopia and difficult walk he has cerebellar
signs on the lt side, I missed it coz of confusion 4/20
St 5.....psoriatic arthropathy 28/28........acute Back pain
in female 35yrs old she has history of cushing disease
removal from brain taking hydrocortisone dd vertebral
fracture, mets, disc 23/28
Score 133/172, but I fail in physical examination minus
one mark , so I fail but alhamdlillah

Alhamdulilah I have passed PACES; got 159/172


..my exam experience Glasgow Feb 2017
I started by st 3
Cardio
Pt e SOB; O/E; mid sternotomy scar, metallic click , no
murmur , my D; MVR, viva inv & manag + valve types!!,
why no saphenous scar?! got 20/20
Neuro; pt with falls; examine his neurological system; I
started as regular greeted the pt and asked to walk;
Parkinsonian gait, I examined tone then tried
Parkinson approach as Ealing vedio but no tremor
evident, then I was stuck and dont know what to do
more, I tried power, reflexes and bulbar ex and time
out, viva about Parkinson, I thought I performed bad,
20/20
st 4
syncope due to OHG overdose in depressed nurse, who
denies the act but confirmed inv and previously told
about insulinoma, task to communicate D and manage
;concerns
concern: confidentiality, financial troubles & support, I
missed in scenario from where she got the OHG,
thought she was D then corrected myself (looked not
)good
viva; what ethics here, from where she got the OHG
and if from hospital any implications, what kind of
support! And how about psychiatry assessment
I got 11/16
BBC1
Systemic sclerosis with swallowing problems, straight
forward, concern; is it curable?! I dont know what I
missed, 26/28
BBC2
young man with Visible Haematuria with normal Ex
and history, concern is it cancer, I missed a good DD
and plan of management , EX were upset , got 22/28
st 1
Abdomen
Scar of liver transplant + drum stick clubbing, viva abt
possible D and Inv , transplant medications and SEs,
got 20/20
chest
COPD with bronchiectasis, viva causes and inv and
management, got 20/20
st 2
Dizzy spells , postural hypotention and tachy, in AF +
DM+IDH+HTN+DVT, gave DD uncontrolled AF / Drug /
Autonomic neuropathy, viva inv and management, got
20/20
I hope this helps
Paces is a very tough but amusing experience, I passed
from 3rd trial all in UK, done courses in Ealing,
Hammersmith in London and paces 4 u in Manch, got
110 in 1st trial and 117 in 2nd , now 159 praise and
favour to Allah
Try to study hard 50% and try to practice in your real
life 150% and in shaa Allah you will Pass
Hope the best for you all, much thanks to this group
and all colleagues, your experiences were so helpful
Have a nice day

:UK PACES experience


I would like to share with the group my exam stations,
.in the UK Luton and Dunstable Hospital - #London
:Station one
:Abdomen
.Pre-emptive renal transplant secondary to DM
:Resp
Bilateral lungs transplan (clampshell scar )with
.evidence of fine end crepts
:Station 2
history suggestive for Core Pulmonale secondary to
advance COPD, lots of social issue to address, advice on
.smoking cessation and LTOT
Concern: Had recent history of IE COPD led to
intubation, worried to happen again, I suggested
.vaccinations and rescue bag of medication
:Station 3
CNS: pastic paraparesis secondary to probable MS
)(Young lady
CVS: Mixed Aortic valve disease with predominant
.)AR (Collapsing pulse
:Station 4
Medical error, wrong dose insulin charted over the
.weekend caused hypoglycemia to the patient
Concern: how could I ensure that doesn't happen to
?other patients? Is the service bad over the weekend
Examiner asked me how would you report a medical
error in the NHS
For this scenario particularly there is a sample in the (
neck website which show how exactly to report and
)address error, I felt quite confident with it
Station 5
BCC1: Right homonomus hemiaopia , secondary to
.Stroke
.Make sure to start Aspirin after CT BRAIN
The patient is a Taxi driver, should be reported to the
.DVLA to stop driving
Examiner asked about investigations and site of the
.lesion
BCC2: Carpo pedial spasm secondary to iatrogenic
hypocalcemia after a thyroid operation. (She want tell
.)you about it till you notice the scar in the neck
?Concern : would I be cured
I mentioned the endocrinology input and the
.treatment of hypocalcemia
Examiner asked about how could I clinical demonstrate
.hypocalcemia (the two signs), management plan

My exam first cycle dubai 20/2


I start with history Yong patient complain of chest pain
with family history father died at 45 years IHD and they
gave that his cholesterol is high upper normal started
by grating the patient introduce my self as Dr told us
then I asked about the pain dull aching not radiated no
any other symptoms
He is smoker
his work as something inthe bulding carrying object
that what I understand and he was seen by other
doctor and stress done for him 9 month back
Then his concerns what is the cause of the pain I said
we need to rule out ischemic cause first as he had
strong family history and smocker and his cholesterol is
high but he is not happy then I said we need to do echo
and possible coranery angio then
I closed by checking understanding
Discussion with examiner's what could be the cause
I said ischaemia. And mention many differential but he
is not happy at the end I said muscular
Then he told me what will be your first diagnosis then
I said ischemic then muscular time finshed i am not
happy they gave me 12
Station 3 first
Cardilogy aortic stenosis with classic murmur but large
volume pulse collapsing and neck vein pulsation
Pulse regular
Discussion what is your finding
Diagnosis
AS and possibly AR but I could not heard the murmur
who you confirm ur diagnosis
echo looking for 1234
Management
Education and counselling avoid exercise
Valve replacement then at end what other i said TAVI
I got 20
Neur examin hands of the patient
No obvious deformity I start with inspection then I
examin ulnar median and redial she had lt hand
median palsy
Discussion what could be the cause investigation
I got 20
station 4 Communication
Bdn of uncertain malignancy
Long scenario female 80 ys presented e
vomiting,obstructive jaundice loss 20 kg of body
weight but still obese for 2 m u/s and CT no mass no
lymph node Stent inserted in common bile duct no
malignant cells in biliary fluid but still malignancy is
suspected
Task inform her daughter and answer her queries l
started as Dr zain taught us identify myself my role,
check relative identity, ask if she want any body to
attend meeting said no I inform her about reason of
meeting, then I ask her about what she knows about
her mother condition she started to talk for about 3
minutes telling the story of her mother so I explain to
her the result of investigation and told her that we are
still suspecting that her mother bad growth she said
what do mean I told her mean malignancy she get the
phases bad news reaction l left her to express herself,
and after becoming calm asked what happened after I
told her we need to take tissue sinp from suspected
area, she told please don't tell mama I answer as in
doctor zain course I handle thus issue gently if your
mam wants to know we will inform her we will not
enforce information to her and this will help her to talk
decision about management plan and if hide
information she might know and then she loose trust
in medical team. and accepted. Another concern want
to take her mama I said now we need to do some test
and I need to consult my senior and oncologist then if
she remain stable after she can be discharged then
discussion was about ethical issues, why you will
inform the pt I answered as said to daughter and the
autonomy of pt as she is competent. as about elment
of competence he asked about involving the senior
I got 16
Yangon (10.3.2017) last round
Station 3 MS AF Pulmonary HT Spastic paraplegia on
anti TB
STATION 2 postpartum thyroiditis
Station 1 pleural effusion
ADPKD neckline scar and AVF
Station 4 steroid UC ( main concern want to take
alternative medicine (herbal medicine) instead of
steroid
Station 5 Bcc 1 TIA Hypertension Grade 2 and 3
hypertensive retinopathy on both sides
Bcc 2 chest pain ( pleurisy) and burnt out RA hand

Malta
Station 1
Hepatosplenomegaly
Chest scar
Station 2
Diarrhea IBS
Station 3
MR AS
Spastic paraparesis with intact sensation
Communication
Cancer pancreas
Station 5
Asthma with pregnancy
Tremors

Hi everyone would like to share my exam experience


Egypt 6 of February New Kasr Aainy Hospital ..first
carousel
I Started with st4
A lady admitted to hospital with UTI found to have
PCKD complicated by ESRD ...task..speak to her
daughter.. I started by greeting her daughter
Asked her about the previous health of the mother
that led to her being admitted.. BROKE the bad news
with empathy,asked about her knowledge...explained
the disease..it's manifestation..complication..talked to
her specifically about ESRD at that her mother is in
need for dialysis..she was depressed because of her
mother condition explained that this disease runs in
family that's why we need to do certain tests to see
if.she is having the disease as she is at the proper
age....she had many questions which I respond to if she
will end like her mom..explained that there is a
possibility at 60but we will follow her closely..so that
we can control the complications..I asked if she had a
partner she said no.. Siblings she had 2 their ages they
were over 20
Which she did not have asked about any more concern
she had told her that it's the proper age for counselling
about the disease... Summarized what we talked
about.. Checked her understanding Checked if there is
any one looking after the mother with whom she was
... living offered help
The examiner asked what is the issues in this
...scenario
I told him BBN and counselling the daughter
He was smiling all through from the start so that gave
me some relief thank god
First didn't get his point why will she not be ok
Second question was do you think if even she had done
the investigation and turned not to have the disease
she will be ok
Then he explained imean for not having the disease
and carrying the gene..Yes of course she needs to be
counselled about the disease before having the tests
done
Thank god I got 16
Station 5 seemed like a nightmare
Even before entering the room..I was shaking so bad
anyway I managed to get to the room
BCC1 was a man 30 yrs weight gain
I took the history it was going with hypothyroidism
which caused him fatigue.. prox.weaknessthen I
proceeded to examine him..finished the past history
d.h...f.h..social..offered some help group as his
prob.was affecting him socially and his work..his
concern was what is my problem..is it
treatable..examiner asked what's his problem..what
did you find when you examined him..you found him
to have prox.myopthy why not examine lower limbsI
answered examined for prox.myopthy..in his upper
limbs..how.are you going to investigate him..he was
smiling all throughI was still shakingI got 24
BCC2 a lady about 30 diagnosed one year with skin
lesion and has DM
I asked about the lesion 1 year ago started as vesicles
itchy affecting her mouth..she was put on steroid
devoped cm 6months...I went to examine her started
with the lesion ..asked for torch they won't offer it
unless you askasked about other side effects of
steroid she was not any prophylaxis..offered help
groups in social history..her concern was was the
diabetes related to my treatment any other options I
told her it's an essential ttt for your condition on
weighing the risk and benefits can go back to the skin
doctor to revise the dose and give you some
prophylactic ttt..regarding other options yes there are
other options but also with side effects so let's refer
you to skin doctor the gland doctor ect...the e
Examiner asked what's your diagnosis I said pemphigus
vulgaris...what is the cause of her d.m..I told him her
steroid..other options I said yes azatiopurine. Igot 28
Station one chestworst
Positive findings bilateral dullness.trachea deviated to
rt..deceased air entrance bilat..increased tactile vocal
fremitus...examiner what's yr diagnosis I said bilateral
fibrosis more on rt side..then he asked about the
causes of fibrosis I got 8
Abdomen..tried to forget about the last case..patient
was jaundiced..pale..Abdomen left subcondrial
scar..liver palpable tender 6cm below coastal margins
14cm span..No signs of ascites...examiner asked what's
your positive findings?!like I said above..what's yr
diagnosis appendectomy..plus hepatomegally possible
)cause haemolytic anemia(thalassemia
How can you explain the hepatomegally I said
secondary to repeated blood transfusion developed
hepatitis or coincided with the anaemia
What will you find in blood film of this patient I said hj
bodiesIgot18
Station 2
years old male with history of rt side chest pain 40
occurred when he is at work mainly of short duration
no aggravating or reliefing factors associated with
sweating. palpitaion.dizziness ..known Hypertensive
last reading was was 150/90 on ttt.not known
diabetic..No history of high cholestrol..clots.positive FH
of sudden death his brother..otherwise not known to
smoke drink alcohol
CONCERN what is my problem as it is recurrent..can
you give me some painkiller.. at that point I had put on
my mind differential to be honest no difinit
cause...so I said to him for right now to be able
to.answer your concern.I have to do some tests and
examine you...as there are many possible causes like
..unstable angina..HOCM..arrhythmia
Examiners so what's your diagnosis I said My
differential diagnosis is unstable
angina..HOCM...Arrhythmia
Why don't.you.give him just pain killer...me he is HTN
plus history of sudden death his brother I would have
to role.out serious.causes then I can give him the pain
killer..so you will admit him...yes to investigate..so
what investigation general..then.ECG..cxr..cardiac
enzymes..echo..Igot 17
Station 3
Cardio...old man lying in cardiac bed dyspnic..pulse
small volume irregular..AF...apex difficult to
detect...1st heart sound loud..2nd heart sound
accentuated at pulmonary area
MDM at the mitral area..so my diagnosis was
M.S..AF..PHTN
EXAMINER what's your diagnosis...I got 20
Neurology..instruction examine lower limbs
Female unable to walk for long time
On examining her she had increased tone.clasp knife
rigiditiy..power of umn pattern..increased
reflex..normal sensation in lower limbs..I asked to
proceed and examine upper limbs which was
normal...Examiner what's your diagnosis I said spastic
paraplegia e normal sensation
What are the possible causes I said hereditary spastic
para...CP..then he said what else would help you to
confirm your diagnosis I told him FH...then he said time
up.I got 20

:My experience in PACES exam 1/2017


I had the exam in Cairo,Kasr Eleiny 3rd carousel
6/2/2017
I got 150 / 172
First I would like to thank our brother and teacher dr
Ahmed Maher Eliwa and all people who supported us
and I appreciate your efforts
.with us for learning and success
My advice to all doctors who will enter the exam in the
future as dr
Shaimaa Mohamed Abdou said is not to put any
previous failure in your mind and try to sleep well
before the exam ,and also try not to read or revise any
.thing the day before the exam ,to relax your brain
;I started with station 4 communication
.Outside the exam room, the scenario was
years old female patient admitted by community 80
acquired pneumonia and received antibiotics then
developed delerium,she became confused, agitated
and refusing to receive any treatment and has idea
that she had been poisoned,and she has past history of
.hypertension
The task was to explain the condition and management
.plan to her son
I started by the usual introduction and asked the son
about what he knows about her condition and after
that started gradually to explain the condition to him
and what delerium means and i asked about her
previous history of any memory problems . and the
surrogate said that she has some memory problems
but deteriorated after the pneumonia
the concerns were, is this reversible ,when my mother
will improve and go home, does she will need imaging
for her brain,how can she will receive medications and
.she is agitated and can hurt herself
I tried to reassure him and that we will form MDT from
chest doctor,ICU doctor and neurologist to assess her,
and that the neurologist may decide to give her
medications to relax her and the medical team and
nursing will be with her and so she will not hurt herself
and we can not expect the time of recovery now but
we will do the best effort for her ,I told him that
delerium is common in old people after infection and
that it is reversible, and that the neurologist is the
person who can decide if she will need imaging for
brain or not, and i provide help for her after discharge
.,summery,checked understanding
During discussion the examiner asked me to
summarize the scenario,asked me, is delerium
reversible,i said yes after treatment of the cause.he
asked about causes of delerium ,I said infections as
pneumonia and urinary tract infection ,he asked about
drugs that cause delerium ,i said dieuretics can cause
dehydration and delerium and also antibiotics can
cause this, then he asked about ethical issues in
.scenario and the bell ring
.I did well but I got 12 / 14 ,I do not know why
;Station 5
First scenario outside exam room was, skin rash in 40
.years female patient
Outside exam room i expected that the case will be
psoriasis or pemphigus but when I entered the room it
appears as vasculitic rash on both upper and lower
limbs but i was not sure and afraid to loose the case,
but in the history with the surrogate the patient is on
.treatment for hepatitis c
The concern was is this rash related to her condition, i
said that yes it may be related to hepatitis c by
affection of blood condeu and, the seconed
possibility is a condition called lichen planus which also
may be related to hepatitis c so we will refer her to
skin doctor and her liver doctor to diagnose and give
.treatment
During discussion the examiner asked what is ur
diagnosis ,I said the first possibility is vasculitic rash,he
said which type of vasculitis ,I said cryoglobulinemia,
he said what other DD of this rash , I said lichen planus
.and porphyria cutenea tarda
He asked how to treat cryoglobulinemia ,I said steroids
.,immunosuppressive drugs and treatment of the cause
He asked how to differentiate between this
differentials, I kept silent then the bell ring then I said
skin biopsy ( the two examiners were very helpful and
)with nice smile
.I got 28 / 28
Seconed scenario outside exam room was, weight gain
.in 40 years female patient
when I entered and took history it was a direct case of
cushing syndrome with all evident manifestations of
the syndrome (do not forget to ask about steroids in
this case) and complications of cushing syndrome and
.weight gain as obstructive sleep apnea
The discussion was about DD and how to diagnose and
differentiate between different causes of cortisone
excess, and the last question was which drug is used to
treat diabetes in patient with cushing syndrome, but i
.do not know what the examiner need
.I also got 28 / 28
:Station 1
; Chest
It was a case of right lower lobectomy with fibrosis on
the same side and bronchiactesis on other side with
.also obstructive lung disease
Discussion was about indications of lobectomy,
.diagnosis and managment plan
.I got 18 / 20
Abdomen; it was not good for me and it was also the
most case I do not like during preparation for the exam
.especially detection of presence or abscense of ascites
There was hepatosplenomegaly and during
presentation I said that I can detect ascites by
examination but I want to confirm by ultrasound but
.the examiner was not happy when I said ascites
I told the examiner that I would like to examine lymph
.nodes but there was no time
During discussion the examiner asked about DD of
hepatosplenomegaly, I said chronic liver disease with
all its causes and myeloproliferative disorders, he
asked about how to diagnose HCC if the patient
develped this ,I said basic investigations,alpha feto
.protein and ultrasound
He asked if this patient has lymph nodes what will be
.the cause,I said metastesis from HCC
I was not happy in this case and has some sort of
.mental block
.I got 13 / 18
:Station 2 History
Outside exam room ,the scenario was 25 years old
female patient who had developed skin rash on the
face and forearm which disappeared after receiving
steroids then recurr again after travelling to Luxor and
also she has fatigue during the last 3 months and
.neutropenia
Inside the exam room it is very important to confirm
that it is the same type of rash in the two attacks
during analysis of the rash and during history she has
. ankle and hand joint pain and has one abortion
The diagnosis was SLE with antiphospholipid
.syndrome
The concerns were, Is this blood cancer causing
fatigue?can I get pregnant now?,I said that it is unlikely
to be blood cancer and I explained the diagnosis to her
and I said that she will be referred to MDT including
rheumatologist and obstetrician to give her the best
treatment and to decide about pregnancy because it
.can be affected by her condition
During discussion the examiner asked
What types of skin rash in SLE?, I said photosensitive
.rash, malar rash and discoid rash
What is the distribution of the rash on the face in SLE?,I
.said rash on the face sparing the nasolabial folds
How can u diagnose SLE and what are the most specific
antibodies in SLE?, I said anti dsDNA and anti smith and
.other investigations
Which inflammatory marker is raised in SLE?, I said
.ESR
What is the treatment of SLE?I said the types and drugs
.used in the treatment
.I got 19 / 20
:Station 3
.Neurology; examine the lower limbs
It was also not good case and I did not detect physical
.findings well
There was peripheral neuropathy and I think that there
were also pyramidal manifestations but were not
.clearly evident as usual
During discussion I said peripheral neuropathy and did
not say other thing because I was afraid to say things
not present, but the examiner said what else , I said
.mostly the patient has combinations of lesions
The examiner asked so what ur other diagnosis, I said
MS and also said to him that MS do not cause
.peripheral neuropathy
.He asked about investigations
.I got 12 / 20
Cardiology; It was a direct case of mitral valve
.replacement
Discussion was about indications of mitral valve
replacement, complications of anti coagulants and
management of cases of metallic valves, the last
question was which antibiotic is used for prophylaxis
against infective endocarditis but I said I can not
.remember then the bell rang
.I got 20 / 20
:Finally I got
.Patient welfare 32 /32
.Physical examination 23 / 24
.concern 14 / 16
.clinical communication 13 / 16
.Physical signs 18 / 24
.Differential diagnosis 22 / 28
.Clinical judjement 28 / 32
.Total score 150 / 172
.I hope success for all. with Ahmed Maher Eliwa

Hi all,I would like to share my station


Day 5 round 2 Yangon
Station 1
Resp - Rt.sided pleural effusion with Lt crepts and
rhonchi
)Abd- Hepatosplenomegaly (Thalassaemia
Station 2
yrs old lady with hypertension20
GP did urine dipstick show proteinuria and haematuria.
.Pt also c/o fatigue for 6mths and fever for 3weeks ago
Station 3
CNS- MND
CVS- MVR with vulvotomy scar
Station 4
Talk with granddaughter about 89yrs old rt. sided
weakness. PMH - TIA and AF with wafarin ,fail to follow
up at wafarin clinic
INR 1.2,CT - infarct stroke and no bleed
Task - explain about CT result and refer to stroke unit
Station 5
BCC1 - H&M due to NSAID and prednisolone overdose
with low back pain and ankylosing spondylitis
BCC2 - Peripheral neuropathy with DM and also taking
anti -TB and diet -vegetatrian

UK Experience
Primary experience in Eastbourne District General
Hospital
St_1
Chest: bronchiectasis + old
Abd: severe hsm without stigmata of cld
St_2
History: presyncope... family with valve replacement &
rupture aneurysm
Tall? Marfan? Discussed aortic stenosis hocm
St_3
Cvs: mvr good function with malar rash, ll edema not
cardiac
Neuro: spastic paraparesis but patern of weakness not
clear lt weaker
With loss touch at level umbilicus on rt side
St_4
Comm bbn colectomy for uc 25 years precancerous
colon
St_5
St 5 ... blurring vision rt eye
laser surgeries in eyes, dm 2
Fundus not seen well
Asked if I did light reflex
St 5 fever.... only +ve is diarrhea

Exam experience
On 8th of February 2017
) In Military Medical Academy ( Almaadi - Cairo
Station 3
Cardio
)Two local examiner( Egyptions
After greating examiners, I washed my hands, quickly
having a look at complaint on the wall, Then I greated
the pt, asked for permission, and asked about pain
(crucial for welfare also for identifying physical signs
for example if he
Pt points to the right upper abdomen look for CV
waves on
JVP and other signs of pulmonary HTN including
.pansystolic murmur of tricuspid reg )my had none that
His pulse was regular with average volume,no special
character (again asked about right upper limb before
raising it for examing for collapsing pulse) .Other
general exam was unremarkable. He had
midsterntomy scar,apex was difficult to localised (don't
panic and waste your time on it if you confronted with
)such patients
First heart sound was soft while the second one was .
metalic. There was pansystolic murmur in the apex
with harsh ejection systolic murmur in aortic area that
radiated to the clavicle . Not in heart failure, No signs
of perpheral infective endocarditis or signs of over
.anticoagulation
Then covered and thanked him
Examiner questions .
?What's your findings
As above. In summary this has aortic valve
replacement with mitral regurgitation I would like to
do echocardiography to confirm my diagnosis to assess
valve function because the murmur in aortic area
.radiates up to clavicle
?How will investigate him
Basic investigation including INR, ECG, CXR and
.echocardiography
?How you treat him
.Social, fiancial and psychological support
Counselling about prophylaxis again endocarditis and
.anticoagulation
.Regular follow up with echocardiography
?When endocarditis prophylaxis indicated
Dental work with blood and work on septic area but
not for endoscopy or other procedures
?Target of INR
3-2
? Role of NOAC
.No role of NOAC in metalic valves
?What do you think the cause of his valve problems
Rheumatic heart disease most likely, also could be
bicuspid aortic valve or degeneration I would like to
.know his age
Time finished
I got 20/ 20
Exam experience
Station 2
History
Young man with long standing
Backache .After greating examiners. I greated the
pt/actor, introduced myself, confirmed pt identity and
ageed agenda . I asked him if there is any thing else
(apart from Backache) bothering him he wants us to
?discuss.what you do for living
.I started by open questions
PC lower back pain which is more in the morning with
stiffness last about 15 minutes for three years (he
)didn't seek medical advice only using analgesics
.No H/O trauma
I asked about symptoms of cord compression(to show
.them that I am a safe doctor)there was none
I asked symptoms suggestive of malignancy and
infections
to show them that I am a safe doctor) there was (
.none
Then I asked about other symptoms of Ankylosing
spondylitis (the A'S ) he had H/O red painful eyes , but
didn't know
.What's the diagnosis was. Also he had heel pain
Then I asked about symptoms of other seronegative
.spondyloarthropathy,there was none
PH there was upper GIT bleeding (asked him
)specifically since he was using analgesics
For which he was admitted to hospital. Apart from that
.No PMH of note
FH I asked specifically about FH of Backache or joints
problems .His father has long standing Backache. Also
FH psoriasis .Then I double checked that the pt doesn't
have skin rash(I asked before when asked about
)symptoms of seronegative spondyloarthropathy
)DH only analgesics (paracetamol and codeine
SH
Impact( how his symptoms affecting his life) and
function
.He is not driving
I summaried and asked him if he wants to add anything
.or if we missed something
I asked him what is the cause of his symptoms from his
.point of view
Upon hearing 2 minutes remaining I asked him about
.his concerns
? What I have
. He asked about exercise
I addressed his concerns told him the plan and thanked
him
.Examiner questions.
?What is your diagnosis
This gentleman gave histry of inflammatory Backache
together with heel pain and H/O red painful eye
.)moreover has FH of long standing Backache(his father
?What's your differential diagnosis
Psoriatic spondyloarthropathy but no history of skin
.rash
Entropathic spondyloarthropathy but no history of
.bowel problems
. Reactive arthritis but no symptoms suggestive of that
?How you investigate him
Basic investigation including CBC, LFT, RFT,
inflammatory markers, CXR specially if I am
Considering biological agents, and Xray of the spine
and sacroiliac joints (I told them about possible
)abnormalities specially of the last two
I told them if X ray is normal MRI is more sensitive in
.showing sacroilitis changes
?How you will treat him
.Social, fiancial and psychological support
Physiotherapy, occupational therapy, Exercise incuding
.swimming
.NSAID with PPI after counselling
If he developed perpheral arthritis sulphasalazine will
be helpful but it has no role in spine like other
DMARDS
If pt fails to respond to 2 NSAID then he will be
.candidate for biological agents
?What are they
Anti TNF alpha like adalimumab
.and etanercept
?What are the precautions for them
Excluding
Active bacterial infection and
.Tuberculosis
Bell rang
.I Thanked examiners while leaving the room
I got 18/20
They are not happy about the way I addressed the pt
concern
The examiner said to me you didn't explain the
diagnosis well
.To him. So they gave 2/4 for concerns I believe

UK Barnet Hospital
St2: lady in 40s headache with symptoms of increase
intracranial pressure
S 3: mixed aortic valve +AF ..not sure
Neuro ..cranial nerve examination 5,7,8,12 plasy for
deffrential
Communication: explain SLE and the need for biopsy
for a young lady
S5;1- epileptic pt came with convulsions and headache.
.pregnant off treatment due to pregnancy
Post MI a few weeks ago came with lethargy and -2
.tiredness. .also has dizzy spells
Respiratory; COPD and discussion about the
management
Abd: Renal transplant causes, side effects of
.treatments
.Yangon (9.3.2017) D4R1
Abd. Bilateral polycystic kidney disease Resp. Rt .
collapse consolidation CNS. Spastic paraparesis
CVS ARMRBCC 1.. gritty eyes with weight loss &
tachycardia Grave ophthalmopathy 2.. reduced urine
output in elderly lady taking treatment for knee pain at
rheumatology clinic History.. altered bowel habit..
got t/m at GP for IBS but still not relieved Comm..
delayed disgnosis of pheochromocytoma
Yangon (9.3.2017) D4R2CVS= AS+ or -ARCNS
=sensory T10.spastic paraplegiaResp
=BronchiectasisAbd=Thalassaemia (only
splenomegaly)BCC2 old lady has Fever and
.underlying hypertension
Yangon (9.3.2017) D4R3-St 4- mother admitted with
UTI found out ADPCKD and CKD stage4 talk to
daughter. St5 a, 56 yr old female - blurring of vision.
Diabetic rertinopathy.5.b 56 yr female feeling dizzy
and faint. Hypopit due to snake bite.St1 abd - COL
with HSM and ascities. I didn't get liver. Resp: COPD
with Brochiectasis.St 2- 25 yr female with bloody
diahorrea and fever 3 days. Returned from Thailand.
Uncle has chron disease. -Ddx infective and IBDSt3-
CNS - multiple CN palsy with ptosis - I couldn't finish all
CN exam and DxCVS- MSMR with AF, heart failure
Asalam 3laikom guys
: I am going to share wz u my paces exam experience in
..ALMAADI MILITARY ACADEMY/EYGPT
.nd carousel2 .. 8.2.17
I started with station 5
BCC 1
..Instruction this pt is complaining of inability to walk
I was stressed and confused stood on z lt side of z pt
started asking z surrogate he said it started 1 yr ago
after few questions I turned out that he has backache
and stiffness for more than 2 hrs and he can walk so
I asked about any restriction of movement of spine and
neck he said am not sure so I started z standard
examination of ankylosing spondylitis asked about z
complications but I didn't explore the other
differentials & didn't hear z examiner saying 2 mins
remaining and so missed z concerns
Z first Q by z examiner was did u answer the concern of
this patient I told him am sorry I ran out of time ..
Then what is ur diagnosis and why, DD, investigations
..and ttt
..I thought I totally messed up but got 20/28
BCC2
..Instruction see this 32 yr old man with poor vision
Hx revealed gradual loss of vision for 2 yrs and he has
behcet disease on ttt including warfarin, I did acquity
he can't see on Rt side & can only appreciate hand
movements on z left.. funduscopy pale discs with bony
specules.. Asked about FH said his brother is blind,
tried to screen for syndromes associated with Retinitis
..pigmentosa
concern was: can he regain his vision,, is it related to
..his behcet's, chances of his kids to be affected
Examiner Qs: what is ur diagnosis and could be related
..to behcet and why?? Mode of inheritance, ttt
Got 28/28
station 1
chest
Young average built gentleman with no peripheral
signs
has left thoractomy scar which I saw in the last minute
while examining the back(it wasn't extending to z
side) for me nothing conclusive in the exam apart
from fine basal crackles more on the Rt base with a
scar of an intercostal tube I guess , I had no idea what
to tell the examiner who kept arguing about z site of z
trachea and said how would u explain that z scar is on z
left while trachea was to the Rt
..Q: cause of lobectomy,, inv,, ttt
..It was my worst station
20/13
abdomen
Young boy pale with supraumbilical midline scar
hepatomegaly and resontant splenic bed, no palpable
..lymph nodes
It was thalassemia, discussion went deep on
thalassemia complications and treatment of each,
what u will see in blood film before and after
..splenectomy
20/19
Station 2
Scenario : 29 yr old lady with one week history of fever
..and bloody diarrhoea after she came from Kenya
Inside she stayed there for 3/52 with her husband and
son ate from local food, took full vaccination&
prophylaxis before travel and continued after she came
back, diarrhoea started on the second day of arrival
home bloody frequent with fever, denied any
diarrhoea from before has weight loss no risk factors
..for HIV, has an uncle with crohn's disease
..So I put infective diarrhoea and IBD
Examiner said which is first I answered infective
diarrhoea coz of z short Hx of diarrhoea and travel
..history but could be as well first presentation of IBD
..All discussion was about work up and ttt of IBD
20/20
Station 3
CVS
double valve replacement discussion about the usual
questions, what u will do for him, target INR,
indications of IE prophylaxis and how frequent u want
..to see him in the clinic
Neuro
..Peripheral sensori motor neuropathy
The loss of sensation and weakness were
asymmetrical, knee reflex was persevered on Rt lost on
z lt
I was not happy as z pt was an old man and has
..difficulty obeying my commands
Examiner Q: possible causes I mentioned DM he said
it's not common in this country so I mentioned other
..courses,, inv ,, ttt of diabetic neuropathy
20/17
Station 4
BBN
Long scenario about a 49 yr old gentleman heavy
smoker diagnosed with dialted cardiomyopathy on
maximum medical ttt, his cardiologist said nothing to
be added.. Came now with cervical lymph nodes and
CT abdomen and chest showed renal cellcarcinoma
invadingelse cell invading z capsule with lung and
.. vertebral metasteses
Task was speak to him about diagnosis , prognosis and
..role of specialist nurse on his palliative ttt
So I started by asking him about his health and how is
coping with his heart condition then asked about did
any one told him why these investigations were done
..for him he said no
Then I told him I have the results with me and
unfortunately it is not as we hope... It showed he has a
growth in his kidney... A nasty growth what we call
cancer.. I gave him time to express his feelings .. then
he told me to be honest with him and tell him every
thing and asked me is it curable I told him am afraid it
is not, it has already gone to ur spine and lungs.. So the
ttt now is to keep u comfortable and free of pain but
no cure
Then he asked how much time left for me I told this
..will definitely shorten your life
Then he told his wife will be very depressed if she
knows this & he doesnt know how to tell her I offered
him to bring her with him next visit if he WISHES& will
.. help him telling her
Then he said he wants to travel and enjoy before he
dies I told u will be assessed by heart doctor and
tumour doctor and they r z one to decide, he told me u
r doctor as well tell me I told him am afraid that ur
heart problem might limit ur options and suggest he
can still enjoy around and spend time wz his family
Then he told me I love gardening but I cant take care of
my garden coz of my dyspnea please give me to help
me ( it was mentioned he is on maximum ttt nothing
to be added) I didn't want to hurt him by saying this he
already had enough I guess so instead I told him u can
make gardening a family time to pass ur experience to
ur kids and wife who will cherish this time forever after
you leave
I concluded by referring him to MDT including
oncologist, psychotherapist, social worker and
Macmillan nurse team who will help him and his family
passing this difficult time
..I showed great empathy all through our conversation
I forgot to summarize and check understanding
:Examiner Q's
What z oncologist will do for him: I told pain control
and may be local radiotherapy for vertebral metastes
..then asked any thing else?? Is he for palliative chemo
..If he insisted to travel how u will help him
Do u think u have convinced him and accepted his
..diagnosis
..What is z role of Macmillan nurse team
16/16
Alhamdullah I passed,, I am grateful to every one who
.. took a moment and shared something in this group
..It was of great help for me and others
..Stay blessed

My experience
Yangon, Myanmar
7.3.17
Day 2, Round 1
Station 1
Abdomen - Anaemia with hepatosplenomegaly
Respiratory system - Rt sided pl effusion
Station 2
yr old woman with tiredness, wt gain, amenorrhoea 28
Station 3
CVS - AR
)CNS - Rt sided 3rd N palsy, (pupil sparing
Station 4
CRF - to explain dx and Rx
Station - 5
BCC 1 - systemic sclerosis with breathlessness
BCC 2- Hypothyroidism with chest pain
BEST OF LUCK for all

Myanmar (Yangon) center (8.2.2017) 2nd round


station 3 CVS..1. AS but firstly i wrongly gave dx of MR,
forgot to tell carotid radiation. But examiner lead to
Aortic stenosis. So i told again AS with gallerverdin
.phenomenom and continue management about AS
MS with AFNeuro..1. Spastic para with sensory .2
about T5. Examiner asked about mx but i answered
general things but she still want other differential may
.be MS or Fredrich, i dun know
Flaccid paraplegiaStation 4.. BBN to wife of SAH .2
pt in ICU with taking warfarin for DVT and thiazide for
HT not regularly taking medications. INR- 4. Neuro
team decided that not fit for surgery. Examiner asked
about ventilator.not including task about removal of
ventilator. Concern... is it due to warfarin? Is it
recover? BCC..1. Rheumato clinic for 5 yrs for
bilateral hand jt pain, now complaint of dizziness and
breathlessness..drugs taking methotrexate, ibuprofen,
aspirin, prednisolone, statin, losartan.Come with
malena and collapse.May be postural hypotension
due to malena.or losartan. BCC 2.endocrine clinic for
5 yrs come with vomiting.Taking sterois for 2
yrs.abruptly stop for 1 wk. HypoadrenalismStation
..1 Abd..1.renal transplant with scar having AVF
HepatosplenomegalyRespi..1.bilateral .2
bronchiectasis with clubbing
Rt UL collapse consolidation with pleural effusion .2
History..recurrent chest infection and cough in pt
with DM and asthma not responding to
antibiotics..cystic fibrosis
Myanmar (Yangon) (8.2.2017) last round
Station 1 resp copd Abd. Thalassemia Station 2
IDA diclofenac induced GI loss Family Hx of Ca colon ,
Coeliac Station 3 CVS MVR or AVR? CNS peripheral
neuropathyStation 4 SLE with renal biopsyStation
5 DM HT with headache ....AcromegalyOptic atrophy

:My exam experience


Medical Military Hospital- Cairo- Feb. 7th
:I started with Station 5
Around 50 yo lady c/o confusion over the past 1 -1
;week. She just received chemo 1 week before
After introduction analysis of complaint OCD any
fluctuations? No Memory problems and to recent or
old events ? coping and home and self care? Getting
lost sometimes and forgetting names of familiar ppl
?like you? Any previous episodes
Then asked about DD of delirium like infections ( UTI
and Pneumonia ) pumps and lumps and wt loss?
Abnormal hand movemts ( Parkinsons plus ) and
neurological system specially headaches with increased
.ICP which was negative
Then moved into the chronological hx of cancer and
.chemotherapy
Examination : general survey by checking the pulse, the
eyes for pallor then quick neurological ex like pronator
drift, quick tone and power and plantar reflexes. All
.were negative and I also offered fundoscopy
.Management: admit for IV fluids and brain imaging
Examiners : D.D: Hypercalcemia due to malignancy,
Brain mets. IVF and Bisphosphonates, brain CT to r/o
Mets. Did u reassure the son? Is it reversible ? A:
Hypercalcemia can be corrected but if there are any
.mets the prognosis should be worst
28/28
;Wt loss in 50 yo female -2
Analysis of complaint, OCD , how many KG over how
long? Appetite was good with no diff swallowing, not
.intentional
I asked about my D.D which I found all negative!
..Malignancy TB Endocrinal Chronic infections
I jumped into PMHX: any long standing problems? So:
like what! Any gland problems particularly the gland in
front of her neck ? yes she had HYPOthyroid 2 years
ago and on levothyroxine. Any change in the dose? Is
. she compliant
I then started to ask about autoimmune associations
joint ? rash ? .. then the only thing I picked was
polyuria and polydipsia when asked about DM .. then I
continued any recurrent infections like in the water
work ? no . Any skin infections particularly fungal? No ,
any discharge from the private part ? YES colour smelly
? No answer is it white cheesy like examiner said
!yes! Is it itchy? Examiner said yes
.I maily examined the thyroid no findings
D.D : DM Levothyroxine overdose Polyglandular
.syndrome
28/28
:Station 1
Respiratory : Young patient with lt lateral -1
thoracotomy scar with reduced chest movement and
expansion on the left side. Trachea was mildly shifted
to the rt!! Percussion was heterogeneous on the left
side dull on the left base. Heterogeneous on the rt side
also. VR was reduced on the lt base. There was bibasel
.end insp crackles not changing by cough
On presentation I was confused .. I didnt mention the

say upper or lower ) with bilateral fibrosis ( did not


mention any areas) I basically wanted the examiner to
direct me as the case was quite difficult didnt want
.to invent signs or come up with a wrong diagnosis
Q: Causes for lobectomy ? what are other reasons for
the the scar said oesophageal and cardiac surgeries!
Management and possible cause in this particular
case> said in this young man with lobectomy and
.fibrosis TB is my first differential
20/20
Abdomen : Middle aged woman do u have any -2
pains ? yes where ? here ( right hypochondrium ) I
was like Huh
Started me palpation trying to avoid causing pains, the
abdomen was distended ( ascites ) while trying to
palpate the liver the patient was uncomfortable I
stopped my examination and said : Sir, the patient is in
pain should I proceed ? Procced gently :D
Couldnt get the border of the rt lobe cause of the pain
and ascites, lt lobe was 6-8 cm below xiphsternum.
Spleen was palpable but couldnt get the border/span
.for the same reasons
Presentation : HSM with Ascites , could get the rt lobe
span and same for the spleen :D
D.D : No signs of chronic liver disease could be
Chronic viral hepatitis with Bilhariziasis causing portal
.hypertension and ascites
Discussion about Investigation and went deep into
diagnosing Bilharisiasis said urine/ stool how about
serum ? wanted to hear ELISA . Also asked about the
HBV markers what if the HBSag ve and AB positive :
Pt is immunised.. what if Core +ve : previous infection
! he was smiling and I was too
20/20
Station 2: Hx- difficulty moving over the past one week.
.He had hx of radiotherapy for lung cancer
I think I got the full mark in this one cause of deep
analysis of complaint ( back pain ) which I got the
incontinence problems then I went deep into how
this is affecting his life and the embarrassment he
suffers with. I stopped at this point showing a lot of
empathy and sympathy as if its a communication
station also asked about occupational hx and
.asbestos exposure
Dx: cord metastasis with compression symptoms
urgent admission - IV dexamethasone and urgent
.neurosurgical review
20/20
:Station 3
Neurology: Rt sided hemiparesis with Rt -1
.hemihpothesia
Discussion was about the possible causes ( thrombo-
embolic ) and site of lesion ( r tint. Capsule ) and
) management ( in acute and chronic presentation
20/20
Cardiology : was a difficult case, Middle aged -2
female, raised JVP and when I asked about abdominal
pains she said yes. I asked the examiner to do the
hepato-jagular reflex he said no need. By then I knew
the patient had TR! I could hear the systolic murmur of
TR which increased on inspiration, I also Heard systolic
murmur over the apex ( not radiating to the axilla ) and
on aortic area ( not reading to carotids and no palpable
) thrill
I then decided to only mention the surest finding ( TR )
and give differentials for the others, trying to avoid
wrong diagnosis or invented signes. ( wanted to get
) some marks better than loosing it all
The examiner asked about diagnoses ( TR ) what causes
( Rheumatic ) he was not satisfied and asked whether
RF affects the tricuspid valve? I said rarely as it
commonly affects the mitral then the most common
etiology for isolated TR should be pulmonary HTN, he
was then a bit satisfied and the discussion went over
the pulmonary htn 1ry and 2ry causes , inv and
.managements
Seems like I missed the loud P2 or a parasternal heave (
the P2 was loud but I didnt want to mention it as I was
) not quite sure
Got 13/20
Station 4 : 50 yo lady collapsed at home the previous
night. Admitted to hospital; with HB of 7.4 g/dl ,
received BT and now feeling better. She is on Ibuprofen
for a long time. Her husband is at home with a recent
.MI and is diabetic who needs someone to take care of
Task: convince the patient to stay at hospital and to
.offer any investigations you may see appropriate
So I realised I needed to convince the patient to stay
for upper GI endoscopy for possible bleeding d.t
.ibuprofen
Introduction , confirm identity , breaking the ice ( hope
uve not been waiting for me for long examiner was
happy nodding his head! )then confirming agenda
exploration and expectations and possible initial
.concerns
She said shes fine now and wants to go home to take
care of her husband, she new she had BT and her HB
was quite low, She had no idea about any possible
.cause
I started to explain possible bleeding and the need for
further inv. When I mentioned the ( camera test ) she
was annoyed and strongly refused it I then realized I
!was on the track
Started to explore the reasons why she refuses she
had a previous bad experience with an endoscopy 7
years ago. She stopped. I asked again ( with a smiley
face ) would u plz tell me more what happened and
what kind of pressures u had ? she said there was no
privacy, uncomfortable as I could feel it all and
!eventually they found nothing
I reassured her regarding every point ( the privacy
skilled doctors more anaesthesia ) and will ask the
.social worked to take care of ur husband
She finally agrees can explain the procedure for you
..I demonstrated with drawing
asks whether I could have discharged her then she
comes back later why? My husband! I reinforced
again about the social services we have and
explained we wont guarantee what happens if she
goes home and we need to know whats going on
.before discharge
She was concerned about any other possible cause, ( I
felt she was asking about cancer ) I asked are u
concerned about any particular condition she said
cancer I said that cant be ruled out I am afraid, but
we need to investigate the most probable cause first
which is the possible bleeding from your stomach due
.to the painkillers
She now agrees and has no further concerns ,
Summarized then said ( I will have another meeting
with u with my consultant after the procedure and may
be we need to do further investigations according to
the endoscopy results ) then told her about the
.consent which she agreed to sign
Examiner: what is the case then asked whether she
may have cancer I said probably asked did u ask
about the alarm signs? Said NO for two reasons; I
didnt want to go into hx taking in a communication
station and I wanted to fulfil my task to convince her to
stay for the upper GI endoscopy. Do you feel any inv.
Should has been furtherly discussed, said yes
Colonoscopy and thats why I told her I am going to
have another discussion with u after the endoscopy :D
Examiner was happy then asked; what if she refused? I
said in that case theres an ethicl issue which is
patients autonomy and I have to respect her wish, on
the other hand Beneficiance should be fulfilled by
trying to convince her to go for it for her best interest.
!He was then smiling
16/16
Got 165/ 172
.Wish you all the best of success
Hi good evening My experience in kilmarnock
crosshouse hospital
Station 1 resp: right sided thoracormy scar with left
sided fine end inspiraory basal crepts diagnosis right
lobectomy due to fibrosis
Abdomen Patient with black pigmentation on hands
dorsal surface with two dialysis fostula on both arms.
no marks of recent use. want to see gum hypertrophy
but he was wearing dentures. was pale. having
laparotmy scar cvp line scar in neck. i cant palpate
masses gave diagnosis of ESRD with renal trnplant.
exmainer was not happy. this patient came in previous
.attempts i think its there chornic patient
St2 55 Y old male smoker DM on metformin and
sitagliptin poor control had MI and atrial fibrialltion
with DVT few years back taking betablocker also. latest
BP sitting 146.78 standing 120.81 INR 1.5 came with
recent onset of dizzy spells for 3 week woth chest
tightness and palpitipns Gave DD of postural hypo
autonomic neuropathy drug induced hypoglymic
attacks cardiac arthmias P.E
Station 3 CVS MVR patient with metallic S1 best heard
in mitral area. Viva was usual. examiner asked me
types of metallic valves
CNS parkinson disease command was patient with falls
examine do appropraite exam. there was tremors
more marked in right side blefroclonus
shufling gait stoped posture cog wheel lead pipe ridigt
masked face with slow speech. viva usual. examiner
asked indocation of MRI or CT in parkinso diseaze
station 4
Y old female nurse came with loss of consousness 33
found to have hypoglecemia she is not DM. high C
peptide junior staff told her it is insulinoma later urine
came positive for sulphonylurea overdose
your task to tell her diagnosis and try to convince her
for phychiatric assessment
i totally messed it up. exmainer told me she had fight
with husband whonis diabetic and she took gliclazide
tabs for suicide
station 5 patient with pain in hands and dysphagia had
typicsl features of Systemic sclerosis viva typical
next patient was 28 y old male with 6 month
intermiettnt hematuria urine positive for RBCs protein
negative i gave DD

Myanmar, Yangon Center (7.3.2017) last round


Resp: collapse consolidation with finger clubbing
Abd: jaundice with hepatosplenomegaly
CNS: spastic paraplegia
History: chest pain.... musculoskeleton pain history of
trauma present
Family history of MI, obesity, hyperlipemia present
Communication: uncertain diagnosis of CA pancreas
BCC1: know RA patient with proximal myopathy and
Cushingnoid face, taking steriod
BCC2: polyuria and polydipsia
Pt has blood disorder and taking blood transfusion for
thalessemia

Hi good evening.... I would like to share my


experiences in London feb 2017
Started with a station 2 young professionals with
history of losses motion for 6 month ..his concern is it
cancer
Station 3 CVS MVR
CNS pt was in wheel chair
Next station 4 was talk to the husband of a pt which
was admitted due to non STMI recived aspirin and she
had allergies to aspirin previous she had GIT upset due
to aspirin and told is allergy to her she advice for
station aspirin plavix lansoprazol pt refused to take
medicines
Station 5 hemochromatosis with DM presented with
....hand pain
Epistaxsis_2
Station 1 chest was not good to me pt was with oxygen
calender and had sternotomy scar next to him was
anticoagulant yellow card...present with SOB I hear
only fine crakels at base .... they ask what you do what
diagnosis
Abd was young African pt with hepatospleenmegaly

)almaadi military hospital second carousel. (8.2.2017


st 4 ; an old man k/c of advance heart failure did
lymphnode biopsy showed Rcc with vertebral mets on
bone scan for palliative care consultation went bbn to
palliative care to consern he has 3 consern 1-he has
acaring wife &he didnt want to tell her bec as she may
become deppressd I said we can help you in this by
telling her e great empathy &our psychologist can help
her if he agree because you need her support 2- how
long he will live as he want to travel abroad, he didn't
enjoy his life I told him yr ca is so advance &is likely to
shorten yr life he ask for estimation isaid monthes
rather than year then he said can you give meds to
prolong my survival to travel I said travel is not good
4you bec of Hf &yr ca (this taken against me ) 3-he has
agarden &he want to continue gardening Isaid I.ll refer
you to OT then isummarize &check understanding ,
offer help. In discussion summarize yr case .why you
insist to tell his wife & not respecting his autonomy ;
bec he is need of her support &I ask him if he agree .
Why you didnt allow him to travel isaid bec he may
deteriorate &of hf he said if he insist how you will
solve problem I said he need to tell his health
insurance &for his hf ikept silent he said assessment
for fitness to fly then he ask can we give meds to
prolong survival of Rcc I said am aware of new meds
used in advance Rcc but couldn't remember he said
why you didnt offer it for him isaid may be he will ask
for it &i have no information about it he laugh then
bell rang alhamdellah iwent out thinking v.bad about
my self as didn't answer his
Consern well but igot 14
St5: first case is young male e difficulty in walking no
other information I was completely disorganized ithink
about neuro inside problem for 2yr no neuro
symptoms I ask 4examination he said ok iask pt to raise
his leg he raised them well again v.bad feelig second
case is lost the it come to my mind to see his gait he
said ok at that time I recognized question mark posture
I ask is he has pain yes in lower back alhamdellah AS
isaid i want to do occiput walltest he said do it then I
ask about other symptom &assoc then iexamine cvs ,
resp .I ask 4 schoper test he said do it again ilet pt to
stand idid it improperly iforget to check for tenderness
& to ask for chest exp bec of time concern what is the
course &need tretment I answer we will do blood test
&image & refer to dpecilized dr iremeber the job in the
last second &he is adriver alhamdellah questions about
diagnosis inv&management really am thinking about
20 but i got 28 . Second case is young male blaring of
vision again no other information inside it is chronic it
started at night & progressd colour vision affected so it
come to my mind RP iask for exam he said do then V/A
he can.t see even moving fingers going to fundus with
typical RP then iask about assoc nothing is there he has
positive fh of blindness his brother iask about med list
serugate said steriod &immunosuppressant isaid 4
what she said for behecit dis I ask e what he presented
she didn't understand me the other examiner answer
me e DVT is he ok now regarding his behecit &no
problem e meds she said yes .consern is it related to
behecit & about his kids .questions is about behecit the
examiner is v.tough he ask why eye signs is not related
to behecit .what behecit can cause in the eye I said
pale disc due to sagittal thrmbosis he is not happy he
want other thing but my mind is stocked why is asking
about behecit &forget RP he gave bad impression
about this &I think he will give me 16 or some thing but
alhamdellah he gave me 26 . Iwill write other statoin in
separate post because ihave problem in my mobile
.rgarding long post
St 1 resp young male with pruductive couph he look
healthy when i ask him to couph it simulate the couph
of copd his chest exam is completly normal only
hyperexpanded chest couldn't appreciate any thing
else ipresent the casevas copd he said where is the
trachea isaid slightly deviated to the right he said what
is the cause of couph isaid infective exacerbation then
he ask about how to diagnose .common organisms.NIV
&LTOT when iwent out one of my colleque he
diagnosed him with pleural effusion ifeel v.bad about
my self how I diagnose copd only with character of
couph &hyperinflation so again this xase is lost
completely ithink he will give me 8but surprisingly igot
. 19
Abdomen :cld male pt pale & jaundiced e parotid
swelling enlarged lt liver lobe .splenomegaly &ascites
&ll odema discussion about causes , diagnosis .acute
.presentation &treatment of varceal bleeding &SBP
St2 young lady e bloody diarrhea e h/o travel to kenya
inside it is for 3week since she came about 12time/day
.fever.shivering .wt loss no other symptoms positive fh
of CDher cousin her family travel e her &are ok & she is
on lopermide from her Gp consern is what is the cause
I said mostly cd but am going to admit you doing blood
test &images to exclude others& to stop lopermide
time is finished before discussing mangement the
examiner again very tough asking about diff igave cd ,
infective diarrhea he said what else isaid ca but far as
she is young &no fh he said what else ikept silent he
said what about malaria as could be but far for bloody
diarrhea like this he ask why you want to admitt isaid
bec of severe colitis according to true love criteria je
said you didnt assess the volume &he is right I forget to
ask again ithink he will give me 14 or15 but he is
generous he gave me 19 alhamdellah
St3 which is my disaster since my previous attempt
neuro aman e difficult y in walking examine LLon
screening he is unable to raise his ul.&ll so hemiplegia
e hemesensory loss I finished early he ask what you
want to examine I said CN he said ok idid 7 , 12both are
affected he said what else isaid heart he said no need
then what isaid carotid he said no need what else isaid
gait as if he is waiting for this question is present yr
finding .diagnosis .dd.inv if he came in acute attack
meds given acutely .if AF what you will give isaid
warfarin he said can you give noac isaid yes he said if
Af isaid no give me samples of noac i gave 3 I got 20
.cvs alady e sob she has small volume pulse reg , raised
jvp , midline sternotomy scar mvr , TR loud p2 ll
oedema , ESM In AA not radiated ididnt mentioned it in
presentation but he ask for it questions about couses
.complication of valve replacement .INR &SBE
alhamdellah igot 20
UK Exam
was in the western general hospital in 3/3/2017
St_1
Chest case with fine crepitations and cushinoid
features I said ILD
Abd with bilateral right and left IF scars with renal
transplant twice
St_3
Cardio mid sternotomy scar old age with metalic click
with second sound
Neuro has partial third with internuc ophthalmoplegia
my first possibility was ms
St_5
BCC1 goitre with no peripheral stigmata he asked me
what have you found clinical signs and how to
investigat
BCC2 case with LMNL 7 th and they asked me why you
didnt examine the 4.5.6 and unfortu I examined only
7.8.3
St_2
Hist
is anemia with family hist of cancer colon
St_4
communication
BBN=Pt with mass in x ray and the task is to tell him
further investigations and possibility of cancer and
patient was insisting to get false hope

My experience in PACES exam 1/2017:


I had the exam in Cairo,Kasr Eleiny 3rd carousel
6/2/2017 ,
I got 150 / 172 ,
First I would like to thank our brother and teacher dr
Ahmed Maher Eliwa and all people who supported us
and I appreciate your efforts
with us for learning and success.
My advice to all doctors who will enter the exam in
the future as dr
Shaimaa Mohamed Abdou said is not to put any
previous failure in your mind and try to sleep well
before the exam ,and also try not to read or revise any
thing the day before the exam ,to relax your brain.
I started with station 4 communication;
Outside the exam room, the scenario was.
80 years old female patient admitted by community
acquired pneumonia and received antibiotics then
developed delerium,she became confused, agitated
and refusing to receive any treatment and has idea
that she had been poisoned,and she has past history of
hypertension.
The task was to explain the condition and
management plan to her son.
I started by the usual introduction and asked the son
about what he knows about her condition and after
that started gradually to explain the condition to him
and what delerium means and i asked about her
previous history of any memory problems . and the
surrogate said that she has some memory problems
but deteriorated after the pneumonia,
the concerns were, is this reversible ,when my mother
will improve and go home, does she will need imaging
for her brain,how can she will receive medications
and she is agitated and can hurt herself.
I tried to reassure him and that we will form MDT
from chest doctor,ICU doctor and neurologist to
assess her, and that the neurologist may decide to give
her medications to relax her and the medical team
and nursing will be with her and so she will not hurt
herself and we can not expect the time of recovery
now but we will do the best effort for her ,I told him
that delerium is common in old people after infection
and that it is reversible, and that the neurologist is the
person who can decide if she will need imaging for
brain or not, and i provide help for her after
discharge ,summery,checked understanding.
During discussion the examiner asked me to
summarize the scenario,asked me, is delerium
reversible,i said yes after treatment of the cause.he
asked about causes of delerium ,I said infections as
pneumonia and urinary tract infection ,he asked
about drugs that cause delerium ,i said dieuretics can
cause dehydration and delerium and also antibiotics
can cause this, then he asked about ethical issues in
scenario and the bell ring.
I did well but I got 12 / 14 ,I do not know why.
Station 5;
First scenario outside exam room was, skin rash in 40
years female patient.
Outside exam room i expected that the case will be
psoriasis or pemphigus but when I entered the room
it appears as vasculitic rash on both upper and lower
limbs but i was not sure and afraid to loose the case,
but in the history with the surrogate the patient is on
treatment for hepatitis c.
The concern was is this rash related to her condition,
i said that yes it may be related to hepatitis c by
affection of blood condeu and, the seconed,
possibility is a condition called lichen planus which
also may be related to hepatitis c so we will refer her
to skin doctor and her liver doctor to diagnose and
give treatment.
During discussion the examiner asked what is ur
diagnosis ,I said the first possibility is vasculitic
rash,he said which type of vasculitis ,I said
cryoglobulinemia, he said what other DD of this rash ,
I said lichen planus and porphyria cutenea tarda.
He asked how to treat cryoglobulinemia ,I said
steroids ,immunosuppressive drugs and treatment of
the cause.
He asked how to differentiate between this
differentials, I kept silent then the bell ring then I said
skin biopsy ( the two examiners were very helpful and
with nice smile)
I got 28 / 28.
Seconed scenario outside exam room was, weight gain
in 40 years female patient.
when I entered and took history it was a direct case of
cushing syndrome with all evident manifestations of
the syndrome (do not forget to ask about steroids in
this case) and complications of cushing syndrome and
weight gain as obstructive sleep apnea.
The discussion was about DD and how to diagnose
and differentiate between different causes of cortisone
excess, and the last question was which drug is used
to treat diabetes in patient with cushing syndrome,
but i do not know what the examiner need.
I also got 28 / 28.
Station 1:
Chest ;
It was a case of right lower lobectomy with fibrosis on
the same side and bronchiactesis on other side with
also obstructive lung disease.
Discussion was about indications of lobectomy,
diagnosis and managment plan.
I got 18 / 20.
Abdomen; it was not good for me and it was also the
most case I do not like during preparation for the
exam especially detection of presence or abscense of
ascites.
There was hepatosplenomegaly and during
presentation I said that I can detect ascites by
examination but I want to confirm by ultrasound but
the examiner was not happy when I said ascites.
I told the examiner that I would like to examine
lymph nodes but there was no time.
During discussion the examiner asked about DD of
hepatosplenomegaly, I said chronic liver disease with
all its causes and myeloproliferative disorders, he
asked about how to diagnose HCC if the patient
develped this ,I said basic investigations,alpha feto
protein and ultrasound.
He asked if this patient has lymph nodes what will be
the cause,I said metastesis from HCC.
I was not happy in this case and has some sort of
mental block.
I got 13 / 18.
Station 2 History:
Outside exam room ,the scenario was 25 years old
female patient who had developed skin rash on the
face and forearm which disappeared after receiving
steroids then recurr again after travelling to Luxor
and also she has fatigue during the last 3 months and
neutropenia.
Inside the exam room it is very important to confirm
that it is the same type of rash in the two attacks
during analysis of the rash and during history she has
ankle and hand joint pain and has one abortion .
The diagnosis was SLE with antiphospholipid
syndrome.
The concerns were, Is this blood cancer causing
fatigue?can I get pregnant now?,I said that it is
unlikely to be blood cancer and I explained the
diagnosis to her and I said that she will be referred to
MDT including rheumatologist and obstetrician to
give her the best treatment and to decide about
pregnancy because it can be affected by her
condition.
During discussion the examiner asked,
What types of skin rash in SLE?, I said photosensitive
rash, malar rash and discoid rash.
What is the distribution of the rash on the face in
SLE?,I said rash on the face sparing the nasolabial
folds.
How can u diagnose SLE and what are the most
specific antibodies in SLE?, I said anti dsDNA and
anti smith and other investigations.
Which inflammatory marker is raised in SLE?, I said
ESR.
What is the treatment of SLE?I said the types and
drugs used in the treatment.
I got 19 / 20.
Station 3:
Neurology; examine the lower limbs.
It was also not good case and I did not detect physical
findings well.
There was peripheral neuropathy and I think that
there were also pyramidal manifestations but were
not clearly evident as usual.
During discussion I said peripheral neuropathy and
did not say other thing because I was afraid to say
things not present, but the examiner said what else , I
said mostly the patient has combinations of lesions.
The examiner asked so what ur other diagnosis, I said
MS and also said to him that MS do not cause
peripheral neuropathy.
He asked about investigations.
I got 12 / 20.
Cardiology; It was a direct case of mitral valve
replacement.
Discussion was about indications of mitral valve
replacement, complications of anti coagulants and
management of cases of metallic valves, the last
question was which antibiotic is used for prophylaxis
against infective endocarditis but I said I can not
remember then the bell rang.
I got 20 / 20.
Finally I got:
Patient welfare 32 /32.
Physical examination 23 / 24.
concern 14 / 16.
clinical communication 13 / 16.
Physical signs 18 / 24.
Differential diagnosis 22 / 28.
Clinical judjement 28 / 32.
Total score 150 / 172.
I hope success for all.

Ygn - 1st day , (6.3.2017) 1st round St 5: nf , c/o


headache , Htn Phaecomocytoma & migrane ( z z line
b4 unilateral headache )Tia : vision lossSt 1:
hepatomegaly e v Long scar on right
sideConsolidation with brochitesis St 2: bloody
d - IBD , INFECTIVE ST 3: indian boy - myotonic
dydrophy Cvs : AS ? , ASD ? St 4: ILD with
amiodarone induced
Myanmar, Yangon (6.3.2017) 1st day 3rd round
st4 menigococcal septicaemia explain condition of
husband Q: can b cure any disablitywhat should I
do for my childernshould I inform to his
officest2 back pain straight forward
h/oankylosing spondylitis family history of
psoriasisexaminar Q : dx ddx mmCVS MS MR
with ht failure MS dorminantexaminar Q : which is
dorminant m/m diff b/t MS MRCNS rt LMN
facial palsy 8 th palsy Q: finding ddx invxresp:
difficult not enough time dullness in rt lower zone
reduced VBS/VF dullness in rt up zone but I heard
nothing give ddx of dullness in rt lower zone other
candidate also didn't notice up zone examinar ask
what do u think about up zone? bad feeling for this
stationabd: deep jaundice up ward enlargement of
liver no sign of chronic Q: ddx invx hep B tmBCC
1 chronic RA with H/ T 5 yr breathlessness for 1mth Bp
160/120 PR 120 spo2 92 sugar 192 RR 18on exam:
fine basal creps not change with cough I think give R
nodule both up limb with limitation of movementdx
p.fibrosis ask another dx hypertensive ht failure
another candidate said OA with ht failureBCC 2
Dizziness with DM on metformin hypoglycemic attack
postural hypodx addisonanother candidate not
same case for CNS foot drop Abd huge splenomegaly
)Ygn - 1st day , 1st round ( my experience
St 5: nf , c/o headache , Htn
Tia : vision loss
St 1: hepatomegaly e v Long scar on right side
Consolidation with brochitesis
St 2: bloody d - IBD , INFECTIVE
ST 3: indian boy - myotonic dydrophy
? Cvs : AS ? , ASD
St 4: ILD with amiodarone induced

This Exam Experience was published before by My


colleague Dr.Lai Yee ,,,but this copy is Well-Organized
EGYPT ,,, Febr. 2017
Hellow Drs am going to write my exam experience.
..actually at that day I had flu and I feel may I couldn't
concentrate bez of that and bez this is first trial for me
igave my self big power try to pass
I started with st3 neuro
it was hemplegia I asked to examined lower limb from
his posture I see it was hemplegia ifiished lower then I
asked the examiner to iwant to examin gait upper and
cardio and iwant to examined cranial he told me just
look at his face of course there's mouth devation then
he start of gave me guestion where the lesion and
investigation and managment really the most
important is to keep good impression from examiner
about how yu comunicate with the pts during yu
examin contine tell every thing yu will do and yur eye
on pt face while examining him it's difficult but try yur
best don't forget covering and thanks him behaviour is
important not only infomation iget20/20
Shifting to cardio
my case metallic aortic replacement with ejection
systolic murmur for echo and mitral regur then he ask
about how yu will investigate how yu will manage and
about INR and about finding in echo get20/20
I go comunication
it was acceptable mother want yu talk to her daughter
she has possibility of cholangiocrcinomer or cancer
head of pancreas but still not found in ct oe ercp and
the mother she has feature of obstructive joundice and
on pain I started to to talk to daughter about her
mother she cry a lot that she felt guilty bez she delay
briging her mother to hospital and her concern that if
cancer not tell her mother about that bes she felt
delaying not bringing her mother early is the cause and
she was really upset but I proceed as I was learn to give
empathy and polity talk about mother right if she ask
bez mother competence she want also discharge her
mothers bez share is busy ienterfer with social worker
and we cannot discharge her bez she need admition
mangining her pain and so on then I made closing
summary and understanding get16/16
Then I started history
young man with back pain I did well but only giving
me13/20 really I don't knowwhy I think it was
straightforward history and iput nice dd going with his
family history father has psoriasis grand mother had
back pain in his history he has chronic eye
inflammation iput inflammatory ankylosing type back
pain then iput
Then I go st5
really case not difficult but time very very short start
with reumatoid hands complicated with rt knee
osteoarthritis I take history then I felt time go soonly I
try difficulty to examind hand and rt knee very badly in
interupted way am afraid then I start to manage her
concern not finish it the bell rangI felt badly and I
answered all his guestions but really I felt down but I
told my self forget get25/28
The next st5
headache really I started it trying to manage my fault I
take history and as same tme I start to examind then
really I put hard work to take high score in this bez I
think I did badly in first oneI did good complete i take
28/28
All history is negative only i found the clue when asked
family history of haemorrhage in head peace and
history of kidney disease is that her mother on dialysis
so I put in mind subarachnoid haemorrhage
so also I advice to make emage for her head and
kidneys to exclude
The Examiner ask me before surgery how can yu help
???her
The Examiner want to hear Nimodopine then ask me
about group of it and the dose and also he ask me
what and when yu do CSF and what yu will find he
want Xanthochromia after 12 hours
Then i go st1
chest copd with corpulmona
Abdomen= 20/20 abd hepatomeglly with splenectomy
examiner kind asking dd
and managment 18/20
at end thanks God I pass score160/172

:My exam experience


Medical Military Hospital- Cairo- Feb. 7th
:I started with Station 5
Around 50 yo lady c/o confusion over the past 1 -1
;week. She just received chemo 1 week before
After introduction analysis of complaint OCD any
fluctuations? No Memory problems and to recent or
old events ? coping and home and self care? Getting
lost sometimes and forgetting names of familiar ppl
?like you? Any previous episodes
Then asked about DD of delirium like infections ( UTI
and Pneumonia ) pumps and lumps and wt loss?
Abnormal hand movemts ( Parkinsons plus ) and
neurological system specially headaches with increased
.ICP which was negative
Then moved into the chronological hx of cancer and
.chemotherapy
Examination : general survey by checking the pulse, the
eyes for pallor then quick neurological ex like pronator
drift, quick tone and power and plantar reflexes. All
.were negative and I also offered fundoscopy
.Management: admit for IV fluids and brain imaging
Examiners : D.D: Hypercalcemia due to malignancy,
Brain mets. IVF and Bisphosphonates, brain CT to r/o
Mets. Did u reassure the son? Is it reversible ? A:
Hypercalcemia can be corrected but if there are any
.mets the prognosis should be worst
28/28
;Wt loss in 50 yo female -2
Analysis of complaint, OCD , how many KG over how
long? Appetite was good with no diff swallowing, not
.intentional
I asked about my D.D which I found all negative!
..Malignancy TB Endocrinal Chronic infections
I jumped into PMHX: any long standing problems? So:
like what! Any gland problems particularly the gland in
front of her neck ? yes she had HYPOthyroid 2 years
ago and on levothyroxine. Any change in the dose? Is
. she compliant
I then started to ask about autoimmune associations
joint ? rash ? .. then the only thing I picked was
polyuria and polydipsia when asked about DM .. then I
continued any recurrent infections like in the water
work ? no . Any skin infections particularly fungal? No ,
any discharge from the private part ? YES colour smelly
? No answer is it white cheesy like examiner said
!yes! Is it itchy? Examiner said yes
.I maily examined the thyroid no findings
D.D : DM Levothyroxine overdose Polyglandular
.syndrome
28/28
:Station 1
Respiratory : Young patient with lt lateral -1
thoracotomy scar with reduced chest movement and
expansion on the left side. Trachea was mildly shifted
to the rt!! Percussion was heterogeneous on the left
side dull on the left base. Heterogeneous on the rt side
also. VR was reduced on the lt base. There was bibasel
.end insp crackles not changing by cough
On presentation I was confused .. I didnt mention the

say upper or lower ) with bilateral fibrosis ( did not


mention any areas) I basically wanted the examiner to
direct me as the case was quite difficult didnt want
.to invent signs or come up with a wrong diagnosis
Q: Causes for lobectomy ? what are other reasons for
the the scar said oesophageal and cardiac surgeries!
Management and possible cause in this particular
case> said in this young man with lobectomy and
.fibrosis TB is my first differential
20/20
Abdomen : Middle aged woman do u have any -2
pains ? yes where ? here ( right hypochondrium ) I
was like Huh
Started me palpation trying to avoid causing pains, the
abdomen was distended ( ascites ) while trying to
palpate the liver the patient was uncomfortable I
stopped my examination and said : Sir, the patient is in
pain should I proceed ? Procced gently :D
Couldnt get the border of the rt lobe cause of the pain
and ascites, lt lobe was 6-8 cm below xiphsternum.
Spleen was palpable but couldnt get the border/span
.for the same reasons
Presentation : HSM with Ascites , could get the rt lobe
span and same for the spleen :D
D.D : No signs of chronic liver disease could be
Chronic viral hepatitis with Bilhariziasis causing portal
.hypertension and ascites
Discussion about Investigation and went deep into
diagnosing Bilharisiasis said urine/ stool how about
serum ? wanted to hear ELISA . Also asked about the
HBV markers what if the HBSag ve and AB positive :
Pt is immunised.. what if Core +ve : previous infection
! he was smiling and I was too
20/20
Station 2: Hx- difficulty moving over the past one week.
.He had hx of radiotherapy for lung cancer
I think I got the full mark in this one cause of deep
analysis of complaint ( back pain ) which I got the
incontinence problems then I went deep into how
this is affecting his life and the embarrassment he
suffers with. I stopped at this point showing a lot of
empathy and sympathy as if its a communication
station also asked about occupational hx and
.asbestos exposure
Dx: cord metastasis with compression symptoms
urgent admission - IV dexamethasone and urgent
.neurosurgical review
20/20
:Station 3
Neurology: Rt sided hemiparesis with Rt -1
.hemihpothesia
Discussion was about the possible causes ( thrombo-
embolic ) and site of lesion ( r tint. Capsule ) and
) management ( in acute and chronic presentation
20/20
Cardiology : was a difficult case, Middle aged -2
female, raised JVP and when I asked about abdominal
pains she said yes. I asked the examiner to do the
hepato-jagular reflex he said no need. By then I knew
the patient had TR! I could hear the systolic murmur of
TR which increased on inspiration, I also Heard systolic
murmur over the apex ( not radiating to the axilla ) and
on aortic area ( not reading to carotids and no palpable
) thrill
I then decided to only mention the surest finding ( TR )
and give differentials for the others, trying to avoid
wrong diagnosis or invented signes. ( wanted to get
) some marks better than loosing it all
The examiner asked about diagnoses ( TR ) what causes
( Rheumatic ) he was not satisfied and asked whether
RF affects the tricuspid valve? I said rarely as it
commonly affects the mitral then the most common
etiology for isolated TR should be pulmonary HTN, he
was then a bit satisfied and the discussion went over
the pulmonary htn 1ry and 2ry causes , inv and
.managements
Seems like I missed the loud P2 or a parasternal heave (
the P2 was loud but I didnt want to mention it as I was
) not quite sure
Got 13/20
Station 4 : 50 yo lady collapsed at home the previous
night. Admitted to hospital; with HB of 7.4 g/dl ,
received BT and now feeling better. She is on Ibuprofen
for a long time. Her husband is at home with a recent
.MI and is diabetic who needs someone to take care of
Task: convince the patient to stay at hospital and to
.offer any investigations you may see appropriate
So I realised I needed to convince the patient to stay
for upper GI endoscopy for possible bleeding d.t
.ibuprofen
Introduction , confirm identity , breaking the ice ( hope
uve not been waiting for me for long examiner was
happy nodding his head! )then confirming agenda
exploration and expectations and possible initial
.concerns
She said shes fine now and wants to go home to take
care of her husband, she new she had BT and her HB
was quite low, She had no idea about any possible
.cause
I started to explain possible bleeding and the need for
further inv. When I mentioned the ( camera test ) she
was annoyed and strongly refused it I then realized I
!was on the track
Started to explore the reasons why she refuses she
had a previous bad experience with an endoscopy 7
years ago. She stopped. I asked again ( with a smiley
face ) would u plz tell me more what happened and
what kind of pressures u had ? she said there was no
privacy, uncomfortable as I could feel it all and
!eventually they found nothing
I reassured her regarding every point ( the privacy
skilled doctors more anaesthesia ) and will ask the
.social worked to take care of ur husband
She finally agrees can explain the procedure for you
..I demonstrated with drawing
asks whether I could have discharged her then she
comes back later why? My husband! I reinforced
again about the social services we have and
explained we wont guarantee what happens if she
goes home and we need to know whats going on
.before discharge
She was concerned about any other possible cause, ( I
felt she was asking about cancer ) I asked are u
concerned about any particular condition she said
cancer I said that cant be ruled out I am afraid, but
we need to investigate the most probable cause first
which is the possible bleeding from your stomach due
.to the painkillers
She now agrees and has no further concerns ,
Summarized then said ( I will have another meeting
with u with my consultant after the procedure and may
be we need to do further investigations according to
the endoscopy results ) then told her about the
.consent which she agreed to sign
Examiner: what is the case then asked whether she
may have cancer I said probably asked did u ask
about the alarm signs? Said NO for two reasons; I
didnt want to go into hx taking in a communication
station and I wanted to fulfil my task to convince her to
stay for the upper GI endoscopy. Do you feel any inv.
Should has been furtherly discussed, said yes
Colonoscopy and thats why I told her I am going to
have another discussion with u after the endoscopy :D
Examiner was happy then asked; what if she refused? I
said in that case theres an ethicl issue which is
patients autonomy and I have to respect her wish, on
the other hand Beneficiance should be fulfilled by
trying to convince her to go for it for her best interest.
!He was then smiling
16/16

Got 165/ 172
.Wish you all the best of success
Am going to write my exp in paces my exam was on
.almaadi military hospital second carousel
First of all I want to thanx dr.Ahmed Maher Eliwa for
his great & generous support
Istart e st 4 ; an old man k/c of advance heart failure
did lymphnode biopsy showed Rcc with vertebral mets
on bone scan for palliative care consultation went bbn
to palliative care to consern he has 3 consern 1-he has
acaring wife &he didnt want to tell her bec as she may
become deppressd I said we can help you in this by
telling her e great empathy &our psychologist can help
her if he agree because you need her support 2- how
long he will live as he want to travel abroad, he didn't
enjoy his life I told him yr ca is so advance &is likely to
shorten yr life he ask for estimation isaid monthes
rather than year then he said can you give meds to
prolong my survival to travel I said travel is not good
4you bec of Hf &yr ca (this taken against me ) 3-he has
agarden &he want to continue gardening Isaid I.ll refer
you to OT then isummarize &check understanding ,
offer help. In discussion summarize yr case .why you
insist to tell his wife & not respecting his autonomy ;
bec he is need of her support &I ask him if he agree .
Why you didnt allow him to travel isaid bec he may
deteriorate &of hf he said if he insist how you will
solve problem I said he need to tell his health
insurance &for his hf ikept silent he said assessment
for fitness to fly then he ask can we give meds to
prolong survival of Rcc I said am aware of new meds
used in advance Rcc but couldn't remember he said
why you didnt offer it for him isaid may be he will ask
for it &i have no information about it he laugh then
bell rang alhamdellah iwent out thinking v.bad about
my self as didn't answer his
Consern well but igot 14
St5: first case is young male e difficulty in walking no
other information I was completely disorganized ithink
about neuro inside problem for 2yr no neuro
symptoms I ask 4examination he said ok iask pt to raise
his leg he raised them well again v.bad feelig second
case is lost the it come to my mind to see his gait he
said ok at that time I recognized question mark posture
I ask is he has pain yes in lower back alhamdellah AS
isaid i want to do occiput walltest he said do it then I
ask about other symptom &assoc then iexamine cvs ,
resp .I ask 4 schoper test he said do it again ilet pt to
stand idid it improperly iforget to check for tenderness
& to ask for chest exp bec of time concern what is the
course &need tretment I answer we will do blood test
&image & refer to dpecilized dr iremeber the job in the
last second &he is adriver alhamdellah questions about
diagnosis inv&management really am thinking about
20 but i got 28 . Second case is young male blaring of
vision again no other information inside it is chronic it
started at night & progressd colour vision affected so it
come to my mind RP iask for exam he said do then V/A
he can.t see even moving fingers going to fundus with
typical RP then iask about assoc nothing is there he has
positive fh of blindness his brother iask about med list
serugate said steriod &immunosuppressant isaid 4
what she said for behecit dis I ask e what he presented
she didn't understand me the other examiner answer
me e DVT is he ok now regarding his behecit &no
problem e meds she said yes .consern is it related to
behecit & about his kids .questions is about behecit the
examiner is v.tough he ask why eye signs is not related
to behecit .what behecit can cause in the eye I said
pale disc due to sagittal thrmbosis he is not happy he
want other thing but my mind is stocked why is asking
about behecit &forget RP he gave bad impression
about this &I think he will give me 16 or some thing but
alhamdellah he gave me 26 . Iwill write other statoin in
separate post because ihave problem in my mobile
.rgarding long post
Cont : my exam is on 8/4/2017
St 1 resp young male with pruductive couph he look
healthy when i ask him to couph it simulate the couph
of copd his chest exam is completly normal only
hyperexpanded chest couldn't appreciate any thing
else ipresent the casevas copd he said where is the
trachea isaid slightly deviated to the right he said what
is the cause of couph isaid infective exacerbation then
he ask about how to diagnose .common organisms.NIV
&LTOT when iwent out one of my colleque he
diagnosed him with pleural effusion ifeel v.bad about
my self how I diagnose copd only with character of
couph &hyperinflation so again this xase is lost
completely ithink he will give me 8but surprisingly igot
. 19
Abdomen :cld male pt pale & jaundiced e parotid
swelling enlarged lt liver lobe .splenomegaly &ascites
&ll odema discussion about causes , diagnosis .acute
.presentation &treatment of varceal bleeding &SBP
St2 young lady e bloody diarrhea e h/o travel to kenya
inside it is for 3week since she came about 12time/day
.fever.shivering .wt loss no other symptoms positive fh
of CDher cousin her family travel e her &are ok & she is
on lopermide from her Gp consern is what is the cause
I said mostly cd but am going to admit you doing blood
test &images to exclude others& to stop lopermide
time is finished before discussing mangement the
examiner again very tough asking about diff igave cd ,
infective diarrhea he said what else isaid ca but far as
she is young &no fh he said what else ikept silent he
said what about malaria as could be but far for bloody
diarrhea like this he ask why you want to admitt isaid
bec of severe colitis according to true love criteria je
said you didnt assess the volume &he is right I forget to
ask again ithink he will give me 14 or15 but he is
generous he gave me 19 alhamdellah
St3 which is my disaster since my previous attempt
neuro aman e difficult y in walking examine LLon
screening he is unable to raise his ul.&ll so hemiplegia
e hemesensory loss I finished early he ask what you
want to examine I said CN he said ok idid 7 , 12both are
affected he said what else isaid heart he said no need
then what isaid carotid he said no need what else isaid
gait as if he is waiting for this question is present yr
finding .diagnosis .dd.inv if he came in acute attack
meds given acutely .if AF what you will give isaid
warfarin he said can you give noac isaid yes he said if
Af isaid no give me samples of noac i gave 3 I got 20
.cvs alady e sob she has small volume pulse reg , raised
jvp , midline sternotomy scar mvr , TR loud p2 ll
oedema , ESM In AA not radiated ididnt mentioned it in
presentation but he ask for it questions about couses
.complication of valve replacement .INR &SBE
alhamdellah igot 20
So my score is 166 this is my second attempt . My
advise is to be confident exam is easy but we
. complicate it

my experience in kasr elaini 6/2/2017


: started with station 1
abdomen: young male pale with huge splenomegaly
"crossing midline" and mild hepatomegaly with
multiple small cervical l.n
i presented the case as huge splenomegaly and
hepatomegaly my diag. CML the other d.d i only
mentioned bilharziasis and i didnt mention anything
else , even lymph nodes i didnt mention
discussion : whether this pat. has cld i said no
whether he has ascites i said no then he asked me how
i know this is spleen i told him i cant insinuate hand
above it and continous with splenic dullness he asked
what else i told him the notch but i didnt appreciate
the notch he asked this is spleen or a mass i answered
this is spleen . cause of anemia and inv. blood film >>
blast cells i was not satisfied with my performance
although i m sure of my findings he asked me at the
end whether the pat. has jaundice i said he has tinge of
jaundice "not sure" i have got 11/20
chest : old male with cachexia , clubbing , tar staining ,
hyperinflated chest but only findibgs bilateral basal
crepitations i said in this patient we should exclude
cancer first then my diagnosis i said he has obstructive
airway disease and interstitial lung fibrosis the
examiner told me ok so you think the patient has
clubbing and interstitial lung fibrosis? i said yes ! then
the discussion was about IPF inv., findings in blood
. gases , pulmonary function and HRCT
i got 19/20
station 2 : history .. young male with recurrent attacks
of typical chest pain with positive family history of
sudden death at young age my diagnosis was familial
hyperlipidemia
the examiner was very irritating that he didnt give any
chance to think he asked what if his inv. is normal i said
i will refer him for cardiac cath. the examiner was
shocked cath. immediately! i told him that the patient
should be considered unstable angina as newly
diagnosed angina but he has no chest pain for the last
48 hours he stares unstable !
asked about treatment i told him that i will give this
patient ASA and b blocker until his inv. declared the
!examiner looks not satisfied at all
he asked what if all his inv. came normal i forgat
!!! everthing about psychic and anxiety
i was very upset by the end of this station
" i got 18/20 !! "to be continued
:my exam experience final word
my little advice if applicable
never give up as the exam. is easy to pass and very -
easy to fail and you will not know when either will
.happen
spend extra time examining patients and searching -
for physical findings in your practice and little time
with books while you are preparing for the exam.
except for history and station 5
don't search for rare diagnoses as they are also rare in -
exam. but don't miss the common ones and don't miss
.the clear signs which are clear to examiners too
examiners are rarely cardiologist or neurologist so -
.don't try to search for what they already missed
!are very helpful a lot of prayer and doaa -
finally not every one smile to you love you and not -
every one give you -sorry- a shit hate you ... and so
examiners
thanks for all our colleagues who helped us and
continue to help others in the hard journey of MRCP
specially dr Ahmed Maher Eliwa Hope Hope.... i like
you

: started with station 1


: Chest
. Rt lower lobectomy
: Abdomen
. Hepatosplenomegaly fo differential
: CVS
Marfan with MR
midline sternotomy scar
. Loud metallis second heart sound
:CNS
Examine LL
Sensory motor peripheral neuropathy
The pt had swollen deformed knees and medial scars
over the both knees and varicose veins
: Station 2
History of travellers bloody diarrhea , fever , abdominal
. pain , just returned from a travel in Kenya
Her concern was her auncle had chron's disease and
. she was afraid from having the same disease
:Station 4
A man who had terminal heart failure and the
cardiologist told him there is nothing more to do for
. him
He discovered a lump in hos neck of which a biopsy
was taken and discovered to be a metastasis from a
. kidney cancer
Break the bad news and discuss the prognosis and the
. ttt options
: Station 5
. A 25 yr old lady presented with joint pains
On taking history and examining her she had Ehlor
Danlos , her brother also had the same disease . She
had a past history of pneumothorax and quit smoking
. ever since then
Concern : joint pains in all her joints , mainly her
shoulder (dislocating and returning it back intentionally
by her self ) and the other concern was her child , she
has a boy and she was asking how to take care of him
The examiner asked how can she screen her child for
Ehlors Danlos
: Station 5
A man presented with daytime somnolence , he was
diabetic type 2
On taking history and examining him he had ankylosing
. spondylitis and obstructive sleep apnea
) He was not obese(
my experience in mrcp diet 1/2017
Cairo almaady military hospital
last day 8/2/2017 3rd carousel
i started with station 1 then 2, 3, 4, lastly 5
station 1 chest & abdomen
i started with chest case male about 20-30 years this pt
has no any sign of chest problem no cynosis, no
clubing, trachea central,other examination normal
except only dullness in the lt base in the mid axillary
line,i noticed also skin changes in this area due to
pevious dressing and plaster marks, and peripheral
venous line in rt arm, i became confused , because i
was expect chest case to have many sign that we see in
the course,and i have not reach any diagnosis after
finishing the examination, i present the case as lt
peural effusion mild to moderate, after that discussion
was very tough with British examiner of African origin
he is not giving time to think discussion about causes i
said postpneumonic, TB, tumour
. investigation i said basic and cxr ct chest
treatment i said pleural taping he ask what else for
taping i said therapeutic and diagnostic , how I said for
analysis protein, cells, LDH, PH , how differentiate
between transaudate and exudate I said light criteria
he asked about light criteria but he not give time to
think, then ask if recurrent effusion I said pleurodesis
asked what indication I said I recurrent pneumothorax
and try to remember another causes but he didnt give
time to collect your thought , asked about type of
pleurodesis I said talc power he what else I didnt
answer at the end he asked he has any thing else,I said
no and I felt I loss the station because I didnt know the
diagnosis and vey tough discussion, but the result came
.19/20
Addomen case : middle age pt average body built , no
sign of chronic liver disease, no jaundice, no anemia, I
felt the rt lobe of liver 6 cm, lt lobe 6cm, but I felt the
total span about 12 cm , spleen felt 6 cm, no ascites,
there is lt axillary lymphadenopathy, I was confused to
said hepatosplenomegaly with lymphadenopathy or
splenomegaly with lymphadenopathy, first I said
hepatosplenomegaly hepatosplenomegaly with
lymphadenopathy, then said no splenomegaly with
lymphadenopathy for DD lymphoproliferative,
myeloproliferative, infection asked like what I said TB,
then asked about investigation I said basic the u/s
abdomen, ct abdomen , LN biopsy asked if LN was
negative I said do bone marrow examination, then he
asked why not chronic liver disease I said in this case
no peripheral stigmata of chronic liver disease, I
remember he this question twice and I answer the
same . at the end I felt I also lost the case but I scored
.14/20
At the end of this station I felt I lost the whole station
and get depressed I said one station not the and try to
.do better in the next
: Station 2 history
Middle age male c/o back pain for 18 month, some lab
investigation I didnt remember that was normal
I start by identify my self, my role, check pt identity,
and agenda of meeting,make analysis pain, gradual
onset for 18 month, localized to whole spine, not
reffered, morning stiffness, improved with exercise and
initially with paracetamol but now not helping, not
increased by cough, straining, cannot bend forward to
pick anything from ground, no trauma, no neurological
manifestation, no sob, no cough, no chest pain, no
palpitation, no GIT symptom, no rash, no mouth ulcer,
no oral ulcer, no fatigue, no muscle pain, no wt gain or
loss no lump or pump , no eye symptom but he had eye
problem many years ago treated with eye drops by eye
doctor but he doesnt know the diagnosis, no joint pain
but he had pain at back of his knee many years ago
treated with analgesia, then I ask him he had anything
he want to add I forget to ask about, then the
surrogate become confused and asked what do want
to known, I said anything with your health he said only
pain affect me so much.the I summarize to him the
.problem
.PH; NO previous similar condition, no chronic illness
FH; his father has skin rash looks like psoriasis, his
.grandfather has similar pain
Social H ; NO alcohol, no smoker
Job ; desk job work
No travel abroad
Hoppies like to play football but now affect by pain,
and this pain affect his daily activity badly
Concerns; I said to him from you have back pain with
previous eye problem and tendon Achilles affection,
and your father has skin rash and grandfather has
similar pain, so you most likely has a condition called
ankylosing spondylitis inflammation of joint of your
spine, then he said it curable, I said Im sorry its not
curable but controllable we give some medication to
control pain and prevent progression of disease . He
.has no other concern
I told him we do some blood tests and imaging of the
.spine and give appointment to tell the result
Discussion : what is diagnosis, I told +ve finding of
history then I said ankylosing spondylitis he said why
not osteoporosis you are not asking about the I said no
( till now I didnt know relation between fever and
osteoporosis ) ask about investigation I said basic cbc,
esr, rf, then HLA B 27 then imagining xr spine MRI,
THEN ask about treatment I said multidisciplinary
team, physiotherapy , occupational therapy , he said
what type of physiotherapy I said swimming ,then ask
about medication I said NSAIDs he said if he has git
pain I said we give PPIs, he said but you dont ask
about manifestation of git bleeding I said I didnt ask
specifically but he has not any git symptom, then he
said you ask about it indirectly when you ask him do
you have anything you want to add I forget ask about
it. Then he ask if pain not control I said to give
biological treatment he asked about precautions I said
.TB screening
I thought I did this scenario vey well and I will get near
full marker but I get 12/20 ididnt know why
station 3 cvs & neuro
Neuro examine lower limb
Old pt about 60 years I strat by screening found that he
has Lt hemiplegia hypertonia, hyperrflexia, extensor
plantar, +ve pathological reflexes, I forget to do clonus,
loss of all modality of sensation except vibration sense,
I present my findings, and I said lt hemiplegia with
hemisensory loss he asked about causes l said
thromboembolic, hemorrhage, and he asked what else
I said AF he asked what else what else I didn't answer
and ask about management I said if acute stage we do
CT brain to rule out hge then give antiplatalate, Statin,
risk factor control he if he came within one hours I said
to thrombolysis
In chronic stage physiotherapy, bowel care bladder
.care. I got 17/20
CVS. Pt has sternotomy scar, pulse regular no special
character, apex can't be located with inspection and
palpation 1st accentued with pansystolic murmur
propagated to axilla, 2nd heart sound metallic with
ejection systolic murmur l AVR with mitral
regurgitation ask about management I said education
and councilling, cbc, coagulation profile, ECHO, ask
about frequency of inr monitoring I said daily till reach
target then every 3 month,ask about new
anticoagulant I said not indicated in valve replacement,
ask about endocarditis prophylaxis I said if pt need
.invasive procedure in septic area, I got 20/20
station 4 Communication
Bdn of uncertain malignancy
Long scenario female 80 ys presented e
vomiting,obstructive jaundice loss 20 kg of body
weight but still obese for 2 m u/s and CT no mass no
lymph node Stent inserted in common bile duct no
malignant cells in biliary fluid but still malignancy is
suspected
Task inform her daughter and answer her queries l
started as Dr zain taught us identify myself my role,
check relative identity, ask if she want any body to
attend meeting said no I inform her about reason of
meeting, then I ask her about what she knows about
her mother condition she started to talk for about 3
minutes telling the story of her mother so I explain to
her the result of investigation and told her that we are
still suspecting that her mother bad growth she said
what do mean I told her mean malignancy she get the
phases bad news reaction l left her to express herself,
and after becoming calm asked what happened after I
told her we need to take tissue sinp from suspected
area, she told please don't tell mama I answer as in
doctor zain course I handle thus issue gently if your
wants to know we will inform her we will not enforce
information to her and this will help her to talk
decision about management plan and if hide
information she might know and then she loose trust
in medical team. She asked not to inform mama for
thee times and every time I answer her the same,
finally she accepted. Another concern want to take her
mama I said no she can't, then discussion was about
ethical issues, why you will inform the pt I answered as
said to daughter and the autonomy of pt as she is
competent. He asked what could be the lesion I said
small intestine tumour compressing from out side he
asked about the I didn't know he said retroperitonneal
.sarcoma. I got 12/16
station 5
BCC1 middle age female c/o hand pain and b/l knee
pain
some investigation normal
When I saw the pt I found ulnar deviation of both
hands history go with Rheumatoid arthritis no PM NO
FH NO SMOKING NO ALCOHOL HOUSE WIFE pain
affecting daily living activities
Concern what I have
O/E swelling tense fingers,palmar erythema, hotness,
at joint wrist, pip,mcp
Ulnar deviation, no sign of carpal tunel syndrome,
function persevered, no nodules, i examine the knee.
And asked to examine the chest he said examine.i ask
to elevet the clothes he said examine while dressed I
auscultate no finding
I answered concerned Rheumatoid arthritis not curable
but controllable we some blood test and images to
confirm the diagnosis Discussion about management
Multidisciplinary team physiotherapy, DMARDs, if
failed biological, then he ask me why you examine the
chest I said due to Rheumatoid lung disease then he
show me the hand and ask what this I said palmar
erythema show me whitish lesion at dip joint of Lt
thumb I didn't answer I didn't know what is it. At this
.time I realized that I have lost the case but i get 27/28
DCC2
Young female presented with sudden onset of severe
headache in back of heade 8/10 no sign of increased
icp, no eye symptoms. no scalp tenderness, no rash, no
joint pain, no muscle pain, PH of migraine this
headache is different to migraine no aura no
neurological symptom, no nasal stiffness, No history of
chronic illness, FH cerebral bleeding, SOCIAL HISTORY
no alcohol, no smoking, house wife
concern what I have
Examination BP normal, no diplopia, no visual field
defect, I ask for fundus examination said no need
I answer concern sah we need to do brain image to rule
out and we'll give medication for pain control, other
concern could be anything else I said yes could be
atypical migraine but our priority to rule out brain hge
Discuss e examiner about diagnoses I said SAH, DD
atypical migraine, investigation CT brain if normal I said
LP when you do i said 12 hour what do you find I said
xanthochromia, then asked about posterior fossa
anyneuresm manifestations I said unstidness but he
want which cranial nerve affected by posterior
communicating anyneuresm l said the sixth abducent
but it's the third oclumotor that affected
Treatment I said abc then BP control he said what will
give I said nimodipine,then asked about neurosurgery I
said yes if has anyneuresm will do clipping or
endcoiling. I got 23/28
After the examination I was depressed because l felt I
did bad in first and five station but I get good score in
this two station chest 19/20,abdomen 17/20, station 5
I get 23/28, 27/28. And station that I expect I do very
good Station 2 I get 12/20, and station 4 I get 12/16
Finally l passed with total score 144/172
Allahamdoollah in clinical communication skill l get I
exact pass mark 11/16

Experience of our dear colleague Dr.Shaimaa


Mohamed Abdou
She PASSED PACES in EGYPT Febr. 2017
dear colleagues here you are my experience in paces
exam 1/2017
concerning pre-exam period I advice to sleep well and
not read from many sources...don't put previous
failure in your mind and keep telling yourself you will
pass...really that seems simple but it is very important
before the previous attempt I read many sources I took
...many courses but I failed
concentrate on cases for paces 2015 and the course of
dr. Ahmed Maher Eliwa really it was very beneficial
...and different from other courses and I'll tell you how
don't put false ideas in advance like maadi centre is
difficult than kasr elaini or try to remember the
!!patients...all this is nonsense
all patients in exams are acute like you see in life they
will obey your orders if you was clear but they may
...waste some time...ok!this what will happen in life
my exam was in maadi military hospital 7/2/2017
I knew I will start from station 3 which was my
nightmare in previous exam...anyway I tried to put out
this idea from my head and tell myself I am ready and
it will be inshallah different
the bell rang I entered the room first case was neuro 55
yr old man(by the way age is not real or scenerio but
you should proceed on information they gave to
you!!the man looked older!!)anyway I examined
without interruption it was examine lower limb it was
clear that it is hemiplegia but when I was comparing
sensation to face he kept saying please stick to lower
limb only!!!ok I am just comparing then he said what
else I told him I'd like to check coordination in upper
limb he said no need...so I wanted to examine gait he
was very happy as if he was waiting for this!!it was
clear then he started to ask where is lesion I actually
had no time to check cranial nerves 7,12 but I told him
it is above pons then asked me about causes in "this
young man"I told him thrombosis or embolism so how
you will manage..I told aspirin and control blood
pressure ...I was upset as I didn't examine for cranial
nerves...he wanted this.anyway I scored 19/20
then cardiology female patient 40 years old with
shortness of breath by examination I found raised JVP
,pulmonary hypertension,mitral regurge ,tricuspid
regurge..no scars..he asked about positive findings and
what investigations I told him ecg..echo..he said how
to manage...here mental block occur so I was silent..I
think I should have told diuretic test first...and he was
waiting what criteria in echo to diagnose it actually I
didn't know what he ment I scored 17/20
continue my experience in paces 1/2017 military maadi
:hospital 7/2/2017
second station for me was communication I was still
thinking in last station so I was feeling bad and not in
desired mood anyway also the surrogate was the same
in.previous exam.again I put this of my mind she was a
patient with anemia and wanted to discharge herself
against medical advice ..she received blood transfusion
improved but you want to convince her to stay for
more investigation mainly endoscopy but she wants to
go as her husband had MI and needed help...ofcourse
outside the room I divided my talk to
introduction,body and summary as I heard in dr.
Ahmed Maher Eliwa video..they discuss with me if she
still insist to go I told them I will tell her that I'll arrange
meeting with consultant but if she still insist she can go
with near appointment for endoscopy..he said you will
let her go??I told him I have to respect her
autonomy...I scored 13/16 for mistake that when she
said too early that she wanted to go for her husband I
should've told her immediately that I'll refer you to
social worker not delay it...and also they wanted me to
take very brief history of her condition to justify my
investigation to her(all this came in my mind again
)!!before I leave
then station 5 with very big marks so I tried to
concentrate as scenerios I find first difficult 55 year old
female with increase forgetfulness and abnormal
behaviour!!!I put alzheimer...renal or liver
failure...that's all I entered the room (again most
patients are normal and they are inventing
scenerio)the only positive from history with suurogate
when I asked about her health before she said she had
chemotherapy for kidney tumour 10 yrs back....oh!!so I
thought of hypercalcemia asked about its symptoms
examined abdomen...then in discussion he asked me
can it be brain mets ??I told yes but no localizing sign
but it is possibility...he said but you didn't examine for
it!!I scored 20/28
the second case 50 yrs old female with history of
weight loss...all negative except he told there is
polyuria and hypothyroidism on treatment..I thought
for autoimmune causes mainly but first I told him over
replacement of thyroxine ...ok he said but seems to
wait for something else ...I told diabetes...and he
wanted this and started asking about
investigations...anyway I scored 21/28
then station 1 chest 30 yrs old male actually I was
exhausted !!I didn't check for clubbing and thin skin for
steroid and time was running I detected creps of
fibrosis but dullness over the left base but I wasn't sure
about dullness so I didn't tell when he asked what
else..but I told him.patient is plethoric(in myself I think
this from steroids)he asked about fibrosis when to give
steroids what investigation and treatment I scored
14/20
abdomen female 55 years with ascites
hepatomegaly..?splenomegaly(this time I was not sure
but I told there is splenomegaly)he asked about liver
span,positive findings,differential diagnosis I told early
liver cell failure he said early can cause ascites I told no
it should be cirrhotic not enlarged then I told him
autoimmune he was happy to hear it ;non alcoholic
liver disease asked me about complications of ascites I
scored 18/20
third and last part of my exam in maadi military
hospital 7/2/2017
last station was history young male 40 years old with
poor mobility??I divided my talk to introduction then
system affected here either neuro or musculoskeletal
cause and end of discussion with summary and telling
patient most probable diagnosis and plan of
management from history he said only feeling can't
walk properly with history of lung cancer before for
which he received radiotherapy and they told him he is
ok..he told he is having problem with control of his
sphincters so the talk was about it is most probably
compression of cord and need urgent hospital
admission and MRI and to deliver that message to
patient discussion was about management and what
differential I COULDN'T find anything except lambert
eaton syndrome..I scored 17/20
total score 139/172
....but alhamdolilah I passed after all
again believe in yourself...practice very much...read
cases for paces for clinical and ryder for history and
communication...attend preparatory course then
advanced with dr. Ahmed Maher Eliwa as he is
different in the way of arranging your thoughts
systematically and in approach of station 5 which
represents big bulk of your total marks
finally thanks to ALLAH then dr. Ahmed maher
good luck for all and sorry for talking too much and
..dividing the post as I was afraid the mobile will be off
good luck and ready for any questions if something not
clear

kochi experience : st1) respi : rt sided thoracotomy


scars(2 scars) all examinations were normal except
widespread wheeze abdo: av fistula in left hand with
no other finding st2) possibly sheehan syndrome, I
guess st3) cardio: bradycardia with water hammer
pulse no findings in precordium possibly slow af in
heart failure neuro: peripheral neuropathy st4)
breaking bad news about young pt in icu due to
hemorrhagic stroke st5) pt with seizure with htn no
findings on examination possibly due to uncontrolled
htn, visual disturbance due to dm with others
complications of dm

Dubai paces today 20/2/2017


third circle
Satation5
Young pt with headache&uncontroled Bp
)Pheochromocytoma(
Pt post Ml
Present with chest pain&fever
Dressler's syndrom
History
Loose motion without loss of weight
)IBS(
Station 1
Abdomen
Transplanted kidney
Chest
Lobectomy scar
Station 3
Cardio
Sternotomy scar
With murmur in aortic
area
Pt have xanthlasma in his face
????
Neuro
Spastic parapariesis withaout sensory loss
For DD
Communication
Case of ca prostate
Bilateral hydronephrosis
With high k
Management plan
Nephrestomy
DNR
Good luck for all
No click sound or
metalic sound in cardiology case and the diagnosis is
Aortic stenosis
Cochin
St.1 Abd: ckd with av fistula peritoneal dialysis catheter
.and cv line bandage, Had some rash all over body
Respiratory- syst. Sclerosis with ILD
History: 35 yr old female, 2 episodes of rash, both
subsided, now with fatigue and neutropenia. prev
history of loss of conception
.Wants to get pregnant
St.3 CVS: mr with pulm htn and probable TR
CNS: Peripheral sensorimotor neuropathy
St.4- UC, given mesal and local steroid, not responding,
now advised to take oral steroid
Doesn't want to take it, looking for alternative
treatment
Discuss treatment & management plan
BCC1. 35, F, had fever& rash 10 days bk, now subsided,
h/o allergy, nothing on exam
BCC2. 80,F, hypertensive, now stooping and difficulty
in walking, knee pain, anaemic & kyphotic
Kochin )22.2.2017(
BCC: 1. Young lady with right upper limb pain
Examination absent pulse and carotid, subclavian bruit,
)Aortoarteritis (Takarasu
Partial seizure , pt with past history of breast ca .2
Resp: Systemic sclerosis with ILD
Abd: CLD
CNS: myasthenia gravis
CVS: ASD
.History... altered bowel habit for 3 months
Cochin )23.2.2017(
CVS: Avr mrtallic for AR moderate in cardiac failure bcs
..of thyrotoxicosis
Abd: postrenal transplant for probably lupus nephritis
..with cyclosporin tox
Resp: left pneumonectomy with compensatory
hyperinflation and upper lobe fib on opposite side
..probbly tb
CNS: young stroke with oral ulcer alopecia pallor
.probbly apla lupus
History postpartum thyrotox .. examiner wanted
.something more in diagnosis
station 4 newly diag tb. Hemoptysis... break the bad
.news counsel everything wants to go out of uk
.Bcc: 1. CAD, t2dm, ischemic dcm, mr
..fever with hsm.2
Kochin evening round )23.2.2017(
St 1. Abd: CLD with massive ascites spider nevi n
dilated veins-portal hypertension
Resp: ILD
History: Syncope with jerky movements - palpitation n
on exertion
St3: CNS: Peripheral neuropathy
CVS: Prosthetic valve
Station4. Meningococcal meningitis informing wife
BCC... 1. Psoriasis
LOw back pain with pain radiating down early .2
morning ? Inflammatory - sacroilitis

UK Experience ,,, Febriuary 2017


st 3
Neuro
parkinsonism with no tremor evident
Cardio
mitral valve replacement
st 4
syncope due to OHG overdose in depressed lady
concern :::confidentiality &support
st 5
BBC1
Systemic sclerosis with swallwing problems
BBC2
Visible Haematuria with no signs
st 1
Abdomen
liver transplant + clubbing
chest
COPD with bronchiectasis
st 2
Dizzy spells ,, post hypotention in AF +
DM+IDH+HTN+DVT

Sharjah today 23/2/2017


St1 Abd splenomegaly with liver cirrhosis
Chest COPD
St2 history Dizzy spells in Dm .. history of Mi DVT .. AF
... family history of DVT .on Ramipril
St.3 CVS MR + TR
CNS spastic paraparesis without sensory involvement
St4 pt with arrhythmia on amiodarone developed lung
fibrosis .. speak to the daughter
St.5 a.. HYPOKALEMIC PERIODIC PARALYSIS
B .. Dizzy spells (again ) IN pt with DM.and on ramipril

It is my pleasure to share my experience in


Muscat,Oman hoping my brothers will have a
benefit from my experience .
I started with station 4:
This lady diagnosed with SLE 3 days ago. And
her urine showed protein. Please explain to the
need for renal biopsy and manage her concern
I started with do u want anyone of ur family to
attend our meeting. She Saied no.
how much u know about ur health, surprisingly
she doesnt know anything
so I explained to her
then unfortunately ur kidneys were affected and
in order to know to witch level ur kidneys
affected we need to do one more test wish is
renal biopsy. Then I explained to her renal
biopsy. and I draw for her the kidneys and how
we will do the procedure
her concern is it by general anesthesia . I said
no but with local and sometimes we might just
let u sleep but it is not general.
She asked me will it affect my university. I told
her we are here to help u, we will do our best ,
we need to involve MDT to give u a proper
management and if it so u will live a nearly
normal life but us should under regular follow
up
then she asked me will it affect me if I want to
get pregnancy. I told her it should be planned
whenever It is planned and under close monitor
, it will not affect u
Then I told her there is a consent she has to
sign
she asked me If she doesnt like to do the
biopsy. Will it affect me.
I told her we do not know to witch extend ur
kidneys affected so we can not give u a proper
management.
Then she agreed and accepted to sign the
consent
examiner q :
do u think u convinced her.
I Saied yes
what is the ethical issues :
do u think renal biopsy is important for her.
I told absolutely, why , I answered the same
answer above
at what stage of lupus nephritis u think this lady
is :
I Saied wt least stage 3
What is the modalities of manager. He asked
about the drugs
I Saied im not sure but methotroxate is one of
drugs plus other immunosuppressant like
azathio.
did u answered her concern about pregnancy > I
Saied yes
then he told me did u speak about methotroxate
during pregnancy
I told him no but I have to
ok thank u
Station 5:
1.Young male with presented with diarrhea for
days after using antibiotics
for ur kind care :
when I entered the room 50 years male
I started with tell me about ur health
he said diarrhea now settled down . so whats ur
problem now> he said this skin rash with hand
pain
I asked to see the rash It was not looks like
psoriasis . so I stunted show me ur hand. There
were in rt hand only
ulnar deviation at metacarbophalengial joint
the left hand is normal
then I analyzed the pain and he said morning
stiffness for more than one hour
I asked all CTD anakysis from hair till symptoms
of myathenai qravis
then I asked him to see his elbow no rash
his back no rash
then finally where else u suffered he told me in
my scalp
yes this is psoriasis although no typical rash
but I Saied this is psoriatic arthropathy
I explained to him . and asked him how this
affect his daily activity and job. He answered a
lot.
then I managed his concern and I will refer him
to multidisciplinary team including joint and
skin doctor and they will provide agood plan of
management for him
is it treatable. Unfortunately but controllable
2. this lady co difficulty in swallowing . for ur
kind care.
I thought I will find systemic sclerosis but when
I went to room I found no evidence of SS. But
she has peaked nose strangely
I analyzed the symptoms which was toward
solid
no loss of weight , lymphadenopathy , sweating
nor alarm signs
I asked her to see her neck
the I found smaaaaaal goiter
then I started to analyze the thyroid symptoms
which was negative
social drug and past medical history non
significant.
I examined her thyroid and no retro sterna
extension
I asked her what is ur concern
she Saied what is going on with me
I explained to her every thing
is it treatable
I told her thankfully no alarm signs but we need
to exclude serious condition but from her
history no alarm symptoms
we need to do upper GI endoscopy
examiner q:
how u will investigate
how u will mange:
do u think this thyroid is the cause for her
symptoms
I Saied no thats why we need to do upped GI
endoscopy
Clinical stations :they asked about whats ur DD,
investigation and how u will manage this
patient.
Station 1:
1. Abdomen:
Please examine this gentleman :
When I saw the patient he is a young male
I do not know if he is overpigmented or not but
he looks so
then I took inspection then after finishing
examination the examiner asked me to present
my findings
then I was shocked but I saied I will present
what I found
This is my pleasure to examine this gentleman
who is lying comfortable with an average build
the patient is not pale jaundices or cyanosed .he
has gum hypertrophy with good oral hyagine
there is no evidence of CLD
this gentleman has mild fine termer
this gentleman has mid laboratomy scan with
some drainage scars
he has hepato-splenomegally
liver span is 10 cm and spleen is 10 cm below
the costal margin
No evidence of ascites and there is no evidence
of lymphadenopathy
I would like to complete my examination by
doing DRE and examination for gentalia.
Examiner q :
What is ur diagnosis?
I said in the presence of gum hyperplasia and
mild fine termer , hepatosplenomegally and mid
laboratomy scan I will put liver transplant at the
top of my DD
although there is no evidence of CLD but I
cannot exclude CLD with portal hypertension
How do u this the cause of CLD in the man?
I saied alcohol , infective like hepatitis,
infiltrative and autoimmune like autoimmune
hepatitis, PBC and PSC
hemochromatosis.
How u will invewstigate this pt ?
I will start with baseline investigation in form of
CBC , urea and electrolyte.
LFT
autoimmune antibody and viral screening
Iron study and copper study
Then I will go for abdominal ultrasound and I
maight need liver biopsy
How u will manage this pt?
Non pharmacological and
pharmacological
2. Chest: bronchiactasis with left lower
lobectomy
Station 2:
a 38 years old male presented with recurrent
chest infection since long time. All labs were
normal. for you kind care.
During analysis he has recurrent ear pain(Otitis
media) and sometimes loose motion. sometimes
he has burning in micturition.
He is smoker and alcohol consumer.
No other symptoms
I have to exclude HIV( sexual history) in a
sensitive way.
Social and family history
it affect his job and has recurrent absence from
work.
no significant drug history apart from recent
antibiotic usage
So examiner q:
what is ur dd
common variable immunodeficiency vs cystic
fibrosis vs HIV vs hypogamaglobulinemia
how u will investigate this gentleman:
baseline investigation in form of CBC
chemistry.
xray
immunoglobulin
sputum for culture and sensitivity . for gram
staining and acid fast bacilli
Na sweat test
how u will manage this pt:
pharmacological and non pharmacological
last q : do u think this pt has CF> I said I ahave
to exclude but for me unlikely
so this gentleman diagnosed with CF> what do
u think. I said wrong diagnosis
Station 3:
CNS:
Neuro case :
Please examine this gentleman:
It is my pleasure to examine this gentleman who
is lying comfortable with an average build
by inspection there is no deformity , scars or
wasting
This gentleman has weakness in a pyramidal
distribution in witch flexar is weaker than
extensor, abductor is weaker than adductor and
distal is weaker than proximal.
The weakness is more in left lower limb than RT
There is loss of sensation up to amblicus to pin
prick (strange finding)
posterior column modalities of sensation in
form of joint position and vibration are intact
He has an evidence of cerebellar signs evident
by impairment of heel shin test
I would like to complete by examining the upper
limb, eye looking for nystagmus , gait looking
for wide ataxic gait
then the eaxaminer told me just forget about
sensiation.
What is ur diagnosis:
I told him this gentelman has cerebellar
syndrome
Whats ur DD?
I told him I couldnt get ur Q
He said what are the causes of cerebellar
syndrome?
I said demylination like MS, degenerative and
dierty , infective, vascular like stroke,
inflammatory , neaoplasm, vit b12 dif and
hypothyroidism.
Whisch cause can be reversible after
treatment?(I thought he mean reversible
completely)
I said Iam not sure
How u will investigate this pt :
I will start with baseline investigation in form of
CBC , urea and electrolyte.
MRI of spinal cord and posterior fossa
CSF analysis, VEP
Vit b12 level, THT
How u will manage this pt? Pharmacological
and non-pharmacological
Cardio: Double valve replacement
Wish you all the very best of luck

UK Experience
Nottingham city hosp 25/2/17
Start with station 5
Bcc 1
Man with lethargy
Got DM, joints pain, loss of sex drive, rashes on face
Dx:Hemochromatosis
Bcc 2
.y.o frequent falls 84
.Instability
.When ask to walk, examiner said normal
When I want check eye, they said normal. Want check
.power they said normal
.So I listen to heart that normal
And he has postural hypotension on citalopram and
.clopidogrel
Ddx : postural hypotension with instability
.I mention post circulation cva
)Now I think it may be NPH(
I ask others candidate, they said mechanical fall as
.patient trip over
St 1
Abdomen
.Rif scar with renal transplant, acf fistula
Got pedal edema
Respi
.Normal lungs!!!! Patient snores. I said OSA
St 2
Clear cut cystic fibrosis from history
Recurrent chest infections
St 3
Cardio AS
Neuro distal mixed sensory and motor neuropathy _
cmt/dm
St 4
Many said difficult
Patient keen active, sister wants paliative... Pt
disseminate bowel ca

Cochin )25.2.2017(
St 1. copd with bronchiectasis
abdo- ascitis with jaundice
.st 2. History of Ankylosing spnd
st3 cardio- I said- AS with MR,not sure
neuro- Lmn Rt facial palsy
st 4. convince pts sister about palliative care of a
metstatic cancer. sister wants sedation. parent wants
agressive Mx. pt had bleeding ulcer. nnow nneed
embolisation sister doesnt want.a bit complicated
st 5. a pt with neck swelling- multinodular goitre
another pt male with fatigue- I found OSAS Due to
obesity, wanted to rule out endocrine and metabolic
syndrome

)Dubai (21.2.2017
Station 1 : resp: pleural effusion
Abd: bilateral palpable kidneys with dialysis catheter
and right iliac fossa mass
Station 3: neuro is peripheral neuropathy
CVS: diastolic murmur de is mitral stenosis vs aortic
regurg
Station 2: lady with palpitations delivered 1 month
ago. Postpartum thyroiditis
Had heat intolerance and loss of weight
Family history : her father died with hemorrhagic
stroke. Brother had a heart attack at age 48
Occasionally uses inhalation for asthma
Her palpitations were on exertion
Station 4: lady with sle and proteinuria. task to explain
the diagnosis and the need for renal biopsy
St 5: sudden loss of vision in a htn patient in one eye
lasted for about 30 minutes with complete recovery
St5: lady with me has numbness of the hands

UK- Experience ((Glasgow ))%% Alexander parade


%%hospital
St1
collapse with consolidation
Ascites for DD
St2
diarrhoea, vomiting in pt with DM ( poor
control)&dialysis for last 3 yrs
St3
AR in downs syndrome
Parkinson's disease
St4
pt with achalasia did esophageal dilatation &ended by
perforation discuss with son who concern about
negligence
St5
headache for 2 months with scalp tender ness (giant
) cell arteritis
)Loss of vision for 10 mins (TIA

Exam 16/2/2017, uk centre


Station 5 : sudden onset visual disturbance
Elderly male , vision disturbance since 1 day , came on
while watching tv, problem with central vision RT
SIded, history of diabetes, Examination , bruit in right
carotid, pulse regular , power normal , fundoscopy new
vessels at disc, D/d diabetic retinopathy /
maculaopathy /Amurosis fugax
BCC 2 : 84 yr old headache past 2 months , central
headache no aggravating, relieving factors taking
analgesics, No history to suggest pituitary tumour, SOL,
GCA,stroke. I gave D/d could be tension haeadche or
analgesic overuse
Abdomen : hepato spleenomegaly , 50 y old male with
abdominal distension, Plamar erythema , jaundice ,
liver enlarged 4 fb, spleen 2 fb, no acites , no
lymphnodes,Gave d/d myeloproliferative disorders,
CLD. There were some other cases in abdomen 1
person just got hepatomegaly ,with no spleenomegaly
Resp 2 patients : Young gentleman with clubbing,
Bronchietasis. d/d cystic fibrosis , ciliary dyskinesias
and immune related problems
Old gentleman clubbing and fine crackles , pul fibrosis
History : bloody diarrhoea d/d infectious , IBD
Station 4 : counselling lady with Ulcerative colitis ,
counsel regarding steroids , she is not willing to take
steroids due to side effects
Neuro : spastic paraparesis with no sensory
.involvement
D/d Hereditary spastic , MND
Diet_1#
7.2.2017: Cairo

Dubai exam today


(St 5 (carpal tunnel syndrome &TIA
ST 2 palpitation for evaluation
Comm SLE for renal biopsy
#######################
unfortunately:::
Candidate was shock &didn't give more details other
than these

Experience of my dear Brother ,, please PRAY 4 him


Ayr university hospital (Edinburgh)
1st carousel 19/2/2017
Station 1
Chest:
Elderly man has left pneumonectomy (scar in the
back NOT apparent anteriorly at all). Cushingoid
face, purpura, thin skin and proximal myopathy.
Right basal fibrosis.
Questions:
- Findings
- Causes of surgery
- Investigations
- Treatment of PF
Abdomen:
Left iliac fossa scar with underlying mass. Evidence
of gum hypertrophy and neck scar. Two anticubital
fossa scars.
- Findings
- Cause of ESRD in the patient
- How will you investigate?
- If the patient comes at night with tender abdomen
and fever what will you do as you are on-call?
Station 2
Full scenario is:
49 year old male has 6 weeks history of chest pain
which he couldn't give clear description. Sometimes
it's related to eating but didn't improve with
omeprazole. Cardiac and chest examination are
normal. ECG is normal. Please see and advice.
- ONLY Positive info from hx = Smoker + HTN +
brother died from heart attack at age of 40 y.
Questions:
- DD
- What are general causes of chest pain
- What are hereditary cardiac causes of chest pain
- Examiner wants HOCM although NOTHING fits at
all.
- How will you treat the chest pain?
- How will you treat his uncontrolled HTN?
- What's investigations and treatment?
Station 3
Neuro:
Lady with difficult walking. Examine LIMBS.
Although survey is NORMAL.
Clear left resting tremors and evident extrapyramidal
signs (Parkinson's disease)
I asked to let the lady walk + talk + write
GAIT looks ok but slight difficult only when turns to
side.
I asked to measure sitting and standing BP, to check
nystagmus or upper gaze palsy. Examiners said No
need.
Questions:
- Findings
- How to diagnose Parkinson's
- When do you need imaging
- Treatment medical and surgical
Cardio:
Elderly patient with dyspnea.
COMPLETELY NORMAL EXAMINATION
Questions
- Positive findings (I told NORMAL
EXAMINATION)
- Causes of dyspnea when normal examination
- How to investigate him
- What will do for him if comes to you in the clinic
Station 4
40 years male was feeling cold while on his office. He
had high fever and suddenly collapsed.
Meningococcal septicemia is diagnosed. He was
started on IVF and antibiotics. Awaiting for ICU
admission.
Talk to the wife.
Station 5
These questions are asked in the the 2 cases:
Diagnosis
DD
Investigation
Rx
BCC1:
58 y male with type II DM on insulin. He has bilateral
charcoat joints. Four weeks ago he has right foot
ulcer on his sole. For your assessment.
BCC2:
28 y male complains of red itchy eyes. His thyroid
function was abnormal and started on treatment. His
eyes are still concern. For your assessment.
Good luck.
Copied from WhatsApp group
(14.2.2017) England Exam
Resp: Fibrosis secondary to RA and lobectomy scar
Abdo: young girl with liver transplant scar with
lobectomy scar with tracheostomy and multiple striae
Cardio: AVR
Neuro: mixed UMN and LMN signs with peripheral
neuropathy ??cause
Station 4: BBN
Station 2: Recurrent hypoglycemia episodes in T1DM
Station 5s: 1. PF in scleroderma patient,
2. RP

(Copied) Egypt 1/2017 Almaadi Hospital


8th February last cycle
Station 1:
*chest COPD + corpulmonale
*Abdomen hepatomegaly with spleenectomy scar
Station 3 :
*CVS Double valve replacment
*Neuro Hemiplegia
Station 2:
*Back pain in young male ,?with family history and
grandther had psoriasis
Station 4:
Speak to daughter whose mother did ERCP and
diagnosed as gallstone cancer, daughter is feeling bad
and doesn't want her mother to be informed about
diagnosis coz she delayed bringing her to doctor
2nd concern in station 4 ,she wants to take her
home
Station 5:
BCC1,female known RA presented with new knee
pain ?osteoarthritis
BCC2, female with blowing headache ,past history of
migrane different from this pain ,family history of
dialysis , father died early with brains hrge , PKD

Cairo 3rd carousel (8.2.2017)


St 1:Hepatosplenomgally with lymph node
Pleural effusion
St2: Ankylosing
St3:Avr with mitral regurge
Hemiplagia
St4: Explain uncertainty diagnosis cancer pancreas
St5:1.Subarachnoid
2.Rheumatoid

(8.2.2017) Egypt
Station 1
HSM
History
Diarrhea with family history of crohn's
Communication
Palliative care in advanced renal cell carcinoma
Neuro
PN
Cardio
AVR MVR
Station 5
1.Ankylosing
2.Retinitis pigmentosa
carousel 1:maadi 7/2/2017
station 3: rt side hemiplegia
cardio:MR,TR,pulm.HTN
station 4:convincing lady with anemia to stay in
hospital for further investigation(endoscopy)she is on
painkillers
station 5-1)female with increased forgetfulness and
abnormal behaviour...history of renal
cancer...hypercalcemia vs brain mets
2)loss of weight with hypothyroid...from history
..polyuria...he wanted DM
station 1:pulmonary fibrosis
abd.hepato ?splenomegaly with ascites
station 2:poor mobility with poor history of lung
cancer and radiotherapy....cord compression

carousel 1:maadi 7/2


station 3: rt side hemiplegia
cardio:MR,TR,pulm.HTN
station 4:convincing lady with anemia to stay in
hospital for further investigation(endoscopy)she is on
painkillers
station 5-1)female with increased forgetfulness and
abnormal behaviour...history of renal
cancer...hypercalcemia vs brain mets
2)loss of weight with hypothyroid...from history
..polyuria...he wanted DM
station 1:pulmonary fibrosis
abd.hepato ?splenomegaly with ascites
station 2:poor mobility with poor history of lung
cancer and radiotherapy....cord compression
good luck
pray 4 me please
I needed anti-mental block at that time!!!!

Carousel 3 (5.2.2017) Egypt


St 5: 1. psoriasis with HCV
2. HTN , DM with fundus
Station 3 : TR MR with PH
Spastic paraplegia
Station 1 : Splenectomy with Hemolytic anemia
COPD with left effusion
Communication : MI

Exam UK Experience
Dundy hospital...Febr.2017
St1
Abdomen: Renal Transplant
Chest :COPD
St 3
Cardio : AVR
Neuro: charcot marie tooth synd
St 5:
Retinitis pigmentosa+Night vision
the second on ?????
That is ALL
St George hospital, London
Carousel 3 Cairo 6/2/2017:
Station1:
Abdomen:hepatosplenomegaly (chronic liver disease).
Chest: right lower lobectomy with bronchiactesis on
left side.
Station2:history:female patient with photosensitive
rash and fatigue and neutropenia. ( SLE with
antiphospholipid).
Station 3:
Cardiology:mitral valve replacement.
Neuro:paraplegia and peripheral neuropathy.
Station4:communication:female patient80years
admitted by pneumonia and then developed delirium
the task was to explain the condition and
management plan to her son.
Statio5:
-weight gain in female patient :Cushing.
-skin rash in female patient who has hepatitis
c:cryogloglobulinemia versus lichen planus
EGYPT ,, CAIRO ..FEBR. 2016
Carousel 3
kasr Eleiny 6/2/2017:
Station1:
Abdomen:hepatosplenomegaly (chronic liver disease).
Chest: right lower lobectomy with bronchiactesis on
left side.
Station2:history:female patient with photosensitive
rash and fatigue and neutropenia. ( SLE with
antiphospholipid).
Station 3:
Cardiology:mitral valve replacement.
Neuro:paraplegia and peripheral neuropathy.
Station4:communication:female patient80years
admitted by pneumonia and then developed delirium
the task was to explain the condition and
management plan to her son.
Statio5:
-weight gain in female patient :Cushing.
-skin rash in female patient who has hepatitis
c:cryogloglobulinemia versus lichen planus

EGYPT ,, CAIRO ..FEBR. 2016


Carousel 1
Communication
APCKD
History
HOCM
Station 3
MR ?
Spastic paraplegia
Station 5
Pemphegus
Hypothyroidism
Station 1
Hemolytic anemia
Chest : ILD

Experience of my friend=EGYPT ,, Kasr


Alaieni,,5/4/2017
St5
My 1st case.. Behcet.. scenario diminution of vision.
Second case scenario diarrhea and weight loss. I
consider the case lymphoma..
HSP+lyphadenopathy+diarhea + night sweats.
I put tb on my dd as there's history of contact with tb
but i said lymphoma more likely.
CVS... Systolic murmer in thalathemic pt for dd.....
MR or TR. And the examiner asked me could me
hematinis murmer i said yes but i don't know what's
hematinic murmer!
Neuro case is different.. Young woman with lt sided
weakness.
Abd case was very huge splen with hepatomegaly also
and axillary lymph node i sade lymphoproliferative
vss myeloproliferative.
Chest... Copd but examiner was hard also straight
forward case.
History... Proximal myopathy for dd... Statin induced
vss polymositis.
Communication.. BBN meningitis.

Carousel 1
Communication
APCKD
History
HOCM
Station 3
MR ?
Spastic paraplegia
Station 5
Pemphegus
Hypothyroidism
Station 1
Hemolytic anemia

Cairo today
St 1 copd .huge spleen
St2 proximalmyopathy for d.d
St 3 monoparesis .mr +pht ?
St4 bbn mengiosepsi
St5 bhcet .abdomial tb
4.2.2017 Egypt Carousel 3
Abd... Thalassemia
Resp...Basal fibrosis and COPD
History ...Anemia and fatigue
CNS ...paraplegia with PN
CVS ...AR
Communication ....Suspected bronchogenic
carcinoma in pt working in shipyard
Station 5
1.Thyrotoxicosis
2.Optic atrophy in MS pt

Today first carousel in Egypt


1) lt lobectomy for DD
Splenomegaly for DD
2)diarrhea &vomiting in DM with RRT
3) mitral valvotomy scar in patient with SOB
Rt lower limb monoparesis with intact sensation
4) functional disorder, the surrogate denies all
stresses!!
5) fatigue in pallor transfusion jaundice splenectomy

2nd carousel Egypt


COPD
Splenomegaly for DD
History
Same as first carousel
Diarrhea with DM and hemodialysis
CVS
PH TR
CNS
Monoplegia with cerebellar signs
Communication
Functional disorder, headache same as first carousel.
Station 5
Anemia with splenectomy and blood transfusion in
febrile pt
Systemic sclerosis with PH and basal fibrosis
Egypt exam today
10 - 10 - 2016
Station 5
Pemphegus
gout
History
DM with lack of awareness
Communication
Steven Jonson
Neuro
spastic parapresis

MALTA paces exam


10 December 2016
ST 5
1= back pain=ankylos. spond
2=abdom. pain+wt loss
Alcoholic liver dis +HCV
Station 1
Respiratory = lobectomy
Abd == bilat enlarged kidneys
ST 2
History = ABDOMINAL PAIN== PARACETAMOL
OVERDOSE
ST 3
Cardio=Mitral sten+AF
CNS = PARAPARESIS
ST 4
communication ==hypoglycemia unawareness,,DM

Khartoum 5th December


2016
Day 3, Cycle 3
Station 1
Chest: Bronchiectasis
Examiner Q:
+ve finding
I mentioned :
Wheezes
Crakles (then he asked which part of the chest ,I told
basal ,then coarse or fine )
Dull percussion note on the lung Base.
Then he asked what about trachea (I told central )and
breathing sounds (l told reduced over the Base of the
lung )
I mentioned clubbing (but I am not sure so I thought
I was invented that because the examiner is
wondering about that)
Then I mentioned reduce chest expansion .
Then he asked about DD (I MENTIONED FIBROSIS
&COPD)
Then he which is more likely fibrosis or
bronchiectasis (I told bronchiectasis )
He told me the patient is not smoker (so I said COPD
is unlikely but can occur )
Then he asked what questions you would ask the
patient (l mentioned sputum production
&occupational history )
Lastly he asked about causes of bronchiectasis and
investigation
I got 18
Abdomen:
Cirrhosis with splenomegaly
The patient is male jaundiced & pale with shrunken
liver and splenomegaly and lower limb edema
Q:
Diagnosis
I mentioned Liver cirrhosis with portal hypertension
Then she asked about causesof cirrhosis (I mentioned
alcohol, viral,autoimmune. Vascular )
Investigation and management of of varices (l
mentioned endoscopic band ligation and beta
blocker,then she asked if beta blocker is CI, I told
nitrate but it is controversial and lastly she asked
How to control bleeding l told mennisota tube (l
forgot the name of sengestaken tube )
I got 20
ST2
Facial and tongue swelling and history of recurrent
abdominal pain. The patient is 28years old adopted
lady.
I started with greating the patient and who am I and
asked her to tell me more about her symptoms then
system reviews is normal apart from recurrent
abdominal pain &no symptoms of connective tissue
diseases
PMH :appendectomy
Drug & allergy is nil
FH (she doesn't know about her family because she's
adopted )
Social history :
University student
Non smoker or alcohol consumer
Concern about disease transmission to her kids and is
it serious and do I need dietary restriction
I told her the possible diagnosis is hereditary
angioedema and I explained it to her and (l told there
is the chance that the kids can be affected and the
disease can be serious if not treated and I will refer
her to dietitian for dietary advice and lmmune
specialist and I told her they are more expert than me
in discussing things further )and I also adviced her to
come to hospital if symptoms develop again and I told
epipen is ineffective for hereditary angioedema)
When 2min left l checked her understanding &any
further questions and I summerize to her and I will
write to the GP
Examiner Q :
1-who is support this lady (l told sorry I forgot to ask
)
2-DD
Hereditary angioedema
Anaphylaxis
Idiopathic angioedema
3-what is difference between anaphylaxis &
hereditary angioedema (l told absence of skin rash in
the later)
4- What is the pathophysiology (C1 estrase inhibitor
defeciency)
5- Drugs causing anaphylaxis (l mentioned ACEI &
NSAID )
6- mode of inheritance (l don't know )
7- Is appendectomy related to her condition (lam not
sure because l told the patient it's not related )
I got 18 (I am not expecting to get high mark)
ST3
CVS
Young man with AR&MR WITH LUNG congestion
(I am not organized I forgot the JVP and I examined
collapsing pulse at the end & didn't present some of
the positive sign )
Q:
About investigation & and management and what is
suitable metallic or tissue
And Q about age limit (I told I don't know but the
tissue valve for elderly and the durability of them 10-
20 years
I got 13
CNS
Upper limb examination
Motor neuropathy (wasting. Hypotonia.
Hyporeflexia. Intact sensation. Inability to do
coordination )
Q: DD
Myopthy
Motor neuropathy
Then investigation and management
I got 20
ST4
(from Dr. Zein course)
Hodgkin lymphoma stage IIA
For Hickman line and chemotherapy (I introduced
my self we agreed agenda withthe patient then I
asked him about his understanding he knows every
thing so I explained the need for chemo and Hickman
line insertion and I explained this to him and the side
effects of chemo .his main concern is fertility and his
job so I told about sperm bank and sickleave, then he
asked me shall I tell my wife? (I told I am happy to
arrange meeting for you and your wife if you agreed.
Then he asked what if decided not to take chemo, I
told I am sorry to tell you might die from the cancer .
At the end no more Q or concerns then I asked what
message you will take home and summerize and close
the consultation
Q:
1-summerize the main issue in the case
2-you told the patient you will offer him sperm bank
do you have it in Sudan (l told I don't know but he is
UK patient )
3-you told him to wear mask against infections what
other measures you will offer? I told vaccine. Q
:against what? I told pneumococal & infleunza.Q :do
you have this in Sudan (idont know I am not working
in Sudan )
4-What is ethical principles here
I mentioned autonomy and beneficence
5. Is the wife has the right to know about her husband
infertility?
I told no unless her husband agreed ,the examiner
tried to shake me but I didn't change my answer.
I got 15 (although I used some medical jargon but
only 2)
ST5
BCC1
JOINT pain with hoarseness of voice and muscle
cramp for 3 month. With history of thyroidectomy
The patient has obvious stridor
Discussion about causes and investigation
I got 22
BCC2
Young lady with dysphagia for solid for 3 month in
patients with scleroderma
Discussion about differential and mangement
I got 26

Again thank you dr.Ahmed for all help you provide.


So, my experience was posted on this group before my
result been release.Here you find it e marks.
I started my exam with station 3
Neurology: examine Young lady upper limbs:
General inspection global wasting but predominantly
small muscles of both hands
Hypotonia, hyperreflexia and weakness almost
equally distributed 2/5.I needed to examine sensory
system and proceed to see the fasciculation over the
thigh but examiner told me skip it and I proceeded
for CN examination where I found obvious tongue
fasciculation.She was not able to speak and she had
drooling of saliva.All of what I commented on during
my presentation.She was a case of MND.
The discussion started with if the pt been admitted
several times to the hospital because of pneumonia.
My response was:asp. Pneumonia due to bulbar palsy
as a part of her condition. Then deep details about
MND, types(I offered),where is the lesion?
investigations(clinically),management(MDT in details
from swallowing therapist up to psychological
support)
Treatment and benefit from it and cost effectiveness(I
offered and I said its not).
20/20
Cardio:
Female of 40s
Pulse irreg irreg+Raised JVP midsternotomy
scar,diffused apex, palpable P2,PSM rad. to axilla, no
click.
MR after MVR e tissue valve AF,Pulm.HTN.
Discussion was about investigation, ttt medical and
surgical, wich valve you are going to use and to
monitor her.
Station 4: BBN
physical trainer presented with eye sight problem and
cerebellar dysfunction first time(inside when you
ask).Your consultant think of MS and he is away,
MRI Brain highly suggestive,I started as dr.Zain
taught us and get through all the important points
but I forget the driving issue and they didn't asked
me.
Examiner questions:
Ethical issues:
BBN. So,you BBN but with uncertainty
Me, yes,because this is the first attack and the MRI
became highly suggestive so I need to revise and
discuss with my consultant again.
autonomy.
examiner:where in the scenario?pt didn't want to tell
his partner.
Ex:Do u think its important to tell his partner
Me:yes,she will help him emotionally and physically.
Ex:asked about fertility,inheritance, chance of the
babies to get the disease?
I responded exactly like in dr.Zain sheet and I havr
already discussed the same q e the surrogate.
Ex:what about his job?same advice I gave to the
surrogate I gave to the Ex:
Its early to anticipate but you need to be in regular
F/UP in order maximise the free of symptoms period
and reduce the attacks.
How the disaese behaves?same answer?behaves
differently in different people later on we may be
more familiar with the type you might,again the F/up
is crucial for you(him to Ex).
I mentioned the need for giving him steroids.And new
medication but they didn't ask the name.
13/16
BCC1:
Young male e recurrent mouth ulcers
i covered all GIT AND MSS and no skin rash all of
were negative.
I asked about DVT which he had 3 years ago. I asked
about PE and its symptoms
(-ve).
I examine the mouth I found about 3 small ulcers
similar to the aphthus ulcers.then immediately I
jumped to the legs and I fully examine the legs for
DVT (Lt leg below knee)good peripheral pulsation.
I examined the precordium and there was loud P2.
Concern:what is the cause of the ulcers
And what you will do for him?
I replied like Behcets disease causing DVT and
PE.both to the relative(not PE) Examiner
He asked me how can you justify that there were no
symptoms?recurrent micro throbi passing unnoticed.
What about the skin rash over the Limb e DVT?
Me:.I don't know.
investigations, ttt
28/28
BCC2:
Pt around 50yrs old
Known RA has back P.
I analysed the BackP:sudden mechanical e no
neurological manifestations, sphincters intact.
RA is controlled.
DH:on prednisone 15mg for 3yrs.
All others not significant
Examination:back is tender on percussion.
LL.exam:upgoin planter with pyramidal weakness
but examiner did not allow me to complete for
sensation.
I diverted to examine the hands for RA (activity
disability) there was drumstick clubbing then I
jumped to the chest thinking of HPOA due to Ca lung
causing mets to the spine as a cause of his Back P
Chest:coarse crackles Wheezing
Concern:what's wrong e my dad.is there any thin
wrong?I replied same as to Examiner.
Examiner :what is the problem e this pt?steroids
induced osteoporosis, causing vertebral #.although I
have DD like Ca lung e mets to the back.
What is the cause of the clubbing?Ca lung
In the context of RA
I don't know.He needed an answer and I hadn't
it.Lung fibrosis unlikely to cause clubbing in the
context of RA and he agreed.Again what is the
cause!!!
So how are you going to manage him?
Urgent MRI spine,neurosurgery consultation bill
rang alhamdulillah.
22/28

Satation 1:
Chest: under built Young lady e sputum pot filled e
yellow to greenish sputum.
Clear case of bronchiectasis but the examiner wasn't
happy about that
He asked what is your DD?
Me:DD for bronchiectasis or the causes of it?
Ex:For Bronchiectasis
Me:Lung fibrosis
Then the discussion diverted to Lung fibrosis,causes,
upper and lower zone fibrosis, unilateral and
bilateral.We ended e TB, investigations.
19/20
Abdomen :
Young male e huge hepatomegaly almost approaching
the Lt hypochondium.e no features of CLD.
I knew that I trapped my self.
I felt duputryn contracture on one hand, and I think I
started to imagine temporal wasting, which were not.
Q:DD :I sayed there causes for hepatomegaly alone
again I trapped my self because I started with what I
was thinking about:Alcoholic liver disease
Ex:is it common in your countryl realised my
mistake, no way to fix it.
investigations, ttt.
13/20
Station 2
75 yrs Male found collapsed.
High s.cr BU S.Na.
After doing my introduction
I started e open ended question, daughter had been
called that her father found collapsed.So the analysis
of collapse was not informative.So I changed the plan
and I went searching for losses by system.GI(GE 5d
ago treated at home but he was doing fine) including
upper or lower GI Bleed, was -ve)
GUS :polurea for more than 3 months
Surrogate was very kind but she spent about almost
5min unstoppable speech it was hard to interrupt her.
I used to go systemically but I couldn't because the
time left were v.limited I was afraid, to lose
marks,however I thought I cover all.
Pt has depression since his wife death,he takes
medicine for it,which well controls his symptoms, I
rapidly assess his mood,sleep,appetite,all were ok I
didn't ask about suicidal ideas but she told he was ok.
Nothing else was significant.
I answered her concerns, Explaining DI, sequences of
Lithium toxicity,the contribution of the DHD(caused
by GE)to the problem.
I replied to the examiner almost in the same manner.
Discussion was about type of DI and which one this pt
has.How to investigate and what you expect to
find.How to treat.She agreed all.
At the end Ex asked:for how long pt has polyuria
Me:6 months.
How did he compensated for this?
Me:I didn't ask.But I should.
17/20
Overall score 152/172
Alhamdulillah
I hope the best for all
Station 5:
1. A 59 yrs old lady presented with a 10 days history
of diarrhea.
VS: pulse 90 bpm 100/60
Inside: a relatively middle aged lady sleepy ,dry
mucus membranes ,history of diabetes 20 yrs ago.
The diarrhea is not containing blood or mucus ,no
alarming symptoms or signs, no autonomic features,
no constitutional symptoms ,there was history of
recurrent antibiotic use for UTI; last course was 2wks
ago.
Concern is it cancer?
My differential: antibiotic induced
pseudomembraneous colitis ,infective diarrhea,
autonomic neuropathy as she mentioned the diarrhea
wakes her at night.
Examiner was interested in something else they try to
push me to say something ,I didn't get them, but when
I finished they told me IBS
2.
A 54 yrs old lady with long standing joint pain for
knee replacement, all vitals were normal, pulse60,
Inside: joint pain is not specific multiple joints.
Surrogate did not know anything about her
everything. When asked why u r here? She replied i
was sent by the surgical unit but I don't know why.
Examination noooooormal ,fit lady , then 2mins left,
and the examinar was very annoyed then he told me
she has fatiguability does it give u any hints,
I digged and found she is hypothyroid. I had to check
the neck and the thyroid status, she had a scar in the
neck the examiners were surprised about it then they
told me it was from trauma to the neck
less than one min, concerned about can we do the
surgery answered the concern and then simple
questions how to investigate, she had diabetes then
what could be the etiology for her hypothyroid, I
scanned her for autoimmune illnesses in the history
earlier.
28/28
Station 1:
CVS :
DVR very clear & straightforward
20/20
Neurology :
veeeeeery difficult case young girl 15 yrs of age.
Instructions: examine this patient neurologically
OK from where to start!!
Face nothing impressive ,she was very shy and i guess
low IQ, very uncooperative, laughing, globally wasted
upper and lower limbs, with pes cavus, power strong,
tone normal, reflexes normal, gait normal,
One min left ,cerebellum one side a bit impaired, time
over..
Qs:
Summarize your findings, very confused thoughts but
tried to organize it. Then I noticed that she is moving
the shoulders strange enough?? What could be
the cause.
I said young girl with possible cerebellar so
hereditary Stacia's, abnormal movement no localizing
sign so chorea is possible ,
What could be the cause I replied Wilson's or
rheumatic fever
How to investigate ,time over..
Surprisingly I got 16/20 I thought only 8
Al7amdolellah
Station 2:
A40 yrs old deliveryman with lower limb swelling
,blood pressure normal urine ++ of protein.. see and
advise.
Inside very clear nephrotic syndrome with all the
water retention symptoms, start to look for the cause,
diabetic but very controlled ,then asked about
vasculitis, there was history of joint pain for six
months did not seek medical advise, taking over the
counter ibuprofen 400 my tds,
Now clear picture of either vasculitis causing
nephrotic, Rheumatoid arthritis causing nephrotic, or
drug induced nephrotic syndrome
The discussion was on the management.
Got 16/20
Station 3:
Abdomen: A young cachexic lady, very pale, with
maaaaasive ascites , not jaundice and no signs of
chronic liver disease & no lymph nodes, Differential:
decompensated CLD but no signs of liver disease,
heart failure but no peripheral oedema I did even
examine the lung bases after permission of the
examiners as I finished early no lower limb oedema.

Abd : young lady cachexic , very pale, with


maaaaasive ascites , no jaundice or signs of chronic
liver disease no lymph nodes, differential :
decompensated CLD but no signs of liver
disease,heart failure but no peripheral oedema I did
even examine the lung bases after permission of the
examiners as I finished early no lower limb oedema &
infections like TB.
Renal disease also remote possibility.
Station 4:

The nurse with conversion disorder ,stroke ,imaging


normal ,she heard the nurses sayi
Khartoum PACES, December 2016
Day 2 - last cycle 3
Station 4,
Communication Skills :-
Dealing with an angry son, his mother underwent hip
replacement , on rehabilitation centre she developed
headache , the doctor suspect gaint cell artritis he
gave her 40 mg predonsolone. She developed steroid
induced psychosis , they reffered her to the general
hospital where she was seen by the rheumatologist
who ruled out gaint cell artertis , and started to
tapper the steroid.
Her son at home and they didn't call him at night &
he is ungrey for that..
his concern , is it reversable
and could my mother returned back to the
rehabilitation program ?
I got 16/16
Station 5
BCC1
A 70 yrs male with joint problem since 15 yrs
developed SOB and cough for 6 months , inside the pt
had obvious RA with deformities , on Hx he is on
methoteraxate ,
O/E had creps all over the chest Concern is it heart
attack?
I got 26/28
BCC2:
35 yrs female developed pain on both hands with
change in appearnce , in side when i went to greet her
i saw fingers deformities and skin rash , it is psoriasis
wih psoriatic arthropathy
I got 27/28
Station 1:-
Abdomen , obese female pt treated for anaemia ,
had nonfunctioning fistula, she had ascites with
peritoneal dialysis cath ,
Iam short stature , the female is obese , the couch
is high, i couldn't palpate the Lt kidney but the right
is palpable , APKD
I got 18/20
Chest:-
Obvious features of scleroderma with bibasal fibrosis
I got 20/20
Station 2:-
35 yrs male with type 1 DM had collapse & black out
, he had all macro & microvascular & autonomic
complications, it's a long Hx
the concern , what could be the cause ,
I finshed the Hx without managing his concern , time
over ,
I got 17/20
Station 3:-
Neuro,
sensory motor peripheral neuropathy , I got 20/20
Cardio:-
midsterniotomy scar , the pt had ESM at aortic area ,
itis prosthetic aortic valve ,
I got 20/20

YANGON CENTRE
Day 1 (7.11.16)
Station1
Resp: male pt, cough & SOB for 3 months
O/E: clubbing,dullness percussion, reduced VBS, no
ronchi,no crepts
I gave first consolidation.
Examiner ask DDx
I gave pleural effusion,pulmonaryfibrosis, Ca lung
Pleural thickening
Invx.

Khartoum paces exam


December 2016
St:3
NEURO:MND e bulbar(dysarthria.tongue
Fasciculations)
Cardio:Mid sternotomy scar+PSM rad to
axilla+pulm htn
AF
St4:Physical trainer.with cerebellar
features.MRI:highly suggestive MS
BCC1:Behcet +DVT+PHTN
BCC2: RA came e Back P sudden onset due steroid
use
OE :CLUBBED hands
Chest full of wheeze and crackled
Examiner wanted bronchiolitis obliterans
St1:chest:chest full ofcoarse crackles +wheeze sputom
pot sputum yellowish colour
Examiner wanted lung fibrosis
Abdomen :huge liver
Polycyctic liver
St2 Hx:lithium toxicity

08/10/2016
Muscat
St 01
Res: ILD with obvious clubbing
Abd:
Young boy with l/s Polycyclic kidney
St2
35yrs old lady with left sided weakness of the body
lasted for one hour. Only positive thing in the history
was taking ocp and headache with the onset of
symptom
St3
Cvs
MVR WITH Recent pacemaker insertion (pt
tachycardic)
Cns
45yrs old man with difficulty in walking. Proximal
weakness more than distal.
Plantar down going.reflexes are very sluggish.
Sensation intact. No cerebellar sign.as pt unable to
walk could not check gait......myopathy...
St4
52yrs old lady known case of AF on warfarin
investigated for anaemia .colonoscopy bx revealed
ca.no distal metastasis.to break the bad news.
Bcc1
Young pt with loss of vision at night.
Retinitis pigment Osama
Bcc2
Pt with numbness of the both feet
Diabetic peripheral neuropathy with charcot joint

Station 1
Middle aged man with
progressive SOB.
On examination patient was dyslexic with
Rheumatoid hand and fine end inspiratory
crepts
Questions What's are the finding ,how will
you investigate and manage.
20/20
Abdomen was a young male with icterus
and Spenomegaly
Again same questions
Finding how will you investigate
I gave a diagnosis of Thalassemia but I seem to have
missed
Some physical findings
How will you manage
What are the complications of blood
transfusion
Score 16/20
Station 2
35 year old male working as a financial
consultant with frequent travel to Africa
Had history of drenching night sweats
and weight loss
On taking history had past history of
travel with poor compliance of taking
prophylaxis for malaria
No risk factors for HIV
Travel to urban areas of Africa
Gave d/d of TB,Lymphomas,chronic malaria
Discussion on TB investigation
Concerns can it be cancer
Score 20/20
CVS: was a tough one not sure of
diagnosis
Discussion indications of valve replacement
9/20
Neuro: Young female with complaints
of having problems with vision.Examine
cranial nerves
On examination patient had right sided
homonymous Hemianopia
Discussion on where could be the lesion
Causes in young female
Investigation you would ask for
20/20
Station 4
Was a long scenario
Mr A is a known case of COPD admitted with acute
exacerbation in HDU. Was started on iv antibiotics.
Culture were negative.
He is not doing well and has developed generalised
body swelling and started on diuretics
Patient is mildly confused. The treating Consultant is
of the view that Mr A does not have a good prognosis
though he has not yet spoken to ICU and no decision
for or against.Also Mr A continued to smoke even
after previous admissions. Intubation has been taken.
Job is to Speak to daughter for which Mr A had given
consent, discuss the management plan and prognosis .
Daughter wanted MR A to attend a wedding which
was after 3 months and Mr A had expressed that
everything be done to help him live longer.
Discussion was on whether to intubate and is always
intubation difficult to waen from MV
Who will take the final decision to intubate or not
If patient is not confused does he have a say
Station 5 case 1
Middle aged man known case Of DM since 3 years
complaining of alteration of sensation on left lateral
thigh.
On OHA
Duration 2 weeks
On examination absent sensation on lateral side of left
thigh( distribution of lateral cutaneous nerve of thigh
No other abnormalities
Diagnosis: Meralgia Parasthetica
Concern : Is it due to diabetes or Metformin
Discussion investigation and management
28/28
Case 2
50 year old female c/o SOB and difficulty in
swallowing
On examination
Systemic sclerosis
Concern will it worsen
Discussion on investigation and management
24/28
##########################################
Detailes of the communication case
As mentioned before the written scenario was long
(full A4 page)
Job was to explain to daughter the management
done,prognosis and future plan of management.
After confirming the identity and being next of kin, I
asked her how much she knew
Of her father's disease
She said that he had been suffering from breathing
difficulties and had several previous admissions after
which he would improve and would be discharged
home
However he would continue to smoke(which was also
mentioned in the scenario
Taking the discussion further I asked whether her
father had any advance directives .she replied that
her father had expressed his view that everything
possible be done to help him live longer
Then I explained to her the present clinical status of
her father and during the present admission he is not
doing well and tried to show some empathy
Also her father had developed complications in
simple terms with generalised swelling and confusion
Daughter then asked me Why we are not shifting him
to ICU
I then explained that the treating Consultant views
that he did not have a good prognosis at the same
time repeatedly showing empathy
Though the consultant has not yet taken a decision
I tried to tell her that once on the breathing
machine,such patient are difficult to wean off
At warning of 12 minutes I asked her for her concern,
she said she wanted her father to attend marriage
ceremony which was due after 3 months
At this I summarised the discussion and told her I
would be informing my consultant of the discussion
and also inform him about your consent as well as
patient desire that everything that can be done to be
done for himm
And the end I said I would be leaving my contact
details and she was free to contact me
For any new concern she may have
Discussion with Examiner was centred mainly on
intubation
Of such patients
Who will take the decision for shifting the patient to
ICU
Do the family members have a say in Taking a
decision on Intubation and ICU tranfer
Suppose If the patient was not confused
Will he have a say in a decision of his transfer to ICU
and intubation
Is it always that COPD patient are not to be intubated
and are difficult to wean off
As usual the examiners were expressionless
To be truthful I was not sure how I have done but
Alhamdullilah got 16/16
My experience at leicester royal infirmary,uk on
20/11/2016
Cardio..severe mr 2ndry to rheumatological
disease...19/20
Neuro..upper limbs..proximal myopathy...20/20
Respiration..left upper lobectomy..13/20..i totally
missed lobectomy scar.
Abdomen...myelofibrosis..20/20
Bcc1.....psoriatic arthropathy with both hands n feet
involvement..23/28
Bcc2...microscopic hematuria...23/28
History..angioedma...02/20...?
Communication...psychogenic hemiparysis...05...?
Total...125
I failed...

Malta 1st day 1st carousel


december 2016
Chest
pneumonectomy e lung cancer
Abd
HSM CLD
Hitory
uncontrolledHTN
Cardio
AS MR
Neuro
Heam. Heamanopia
Comm
COPD terminal for discharge
BCC1
migrain e headache analgesic misuse vs tension
headache
2 infective diarrhoea in pt. E crohns received ABX
pseudomem.cholitis

Manipal hospital- Bangalore,INDIA, day 2


Station 4
26yrs old lady physiotherapy as st. Working in stroke
unit .admitted with flaccid limb weakness , CT and
MRI normal explain about functional weakness . Pt
was reluctant to accept the diagnosis and wish to see
neurologist urgently don't want to see psychiatrist
.social issue about job , and grandmother died 3
months back with stroke.discussed about psychiatric
referral and physiotherapy
Station 5
1 st case 30 yrs old lady with high prolactin levels and
normal TSH c/o scanty and irregular menstruation.
2nd case- 26 yrs old lady with SLE since 6 yrs
presented with right sided pleuritic chest pain ,with
fever. Discussion about DD of chest pain.
Station 1
- Respiratory -- lung fibrosis
Abdomen = ascites with chronic liver disease ,
jaundice, parotid swelling,flapping tremor, spider
nevi , examiner asked about if there is fever what can
be the cause and how to treat
.management of ascites .
Station 2
30 yrs old lady with facial and neck swelling sudden
onset ,adopted child , no other positive history ,
concern about allergy .DD- hereditary angioedema .
Investigation and treatment .
Station 3
Neuro - right sided weakness , with proximal wasting
hypertonia, hyperreflexia ,dyddiadokokinesia
,sensory normal .
Cardio -- young lady with MS - tapping apex sinus
rhythm ,loud S1 diastolic murmur , phtn and raised
jvp .

Copied from telgram channel of


Dr. Mahmoud Abo-Khadija
** Paces uk study group** On Telegram
1. Station 1
A. Respiratory : ILD due to SLE in a male patient.
Fine end inspiratory Crackles were heard on
auscultation in both lung fields, more prominent on
left upper to mid zone.
Patient had no other stigmata, only had hairloss of
scalp and eyebrow.
Asked about causes , investigations and management.
I scored 19/20.
B. Abdomen: a middle aged lady with AV fistula ,
active, old scars in her neck for venous access, huge
ascites with everted umbilicus. There was a mass
palpable and ballotable even though there was huge
ascites.
I carefully percussed and tried to figure out the get
above the swelling and was quite confident about
palpable kidney.
So it was actually an ESRF on MHD patient with
ADPKD with huge ascites .
I got 19/20 lost one mark probably for not checking
fluid thrill , i thought only shifting dullness would be
enough. But examiners asked about fluid thrill and
probably cut 1 mark in clinical examination.
2. History station
A 24-year-old lady presented with fatigue and
tiredness , lab reports suggestive of IDA, has history
of menorrhagia , no thyroid problem. This lady has
been receiving treatment for IBS
A nice surrogate , i elicited all the informations acc to
the format which helped me find out the actual
reason of anaemia was malabsorption, it was celiac
disease but the were no precipitant food item. I
excluded all the causes of Malabsorption.
Crossings were centred around DDs , reasons to
establish celiac as a cause , investigations and mx of
celiac in 3 words.
Got 20/20
Station 3:
A. Cardio :
Middle aged man with mid line sternotomy scars ,
audible metallic click from outside, 2nd heart sound
replaced by metallic click and a flow murmur more
prominent in aortic region.
Completed examining him in 5 minutes and i was
asked hundreds of questions regarding aoric and
mitral valve diseases , indications, signs of severity,
Investigations, management , INR.
20/20
B. Neuro :
Young lady with stroke - clonus/ spasticity/ hyper
reflexia / weakness / extensor plantar- of left side /
intact sensory, didnt let me check the gait. I asked to
see the upper limb and allowed to inspect only and
found her to have left sided hemiplegic posture .
Asked permission to check pulse , carotids and
precordium along with thorough history onset and
progression. Times up then.
Questions were around DD , causes of young stroke,
cardiac causes of stroke, whether it was an embolic or
thrombotic one. Asked y would i wanted to check
pulse carotids and precordium. I was asked to imitate
a hemiplegic gait. Investigation and management.
Examiners seemed to be really pleased
Got 20/20
Station 4:
Task was to talk to the daughter of a 70-year-old
gentleman who was diagnosed with parkinsons for 7
years. He got admitted with fall 2 days back . Nurse
observed today that the patient is increasingly stiff
and having difficulty with standing up from chair. He
also had some cough and abnormal breathing. Nurse
on duty today checked and found out that patient was
not given stelavo since admission as the medication
was not available in the pharmacy.
My task was to explain the mishap and discuss
further management.
It was difficult, had to cover a lot of thing, managed
time well completed it in 14 minutes
Apologised 1st , calm the angry surrogate, discussed
about the occupational health team , SALT team ,
neurologist , adressed chest infection , managed her
concern about hospital acquired infection, advanced
directive related issues , social care , asked the lady
for any help.
Examiners were discussing regarding the issues and
answered everything confidently.
I was happy about the station as i thought this was a
completely new and unusual scenario, it would be
tough. But it went well.
16/16.
Station 5
A. 34 year old lady presented with fever and weight
loss. I asked for all possible details including exposure
history and travel. Sign i could elicit was only an
anterior cervical chain lymphadenopathy. Asked for
temperature chart.
DD given TB lymphoma leukaemia HIV
Crossing was all about investigation, histological
finding of tb lymphadenitis. Out line of management.
28/28
B. A 58-year-old lady presented with bluish
discoloration of fingers and shortness of breath. It
was a case of systemic sclerosis with ILD.
Asked about investigation and management.
So its quite possible to obtain full marks in all the
stations. Just you need to be well rehearsed and little
bit of good luck to get good cases and cordial
examiners.

EXAM Malta today


11DECEMBER 2016
St1
(respiratory OLD with pulmonary fibrosis
Abd transplant kidney
St2
haematemesis in heavy alcoholic drinking
St3
AS cardio
Neuro cerebellar SYND
ST4
discuss delayed of cancer discover in a lady came with
vomiting of blood &many investigation done do her
(endoscopy, x-ray &Abd US )then CT chest
&abdomen show cancer in carina with mets
Discuss with her daughter even if cancer discover
early it's for palliative treatment
ST 5
(back pain with fever in old lady with Hx of back
surgery 6 wks ago
2nd one us caeliac disease (loose motion with
anaemia)

PACES EXAM MALTA


11 DECEMBER 2016
St 3
Cardio = MIXED (AS & MR)
Neuro PARKINSON dis
St2
SOB (COPD)
St 1
Abd ,,, ABDOMINAL MASS
Chest pneumonectomy
St5
(night sweat )
Asthma uncontrolled with charge straous
St 4
ca pancreas (delayed diagnosis &for palliative
treatment )came with multiple visit with Abd pain
lastly CT done &show ca
Pt angry &want complain

Thanks for this group for much support. My exam


second day first round in Mandalay centre (
15.11.16).
Station 2
Analgesic induced headache with underlying
migraine
CVS- ESM at apex with radiation to mid axilla &
ESM at aortic area with no radiation no AF No
HEart failure no IE- I give MR . Other candidate give
MR too. DX- MR
CNS- prompt- examine cranial nerve- right 12 CN
palsy with hard voice with salivation & right hand
small muscle wasting with claw hand
Only have time for finger adduction& fromen sign.
No time for motor& sensory& jerk
Examiner ask me what do you want to do if I have
time ? Said sensory& cerebellar & motor.
Common cause in her middle age - MS,
syringomyelia, pseudo bulbar DX- Syringobulbia :)
St4- Steven Johnson due to penicillins during birth
Abd- ascites with no organomegaly- COL with portal
hypertension
Respi- Rt lower lobe consolidation
St 5- 1)Acromegaly
2) fatigue all the time with RA deformed hand in
middle age female with bilateral ptosis with I think
pallor
I rule out MG with evening worse fatigue ,
hypothyroid , Addison,
Drug history- NSAID according to doc three times
per day with increasing pain in her hand
Can't find anything- give DX as analgesic induce
blood loss
Anaemia due to chronic diseases
I passed

MALTA paces exam


December 2016
ST 5
1= fresh bloody diarrhea==crohn dis
2=HEADACH==may be migrain or medication
overuse
Station 1
Respiratory = Interstitial lung dis
Abd =hepatosplenomegally
ST 2
History = poor control ABP
important to say in ttt AMPULATORY &HOME BP
monitoring
ST 3
Cardio=Mitral reg
CNS = Examin the EYE????!!!
ST 4
communication ==Angry son refusing Discharge of
his mum from hospital

MALTA
DECEMBER 2016
St5
(night sweat )
Asthma uncontrolled with charge straous
St2
sob (copd)
St 3
Ms with Af
Neuro Essential tremer
St 1
Abd hernia
Chest pneumonectomy
St 4
ca pancreas (delayed diagnosis &for palliative
treatment )came with multiple visit with Abd pain
lastly CT done &show ca
Pt angry &want complain
EXAM Malta today
2nd carousel
11DECEMBER 2016
St1
respiratory Pnumonectomy
Abd Abd mass
St2
haematemesis in heavy alcoholic drinking
St3
cardio VSD
Neuro spastic paraparesis with no sensory level
ST4
discuss delayed of cancer discover in a lady came with
vomiting of blood &many investigation done do her
(endoscopy, x-ray &Abd US )then CT chest
&abdomen show cancer in carina with mets
Discuss with her daughter even if cancer discover
early it's for palliative treatment
ST 5
(back pain with fever in old lady with Hx of back
surgery 6 wks ago
2nd one us caeliac disease (loose motion with
anaemia)

2016/3
17/11/2016 Chennai (Sundaram Medical Foundation
Hospital)
Station 3
CVS MVR
20/20
CNS Examine cranial nerve: Bell palsy
18/20
(I cannot answer when examiner asks me why there is
loss of nasolabial fold on the contralateral side and
role of nerve conduction study for 7th nerve palsy)
Station 4, End stage COPD lady, Admitted for Type 2
respiratory Failure, not responsive to non-invasive
ventilation and getting deteriorating. pt has her own
nebulizer and oxygen cylinder at home. Patient is
keen for self discharge (mentally competent) Task is
to speak to the son and explain about self discharge.
It is like breaking bad news. I did badly and I thought
I will fail this station as I have left nearly 5 minutes
by the time I finished all the tasks and solved his
concerns : (
13/16
Station 5
BCC 1 65 yr old lady with knee pain > 6 months
Dx: likely OA
28/28
BCC2 30 yr old man, present with fever ,cough X 1
month (pt give history of night sweats, poor appetite,
weight loss, hemotypsis, but no exposure to TB, no
risks of contracting HIV ) Examination findings
seems to be normal.
DDx TB, Ca Lungs, Lymphoma
26/28
Resp
COPD with bronchiectasis
I did badly in this station because of limited time,
Unfortunately, I started examination at the front
which I found nothing abnormal, except reduced
cricosternal distance and barrel shaped chest wall
which is not quite obvious..There was no clubbing/no
cyanosis on general inspection.
When I started to check the patient's back, only 1
minute left, so, I listened only his back which I heard
crep on the right lung base.
But examiner asked me whether crep is unilateral or
bilateral.So seem like bilateral.
17/20
Abdomen
This is my worst station.
Young Lady with functioning AVF (but not recently
used) , no features of chronic liver disease.
I found only very small splenomegaly which I am not
quite confident to tell the examiners.
Examiners lead me questions about Chronic Liver
Disease.
13/20
Station 2
30 year old lady, mild anaemia (normochronic
normocytic anaemia), BP 140/80, present with
Fatigue
Dx: SLE with Antiphospholipid antibody syndrome.
main concerns: pregnancy
20/20
Total 155/172 and I passed PACES finally!
Thanks to my parents, my teachers, friends and
colleagues, especially my husband who helped me
intensively before my exam. Without his support and
encouragement, I won't be able to pass this exam.
Thank you everyone!
Malta 1st day 1st carousel
december 2016
Chest
pneumonectomy e lung cancer
Abd
HSM CLD
Hitory
uncontrolledHTN
Cardio
AS MR
Neuro
Heam. Heamanopia
Comm
COPD terminal for discharge
BCC1
migrain e headache analgesic misuse vs tension
headache
2 infective diarrhoea in pt. E crohns received ABX
pseudomem.cholitis
Manipal hospital- Bangalore,INDIA, day 2
Station 4
26yrs old lady physiotherapy as st. Working in stroke
unit .admitted with flaccid limb weakness , CT and
MRI normal explain about functional weakness . Pt
was reluctant to accept the diagnosis and wish to see
neurologist urgently don't want to see psychiatrist
.social issue about job , and grandmother died 3
months back with stroke.discussed about psychiatric
referral and physiotherapy
Station 5
1 st case 30 yrs old lady with high prolactin levels and
normal TSH c/o scanty and irregular menstruation.
2nd case- 26 yrs old lady with SLE since 6 yrs
presented with right sided pleuritic chest pain ,with
fever. Discussion about DD of chest pain.
Station 1
- Respiratory -- lung fibrosis
Abdomen = ascites with chronic liver disease ,
jaundice, parotid swelling,flapping tremor, spider
nevi , examiner asked about if there is fever what can
be the cause and how to treat
.management of ascites .
Station 2
30 yrs old lady with facial and neck swelling sudden
onset ,adopted child , no other positive history ,
concern about allergy .DD- hereditary angioedema .
Investigation and treatment .
Station 3
Neuro - right sided weakness , with proximal wasting
hypertonia, hyperreflexia ,dyddiadokokinesia
,sensory normal .
Cardio -- young lady with MS - tapping apex sinus
rhythm ,loud S1 diastolic murmur , phtn and raised
jvp .

Copied from telgram channel of


Dr. Mahmoud Abo-Khadija
** Paces uk study group** On Telegram
1. Station 1
A. Respiratory : ILD due to SLE in a male patient.
Fine end inspiratory Crackles were heard on
auscultation in both lung fields, more prominent on
left upper to mid zone.
Patient had no other stigmata, only had hairloss of
scalp and eyebrow.
Asked about causes , investigations and management.
I scored 19/20.
B. Abdomen: a middle aged lady with AV fistula ,
active, old scars in her neck for venous access, huge
ascites with everted umbilicus. There was a mass
palpable and ballotable even though there was huge
ascites.
I carefully percussed and tried to figure out the get
above the swelling and was quite confident about
palpable kidney.
So it was actually an ESRF on MHD patient with
ADPKD with huge ascites .
I got 19/20 lost one mark probably for not checking
fluid thrill , i thought only shifting dullness would be
enough. But examiners asked about fluid thrill and
probably cut 1 mark in clinical examination.
2. History station
A 24-year-old lady presented with fatigue and
tiredness , lab reports suggestive of IDA, has history
of menorrhagia , no thyroid problem. This lady has
been receiving treatment for IBS
A nice surrogate , i elicited all the informations acc to
the format which helped me find out the actual
reason of anaemia was malabsorption, it was celiac
disease but the were no precipitant food item. I
excluded all the causes of Malabsorption.
Crossings were centred around DDs , reasons to
establish celiac as a cause , investigations and mx of
celiac in 3 words.
Got 20/20
Station 3:
A. Cardio :
Middle aged man with mid line sternotomy scars ,
audible metallic click from outside, 2nd heart sound
replaced by metallic click and a flow murmur more
prominent in aortic region.
Completed examining him in 5 minutes and i was
asked hundreds of questions regarding aoric and
mitral valve diseases , indications, signs of severity,
Investigations, management , INR.
20/20
B. Neuro :
Young lady with stroke - clonus/ spasticity/ hyper
reflexia / weakness / extensor plantar- of left side /
intact sensory, didnt let me check the gait. I asked to
see the upper limb and allowed to inspect only and
found her to have left sided hemiplegic posture .
Asked permission to check pulse , carotids and
precordium along with thorough history onset and
progression. Times up then.
Questions were around DD , causes of young stroke,
cardiac causes of stroke, whether it was an embolic or
thrombotic one. Asked y would i wanted to check
pulse carotids and precordium. I was asked to imitate
a hemiplegic gait. Investigation and management.
Examiners seemed to be really pleased
Got 20/20
Station 4:
Task was to talk to the daughter of a 70-year-old
gentleman who was diagnosed with parkinsons for 7
years. He got admitted with fall 2 days back . Nurse
observed today that the patient is increasingly stiff
and having difficulty with standing up from chair. He
also had some cough and abnormal breathing. Nurse
on duty today checked and found out that patient was
not given stelavo since admission as the medication
was not available in the pharmacy.
My task was to explain the mishap and discuss
further management.
It was difficult, had to cover a lot of thing, managed
time well completed it in 14 minutes
Apologised 1st , calm the angry surrogate, discussed
about the occupational health team , SALT team ,
neurologist , adressed chest infection , managed her
concern about hospital acquired infection, advanced
directive related issues , social care , asked the lady
for any help.
Examiners were discussing regarding the issues and
answered everything confidently.
I was happy about the station as i thought this was a
completely new and unusual scenario, it would be
tough. But it went well.
16/16.
Station 5
A. 34 year old lady presented with fever and weight
loss. I asked for all possible details including exposure
history and travel. Sign i could elicit was only an
anterior cervical chain lymphadenopathy. Asked for
temperature chart.
DD given TB lymphoma leukaemia HIV
Crossing was all about investigation, histological
finding of tb lymphadenitis. Out line of management.
28/28
B. A 58-year-old lady presented with bluish
discoloration of fingers and shortness of breath. It
was a case of systemic sclerosis with ILD.
Asked about investigation and management.
So its quite possible to obtain full marks in all the
stations. Just you need to be well rehearsed and little
bit of good luck to get good cases and cordial
examiners.

EXAM Malta today


11DECEMBER 2016
St1
(respiratory OLD with pulmonary fibrosis
Abd transplant kidney
St2
haematemesis in heavy alcoholic drinking
St3
AS cardio
Neuro cerebellar SYND
ST4
discuss delayed of cancer discover in a lady came with
vomiting of blood &many investigation done do her
(endoscopy, x-ray &Abd US )then CT chest
&abdomen show cancer in carina with mets
Discuss with her daughter even if cancer discover
early it's for palliative treatment
ST 5
(back pain with fever in old lady with Hx of back
surgery 6 wks ago
2nd one us caeliac disease (loose motion with
anaemia)
Oman 10/4 cycle 1
Station 3 :
Cardio MVR
CNS CMT
Station 4 :
New diagnosis of SLE
proteinuria 4 g/ L haematouria
Discuss the diagnosis and renal biopsy
Station5 1:
Diarrhea nail lesion skin rash joint pain in hands
? Psoriatic arthropathy
2: dysphagia thyroid nodule
euthyroid, no retrosternal extension
At the end the examiner asked would you expect a
small goitre to cause dysphgia
Station 1: chest lt side bronchiactasis
Abd: ill pt 2 paramedian scars multple other scars
Rt iliac fossa mass i gave differential diagnosis
transplanted kidney? Appendicular mass? Chrons?
Caecal mass? Discussion was about transplanted
kidney
Station 2: pt 29 male hx of asthma and type2 DM CO:
recurrent chest infection
In hx: cough productive of large amount sputum..
bulky difficult to flush motions.. DM on insuline.. hx
of infertility.. I gave diagnosis of CF what other
possible causes

PACES EXAM MALTA


11 DECEMBER 2016
St 3
Cardio = MIXED (AS & MR)
Neuro PARKINSON dis
St2
SOB (COPD)
St 1
Abd ,,, ABDOMINAL MASS
Chest pneumonectomy
St5
(night sweat )
Asthma uncontrolled with charge straous
St 4
ca pancreas (delayed diagnosis &for palliative
treatment )came with multiple visit with Abd pain
lastly CT done &show ca
Pt angry &want complain

Thanks for this group for much support. My exam


second day first round in Mandalay centre (
15.11.16).
Station 2
Analgesic induced headache with underlying
migraine
CVS- ESM at apex with radiation to mid axilla &
ESM at aortic area with no radiation no AF No
HEart failure no IE- I give MR . Other candidate give
MR too. DX- MR
CNS- prompt- examine cranial nerve- right 12 CN
palsy with hard voice with salivation & right hand
small muscle wasting with claw hand
Only have time for finger adduction& fromen sign.
No time for motor& sensory& jerk
Examiner ask me what do you want to do if I have
time ? Said sensory& cerebellar & motor.
Common cause in her middle age - MS,
syringomyelia, pseudo bulbar DX- Syringobulbia :)
St4- Steven Johnson due to penicillins during birth
Abd- ascites with no organomegaly- COL with portal
hypertension
Respi- Rt lower lobe consolidation
St 5- 1)Acromegaly
2) fatigue all the time with RA deformed hand in
middle age female with bilateral ptosis with I think
pallor
I rule out MG with evening worse fatigue ,
hypothyroid , Addison,
Drug history- NSAID according to doc three times
per day with increasing pain in her hand
Can't find anything- give DX as analgesic induce
blood loss
Anaemia due to chronic diseases
I passed

MALTA paces exam


December 2016
ST 5
1= fresh bloody diarrhea==crohn dis
2=HEADACH==may be migrain or medication
overuse
Station 1
Respiratory = Interstitial lung dis
Abd =hepatosplenomegally
ST 2
History = poor control ABP
important to say in ttt AMPULATORY &HOME BP
monitoring
ST 3
Cardio=Mitral reg
CNS = Examin the EYE????!!!
ST 4
communication ==Angry son refusing Discharge of
his mum from hospital

MALTA
DECEMBER 2016
St5
(night sweat )
Asthma uncontrolled with charge straous
St2
sob (copd)
St 3
Ms with Af
Neuro Essential tremer
St 1
Abd hernia
Chest pneumonectomy
St 4
ca pancreas (delayed diagnosis &for palliative
treatment )came with multiple visit with Abd pain
lastly CT done &show ca
Pt angry &want complain

EXAM Malta today


2nd carousel
11DECEMBER 2016
St1
respiratory Pnumonectomy
Abd Abd mass
St2
haematemesis in heavy alcoholic drinking
St3
cardio VSD
Neuro spastic paraparesis with no sensory level
ST4
discuss delayed of cancer discover in a lady came with
vomiting of blood &many investigation done do her
(endoscopy, x-ray &Abd US )then CT chest
&abdomen show cancer in carina with mets
Discuss with her daughter even if cancer discover
early it's for palliative treatment
ST 5
(back pain with fever in old lady with Hx of back
surgery 6 wks ago
2nd one us caeliac disease (loose motion with
anaemia)

2016/3
17/11/2016 Chennai (Sundaram Medical Foundation
Hospital)
Station 3
CVS MVR
20/20
CNS Examine cranial nerve: Bell palsy
18/20
(I cannot answer when examiner asks me why there is
loss of nasolabial fold on the contralateral side and
role of nerve conduction study for 7th nerve palsy)
Station 4, End stage COPD lady, Admitted for Type 2
respiratory Failure, not responsive to non-invasive
ventilation and getting deteriorating. pt has her own
nebulizer and oxygen cylinder at home. Patient is
keen for self discharge (mentally competent) Task is
to speak to the son and explain about self discharge.
It is like breaking bad news. I did badly and I thought
I will fail this station as I have left nearly 5 minutes
by the time I finished all the tasks and solved his
concerns : (
13/16
Station 5
BCC 1 65 yr old lady with knee pain > 6 months
Dx: likely OA
28/28
BCC2 30 yr old man, present with fever ,cough X 1
month (pt give history of night sweats, poor appetite,
weight loss, hemotypsis, but no exposure to TB, no
risks of contracting HIV ) Examination findings
seems to be normal.
DDx TB, Ca Lungs, Lymphoma
26/28
Resp
COPD with bronchiectasis
I did badly in this station because of limited time,
Unfortunately, I started examination at the front
which I found nothing abnormal, except reduced
cricosternal distance and barrel shaped chest wall
which is not quite obvious..There was no clubbing/no
cyanosis on general inspection.
When I started to check the patient's back, only 1
minute left, so, I listened only his back which I heard
crep on the right lung base.
But examiner asked me whether crep is unilateral or
bilateral.So seem like bilateral.
17/20
Abdomen
This is my worst station.
Young Lady with functioning AVF (but not recently
used) , no features of chronic liver disease.
I found only very small splenomegaly which I am not
quite confident to tell the examiners.
Examiners lead me questions about Chronic Liver
Disease.
13/20
Station 2
30 year old lady, mild anaemia (normochronic
normocytic anaemia), BP 140/80, present with
Fatigue
Dx: SLE with Antiphospholipid antibody syndrome.
main concerns: pregnancy
20/20
Total 155/172 and I passed PACES finally!
Thanks to my parents, my teachers, friends and
colleagues, especially my husband who helped me
intensively before my exam. Without his support and
encouragement, I won't be able to pass this exam.
Thank you everyone!

My exam was at chennai Sundaram medical


foundation on 18.12. 2016
Thanks all in this group. I got much help from you
all.
Started with st 2: young unmarried lady complaining
Transient Left arm weakness.....taking OCP....dx was
Hemiplegic migraine....dd was TIA. got 18/20
Then st 3: cvs case was AVR with CABG....discussion
was about causes of
AVR....inv.....treatment....warfarin.....INR...IE
prophylaxis....got 20/20
Neuro case was Stroke....findings were not so typical.
But old stroke may give such findings. So pls examine
stroke pt as many as possible. got 20/20.
st 4: ADPKD, concern was about perinatal
testing....family screening...surrogate was happy but
indian examiner was so tough.....asked so many
questions. got 10/16
St 5: 1. Neck lump....euthyroid....concern was about
surgery....discussion was about dd....indication of
surgery....got 28/28
St 5: 2. Chronic diarrhoea....strong family history of
CRC....RIF tenderness on examination....dd were
Chron's disease.....CRC.....discussion was about
investigations....got 27/28
St 1: Abdomen case was PKD. Discussion was about
associations.....mx. got 20/20
Resp case was COPD, bronchiectasis....discussion was
about treatment of COPD and complications of
bronchiectasis....got 20/20
Total score 163. Passed. Thank you all again. May
Allah bless us all.

Exam experience in Chennai


Nov 16--2016
St 1.
- Resp. Middle aged lady with COPD and
fibrocavitatory lesion.
Qs: causes of COPD, what are the possible cause of
the lesion (old TB, ABPS), what investigations to do
and how would you manage. When I was presenting
my findings I got confused and forgot to mention the
bronchial breathing and VR over the cavitatory
lesion. 15/20
- Abdomen. Young male with functioning AV fistula
and HSP, no signs of CLD. I wasn't sure exactly how
to link all findings together when asked. I suggested
2ry amyloidosis or CTD leading to ESRD. Also asked
what investigations would u do for him. 19/20
St 2.
Young male known to have asthma with worsening
symptoms over 4 months. The key point in history
was a new pet cat he purchased 3 months ago. His
concern was losing his job because of recurrent
absence.
Examiner asked me about differential diagnosis, tests
to be done. I said skin allergen test then he asked
about the latest test -> RAST. he asked about the
method of RAST test which I didn't know then he
went on to ask about the difference between atopy
and anaphylaxis but thankfully time was up. 19/20
St 3.
Neuro. Young lady with cerebellar syndrome and pes
cavus.
Qs: what is pes cavus associated with? Differential
diagnosis for this case (FA, MS, vascular, tumor)
What sensory findings to expect if she had friedrich's
ataxia -> peripheral neuropathy. What
Investigations? 20/20
Cardio. Confusing case. Lady with midsternotomy
scar, palpable S2, LPH. I couldn't hear any prosthetic
valve clicks or murmurs. S2 was split. I gave
differentail diagnosis of ASD with previous corrective
surgery or pulmonic valve disease and pulmonary
HTN. Examiner asked about investigations only. And
surprisingly I got 20/20
St 4.
25 yr old lady with diabetes was admitted to the
hospital with pneumonia and while she was admitted
ahe received the wrong type of insulin when
compared to her GP notes and developed only 1
episode of mild hypoglycemia. Task was to explain
the error to a "somewhat" angry patient.
Concern was having another hypoglycemic attacks at
home. Examiner asked who was responsible for the
error I said it was a medical team responsibility as
there are multiple factors leading to it. While the
doctor was overwhelmed in the emergency room his
senior or the nurse could have contacted the GP for
the medication list. Examiner seemed happy with that
response and asked what could have been done to
avoid it. I suggested a double signature system for
medications and a pharmacy policy to review GP
records of long term medications before prescribing.
16/16
St 5.
Case 1. Middle aged female with worsening exertional
dyspnea and ankle swelling. From history she said she
was hypertensive for 2 years but didn't take any
medications for it. On examination she had an
inframammary scar, raised jvp and bibasal crackles
as well as lower limb edema up to the knees. I
couldn't hear any murmur probably because she was
a little obese and I was rushing to address the
concerns. I said the pt was in heart failure, the scar
suggests mitral valve disease that was repaired and
probably recured. The other possibility is untreated
HTN. Examiner asked about management of heart
failure and hypertension. 23/28
Case 2. 30 yr old male with uncontrolled high BP
170/100.
History was only positive for similar problem in his
father who had high BP and developed an
intracranial hemorrhage. Examination was negative
but I forgot to check for radiofemoral delay so of
course the examiner asked about coarctation of the
aorta, what were the other diffentials (APKD,
phaeochromocytoma) and if I would admit him. 24/28
Overall it was a tough exam but I passed with the
praise of Allah
Exam experience in wishaw general hospital wishaw.
Glasgow centre 2nd November.2016
Unfortunately i failed. Marks obtained 124. I started
from station 5. 1st scenario was of a lady with RA
complaining of recent SOB and family physician did
chest xray which doesnt show any malignancy but
some scarring. She was on methotrexate and she had
pernicious anemia too. I found right sided fine creps
(i guess left sided too) i gave diagnosis of pulm fibrosis
secondary to either RA itself or methotrexate. Q:
D/D. i missed taking h/ o smoking and examiner
asked : did she smoke. I sa.id sorry i didn't ask, then
he asked, in case.of smoking what will be differential.
How will u manage. Marks 23/28.
BCC2: a lady with multiple swellings in neck and
some problem in mouth, dryness.(i couldn't
undesstand well about mouth features, i guess
decaying teeth. No characteristic lesions). She had
almost all neck glands palpable. (Submandibular,
parotid, cervical lymph nodes, some drs noticed
surgical scar below jaw which i didn't notice ). She
didn't have any weight loss etc (i gave d/ d of miculicz
syndrome as she had pernicious anemia also, thinking
autoimmune process and examiner asked.what else, i
said lymphoma, he asked which is most probable, i
said she doesn't have B symptoms :-) he said u didn't
ask about it in history =-O). Which made me nervous
and i couldn't give good management plan :'( marks
18/28.
Station 1: abdominal examination was
hepatospelnomegaly and i guess was simple.he had
tattoos. I didn't find any signs of CLD but now i
doubt that maybe there were spider nevi. :-P what is
dd.how will u investigate.13/20. respiratory was
classic COPD case, went fine. Q: d/ d of wheezy chest.
How will u differentiate asthma.and copd.
Investigation, management plan.20/20.
History: lady with lethargy and fatigue for 3 months.
Gp started citalopram and now referred for
hyponatremia 124, history exploration revealed
weight loss and cough (gp started managing as
asthma.). She also had diverticulosis previously and
family hx of ca colon. Was concerned about that. I
gave d/d of SIADH due to paraneoplastic syndrome,
citalopram induced hyponatremia. How will u
investigate. i did not managee concerns well. I didnt
tell her that it can be ca lung.marks 13/20.
Neuro : lady with.clumsiness, examine upper limb. I
found pill rolling tremor (in happiness i missed exam
routine for which they categorically asked.. what
about reflexes...proprioception, which i missed
examining. Key is to do full scheme of examination ).
Viva: d/d. How will u differentiate PD from
parkinson plus (and asked whether u looked for any
signs, which i missed checking ). 16/20.
CVS: man with midline sternotomy scar, has sob ( i
didn't concentrate on command so missed important
d/d), i gave diagnosis of tissue valve replacement as no
metallic click or vein harvesting scar. They asked why
he has sob. and i became confused that only then i
noticed that scenario was for sob. Anyhow i managed
to answer a bit but as sob was not in my mind so
cpuldnt notice valve functioning etc. People say he
had aortic stensoaia murmur which i didn't
appreciate. 9/20.
Station 4: counsel.angry son of patient. Patient had
femur fracture and she shifted to rehab where she
was started on steroids as doctor on duty suspected
giant cell arrteritis when lady complained of
headaches. After steroids she developed psychosis and
MDT thought that she should be shifted to hospital.
Son was next of.kin and was angry why she was
shifted without informing him and why dr gave
steroids and wanted to complain. Asked about further
management and issues. I guess i did well in it. Viva :
how did this meeting went, (plz notice that for station
4 usually examiner ask this question that how did this
neeting went). Asked about ethical issues and what
ethical issues u know. Asked me in real life if u
encounter an issue like this how u will manage. also
asked whether i missed some issues or not. 16/16. Key
points : dont miss exam.routine. plz follow full
scheme of examination. read scenarios and
instructions well. 5 Minutes be
MALTA paces exam
December 2016
ST 5
1=ASTHMA with pregnant afraid from steroids
2=TREMORS
Station 1
Respiratory = pneumonectomy
chest scar
Abd =hepatosplenomegally
ST 2
History = Diarrhea=IBS
But his father &grand father develop CANCER
COLON at age more than 60 y & uncle at age 39
discussion about COLONOSCOPY
ST 3
Cardio=Mitral reg+AORTIC STENOSIS
butgood volume pulse
CNS = spastic paraparesis +normal sensation
ST 4
communication =cancer pancrease = BBN
Khartoum paces exam
December 2016
Station4
lady with femur fracture operated and refered to
rehabilitation program and developed headache the
doctor gave her steroid as he is suspecting gaint cell
arteritis then she developed steroid induced psycosis ,
her son is angery talk to him and address his
concerns.
Station 2
young man with DM 1 for 18 years all diabetic
complication presented with recurrent attacks of
collapse
: Station 3 cvs
mid sternotomy scar in old lady with anaemia and
s.o.b ??
: Neurolog
young lady with inability to walk flaccid paraperesis
with distal wasting and peripheral neuorapathy
: Station 1 abdomen
ascites with peritoneal dialysis catheter D/D for
ascites
: Respiratory
lower lobe fibrosis in patient with scleroderma
: Station 5
old man with RA on methotrexate and NSAIDS
presented with S.O.B chest exam crackles all over the
chest
: The other one is lady who is having swelling and
pain of the hand and change in the apearance

PACES Exam in Khartoum 5th December


2016
Day 3
Cycle 3
ST1
Chest
Bronchiectasis
Abdomen
Cirrhosis with splenomegaly
ST2
Facial and tongue swelling and history of recurrent
abdominal pain. The patient is 28years old adopted
lady
ST3
Cvs
Young man with AR&MR WITH LUNG congestion
CNS
upper limb examination
Motor neuropathy (wasting. Hypotonia.
Hyporeflexia. Intact sensation. Inability to do
coordination )
ST4
Hodgkin lymphoma stage IIA
For Hickman line and chemotherapy
ST5
BCC1
JOINT pain with hoarseness of voice and muscle
cramp for 3 month. With history of thyroidectomy
The patient has obvious stridor
Discussion about causes and investigation
BCC2
Young lady with dysphagia for solid for 3 month in
patients with scleroderma
Discussion about differential and mangement
Khartoum paces exam
December 2016
ST 5
Foot drop On anti TB
Peripheral neuropthy
dizziness e blurr of vision
Inside she has has vomiting for 3wks
Time went she has missed cycle
Something to do with BIH exacerbated with the
pregnancy
Station 1
Respiratory Copd
Abd splenectomy&Pallor
Discussion was on the splenectomy
ST 2
History son of a father recently had been diagnosed e
can colon
Son is stressed had abd pain and erratic bowel habits
ST 3
Cardiovascular aortic valve replacement
CNS HSMn
ST 4
communication Steven johnson syndrome in a lad
Who delivered recently

This is the ( Amulet luck) of this Diet ::


St 4
Breaking Bad News to husband Whose wife received
inj penicillin for strep,,then allergy + organ failure

Bruni December 2016


my experience 3rd cycle brunei:
history : collapse , mostly cardiac.
communication : breaking bad news polycystic
kidnay for renal dialysis.
cardiac : double vavle replacement.
respiratory: ILD but the examiner ask what the
cause? patient has ammutputed leg and 3 digit in
both side , no idea
abdomin: poly cystic kidney, flank mass with
fuctional AV fitula.
neuro: really bad , no idea , one leg there some
weakness , almost not detectable with exaggerated
knee reflex but absent ankle jerk , sensation intact...
station 5
bbc 1: hx of short period of vertiogo , nothing else ,
now he is free, asked for d(we meet the guy outside
and he said i have nothing...)
bbc 2 : skin rash , SLE but i thinks there levido
Reticularis ?
dont know
(even collage said its diffcult 2 station
PLEASE DOAA FOR us

My examination today cycle one


Khartoum paces exam
December 2016
: Started with
station 3 young man with ejection systolic murmur
over left 2nd intercostal space? ? Pulmonary stenosis
I think it was correct..
: Neuro young lady with cerebellar poserior colum
pyramidal wasting small hand muscles for
differential. .time was critical. .my advice don't waste
time seconds is important
: Communication...the girl with functional
disorder...normal ct mri brain
: Station 5 diarrhea end up with ibs concern is it
cancer
: Bcc2 came for knee surgery..the rest u need to get it
from history constipation weight gain...thyroid
surgery..hypothyroidism
: Abdominal distension pallor ascites. .abdominal tb
according to discussion
: Chest..lung fibrosis. ..obvious is packed nose...skin
tightness...systemic sclerosis
: History ankle swelling sob history oh
mi...proteinuria 3 cross..end up with ibuprofen
induced nepropathy so mi ia distraction..
PACES exam experience
Exam Alain center=Emirates
05/12/2016
St 1
Resp: CFA
Abd- splenomeg with lethargy for differentials
St 2 -HISTORY- pt has hyponatremia --- hx
St 3- cardio- metallic
Neuro- rt hemiparesis... Assess motor
St 5- skin lesion? Pemphigus vulgaris?
Rt leg swelling, on anticoag- oral...regular/ what can
the swelling be
St 4
Breaking news to husband that wife who received inj
penicillin for strep inf.
Upon labour, has gone into anaphylaxis/ multi organ
failure...
May die
My experience in MRCPI DUHBAI SEPT.
2 long cases
crohns with colostomy.
steroid induced proximal myopathy.
received infliximab. Tuberculin was positive. received
anti T.B. for sometimes. very complicated case.
discussion almost every thing about crohns and S.E.
of alot of drugs as azathiopurine. infliximab. ....
anklyosing spondyl. complicated with uveitis.
discussion about complications and new drugs
short cases.
1. it was the 1st station for me europian pt. with renal
transplant and hearing aids. horrible examiner bad
presenation and discussion
2 young lady with unilateral exophthalmus. and led
lag. couldnot appreciate goitre . I told graves with
other DD discussion management of graves and DD
3 young lady with clubbing has early rheumatoid
hands with boutonniere in 3 fingers. rt interscapular
fine crackes with bronchial breathing.
I told fibrosis with cavity. some told bronchiecatasis?
4 Examine cranial nerves RT LMN facial. discussion
causes of bells treatment. causes of bil facial palsy.
5 communication. angry lady her father on warfarin .
admitted with UTI I prescribed ciprofloxacin. inr
became 8 bleeding.
she was crying all the time.
Alhamdullelah pass
I was waiting for feedback for more benifit, but
unfortunately it seems they are not sending to any
body.
I would like to thank my dear teacher Dr.Maher who
taught me a lot. his amazing way of teaching. always
encouraging me . before,
during and after the course. we cannot reward you sir
whatever we said. Allah only may rewards you.
I will never forget both Khalids, always beside us.
watching us during practice. if any mistake done by
any candidate, will not pass easily, they catch it and
correct it in a fantastaic way.
I will not forget Dr. Abdulfattah . it was a very nice
preparation with him.
Gazakom Allah Khair to evey body help me and
others by any mean of help.
May Allah bless all of them
Khartoum paces exam
History
Facial and tongue swelling
(Hereditary angioedema)
ST4
Hodgkin lymphoma llA
For Hickman line
And chemotherapy
this is ONLY
as came from the source

Khartoum paces exam


December 2016
St5
1.pt persented e headach and plurring of vision
.htn..pitiutary adenoma
2.young male e jundice and fatigue..gilbert and
differntial
: St1
heptomegally ..liver cyst ..discusion of causes
hepatomegaly
chest..lung fibrosis e lopectomy
St3
AVR
Pseudopulpar palsy,, examin cranial nerve
: St2
b blocker induce asthma
Pt had hay fever
: St 4
COPD discussion management plan

passed the exam in yangon ,myanmar


station 1) resp ---- (?effusion )
but i mentioned collapse with effusion
discussion on ddx ,inx ,mx
I ONLY GOT 12 /20
abd ..thalassaemia ..19/20
station 2----lithium toxicity
i forget to explore frequency ,character of motion
i forget his definite past history
discussion ..dx ,ddx ,interpretation of biodata
i got 17/20
station 3= cvs .AS+AR...20/20
Cns...flaccid paralysis
i forget to test about joint position sense ..16/20
station 4...Lymphpma with hickman line
14/16
bcc1 --gout --25/28
bcc 2 --cushing disease --18/28
really thanks to PEC groups for sharing experiences

passed the PACES in 3/2016 diet New Yangon


General Hospital, 2nd day, 3rd Round.
This is my 2nd attempt!
I like to share my exam experience.
I start with Station 3
Neuro.. simply Parkison d/s I got 20/20
CVS.. i didn't do well.. I heard PSM n midline
thoracotomy scar but i can't hear metallic click n
forgot to check venous graft scar at legs. I give the
differential for PSM n i only got 14/20.
station 4 is yg lady with SAH n CT normal, came
from Australia planned to go back. I hv to tell the Dx
n need of LP. Examiner ask about Autonomy. I got
16/16.
Station5>>> BCC 1 Psoariasis.. examiner asks for
differential n i was thought-block. so i got only 23/28
BCC 2 >>> DM retinopathy with other DM
complication and Hypertension. Fundoscopy is my
weakest skill n i was shaking :P Examiner asked me is
there any hypertensive retinopathy and the bell rang
when i was answering. I got only 21/28
Station1
Abdomen is moderate splenomegaly with jaundice.no
chronic liver insuff: signs. I m not sure about
hepatomegaly so i told the examiners honestly and i
want to confirm with USG. I give the Dx as
Thalessemia n other hemolytic anaemias. I got 20/20.
Respi is RUL lobectomy wih Rt thoracotomy scar n
Rt pleural effusion. I got 20/20.
station 2 is h/o collapse .. middle age lady with Poorly
controlled DM with autonomic neuropathy n
Hypertension taking OHA and diuretics and ACEi.
she is not aware of hypoglycemia. I give the Dx as
autonomic neuropathy with hypoglycemia and 2nd
differential is Postural hypotension due to drugs. I got
19/20.
Total 153 and i passed although i was not doing well
during exam. Only bcoz of hard praying n strong
belief in GOD!!

Sri lanka =26 November 2016


St5
TIA
HAEMOPTYSIS
St1
Abdom hepatosplenomegally=haemolytic
anaemia
Reapir pleural effusion
St3
SPASTIC PARAPARISIS without sensory level
Mitral regurg
St 4
DM WITH HYPOGLYCEMIC ATTACK
St2
SLE
i passed the exam in yangon ,myanmar
station 1) resp ---- (?effusion )
but i mentioned collapse with effusion
discussion on ddx ,inx ,mx
I ONLY GOT 12 /20
abd ..thalassaemia ..19/20
station 2----lithium toxicity
i forget to explore frequency ,character of
motion
i forget his definite past history
discussion ..dx ,ddx ,interpretation of biodata
i got 17/20
station 3= cvs .AS+AR...20/20
Cns...flaccid paralysis
i forget to test about joint position sense ..16/20
station 4...Lymphpma with hickman line
14/16
bcc1 --gout --25/28
bcc 2 --cushing disease --18/28
really thanks to PEC groups for sharing
experiences
Alhamdulillah, I passed my exam in Golden
Jubilee National Hospital, Glasgow on 18th
November 2016.
Station 2: Painful right knee in a patient with
ESRF on regular HD, AF on warfarin.
Cardio: MR in AF with CABG scar.
Neuro: UMN signs in bilateral lower limb- MS
Station 4: Breaking bad news- multiple
sclerosis.
BCC1: multiple joint pain- osteoarthritis
BCC2: light headedness and sob in a 52 year
old man with history of UGIB. On examination
he has MR (likely MVP) in AF.
Respi: left Lung transplant, right lung fibrosis,
underlying psoriasis.
Abdomen: Tinge of jaundice in middle age man.
otherwise no other signs. Diagnosis: likely
autoimmune chronic liver disease

My exam experience at the Golden Jubilee


Hospital, Glasgow on 17/11/16
Abdominal - renal transplant functioning well
with old capd scars. No fistula,no neck catheter
scars. Questions on management,
immunosuppression 20/20
Respi - right lower lobe lobectomy, as trachea
central and apex not displaced. Questions on
causes, and then went on to sx ix and mx of PE
20/20
History - stem: collapse. Patient had focal
seizure with generalization, hx of breast ca on
anastrozole, no other sx. Questions on ddx,
immediate ix and mx. dx: seizure sec possible
brain mets
20/20
CVs: irregularly irregular pulse, otherwise all
normal. Mistakenly said loud S1 thinking it's MS
but examiners not happy. In the end questions
all about AF only, who needs warfarin, what new
drugs for anticoagulation, mx in emergency.
11/20
Neuro : complain of double vision, eyes all
normal (no overt ophthalmoplegia)except
worsening diplopia on sustained uPgaze and
also involving UL. has midline sternotomy scar.
Dx is MG. Questions about pathophysiology,
other ddx (lems, other myopathies) ix of choice
and mx. Was also asked about congenial MG
but said I don't know. 20/20 :p
Comms: break bad news: skin mole turned out
to be stage 1 malignant melanoma. Surrogate
was concerned her fruqent tanning caused it.
15/16
Bcc1: c/o bl LL skin itchiness and rash. On
examination looks like diabetic dermopathy, ddx
is venous insufficiency. Examiners asked about
dm, ddx, mx 27/28
Bcc 2: this one weird: stem was headache and
bp 140/90. On hx headache wakes him at night,
morning, had some accident. On examination of
eyes no visual field defect. Funduscope shows
grade 2 htn retinopahty, didn't see
papilloedema. Presented findings but not sure,
ddx was intracranial lesion. Ix CT scan no time
to discuss other. Examiners not too happy 16/28
Praise God is passed


.



..

(.
. .
. .
. .
.



(. )Ahmed Maher Eliwaa








.
..
.

My exam experience ,perth Royal infirmary,


UK,9/11/2016
I didn't take any courses in UK bcs my Visa
valid only after 31 /10 and no courses available
in UK after October.i depended on Dr Ahmed
Maher course only (I attended 3 courses with
him ) .this is my 1st attempt
I would like to thank my dear prof dr Ahmed
Maher Eliwa for his great efforts with me
,dr Hady Gad,Dr Marwah Yehia,dr
,Dr Khaled Magdy,special thanks to my study
partner dr Mohamed El Aghory
Station 3
C.V.S
68 yrs old male with SOB by examination
Pulse irregular ,ejection systolic murmur at
aortic area
Disscussion ,what is your positive finding ,why
not aortic sclerosis, indication of replacement
I GOT 15
C.N.S ,,,examine his eyes
50 yrs old male wheelchair with nystagmus on lt
eye and faliure of adduction in rt eye ,INO it was
surprise ,I STOPPED
Examiner asked me to examine cranial nerves
,there was 7th cranial UMNL
Disscussion, what is ur positive findings ,what
is your DD ,Treatment of MS
I GOT 17
Station 4
25 yrs old female US showed multiple
cysts in kidney and his father on dialysis due to
ADPKD
many concerns ,will be on dialysis like my
father ?I love my boyfriend and we prepare for
marriage can I tell him ?what about my kids ?
I GOT 8
Station 5
BCC1
82 yrs old female with fullness in her neck ,I
forgot to wash my hands and patient asked me
plz Dr wash your hand
it was thyroid case since 10 yrs but recently the
swelling enlarged
No sign of thyrotoxicosis or malignancy but
there was slight change in her voice
Here concern ,what is the cause and what will
you do for me?
Examiner asked me the same Qs
I GOT 21
BCC 2
80 yrs old female with blurring of vision there
was Bitemporal hemianopia since 3 weeks ,I
asked for fundus examination he dimmed the
lights then he told me no need
Concern ,what is the cause ,does my vision will
be normal ?
Examiners Qs ,what is ur dx ,what is the causes
?
I GOT 23
Station 1
Respiratory
45 yrs old female with lateral thoracotomy scar
,PNEUMONECTOMY
examiner Qs what is cause of pneumonectomy
in this patient ,deff bet lobectomy and
pneumonectomy
I GOT 19
ABDOMEN
renal transplant (functioning)with rt iliac fossa
scar ,rt subclavian scar and hepatomegaly
Examiners Qs what is your positive finding
,what is the cause of renal transplant in this
patient ,why liver enlarged
I GOT 20
STATION 2
30 yrs old female with abnormal liver
function (elevated ALP ,GGT) and fatigue
My dx was primery biliary cirrhosis
Concern .what is the cause ,what is my
treatment, the cause of tiredness is my thyroid
illness or my liver problem?
Examiner was young British lady with blue eyes
and blond hair like the serrogate so
how can I concentrate !!!!!
I GOT 11
Total score 134

UK exam experience
November 2016
Hx : Abd pain and vomiting
Paracetamol overdose
Concerns :is it going to damage my liver ?
Has exams coming
Problem with boy friend :impulsive not suicidal
now ,regretting it.
Q: diagnosis /differential
Why did u ask about urinary symptoms?
Gyne questions very important to exclude
ectopic .
How are u going to manage ?
If she wants to leave what r u going to do ?
Psych review for capacity.
Important to ask about suicidal risk /support at
home or at university.
Time off and letter to university(she has exams)
Communication: Lady in her 50s with ESRF
High urea /creatinine more than 400 .uss
:bilateral small kidneys .
Had check up with insurance company 8 years
ago and BP was 140s/90 (high any way)
+blood + protein
Task :BBN and need for medical treatment and
dialysis in the future .
I think also to mention about link between
previous urine dip and this result (can not
remember wording exactly -but 2 tasks )
Concerns :job -accountant ,single and has one
daughter .
What is going to happen .
Angry as insurance company should have
warned her and told her to go to see GP before
is too late .
Questions:
Examiners said do you think I addressed her
concerns ?
What is best option for her ?PD/transplant as
young and active .
FH is it relevant here and why
Started with station 5:
1/47 y o lady with chest pain ,you are doctor in
emergency department .
classical angina Hx
Central ,worse with excercise ,lasts few minutes
and then stops .Husband brought her as
becoming more often .no obvious risk factors
,does not smoke ,no BP ,DM ,high cholesterol
.FH of IHD ,gi bleed
Concerns:is it heart attack ?what is it?
Normal cvs ,BP 135/80(not sure but was ok
),pulse :normal
When examining her back ,I saw the thick skin
(pseudoxanthoma elasticum ).
I asked of what is this ?(don't be afraid of
asking they will tell u or give you hints if
relevant or not ).
She said Oh I don't know ,someone mentioned it
recently but I don't know what was it .so I knew
it is relevant .
(If not relevant then she would tell u don't worry
about it or old injury or what ever.They don't
hide things from u )
I tried to exclude Elgar danlos by asking her to
try to pinch her skin and lift it up ,but it was
negative .
I explained to lady stable angina needs further
testing ,bloods,ecg,echo .
To be honest I was not sure if Pseudoxanthoma
or not .
So presented as angina possibly
Due to CT disorder as she has FH of GI bleed
and they can have GI perforation and aortic
dissection .
Examiner : what are the risk factors for IHD .
What are the names of CT disease you know
that can cause chest pain ?
Other differential for chest pain ?
Other case was :Neck lump and tiredness .
Hx of tiredness for 3 months ,lump 2/52,no
thyroid symptoms
No B sym /wt loss / night sweats
/no cough no other symptoms,
Pmh had tonsillectomy ,no medication
Concerns :back from Egypt 3 weeks ago ,is it
related ?is it thyroid
Had multiple LN :submandibular ,cervical
,Axillary
Did not do organomegally
Thyroid not palpable .not mobile when drinking .
Explained unlikely from his travel to Egypt
because sym started after his tiredness .
Needs to be seen by blood specialist to do
some bloods and sample from the glands.
Is it cancer ?possible blood cancer but needs
further tests .
We also need to exclude infections .we will do
tests and discuss with specialist .
I think I said admission
Examiners :what do U think ?
Differential
Investigation
Management
Other types of leukaemia you know
What infection can cause lymph node in 2
weeks ?
I said tiredness was for months so not Acute
,but infections like HIV/TB
(Long list but these what I remembered )
Exam today in Sri Lanka
St1 basal fibrosis ,hemolytic anemia
St3 ulna nerve palsy, AR with pul Htn
St 5 lt hemiparesis AF diastolic m in mitral area
Bcc2 psoriasis got worse after taking
hydroxychloroquine
St4 pt in rehabilitation given steroid got
psychosis
St2 epilepsy

Maadi hospital exam in 12 Oct 2016 laste


carousel
1- abdomen :HSM +Pallor+left sub mandibular
LN +left axilla scare
Mostly lympho proliferative disease
Q about diagnosis ,DD,causes of generalised LN
viral EBV,.HIV,TB,SARCOIDOSIS plus
myeloproliferative
investigation and management
2_chest:
Left upper lobectomy +trachea shifted to left
+course crackles on bases mostly
bronchiectasis
Q about causes of lobectomy
Investigation when I mention CXR asked what
you will see I replay after hesitation tram lines
,then in CT signet ring appearance
Then asked about treatment
Station 2-asthmatic worsening symptoms in
young guy I explored pet animal at home
+taking propranolol from wife medicine
.concern about causes of deterioration. I ask
him to stop taking propranolol and forget to tell
keep away from the pet animal.examiner asked
why not tell him to keep away from that pet
animal I reply I was planning to ask him but
forget .
Then Q about Causes ,investigation
steps of BA ttt
3- cvs left infra mammary scare +AF+MS so
mitral restenosis post valvotomy with pul HTN
Q investigation and ttt
-Cns examin motor system in patient with
walking aid co difficult walking
He had left hemiparesis+left UMN facial palsy
(should examin LL +UL+motor CN )
Q investigation and ttt
Station 4-young visitor lady from USA with
cruciating occipital headache with
vomiting 1st CT normal ,planned for lumbar
punture but she want to go DAMA to fly back to
USA
I start as usual ,exploring her idea about her
illness then explain to her suspension of SAH
so need LP .any idea about it reply no so
explain about procedure advantages
She ask about altrnative investigation I reply
serial imaging of head peace (right answer MRI)
but LP is the best.
She agree for LP .concern she want to go DAMA
for fly back to USA I reply if you have SAH it is
dangerous to fly .then I concluded .checked
understanding then time finished
Q ethical issues .consenting, autonomy vs
beneficence
He asked if she refuse LP you told serial
imaging within how much time I replied 48 hrs .
Then he asked after LP if negative for SAH when
she can fly .I told needs bed rest for 6 hrs then
can be DC he asked when to fly I replied I will
take 2nd opinion from my consultant. He aggree
then time finished. I got 13
Station 5-BCC1- patient co
joint pain I notice psoriatic plaque so I do it as
usual asked about systemic complication
.examined for rash distribution sites ,heart,
chest I ask for examining back they replied no
need
Then I conclude Diagnosis psoriatic arthropathy
and replied concern
Q about D,DD .investigation and ttt
Bcc2-generalized strange rash never I saw
before started 2 weeks back with alopecia
totalis in HCV patient on interferon 5 months
ago +ribaverin
The rash wasn't bolus
I don't know the case so I ask about concern it
was what's the Diagnosis. I replied my be
related to interferon but don't stop your
medication till taking opinion of hepatologist.
I offer admission.
Examiner ask why admission I replied to make
comittee and take opinion of hepatologist and
dermatologist he told can be by telephone I told
yes.
This was the worst case in my exam no
candidate know the case
Examiner told one of our collegue may be lichen
plans?
Alhamd lillah I passed so every body should do
his best and when your time came in shaa Allah
you will pass.
Deal with exam as you deal in ER .practice as
much you can .in my work I took history and
examin as we trained for mrcp in courses
Best of wishes for all to pass soon in shaa allah

kuwait==November 2016
copied from Dr.Zain group
I started by station 2
Hx: instructions,: 45 years old male with
generalized body weakness for 4 month,, he
consulted his GP a month ago and given
ESCITALOPRAM for presumed depression, he
noticed no improvement, at f.u with GP labs
taken and found to have Na of 124 other labs are
normal.
Inside I started analyzing weakness and it is
fatique only, no symptoms of depression,
otherwise he has wt loss, dry cough for 6
months. All other systemic review is
unremarkable.
Pmh: Dm on metformin last HBA1C is 7.8,
Asthma diagnosed 5 months ago without proper
investigations and no previous respiratory
symptoms.
Dh: no diuretics, uses blue and brown inhalers
F.H: unremarkable.
SH: Smoked 20 / day for 20 yrs stoped 5 years
ago when diagnosed with asthma.
Works as instructions engineer in building
blocks factory.
Concern was
1-The cause
2-can I take salts to correct Na
Exammier: DD I answered SIADH due to 1-Lung
ca 2-ESCITALOPRAM.
Asked also about employment hazard
I GOT 20
Examminer asked is it likely due to
ESCITALOPRAM or less likely, I answered less
likely
CVS: I didn't do well but actually it was mixed
aortic but got abit confused, got 10
CNs: examine upper limb, bilateral cerebellar
syndrome without sensory impairment
Qs DD main discussion on Friedeick
Got 20
Communication: clear BBN 45 F ESRD HTN
Many qstions, wanted transplant now asking
very detailed questions regarding HD and PD
inspite of my early advise for nephrology
referral
I explained everything about ESRD, and related
issues, social issues and options, the surrogate
kept asking.
Got 12
BCC1: DM HTN with blurring of vision: it was
bilateral, gradual , can't read well, can't see
trafic lights well, no other symptoms.
Pmh: poorly controlled DM &htn, has peripheral
neuropathy.
Acuity + field were normal, fundus bilateral soft
and hard retinal exudates. Concern was driving.
My plan was referal to eye doc diabetes
management podiatrist and to stop driving.
Examminer was surprisingly unhappy with my
findings and asked why to stop driving if normal
acuity, I answered he has difficulty reading and
seeing traffic lights, wasn't satisfied
Got 18 only.
BCC2: 54 years old f with palpitations and
sweating
Normal BP
no thyroid symptoms
No CVS
Palpitations are self terminating no associated
symptoms
I thought I got lost and just a minute before
finishing I considered post menopause and it
came to be the right one with hot flushes dry
vagina and mood changes
I examined thyroid, CVS asked for BP
But un fortunately no time to answer all her
many questions.
DD Postmenopausal
Pheo
Arrhythmia
Thyroid
Examiner was a tough female was angry about
why I didn't discuss the issue of menopause in
depth I answered I should do if I got time
Got 21
Chest wad left upper fibrosis in a young thin
They asked about the single most likely
diagnosis I answered post TB, then general
management
Got 20
Abdomen thalassemia
Got 18
Finally passed and I would like to say ; don't
depend on any station, my marks in station 5
seems to be of a failing candidate but
fortunately I compensated by other stations

My exam dubai 11/10/2016


Station 4: old female patient in rehabilitation for
physiotherapy because fracture femur she
develop headache, the doctor on rehab
suggested temporal arteritis, steroid started
then she develop psychosis, they reffere her to
hospital she assessed there by rheumatologist
he said temporal arteritis less likely and adviced
tappering of steroid, ur role to speak with her
angry son.
His concern why they gave her thos steroid?
Why they refere her without informing him?
What is out come and prognosis of her
condition?
They will send her back to rehab after that?
Station 5:
2 patient presented by dysphagia 34 years old
and 35 years old( case one systemic sclerosis,
case 2 multinodular goitre euthyroid with
pressure symptoms), both concern is it
acancer?
Station 1: Abdomen splenomegally without
peripheral feature of CLD), egyption, most
probably shistsomiasis.
Chest: young male left lower lobectomy
because of aspergiloma.
Station 2: history.
54 years old collapse (epilepsy feature)
Station 3:
Neurology: young female flacid paraparesis
without sensory level.
Cardio: AF+raise JVP + double valve
replacement (aortic and mitral), functioning well.
wanna share my exam experience
Kuwait , Al Amiri center November 2016:
Station 1:
Chest:
Middle aged male cyanosed clubbed with raised
jvp and mild pedal edema
He had bilateral symmetrical fine to medium
sized inspiratory rales and midline sternotomy
scar
Tattoo on left arm
Impression ILD
Viva: what are ur findings?
What could be the cause and how would u
manage him
I got 20/20
Abdomen:
Middle aged patient average built
Pale with palmar erythema and purpuric
eruption on both shins ( he was happy when I
said cryo)
No ascites
Hepatosplenomegaly
Viva was on etiology and management
I got 20/20
Station 2:
40 year old lady
Diabetic
History of parathyroid surgery for high calcium
levels
Came with bouts of palpitations and irritability
with high bp
Impression was pheo as part of MEN 2a
Concern was what do I have and how will it
affect my job as a businessman
Viva
On differential diagnosis
Men syndrome
Management of high bp in pheo
Medullary thyroid carcinoma
I got 20/20
Station 3:
Neuro:
Middle aged male
Sensory motor neuropathy till knees
Deep sensory loss also
Stamping gait
Cerebellar signs could not be properly assessed
due to weakness but roughly absent
DDx sensory motor PN mostly hereditary
Viva
On causes
What would do for him
If u did a CXR as u say what may u find? I told
malignancy or sarcoidosis
I got 20/20
Cardio:
Middle aged male with hyper dynamic apex
Muffled S1
MR murmur
I appreciated a collapsing pulse and AR but they
were not happy with it !
Viva was on causes of MR in this age group
Management and indications of surgery
I got 13/20
Station 4:
Breaking bad news:
A medical student diagnosed with MS by MRI
and CSF after she got an episode of ON
The neurologist did not have time to explain to
her and now it's ur job
It's a common scenario but the surrogate was
very bad
Concern was complex:
What do I have?
Will I end up in a wheel chair?
Will I end up with a urine catheter?
Will it affect me as a medical student and will be
able to be a doctor !
How about my fertility! Will it affect my ability to
get married and have kids
I got 11/16
Station 5:
Case 1
25 yo Male previously healthy came with LL
edema
Systemic review excludes any cardiac, hepatic,
nutritional problem or anaphylaxis
He denied any face puffiness
Only positive for frothy urine (bubbles)
No evidence of autoimmune disease, viral inf or
malignancy
Concern: what do I have
I did not take a consent for renal biopsy!
Impression: nephrotic syndrome
Viva
How to diagnose
What types do u know
What to exclude in this case ( solid malignancy)
Anticoagulants in membranous nephropathy
Many question on membranous Nephropathy:
Why didn't u take consent for renal biopsy? I
told after I confirm my diagnosis with urine
albumin creatinine ratio first
I got 28/28
Case 2:
45 yo male with a troublesome skin lesions and
uncontrolled BP
Impression
Neurofibromatosis
Concern: high bp and ll edema caused by
norvasc
Viva
What are causes of htn in neurofibromatosis (
renal arteries stenosis, pheo and CPA lesion)
What is management of renal a stenosis
Is there any benefit from surgery in renal artery
stenosis ( no, multiple drug therapy with CCB,
central agents and direct vasodilators is equally
effective)
How can u elicit the presence of renal artery
stenosis
I got 23/28
Finally I passed thank god 155/172
I wish u all the best

paces exam cochin india 21/11/2016


st 1 ILD , ascites plus bilateral pitting oedema
st 2 DM type 1 with attack of hypo and
hyperglycemia plus hypothyroidism.
st 3 AS plus pansystolic murmur at apex,PNP
uncoperated pt
st 4 cs discuss w relative poor px of father with
advance copd and resp failure
st 5 Rh A , TIA with family hx of stroke

Colombo (Sri lanka) day 1 ... (first exam in This


country) :
Neurology : Command examine lower limbs ,
Spastic paraparesis without sensory
involvement
(some other candidates got another patient with
command examine upper limbs : blilateral
wasting small muscles of hands)
Cardiology : Some candidates gave diagnosis
as MR and some as Mixed mitral valve disease
Respiratory : ILD (clubbing with bibasal fine
crepts, character doesn't change on coughing)
Abdomen : hepatosplenomegaly
Station 5 a) SOB in longstanding RA , also on
Methotrexate
Station 5 b) Probably hypokalemic periodic
paralysis
History : Acute monoarthritis
Communication : Pacemaker malfinctioning,
needs readjustment. Patient doesnt want to stay
in hospital

Dubai 10/10/2016
St 4 polycystic kidney bbn concern about job
and her kids.
History: uncontrolled asthma after yrs of
control, new factors was pet at home and
propranolol for anxiety
Neuro: upper limb examination in ESRD pt,
there was wasting of thenar group.
Cardio: aortic stenosis probably aortic sclerosis
Abd: hepatosplenomegaly and i missed lymph
nodes, there was hickman line in place probably
lymphoma
Chest: was very difficult very old man
uncooperative. Obstructive changes with
depressed lt side. Probably copd with lt fibrosis.
St 5, 1 recurrence of grave's in a young man
St5, 2 fever and sweating with artificial valve

Allhumduliah I have passed my exam held on


1st November 2016 .
My exam experience of wishaw general hospital
as below :
I started with station 2 .
Case outside was this lady was admired with
facial n tongue swelling now came for follow up
.
Young female student living in abroad had this
episode first time ever in morning with no
precipitating factor . Adopted child . No part
time job . No asthma and other allergies . No
atopic symptoms .
Explained her about medic alert bracelet ,
epipen,
Nurse specialist , further support , help , dial
999 in case of emergency let your friends know
about it .
Specialist input , exclude pregnancy ,
Psychosocial support .
Stress was upcoming exam and financial issues
.
I gave d/d C1 esterase inhibitor deficient .
Hereditary angioedema
Examiner discussion :
Asked about definite diagnosis , d/d
What are types of skin reaction type 1-4 skin
reaction which one should be found in patch
test .
Definitive diagnostic test ,
Management
Precautions
Allhumdulliah got 18/20
Station 5 :
BCC1 : this man presented with rt sided chest
pain .
Pt said its ache and sometimes feel palpitation
rather than pain , asked about Socrates ,
previous episode , smoking , job , had previous
asbestos exposure , constitutional symptoms ,
sun exposure , vit D , forgot about trauma but
mentioned him don't lift heavy things , any fever
, rash for herpes zoster , pt had ANKYLOSING
APONDYLITIS , allergic to multiple medication ,
had perforation of gut due to ulcer now taking
analgesic by patch ,
Excluded all cardiovascular risk factors ,
Examination findings : features of AS .
Pacemaker , median sternotomy scar ,
Laporotomy scar
On rt side chest no tenderness no rash .
D/d : musculoskeletal , trauma , part of AS ,
bone metastasis , vit D deficiency .
Examiner more interested about palpitation
explained that given hx of significant cvs histry
would definitely do ECG,echo as well along wth
other tests .
Got full marks 28/28 Allhumdulliah
Station 5
BCC 2:
This lady presented with difficulty swallowing ,
Tried to exclude malignancy , mechanical
causes , HIV risk factors ,
Smoking , job , Alcohol ,
Social hx ,
Impact on life .
Examined for thyroid , retrosternal goiter , Oral
cavity examination , cervical lymphnodes ,
axillary lymphnodes,
Examiner discussion :
Diagnosis , D/d .
Tests barium studies barium swallow . Thyroid
profile ,
Management , d/d
Got Full 28/28 Allhumdulliah
Cvs : prosthetic tissue valve REPLACEMNT with
no harvest scar in leg valve dysfunction
diastolic murmur on left stern border valve
REPLACEMNT secndry to b calcification of
aortic valve
Got 19/20
Abdominal
Elderly Caucasian cachetic , Icteric
No peripheral stigmata of CLD with a mass in rt
hypochondrm and epigastrium no cervical
lymphadenopathy
Pancreatic mass causing obstructive jaundice
Got 15
Respiratry
Pulmonary fibrosis secndry to sarcoidosis
Got full 20/20
Neuro
Double vision .
Don't know what was that
Pt had fatiguiblity with left partial ptosis no
Horner signs
Totally mess this
Station 4
My case was dealing with daughter whose
father copd admired with pneumosepsis curb
Score high n bed not availbl in medical icu so in
surgery ward
Got good marks 14/16
Allhumdullilah for everything
Good luck everone
May Allah Pak grant success to all
Aammeen

Today last round 15/11/2016 chennai MMM


hospital
Cvs:::: MSMR PTH (also got systolic murmur in
aorta area)
Cns :::::small muscle wasting in both upper
limbs with aflexia (sensory n cerebellar intact)
History ::::;IBS (strong family history of ca
bowel)
Resp ::::::left fibrocavitory lesion
Abdomen :::avf with liver enlargement
Comm ::::talk with pt' dg about pt refuse for
BKA . Pt admit to ward for 1/12 for infective left
DM foot
Last night got sudden critical ischemic change
Vascular consultant n medical consultant offer
for BKA ,,,pt refuse pt mental capacity
competent
Bcc1:::: 50yr old man Melanae
Pt taken naproxen n aspirin
Bcc2::: 30yr woman k/c sle .... right sided chest
pain for 2/52wk
Recent fever joint pain +

UK Experience on 18th of November2016


St:1
Resp....effusion and lobectomy
ABD....dnt know what it was....scenario was
ABD.distension
I tell them that may be asceties but not clinical
clue
Neuro....multiple sclerosis...spastic parapareses
Cardio
Only afib
Hist....melena due to cardiac medication
Comm.skill
DM,WANT BARIATRIC SURGERY
St 5
BBC1: DVT....done well
BBC2::Amiodaron induced hyperthyroidism
intermittent palpitation &wt loss
.....I cudnt pick

Mandalay center, 16.11.2016 last round


Station 1- Abd- polycystic kidney disease
Resp- pulmonary fibrosis ( bilateral basal)
Station2 - 30 yr old lady presented with 6
months history of tiredness. Hb 102 mg/do, MCV
75, menorrhagia(+) , Normal thyroid function
tests. PMH - IBS, history of wt loss(+). Concern-
causes of tiredness.
Station 3- CVS- 4 candidates answered MS with
AF, the one -dextrocardia
CNS- left ulner nerve palsy
Station 4- 75 yrs old lady with known CKD with
HT on Ramipril od presented with urinary
sepsis.Give amoxicillin and gentamicin. No
blood tests were done for 2 days. On Monday,
patient got oliguric and acute kidney injury.
Stop genta & Ramipril. IV fluid were given .
Nephrologist see and concluded that no renal
replacement therapy is required. Doctor 's task
is to explain her son( Angry patient).
Station 5
BCC1- 30 yrs man presented with Rt leg
swelling. ( Dx DVT)
BCC2- 55 yrs old lady with known HT with DM
presented with blurred vision. On fundoscopic
examination of Lt eye shows blackish pigments.
I can't answer well in this station.

first day first and second round


(14.11.2016) Myanmar, Mandalay Center
History recurrent hypoglycemia only sweating
RBS wide range hypo to 20 : irregular diet: dose
of insulin adjust herself.postural drop. fullness
of stomach.on levothyroxine . concern why she
has the attack. plan for getting married .can
have baby in the future?
CVS coarctation of aorta operated with aortic
valve replacement
CNS rt complete ptosis.3'4'6 th CN no pyramidal
symptom no cerebellar symptoms
BCC collapse causes ? GI bleed.Drug due to
propranolol 40mg BD
BCC Chest pain with unilateral leg swelling
diagnosis PE
Resp rt lung collapse with bronchiectasis
Abd haemochromatosis
communication in first and second round
parkinson disease for 7 years now two day
history of hospital admission .fall history with
carpet edge. stiffness more severe.swallowing
problem. no medication recently .because the
drug is not available. daughter want to know
why her father became deterioated. miss
medication. angry for that

UM red team Malaysia..26/10/16


Last carousel
St 1: hepatosplenomegaly with pallor- CML..got
20/20
RA with cushingoid features with interstitial
lung disease..got 20/20
St2- syncope with collapse-sounded
cardiogenic syncope.got 19/20
St3- MVR in failure..got 14/20
Bilateral claw hand with small muscle wasting
of hands probably bilateral ulnar nerve palsy
disucss along differentials..got 19/20
St 4: breaking a news of multiple sclerosis...did
badly got only 6/16
St5: BCC1- headache, young man has h.o acne
taking isotretinoin and pcm, gave differentials
BIH, migraine, tension headache..got 26/28
BCC2: diarrhea with vommiting and fever, skin
lesions looks like mix of pemphigus and
psoriasis, taking aza and prednisolone..gave
differentials of infective diarrhea, inflammatory
bowel,drug related....got 21/28
With gods grace passed 145/172...
16.11.2016 first round last day
Myanmar, Mdy Center
CNS proximal myopathy
CVS double valve replacement
Resp right upper lobe collapse with
bronchiectasis feature
Abd marked pallor with splenomegaly
BCC 1 joint pain 2days
DM.Beer drinking
On examination first metatarsal joint swelling
and tophi
Diagnosis gout
BCC2 c/o 43yr reduced vision one year
gradual not painful
initially right eye noticed during face washing
later left eye involved
hypertension history present
on examination bilateral optic atrophy and
hemianopia
History 28 yr c/o first time collapse during quee
.light headache and black out.urinary
incontinence present .not answer post ictal
confusion
family history of epilepsy present
no social problem
single . phone operator
no night duty
apply for driving licence
concern epilepsy?
what investigations needed
tell me do and do not
can get driving licence or not
Diagnosis first time unprovoked seizure
Communicatio 60 yrs male patient with COPD
six month ago lung function test FEV1 24%
history of NIV
now have AECOPD
treatment given at ICU
now out to general ward today is 8th day
but patient frequently get confusion with
increase CO2 with spO2 95%
explain poor prognosis to son ,not rule out
ventilator support.not rule out ICU care.not rule
out invasive ventilation
his son wants to be present of his father in his
grand daughter wedding
angry for not doing the above possible
treatment plan
request second opinion

15.11.2016.Mandalay last round second day


Resp consolidation
CVS MR pulmonary hypertension
Neuro spastic paraplegia
Abd splenomegaly
BCC hand tremor
on sodium valporate for epilepsy
last attack yesterday
alcohol drinking previously
postural tremor more marked then resting
tremor on examination
BCC chest pain
increased onexertion
relieved by antacid
hypertensione
smoking since 15year of age
ddx GORD IHD
History episidic headache and palpitation for 6
months in menopause lady
CCB for hypertension but side effect rash
omit it and now on losartan and thiazide but
uncontrol BP 160/100mmHg
sweating present
no panic attack. wt loss 3months
no LOA
Commumication UC flight attendent
concern married plan,need to tell boyfriend or
not
family planing
loose motion and problem with long hall flight
can continue her job or not
ask
medical report to show her boss
can write it for me
does not want to tell her boss, do not answer
her boss if ask
Myanmar, Mandalay Center
second day second round 15.11.2016
History35yr unilateral headache
took co-codamol 4tabs per day for one month
now headache became the whole headache
no stress
no family problem
concern why headache? migraine? SOL?
request for CT?
Communication
vaginal swab group B streptococci isolated
give IV benzyle penicillin to prevent child
getting infection during delivery
child condition well but mother developed
SJSyndrome
now unconsicious, renal impairment, liver
function impair,
plan for ICU care
concern can infection transmitted to baby
can recover after stopping medication
can predict outcome
what treatment will be given at ICU
can get recovery in ICU
complain procedure
CVS psm at lower sternum edge
CNS Motor neurone disease
Resp left massive pl effusion
Abd liver transplant
BCC c/o headache and hypertension
acromegaly
BCC fatigue and tiredness2months
known RA on methotrexate CQ NSAID
on examination partial ptosis with fatiguibility
test positive
dx RA and MG

Myanmar, Mandalay Center


Day2 round 2
Station 1 - left sided pleural effusion
Liver transplant
Station 2 - Headache
May b analgesic induced
Station 3 - PSM DDx
Bulbar + 12 CN palsy ( MND )
Station 4 - Explain to husband about Steven
Johnson's syndrome due to penicillin during
delivery
Station 5 - Acromegaly
- RA + Myasthenia Gravis
Good luck to all !!

St 2 Type 1 DM erratic sugar control with


frequent hypo attacks
DDx APS
Autonomic neuropathy
St 3
CVS AVR
CNS complete 3 rd N palsy
St 4
Dealing with angry pt's daughter-Parkinson
admitted with fall , missed medication since
admission
St 5
BCC. 1 collapse ( further exploring he said he
has heavy drink with previous ho of H&M)taking
propranolol
Postural BP drop +
Dx orthostatic hypotension due to DDx GI blood
loss, propranolol, autonomic failure due to
alcohol ( i said that apart from orthostatic
hypotension, hypoglycemia need to be excluded
due to underlying liver problem)
BCC 2. 37 male hemoptysis with chest pain _
Pul embolism
( on exploring ho- gave long haul flight)
St 1
COL
COPD , bronchiectasis

(14.11.2016) Myanmar, Mandalay Center


first day last round
History bronchial asthma with beta blocker
Communication Interstitial lung disease with
palliative care
BCC proximal muscle weakness with Graves
underlying Myasthenia gravis
BCC tall young man with chest pain underlying
Marfan syndrome
Resp pleural effusion
Abd J and hepatosplenomagaly
CNS flacid paraplegia
CVS pure mitral stenosis

Myanmar 14.11.16
Day 1
Round 2
1. Splenomegaly with anaemia
Bronchiectasis
2. Hypoglycaemia with Type 1 DM
Also have thyroid problem and autonomic
neuropathy
DDx APS
3. Proximal myopathy due to steroid
ASD???
4. Angry patient daughter about her father's
Parkinson disease miss to pay medication at
ward
Patient have aspiration pneumonia at currently
5. Collapse with COL
pulmonary embolism with DVT
Other people got 3rd CN palsy for Neuro
COL for abdomen
Aortic valve replacement for CVS
Good luck to all
(9.11.2016) 3rd day last round, Myanmar,Yangon
center

BCC 1. Joint pain in thalassemia

BCC 2. Diabetic scar

S2- SLE with antiphosholipid Symdrome

S4, breaking the bad news MS

1st day 1st round, Myanmar center, Ygn


(7.11.16)
History - hemiplegic migraine
Comm - wrong insulin type injection / medical
error insulin injection
BCC1- OSA
BCC2-myotonic dystrophy
Resp... UL collapse with effusion
Abd... COL with gynecomastia surgery
CVS... MS+AF
CNS...Parkinsonism
(11.11.2016) last round last day Myanmar
Center, Yangon
CVS - AS ,
Resp - ?effusion with consolidation ,
Abd - PKD ,
CNS- peripheral neuropathy motor sensory ,
History- lithium toxicity ,
comms- hickman line ,
bcc - cushing ( pitutary) , gout

(11.11.2016) last day first round Myanmar


Center, Yangon
CNS... 5th+7th CN palsy
CVS.... AS???
Resp... ILD
Abd... Thalessemia
History.. confusion for 2-3 hrs dx.. transient
global amnesia
Comm... metastasis malignant melanoma
BCC... Acromegaly, Vitiligo+ goiter

(10.11.2016) 4th day 2nd round Myanmar,


Yangon Center
CVS... AS
CNS... 12th CN palsy+cerebellum
Resp.... UL collapse+ LL pleural effusion
Abd... renal transplant
History... Fatigue+ SOB, valve replacement done
taking warfarin, hypothyroid present.
Communication... Clostridium difficle
BCC... 1. Neurofibromatosis
2. Hypopitutarism

06/11/2016...3 pm...10 examinee...


Frimley park hospital...
Fracture clinic...
1. Respiratory- left lower lobectomy and COPD...
Abdomen - spleenomegaly and ascites, no
stigmata of CLD...I said CLD...the examiners
wanted something else...I think non cirrhotic
portal htn...
2. History - IBS reassurance...
3. Cardiology- sternotomy scar, pacemaker
scar, without graft scar or metallic sound, ESM
in Aortic area- I think tissue valve with re
stenosis...got completely busted in this
station...
Neurology - retinitis pigmentosa...examine the
vision...
4. Communication - missed medication
5. BCC 1- rheumatoid arthritis with malena...
Gastric erosion due to NSAID
BCC 2- a 40 yr old lady with progressive
weakness of rt hand...MND or, motor
neuropathy (no sensory loss)

(10.11.2016) 4th day last round Myanmar,


Yangon Center
History... sudden onset facial swelling and
tongue swelling Dx idiopathic angiodema ddx
hereditary angiodema
Comm.. BBN for ADPKD
CNS... Rt 3,6,7 CN palsy
CVS... MVR
Abd.... CLD with ascities
Resp... Lt lower lobe effusion
BCC...1. TIA with AF
2. SLE with wt gain with goiter and moon face
dx- Cushing, hypothyroid

YANGON CENTRE
Day 1(7.11.16)
Station 4
Tough station of all for me.
24,female,c/o abdominal pain
USG shows polycystic kidney disease
Father also had ADPKD & on PD & peritonitis
Task: explain dx
Concern: worried that she had to take RRT like
her father
Planning to marry & have a baby
Ethic: she ask if she told her fiancee about ds &
to screen her brother
Examiner asked me if she told her fiancee about
her disease, he might not marry her? I don't
know.
She is afraid of invasive Inc& don't come to
renal OPD, how would u do
To many difficult questions.

UK Experience
1-11-2016
JAMES COOK HOSPITAL 3RD CYCLE, 1ST DAY
Chest: rheumatoid hand with lobulated rt pleural
effusion
Abd : transplanted kidney ,
Cardio: valve replacement (tissue), with mital
regurge
Neuro: ?? , very difficult, last second I know
Parkinsonism
History: acute gastroenteritis, Acute kidney
injury and lithium toxcicty
Communication: BBN Cancer esophagus with
metastisis, inoperable, depreesed pt.
BCC1:BACK PAIN AND HEADACHE , POST
PITUITARY RESECTION, TANNED SKIN ON
REPLACEMENT THERAPY ( NELSON
SYNDROME)
BCC2: ACTOR WITH COLLAPSE AT WORK ON
CITALOPRAM WITH HISTORY OF SUDDEN
DEATH OF HER FATHER ??FAMILIAL
PROLONGED QT

(8.11.2016) 2nd day last round


Myanmar (Yangon Center)
Abd....anemia with splenomegaly ,
?hepatamegaly , prominent malar face Resp -
lobectomy CVS - midline sternectomy scar with
?PSM , no click and no CABG scar, History-
frequent collapse , comm - SAH and LP, BCC1
diabetic and hypertensive retinopathy with
uncontrolled hypertension and diabetes , visual
acuity reduced for two months, BCC2 psoriasis
with beta blockersNeuro parkinsonism
(9.11.2016) 3rd day 1st round Myanmar Yangon
centre
S1 Collapse conso+ effusionHSmegaly with
ascitesS2 collapseS3 MS+AF, Flaccid
paraS4 GCA on steroid become
psychosisS5 vitiligo+thyroid, Anky Spond

Myanmar Center, YANGON


1st day Last Round (7.11.2016)
Station 2_History taking
34,male,accountant
Known case of irritable bowel syndrome
On antispasmodics
Father had colon cancer & took colectomy &
radiotherapy
Complaint: worsening crampy abd pain&
Erratic bowel function
Concern: worried that he have colon cancer
Q: PDx,DDx,Inx.
IBS
IBD
Celiac disease
Tropical sprue
Chronic bacterial overgrowth
Chronic pancreatitis
Whipple disease
HNPCC
Familial CRC
Inn: F BC,USG,colonoscopy,faecal
elastase,Anti_endomysial Ab,anti _TTG
His uncle also had colon cancer
CVS
1.AS,PS
2.MS+AF+ pulm HTN
Neuro
1.Difficulty in walking
Ex of LL
On Rt LL,Tone increased,power 3/5
Knee exaggerated,ankle reduced,EPR on both
side, sensory level +
Spinal cord compression
Transverse myelitis
Inx
2.Common Peroneal nerve palsy
BCC 1
Asthma with blurred vision
Worse in dim light & at night
Retinitis pigmentosa
Q: syndromes associated with RP
BCC 2
Long standing blood disease with wt loss
despite GOOD appetite
HO of more than 100 bags of blood
transfusion,hyperglycemia symptoms
How operation for blood ds
O/E Thalassemia, Hepatomegaly, splenectomy
scar
PDX,reason for splenectomy
Station 4
24,female,c/o abdominal pain
USG shows polycystic kidney disease
Father also had ADPKD & on PD & peritonitis
Task: explain dx
Concern: worried that she had to take RRT like
her father
Planning to marry & have a baby
Ethic: she ask if she told her fiancee about ds &
to screen her brother
Examiner asked me if she told her fiancee about
her disease, he might not marry her? I don't
know.
She is afraid of invasive Inc& don't come to
renal OPD, how would u do
To many difficult questions.
Station1
1.Resp: male pt, cough & SOB for 3 months
O/E: clubbing,dullness percussion, reduced
VBS, no ronchi,no crepts
I gave first consolidation.
Examiner ask DDx
I gave pleural effusion,pulmonaryfibrosis, Ca
lung
Pleural thickening
Invx.
2.Rt sided pleural effusion
Abdomen
1.A man with abdominal discomfort
O/E Hepatosplenomegaly,smooth
surface,dilated abdominal veins,no spider naevi,
jaundice,palmar erythema
Q: causes of HS , invx
2.Pallor+Jaundice+hepatosplenomegaly
Day 2 (8.11.2016) Myanmar center, Yangon 1st
roundStation 1 Rt upper lobe collapse &
effusionPCKD with AV fistulaStation 2
palpitationPheochromocytomaMEN2Stati
on 3Rt 7th cranial nerve palsyBilateral 6th
cranial nv palsyMS AFStation 4CURB 65
pneumoniaTalk to daughter about patient's
deathStation 51)Rt homonymous
hemianopiaAF2)systemic
sclerosis,malabsorption

UK west middlesex university hospital...


29/10/16
St1... bronchiectasis...kidney+pancreas
transplant...
st2...young female with an episode of loss of
conciousness and incontinence.. her brother
has epilepsy
St3... AVR... diabetic neuropathy sensory only
upto mid sheen
St4... old man with pul fibrosis.. talk to son
about palliative care
St5... OSA + gouty arthritis is hands...
YANGON CENTRE
Day 1(7.11.16)
Station 2_History taking
34,male,accountant
Known case of irritable bowel syndrome
On antispasmodics
Father had colon cancer & took colectomy &
radiotherapy
Complaint: worsening crampy abd pain&
Erratic bowel function
Concern: worried that he have colon cancer
Q: PDx,DDx,Inx.
IBS
IBD
Celiac disease
Tropical sprue
Chronic bacterial overgrowth
Chronic pancreatitis
Whipple disease
HNPCC
Familial CRC
Inn: F BC,USG,colonoscopy,faecal
elastase,Anti_endomysial Ab,anti _TTG
His uncle also had colon cancer
YANGON CENTRE
Day 1(7.11.16)
CVS_ I'm not sure
AS,PS
Neuro
Difficulty in walking
Ex of LL
On Rt LL,Tone increased,power 3/5
Knee exaggerated,ankle reduced,EPR on both
side, sensory level +
Spinal cord compression
Transverse myelitis
Inx

YANGON CENTRE
DAY 1 (7.11.16)
BCC 1
Asthma with blurred vision
Worse in dim light & at night
Retinitis pigmentosa
Q: syndromes associated with RP
BCC 2
Long standing blood disease with wt loss
despite GOOD appetite
HO of more than 100 bags of blood
transfusion,hyperglycemia symptoms
How operation for blood ds
O/E Thalassemia, Hepatomegaly, splenectomy
scar
PDX,reason for splenectomy

Dear colleague
I would like to share my exam experience in
Maddi armed hospital
St3: Examine motor system
By inspection patient has hemiplegic posture in
the left side and on screening there is weakness
in elevation on limbs on left side--- I said dont
forget crainal nerve or heart examination
On exam left side hemiparesis Then I examined
the cranial nerves (7th,12th) , Both carotid ( as
per dr Ahmed recommendation ) ,Pluse. Finally I
asked to examine heart he has
midlinestrenotomy scar and Prothetic valve
(MVR on auscultation)
Examinaer Qs: what is your finding?
How would you manage the patient?if in acute
stage urgent Ct scan to rule out hemorrhagic
stroke if the patient in window phase to benefit
from anti thromotic therapy Vs chronic non
pharmacological Physiotherapy and
phamrcological ttt addressing the risk factor
Examiner went to discuss the cardiology in the
case by asking about the prophlyaxsis of
Infective endocarditis.
Cardio: patient is complining of shortness of
breath
Patient was young pale with congested neck
vein, midline sternotomy scar and
hyperdynamic apex left parasternal heave with
Af and 1st and 2nd heart sound is metallic
Examiner Q what is your finding ?
How would you mange the patient ? mentioning
the target INR 2.5-4 examiner said are you sure I
said yes ( I should have to say from 2.5-3.5 and
in his case additional risk factor as Af it should
be till 4 (dr AME)
St:4 Title : Iatrogenic renal impairment
She was a female patient 60 years old admitted
in the hospital. she has been having urinary
tract infection and she was given gentamycin
antibiotic and unfortuntly dose has not been
checked at the weekend and reached the toxic
dose and has been stopped afterwards.
Nephroplogy team came to assess the patient
and in their opninion she is not in need of
replacement. The patient was on ACEI and renal
function test was done previously to the
medication was normal
It was a case of Negligance and I proceed as
Appologise, Admit the mistake, Write incident
report, solving the current problem and
explaining the future plan for the patient
Surrogent question: who is responsible for this
mistake? Give me his name? I want to fire a
complaint? Will my mother get better?
I answer this is the mistake of whole team not
one member of it and all my apology on behalf
of the team and as you like if you want to fire a
complaint it is your right and from my side I ll
guide to proper place and person.
Examiner question:
What kind of medical problem you are facing
in this case
I said it is a case of Negi lance
How would you avoid such problem from
happening again?
By doing meeting with risk managerial team and
do through investigation analyze the root cause
analysis ( as per dr Ahmed words) the root
cause analysis of such problem to about why
and how such problems happened and doing
ordination to the staff to avoid it in the future
St5:
BCC1: 18 years old patient complaining of short
stature? ( Same case I Took at dr Ahmed
course)
At first glance I saw the patient she was having
thalassemic features and genelised pigmention
I first asked about the height previously? Height
of parents? Then chronic medical condition
(patient was on iron chelating agent and has
frequent blood transfusion in the past)? The I
asked about symptoms of panhypopitutrism ,
social and mood History of surgical operation
(splenectomy).
Examination I asked to let the patient sit so as
to mesurse the height and span (examiner told
me assume it is proproniate)
Then I asked to look at the (breast and axillae)
examiner told me absent
Then I do general survy (she was having
thalassemic features and genelised pigmention)
palor, then in the abdomen there was scar in the
left hypochrondrim then I palpate the liver and
percuss splenic bed to confirm splenectomy
Concern was
will I gain height again or not ?
I said frist we have to do imaging to look for
your bone age if it already closed or not and
accordling the management will be wather to
give you growth hormone or not
Will I ll be able to see my menses ?
We ll refer you to MDT including the women
doctor and gland doctor they may give you
recplacment hormnes in the form of Estrogen
and progesterone for your period
Examiner Q what is your finding? What will you
for this patient to get secondery sexual
character ?E+P ? will she be able to get
pregnant? She can be given Gnrh
BCC2: patient is having shortness of breath
On entering patient has hand deformity
characterstic of RA She has been complaining
of shortness of breath for 6 months and is
getting worse
Patient has been diagnosed as Rhumatoid
arthritis and she was on methotrexate for 2
years in addition she took NASID, No lower limb
odema, No dyspepsia or melana and no
associated other rhematological disease
On exam. She has RA deformity I assess for
Activity and function of the hand then I look for
pallor,PM, Lower limb odema, palapte the back
of the chest of the patient and then auscultate (
fine inspiratory crepitation in the base ) then
auscultate the pulmonary area (+P2)
Concern: what is the cause? Either the RA itself
or the medication she took I ll do for her
imagaing on the chest and refer to MDT
including the chest and joint doctor.
Examiner what is your finding? And what is the
cause
St:1 Abdomen
Patient complains of bleeding per gum
Pt was young with pallor, splenomegly and cx
and axillary LN
Examiner what is your finding? What is your
differnatial diagnosis? What is the cause of
bleeding per gum?
inflerative disease ex lymphoproliferative (LN),
chronic Infection, connective tissue disease.
How would you investigate? Basic investigation
including blood film
CBC, LDH, B2 microglobin, immunophentyping,
LN bipsy or bone marrow bipsy.
Chest
Patient complains of dyspnea
Patient had clubbing, dullness on the lung base
and breath sound was vesicular with prolonged
expiration with fine inspiratory crepitation on
base of the lung Dx bilateral basel lung fibrosis (
he has compensatory emphyema in the upper
lung zone)
Examiner ask about the cause? Management?
St 2 patient was 60 years old diabetic on and
hypertensive was sitting in the restaurant with
her friend and then got confused without losing
the consciousness for one hour she doesnt
remember anything about what happened.
Through history all is NO.she concerned about
driving and if it will recour again? Examiner ask
about the cause? And management?
It was confusion for DD? TIA, subdural
hematoma, stroke
I would like to express my gratitude to Prof.
Dr Ahmed Maher Eliwa
i can't find a word to describe a single thing you
have done to me for longtime. You gave me
confidence in my self so as to beat all my
weakness and not only to take but also to give
the best to others. DR AHMED MAHER ELIWA
YOU ARE THE KEY MAKER OF SUCESS WITH
HELP OF ALLAH .

YANGON CENTRE
Day 1 (7.11.16)
Station1
Resp: male pt, cough & SOB for 3 months
O/E: clubbing,dullness percussion, reduced
VBS, no ronchi,no crepts
I gave first consolidation.
Examiner ask DDx
I gave pleural effusion,pulmonaryfibrosis, Ca
lung
Pleural thickening
Invx

YANGON CENTRE
Day 1(7.11.16)
Abdomen
A man with abdominal discomfort
O/E Hepatosplenomegaly,smooth
surface,dilated abdominal veins,no spider naevi,
jaundice,palmar erythema
Q: causes of HS , invx

(Courtesy of Dr Kefah Bashir who passed in this


diet)
My exam Dubai 11.10.2016
Start with communication :
Old lady was admitted on rehabilitation for
physiotherapy because she have fracture femur
,she develop headache there the doctor on
rehab centre suspected temporal arteritis
,received steroid then she develop psychosis
,referral to the hospital was done re assessed
by rheumatologist ,said temporal arteritis
unlikely , and advice tapering of steroid , ur rule
to speak of her sun , explain to him and answer
his question ?
His concern why they gave her this steroid ? He
is upset from doctor ?
What is the out come of her condition ,?
Why they refers her to hospital without inform
him ?
Station 5: 2 cases dysphagia ( 35 years old and
34 years old)
Case one systemic sclerosis case 2:
multinodular goitre ( euthyroid)
Chest: Lt owed lobe lobectomy because of
aspergilloma.( young male)
Abdomen: splenomegally ( no signs of CLD)
CNS: flacid paraparesis without sensory level(
young female)
CVS: AF+ raised JVP + double valve
replacement (AVR+MVR)functioning well.
History : 54 years old female presented by
collapse

Kuwait adan hospital


November 2016
Resp
ILD with rheumatoid hand
Abdomen
Polycystic kidney
Neuro
Peripheral sensory motor neuropathy
Charcot marie tooth
Cadio
Mixed aortic
History
Lithium toxicity
Communication
Breaking bad new pancreatic ca
Station 5
Diabetic maculopathy ? Not sure
IBD
Hope u all the best

Western General Hospital Edinburgh..


November 2016
Station 1
Abdomen CLD with hepatomegaly
Resp Left Sided Consolidation
Station 40y old female KC ulcerative colitis with
ileostomy bad having lightheadedness with
weakness.. creatinine 265 urea30 no proteinuria
no hematuria
i messed it up.. it was high output stoma.. i gave
dd of flare up of disease. saw examiner puttinf
unsatisfactry
Station 3
CVS ESM not radiating anywhere
i gave DD examiner seems satiffied
CNS sensrimotor neuropathy for DD
Station 4
man admitted with hemetmesis confused
alcoholic explain to wife and expalin need for
urgent endoscopy
Concerns He is not drinking much
any other alternatvie
Station
Pulmonary embolism in 20 week pregnant lady
with previous misscariage she came in e.r
collapsed on inquiry she said chest pain and leg
swelling
2nd scenario weight loss palpitations on exam
there is thyroid swelling.. i gave dd pf thyroid
cancer but shud say toxic multonodular

Royal Hospital
MUSCAT, OMAN
Day 2 cycle 2
Station 1
Abdomen Renal transplant with failure pt on
Hemodialysis....functional AV fistula left arm.
Questions about signs and complications of
ESRD and cause of transplant failure.
Score 18/20
Chest young male with rt lateral thoracotomy
scar and Rt. Lower lobe lobectomy.
He was clubbed.
A case of bronchiectasis.
Questions on causes of bronchiectasis
Management
Organisms
19/20
Station 3
Cardio young male with AS+/-AR .....dominant
AS.
Questions on causes of AS
Clinical severity markers
Dx
20/20
CNS a case of Charcot Marie tooth.
Questions on investigations and management
20/20
Station 5
1. Middle aged male with headache and visual
disturbance ...a clear case of Acromegaly
27/28
2. Young male known HTN with recent wt gain,
headaches and day time somnolence....OSAS
Viva on invx and differentials.
26/28
Station 2
A case of MS medical student admitted with
attack (vertigo and diplopia) recovering MRI and
LP confirmed dx. Not satisfied with neurologist.
Need second opinion. Your role to explain
diagnosis and future outcome.. address
concerns.
Examiners very rude.
16/16
4. History
Young male with Abd pain and erratic bowel
habits.
Diagnosed with IBS.
Sx related with stress.
No ALARM
Father dx with colonic cancer and recently
operated.
Pt got worsening sx for. 6 weeks.
Concerned could be bowel cancer.
Viva on differential.
Since he has very strong FH of bowel cancers in
his Father, Grand Father and paternal uncle
@39 yrs.
One aunt with uterine Ca.
So I also included screening and genetic
testing.
19/20

Sharjah 2016/3
Station 3
-Cardiology: CABG with bilateral Harvesting
scars and AV fistula functioning. Have AS,MR.
-Neurology : Spastic paraparesis with out
sensory level.
Station 4
The common scenario, speak to daughter of
Mr:X who is known case of COPD had been
admitted overnight with pneumonia to surgical
ward because no bed in medical ward, Pt
missed antibiotics dose because there was no
cannula, Pt was deteriorated shifted to ICU and
arrested there, CPR was not successful.
Station 5
-Neurofibromatosis.
-Multiple Myeloma.
Station 1
-Abdomen: Thalassemia with hepatomegaly and
splenectomy.
-Respiratory: Pleural effusion.
Station 2
acute renal failure to find out the cause
I hope this might help, if any one needs more
detailed feedback kindly contact me.
All the best.

Alhamdullilah I have passed


Score 166/172
PACES DIET 3, Royal Hospital
MUSCAT, OMAN
Day 2
Station 1
Abdomen; Renal transplant with failure pt on
Hemodialysis....functional AV fistula left arm.
Questions about signs and complications of
ESRD and cause of transplant failure.
Score 18/20
Chest; young male with rt lateral thoracotomy
scar and Rt. Lower lobe lobectomy.
He was clubbed.
A case of bronchiectasis.
Questions on causes of bronchiectasis
Management
Organisms
20/20
Station 3
Cardio; young male with AS+/-AR .....dominant
AS.
Questions on causes of AS
Clinical severity markers
ECHO findings
20/20
Neurology; a case of Charcot Marie tooth.
Questions on investigations and management
20/20
Station 5: BCC
1. Middle aged male with headache and visual
disturbance ...a clear case of Acromegaly
27/28
2. Young male known HTN with recent wt gain,
headaches and day time somnolence....OSAS
Viva on invx and differentials.
26/28
Station 2: History taking
Young male with Abd pain and erratic bowel
habits.
Diagnosed with IBS.
Sx related with stress.
No ALARM
Father dx with colonic cancer and recently
operated.
Pt got worsening sx for. 6 weeks.
Concerned could be bowel cancer.
Viva on differential.
Since he has very strong FH of bowel cancers in
his Father, Grand Father and paternal uncle
@39 yrs.
One aunt with uterine Ca.
So I also included screening and genetic
testing.
19/20
Station 4: Communication
Young female admitted with vertigo and
diplopia. Diagnosed as Multiple sclerosis.
Diagnosis was explained by Neurologist but pt
wasn't satisfied and wants to seek 2nd opinion
from candidate.
Explain her diagnosis.
Future implications and address her concerns
She is a school teacher and studying medicine.
Sole carer of her mother.
So raised questions about her careers future
and regarding care of her Mother.
Questions were asked related with ethical
issues like autonomy.
Examiners was rude.
Score 16/16
MANY MANY THANKS TO DR. ABDUL FATTAH
WHO TAUGHT ME AND IT WAS REALLY VERY
HELPFUL.

Hospital Serdang, Malaysia.


22nd Oct 2016.
BCC
1. LL swelling, discoloration, with diarrhoea 3
months.
2. SOB, acute onset in a dialysis patient.
Station 1
Resp
Bronchiectasis
Abdo
Failed renal transplant
Station 2
Hx of headache
Station 3
CVS AVR
Neuro: proximal myopathy
Station 4
ADPKD counselling
Egypt =Cairo
Maadi hospital
I have started with station 4: 38 yrs old male
with type 1 DM,
Had macro &micro complications, had Un
awareness of hypoglycaemia, & he lost his
driving lisence bz of this....
In side young age male.i asked about DM :
onset, duration, FU, cotrol ... Rx: insulin lantus
and apidra . He increased insulin dose e out Dr
opinion. ... decreased his meals after insulin...I
asked y? He told bz he wants to avoid further
complications. .. I showed empathy and
appreciate his effort but I told him unfortunately
this is not the right way as it resulted in frequent
hypo attacks. ..
I asked about all autonomic dysfunction
symptoms and r negative. .He has no FU e GP,
diabetes Dr, neuro, ophtha or chiropodist. ...I
advised for all and suggest referral...
He is covering his finances well and good family
support... I was afraid to ask further Qs not to
simulate history station...
Exam.Qs:
He was totally upset and asked:
Did u ask about other CV risk factors " HTN,
dyslexia, smoking?"
Drug list? He is on atenolol ... what's u r
opinion?
Do u think u Rx his concerns well?"how r u
going to help me? This is the concern"
I apologise, show that it a big mistake that I
did.... I act e facial expressions. ...
I realized that I lost this station....
They gave me 11....
Went out I reassure my self: always 1st station
is the worst...I will do my best for other
stations.... I remembered advise of Dr zain...
The 5 mins before S5 i was only reassuring my
self... only read the scinarios e no mental
preparation. ...
BCC1:
From out side middle age male e diarrhoea and
abd colic....
diarrhoea was bloody, in &off no wt loss...
I jumped to eam. Pt e psoriasis. ...rash
present....abd: nothing specific. ..drug list: I
forget but include MTX and NSAID, and one
other ttt. ..
Ph : only psoriasis. ..
Asked about precipitants in brief. ...All this & I
am not sure what is the relation. ..
And that was the concern and exam. Q also:
My brain is empty that time, then I decided to
tell the truth' that I don't know the relation" to
psoriasis right now, but I need to do some tests
that my include camera test, and involve tummy
and skin Dr & at that time we will tell u whether
related or no...
Exam.Qs:
Is it related? Then the same answer
DD: IBM
infections
Ca need to be ruled out
If IBD: which one? confidently I told crohns " I
don't know why I told this? I am sure no one
else can do such mistake...at this point I feel I
will get a big zero in this station and this mean I
lost the exam... so I decided to complete my
exam v cool bz I lost this already and it will
never differ again. . ..
Strangely they gave me 24
BCC2:pt e sore throat from out side . .and I have
differential in my mind other than flu, urti..
Inside : young male, analyse sore throat. ..no
feature of flu... then I asked did he took any ttt "
I mean for current condition".then she was v
generous and told he is taking Carbis azole, ok
god gave me this... ...so he is hyper thyroid?
Yes.. then I jumped to exam....throat and full
thyroid exam.... only positive? Diffuse goitre
and fine tremor?
Concern; is it related to his ttt?
I answered may be related, that's why I need to
admit do some tests then will tell whether
related. .. involve gland Dr bz he may need to
stop u r ttt for awhile and then resume it?
Exam.Qs; u r D. ...
Invest
Ttt
Admission or no?
I got 28
S1: I think modest station but was the worst....
Chest: elder man , lying at 90 digree almost, I
asked can I put him at 45 ? Answer he is
comfortable now... so I proceed... looks ill... I
search around for O2 or nebuliser machine.
..nothing...trachea central. .. crichosternal notch
distance reduced. ...
Has v quite and distant breathing... ..
Back v. Scattered rhonchi and basal creps. ..so I
present the Pt as lung fibrosis e obstructive
elements. .. discussion went thru this...at last 30
sec. Egyptian exam asked what about his ant.
Post. Diameter. ... it was increased. ..I told D
is copd complicated by fibrosis. .. He was
showing me how much I was wrong....
Onnnnnnly 7/20
Abd:HSM in
apt e valve replacement. ... I am not sure about
ascites so I told negative. ... exam.Qs:.
DD..invest....ttt. ....
Possibly I missed ascites so they gave me 11
St2:
37 yr lady ... has exhaustion...BP 145/95.... hb:
11 normo. ...
Inside; joints pain, malar rash, h/of abortion.
..h/of DVT. ..
not on pills. ..no features of other MCTD or
overlap syndrome
So SLE e antiphosphlipid syndrome. ...
Concern : can she have a child? I answered
after D and Pl good control she can get kids ...
but it should be planned. ..and need to involve
women & joint Dr. ..
19
St3; CNS: hemiplegia e facial palsy lower in Pt e
prosthetic valve....
Qs :findings
Diagosis
Cause
Invest
Ttt
20
CVS: lady with A., AS, P.HTN e TR
Qs:
D
Invest
Cause
20
AR*
In s4 dyslipidemia* sorry for mistake
Carbimazole*

kuwait amiri Hospital


November 2016
:abd cld hepatomegaly ,chest copd
,bronchiectasis ,cardio mR ,neuro mixed upper
moto and peripheral neuropathy LL,
St5 gravis d,ankylosing spondyliits
Communication medical error pt received wrong
dose of insulin
History 1ry biliry cirrosis
Kuwait exam ,,October 2016
mubarak hospital
Station 1:
HSM fot diff.
Localized bronchiactasis asking
cause,management.
Station 2:
Malabsorption synd. For 2years changed in ccc.
In last 6months with steatorrehea+abd pain
came from carribian 6 months ago she can't
remember the relation of the timing.
So malabsorption for DD.
Station 3:
flassid paraparesis with intact sensation for DD
MVR.
Station 4:
Pt with suspected SAHge want to LAMA. after ct
is normal but advice to do LP.
Station 5:
-MCTD with joint pain in boh hands.
-Goiter and hyperthyroid came with difcult
swallowing
Kuwait Amiri hospital
october 2016
Station 1 pulmonary fibrosis
Abdomen hepatimegaly
Station 2 pheochromocytoma MEN2
Station 3 MR
Motor sensory p.n
Station 4 explain diagnosis of MS and impact on
life
Station 5
NF Neurofibromatosis :skin problem and high
bp
Management
I didn't do well)
Nephrotic syndrome
Nephrotic:young man with lower limb swelling
Causes

Station 1:
HSM fot diff.
Localized bronchiactasis asking
cause,management.
Station 2:
Malabsorption synd. For 2years changed in ccc.
In last 6months with steatorrehea+abd pain
came from carribian 6 months ago she can't
remember the relation of the timing.
So malabsorption for DD.
Station 3:
flassid paraparesis with intact sensation for DD
MVR.
Station 4:
Pt with suspected SAHge want to LAMA. after ct
is normal but advice to do LP.
Station 5:
-MCTD with joint pain in boh hands.
-Goiter and hyperthyroid came with difcult
swallowing.

Kuwait exam today


Station 1:
HSM fot diff.
Localized bronchiactasis asking
cause,management.
Station 2:
Malabsorption synd. For 2years changed in ccc.
In last 6months with steatorrehea+abd pain
came from carribian 6 months ago she can't
remember the relation of the timing.
So malabsorption for DD.
Station 3:
flassid paraparesis with intact sensation for DD
MVR.
Station 4:
Pt with suspected SAHge want to LAMA. after ct
is normal but advice to do LP.
Station 5:
-MCTD with joint pain in boh hands.
-Goiter and hyperthyroid came with difcult
swallowing.

UK : Experience of a candidate who passed in


this diet)
My exam started with station5 .
BCC 1 was back pain...i went in and started
history with differential of ankylosing spondilitis
in mind but patient told me has has rashes as
well.on examination back movements were fine
and he had nail pitting.i gave differentials of
psoriasis ..as pain was in small joints so
examiner asked me is it something else.i told
although it is typical presentation ofpsoriasis
but i will like to rule out RA as well.then he
asked management. ...got 28/28
BCC 2 was hoarseness....i saw a scar on neck of
patient while taking history.she told me she had
thyroidectomy about 10 years ago....and now
she is having hoarseness for last 3 months.on
further questioning she told she has stopped
taking thyroxin and is gaining wt as well.i asked
any other medical problem.she told me that she
is having asthma n takes brown inhaler but
does not rinse mouth afterwards. I advised her
about inhaler technique and rinsing mouth n
starting on thyroxin.examined neck n offered to
examine tummy to rule out any stria as she told
me she was gaining weight....examiners asked
about other differentials .i told it might be ca
larnyx as well.got 28/ 28
Station1 ABDOMEN was Renal transplant.
....was staright forward....was asked about
management of ckd n investigations.
Respiratory was also a lady with clubbing n fine
crackles with small scars on rt chest...she had
fine crackles so i gave diagnosis of pulmonary
fibrosis.examiners asked about scar ....i said it
might be lung biopsy scar.
Got 19/ 20 in respiratory and 16/ 20 in abdomen.
Station2 was Shortness of breath in 75 years old
smoker....i took history n ruled out all
differentials.told possibilities of pE, LRTI ,
CAlung.got 16/20
Station 3
Cardiology metalic aortic valve was really
straight forward...got 20 / 20
Neuro a lady with proximal muscular weakness
n intact sensations...i gave dd of muscular
dystrophy n MND ...viva was about investigation
n causes .got 18/20
Station 4 to council hypoglycemic
unawareness...i forgot to ask about smoking but
satisfied patient so well that he told me that
thank you for very good explanation as you
have explained everything....Got 14/16
I think it is all blessing of Allah .
I would advise all my fellows to do as much
paractice as you can.see as many patients as
you can with exam cases in mind and finally do
a revision course one week before exam to get
into mode of exam.
This is bit different exam but if you practice it is
very easy otherwise very difficult but one
should never be disappointed....GLASGOW
college is better for overseas candidates as it
looks to me examiners are very very fair

Alhamdullilah I have passed


Score 166/172
PACES DIET 3, Royal Hospital
MUSCAT, OMAN
Day 2 cycle 2
Station 1
Abdomen Renal transplant with failure pt on
Hemodialysis....functional AV fistula left arm.
Questions about signs and complications of
ESRD and cause of transplant failure.
Score 18/20
Chest young male with rt lateral thoracotomy
scar and Rt. Lower lobe lobectomy.
He was clubbed.
A case of bronchiectasis.
Questions on causes of bronchiectasis
Management
Organisms
19/20
Station 3
Cardio young male with AS+/-AR .....dominant
AS.
Questions on causes of AS
Clinical severity markers
Dx
20/20
CNS a case of Charcot Marie tooth.
Questions on investigations and management
20/20
Station 5
1. Middle aged male with headache and visual
disturbance ...a clear case of Acromegaly
27/28
2. Young male known HTN with recent wt gain,
headaches and day time somnolence....OSAS
Viva on invx and differentials.
26/28
Station 2
A case of MS medical student admitted with
attack (vertigo and diplopia) recovering MRI and
LP confirmed dx. Not satisfied with neurologist.
Need second opinion. Your role to explain
diagnosis and future outcome.. address
concerns.
Examiners very rude.
16/16
4. History
Young male with Abd pain and erratic bowel
habits.
Diagnosed with IBS.
Sx related with stress.
No ALARM
Father dx with colonic cancer and recently
operated.
Pt got worsening sx for. 6 weeks.
Concerned could be bowel cancer.
Viva on differential.
Since he has very strong FH of bowel cancers in
his Father, Grand Father and paternal uncle
@39 yrs.
One aunt with uterine Ca.
So I also included screening and genetic
testing.
19/20

One of my colleague examined in UK


Westmiddle hospital,
I started with st 5
BCC1: middle aged woman with transient LOC
,?! TIA, normal ex, conc. will it recur?
BCC2: 60 yrs lady with tiredness, h of prev
pituitary s ,, ?!!adrenal insuff
Resp: copd
Abd: fistula and scars not tranplant
Hist; ref syncope/epilepsy,, conc is it
epilepsy?!!
Cvs: MR
Neuro: median nrve inj, scars of carp t
St4: palliative care for an ILD pt,, con what
support/ end of life issues

9/10/2016
*chest :clubbing with bilateral basal creps,
discussion about possibly ILD&broncheictasis
*abdomin ,young female with scar RUQ&LIF ,
?renal &liver transplant secondary to polycystic
kidney &polycystic liver disease
*Neurology, young male with proximal
myopathy with normal sensation and
coordination? Becker dystrophy? Other causes
of myopathy
*cardiology, midline sternotomy scar with
miteral valvotomy scar and metalic S1
*history, migraine headache
*communication, physiotherapy staff nurse with
functional weakness confirmed by normal brain
MRI , I started by reassuring her that normal
imaging mean nothing serious in your brain,
she said you mean I am faking symptoms of
weakness, I replied, no you are not faking
symptoms and there is a real problem and we
are here only solve your problem
Then she asked what is my problem? I said
because of your stressful job of physiotherapy
and stroke units and always seeing crippled and
disabled patients, this makes your brain to
misinterpretate the stressful triggers in to a
weakness
Then I asked about her social life, which is also
stressful due to after her duty she used to help
her younger sisters at home, there is no time to
enjoy her hobbies, she has no friends and
single
Then reassured her again this is functional
weakness and it's curable condition
Regarding treatment is mostly live style change,
change or modify her job, referral to psychiatrist
for behavioral therapy ,talk to social worker for
home support, you are still young try enjoy your
life, have friends, enjoy your hobbies, finally
summarized check understanding, give
supports.
Hope all of us to pass
*BCC1,headache with visual problem?
Acromegaly
*BCC2, young male with heart valve problem
and back pain? Ankylosing
** This is my exam yesterday in Mascut
Kuwait October 2016
History: diarrhea
Comunication: convince the son to do life
saving procesure
Resp: pleural effusion
Tb is common cause in kuwait esp indian
Abd: was normal exam with scar in RIF
Dont panic just give DDx
Cvs: mixed aortic valve dis
Neoro: examine cranial case of MG with
thymomectomy scar
Bcc: acromegaly
Bcc2: behcet

Oman - Muskkat
10/10/2016
*chest :clubbing with bilateral basal creps,
discussion about possibly ILD&broncheictasis
*abdomin ,young female with scar RUQ&LIF ,
?renal &liver transplant secondary to polycystic
kidney &polycystic liver disease
*Neurology, young male with proximal
myopathy with normal sensation and
coordination? Becker dystrophy? Other causes
of myopathy
*cardiology, midline sternotomy scar with
miteral valvotomy scar and metalic S1
*history, migraine headache
*communication, physiotherapy staff nurse with
functional weakness confirmed by normal brain
MRI , I started by reassuring her that normal
imaging mean nothing serious in your brain,
she said you mean I am faking symptoms of
weakness, I replied, no you are not faking
symptoms and there is a real problem and we
are here only solve your problem
Then she asked what is my problem? I said
because of your stressful job of physiotherapy
and stroke units and always seeing crippled and
disabled patients, this makes your brain to
misinterpretate the stressful triggers in to a
weakness
Then I asked about her social life, which is also
stressful due to after her duty she used to help
her younger sisters at home, there is no time to
enjoy her hobbies, she has no friends and
single
Then reassured her again this is functional
weakness and it's curable condition
Regarding treatment is mostly live style change,
change or modify her job, referral to psychiatrist
for behavioral therapy ,talk to social worker for
home support, you are still young try enjoy your
life, have friends, enjoy your hobbies, finally
summarized check understanding, give
supports.
Hope all of us to pass
*BCC1,headache with visual problem?
Acromegaly
*BCC2, young male with heart valve problem
and back pain? Ankylosing
(Experience of Dr Iqbal, Copied from another
group)
Alhamdullilah I have passed
Score 166/172
PACES DIET 3, Royal Hospital
MUSCAT, OMAN
Day 2 cycle 2
Station 1
Abdomen Renal transplant with failure pt on
Hemodialysis....functional AV fistula left arm.
Questions about signs and complications of
ESRD and cause of transplant failure.
Score 18/20
Chest young male with rt lateral thoracotomy
scar and Rt. Lower lobe lobectomy.
He was clubbed.
A case of bronchiectasis.
Questions on causes of bronchiectasis
Management
Organisms
19/20
Station 3
Cardio young male with AS+/-AR .....dominant
AS.
Questions on causes of AS
Clinical severity markers
Dx
20/20
CNS a case of Charcot Marie tooth.
Questions on investigations and management
20/20
Station 5
1. Middle aged male with headache and visual
disturbance ...a clear case of Acromegaly
27/28
2. Young male known HTN with recent wt gain,
headaches and day time somnolence....OSAS
Viva on invx and differentials.
26/28
Station 2
A case of MS medical student admitted with
attack (vertigo and diplopia) recovering MRI and
LP confirmed dx. Not satisfied with neurologist.
Need second opinion. Your role to explain
diagnosis and future outcome.. address
concerns.
Examiners very rude.
16/16
4. History
Young male with Abd pain and erratic bowel
habits.
Diagnosed with IBS.
Sx related with stress.
No ALARM
Father dx with colonic cancer and recently
operated.
Pt got worsening sx for. 6 weeks.
Concerned could be bowel cancer.
Viva on differential.
Since he has very strong FH of bowel cancers in
his Father, Grand Father and paternal uncle
@39 yrs.
One aunt with uterine Ca.
So I also included screening and genetic
testing.
19/20
(Copied, From UK : Experience of a candidate
who passed in this diet)
My exam started with station5 .
BCC 1 was back pain...i went in and started
history with differential of ankylosing spondilitis
in mind but patient told me has has rashes as
well.on examination back movements were fine
and he had nail pitting.i gave differentials of
psoriasis ..as pain was in small joints so
examiner asked me is it something else.i told
although it is typical presentation of psoriasis
but i will like to rule out RA as well.then he
asked management. ...got 28/28
BCC 2 was hoarseness....i saw a scar on neck of
patient while taking history.she told me she had
thyroidectomy about 10 years ago....and now
she is having hoarseness for last 3 months.on
further questioning she told she has stopped
taking thyroxin and is gaining wt as well.i asked
any other medical problem.she told me that she
is having asthma n takes brown inhaler but
does not rinse mouth afterwards. I advised her
about inhaler technique and rinsing mouth n
starting on thyroxin.examined neck n offered to
examine tummy to rule out any stria as she told
me she was gaining weight....examiners asked
about other differentials .i told it might be ca
larnyx as well.got 28/ 28
Station1 ABDOMEN was Renal transplant.
....was staright forward....was asked about
management of ckd n investigations.
Respiratory was also a lady with clubbing n fine
crackles with small scars on rt chest...she had
fine crackles so i gave diagnosis of pulmonary
fibrosis.examiners asked about scar ....i said it
might be lung biopsy scar.
Got 19/ 20 in respiratory and 16/ 20 in abdomen.
Station2 was Shortness of breath in 75 years old
smoker....i took history n ruled out all
differentials.told possibilities of pE, LRTI ,
CAlung.got 16/20
Station 3
Cardiology metalic aortic valve was really
straight forward...got 20 / 20
Neuro a lady with proximal muscular weakness
n intact sensations...i gave dd of muscular
dystrophy n MND ...viva was about investigation
n causes .got 18/20
Station 4 to council hypoglycemic
unawareness...i forgot to ask about smoking but
satisfied patient so well that he told me that
thank you for very good explanation as you
have explained everything....Got 14/16
I think it is all blessing of Allah .
I would advise all my fellows to do as much
paractice as you can.see as many patients as
you can with exam cases in mind and finally do
a revision course one week before exam to get
into mode of exam.
This is bit different exam but if you practice it is
very easy otherwise very difficult but one
should never be disappointed....GLASGOW
college is better for overseas candidates as it
looks to me examiners are very very fair

One of my colleague examined in UK


Westmiddle hospital,
I started with st 5
BCC1: middle aged woman with transient LOC
,?! TIA, normal ex, conc. will it recur?
BCC2: 60 yrs lady with tiredness, h of prev
pituitary s ,, ?!!adrenal insuff
Resp: copd
Abd: fistula and scars not tranplant
Hist; ref syncope/epilepsy,, conc is it
epilepsy?!!
Cvs: MR
Neuro: median nrve inj, scars of carp t
St4: palliative care for an ILD pt,, con what
support/ end of life issues

Kuwait October 2016


History: diarrhea
Comunication: convince the son to do life
saving procesure
Resp: pleural effusion
Tb is common cause in kuwait esp indian
Abd: was normal exam with scar in RIF
Dont panic just give DDx
Cvs: mixed aortic valve dis
Neoro: examine cranial case of MG with
thymomectomy scar
Bcc: acromegaly
Bcc2: behcet

(Courtesy to Dr Ashwag)
I will share my experience
wish one found it helpfull
i did my exam in Royal Hospital Oman 7/10/2016
start with station 1
when bell ringing i fell stress i couldn't see
where hand Sanitizer so i just look around
searching and examiner ask me to start i run
wash with water examiner said it is ok just start
give me tissues i feel stupid but no time for
feeling i just say hi to pt and ask him to expose
his abd and chest it is case of renal transplant
came with abd pain for investigation i think i did
it v. fast and did well i answer all Q what is your
finding , diagnosis ,how you know his
transplant kidney is functioning ,investigation
and looking for what for any test he ask me you
miss to auscultate kidy did you think it is
important in this pt i said yes he said why ? i
said renal artery stenosis he said ok how you
investigate for this i said i will start simply by
US then MRI if needed
second case bronchiectasis with lobectomy
when i start pt sleeping deeply examiner wake
him he take second to concentrate then i ask
him to examine did the usuall , i forget to tell
this time examiner show me where hand
sanitizer it is fix in door from outside i
clean my hand and start i examine v. fast after i
finish examiner tell me i have 1 min left i
ascultate again he is english examiner v. nice
also discussion go smooth what is your finding
Diagnosis , investigation i forget sputum test he
ask you miss sputum i said yes am sorry i need
to do sputum FB , treatment
i go to next station 2 case od young male
23years with IBS treated symptomaticly with
strong family hx of colon cancer
[10/29, 12:11 AM] .: when i start i want to shake
hand surrogate said i didn't shake hand
female so i said with smile hello am Doc
.... i just started i take detail hx of diarrhea no
alarm sign i finish all part of hx answer concern
he afraid as his father diagnose befor 1 month
of colon cancer with strong hx in family i
reasure him as far as no alarm sign no need to
do invasive test and i suggest to referral to
psychological i said that the cause of your
diarrhea due to stress and you need to reduce
stress on your life as much as you can will
improve your symptom i think to if you do
convulsation with my colleges in psychological
department will help you he agree then he ask
what about family hx i said i will come of coarse
to this point then i reassure him more and
explain we need to to some blood test to
exclude any cause or complication am thinking
that time not to forget about celiac disease and
malabsorbtion , then i said regarding family hx
sure we need to to some screening test and
genetic test we can make another appointment
to talk in detail
[10/29, 12:34 AM] .: he agree then i summarise
and check understanding i agree plan .
examiner English and other Omani one said so
you think he had no cancer i said no alarm sign
and his diarrhea chronic with stress he said why
you not put possibility as this attack of diarrhea
more sever and prolong as he claim i said now
pt on stress that is why symptom more sever no
need to increase stress as we have nothing said
it ca. regarding family hx need refer for
screening test said which test i said genetic
said then i said colonoscopy then discussion
about plan of treatment possible DD
third station is hardest one for me first case is
young female v. pall , tachypneac with metallic
valve sound going with aortic valve replacement
with obvious sternetomy scar , with sign of
pulmonary hypertention , active neck pulsation
,basal crepetation and LL edema to be honest
when am still examining pt i thought she is
young female and on AF so mostly the lesion is
on mitral no aorta but i heard it going with first
sound and the second sound is free with clear
high volume but i decide not to think and just
said finding as i get also i notice pt have big
neck scar so may be have hx of thyroid problem
which explain AF???!! any way i present my
case as this keen leady on 45degree tachypneac
... etc so my diagnosis have aortic valve
replacement with pulmonary hypertension and
AF , on failure no prepheral sign suggest
endocarditis examiner English ask me so many
Q causes of replacement
: causes and indication for replacement in
stenosis or Regar. type of valve advantage and
disadvantage, how you investigate this pt what
you will see in ECG , echo , last Q about
coagulation is any place for other antiagulant
apart from warfarin ?? this only Q i stop and
said am not sure he smile and said No place ,
actually he ask soooo many Q i answer fast he
ask next Q
neuro case is my bad one i got 8/20 on it case
of young male with paraplegia with sensory
level to T4 i got Rt limb spastic with aggressive
clonus second limb down goin with hyporeflexia
and no clonus but am not sure about the case i
got confuse because i think what could be the
cause i think simply may be MS , or
Compression in which side there is destruction
of vertebrae with compression of root in one
side till now am confuse i said may be there is
prepheral neuropathy ?! actually no place to
may be either sure or not so the examiner is
Indian v. tough he ask me So many Q i feel bad
but i think i manage not good way
: i already share this case on detail befor i will
search and copy best it and i will she feedback
of it when i get it.
station 4 case of young 37 years leady she work
as part timer teacher also she start with ( mature
education ... etc not remember exactly but
means no she start to study medicine collage
she experience tinnitus and blurred vision
which MRI and LB done diagnose as MS pt
already seen by neurologist who tell her about
her condition but she have some issue which
confuse about it and some concern so she ask
for adoctor i read scenario twice i understand i
have no clear plan i need just to set and answer
what she may ask when i enter examiner
ask me my evaluation sheet i forget
out side i go back one from out side ask me to
wait she will bring so i wait her all this on my
time so i enter then told me instruction
then start 1 min may be already left
: i introduce my self and confirm pt then agree
agenda of meeting and permission to discuss, i
ask her what she have been told so far she said
she is confuse about it i explain the disease the
behavior of it and it may be deference from one
to other i don't now why i offer pt i will give her
leaflet and website , supporting group can help
her more to know about disease (although this
step usually use to close meeting but may be
she already informed and still confuse i notice
expression of examiner follow me so just i go
on and ask her what she confuse about she said
am study medicine i said it is great as far as you
have nothing disabling you can study she
repeat it is medicine i thought she may give
me clue for some concern i said yes now you
regain your did you have any problem in your
vision she said no i said so great as far as you
free and you can do something i encourage you
to do it (in discussion examiner ask me did you
think in problem to study medicine?? then i
make sure that there is some thing i miss i said
now problem as far as she can but after
graduated some issue may will be concern she
said which issues i said may be affect her field
as may she need not to be post in ER or any
field deal with surgical skill as Surgery, OBG
according to her health that time, examiner said
so you think this important things to tell her
about it now i said yes i wounder may be no
need to tell all bad news this admission and to
make other appointment she ask then all Q of
surrugate agin as surrugate ask me she care of
her old mother i offer referal to social
department she said what they will do for me i
said can give you some expert advice also can
offer you nurse part-timer,a lot of option they
can discuss with you as nurse care or home
care for elder but if you want could you tell me
about your mother i close my Q fast by i means
did she have any medical problem??
: she said yes have limb pain and couldn't walk i
said am sorry so i think also your GP can help if
any treatable or she need medical care at least
can give you suggestion she agree then she ask
she need financial support i said who support
you before? she said no one she is work as
teacher i said it is great now you can continue
you work as you are free now but i will envolve
occupational department they can help you
regarding this issue
: she ask about family planing she plan to
marriage also get pregnant also she afraid to tell
her partner
alot of Q time over she still asking am not do
any summarization or check understanding nor
doing any thing and surprising things i got
12/16
: last station 5 first case is acromegaly i did it
good get 28/28
second case OSA i think due to hypothyroidism
i get 25/28
: conclusion exam about skill and how you can
manage cases by defrant way

paces exam date 6/10/16 at


ARMED FORCES HOSPITAL MUSCAT
--------------------------------------------------------------------
I started with :::::::::STATION 5/3/2/1/4
STATION 5
1- PLS EXAMINE THIS LADY HAVING
ARTHRITIS SINCE 5 YEARS COMPLAINING OF
HAND PAIN
2- THIS GENTLEMAN WITH HYPERTHYROIDISM
ON TREATMENT , HAVING EYE SYMPTOMS
INTERMITTENT DIPLOPIA AND WATERING ,
PLS TAKE FOCUS HISTORY AND
EXAMINATION
STATION 4
SHORT SCENERIO- PLS SPEEK TO THE SON
OF THIS PT AGED 68 Y HAVING DIAGNOSED
AS ILD 1 YEAR BACK , NOT RESPONDED TO
STEROIDS AND HIS LONG STANDING DM IS
WORSENED WITH THERAPY , LATER TRIED
NEWER DRUGS BUT DROPPED FROM TRIAL
.IN VIEW OF HIS ADVANCED DM AND
PROGRESSIVE ILD , PLS SPEEK WRT LONG
TERM PLAN OF MX OF ILD AS RESPIRATORY
TEAM HAS SUGGESTED ONLY PALLIATIVE
CARE
STATION 3-
RS- PLS EXAMINE THIS MAN AGED 62 WITH
SOB AND CHEST PAIN RT PL EFFUSION
P/A- PLS EXAMINE THIS MAN AGED 40 Y WITH
ABD PAIN AND FATIGUE - MODERATE
SPLEEN
STATION 2-
HISTORY SCENORIA THIS LADY 30 Y OLD
HAS OFTEN TIERDNESS AND EXHAUSTION ,
O/E- BP- 140/90 MMHG AND CBC HB- 10 , PLS
TAKE HISTORY AND ADDRESS HER
CONCERNS
STATION 1
CVS- 50 Y OLD MALE WITH CHEST PAIN
MURMURS AS/ AR/ MR TALL STATURE ,
HIGH ARCH PALATE , GYNECOMASIA , SCAR
LT ELBOW JOINT , SCANTY FACIAL HAIR
CNS- 35 Y OLD MAN WITH DIFFICULTY IN
WALKING HAS UMNTYPE WEAKNESS
CLONUS ASYMETRICAL WEAKNESS MORE ON
RT , NO CEREBELAR SIGNS , POST COLUMN
SENSATIONNORMAL NO SENSORY LEVEL

Expertience of a candidate with 16/16 in a


communication case
Scenario out side said : ( A lady with extreme
anger about her lost FNA result which lost with
a doctor who is on leave for 2 weeks... Which is
done for suspected mass on Cxr and confirmed
by CT scan ... Abd U/S show mass on liver.. We
don't know which is primary and which is
secondary that is why FNA and decision is to
repeat FNA again....!!! )
Candidate: Hello , This is Dr Jack , I am senior
house officer in MAU clinic , Nice to meet you ,
Would I get you to confirm your name and age
please?? You are Mrs : Jhones and you are 55
years old???
Surrogate: Nice to meet you Doctor , yes I am
the one
Candidate: I came today to discuss with you
your condition and results of tests done for
you... is that OK with you?
Surrogate:Yes I am waiting for that... please Doc
tell me what about my results ..is it some thing
bad?
Candidate:First of all Do you want any one be
with you here in this meeting?
Surrogate:No Doc I am OK...
Candidate:And would you tell me what do you
know so far about your condition?
Surrogate: As you know .. I am waiting for the
sample reusult took from my tubes .. I don't
know why they did this but I came with pain on
my tummy here in the right side ..then they did
scan of my tummy ..then again they asked about
chest scan... I am confused Doc .. No body told
me any thing...they talk about mass in my liver
and tubes... Is it CANCER Doc???
Candidate: I am afraid to tell you that some
thing serious going on..( silence )...The results
altogether is showing that you have a spreading
Cancer on your liver and tubes ...so sorry to tell
you such bad news....( looking around for tissue
, patient cried , I gave her tissues , still crying ..I
wait for her to raise her head about 1 mintue
then I start to say ) There is another bad news
for you as we did not know the origin of this
Cancer, is it from liver go to your tubes or vice
versa....That is why we did a sample test for
you....But I am sorry again to tell you that..my
colle...
Surrogate: Yes what about the result..?
Candidate:I am sorry again to tell you my
collegue doctor who saw you last time ..he is on
vaccation for weeks and unfortunately your
result is with him...so we lost it..
Surrogate: What a bad hospital is this ...what is
that mean??
Candidate: I am deeply sorry to tell you that we
search here and there for your result but
unfortunately we did not find it .... we tried to
call him many times but his cell phone was
switched off all the the time and this is the only
way to reach him...
Surrogate: Why this always happen to me...
your system is very bad really .. and i think you
are playing with people lifes..( Got extreme
anger ).
Candidate: You are absolutly right regarding the
abscence of your result ... and sure this fault
will be ivestigated by our team here ..so not to
happen again for other person.
Surrogate: You are making me confused Doc... I
want to complain ...
Candidate: You have Right to complain ...but I
need to say to you.. again we want to repeat the
needle sample for your tubes again...
Surrogate: What ..What ..No No Doctor ..Last
one is bad experience to me and I will not repeat
it again..
Candidate: I am afraid to tell you that ..this is the
only way to know what kind of cancer you
have..by knowing the nature of it..we can deal
with it
Surrogate: No I want to complain...
Candidate: As I told you Mrs Jhones ... This is
your Right .. and if you want this we have here
in the hospital a complain system and I will tell
you what channels to go through..But Let us
think positive about things..You have this
cancer and we are not sure if it is treatable or
not(really I dont konw)..and if so what kind of
treatment..is by chemos or Radiation or Surgery
or all together..and to answear all these
questions we need to Do the needle sample test
for you...and I am sure that the procedure
discussed with you before and you know every
thing about it..
Surrogate: It is painful doctor...crying
Candidate: I will be sure that they will make you
pain free this time and I will be with you to make
sure of this..
Surrogate : Please Doc I am alone ..would you
help me..?
Candidate: Sure and it is my pleasure..But let
me know about your family where are they ?
Surrogate : I left My husband after 20 years
marriage last month..and I fired from my job
also..(crying -Tissues... At this time I said to my
self How much RCP gave these surrogates ? As
she able to cry with many tears at any point of
time..and surprised that this will be repeated for
other 4 candidates with me in the pannel ...Good
actress Really..)
Candidate : I will ring now For the social worker
and psychologist to attend our next meeting in
Thursday with all of the Team concerning about
your condition after the result of your needle
sample test...and this team including cancer
doctor, our consultant, liver doctor ,me and
chest doctor...to decide about the way of
making you symptoms free for your rest of
life...and I am afraid to say that your cancer is
spreading now and we need to control it by
palliation only..that mean to make you as
comfort as we can but No total cure for this..
Surrogate: Crying...WHEN IT WILL BE DONE ?
Candidate: I will make the nearest apointment
and ask them to realease the result on the same
day..so as not to delay things
Surrogate : Thank you Doc..
Candidate: As before you need to sign a PAPER
if you will Go for this..
Surrogate : OK doc
Candidate: We go through alot of information
would you recap for me what you get from our
discussion today..?
* AT this point examiner stop me that time is
finish..I felt worried about that as I did not
Summarize and check
understanding...Ohh..God
* They ask many questions? I automatically
replied by defending mechanism. I think this
helped me to pass this station

Um last group. 26/10/16


Bcc - elderly gentleman, blind since young.
Came with ?lethargy and memory impairment.
Went in...he has extensive psoriasis... was given
some hormonal treatment....
Havent got a clue how to link all.. gone
the next one is a thyroid pt...eye sign. Stem Sob
and ankle swelling.
Counselling - ms break bad news.
History - collapsed.
Cvs - mvr.
Neuro - blank.again. i think it is myotonic
dystrophy.
Respi - inpatient. Clubbing. tracheal deviated to
the left. Reduced air entry. Not sure - said it was
left upper lobe collapsed. Ca. Didnt do well.
Abdo - renal transplant.
Looking forward for next seatin

Hospital Serdang, Malaysia.


22nd Oct 2016.
BCC
1. LL swelling, discoloration, with diarrhoea 3
months.
2. SOB, acute onset in a dialysis patient.
Station 1
Resp
Bronchiectasis
Abdo
Failed renal transplant
Station 2
Hx of headache
Station 3
CVS AVR
Neuro: proximal myopathy
Station 4
ADPKD counselling

UM red team 26/10/16


Last carousel
St 1: hepatosplenomegaly with pallor- CML
RA with cushingoid features with interstitial
lung disease
St2- syncope with collapse-sounded
cardiogenic syncope
St3- MVR in failure
Bilateral claw hand with small muscle wasting
of hands probably bilateral ulnar nerve palsy
disucss along differentials
St 4: breaking a news of multiple sclerosis
St5: BCC1- headache, young man has h.o acne
taking isotretinoin and pcm, gave differentials
BIH, migraine, tension headache
BCC2: diarrhea with vommiting and fever, skin
lesions looks like mix of pemphigus and
psoriasis, taking aza and prednisolone..gave
differentials of infective diarrhea, inflammatory
bowel,drug related....just hoping for the
best...dunno whether gud enuff to pass

Um red group. 2nd cycle 25 october, centre UM


Start with BCC 1
Heart pain? Patient said. History more on gerd.
Obvious facial rashes with scar, lucky pt said it
is pemfigus on steroid. Has lower limbs
weakness with cushingoid on steroid.
Ddx : I give GERD, IHD, side effects steroids.
I ask concern and examiner said times out
.Sad.
Bcc 2
Knock into object, vision blurring.
He got DM dermopathy, right BKA.
Vision until waving fingers and light perception.
I offer funduscope,.and ecaminer said ok = eye
not dilated, no.red reflex. I.cant visualize.retina.
Ddx. I give poorly controlled dm. Concern
is.driving, I said need medical board and license
agency. Multidisciplinary team.
Funduscopy I said I cant visualize
Station 1
Lady with mass at abdomen, so nodular, I think
both kidney balotable, dont know if its kidney or
liver... I said ADPKD.
Respi
Inhaler on table - before bell.rang than I
mentioned ...
Trachea deviated to right, I said ddx fibrosis,
mitotic.
No ronchi no wheeze. I said possible ashma or
copd...
Mx I said pulm rehab and stop smoking. And
goh bak liong ask how u know pt is smoking? I
said I can see inhaler, possible copd
St 2
Diarrhea 2days,.he is gardener. Bloods renal
failire....On lithium.
Ddx infective age, hus/ttp, and lithium causes
nephrohenic DI
St 3
Metalic dual valve replacement. Why pt is sob? I
said related to heart and may not.related. No
murmur. Can hear click from bedside.
Neuro, see gait n proceed
Lady with broadbase gait. Cant do knee jerk
because pt pain, lucky I do elbow jerks.= hyper
reflexia.... Very sartle nystagmus to horizontal
gaze....
Ddx cerebellar, acquired congenital drugs
End with.st.4....
No more adrenaline left... Lucky Miss Ismail so
coperativr...
Angry pt to consultant cardiologist, pacemaker
not function, no body explain, she in ccu, want
to home wedding nephews... I adress one by
one slowly, and pt keen to stay and do another
repositioning pacemaker... Thank god.
All the best others

um26/10
morning session blue carousel
1)
abdo thalassaemia
respi : lobectomy
2) hx taking : low, night sweats
3) cvs : ms/mr some people got single valve
disease ive got mixed mitral valve.. dunno
neuro : bulbar palsy with hearing loss
4) BBN esrd 2' adpkd
counsel for possible rrt
5) bcc 1 abdo discomfort and constipation 80 yo
on morphine/ codamol
bcc 2 recurrent syncopal attack
hoping for the best
Hospital Serdang, Malaysia.
22nd Oct 2016.
BCC
1. LL swelling, discoloration, with diarrhoea 3
months.
2. SOB, acute onset in a dialysis patient.
Station 1
Resp
Bronchiectasis
Abdo
Failed renal transplant
Station 2
Hx of headache
Station 3
CVS AVR
Neuro: proximal myopathy
Station 4
ADPKD counselling

2nd cycle 25 october, centre UM


Start with BCC 1
Heart pain? Patient said. History more on gerd.
Obvious facial rashes with scar, lucky pt said it
is pemfigus on steroid. Has lower limbs
weakness with cushingoid on steroid.
Ddx : I give GERD, IHD, side effects steroids.
I ask concern and examiner said times out
.Sad.
Bcc 2
Knock into object, vision blurring.
He got DM dermopathy, right BKA.
Vision until waving fingers and light perception.
I offer funduscope,.and ecaminer said ok = eye
not dilated, no.red reflex. I.cant visualize.retina.
Ddx. I give poorly controlled dm. Concern
is.driving, I said need medical board and license
agency. Multidisciplinary team.
Funduscopy I said I cant visualize
Station 1
Lady with mass at abdomen, so nodular, I think
both kidney balotable, dont know if its kidney or
liver... I said ADPKD.
Respi
Inhaler on table - before bell.rang than I
mentioned ...
Trachea deviated to right, I said ddx fibrosis,
mitotic.
No ronchi no wheeze. I said possible ashma or
copd...
Mx I said pulm rehab and stop smoking. And
goh bak liong ask how u know pt is smoking? I
said I can see inhaler, possible copd
St 2
Diarrhea 2days,.he is gardener. Bloods renal
failire....On lithium.
Ddx infective age, hus/ttp, and lithium causes
nephrohenic DI
St 3
Metalic dual valve replacement. Why pt is sob? I
said related to heart and may not.related. No
murmur. Can hear click from bedside.
Neuro, see gait n proceed
Lady with broadbase gait. Cant do knee jerk
because pt pain, lucky I do elbow jerks.= hyper
reflexia.... Very sartle nystagmus to horizontal
gaze....
Ddx cerebellar, acquired congenital drugs
End with.st.4....
No more adrenaline left... Lucky Miss Ismail so
coperativr...
Angry pt to consultant cardiologist, pacemaker
not function, no body explain, she in ccu, want
to home wedding nephews... I adress one by
one slowly, and pt keen to stay and do another
repositioning pacemaker...

Malaysia - UMMC
25/10/2016
Respi : COPD, rhonchi, hyperexpanded chest,
on NPO2, tachypnoeic, greenish sputum dd
pneumoni, TB.
Abd : Right transplanted kidney functioning
well, no CVL no AVF. Cushingnoid
History : transient global amnesia
CVS : ?MR in failure ???
Neuro : Charcot-Marie-Tooth / dystrophia
myotonica
Comm : 50yo man T1DM with hypoglycaemia
unawareness. Counsel regarding hypo.
BCC 1 : 30yo male, LOC with vomitus beside
him, PCM 20 tablets, social drinker with recent
increase past 2 weeks stress marriage affair.
Physical all normal.
BCC 2 : Dysphagia, typical Dermatomyositis.
Likely mixed CTD. Skin tightness as well. ?hx of
breast ca. ?esophageal ca.

Exam cases October 2016


Station 5
RA with CTS
Palpitations secondary to anxiety
Station 1
Pulmonary Fibrosis
PBC
Station 2
Upper GI bleed Mallory weis + on NSAIDS
Station 3
TOF
Peripheral Neuropathy
Station 4
BBN of renal failure requiring dialysis.
Kuwait 2016
Our cases in kuwait
In two days
Neoro: MND
Cardio: Af + MS
Hx: 1) diplopia and fatigue (MG)
2) back pain and hypercalcemia
ABD: splenomegaly >>> Hereditory
spherocutosis
Other case: polycystic kidney
Resp: Lobectomy
Comunication: clear medical error FBA sample
was lost and you need to repeat it
Station 5: joint pain in hands
Other pt: garves eye dis

Another Feedback from colleague in Dubai


today:
St 1:
Chest: Rt side lung fibrosis.
Abdomen: Renal transplant.
St 2:
DVR, AF, DM, on warfarin, hypothyroidism on
thyroxine has IDA on iron but not responding
referred to you to look for a cause. A CASE For
DD
St 3:
CVS: DVR.
CNS: spastic paraparesis without sensory level.
St 4:
PT with chest pain who underwent stress ECG
which is positive.
Your role to tell him test result & to explain for
him that he needs c angiography & may be
CABG after that.
St 5:
BCC1: Pt has SOB: Then found to have Wt loss,
diarrhoea, thyroid nodule & neck scar.
BCC2: ALSO SOB: progressive then found to
have RA on MTX.

My colleague exam in dubai


5th October
Station 1
1 /Ascites for differential
2/ rt lower lobe consolidation pt had cannula
also
Station 3
1/Aortic regurgitation /AF marfan?
2/ hemiplegia ( examine upper case limbs)
Station 2
Back pain ,ankylosing
Station 4
Somatization disorder
Station 5
1- DVT
2- Amurox fugax

Dubai today:
St 1:
Chest: Rt side lung fibrosis.
Abdomen: Renal transplant.
St 2:
DVR, AF, DM, on warfarin, hypothyroidism on
thyroxine has IDA on iron but not responding
referred to you to look for a cause. A CASE For
DD
St 3:
CVS: DVR.
CNS: spastic paraparesis without sensory level.
St 4:
PT with chest pain who underwent stress ECG
which is positive.
Your role to tell him test result & to explain for
him that he needs c angiography & may be
CABG after that.
St 5:
BCC1: Pt has SOB: Then found to have Wt loss,
diarrhoea, thyroid nodule & neck scar.
BCC2: ALSO SOB: progressive then found to
have RA on MTX.

My colleague exam in dubai


5th October
Station 1
1 /Ascites for differential
2/ rt lower lobe consolidation pt had cannula
also
Station 3
1/Aortic regurgitation /AF marfan?
2/ hemiplegia ( examine upper case limbs)
Station 2
Back pain ,ankylosing
Station 4
Somatization disorder
Station 5
1- DVT
2- Amurox fugax
EGYPT== CAIRO
Maadi hospital exam
12 Oct 2016
laste corrosal
1-HSM +Pallor+left sub mandibular LN +left
axilla scare
Mostly lympho proliferative disease
Left upper lobectomy +trachea shifted to left
+brochiectasis
2-asthmatic worsening symptoms with pet
animal at home +taking propranolol
3- cvs left infra mammary scare +AF+MS so
mitral restenosis post valvotomy with pul HTN
Cns left hemiparesis+left UMN facial palsy
4-young visitor with cruciating occipital
headache with
vomiting CT normal for lumbar punture want to
go DAMA to fly back to USA
5-BCC1-
psoriatic arthropathy
Bcc2-
generalized strange rash never I saw before
started 2 weeks back with alopecia totalis in
HCV patient on interferon 5 months ago
+ribaverin
Examiner told one of our collegue may be lichen
plans?
Serdang Hospital, Malaysia. MRCP PACES 22nd
October. 3rd carousel
1. Abdo: Pt came in with Abdo pain.
Transplanted kidney. Both sides have inverted J
shape scars. Both sides palpable mass. Left
wrist AvF scar but no palpable thrill
Respi : Long thoracotomy scar on the right side.
I think had fine creps over the base of right
lung. ? Lobectomy secondary to malignancy
/abcess/volume reduction surgery? Transplant.
No idea
2. History : Young chap with history of asthma
and cough. Cough and wheeze worsening for
past 4 months despite escalation of
therapy/Inhaler. Usually 3,4 episodes per week
at night. No failure symptoms. On further
history, had problem at work, possible lay off.
Wife seeing psychiatrist for depression. Patient
taking propanolol prescribed for her wife for his
own symptoms.
3. Neuro : Young lady with difficulty walking.
Broad based gait, positive romberg. Spastic
paraparesis, increased tone, brisk reflexes,
upgoing plantar on left. No sensory or
propioception impairment. Heel shin okey. I
think MS or SCA or Friedrich. Have few other
differentials for upper motor neuron lesions
CVS: Very young chap with pacemaker or
maybe ICD. Came with palpitations. Really can't
appreciate double apical impulse. Just some
systolic murmur LSE and at the apex, I'm not
sure what murmur. Maybe systolic
murmur.Gave HCOM or MVP as differential
4. Comm skills : Talk to patient's son about his
father prognosis. Diagnosed ILD. Multiple
admissions. Baseline function deteriorating.
This admission with chest infections covered
with iv antibiotics. Respi team suggested
palliative care. Patient doesn't want more
admission.
5. Case 1: Young chap with double vision and
diarrhoea. Looks like Grave ophthalmopathy.
Unsure if AF or not. Visible palpable goitre.
Case 2. Young lady with gum swelling. Not so
pretty but make-up so thick. History of seizure
on phenytoin. It's gum hypertrophy.I asked for
any rashes or skin changes . She denied. Later
after exam they said its tuberous sclerosis but I
really can't find the rash because of the make-
up. Died
Malaysia Hosp serdang 22/10/16
Station 1 i think ILD and Renal transplant
Station 2 headache
St 3 proximal.myopathy and AVR
St 4 explain ADPKD diagnosis
St 5 pretibial myxedema with goitre
And Esrf pt with AVR came with 1day of SOB

UK exam 2016
History taking
Jaundice in a traveler after returning from
Kenya.
Communication Skills
A patient with end stage COPD: explain to his
daughter about the risks and benefits of
mechanical ventilation.
Station No. 5
A. Neurofibromatosis
B. A female patient with tiredness, weight loss
and history of Graves disease/Rheumatoid
arthritis? --Coeliac disease/Addisons?

Oman 09/10/16
1-Abdomen: Young adult with mid line upper
abdominal scar. Hepatosplenomegaly. No
peripheral stigmata of CLD, not pale nor icteric
and no palpable LNs. Subtile parotid
enlargement.
1-Resp: Young not in distress, well built, no
clubbing, apex not palpable at Lt side, but
indeed it is on Rt side, with mixed corse creps
and some rhonchi.
2-History taking: Young male with typical
migraine headache without aura not responding
to overcouter codamol, plus mild
chronic tension headache on top of his
migraine. His concern was he has difficult time
in his job and others with his headache.
3-Cardio: Metalic valves.
3-Neuro: Young, obese with bilateral lower limb
weakness, mainly proximal, with good distal
power, normal sensory and cerebellar
examinations. Planters downgoing. There was
scar at Lt thigh, probably a muscle biopsy.
Impression: Myopathy.
4-Ethics/Communication: Female,
physiotherapist at stroke unite, admitted with
acute hemiplegia involving limbs but sparing
crandial nerves. Examinations variable and not
conclusive plus normal work-up included CT
and MRI brain. Impression was functional
weakness. She is angery and wants to talk to
doctor now as she heard somebody saying that
she is faking her symptoms. Plz see her and
discuss the management.
5-Case1: Young female with headache. Please
see her. Fruther history revealed headache,
visual distrubance, amenorrhea, changes of her
shape consistent with acronegaly.
Examinations: acromegalic features and
bitemporal hemianopia.
5-Case2: Young adult with back pain and found
to have cardiac murmur. Please see him.
Fruther history: chronic back pain with morning
stiffness and restriction of movements. No rash,
no diarrhea, no trauma, not fever, no
neurological deficit. Examination: typical AS.
AR murmur.

Feel very happy and privileged to tell that by the


grace of Almighty passed my paces exam with a
score of 170/172 in Glasgow this diet...very
thankful to all the group members for their
efforts of posting such useful information
constantly... So here it my experience of paces
in Queen Elizabeth Hosp Glasgow... St1- abd
was ascites due to cld with portal htn, resp was
lobectomy due to pul avm/lung abscess...st2-
Dx was transient global amnesia,many got it a
bit wrong thinking of tia,examiner actually at the
end of viva told me that I was the only person
giving the right Dx,so that helped to boost the
confidence...scenario was a 50yr old lady
became confused for 2hrs and became normal
again after the attack.she had a friend along
side who said that she was saying and behaving
abnormally,there was no weakness,no loss of
consciousness,no history of trauma or
seizures,sweating,mood changes..this was the
fort time she had it...she is all well
now..previous history of diabetes...my Dx was
transient global amnesia and dd were
tia,hypoglycemia,electrolyte imbalance and
seizures...examiners were very happy..
Concern as usual is it stroke?...st3- cardiac was
aortic stenosis with ejection systolic murmur
radiating to carotids,low vol pulse and a
Pansystolic murmur in apex due to gallavardin
phenomenon... Neuro case gave me nightmares
as a young male who on examination had only
left up going planter and all normal findings...it
wasn't a hemi,everything else was normal.
I thought I missed something but as it turns now
that was the only sign present...Dx was multiple
sclerosis,i reached the Dx thinking of having an
upper motor sign with a 30-40yrs old male... St4-
counsel a male pt of 32yrs for Hickman line for 6
cycles of chemotherapy for Hodgkin's
lymphoma...concerns were-why him?how to tell
his wife?what about fertility as he just been
married and wants kids?what is the
prognosis(pt had stage two A)?...st5- one was
Turner's syndrome with
hypothyroidism...concerns were about mostly
all the complications of Turner's...and sec one
was typical psoriatic arthropathy on mtx but not
controlled....examiners main focus was of
biological agents and multidisciplinary
approach... Lost a mark in neuro and
Turner's...except that it was a blessing from the
doors of Allah and thankfully now can say
passed the biggest hurdle of paces...my advice
like most of the experienced doc here will tell u
there is no alternatives to practice...make a
scheme,make a plan and practice... Practice the
common cases for st1,3...collect all the past
cases as much as possible for st2,4,it gets
repeated so many times...solve it and practice
with friends or anyone... And also practice very
very hard for st5... St5 carries one third of marks
of whole exam,a bad st5 makes passing so
much difficult...about books I followed safely al
rokh sir's pdf for st4, made my own notes for
st2, cases for paces for st1,3...and ost and
sadek al sir's pdf for st5 ... And lastly thankfull
to all the group members for such high standard
of work and helping candidates throughout this
tough hurdle of paces....

Glasgow UK on 17/10/16
St.1 - abd- transplanted kidney
resp - pnuemanectomy scar
St.2 - c/o palpitation ,headache - MEN
St.3 - card - AS & Neuro - Parkinson's ds
,examination of lower limb.
St.4 - explain for OGD for bleeding varieces
St.5 - 1)arotic valve replacement in c/o
palpitation
2)headache
Malaysia
Cvs- mr
Respi- coad
Abd - renal transplant
Cns- ms
Hx taking -hemoptysis
Comm skill- first unprovoked seizure ,update &
advice to wife
Bcc - ankylosing spondy
- churg strauss synd

London. bedford.
my friend cases in uk: respi - kyphoscoliosis
and? Rheumatoid arthritis with crepitations and
tracheostomy scar, likely bronchiectasis.
Abd - Mercedes Benz scar.
Hx - SLE.
CVS - AS.
CNS - Right frontal scar with right CN I, partial
CN III, LEFT V2?, CN VII involvement. Sorry
don't know what is going on here. Anyone, any
input?
Com-speak to daughter of patient who has
advanced copd who is doing poorly. Your
consultant thinks the prognosis is poor but
intensive Care has not been ruled out. Patient
has mentioned that he would do anything to
attend his granddaughter's wedding in 3/12
time. Your task is to explain to the daughter the
patient's current condition, inform her of current
prognosis, and explore patient's wishes.
Bcc: 50/Caucasian lady post partial
thyroidectomy presents with lethargy
(hypothyroidism sx).
Bcc: 72/Caucasian gentleman k/c parkinson's
come with frequent falls and fluctuating BP(
postural hypotension&dizziness ).
The cns one of my friends thinks it is operated
npc
The bcc parkinson's, pt is on bisoprolol n
warfarin for his heart as well

Exam experience in St George hospital London


Start station 5
BCC 1 pto with uncontrolled HTN, previous
history of carotid body tumor surgery, with neck
scar, concern is tumor is back, I said it might be
possible need further investigation didn't get
much information about recent control so that
was mistake, examination was normal,
BCC2 ankle pain acute with urinary problem, it
was fake patient because he was walking
outside normally, on exploration multiple sex
partners so gave diagnosis of gonococcal
reactive arthritis, examiner was interested in
Reiter syndrome
Station 1 I could only find basal crepts and
pedal edema couldn't get it right May be I was
also getting prolong expiration, cough was dry,
so I gave ild with pulmonary hypertension, I
don't know it's right or not
Abdomen pt with abdominal pain and fever with
peritoneal catheter in place I think so I got
fullness in flank but could not get any
visceromegaly they were asking about causes
of abdominal pain I said peritonitist , asking
more I couldn't recall more,
Station 2 hemarthrosis, in old patient with
differentials of gout, septic trauma INR 3.5, I
forgot to ask about compliance though asked
about any problem with medication but still I
miss important issue
Station 3, cardiology I ran out of time in station
1 and 3 due to less practice, there was long scar
in middle age lady with tremors in hands , I
couldn't appreciate any murmur, they were
asking about causes of long scar and tremor I
said amiodarone but couldn't recall more I think
ciclosporin tremor with heart transplant but I
don't know about isolated heart transplant scar
Station Neuro peripheral neuropathy
Station 4 Esrd need lot of issue as cause was
polycystic kidney disease, with husband blind,
issue with dialysis, genetic counseling
Uk Truro (London college)
17/10/2016
Cvs
Congenital heart repair
Median sternotomy scar
No murmurs
Discussion on previous possible causes and
complication eg asd ,vsd
Questions focus more on asd
Neuro
Lower limb
Spastic paresis
Mnd likely
St 2
Hx taking
Hx abd pain and diarrhea
Patient has hx irritable bowel syndrome before
Worry as has strong family hx of colon ca
St3
Respiratory
Rt lobectomy with clubbing
Abd
Jaundice with splenomegaly
Com
Newly dx ckd
Previous gp had nt conveyed enough
information as she has health screen which she
choose nt to investigate
Counsel on ckd and posibility of dialysis in
future
Bcc1
Hx dm type 1
Worsening eye vision
Dm retinopathy
Bcc2
Hx uncontrolled dm with fever and lower limb
ulcers
Gt charcot foot changes with dm neuropathy as
well as skin redness at left foot ? Cellulitis
Best of luck to all

Copied
16/10/16
Station1 pulmonary fibrosis
Renal transplant
Station 2 sob
Station3
Avr tissue
Neuro peripheral neuropathy
Station ca lung
Station5 headache likely hemiplegic migraine
Back pain. Ankylosing spondylitis
Cairo 13-10 (Courtesy of Dr Ahmed Farouk )
Abdomen: HSM with ascites.
Chest: lung fibrosis, although clear chest, she
has clubbing and thin skin,she has also
cachexia..
Neuro: celebellar syndrome, flaccid paraplegia
and areflexia and downgoing planters with P N,
old age excludes f. Ataxia, so it could be due to
multiple strokes
Cardio: AVR and ASand probably Aortic flow
murmer or aortic regurge for echo assessment
History: headache
It was migraine vs drug induced
Communication: angry patient as her dad died
due to no beds in HDU also a missed dose of
antibiotics
Station5: blurred vision in one eye, painful eye..
Fundux not accessible.. Diagnosis was Behcet
with anterior uveitis, then he said make the
complaint Acne
The other case, rash on elbow and knee, firm
nodule, the only positive finding is Shortness of
breathing, xanthoma

Exam cases- bilateral UL lobectomy (resp) renal


transplant with functional but unused AV fistula
(abd) MEN1(history) bioprosthetic AV (cardio)
sensorimotor peripheral neuropathy (neuro)
explaining diagnosis of MS(comm/ethics)
Neurofibromatosis 1(Bcc1)& Ankylosing
spondylitis (Bcc 2)
My friend cases

Uk grimbsy 15/ 10/16


Station 1
Pulmonary fibrosis
Cld only palmar erythema was present.complex
patient
Station2
Wt loss night sweats
Dd lymphoma / tb / hiv
Station 3
Cvs
Aortic valve replacement tissue valve only flow
murmur was present
Neuro clubbed foot , proximal myopathy
No sensory loss DD Muscular dystrophy
examiner asked what else i said i would
examine spine n also do upper limbs to rule out
mnd.
Station 4 hypoglycemia with unawareness.
Address concerns i forgot to ask
smoking.everything else was ok
Station 5 back pain n hand pain with rash
.....psoriasis
Dd psoriasitic arthropathy
Second station 5 hoarseness
Complex case
Lady was smoker n had stopped thyroxin , was
also taking steroid inhalers , actively somoking
no lumps n bumps in neck
I told examiners that i would restart thyroxin n
tell correct technique of inhalers to rinse mouth
after use....he asked what else , i said ca larynx
.....examiner was happy.bell rang

UK- Chester 15/ 10/16 (Courtesy of Dr Umar)


Station 1 lung transplant
Polycystic kidney
Station2 hypertension
Station 3 double mettalic valve
Neuro wasting of hand with no sensory loss dd
mnd
Station4 hypoglycemia with unawareness
Station 5
Osa
Rheumatoid arthritis with sob

Cairo 13/10/2016
1st carousel
ST3
NEURO: bilateral cebellar lesion, loss of deep
sensation, high stocking hypothesia to
superficial sensation+nystagmus bilateral (M.S
with peripheral neuropathy, cerbellospinal
degeneration, multiple strokes)
CVS:AVR+AS+??MR
ST4: Death of father 75 yrs copd, pneumonia
crub 5, admitted to surgical ward 2 days ago,
detoriated, transferred to HDU, cpr failed,
cannula dislodger and miss 1 dose of ab...
It was tough one
ST5:
Male 25, blurring of vision in lt eye with
retroorbital pain 3 months ...mother is blind
56yrs.. was not cooperative on fundus ex....lt
eye catract & pigmentation.....i can't appreciate
any thing else in both eyes.....he had acne on no
rx, stria rubra in his arms
D.D (what i put)
Lebers
RP
Optic neuritis
I did it badly
BCC2
Rash on lt elbow+htn
I misses analysis of htn...chest pain...yellow
rash on elbow and knee...adress concerns as
pemphigoid, D.H,
D.D
Pemphigoid
DH
PSORAISIS
Examiner ask me what is relathion to htn? NF
with pheo
He ask again with relation to chest pain, +F.H of
stent in mother?....i answer tuberous
xanthomata, then bell ring
St1
Chest: COPD
Abd: hepatomegaly in morbid obese pt
ST2
Headache (1ry type, migrane without aura,
cluster, analgesic misuse)
I feel not happy with ST5
Ask god 4 me, it is my 2nd attemp, last one
130/172 fail in identifying signs
I book the next diet...as i had only two attemps
then 7yrs will be finished
Again...ask god 4 me

Egypt 13/10
Elmaady
Station 1
COPD
Thalassemia
Station 2
Lithium toxicity nephrogenic DI
station 3
AVR +MR
Facioscapulohumeral
Station 4
Cl. Difficile diarrhea
Station 5
Epigastric pain indomethacin
Paroxysmal nocturnal hemoglobinuria

EGYPT
Cairo 12-10 - 16
St 5
1- Male patient with diarrhea (sometimes
bloody) and abdominal pain.. He has psoriasis
and taking methotrexate.. Concern about cause
of diarrhea and abdominal pain ? IBD, NSAID
induced errsions, IBD, methotrexate, cancer.
2- Male patient is complaining of sore throat.. By
history and examination he has thyrotoxicosis
and on carbimazole.. Concerned about the
cause of sore throat.. Carbimazole induced.
St 1
Chest..COPD with? Basal fibrosis.
Abdomen..? CLD but without signs could be
early cirrhosis
St 2
Young female presented with fatigue and by
history she has joint pains, photosensitivity and
malar rash with previous dvt and miscarriage...
Diagnosis is SLE and antiphoshpolipid
syndrome.. Concern about if she can get
pregnant.
St 3
Neuro.. Young male with difficulty in walking
examine cerebellar syndrome.. Patient has
Upper motor pyramidal lesio and cerebellar
signs.. MS
Discussion about DD of cerebellar syndrome.
Cardio.. AVR with many murmers! (Not sure of
them)
? Aortic stenosis? AR ?MR.. AF
St 4
Young patient type 1 DM on insulin and has
anawareness of hypoglycemic attaks... This
case is a history taking case.. Should ask about
insulin dose change, type of food, increased
activity, smoking, drugs.. On this patient he is
not compliant to insulin dose written for him,
takes b blocker for htn, history of IHD, smoking..
All these factors should be asked about and
corrected to solve his concern

Egypt 12/10/2016
Communication
Lumber puncture to exclude subarachnoid
Station 5
Skin rash in HCV
Joint pain in psoriasis rheumatoid type
Station 2
B. Asthma uncontrolled
Pets
BB
Station 1
Scare with lobectomy
HSM+LN
Station 3
Spastic paraparesis
MVR+AF

Egypt Cairo 12-10 - 16


second carosel (details will follow later)
St 1
Abdomen : splenonomegaly with shrunken liver
for dd? CLD.
Chest COPD.
St 2 young female with joint pain, skin rash, h/o
dvt and miscarriage.. SLE And
antiphospholipid.
St 3
Neoro MS
Cardio AVR with some other murmurs!
St 4
Young man type 1 DM on insulin with
anawareness if hypoglycemic attaks.. Discuss
(as if history case!)
St 5
1- Psoriasis with abdominal pain and diarhea:
could be IBD, nsaid effect,
methotrexate,cancer...
2- Hyperthyroidism with sore throat due to
carbimazole

UK PACES EXAM Experience--Nottingham


10/10/2016
Resp.
Double lung transplant
Abd.
chr. liver dis
Cvs.
metallic valve
Neuro .
peripheral neuropathy
History.
Upper GIT Bleed.
Communication.
Chr. kidney dis 2ry to APKD (breaking bad
news) and mgt
Station 5.
palpitations
carpel tunnel syndrome
Cairo Egypt 11-10-16
second carosel
St 1
Abdomen CLD
Chest COPD with bilateral basal dullness
St 2 night Fever and sweats for dd : tb,
lymphoma, hiv, malaria, seretonin syndrome.
St 3 Cardio?? VSD with cyanosis and clubbing
(not sure)
St 4 angry patient post pacemaker insertion and
one the wires dislodged.. For another session of
insertion.
St 5
1- Addison with family history of hashimoto.
2- vitiligo with pernicious anaemia.

Cairo 11-10
Station 1
Splenomegaly with lymph node
Clubbing with basal fibrosis
Station 2
Confusion
Station 3
Hemiplegia
AVR
MVR AF
Station 4
Gentamycine toxicity
Station 5
Short stature
Rheumatoid with basal fibrosis

Cairo Egypt 11-10-16 second carosel


St 1
Abdomen CLD
Chest COPD with bilateral basal dullness
St 2 night Fever and sweats for dd : tb,
lymphoma, hiv, malaria, seretonin syndrome.
St 3 Cardio?? VSD with cyanosis and clubbing
(not sure)
St 4 angry patient post pacemaker insertion and
one the wires dislodged.. For another session of
insertion.
St 5
1- Addison with family history of hashimoto.
2- vitiligo with pernicious anaemia
Dubai
11/10/2016
History
Collapse
Pt known case of breast cancer
St 1
Hepatomegally
CLD
bronchiactesis
St3
VSD
flaccid paraparesis
St4
Pt with history of #
Suspected to have gaint cell arteritis given
steroid
So angry
St5
Scleroderma
Graves

EGYPT -- CAIRO= 10/ 10/2016


St 1 : Resp
ILD/COPD
Abdomen
HEPATOSPLENOMEGALY WITH ASCITIS/ A
SINGLE SPIDER NAEVUS
ST 2 HISTORY
35 , female k/c T1DM,
RETINOPATHY,NEUROPATHY,NEPHROPATHY,
AF ON WARFARIN, HTN, WITH PAST HX OF
MI,HF 5 months ago,
presented with a hx of recurrent collapses since
three months. She also had night time diarrhoea
since few weeks. She has lost awareness of
hypoglycemia.She is on warfarin 5mg ,
bisoprolol 2.5mg Lisinopril 20mg,Amlodipine
10mg,Digoxin 0.5 micgm ,Frusemide 80mg.
Viva on DD. Hypoglycemic
episodes/arrythmias/autonomic
neuropathy/Addison's dx/ drug induced
postural hypotention/ diarrhoea for similar
reason. Investigations (of autonomic
neuropathy especially) / management.
St 3
CNS :
25 yr old with spastic paraparesis since 16 yrs.
Viva on hereditary spastic paraparesis.
CVS :
35 yr old with SOB and hx of fever. Soft S1 with
soft Pansystolic murmur radiating to the axilla.
S 2 was normal.Could not appreciate a diastolic
murmur. Viva on IE.
St 4 COMMUN
Young female admitted for delivery. Had a
normal vaginal delivery. Was given benzyl
penicillin for vaginal streptococcal infection.
After being shifted to the ward ,she developed a
mild hyperemic rash which progressed rapidly
in a day to involve the whole body with
blistering and oral lesions. She is hypotensive
with breathing difficulty and has developed liver
and kidney dysfunction. Medical team is
thinking of shifting her to the ICU with a view to
a possible intubation. Talk to her husband who
is concerned about her wife's situation.
His concerns :
What happened?
Could it be avoided ?
Was there a negligence involved ? If so , he
would demand compensation.
What will happen ?
He overheard some doctors saying that she
might need to be put to "Sleep". What does it
means ?
Discussion on:
Could it be avoided ? What might have
happened ? What if the skin test negative ,could
one still get a Steven Johnson syndrome ?
How would treat a case of Steven Johnson
syndrome ?
What is autonomy ? Under what circumstances
could you breach it ? Did she have autonomy ?
St 5
Case 1 : 40 yr old male with itching for 10
months . No skin lesion (actor)
HX of foot joint pains on and off , for which he
takes analgesia on and off.
No past medical hx.
Father died of some blood cancer.
Smokes 40 cigs per day for 10 yrs.
St 5
Case 2 :
45 yr old male developed a blistering rash one
month ago after his brother died. He has oral
lesions.
No past medical hx of any kind accept some
wheezing on and off. He is taking aspirin for
some unknown reason since three months.
No family hx.

EGYPT -- CAIRO= 10/ 10/2016


Station 5:Cushing
Retinitis pigmentosa
Chest thoracotomy scar
Copd bronchiactasis
Abd spleenomegaly
History cramps & diarrhea
Father colon cancer
H/o ibs
Neuro
Flaccid paraperisis
With sensory level
Cardio
Loud s1
AF
Loud s2
Most likely ms phtn
Communication
Copd acute attack improved with bipap not
returned to baseline
Daughter wants talk to you

Corrections of date yesterday on 9/10/2016.


I started with stn: 2
Headache which make him suffering in his job
and taking co- co- codamol not improving. He
feel fine when rest in a dark room.
Stn: 3= Collapsing pulse and Predominant AR
AND mild AS.
Neuro= hypotonia with cerebellar syndrome
heel shin test positive with dorsal column
affection. = MS
Stn=4
Pt is a staff nurse admitted with Rt leg and arm
flaccid paraparesis ; CT scan normal and
neurologist consultant evaluated and advice for
MRI Brain which is also normal .
Counselling pt about the disease and further
treatment.
Stn 5=
Bcc 1= Headache with visual disturbance.
Acromegaly.
BBC 2 =
Pt has heart valve problem gp refer for echo.
And he complaints of back pain and neck pain=
A. Spondylitis.
Stn 1=
Abdomen: Young female 15 yrs has mid line
scar and lt subcostal scar and lt iliac fossa scar.
No AV Fistula. BEneath lt iliac fossa scar mass
with spleenomegally.
Diagnosis all over =poly cystic kidney disease ,
findings are transplantation of lt kidney with
cyst in spleen and liver.Hepatectomy due to
cyst in liver.
Respiratory:
Clubbing ,COPD and Pulmonary fibrosis . D/D =
Bronchiectasis.
Pls pray for me
Dr. MOHAMMAD SAZZAD HAIDER, Rustaq.

Egypt exam today


10 - 10 - 2016
Station 5
Pemphegus
gout
History
DM with lack of awareness
Communication
Steven Jonson
Neuro
spastic parapresis

Egypt exam today


10 - 10 - 2016
Station 5
Pemphegus
gout
History
DM with lack of awareness
Communication
Steven Jonson
Neuro
spastic parapresis
Dubai 10/10/2016
St 4 polycystic kidney bbn concern about job
and her kids.
History: uncontrolled asthma after yrs of
control, new factors was pet at home and
propranolol for anxiety
Neuro: upper limb examination in ESRD pt,
there was wasting of thenar group.
Cardio: aortic stenosis probably aortic sclerosis
Abd: hepatosplenomegaly and i missed lymph
nodes, there was hickman line in place probably
lymphoma
Chest: was very difficult very old man
uncooperative. Obstructive changes with
depressed lt side. Probably copd with lt fibrosis.
St 5, 1 recurrence of grave's in a young man
St5, 2 fever and sweating with artificial valve

Oman on 9/8/2016 i started with stn: 2


Headache which make him suffering in his job
and taking co- co- codamol not improving. He
feel fine when rest in a dark room.
Stn: 3= Collapsing pulse and Predominant AR
AND mild AS.
Stn=4
Pt is a staff nurse admitted with Rt leg and arm
flaccid paraparesis ; CT scan normal and
neurologist consultant evaluated and advice for
MRI Brain which is also normal .
Counselling pt about the disease and further
treatment.
Stn 5=
Bcc 1= Headache with visual disturbance.
Acromegaly.
BBC 2 =
Pt has heart valve problem gp refer for echo.
And he complaints of back pain and neck pain=
A. Spondylitis.
Stn 1=
Abdomen: Young female 15 yrs has mid line
scar and lt subcostal scar and lt iliac fossa scar.
No AV Fistula. BEneath lt iliac fossa scar mass
with spleenomegally.
Diagnosis all over =poly cystic kidney disease ,
findings are transplantation of lt kidney with
cyst in spleen and liver.Hepatectomy due to
cyst in liver.
Respiratory:
Clubbing ,COPD and Pulmonary fibrosis . D/D =
Bronchiectasis.
Pls pray for me
Dr. MOHAMMAD SAZZAD HAIDER, Rustaq.
OMAN 9- 10-2016
Another experience
st 5
Acromegly
heart valve problem & complaints of back pain
and neck pain
A. Spondylitis
comm
functional weakness physiotherapy nurse
Abd
Heptomegly
Scar mid line
Abd thalassemia
Haemochromatosis
Cvs
AR
PLUS
AS/mr
Chest lt side fibrosis
rt side consolidation /bronchiecta
Stat 2 history
headache migraine aggravated by analgesic
over use
Neuron proximal weakness ,cerebeller
DD
Exam cases of my frnd uk
Station 4: explaining and bbn of POLYCYTIC
kidney disease young lady . Concern was about
work .
Station 2 : Young pt with collapse histry of
sudden uncle death . HOCM : brugada
Station 5 : CKD WITH knee Joint pain 1 day ..
Gout/ pseudogout
Bcc : young pt with feet pain. ANKYLOSING
Chest: pulmonary fibrosis
Cvs : AS
NEURO : spastic Parapersis with dorsal column
Abdo: renal transplant

Exam in Egypt today


9- 10- 2016
abdomen
hepatomegaly and splenectomy and
lymphadenopathy pallor and jaundice,
chest
COPD with bibasal fibrosis
history
left sided weakness in young female on OCP
recurrent attaks of headache DD hemiplegic
migraine and sinus thrombosis and TIA
neuro
spastic paraparesis without sensory loss MS
cardio
mitral valve replacement and AF and AR
commun
Hodjkin lymphoma for Hickman line and
concern about infertility
st 5
psoriasis and back pain
hypothyroidism

Oman 8-10-2016
St1:
Chest: young pt. With multiple scars in his
abdomen and one small scar in rt.lower lobe +
rt.lower lobe dullness + cerps
DD
Abd: middle aged man ..multiple scars in
abdomen in lt.iliac and rt and lt.iliac mass
St 2 : 40yrs ...dm +htn + parathyroid
ectmy+smoker c/o: palpitations
Examiner ask for issues in this hx
DD for htn
Investigations
St:3
Neuro: middle age male
Catheterize
Both l.l weakness
Hypotonia
Hyporeflexia
Sensory level at t4
Q:
DD
Investigations
Cvs: midsternotomy scar
AVR
Some candidates say both MVR +AVR
Examiner ask what is cause of s.o.b
Station 4
CKD come with urosepsis given gentamycin
+amoxicillin develop exacerbation of renal
function and they didn't do measurement for
gentamicin level for 3days
Now pt.not need the RRT..gentamycin is
stopped ..ivf started
Station 5
Bcc1
Ant.neck swelling
Bcc2
Dm with deterioration..
Neck swelling not clear
In hx surrogate say hand shaking and prefer
cold
O/e
No tremor
OMAN ,,, Muscat
08/10/2016
St 01
Res: ILD with obvious clubbing
Abd:
Young boy with l/s Polycyclic kidney
St2
35yrs old lady with left sided weakness of the
body lasted for one hour. Only positive thing in
the history was taking ocp and headache with
the onset of symptom
St3
Cvs
MVR WITH Recent pacemaker insertion (pt
tachycardic)
Cns
45yrs old man with difficulty in walking.
Proximal weakness more than distal.
Plantar down going.reflexes are very sluggish.
Sensation intact. No cerebellar sign.as pt unable
to walk could not check gait......myopathy...
St4
52yrs old lady known case of AF on warfarin
investigated for anaemia .colonoscopy bx
revealed ca.no distal metastasis.to break the
bad news.
Bcc1
Young pt with loss of vision at night.
Retinitis pigment Osama
Bcc2
Pt with numbness of the both feet
Diabetic peripheral neuropathy with charcot
joint

Oman exam 8/10/16


St1 -transplanted kidney
-copd with bibasal lung fibrosis
Station 2 male 42 post MI 6 weeks dizziness and
epigadtric pain
Sat 3 AVR, Motor nruropathy
Sat 4 diabetic since 32 years with micro and
macrovascular complications he lost his
glycemic awareness
Sat 5 systemic sclerosis with skin rash no any
signs look MCT
Had recurrent hyloglycamic attack and heart
attack
Eye pain and double vision graves
opthalmopathy

EGYPT
Cairo
first courasel:history:hypothyroid patient with
history of valve replacement complain of
tiredeness.she is on warfarin,simvastatin and
thyroxine,on asking she has bleeding per
rectum mostly piles,DD warfarin induced pr
bleeding........communication Multiple Sclerosis
new diagnosed

Egypt Cairo
8_10_2016
History:young lady,prosthetic valve on warfarin
also hypothyroid on replacement presented with
s.o.b and anaemia
Communication: multiple sclerosis (breaking
bad news)
Cardio
Double aortic with MR
Abd
Massive spleenomegaly
Neuro
MS

EGYPT == cairo
8-10-2016 == first courasel:
History:hypothyroid patient with history of valve
replacement complain of tiredeness.she is on
warfarin,simvastatin and thyroxine,on asking
she has bleeding per rectum mostly piles,DD
warfarin induced pr bleeding........
communication Multiple Sclerosis new
diagnosed ( BBN)
Latest exam experience from UK (Courtesy of Dr
Sheraz)
PACES EXP 06.10.2016
queen Elizabeth hospital glasgow
I entered thru station 5..
55yr lady..Turner syndrome, hx of recurrent UTI
n Ear infections, never had daignosis before,
physical findings of turner were short stature,
low hair line, shield chest, short stature, squint
Concenred abt future prospects
Discussion abt DD, what can be done now?
50yr old lady vitiligo..presnets with fatigue
Had to rule out all assoctaions on history, when
asked had postural drop , BP at presentation
was 95/65
Dx Addison disease
Discussion abt DD, Inv, Mx
Station1
Abdomen renal transplant secondary polycystic
kidneys, previous fistula scar on left radius
Resp Copd superimposed LRTI with
parapneumonic effusion left sided
Examiners were not happy
Station 2 was Odd..confusion for 2 to 3 hrs..only
prssenting complaint, previously diabetic..but
everything was normal..no presyncope or
syncope..was just confused for long 2 to 3 hrs
and then revived on its own..no neurological or
cardiac symptoms or association with
posture..gave diff of
TIA/stroke/seizure/cardiogenic..
Station3
CVS midline sternotomy scar..metallic AVR with
ESM but pulse was waterhammer..presenting
comp was palpitations, reasons?
CNS classic diabetic peripheral
neuropathy..with big toe amputated and
neuropathic ulcer, Discussion on Dx DD Ix Mx
Station 4..
Newly diagnosed hodgkin
lymphoma..hematologist asked for
chemotherapy..wanted to discuss
Issues..fertility, employment, hicline, why me ?
How to tell wife...
8/10 Muscat
Station 5 c/o difficulty swallowing systemic
sclerosis
Thyroid eye disease with no other manifestation
of hyperthyroidism... Rt lobe is multinodular
howcome it should be grave's
Station 4 c/o dizziness on standing up and
melena.. had mi 6 weeks ago
Forgot to ask about acei
Station 2 type 1 dm with no awareness of
hypoglycemia. Who does not want to change
his insulin regimn
Chest lobectomy bronchiectasis
I hope it's not pneumonectomy
The trachia shifted to rt but there are signs of
fibrosis upper rt also
Neuro
Proximal myopathy areflexia adductor more
weather than abductor.. on hand shake lefts his
arm
Coordination could not be assessed due to
weakness
No sensory affection
Plus umnl in the form of spasticity
Cardio
Old patient double valve replacement
Young patient mitral valve replacement
Oman 7/10/2016
st 1:abd renal transplant
other bronchiectasis with lobectomy
st 2: young male have diarrhea and abd pain
more than 16 year with strong family Hx of ca
colon he concern as his father diagnosed
recently
st:3 mid sternotomy scar with metallic sound
low volume irregular pulse with pulsating neck ,
there is pulmonary hypertension and lower limb
and sacral edema for me it is aortic valve
replacement for some candidate mitral valve
discussion a very one as he said
neuro:young male paraplegic with one limb
spastic with positive clones other limb hyponia
with down going planter absent ankle reflex and
there is sensory level to T4 examiner Indian
aggressive
st4:young female in medical coll. diagnosed
with MS Already inform about disease but she
confused about it with some concern she ask
tooooo much examiner female aggressive with
English examiner only observing
st5: acromegaly ,,
obstructive sleep apnea
UK (FRESH) Experience exam
1/10/2016 == FRESH
Resp right lobectomy scar most likely
secondary to lung ca. Pt was middle age.i forgot
to palate tracheal position
Abdo renal transplant Pt. Did everything ok.pt
was in constant pain.examiner said Pt has
hyperthyroidism do you find any sign.i can say
Pt had exophrhalmos and she was a bit
overweight.i may pass or fail
Neuro Pt had charcoal marine tooth ds
I did examination ok gave diabetic neuropathy
diagnosis .I
Did not get time to ask him to walk when
examiner gave me clue about high arch foot I
said charcoal marie tooth ds and sorry I should
have ask him to walk so again not pass
CVS central and left leg scar so defiantly had
CABG otherwise very difficult finding I
presented as AS but don't think they were
happy
History taking _ Pt with collapse I think it was ok
from my side
Communication_ father had COPD got chest
infection not recovering again I think it was not
that difficult and I feel at least I did ok to get
pass rest depends on them.
So overall fail
But I will continue this because I really worked
very hard and did nit get chance 4 course
But incase pass then pure luck so less chance
Good luck to you.
Latest history case (Courtesy of Dr Hassan
Abuali )
6/10/2016
Oman
Hx
30 yrs female came complains of fatigue and
having normocytic normochromic anaemia
Sle
Antiphospholipid syndrome
Has hx of dvt and miscarriage
Case encounter before in Egypt 6/15

UK Experience exam
3/10/2016
Station 1 resp:pulmomary fibrosis secondary to
RA
abdo:failing renal transplant with lots of abdo
scars-no idea what they were all for.
Station 2 irritable bowel syndrome in a
demanding patient who wants scans etc
station 3 cardio-aortic regurg with collapsing
pulse in a patient with marfans ,
neuro-no idea-absent reflexes in upper limb with
not much other signs except for mild weakness
of some muscle groups....tough one.
Station 4 uhnappy relative blaming the system
for delayed diagnosis,
staton 5; second epileptic fit and
pcp pneumonia in a hiv patient (im guessing)..

James cook university Hospital UK.


23/8/2016
I started with stn. 5 and my first case was vision
problems. A young lady having visual problem
started suddenly few days ago. I asked to tell
me the story in her own words. She told me that
she is having vision problems at the end of the
day mainly, unable to read. No headache,
vomiting, limb weakness. I exclude. She was a
diabetic and on insulin since last 16 yrs. I saw
her drug list. Then started examing also. She
can't see in her rt. Eye. I tested lt. Eye
movement and field of vision. Then I did
fundoscopy for the rt. Eye. I couldn't see fundus
what I saw diffuse redness all over the visible
part of retina . Pupils were not dilated. I couldn't
see any retinal vessels. So I became a bit
confused about the findings , time finished and
examiner asked me what is your diagnosis? I
told, this lady with long standing diabetes and
sudden onset blurred vision might be having
some diabetic complication . He asked me what
did you saw in fundoscopy. I told there is
diffuse retinal hge. He asked what is your
diagnosis? I was wondering and time finished. I
explained to her that I will refer you to eye
doctor for further evaluation and management.
Then I started 2nd case in Stn 5.
My 2nd case in Stn. 5 pain in one eye. It was
excruciating pain behind the lt. Eye. Several
attacks before. Stays 4 to 5 hrs.
No vomiting or other alarm symptoms. There
was watering from that eye. I took all the history
and examine optic NV, exclude Trigeminal
neuralgia. I diagnosed cluster headache.
Address his concern that this is not brain
tumor. Examiner asked me what treatment. I told
analgesic to Nsaids to sumatriptan. Time
finished . I told it's clinical diagnosis so I will
not advise CT. He accepted.
Then I started chest. Middle aged male with
SOB. There was a chest drain on lt. Axilla. Lt.
upper chest expansion, movement was reduced.
Breath sounds diminished to absent on lt.
Upper chest. Vocal resonance was also
diminished. Examiner asked me what is your
diagnosis. I told lt. Sided lobectomy with plural
effusion. He asked me why you are saying
lobectomy. I told there is flattening and
depression on lt. Upper chest. He asked me did
you see the scar. I told no. Then he showed me
very faint scar on lt. Infra scapular region. Now
he asked me as this is very faint scar so
lobectomy is done long ago, and then why the
drain now. I told he might have CA Lung for that
lobectomy was done before and now again it
might have recurred with pleural effusion. He
asked me this drain is temporary or permanent.
I told temporary. He asked me it's lobectomy or
pneumonectomy. I said it is lobectomy because
the drain was high up in the lt. Axilla. Time
finished.
Abdomen :- Elderly male. With full flanks. Large
rt. Iliac fossa scar. There was rt arm AV fistula. I
could not feel thrill but as I saw fresh puncture
mark so I put my stethoscope on the fistula and
I heard the brui so I am sure now that it was
functioning fistula. I could palpate lt. Sided
enlarged kidney
No shifting dullness or hepato-spleno mealy. I
couldn't appreciate clear lump on RIF. I find
some scar on lt. Infra clavicular area. I present
the case as failed transplant with HD. He asked
me about the masses I palpated . I told lt.
hydronephrosis and right illiac fossa renal
transplant. He asked me, do you think this scar
is on RIF only or. .. I told lt is a large scar
extends from RIF to touch the flank. I wanted to
see gum hypertrophy but he had artificial
dentures. Examiner asked me what is the
etiology here. I told hydronephrosis,
glomerulonephritis, DM, HTN. Time finished
Station 3:- Cardiology, elderly male with SOB.
There was low volume regular pulse HR- 60bpm.
There was pansystolic murmur in the apex with
radiation to the axilla . There was another
ejection systolic murmur in aortic area with
upward radiation. Normal 1st heart sound and
soft 2nd heart sound. I presented the case as
double valves pathology MR and AS. Examiner
asked me what is the etiology here. I told
degenerative, as in old age. But might be
rheumatic also. Asked me investigations, I told
echo. She asked me, you told you would like to
finish examination by doing urine dips tick.
What is your your purpose of doing that. I told
by that I can exclude endocarditis. Time finished
Neurology :- middle aged male with walking
difficulties. I started with gait, it was high
stepping gait . Both legs were wasted and more
on rt. lower leg. There was scar on rt. foot. Tone
was normal, reflexes were diminished to absent,
because I saw some muscle flickering on knee
reflex. Planters flexor. There was pest cavus. My
diagnosis was freidreick's ataxia, examiner
asked me what other possibilities, I told HSMN
because there was loss of vibration sense also.
He asked me how you will investigate the case. I
told Nerve conduction studies. Time finished.
Due to time constraints I couldn't see the back
and I forgot to do co-ordination . Overall
examiner was satisfied as l felt.
Station 4, The story was one 55 yr. Old female
who was admitted to the hospital 6wks ago with
bronchial asthma and she was discharged with
PEFR of 90 -100% of predicted. She came today
in follow up clinic but there was a chest X-ray
during her last adm. 6wks ago which revealed 2
opacities and it was not written in discharge
summary nor any body informed her about that
report. Though it was not certain about the
report whether it was recurrence of her breast
cancer which she had 6 yrs ago and for that she
underwent mastectomy and chemotherapy. It
was cured and she has been following up in
cancer clinic. They told she is fine. Today
another X-ray done which shows the same
uncertain shadows 2 in number. You have to
discuss the matter to that lady . So I started by
introducing myself and go ahead with the matter
as Dr. Zein Taught us. I apologized repeatedly
for not informing her about the previous X-ray
report. I showed empathy when she told that her
another sister died because of recurrence of
breast cancer. I told her about putting her in
priority for CT scan and refer her to chest
specialist. I mentioned about PALS she can put
her complaints. I told I'll discuss the matter with
my consultant to invest the matter of
communication gap that it might not happened
again. I advised about smoking cessation clinic.
I asked about social support and family support
and asked how she will go back home. Offer
support to drop her home if she is hesitate to
drive today. Examiner asked me what the theme
here. It was uncertainty. He asked me why I
didn't tell her today 's X-ray report. I told she is
already upset and as there are is no charge in
shadow so I didn't want to give her extra mental
stress. Before that examiner asked me what
ethical issues involved here. I told Autonomy.
She had the right to know her X-ray report. Then
he asked me that why I didn't disclose today 's
report, which I answered already. Time finished.
History stn. It was an young lady 25 yrs. Old got
some blurred vision sudden onset at the time of
coming back home from a party with her friend
and she was driving at that moment. She had
several same attacks before since last 6
months. This time her friend was witness of the
attack. She became unconscious for few
minutes and she had few low grade jerky
movement of the hands and arms. No headache,
vomiting, no tonic clonic shakes of the body or
limbs happened. No fever, neck rigidity or any
skin rash or purpura, was there. Giant cell
arteritis excluded. As there was history of
tongue bite so I took details history to rule out
epilepsy. No history of clothes wetting was
there. She wasn't on any regular medication. No
significant past medical history except the
recurrent similar attacks. At this episode BP
was 96/50 and pulse was 56 per minute. She had
that black out on the wheel and her friend any
how stop the car and take her out of the car and
took her to the hospital. Her alcohol intake was
in excess of the recommended limits. She used
to drink more than 20 units of alcohol. Not
smoking much. I advised about smoking
cessation clinic and also the alcohol cessation
clinic. She had a family history of premature
death. Her brother died suddenly at the age of
35 yrs. So in her case I discussed to exclude
arrhythmia also, including investigation for
arrhythmia. I checked understanding and advise
investigation. Examiner asked DD. I told
Epilepsy, arrhythmia, hypoglycaemia(blood
sugar level was 4.3 ), vasovagal syncope. Then
he asked me tell me one bedside test to confirm
the diagnosis. I told tilt table test, he said no. I
told Holter monitor, still he said no. Then he told
me BP, and then I said yes, standing and supine
BP measurement. Then time finished. Pray for
me and I wish you all the best to those who are
going for exam.

UK experience
My experience at whipps cross hospital
31/8/2016.
Started with station 5
1.young female referred from surgical
department due to recurrent abdominal pain.
History was negative, no diarrhoea, no loss of
weight.
No relieving or aggravating factors.
Systemic review showed rash at forearm, mild
headache and some joints pain. No weight loss
Periods normal
Examination; no jaundice, abdomen soft
nontender and no viscromagely
Concerns;
1.what is the cause
2.why ultrasound normal.
I explained likely vasculitis or porphyria.
Needs other blood and urine test to confirm .
Examiner asked about differential i said as
above and the next question was investigation
of porphyria
2.25 years old university student with collapse. I
started what happened he told he passed out
while watching movie.
I ask if happen before, Pt told 3 weeks ago while
he was working on computer in library. I started
with prodormal symptoms, they were none.i ask
any friends observed jerky movements, Pt told
yes.
Than history goes on with incontinence and
fatigue after recovery.
I ask about any thing unusual a night before
(lack of sleep ),Pt told no . then asked about
driving, drugs, and hobbies (keen
swimmer).grossly examine tone power in both
limbs,gait and ask for fundus. (Examiner
refuse).
Concerns 1.what is my problem
2.what you will do (scan +eeg).
Consouil about driving and any attendant while
he swims.
Examiner ask! What will be finding in ct? I told
him likely to be normal as there is no
neurological deficit but would like to have com
Complete neurological examination.
Is it possible to have any cardiac problem to
this patient.
I explained possible but less likely as both
events occur while Pt was sitting, however
tacyarrthmias can be possible.
Would you start treatment. I said refereed to
seizure clinic and neurologists will decide
Abdomen# young female with central larotomy
scar,subclavian
Dialysis catheter and right palpable kidney. Not
sure about larotomy scar (which was the main
question by examiner),other question was about
causes of fatigue in this patient ??I told him
uremia, possibility of underlying
hypothyroidism, anemia and infection. Overall
not very good
Respiratory # young female, no rheumatological
manifestations, wheezing from bedside. Minimal
basal crepetations.
Indian examiner started with respiratory rate
(forget??)
Next question was jvp findings (??),followed by
did this patient had loud P2 (??).
I said sorry for above 3 questions
Than he ask differential i told him copd
/fibrosis.
He ask which will be your priority diagnosis, I
told copd due to prominent wheezing than
investigation of copd with xray findings and
pulmonary function test. Overall it was tough
History ##50 years old women complaints of
abdominal discomfort and bloating.
I started with usual pattern of pain,location,
bowel changes, all none. Nonspecific pain not
related to any thing . half stone weight loss.
Than I asked any tummy distension, she said
yes her trouser are tighter and she is using
large size from before. I switch to orthopnea,
pnd, negative. No lower leg swelling no
periorbital swelling no problems with water. No
signs of liver disease. Clueless I proceed to past
history which was significant for mastectomy
secondary to malignancy. Family history
positive for ca breast in sister . mild low feeling
due to recent mother died because of ca
breast.post menopausal (no dysparunia/break
through bleeding).
Concern 1. What is cause of tummy distension. I
explained likely that some tumour cell spread .2.
Is it too late as I have symptoms since 3
months. I told her we have to investigate and
don't worry we will do your test on priority
Examiner ask# diagnosis i told him metastasis.
He ask if Pt don't have distension than what do
you think. I told I consider irritable bowel as
recent death of her mother and only half stone
of weight loss.
What other possibilty I told ca ovaries. Then
tumour markers of ca ovary. What do you do?
Scan ct . any investigation would you like to
offer while she was in opd. I don't have any
answer. He told chest xray.
What measures you told to other sisters and
daughters. I told repeated manual breast
examination and after 40 years of age
mammogram. Got full marks
Cvs # 75 years old male with sob . murmer of
AR. I checked collapsing pulse.
Routine questions about causes.
Causes of acute AR (dissection of aorta,
endocarditis and ruptured sinus of valsulva)
Type of valve
Cns# 50 years old gentle man with difficulty in
walking please examine upper limbs ??
It was parkinsonism. I mentioned to check
sitting and standing BP, micographia and gaze
palsy
Examiner ask about causes.
Treatment
New treatment, mention deep brain stimulation
and dopamine containg implants.
Who will be involve in management of this
patient #MDT.
She asks what occupational therapist will do??
I told occupational therapist will visit the home
and arrange some rails and support to prevent
patient from falling.
Alhamdillah went well. Got 19 in both
Communication # spoke to wife, husband in icu.
Keen cycle rider and went for long marathon
and take extra fluids to prevent dehydration. At
home he also drink water continously till he was
found to seize in garden and brought by
neighbours. CT and all other labs normal.
Sodium 114.
Better but still confused with gcs 15 . two weeks
ago started on bendrafluthiazide for htn (Pt age
45)
I started with wife with sympathy, what she
Knows so far regarding husband.
Gave good news that scan is normal. Likely
seizure due to low salt in body.
She asks why salt become low. I explained . she
asks why still confused I told her take time to
correct sodium slowly . she asks about
discharge.,explains it will take coupleof days.
She asked they are moving to dubai, so he can
do cycle ride there. Its will happen again
????like little puzzle with this question but told
her that chances are low but instead of taking
plane water if he took carbonated water it
contains some salt!!.
She asked about BP medication attributing. I
told possible. She asks continue
bendrafluthiazide. I told we ask cardiology
colleges.
Came back to driving and profession .Pt was
enginer but not exposed to heavy machine. I
told dvla.
She asks follow up for how long as they are
moving to dubai. I told we don't need long term
follow up as prognosis is good and we're will
gave detail medical report to be shown to
doctors in dubai.
Last concern where he will ride cycle in dubai
as it is very hot there ????
I just mention i am not sure But in dubai you
may find indoor cycling track as most of the
activities there are indoor even ski
Examiner ask why Pt confused I told still
sodium is not correct. He ask other reason I told
him possible cerebral edema due to seizure and
low sodium.
He ask at what rate you will correct sodium. I
told 5 -8 meq/day . then he ask what happens
with rapid correction. I answered. He ask at
what sodium level you are happy to discharge. I
told him 135 -140. He ask what about cycle
riding rules after seizures in uk.
I told him I have no idea, but advisable not to do
in early few months . last question is
bendrafluthiazide was a good choice of anti
hypertension for this patient. I told no as patient
ids less than 55 an ACEI should be considered.
Alhamdillah Got full marks
Overall experience of exam in uk was good .
there is no problem of understanding of English
with surrogate in station 2 and 4.

Chennai 1st day 3rd cycle


BCC... 1. persistent htn with knee pain. 2.
Frequent headache within 2-3mths and impact
on job.
History.... 25yr old lady with hypertension and
URE shows RBC and protein.
Communication.... 25 yr old lady come to
yesterday ED with haemoptysis and fever and
done CXR show bilateral apical fibrocavitatory
lession and sputum show lots of AFB positive
bacilli. Pt discharge from hospital without the
result. ED ph her to come to hospital for result
and pt is reluctant to come to hosp but today
come to hosp.
Task... explain the risk to the pt herself and
others and advice to protect of spred of
infection to others.
CVS... restenosis MS with AF. Complaint... SOB
CNS... Hemiplegia, only examine LL.
Complaint... difficulty in walking.
RESP.... complaint... SOB. lt upper lobe
fibrocavitatory lession and lt lower lobe pleural
effusion.
Abd.... complaint... abd discomfort. Lt arm AV
fistula functioning and recent puncture mark
present with hepatomegaly.
That's all. Good luck to all.

18.3.2016 last round Chennai


Station 5 loose motion for 3 months
Bilateral knee pain
Station 1 Respiration COPD Bronchiectasis
Abdomen. APKD
Station 2. Headache with menorrhagia
Station 3 CVS MVR
CNS Facial palsy
Station 4 Type 1 DM with proteinuria
Poor drug compliance
I have passed my PACES exam in Mandalay
center recently.Thanks to Dr Bebo Bebo and
other friends in this group for sharing
invaluable experiences.I w'd like to share my
experiences.
1.Abdomen.heptosplenomegaly w
anemia.Q.finding,dx,ddx,mx.14/20
Resp.moderate pleural effusion.w tracheal shift-
Q.finding,dx,ddx,mx.20/20
2.History.
unilateral Headache.in female 30 yr.not relieved
by simple analgesics,pizotifen and sumatryptan.
Pt have used OC pill for 6 mth then GP asked to
stop.not related to OC pills and not improved by
stopping it.no features of
migraine.cluster.increased ICP.stress present at
work and related to HA.
I said tension HA and migraine as DDx.
Q.how to invest.to differentiate.I said clinically
and by response to drugs.
Q.how to manage.I said I want to do full
neuro.exam and trial of other analgesics like
ibuprofen,diclo. and reduce stress and follow up
for new symptoms.
Q.how to reduce stress .l said biofeedback and
CBT.
It is not fit to typical history of any paticular HA
and I think examiners want discussions about
possible ddx.18/20
3.CVS.MS with valvotomy scar with AF
Q.finding,dx,ddx,mx.simple case 15/20
CNS.examine the lower limbs neurologically
flaccid paraparesis with indwelling catheter .
I examined tone.power.reflexes,planter.pinprick
and joint position sense and heel shin test in
time.Forgot and do not have time to examine the
spine.
DDx.cauda equina and peripheral neuropathies .
I said cauda equina and ddx are peripheral
neuropathies like lead poisoning,porphyria,DM.
Examiner asked about pattern of neurological
deficit in each d.dx,then mx.I said CT or MRI
spine,bowel and bladder care..treatment of
underlying cause.20/20
Station4-medical error.
pt with psoriatic arthropathy taking
methotrexate was given trimethoprim for a UTI.
pt was admitted for nosebleed with
pancytopenia.
I apologize very early after taking rapport and
checking pt's prior knowledge about her
condition,I said we shouldd't have given that
combination as it have led to serious damage to
you.Surrogate show only little anger and with
repeated apology ,she accepted.Ask if she can
conplain,I said yes and explain I will help her to
write conplaint to PALS.
Concern.if she can get recovery and when can
she restart methotrexate or not.I said it depends
on recovery of her blood cells and I will ask my
consultant and if necessary will get opinion of
joint specialists.when can she go home.? It
depends on her codition and I will let her know
after checking her recovery.Then I summerized
and checked pt's understanding and said thank
you.We finished early and we have to sit in
silence for 5 mins!
Examiner warned me to say something to pt but
we have not much to say at that time.
What ethical issues,?I said truth telling about
our mistake,.non.maleficience, beneficience
14/16
My BCC cases are interesting and I got dx only
in last 2 minutes somewhat luckily!
BCC 1.a 25 yr old man with repeated blood
transfusions since 5 yrs of age ,presented with
fever.,high colour urine ,tiredness
Examination show moderate splenomegaly and
pallor.
Pt's concern.what is his problem?I said
thalassemia intermedia.Why he has fever?
l said UTI or malaria or other sort of infection
and I will do blood tests.How can you help me to
reduce transfusion interval,?
I said you have a big spleen ..that is why it
destruct your blood cells and U need blood
transfusion.You need operation to remove
spleen to reduce transfusion interval.
Examiner ask finding .dx.Why he has fever.?I
said UTI or other infection.not satisfied.Why
fever in this pt with splenomegaly,.?
I thought long way and said he may have
hemochromatosis leading to diabetes leading to
immune suppression and infection.Any other
pissibility.? I said hypersplenism leading to
pancytopenia leading to infection.Examiner was
very happy to hear it.How to mx,,,? I said
neutropenic regime.not satisfied.What is
definitive mx,?I said
splenectomy.Examiner.happy!
what will u do before splenectomy.?
I said vaccination.For what? for encapsulated
bacteria.Time was up.
24/28
BCC.2.50yr old smoker present with cough for 2
weeks not responding to 2 courses of
antibiotics.pt said cough worse on lying down
but no other symptoms.I ask other chest and
CVS symptoms and did chest examination and
found no abnormality.Pt asked what is his
problem and I didn't know dx!
I replied it will be chest infection or heart
problem and I can't tell exactly at this stage and
I will do some blood tests and imaging of chest
.Is it serious?is it cancer? I said he has no sign
of cancer at this stage although it is still a
possibility as he smoked heavily.I will do tests
to make sure that everything is OK.Then.I
thought that this pt must have some signs to be
in exam and it appeared in my mind that he had
a hyperresonant percussion and reduced BS. I
quickly said to the pt that he has a condition
called COPD and I will give him inhalers and
some tablets.pt quickly asked is it related to
smoking and I said yes and advised to quit
smoking.Time left only 2 min for discussion.
Examiner asked my findings and accepted.Any
other sign that show other specific dx?I said
no.He accepted.As my dx is COPD any other
ddx?I said asthma but no wheezing and
rhonchi.Any other ddx?HF but no other CVS
symptoms.accepted.Any other dx for cough
worsen by lying down?
I said GERD and examiner was very happy to
hear.What advice will U give to pt?I said high
pillows and to avoid food at bed time.Time was
up. 23/28
There are 2 types of candidates.The first one is
very bright ,smart ,lucky and they can easily
pass exam after studying 2 to 3 months.The
second type is majority of candidates and they
have to work very hard and take a year or more
of studying time to pass.I am the second type
and have to study a long time waiting to get a
seat in Myanmar for about 2 years.This is my
first attempt.
Exam luck is also an important factor.
Then,can we do anything to improve our exam
luck?
As for me, yes.
I shared my knowledge to others and shared
some books and mp3 podcasts in this group by
my another account.I had also helped other
candidates with their study and practice so that
my exam luck can be good.I have met with good
natured examiners!
In the exam, some candidates said they have
time only to discuss ddx.They will lose marks
for judgements.
As for me, I have my own note of common
causes,inv,mx and I memorized them so I can
discuss fluently in 2 to 4 minutes of discussion
time and I reached to management in every
station and passed every station.
Best of luck to all future candidates!

My PACES experience in Golden Jubilee


Hospital, Glasgow, UK in June 2016
Station 1:
Respi: A elderly man with obvious pectus
excavatum. However, the chest signs were
subtle. I got left LZ crepitations with reduced
breath sounds, giving the diagnosis of pectus
excavatum with left LZ bronchiectasis. Another
candidate got right LZ crepitations, the 3rd
candidate got bilateral LZ crepitations. Turned
out the answer was right LZ bronchiectasis.
Lost all marks in physical signs component.
(12/20)
Abdomen: Another station with subtle clinical
findings. Stem: this man has abdominal pain;
please examine and find out why. This middle
aged man has very subtle hepatomegaly.
Discussion on causes and management.
Another candidate reported
hepatosplenomegaly, and the 3rd candidate
reported normal findings. The answer was
hepatomegaly, but I missed the gynecomastia,
so identifying physical signs marks were
deducted. Gave the correct DDX of alcoholic
liver disease. (18/20)
Station 2:
A middle aged lady with prolonged fever,
symptoms persisted despite admission and
treatment for UTI. Further hx revealed prolonged
fever with weight loss. She will also mention a
lump in the inguinal area. DDX given was
lymphoma, occult malignancy, CTD, TB, IE.
Concern: What is causing my symptoms? Spent
a lot of time explaining diagnosis, the need for
biopsy, admissions, further tests. Need to
explore how the fever has affected her daily life
and offer solutions.(19/20)
Station 3:
Neurology: Stem: this lady complained of
double vision. Please examine her. A case of
Myasthenia gravis with thymectomy. The only
sign was double vision with fatiguability and
thymectomy scar. Questions were standard.
(20/20)
CVS: An elderly man with central sternotomy
scar, vein harvest scar, and MR. Got panicked
and gave the wrong diagnosis of AS. Did badly
overall. (10/20)
Station 4:
A elderly man was admitted for pneumonia with
confusion. Given amoxicillin in ward and
developed anapylaxis. He recovered but still
remained confused. Talk to the daughter and
address her concern. Need to elicit the fact that
the daughter mentioned to a doctor regarding
patient's allergy to penicillin. Thus, this is a
case of error of drug administration. Need to
apologize profusely. Lodge critical incident
reporting. Need to address her concern and
reassure her in every way this will not happen
again, and provide her the example how you
intend to avoid this from happening again. She
will have a lot of concerns and anger and you
need to apologize, reassure, offer solutions and
answers to her concern. I didnt mention about
PALS as she never mentioned lodging a
complaint but if she did, offer her ways to lodge
a complain.(16/16)
Station 5:
BCC1: A elderly lady with dark pigmentations
over her shins. Further hx: long standing DM on
OHA, long standing pigmentation for years, not
causing symptoms apart from itchiness. It is a
case of necrobiosis lipoidica diabeticorum
(most likely healed lesions). Given differentials
of chronic venous insufficiency with stasis
eczema, diabetic dermopathy. (28/28)
BCC2: A case of a young man with headache. A
challenging station as there is a lot to get from
history and to examine, and all need to be done
within 8 minutes. Further history revealed
symptoms of headache worse in morning and
with sneezing, vomiting and blurring of vision.
Examinations were normal. Didn't perform
fundoscopy but did mention it. Concern: Is it
brain tumor? My mom had brain tumor at age of
40. DDX: headache due to raised ICP, e.g. IIH,
less likely SOL, migraine. Mx: Offer urgent CT
brain. (25/28)
Overall: 148/172 (PASS)
Personal opinion:
Exam case in UK are generally fair. It has
tendency to put up cases with subtle clinical
findings esp. BCC. Normal surrogates are
frequently used in BCC, with scenarios like
headache, syncope, fever etc being not
uncommon.
The examiners were rather strict and particular
about identifying correct physical signs. This is
the component that scared me the most. This
applies to PACES everywhere and a lot of
practice is required to be able to pick up subtle
signs. Never create signs as this is really fatal.
Station 4 is very unpredictable. Cases can be
easy or complex with multiple agendas. Suggest
to review all the cases posted up here
previously and practise them. Need to have
some knowledge regarding DVLA, Mx of
meningococcal ds and prophylaxis etc... Need
to really elicit the concerns, and offer
solutions/answer as much as you can.
Good luck and all the best.
This group is very helpful, keep sharing cases
and experience guys.
7/ 2016
Station 1
Abdomen: Lady around 50y.o with cushingoid
features, Perma cath, scar on the Right iliac
fossa ( failed renal transplant) and multiple
scars around the umbilicus ( previous
Peritoneal dialysis)
The examiner asked about the complications (
esp. bone complications and he asked about
dietary restriction {Shappati} as the pt and
examiners are Indian)
Respiratory: Male pt around 55y.o well-
nourished with right thoracotomy scar on the
back+ end-insp crackles. No clubbing, no
cyanosis, no signs of pulmonary HTN
Dx ILD, the scar is for lung biopsy ( I said to the
examiner it's for lobectomy but he asked me
what else it could be for, I said for lung biopsy
then he agree with me)
Station 2
Lady aged 55y.o heavy smoker with 3months
h/o SOB, coughing blood and loss of weight.
She sought medical advice recently and given
antibiotic ( she doesn't know the name of it) by
GP who diagnosed her as acute bronchitis, but
no improvement. One week ago she developed
dysphagia for solid food. No h/o fever, no
vasculitis symptoms, no other GI symptoms.
Station3
Cardio: young lady with mid-sternotomy scar
and palmar erythema. No signs of pericarditis.
S1 is metalic. No murmurs or additional heart
sounds. No signs of pulm HTN or pulm
cngestion
Dx Mitral valve replacement ( metalic)
Neuro: instruction: examine lower limbs
old man with walking aids beside him,
indwelling Foley's cath. Perioheral neuropathy
for DD. I mensioned them specifically
paraneoplastic syndrome ( ? Prostatic cancer)
Station 4
Middle age lady diagnosed to have bird fancier
lung disease. She presented today to know the
result ( BBN) and to discuss with her the need
for corticosteroid treatment and to avoid
exposure to pigeon ( she's breeding pigeon and
she's famous in her region )
She resisted first to take the steroid but when I
explained to her its benefits and risks (
including osteoporosis) and the prophylaxis for
the side effects she accepted. Also she got
angry when I suggested to her to avoid
exposure to pigeon.. I appreciated her upset and
I explained that she will not get better unless
she avoids exposure. I suggested to wear mask
in case she has to see her pigeon or to train
somebody to feed them. She said her son may
help her in taking care of the pigeon finally
agreed.
Station5
Case 1
25 y.o. Lady presents with fever (39.5) and
diarrhea. She admitted eating from restaurant.
When I asked about travel she said she came
from Thailand. I asked about insect bite
including mosquitos she said yes. Then I asked
about malaria prophylaxis before during and
after travel she said yes. I also asked about HIV
risks.
O/E : no signs (surrogate)
Case 2
30 y.o male with headache, high blood pressure
(180/100) and urine dipstick showing proteinuria
and microscopic hematuria. He had h/o
childhood chest infection and family h/o SLE.
O/E no signs
There is ophthalmoscope on the table. I noticed
it late. I said " I would like to do fundoscopy but
no time :)"
Dx AkI ( Glomerulonephritis needed kidney
biopsy and Autoimmune profile+ Renal US)

Paces exam cases-july 2016,london


Station 5-35 year old lady with loss of consciousness
for few minutes,no warning particularly,sometimes
some tingling sensation of face.previuosly investigated
for arrhythmia.examination revealed,heart rate-
60/min,i think it was irregular ,but when I checked for
.15 sec ,it was regular.but examiners asked about it
questions-DD,expected to tell about the possible
. cardiac causes
Iinvestigations21/28
station5-2 weeks hx of cough ,phlegm and
+haemoptysis.smoking
-questions
o/e-no clubbing,calf-no swelling.no SOB,but examiner
stopped me halfway thru the chest examination and
told left basal crackles.DD-pul.embo
LRTI
TB
vasculitis
malignancy
.patient was concerned abt cancer
investigations inclTB,vasculitis.28/28
should ask patient concerns and address
-station one
abdomen-multiple scars including RIF scar,large mass
right hypochondrium ,possibly loin,ran out of time.i
was too slow and could not finish exan/.asked the
diagnosis-possibly polycystic kidney with enlarged liver
.due to liver cycst.i missed the kidneys due to time
didn't get time for further discussion.7/20

respiratory system-most probably pulmonary


.fibrosis,obvious clubbing
questions-DD,for clubbing and crackles
Investigations in detail such as what will you expect in
xray,CT in fibrosis
causes for bronchiectasis
20/20
.station2 history-diag cluster headache
questions-summary
diag and DDS
investigations
.management of cluster headache
prognosis
.patient concern-concerned abt brain cancer
I also asked about how did affect her work and life and
.all sort of concerns and possible effects
20/20
cardiology-CABG(midline sternotmy scar,venous
harvesting scar)
aortic stenosis,slow rising pulse was present,ankle
oedema +,no metallic click
questions-DDS
investigations
features of severe aortic stenosis.what were the
features in this patients, 20/20

.neuro-asymmetrical parinsoons features


dds or causes
investigations
management in detail,phrmacological,non
phrmacological
how does it affect people,what are the difficult
tasks20/20

communication-quite simplescenario,no hidden


issues.daughter is concerned about mothers discharge
after sever pneumonia as she is tired and lethargic and
lives alone.mother has got mental capacity and wanted
to go home.talked about autonomy,asessd by physio
and occ.explained fit to discharge,promised to arrange
another r meeting with physio,OT and possibly mother
with mothers permission,expected to offer some help
like community team visit or similar to make sure
things are ok.20/20
.hope its helpful
have passed my PACES exam in Mandalay center
recently.Thanks to Dr Bebo Bebo and other friends in
this group for sharing invaluable experiences.I w'd like
.to share my experiences

Abdomen.heptosplenomegaly w .1
anemia.Q.finding,dx,ddx,mx.14/20

Resp.moderate pleural effusion.w tracheal shift-


Q.finding,dx,ddx,mx.20/20

.History.2
unilateral Headache.in female 30 yr.not relieved by
.simple analgesics,pizotifen and sumatryptan
Pt have used OC pill for 6 mth then GP asked to
stop.not related to OC pills and not improved by
stopping it.no features of migraine.cluster.increased
.ICP.stress present at work and related to HA
.I said tension HA and migraine as DDx
Q.how to invest.to differentiate.I said clinically and by
.response to drugs
Q.how to manage.I said I want to do full neuro.exam
and trial of other analgesics like ibuprofen,diclo. and
.reduce stress and follow up for new symptoms
.Q.how to reduce stress .l said biofeedback and CBT
It is not fit to typical history of any paticular HA and I
think examiners want discussions about possible
ddx.18/20

CVS.MS with valvotomy scar with AF .3


Q.finding,dx,ddx,mx.simple case 15/20

CNS.examine the lower limbs neurologically


. flaccid paraparesis with indwelling catheter
I examined tone.power.reflexes,planter.pinprick and
joint position sense and heel shin test in time.Forgot
.and do not have time to examine the spine
. DDx.cauda equina and peripheral neuropathies
I said cauda equina and ddx are peripheral
.neuropathies like lead poisoning,porphyria,DM
Examiner asked about pattern of neurological deficit in
each d.dx,then mx.I said CT or MRI spine,bowel and
bladder care..treatment of underlying cause.20/20

.Station4-medical error
pt with psoriatic arthropathy taking methotrexate was
.given trimethoprim for a UTI
.pt was admitted for nosebleed with pancytopenia
I apologize very early after taking rapport and checking
pt's prior knowledge about her condition,I said we
shouldd't have given that combination as it have led to
serious damage to you.Surrogate show only little anger
and with repeated apology ,she accepted.Ask if she can
conplain,I said yes and explain I will help her to write
.conplaint to PALS
Concern.if she can get recovery and when can she
restart methotrexate or not.I said it depends on
recovery of her blood cells and I will ask my consultant
and if necessary will get opinion of joint
specialists.when can she go home.? It depends on her
codition and I will let her know after checking her
recovery.Then I summerized and checked pt's
understanding and said thank you.We finished early
!and we have to sit in silence for 5 mins
Examiner warned me to say something to pt but we
.have not much to say at that time
What ethical issues,?I said truth telling about our
mistake,.non.maleficience, beneficience 14/16

My BCC cases are interesting and I got dx only in last 2


!minutes somewhat luckily

BCC 1.a 25 yr old man with repeated blood


transfusions since 5 yrs of age ,presented with
fever.,high colour urine ,tiredness
.Examination show moderate splenomegaly and pallor
Pt's concern.what is his problem?I said thalassemia
?intermedia.Why he has fever
l said UTI or malaria or other sort of infection and I will
do blood tests.How can you help me to reduce
?transfusion interval
I said you have a big spleen ..that is why it destruct
your blood cells and U need blood transfusion.You
need operation to remove spleen to reduce transfusion
.interval
Examiner ask finding .dx.Why he has fever.?I said UTI
or other infection.not satisfied.Why fever in this pt
?.with splenomegaly
I thought long way and said he may have
hemochromatosis leading to diabetes leading to
immune suppression and infection.Any other
pissibility.? I said hypersplenism leading to
pancytopenia leading to infection.Examiner was very
happy to hear it.How to mx,,,? I said neutropenic
regime.not satisfied.What is definitive mx,?I said
!splenectomy.Examiner.happy
?.what will u do before splenectomy
I said vaccination.For what? for encapsulated
.bacteria.Time was up
28/24
BCC.2.50yr old smoker present with cough for 2 weeks
not responding to 2 courses of antibiotics.pt said cough
worse on lying down but no other symptoms.I ask
other chest and CVS symptoms and did chest
examination and found no abnormality.Pt asked what
!is his problem and I didn't know dx
I replied it will be chest infection or heart problem and
I can't tell exactly at this stage and I will do some blood
tests and imaging of chest .Is it serious?is it cancer? I
said he has no sign of cancer at this stage although it is
still a possibility as he smoked heavily.I will do tests to
make sure that everything is OK.Then.I thought that
this pt must have some signs to be in exam and it
appeared in my mind that he had a hyperresonant
percussion and reduced BS. I quickly said to the pt that
he has a condition called COPD and I will give him
inhalers and some tablets.pt quickly asked is it related
to smoking and I said yes and advised to quit
.smoking.Time left only 2 min for discussion
Examiner asked my findings and accepted.Any other
sign that show other specific dx?I said no.He
accepted.As my dx is COPD any other ddx?I said
asthma but no wheezing and rhonchi.Any other
ddx?HF but no other CVS symptoms.accepted.Any
?other dx for cough worsen by lying down
I said GERD and examiner was very happy to
hear.What advice will U give to pt?I said high pillows
and to avoid food at bed time.Time was up. 23/28
There are 2 types of candidates.The first one is very
bright ,smart ,lucky and they can easily pass exam after
studying 2 to 3 months.The second type is majority of
candidates and they have to work very hard and take a
year or more of studying time to pass.I am the second
type and have to study a long time waiting to get a seat
.in Myanmar for about 2 years.This is my first attempt

.Exam luck is also an important factor


?Then,can we do anything to improve our exam luck
.As for me, yes
I shared my knowledge to others and shared some
books and mp3 podcasts in this group by my another
account.I had also helped other candidates with their
study and practice so that my exam luck can be good.I
!have met with good natured examiners

In the exam, some candidates said they have time only


.to discuss ddx.They will lose marks for judgements
As for me, I have my own note of common
causes,inv,mx and I memorized them so I can discuss
fluently in 2 to 4 minutes of discussion time and I
reached to management in every station and passed
.every station
!Best of luck to all future candidates
My PACES experience in Golden Jubilee Hospital,
Glasgow, UK in June 2016

:Station 1
Respi: A elderly man with obvious pectus excavatum.
However, the chest signs were subtle. I got left LZ
crepitations with reduced breath sounds, giving the
diagnosis of pectus excavatum with left LZ
bronchiectasis. Another candidate got right LZ
crepitations, the 3rd candidate got bilateral LZ
crepitations. Turned out the answer was right LZ
bronchiectasis. Lost all marks in physical signs
)20/12( .component

Abdomen: Another station with subtle clinical findings.


Stem: this man has abdominal pain; please examine
and find out why. This middle aged man has very
subtle hepatomegaly. Discussion on causes and
management. Another candidate reported
hepatosplenomegaly, and the 3rd candidate reported
normal findings. The answer was hepatomegaly, but I
missed the gynecomastia, so identifying physical signs
marks were deducted. Gave the correct DDX of
)20/18( .alcoholic liver disease
:Station 2
A middle aged lady with prolonged fever, symptoms
persisted despite admission and treatment for UTI.
Further hx revealed prolonged fever with weight loss.
She will also mention a lump in the inguinal area. DDX
given was lymphoma, occult malignancy, CTD, TB, IE.
Concern: What is causing my symptoms? Spent a lot of
time explaining diagnosis, the need for biopsy,
admissions, further tests. Need to explore how the
fever has affected her daily life and offer
)20/19(.solutions

:Station 3
Neurology: Stem: this lady complained of double
vision. Please examine her. A case of Myasthenia gravis
with thymectomy. The only sign was double vision with
fatiguability and thymectomy scar. Questions were
)20/20( .standard

CVS: An elderly man with central sternotomy scar, vein


harvest scar, and MR. Got panicked and gave the
)20/10( .wrong diagnosis of AS. Did badly overall

:Station 4
A elderly man was admitted for pneumonia with
confusion. Given amoxicillin in ward and developed
anapylaxis. He recovered but still remained confused.
Talk to the daughter and address her concern. Need to
elicit the fact that the daughter mentioned to a doctor
regarding patient's allergy to penicillin. Thus, this is a
case of error of drug administration. Need to apologize
profusely. Lodge critical incident reporting. Need to
address her concern and reassure her in every way this
will not happen again, and provide her the example
how you intend to avoid this from happening again.
She will have a lot of concerns and anger and you need
to apologize, reassure, offer solutions and answers to
her concern. I didnt mention about PALS as she never
mentioned lodging a complaint but if she did, offer her
)16/16(.ways to lodge a complain

:Station 5
BCC1: A elderly lady with dark pigmentations over her
shins. Further hx: long standing DM on OHA, long
standing pigmentation for years, not causing
symptoms apart from itchiness. It is a case of
necrobiosis lipoidica diabeticorum (most likely healed
lesions). Given differentials of chronic venous
.insufficiency with stasis eczema, diabetic dermopathy
)28/28(

BCC2: A case of a young man with headache. A


challenging station as there is a lot to get from history
and to examine, and all need to be done within 8
minutes. Further history revealed symptoms of
headache worse in morning and with sneezing,
vomiting and blurring of vision. Examinations were
normal. Didn't perform fundoscopy but did mention it.
Concern: Is it brain tumor? My mom had brain tumor
at age of 40. DDX: headache due to raised ICP, e.g. IIH,
.less likely SOL, migraine. Mx: Offer urgent CT brain
)28/25(

Overall: 148/172 (PASS)

:Personal opinion

Exam case in UK are generally fair. It has tendency to


put up cases with subtle clinical findings esp. BCC.
Normal surrogates are frequently used in BCC, with
scenarios like headache, syncope, fever etc being not
.uncommon

The examiners were rather strict and particular about


identifying correct physical signs. This is the
component that scared me the most. This applies to
PACES everywhere and a lot of practice is required to
be able to pick up subtle signs. Never create signs as
.this is really fatal
Station 4 is very unpredictable. Cases can be easy or
complex with multiple agendas. Suggest to review all
the cases posted up here previously and practise them.
Need to have some knowledge regarding DVLA, Mx of
meningococcal ds and prophylaxis etc... Need to really
elicit the concerns, and offer solutions/answer as much
.as you can

.Good luck and all the best


This group is very helpful, keep sharing cases and
.experience guys

Experience in Mater Dei Hospital Malta on 2/4/16 first


carousel

Station 1
: Chest
A young patient with spares head hair( I Said possibly
2 to chemo later on upon discussion and actually I
picked it up as I used to see this finding a lot in my
practice in oncology).. RT side of the chest is depressed
and moving less, RT thoracotomy scar and decreased
chest expansion, impaired percussion and dec breath
sounds
Diagnosis: RT pneumonectomy
DD of etiology was bronchiactssis, fibrosis, Abcess and
malignancy
Discussion was about cancer causes in young patient
(germ cell, and Satcoma ) and workup also asked if he
developed SOB what might be the cause , I mentioned
infection and thrombosis PE
?How to investigate him
)I got 20(

: Abdomen
A middle aged male with features of CLD (D
contracture, P erythema, thenar wasting and Tinge of
jaundice) and splenomegaly I said no ascites
DD and work up
Honesty I felt that I missed hepatomegaly
)I got 16(

:History
A 50 years female , married , works as hospice nurse,
travelled to Kenya with her husband and came back
with nausea,vomiting, fever and upper and pain
radiating to back
Heavy alcohol intake
Had 3 miscarriages at Gestational ages of 26,28,28 no
personal or Fx history of VTE
Gp letter mentioned high T bilirubin 70 and high all
Liver enzymes
? Concerned is it cancer
DD : I mentioned Alcoholic hepatitis, viral hepatitis(A)
and dengue, autoimmune hep, and malignancy
discussion was about working her up , and how to
manage, I mentioned that she needs admission, clinical
assessment and rehydration if dehydrated, pain control
and fever ttt with NSAID and avoidance of
acetaminophen and teat etiology
I emphasize on alcohol cessation referral
)I got 20(

:Station 3
CVS: old male has peripheral features of AR
apex displaced
Systolic murmur all over radiates to carotid
I said AS and AR although I didn't hear the diastolic
murmur , I was not comfortable to the auscultatory
findings and I felt may be something is missing, anyway
, they discussed with me what might be the causes of
systolic murmer in this age and how to differentiate
between AS and sclerosis, investigations to do
)I got 20(

: CNS
A middle aged patient
Instruction was : this patient has problem lifting
objects
I examined his upper limbs , he was sitting on a chair ,
he is non English speaker however examiners helped
with instructions and I passed few instructions in
Maltese my self( most of them sounds as in Arabic)
Findings are pure proximal atrophy and weakness at
shoulder girdle and scapular muscles with defined
supraclavicular and scapular margins, no facial
involvement

DD : proximal myopathy likely congenital causes as


patient has an atrophy
And I suggested scapulohumeral variant I enlisted few
other causes as well
Investigations including EMG,NCS, and muscles biopsy
He asked me about mode of inheritance I answered
that I can't recall
Management is supportive and I motioned that few
Novel therapies is under study
)I got 20(
:Communication
Speak to an angry son of 70+ female admitted initially
in orthopedic ward with # femur and underwent
arthroplasty 2 weeks ago , 1 week after she felt while
doing rehabilitation, since this last fall she is on and off
confused, orthopedist assure son that this because of
UTI and she is receiving ttt for that , then patient
transferred to medical ward as her confusion
continues, CT scan arranged , showed intracerbral
bleed with midline shift, neurosurgery advised to hold
enoxparin ( which was started as prophylaxis) and her
.usual aspirin and stop her oral feeding until he see her
Role : speak with son about CT findings and
subsequent plan and discuss the clinical judgment
.when outweighing benefits and risk of LMWH

Son was angry but I listened to him empathetically and


reassured that I'm here to help, I broke the news of the
CT findings and explain the role of Neurosurgery
opinion, his concerns : what is the cause of her bleed,
why giving anther blood thinner while she is on ASA ,
could the fall be avoidable, why he has been told that
?she has UTI
Actually examiner's discussion revolved around
whether LMWH has caused her bleeding or not and
wether there is a way to know that I said unlikely it
was the direct cause however above therapeutic level
of anti factor Xa might give a clue that helps to reveal
.the uncertainty of her bleeding cause
)I got 16(

Station 5
: BCC1
An old male , c/o slurred speech for 30 minutes, three
previous episodes of near fainting , during episodes he
. feels "fluttering" sensation of his heart
PMHx : HTN on amlodipine 5 mg , AF on pacemaker
and warfarin 3 mg and regular check, ranitidine for
gastritis
Exam : AF with rate of 80

Discussion was about DD


I mentioned TIA , orthostatic hypotension
How to investigate, he ask me will you change his anti
?hypertensive or not
?How do you know if the pacemaker is non functioning
)I got 28(

BCC 2
A young lady, pregnant in 18 weeks gestation with SOB
for 2/52 and cough with occasional whitish phlegm and
occurs at late night and early morning,no any other
symptoms upon discussion
KCO bronchial asthma was controlled before
pregnancy on INH SABA & INH steroids but she
stopped them both after got pregnant as she thought
they're harmful
Examination: all clear , LL clear
I explain for her the role of inhaled Mx in controlling
her asthma and that why she got these sympx ,
reassure about safety in pregnancy, adviced PFM diary
and FU with GP
Discussion: DD chest infection and less likely PE
Examiner asked what've s against infection, also asked
? if PE need to be rolled out what to do

Actually I peaked my marking sheet within the


examiner hands while pill was ringing and I'm about to
leave the room with all marks in satisfactory area , I
felt it was a comfort message from Allah at the end of
the exam
)I got 28(

Over all I scored 168/172


My conclusion that PACES is a MOSIAC experience,
it concludes different roles and various methods and
the probability of passing lies in practising as many as
.. one can do of these roles and methods
Inhance your best qualities and fill your defects and as
Prof Zein says eliminate your chance of failure by
.avoiding the failing practice

Thanks Dr Zain again and again for your support and


effort and may Allah grace you with health and
. serenity

Thanks all members of the group for the endless


effort that helped me and others, may Allah bless you
.all

My exam was in July and exam center was Mandalay,


.Myanmar

Station 1(chest) : Middle aged male patient with


clubbing, trachea shift to the right and crepitations in
right upper zone that were cleared with coughing and
dullness in right lower zone
I gave diagnosis as Right lower lobe collapse with
.broncheatasis
Examiner questions were differential diagnosis of
dullness at lung bases, etiology in this patient and how
.would you manage him

Station 2(Abdomen) : Middle aged male patient with


jaundice and splenomegaly
I gave differential diagnosis as cirrhosis of liver, chronic
haemolytic anemia, tropical splenomegaly syndrome,
.myelopoliferative and lymphopoliferative disorders
Examiner questions were another name of tropical
splenomegaly syndrome and then how would you
.manage

Station 2 : Young patient presenting with chronic


diarrhea, he also has history of repeated chest
.infection, sinusitis, deafness
I gave diagnosis as primary immune deficiency most
.probably due to CVID
.Concerns are is it cancer and is it HIV
Examiner questions were causes of diarrhea in this
.patient and how would you manage
Station 3 (Neuro) : Middle aged patient with ulnar
nerve palsy due to leprosy. Examiner questions were
where is the lesion, which muscles are spared as
.patient didn't have claw hand and management

Station 3 (Cardiac): Middle aged female with diastolic


murmur only at apex. I told examiner that this is not
MDM and I heard EDM at apex. But, I told him that I
.didn't hear EDM along left sternal edge
I was very stubborn at that time and I gave diagnosis as
.AR
Examiner questions were severity of AR and
.management

Station 4 : The firefighter who is planning to get


married had allergy to smoke. He is also chronic
smoker and breadwinner. My task is to tell the patient
.to change the job and further management plans
Concerns are he didn't want to change the job, didn't
want to tell her fianc, he is afraid of losing job as he is
financially dependent on this job and also want to
.continue smoking
Examiner questions were what concerns the patient
had and did you solve all concerns and what are ethical
.and legal problems in this case
Station 5: Outside question was 40 yr female patient
.presenting with fatigue
.Diagnosis was OSA with hypothyroid underlying DM
Examiner questions were what is main problem and
.how would you manage

Station 5: Outside question was 50 yr female patient


.with double vision
.Diagnosis was basilar migraine
Examiner questions were differential diagnosis and
.management

Experience of a DEAR Friend ,,, Please : pray 4 Him


North Cambridgeshire Hospital, Wisbech, UK
st day, 3rd cycle 07/07/20161
:Station 1
:Abdomen
middle age lady with signs of scleroderma in rt hand,
my findings was only hepatomegaly and mild lower
limb oedema, I did badly in this station because of
stress, I dont know why I said to examiner that Dx is
CLD, he asked me if this patient came to u in clinic
what will you do ? I said take a full history, then he
interrupted me, history of what ? I said of what might
be the cause like alcohol Hx or any risk factor of getting
viral hepatitis, travel Hx, then he asked about Ix ? then
he interrubted again asked about her left hand if I
noticed anything, I have the feeling that I missed an AV
.fistula because that will make sense
.it was a very bad station for me to start with
:Chest
Middle aged man with clubbing, crackles on both bases
which changed after coughing but it was not coarse
crackles, I think examiner noticed from previous
station that I was in too much stress, he wanted to
smooth it down and asked me now you found
clubbing in a pt with bilateral crackles if you put them
together what will be the diagnosis? I told most
propable Dx is ILD but I can not rule out bronchiactesis
because character of crackles changed with coughing,
he asked about Ix? findings you are looking for in
HRCT? How to asses severity clinically ? how to manage
?
not a bad station I guess
Station 2
plus year old lady diagnosed with breast cancer she 50
did surgery and received chemo/radio therapy
sent by GP because family are asking to admit here as
she cant cope at home anymore
The lady told that recently her mode is going more and
more down and she is depressed, I asked here to tell
me more about here condition then she told me that
tow years back they discovered that the cancer
reached my bone, in system review she started to tell
me that she had abdominal pain but she thinks its due
to conistipation , when I asked about water work she
told me Im going frequently to bathroom, asked about
polydipsia and it was there also, and the feel short of
breath when she walks to the bathroom, her concer
that this depression may be due to the liqid she is
taking for pain and the abdominal pain might be
caused by painkillers and if I can admit here because
here daughter who is caring for her had to travel for a
short period
explained to her about hypercalcaemia and the
management including the need of admission
examiner asked about my differential ? management?
?What will you consider before discharging her
Station 3
CVS
MR and AS, examiner asked about type of murmur? DD
? of pansystolic murmur ? how to investigate
CNS
Could not complete examination, the pt wasted a lot of
time during examination of tone because she was
moving, I found spastic paraparesis without sensory
level
examiner asked about DD, investigations and
management of demyelinating disease (MS) including
new lines of Rx
Station 4
Patient with psoriasis and psoriatic arthropathy well
controlled with methotrexate her GP prescribed to her
a course of trimethoprim for UTI that interacted with
methotrexate and caused pancytopenia
She presented to ED with nose bleed
Task : explain medical error
Station 5
Lady with neck swelling for 2 weeks -1
from history she has the swelling for years but changed
in size for the last 2 weeks, clinically Euthyroid ,
concerned about cancer
?discussion about management
Lady with skin rash -2
palpable non-blanching purpura affecting both lower
limbs and back
HSP
? discussion about management
I had my exam in Brunei on the last day in second
schedule. Exam was tough with some atypical cases,
but ALHAMDULILLAH (All praise to Allah), I passed it. It
was my first attempt. My sincere thanks to PACES
EXAM CASES and all it's contributors, esp. Bebo bebo
and Mahiuddin. I had been a silent observer here. Dr
Mahiuddin gave a lot of useful tips here which really
helped me. I also thank to my all teachers esp Dr
Abdulfattah, who taught me the basics of this exam in
.a very simple way. I would like to share my cases here
Respiration: Young short lady, with SOB. Patient .1
could not lie down, so all examination in sitting
position. No clubbing, central trachea, B/L basal
crackles not fine but doesn't change with cough as
well. My diagnosis Pulmonary fibrosis, Other DD
Brochiectasis. Examiner asked about diagnosis and
different causes. British lady examiner was very
cooperative and she sensed my nervousness as it was
my first ever PACES station, that also respiratory (time
.taking) and plus young lady
.I got full marks
Abdomen: Obese man, round face, and abdominal .2
striae; with active fistula at left wrist. Few scars in the
neck, left subcostal scar with few scars beside it. No
hepatosplenomegaly. I felt some fluid hitting my hand
when patient turned his body. It was a very difficult
palpation. I got shifting dullness as well (??). My
diagnosis- Patient with end stage renal disease on
haemodialysis, most probably on steroids, cause could
be due to Glomerulonephritis. Examiner asked me why
he had ascites. I said due to volume overload
(uraemic). Then why not pedal edema? I told may be
partially treated. He asked for any other reason for this
ascites in renal patient. I told he might have peritoneal
dialysis, which could be reason for fluid. He asked me
for any proof? I showed him the scars on abdomen. He
said it could be due to surgical drainage. I said it could
be. Then he repeated the question, any other reason
for ascites in renal patient. I was very nervous and
.couldn't answer further and the bell rang
History: Middle aged man with SOB and leg swelling .3
and past history of recurrent chest infection. I finished
before time. Examiner asked me about diagnosis. My
diagnosis Bronchiectasis with cor pulmonale (right
heart failure). He asked me of any other possibility. I
could not get it. He asked me about complications of
bronchiectasis, I said local and systemic. He asked
further about systemic. When I told amyloidosis, he
asked, "could it affect kidney" . I told yes, it can cause
Nephrotic syndrome and that is one of the possibility
in this case. He was very happy to hear this from me
.and he gave me thumbs up
Nervous system: Middle aged lady lying down with .4
her right hand near body and wrist looks dropped. I
asked her to put her hands in front and turn the hands
up. Initially the right wrist was dropped but slowly she
raised it. That added to my confusion. I immediately
started typical upper limb examination. Power 4/5 in
the right upper limb. Tone - normal, reflexes - absent
bilaterally with negative Hoffmann. Sensations - I
checked pain and vibration only, due to shortage of
time. And both were reduced on the right side. There
was no obvious facial deviation. I was fully confused. I
went for common thing first and said it could be stroke
in spinal shock. British examiner asked me the proof to
support my diagnosis. I told it is difficult to say without
examining the lower limbs and cranial nerves. But the
typical pyramidal pattern of weakness with unilateral
sensation loss of all modalities could be the clue. She
asked what did it mean by pyramidal weakness, I said
"even though it is more typical in lower limb here I can
see that abductors of shoulder and extensors of elbow
".and wrist are weaker, giving the typical posture
I got full marks ( I can't believe, I am still not sure about
.diagnosis)
CVS : Middle aged man, with midline sternotomy .5
scar. Dual valve replacement with MR, AR and AS, with
chest congestion but no pedal edema. I forgot to check
thrills. British examiner did not agree with my apex
finding, which I immediately accepted. He asked me
about diagnosis and complication. It was a typical
.station
Communication skills: Young man from military was .6
referred by GP for further check up as his brother died
of HOCM last year. His ECG done by GP was normal. He
had appointment for Echo after 2 weeks but still
couldn't get appointment for genetic studies. He was
not eager for further tests and had concern that his life
would be disturbed and he might lose job if it came out
to be positive. He started aggressively, Alhamdulillah, I
tamed him and convinced him. My MRCGP skill helped
me. Examiner asked some typical questions and also
what would I do if he didn't turn up for further
investigation. I told I would take the help of GP or
employer to trace him back. Chief examination
.coordinator was present during this consultation
.I got full marks
BCC1: The coordinator confused me with other .7
case. I lost some time in confirmation. Young lady with
decreased vision of sudden onset in both eyes for 2
days. Diabetic for 6 months, not following up, not
controlled. Father had glaucoma. Past history of
gestational DM. She could only read the top line of
chart. Field normal. Before I started fundoscopy,
examiner informed that two minutes were left. I
looked in the right eye, there were black pigments
suggesting retinitis pigmentosa. I had no time to look
at optic disc or macula. I told I would like to refer her
urgently to Ophthalmologist and also check her blood
sugar. Examiner asked me about diagnosis. I said it
could be due to osmotic changes in the eye due her
uncontrolled sugar. She asked me about anterior
chamber. I said I could not examine due to shortage of
time. As there is no pain the chances of glaucoma is
less. As it is acute and bilateral, Retinitis pigmentosa
can't explain this. She asked me about complications of
DM, I answered everything except Retinopathy (funny?
I felt very depressed that how I forgot this... Exam
.tension). I am still not sure about diagnosis
BCC2: Young lady with hand deformity. She had .8
pain in hand joints and backache. Fingers were
deformed just like rheumatoid arthritis. Nails were
normal. On asking I got to know she had rashes over
elbows which were well hidden with clothes.
Alhamdulillah I got it. I examined her properly. I
managed the time very well here. Examiner asked me
about diagnosis I said Psoriatic arthritis. Then he asked
about type of deformities, signs of activity of disease,
.chest findings and management
.I got full marks
Alhamdulillah, I passed the examination comfortably.
.All praise to Allah

DETAILED Experience of colleague Dr Aisha Elamin


This is my paces exam experience in eygpt,it was a
.tough one ,but al7mdolellah kathiran i passed
I started with station 1
Chest: Copd+bilateral lung fibrosis+ some brochiectetic
.changes on z right side
I took 5 mins examining the pt generally and the ant
chest ,the examiner told me that i have just 1 min left
,so i examined z pt back and lymph nodes and sacral
.oedema
?the examiners ,asked me for z positive findings
i told her there are obst changes with end insp crackles
bilaterally ,and medium sized crackels littly changed by
cough ,so there r brocheictatic changes
She asked me what type of crackels again,what invs
? you want to do for him
when i told her lung f test and it will be obst changes
. she asked me just that, i said mixed
why he has these changes i told her pcoz he may have
repeated infections on top of copd
like the usual bacterial inf ,she asked me what other inf
i told TB
what management ,i told pharmaclogical and non
. pharmaclogical ,and i told her all till steriod
I got 19/20
abdominal station
D: decompensated chronic liver disease +huge
splenomegaly +ascites
They asked me what the cause,then what other
infections cause huge spleen i told kalzar and
shcistosoma, what invs i told all till i came to ascitic
tapping ,she said for even small ascites i told her
.according to US
when i said check serum albumin she asked why you
? want to do it
What mangement ?accordingly to dd,complications
?she asked when you want to give antibiotics
Why is he decompensated i said j+ ascites,she asked is
? he j
20/20
Station 2
History: it was advanced breast ca + hypercalcemia
The scenario was tough they just told us she has breast
ca and she was treated with chemo and radio ,she feels
.unwell pls asses her
So i couldnt figured what is happening and i thought
.that i am going to talk to z daughter
So when i entered i shaked hands i was blank,i greet
r u z daughter of ms.maha, she said no i am ms her
maha,so i surpreised and said sorry,then i told her
would you just tell me about your condition,she told
me the story ,she feels drowsy and unwell recently ,i
said may be brain matestsis ,so i asked about all cns
system,then i didnt get anything i said may be
dermatomyositis ,but nothing ,then i asked her about
the treatment ,what she was given and for how long,i
thought it may be tamoxifen induced cardiomyopathy
,but no hf .just sob on moving to bath
Till i came to z water system ,she has ploy uria at night
.
and she is so depressed ,i was lost,then i told her i
want to reherse what i get from her ,i said you have
increase water frequency +depression+ constipation (i
think it may be from morpheine)+ back pain
(metastasis)
The examiner told me u have just 2 mins left ,so iasked
about smoking,alchohol,impact & drug history ,then
concern,i told her i want to admitt you and do some
imaging n blood tests ,may be you have some
metastasis,and i want to ask the phsycatry to asses you
and give you some nuritional support and fluides then
time finish
examiner asked me what do you think,and why you
?want to admitt her
i told him i want to give her nutritional support + iv
.fluides+ do imaging
Asked why you want to give her iv fluides i said pcoz
she is not eating,and dehydrated,i want to asess her
.first
?then what else
she is dehyderated
And has polyuria and polydepsia,so it may be
hyponitremia ,then the examiner told me so z pt has
polyuria,depression,abd pain ,what do you
think
i siad hypercalcaemia
What is z management?Rehydration + calcitonin ,he
asked what else ?i forgot z besphosphonate totally
.so replied i couldnt remember
Then asked me what other speciality you want to
.consult,apart from the psychatrist ,i said z oncologist
?Finally what z dd o her sob on moving
I said PE, or metastasis or pleural effusion ,n i will do
.imaging ,but i think he was looking for anaemia
What is z cause of her abd pain ? I said could obst or
metatsis,he said could z hypercalcamia
I said yes ,lastly he asked what abour her social issues
I said i am so sorry i couldnt ask her with whome she
lives ,he asked is it important i said sure because if i
want to admitt her ,she may have some issues to be
.solved.(lives with her daughter who travelling now
I got 10 /20
:Station 3
Cvs :As+Ar with dominant AR
?They asked me what is d?what you want to do for him
What is cause? In this young pt bicusbed aortic valve or
.rhuamtic heart disease
What about his pulse rate? large volume collapsing and
.regular
What you want to see in echo? What r signs o severity
?on echo
then what else?what about complications ?IE,but he is
.not febrile and has no signs
What management? Accordingly,duretics if he present
in Hf,asked me is he in hf ? No,i couldnt appreciate any
.crackles or ll oedema
then ACEI ,examiner :*even with this AS,i said
.according to ECO if is it significant or not
Then surgical,aortic valve replacement most probabely
.metalic pcoz he is young
20/20
Neuro: Rt hemiparesis((upper motor neuron
lesion+cerbellar signs))
DD:(Ms or multiple strocks or spino cerbellar
degeneration)
.The instructions was examine z motor system
I started by the LL,then UL finally the face i examine for
horzintal nystagmus, facial nerve and hypoglossal
.nerve
pt has rt hemiparesis,has cerbellar signs in form of
dysdyadokinsia ,rebound phenomenon,finger nose
test,all evident on the rt upper limb plus horzintal
.nystagmus
In addition he has UMN signs in form of upgoing
planter in the Rt side , the refelexes r normal in the LL
.but increased in UL on the rt side
want to examine his gait and speech (what type of
?speech
? examiner asked
. what about the Lt side i said it was normal
.what about z tone ? hyptonia
.asked why ?due to cerbellar lesion
?What diagnosis?DD
What investigations? MRi brain looking for plaques of
? ms,Ncs (he asked what do you see
Lumbar puncture(looking for what ? Oligoclonal band
? (what is it
? What management
Pharmaclogical and non pharmaclogical
Staion 4: the senario was about an elderly lady which
had multiple strocks and recent brain
heamorrhage,known DM and ESRD on regular
heamodialysis ,now she is deteriorating ,and her
wishes was to stop the dialysis if she is getting
deteriorating,and the treating team decided to follow
.her wishes
My task was to inform her son about her wishes and
.the team decision
I started by asking the son ,is he z next of keen,does he
want anyone to attened this meeting with us,did he
see his mother recently and what does he know about
?her condition
Then i told him unfortantely her condition is
deteriorating as he told me ,and about her wishes,and
. that our team decided to respect her wishes
?Surrogate: if you stop dialysis what will happen
?S: is she going to die?and when
S: ok if so ,let me to take her home ,i will bring a nurse
?to stay with her
Me : i apprecite your feeling ,i know yr keen about your
beloved mother,but it is difficult to be managed at
home,pcoz there is substance called k ,it is going to be
we need to monitor her closely. to give her the high
.proper management
?S: what about her Dm and other things
Me: i assure you ,we are going to treat her respectfuly
and with diginty, taking care about all her needs and
manage her blood sugar.only the dialysis was stopped
?S : i am afriad she is feeling pain
Me : she is not aware about her surroundings most
.probabley
?Do u want me to call any one for you
What about you? Who was taking care of your mother
?at home ? And with whome she was living
S :i am a business man,was so busy recently ,i couldnt
stay with her,i hired a nurse for her, i have no siblings
.or other family member
Me: i can understand how is difficult for you,and
. appreciat yr feelings
? Do you want us to offer any social support for you
?What is concerning u more about her
S : ok thanks dr, i jusr want to be sure that she is not
.feeling pain,and to stay with her for now
Me :your more than welcome ,if u want i can arranged
a meeting with my consultant ,and the kidney
consultant to discuss with them.and your welcome to
.visit her at any time
. Only 2 min was left
Me : did your mother has any advance directive or did
she tell you about her wishes ? Or anyone told you
?about that
S : no she didnt
.Then i summarize for him and he agreed
?Examiner asked me? What z issues in this senario
.Bbn, empathy,autonomy of z pt ,advance care of ill pt
? What z issues of her son
?How do you konw this is rt decision
Me : i trusted z senario & my team so most probabely
.they r sure that z pt was competent when she decided
?E : how do u know z pt is competent
Me : that she can understand z information ,recall
E : no there r 4 componenets of it
Me : recall and weight benifits and risks and no one
.inforce her
E :not recalling it is retaining
Me :yes that what i mean
? E :what ethical issues in it
Me : autonomy ,empathy
E : empathy isnt an ethical issues
? Me : benfecience (what is it
Malefecience (what is it )
. Finally finished
16/16
: Station 5
:1
Female 40 ,came with headache
I was totally exhuasted and it was my last station,when
.i read i suppose it was a male and i put different dd
So when i entered the room,examiner told let us start
with female pt ,i was shocked
she has headache for 2 months,no signs of ICP ,no
fever or symptoms of manangism,no trauma,no cns
.symptons,no aura,i felt i was lost .no drug history
Till asked about her period ,she have just gave birth to
.her baby 2 month ago,period stopped from that time
asked did she bled a lot,she said yes
? what happen,asked about lactation
.she couldnt lactate her baby since that time
).it is shehan syndrome(
asked about symptoms of panhypopitutrism.she is
depressed,feeling hot ,fatigue,etc
:examination
Started by hands ,checking PR,rough skin,i asked to do
bp standing and sitting.examiner told me it is written
behinde you in z wall
I asked to examine her neck,gave her water to drink for
thyroid examination
to check her for breast atrophy and examine the axilae(
for hair distribution). Examine abdomen (for straie)
. ,back for interscapular fat, examiner told me no need
.to do fundoscopy ,examiner told no need
. I forgot to do visual field
Then i answered concern, the need for urgent
admission,give iv fluides.do some imaging and blood
.tests
?Examiner asked what is d
pitutary apoplexy due to post partum haemmorrahage
.causing panhypopiturism
iv fluides ,iv What management? Urgent Admission
.steriods,thyroxin
What investigations?MRI brain for pitutary and blood
tests,etc
?What dd
.I said migrane but there was no a typical aura
.Infection but no fever
I think they r were looking for bengin ICP,and pitutary
.tumor i forget to say
28/26
:Bcc2
yrs male with facial weakness,vitals r normal 54
He has rt facial weakness for 1 week ,no other cns
symptoms ,when came to hearing problem,surrogate
told me he has rt ear vesicles 1 week before with ear
.pain
.Also he is a heavy smoker
I examined facial nerve,rough examination for hearing
,asked for torch to examine the mouth for 9th
.crn.examiner told no need
.Examine the ear for rash
.Examine arms for pronator driift
Asked to examine for upgoing planter,speech and
walking. and to do chest examination ,examiner told all
.r normal
Concern was what z d? Is it strock?does he need
.admittion
I said it facial nerve affection most probably due to
recent viral infection, no need for anti viral pcoz it is
not active now .it is unlikely to be strock because he
has no signs of cva or weakness ,but we need to refer
him to nerve dr ,do MRI brian as out pt ,to be sure
.there is no lesion in z brain,esp he is a heavy smoker
.I adviced regarding to stop smoking
We will gave him drugs called steriods,he should cover
.his eye and eat gums to move his mouth
We will give him refreshing eye drops and refer him to
.physiotherapist
?Then examiner asked: what z d
Is it strock? I said it is unlikely pcoz most probably it
.will be in the brain stem,has weakness and more ill
He said but it could be strock .i said may be
?Asked what invst
Brain MRi to be sure there is cerebellopontine lesion
.esp he is a heavy smoker
.Then basic invt
.I replied the same managment i said to surrogate
What complications ? Eye keratitis
What speciallity dr you need to ask him to see the pt a
?part of the nerologist ,ENt,physiotherapist
Opthalmologist
?What abou his speech
28/28
Regarding the books ryder book2 ,for history and .3
communication.(however i didnt complete half of z
.book)
As well as,please have a look on the sample cases . 4
on the mrcp uk.(i came on few of them but it was really
good and give you a clue about what they need u to do
.in z exam
. Oxford bocket book for station 5 .5
. The only thing i found it useful in OST books .6
. OST book 2 ,is really good in history
OST book1: the first 20 cases in station 5
.section,(although i knew that very late
Thanx a lot for all of you i tried my best to write in
detial to prove that the exam is simple and you can lost
inside the exam,and say a silly thing but finally u can
pass

Mandalay Center Q 2015/2nd diet


Day 1, R 1 & 2
History Middle aged lady with fatigue, Hb% - 8 g%,
MCV 77, Previous history of IBS, Dx Coeliac disease
Communication 56 year old lady with PCKD by USG,
Creatinine 450, BBN
BCC1 25 year old lady, C/O Blurring of vision x 6
month, Dx Bilateral OA (?MS)
BCC2 36 year old male with neck swelling x 9 months,
palpitation & SOB x 1 wk, Dx Diffuse Toxic goiter
Resp Left sided pleural effusion
Abdo Massive splenomegaly with left supraclavicular
lymphadenopathy DDx lymphoma, CML
CVS Midline sternotomy scar, Mitral valvotomy scar,
loud P2, sinus rhythm
CNS C/O diplopia, Examine CN - ?Bilateral ptosis, CN
normal Dx - MG
Day 1, R 3
History 25 yrs old man C/O palpitation & chest
discomfort, glycosuria (+). Past H/O intermittent
hypertension (GP told him it would be white coat
.)+( hypertension). Frequent panic attacks & anxiety
DDx Pheochromocytoma, Thyrotoxicosis, Anxiety +
white coat hypertension, ?Type 1 DM & heart disease
Communication Known case of DM, hypertension &
AF presented with ?TIA (cant speak for mins). Past
history of bleeding d/t warfarin use & also had bad
experience about warfarin (his relative died of ICH
during taking warfarin). So he dont want to take
warfarin. He is now taking aspirin. Counseling about
.anticoagulation
BCC1 Puffy face & body ache & pain. Past history
)+( HT, DM, Bleeding d/o x 15 yrs. Methylpred 4mg od
O/E Proximal myopathy (+). DDx Drug induced
Cushings $, Cushings disease
BCC2 Known Parkinson d/s presenting with frequent
.falls
Day 2, R 1 & 2
History Young lady, presenting with fatigue. Hb% 10,
$BP 150/90. Joint pain (+). Dx SLE with APL
Communication Peanut allergy in a chef
BCC1 Chronic smoker presented with productive
cough worse at night. Concern Ca?. DDx GERD,
COPD, ?Ca
BCC2 Joint pain. Dx Systemic sclerosis
Day 2, R 3
Histroy Known type 1 DM with good control, C/O wt
loss - hypoglycaemic attacks (+), postural dizziness
Dx - Addisons disease
Communication Known Parkinsons disease admitted
for UTI. Antibiotics were given. The patients daughter
blamed that her moms Parkinsonism became worse.
Counseling the patients daughter about her mothers
disease and ongoing treatment (ask detailed side effect
.of drugs in PD)
BCC1 Chest pain DDx Angina, GERD,
Musculoskeletal pain
BCC2 Poor DM control, HTN, ?Goiter Dx
Acromegaly + Hyperthyroidism
CVS - MS with PHT
Neuro - flaccid paraparesis / left sided complete ptosis
& complete opthalmoplegia
Respi - bronchiectasis
Abd - COL / Hepatomegaly
Day 3, R 1 & 2
History AKI due to excess fluid loss from ileostomy
(watery diarrhea) with underlying UC
Communication Asthma management. The patient
was afraid to take steroid inhaler & prefer to home
.nebulizer. Counseling for correct treatment
BCC1 Recurrent fits
BCC2 Right shoulder pain haemarthrosis with
haemophilia
Day 3, R 3
History Headache with blurred vision, Dx SOL brain
Communication - Newly diagnosed UC
BCC1 Difficulty in walking, DM with proximal
myopathy, Dx - ?Polymyositis
BCC2 Palpitation, Dx MS, AF

COPIED from Paces uk study group on TELEGRAM


Exam experience of Aberdeen Royal Infirmary 24/6/16
Station 1
Abdomen renal transplant,/pckd
Resp fibrosis due to RA
Station 2 Exertional syncope
St 3 cvs aortic stenosis
Neuro umn and lmn in lower limb
St 4 dicuss with pt PMR diagnosis, management,
treatment, prognosis
St 5 left superior homonymous quadrantopia
Paracetamol poisoning
Collection of U.K. Exam cases this diet by Dr.Nazia Asim
Scleroderma e swallwaing diffeculty ..and
rynouds...Dibetic pt + addison With skin rash
.Necrobiosis lipeditecorum
asthma does not want to have a steroid inhalers due 4
to horse voice
.Scleroderma with SOB , Neurofibromatosis
Histry HOCM
Station 4 bbnews. Mesothelioma fr palliative care,
Brain tumor
CNS.Huntington disease
Station 5. TIA,Collapse
Neuro ,,, eyes examination

Mandalay, 2015/3rd Diet


Day 1 round 1
St 2- back pain
St 4- barette oesophagus
St 5- recurrent fits, OSA
st day 3rd round station 3 neurology was ask me to 1
assess pt speech n proceed. cerebellar speech with
cerebellar sign. cvs MVR with mid line scar n valvotomy
scar comm copd FEV1 24%, spo2 -94%. confused. talk
to daughter. consultant thought poor prognosis,
enquire about ventilation. concern whether can go
home n attend grand daughter wedding yr end.
examiner ask will i surprise if pt died tonight, i said no.
n then asked how about daughter, i said yes. he want
me to talk to pt he is having a life threathening
condition but i only mentioned severe to daughter 5:1
hand n foot pain for 3mth, malar rash, oral ulcer, hand
stiff morning relief with exercise n raynaud
phenomena present 5:2 h/o chest injury 3mth ago.
chest pain 2mth. worse on walking n after meal. relief
to certain extand after pain killer station 1 resp
broncheatasis with right upper lobe collapse n
consolidation with trachea deviation abdo only
anaemia with no hair in axilla. liver 1f n only tip of
spleen palpable tender. no lymph node. d/d history
new onset unilateral headache for 3mth. all symptoms
fit migraine n no neurological deficit at all. cocodamol
tanken more than 15d/mth. concern brain tumor
Day 2 round 1
H/o - Asthma d/t beta blocker
Comm - GE with DVT
BCC - foot drop
AS with fixed drug eruption -
Day 2 round 3
St 2- angioedema
St 4- MS and Mx
St 5- painful hand - systemic sclerosis
Blindness at left eye ? OA
Last day second round
History - traveller diarrhoea
Comm - ECG ST depression for explain coronary
angiogram
BCC 1 thyroid, neck swelling - palpitation d/t Post
partum thyroiditis
BCC 2 blood disorder with facial swelling - Dx - drug
induced Cushings underlying ITP
Last day last round
bcc 1 - bilateral leg swelling due to amlodipine
bcc 2 - double vision. pt is normal. Dx not sure (IIH)
history - ankylosing spondylitis
comm
pnia curb 65 -5, sudden death of his father
explain abt the management and care, daughter is
angry
abdo liver transplant
resp right upper lobe collapse consolidation
cvs MR with pul hypertension and functional TR
neuro lower limb distal weakness. I gave ddx of GB and
motor dominant peripheral neuropathy. sensory is
.normal

Here is my exam experience hopefully it will be useful


Maadi Military Hospital 2/6/3016 second cycle
Well organized very good atmosphere
: I started my exam with cardiology
MVR with A.FIB ,valve functioning well
Q1:causes,RHD then immediately
They ask me about management as he SOB;diuretics
and anticoagulation
Then Q2 if patient had fever what he could have
??infective endocarditis and what is the Target INR2.5-
3.5
Then neurology station :while am examing young man i
can hear the click he has pyramidal weakness on left
side with clonus ,and they ask d/d left sided
hemiparesis stroke in young ;then i said as i could hear
click cardiac cause A.FIB and then ask how you will
decide about Anticoagulation i said CHADS2 score
other D/D demyelination then ask how you investigate
for MS I said MRI VEP and LP
Then communication station i felt i did Bad
elderly with UTI and Parkinsonism which was
diagnosed 3 years but not on treatment ,now she is
admitted with UTI ,her parkinsonism become evident
and started on treatment carpidopa ,role to D/W her
daughter management
My D/about that she is elderly fragile with uti her
symptoms appear
Then daughter ask about side effect of Parkinson drugs
Then i asked social history she told my father bed
bound with stroke and my mum is only care giver ,then
i discuss other modalities of treatment like deep brain
stimulation
Then i told we will involve social worker if no solution
then might need to think about nursing home for your
parents
Examiner ask me about treatment of Parkinsonism
Feeding i said PEG tube then i need family ,they told
me to why u need family to Discuss ;i said she might
have LPA or advance directive As she is incompetent
I felt am of point as i didn't talk much about UTI
But i score 16/16
; Then i moved to station 5
Cushing i asked what is your concern but I didn't
answer the concern cuz no time ,examiner ask me
what is your D/D cushing ,hypothyroidism ,then he ask
me about how to investigate and treat cushing and
what is the difference between cushing disease and
.syndrom
:Second st5
years old presented with polyurea 29
History of RTA three years concer could it be cancer
? Examiner ask what is your diagnosis
Diabetis insipidus also ask about investigation i told
water deprivation,desmopressin then examiner said
more simple one i said urin and serum osmolality and
.treatment
I forgot to refer both patients to speciality,may be that
is why i score less
Then Abdomen:hepatosplenomegaly with heart failure
and also she had auidble click .gum bleeding,echemotic
patch in her hands
D/D Decompensated CLD i told hepatitis C ,then he ask
management i told referral to hepatologist and ask
about latest treatment for Hep C i told bocepravir
Chest:COPD ,with basal crepirations i told with fibrosis
examiner didn't like it he want COPD only then he ask
me about non pharmacological therapy and then
.pulmonary rehabilitation
History station:35 years old with hearing difficiency
from recurrent infections with meningitis at age 17
,chest infection ;UTIs and came with dirrhoea and
weight loss ,i told examiner
hypogammaglibulibaemia,he told me what else then
HIV ,then he helped me cuz I forgot Cystic Fibrosis,till i
said cystic fibrosis so am not sure what he wanted or
how he will judge me cuz i gave only
hypogammaglibulibaemia and CF after his help
Concern newly married will my children will get it then
i said Advance technologies like gene selection and IVF
20/19

My advise is practice more than studying books

COPIED
Exam experience Kasr AlIny hospital
6016/6
first day 3rd cycle
CVS -1
prosthetic valve mitral with AF
Discussion was so long I finished my examination early
he asked me about indication for replacement ,
treatment, and cause of chest pain in such case, target
INR
Score 19/20
Abdomen
Pale pt with hepatosplenomegally
DD start with hematological cause and still CLD on my
list then he asked me about common cause of CLD in
egypt then how to approach pt and treatment
20/18
Chest
Female with rt apical fibrosis and pleural effusion
Discussion was about causes and treatment but I
scored bad because I didn't exposed pt completely she
asked me not to do she was young and I respect that
but examiner didn't like it
20 /8
Neuro
Peripheral neuropathy gulliam barri and discussion was
about DD and treatment when to admit pt
The funny thing in this station
That before i start i asked her if she has pain any where
and if she felt and to tell me then while am doing tone
she scream of pain I stopped immediately i told
examiner i
Station 5
proximal muscle weakness wt gain -1
History everything was negative the only positive that
he is on thyroxine i asked surrogate why he is on
thyroxine because i asked about previous medical
illness he said nothing he told me I don't know
My DD at this point cushing hypothyroidsm
I examine to role in or out one of them it was
hypothyroidism diffuse goiter
Discussion was about investigation treatment
28
middle age pt with lower limb weakness with oral -2
ulcer
Hx was suggest to behecet disease i examined lower
limb neuro and for erythema nodosum,And oral ulcer
Discussion was about cause of weakness how to
diagnose and treatment
28
]12:14 25.06.16[ Muna Moon
]Forwarded from Muna Moon[
My exam experience gasr al3eni hospital first day 3rd
cycle
CVS -1
prosthetic valve mitral with AF
Discussion was so long I finished my examination early
he asked me about indication for replacement ,
treatment, and cause of chest pain in such case, target
INR
Score 19/20
Abdomen
Pale pt with hepatosplenomegally
DD start with hematological cause and still CLD on my
list then he asked me about common cause of CLD in
egypt then how to approach pt and treatment
20/18
Chest
Female with rt apical fibrosis and pleural effusion
Discussion was about causes and treatment but I
scored bad because I didn't exposed pt completely she
asked me not to do she was young and I respect that
but examiner didn't like it
20 /8
Neuro
Peripheral neuropathy gulliam barri and discussion was
about DD and treatment when to admit pt
The funny thing in this station
That before i start i asked her if she has pain any where
and if she felt and to tell me then while am doing tone
she scream of pain I stopped immediately i told
examiner i
Can't continue examination she is on pain he told me
proceed I thought i lost it but al7amdole ALLAH
Score 20/20
History
Pheochromocytoma men
Young pt recently diagnosed with HTN and he had
panic attack he was started on diazepam
Discussion
DD add hyperthyroidism he asked me how u will
explain wt loss in Pheochromocytoma i told him 10%
can be malignant
?Why men
ve family hx+
Symptoms of hypercalcemia
20/16
Communication
I scored bad and I didn't read scenario good
Middle age pt newly started on thiazide for HTN he
was walking on hot weather he drink water then he
had fit
Na was 114
Explain to wife about conditions and prognosis
What i did i explained why he had fit and the idea of
dilution hyponatremia and the effect of thiazide and i
told the wife its provoked seizure but still we need
image to role out other causes
But this part upset examiner he said no need for
further image no need to discuss job and driving
16/8

The Experience of the Exam of our colleague


)copied from Dr. Zain group(
Examined in Egypt last diet
start my exam with station 3
Cvs:it was case of shortness of breath diagnodis
wasMVR with pulmonary HTN In AF question was
about AF managment , B blocker contraindication ,
target INR for mitral valve replacement
Score 19/20
CNN case of difficulty in walking in young patient
finding was pallor, jaundice with hemiparesis lt side q
was about causes of hemiparesis how to investigate
and how to ttt
Score 18/20
Station 4
yrs old lady on renal dialysis with past history of 80
stroke after which she become blind she experience
wish to stop dialysis if her condition become worse and
the renal team decide it is time to stop dialysis she is
drowsy with shortness of breath and expected to die
after 3 day if dialysis is stopped speak to her son about
his mother condition
It was tough and I don't know how I will manage I
Remember the consequence of Dr. Zain I start with
same manner after greating and permission of note
and if any relative wont to attend , how much he know
about his mother condition he know little about it I
clarify her condition and the need to stop dialysis to
her and I ask if he know that his mother she has any
wish and he know about the wish of his mother he ask
to take mother home since dialysis is stop I counsel
him about the need for her to stay in hospital for her
best interest his concern was about his mother
condition and if he is able to take her home and after
how many day she will die I tell it will shorten her life
then summarize and check his understanding the till
me still u have time I don't know what I will tell more I
didn't discuss about DNR
Examiner q was about issue
And why u will keep patient in the hospital and what
about the wish if her son
Itwas v.bad station for me
Score 10/16
Station 1
Respiratory was case of copd with bibasal fibrosis q
was about how to investigate and how to ttt and lung
function test
Score16/20
Abdomen
CLD
Q was about finding and how to investigate and how to
ttt
Score 20/20
Station 5
Case of headache only complain in history she has
Sheehan syndrome and not on ttt
Concern cause of headache
Q what us the cause of headache and how to
investigate and ttt of Sheehan syndrome
Score 26/28
2
Case of lt facial weakness LMLN
No other CN affection no weakness concern if this
stroke and what is cause of it
Q what is causes of facial weakness how to ttt
Score 28/28
History role sho in medical admission unit case of CA
breast received chemotherapy and radiotherapy she
can not copy at home and insist for hospital admission
I was though it is communication not history
In the history she has symptoms of hypercalcaemia
with bone metastes back pain and shortness of breath
her concern is the admission and about the cause of
her symptoms
Q how to treat hypercalcaemia and palliative care
Score 20/20

My exam experience first day second round new kasr


el aini hospital
Dr/yousif el malahy
Score 140
Firstly i like to share how i prepared for the exam
This was my first attempt
Not take any courses
Just listen to dr ahmed maher in you tube for
communication senarios but not for hx taking and it
will be reflected by my score in station 2
For physical examination stations i prepared for it by
collections from many books and videos and start to
practise it in my clinic with every patients
I started with station 3
First one neuro
Examine lower limbs
It was spastic paraparesis with abcent knees and
upgoing planters with no sensory impairement except
for in L5 dermatome in right side it cause me cofusion
not reach diagnosis but discussions was good i reached
diagnosis only while driving back home as pure lateral
sclerosis i score 13/20
Second one cardio
Obese bad exposure
Not cooperative
Midsternotomy scar with no vein harvesting scar
Metallic sound before systol i diagnose it Prosthetic
mitral valve
No signs of infective endocarditis nor heart failure
Discussion was about investigations
I was not sure about diagnosis as while we out 2 of us
say it was mitral 1 say it was aortic because patient
was tachycardic
Score20/20
Station 4
Explain diagnosis of IBS 7 for 7 years not improving
little angry want mor investigations and second
opinion
I think i do it good
Dicussion was about why not do more invextigations
??may it be cancer
If he has the right to seek second opinion
Score13/16
Station 5
First one acromegaly
Complicated by carpal tunnel obstructive sleep apnoea
visual problems but no symptoms or signs of field
problems discussions about cause i say papilloedema
or optic atrophy
Score28/28
Second one pemphigus vulgaris
Generalized erosions oral erosions
No other signs
One of concerns about complications i said im not sure
Dicussion about DD i forget steven jonson
Surprisingly i score 26/28
STATION 1
First one abdomen
Jaundice pale splenomegaly ascitis liver not felt
Discussion about DD
Score20/20
second one chest
COPD
There was crepitations but i dont mention i think there
was bronchiectasis or fibrosis also
Dicussion about investigations and treatment
Score14/20
Station 2
My nightmare
Abdominal swelling pain diarrhea
I miss past medical disease of exised breast cancer
however i ask about family history of cancer
I miss diagnosis all my focus was about coeliac disease
and inflamatory bowel disease this was simply because
i gave no attention to abdominal swelling and didnt
analysed it
Score6/20
overall grade pass
Thanks to dr ahmed maher eliwa
I share this photo because it helps me alot in practice
of physical examinations at home

kuwait 23/3 cycle 3


station 1
Abd
About 40 years man looks very well no sign of chronic
liver disease not pale or j' no av or central v catheter
scar or fistula then I notised right iliac fossa scar so I
thought this will be a case of crohn's disease butt later
I found a mass below this scar resonant can be bi
manually palpable move less with respiration . No
other mass no ascites no bruit .then I returned back
searching for sign of immunosuppressins , infection .
Volemic status . Scar or fistulla peritoneal cath scar,
this patient have very small central v cath scar in his
chest . Later on I met this patient he told me I did very
well and I will get the full mark because he have the
. experience to judge the candidate performance
Examiner questions
Present your POSITIVE finding and diagnosis. The
causes of renal failure ,I said I couldn't find any clue
helping me to know the cause but most likley it is dm
. ,htn , glomerulonephritis
? What is the immunosuppressant drugs for R trans
.?What is their side effects
How you know this patient have a faild renal
?. transplant
Respiratory case
years old man dyspnic .clubbing with sings of 50
obstructive lung disease decreased cricosternal
distance ,prolonged expiration with wheezes. resonant
exept the bases . Harsh Crackles changes with cough
anteriorly ,soft crackles not change with cough in the
bases . When i said that the british examiner shocked
he said to me are sure I said yes very sure, he sked me
are you sure the crackles in the base are not change
with cough i said yes and there is dullness . He said you
have to chose between bronchiectasis and fibrosis , I
said could be both fibrosis due to repeated infection,
he said no you have to choose , I said fibrosis. Then he
said suppose this patient have bronchiectasis how are
?you going to investigate
What is treatment ? What is the most immportant part
?of non pharmacological treatment
Station 2
For history station if your exam at center running late
like me you have to read and practice all cases came in
the other center uk and overseas. Because of that this
station was very easy for me . I knew that this is the
case of postpartum thyroiditis. I just need to rule out
the other d diagnosis of palpitation and other causes of
.hyperthyroidism
. A lady complain of palpitation
recent delivery , weight loss. Feel warm , loose motion
. Postive familly history of heart disease her mother
died due to heart attack at age of 60 . No cardiac or p.e
risk factor, no neck lump or eyes changes , no tremor .
Negative every thing else . Past history of asthma no
drugs other than sulbatamol inhaler , no smoking,
house wife , live with her husband and 2 kids satisfied
financially when I asked her are you satisfied
financially she said look if you are asking about my
mood I am ok and I dont have anxiety then she
laughed, me and the 2 examiner also laughed alot
.when I asked her how this problem affected your life
she said I dont know and she looked to the examiner
. with big smile
? Concern :- is it heart attack like my mother
? What is my problem
? Is it treatable
I explained to her the problem and answered her
. concern
Very lovely examiners British man and young lady I
. think she is a sudanese
She asked me about d diagnosis ? P p thyroiditis, pp
cardiomyopathy and h attack and arrhythmia are very
.unlikley
Investigations ? Some questions about what I expecte
? about antibodies and thyroid isotope scan result
Treatment? I said the treatment is b blocker but
because this patient have asthma I have to seek a
senior advise . And close f up of her next pregnancies
.because this problem may re occur again
Station 3
Cardio
Berfore I enter the room I saw the patient coughing .
large amount of sputum I said most likely this patient
have pulm edema . Then no right hand pulse, left
hand regular large volume collapsing , raised jvp but I
wasn't sure .s 1 was soft normal s2 loud at the
pulmonary area left parasternal heave palpable 2nd h
sound . Pansystolic at the apex radiate to the axilla but
you can hear it all over the precordium but with
diffrent intensity at upper left sternal edge . And I am
pretty sure that there was a radition to the right
carotid . There was also lung bases crackles and no
lower limb edema . That what I found but I said to the
axaminer no right hand pulses he asked me why ? I
said catheter and clott, embolism . Then good volume
pulse I didnt mention collapsing. I said systolic murmur
radiate to axilla with different intensity I will do echo
to make sure about this . with pulm htn . He asked me
without echo how you can know if there is tricusp reg
or not . At is moment I knew that mean the jvp was
raised I said to him sorry I forgot that the jvp was
.raised
. so there is tricusp reg also
When I told him that this patient have basal crackle he
told me forget about that . He asked me does this
?patient need anticoagulation
?Investigation
?Treatment
after that the other examiner asked me are you sure
that you didn't hear any diastolic murmur ? by this
question I thought that mean I missed some thing
serious but I said no I didn't hear . He looked straight
. to my eyes and I kept silent
Cns
Examine upper limbs
years old man 50
Have an amputated left leg below the knee. large right
upper arm scar with
Power zero hyptonia no reflex loss of all types of
sensations in the same side of the sacr
Normal left upper limb normal back examination,
normal co ordination . I asked can I examine the lower
limb he told me I dont think it will help you .to be
honest for unknown reason I didn't compare the 2 limb
in reflexs and sensation. I didn't asses the sensation by
.dermatomes only by level
_: Examiner
?Your positive findings
?The cause
Nerve injury due to trauma most likely accident .
?Which type of trauma
? Which nerves
?Investigation
. Treatment? Just support
Communication
Very long scenario about 14 lines . It is in d. Zain
sheet but with a little difference . I took what I thought
a bad performance in station 3 so I wasted more than 1
. minute doing nothing
A man about 50 admitted MI found to have low HB
about 110 g/l , nothing was done and they told him it is
.due to NSAID discharged on aspirin
month later his GP found his Hb about 70g/l (I am 2
not sure about those numbers ) and he did some
investigations and found nothing( here I dont know
why I thought the cardiologist was the one who did the
investigation and all returned normal) . That all I can
remember, actually that all what I knew when the bell
rang. But I kept calm because I know I can gather all
.the Information from the surrogate
Then can you share with me what you know about
your problem ? He told me every thing about his case .
More than that he told me he afraid it may be a cancer
and the gp told him that he may need blood tranfusion
. Why the cardiologist didn't do any thing to
discover this cancer as the cause of his anemia ? He
afraid of blood tranfusion because he think he can get
. hepatitis and hiv
So dealing with angery patient keep calm listen carfully
allow him to talk dont disturbe . I told him he can't do
camera for at least 6 weeks after MI . And it was wrong
to be on aspirin so we need to stop it till see the result
of the investigation we will contact the cardiologist to
give you alternative medication .And let us go for ward
. to know the cause of your low hb
. The blood is tested and he will not get Hiv or heptitis
I took quick history seerching for common causes of
. anemia and he had nothing positive
I asked him about endoscopy I found that he have alot
of information about It . Then i explained the
endoscopy . Offer a leaflet to read and the endoscopy
doctor will answer any questions before the procedure
.
I dont know what is the cut of for transfusion when
using g/l value i know it 6 when you use g/dl . So i
told him we need to involve the blood doctor and also
the bowel doctor
? The patient asked me if he have a cancer
I said the cause of your low hb is not obvious so we
need to do full check up to know what is the cause it is
ranging from mild cause like diet and bleed due to
. aspirin to more worrying like cancer
_: Examiner questions
He asked my why the cardiologist didn't do anything
for low hb ?I told him he did but all normal
He told me that was the Gp . I told him sorry because
of the scenario was very long I thought the cardiologist
was the one who did that . But if that was the case
there is negligence . He asked me is it nice to expose
the mistakes of our colleagues? I told him to be honest
. is the best thing
? Then he asked me about the issues in this case
What if this patient refuse the blood tranfusion? He
. have the right to refuse
.What we call that ? Patients autonomy
Do you think this patient have cancer ? I said it is
.possible
Station 5
First case a man complain of joints pain
So main d diagnosis :_ 1/R arthritis 2/osteoarthritis
.3/psoriasis 4/ank spond
Open question then
.Targeted questions to reach the diagnosis
He have symmetrical metacarpophalangeal joints,
proximal interphalangeal and both wrists some times
shoulders and knees pain . Morning stiffness . Releived
by exersice.this problem For more than 10 years . No
back or neck pain . No skin rash no eyes changes. No
breathing problems . Normal water work normal bowel
.habits
P. history of viral hepatitis but have been treated many
. years ago
Drug on steroid, azathioprin , NSAID. But no great
benifit
. He doesn't know what is his diagnosis
IT engineer . Negative f. History . No great impacts on
. his daily life
. Normal physical examination normal hands function
Most likely r arthritis it controlable ,we need to carry
out some tests, blood and x ray . Appointment to
.discuss the result of this tests
Gruop of specialist will be involved in your
management including joints doctor who may need to
change or add some medications physiotherapists and
.occupational therapist
Concern? He told me I already answered all his concern
thank you . He was very nice indian man he told me
after the exam :- actually I helped you alot . I said
yeah thank you so much . And he was right because he
gave me most of this information after the opening
question. And he was easily satisfied by my
.explanation
_: Examiner
?D diagnosis
Any relation between his liver problem and this joint
. pain I said no relation
?Inv ? And treatment
?What medication we can add
?Biological medications
Station 5
hours right side weekness 50 . years old lady .bp 2
150/95
So main d diagnosis
. TIA , complex migrane
Less likely m sclerosis antiphosphlipids syndrome.
.vasculitis and Psychological
Sudden onset right side weekness continued for 2
hours no headache no face weekness no vision or
speach problems no loss of consciousness. negative
.other cns symptoms
No palpitation no chest pain or breathing problems. No
. skin or joint problem
P history of htn no other vascular risk factor no p
history of similar condition no history of unsteadiness
.or bowel or bladder control . No p.h of migrane
F history of stroke mother at about 50 years old
. Drug :- lisinopril , no oral contraceptive
She is a nurse . She doesn't drive nor smoke. No
.alcohol
.No mood problems
physical exam :_ normal pulse no carotid bruit normal
uper limbs examinatin . They stoped me doing heart
auscultation and cranial nerves examination including
. fundus after I affored to do them
She want to know what is going on ? Is it stroke ? Will
?it happen a gain
I said it is mini stroke with explanation . Yes it may
happen again so we need to admit you and carry out
some tests blood test heart tracing and scan . Brain
image and ultra sound scan to your neck .the nerve
.doctor will be involved in your management
-: Examiner questions
What is your positive findings? Nothing
Why do you want to admitt her despite the weekness
. has resolved? Abdc2 score
?Investigations

Sharing my experience with Paces: Never give up & u


.will reach your goal
I had 4 attempts for paces, the first attempt was 2013
where i was quite immature and had a nightmare.
Went on to 2nd attempt had a heart breaking score of
129 passed all components but failed total mark. 3rd
attempt was in Brunei December 2015 where i scored
150 but failed in a heart breaking component of
.concern with 9 marks
Finally manage to complete my marathon in 2016 4th
:attempt
:Exam case
Started with the station i dread the most and least well
.prepared - station 4
:Station 4
MND counseling: break bad news about the diagnosis
)16/12( .and address patients concern
:Station 5
)28/20( Thyroid eye disease with cataract
)28/21( Psoriatic arthropathy with lower limb OA
:Station 1
Respi: scleroderma with pulmonary fibrosis (obvious
sclerodactyly with mouth furrowing and unable to put
in 3 fingers into mouth, bilateral fine basal crepitations
with bronchial breathing. Chemoport seen at right
chest region. Was asked about the use of chemoport &
how to correlate with the diagnosis. I was unable to
)answer that.(20/20
Abdomen: Chronic liver disease with
hepatosplenomegaly, was asked what one single test
you would like to do to confirm your diagnosis? Finally
was told to be liver biopsy by the examiner. I was
)20/20( surprised by the marks
:Station 2
Pulmonary fibrosis secondary to methotrexate /
bronchial asthma / bronchiolitis obliterans. Questions
)20/15( .about bronchial asthma and treatment
:Station 3
CVS: ASD with ?TR in failure. (9/20), i guess i messed up
this station and created the signs of tricuspid
regurgitation. Perhaps there is none. Lesson learnt is
!not to create signs
Neurology: Guillain barre syndrome with patchy areas
of sensory loss and bilateral lower limbs weakness
(13/20), was questioned about findings in LP for GBS.
Unable to reach management & thus affected my
.mood. Luckily this is my final station
Total score 130/172. I wish those who are going for
exams good luck & work hard. Play hard and enjoy the
.learning process too

Detailed and very useful Experience ,,, from a colleague


who appear in Brunei 10/6/2015
Brunei 10/6/2015 these is my third trial and worst trial
and the evidence for that in the trial before I get full
mark in more than 4 station but I didn't pass don't
astonished that is PACES and in the last exam I didn't
get any single full mark any station but I pass my exam,
why I told all that story because after my shocking
result in the past I had wrong believe if I didn't get full
mark in most of the station I will not pass and that
proved totally wrong
I started by station1 my first case is respiratory young
man looks good complaining of S.O.B I find only
trachea shifted to the RT and crepitation bilateral
changed by cough I finished in time the British
examiner was the leader
He asked my did u finish your examination I started my
presentation I rich the diagnosis of bronchiectasis he
asked about investigation and management was
ringing we moved to next case me and the local
examiner and I started my examination for middle age
man and I rich looking for the hands and until that time
the British examiner not came in the time I started
shaking because I said to my self maybe I have bad
presentation or wrong diagnosis in the respiratory and
he marked me bad and the local examiner also
went I become alone with pt ,the local examiner he
asked the British to come when they came I rich the
face of the pt and I asked my how they can assess me
in the examination of hands and difficult to start again
because the time is already gone I complete my
examination and it was case of hepatosplinomegaly I
started my presentation in the end of the discussion
they asked me if we said for u the pt is stable for long
time , in that moments I feel lost and that means I
didn't rich the diagnosis and become silent for
moments and after that I don't Know why I opened my
mouth and I said it could be APKD I think the face of
examiner became like and is ringing in the same
time really in that time I pray to not hear me at least
one theme because I'm sure these is case
hepatosplinomegaly I went for st2 really depressed
because I compared my previous performance in the
previous exams and got full mark really I become
depressed for me be for 2 minutes I didn't looking for
scenario of the history and I told my self with full mark
I didn't pass in the past how I will pass with the bad
performance and that is totally wrong I get 19 and 18
And after that I feel like I'm in dream I heard yours
voice ,dr.zain and all of u , u said don't worry that is
only one st u can compensate in the next coming st
really like I wake up from sleep I tried to forget and I
realized that I have very long scenario I have ever seen
in the history it's like communication scenario
It talk about young female with long history of
uncontrolled DM type1 and now presented with
recurrent history of hypoglycemia they written
investigation and long story I feel depressed
again I know the history of DM very long also and the
examiner will be so sophisticated in marking ,I enter
the st I found the same surrogate of my last exam but
that time she was in st4 I remembered her she was
very nice in that time I feel relax and started asked her
in these time she become talkative and also not
understand some layman English like water system
always she speak about diarrhea and I found difficult
to control her when I rich family history they remind
me for 2 minutes I feel shaky I want to ask her
for many thing I realized that I did badly managed my
time. They start to ask about her problems I said most
of DM complication prephral neuropathy and
autonomic neuropathy and retinopathy and the cause
of hypoglycemia it could be CKD or Addison and they
discussed with me about management and ringing
when I rich the door I remember that women I didn't
ask her about her job and driving and I felt that I did
fatal mistake because the last exam I fail skill C I said
allhamdolilah and felt depressed again I heard your
voices and I remembered I came here to do all the
exam not only 2 st they give me 14
I refresh my self by drinking water because I feel very
thirsty I enter st 3 and I full of hope it will be very easy
because I practiced very will with imtithal by the way
the history also I practiced with her so nicely when I
remembered I said to my self ah ah ah I started by
cardio meddle age it looks like COPD pt I examined
him and find nothing in cardio after I rich the back
searching about any thing even basal crepitation and I
didn't find any thing ,still I have time can u imagine I
started agin the examination of precordium only I
realized that man has distant heart sound ,when he
said tell your finding I want to cry I said for him I
found nothing than distant heart sound I put
deferential diagnosis they discussed the management ,
in that moment I felt that my destiny not pass that
exam we went for neuro the British said to me these is
sister she will help u with your case really I asked my
self what that case need for help in his examination
when we enter the room I read the structure it written
women she blurring of vision please examine her
cranial nerve women about 75 years old she can't
understand single word of English and sister translate
for her ,the problem when I give her the order and
nurse translated for her she understood wrongly
especially in visual field and eye moving she started to
laughing with high voice because she find it difficult
and the British examiner Also laughing with her in that
time I said to my self I rich here by 3 airplane and they
laughing , in end I find only bilateral tossing , I
said for him I want to pursued he said carry on I asked
to count and looking for ceiling it was +ve and also
fatiguability test is not clear I said for him I want to
examine her chest he said only from outside also she
had difficulty to release my hand and I find that
difficult to me is that from bad communication or true
sign I put deferential diagnosis myasthenia gravis and
muscular dystrophy and from discussion they want
only myasthenia they give me 16,14
In front of st 4 just I remembered your talking about
istigfar and alsal ala alnabi
I did that and I felt relaxed and starting study st4
scenario about meddle age women she had past
history of breast ca and bronchial asthma ,she done
surgery and received chemotherapy before 2 years
now she has pneumonia before 6 weeks ago and she
did x Ray now the radiological department they found
shadow and today also repeat the x Ray with the same
lesion they said talk with her and dealing with
uncertainty
I enter relaxed she was angry at first because the delay
but when I said I will revise your file I will reported if I
find any delay she became relaxed I speak with her
about the possibility of cancer or infection and we have
to further test like ct scan and bronchoscope and I
explain for about it and I asked about allergy for dying
and contrast also did she do it before also I asked
about kidney function also I check understanding many
time and make summery and closing ,they asked about
the ethical issues I said ,BBN,dealing with
uncertainty,empathy and counseling about the
investigation in the last time he asked me what do you
think the main issue of these scenario, I just keep silent
and after that the ringing I went out relaxed I said to
my self it will be full or 14 and I feel these is best st
from start and no big difference from my performance
of my last exam and i get full mark in that time but that
also is wrong feeling because I get only 10
I rich my last st and most challenging st for me because
my previous trials i lost all my scores in st 5 first
scenario about elderly man has history of Mi came for
cardio clinic complain of difficulty of walking they given
normal vital ,case 2 about young female complain of
chest pain also with normal vital when I enter I found
old man with good dressing sitting in chair beside him
young lady I greet him really he masked face but I think
firstly that's normal because the old Asian looks like
these I started to ask his relative when I asked about
shaking hands she said yes he has after these point I
target my questions and examination of Parkinson's I
think the most thing cause the examiner happy I tried
to exclude vascular cause and also the examination of
babinisky sign because the he has shoes with socks I
volunteer to help him because I felt the time is running
pt refused I did it and knee in the ground the
British examiner came near to me after that I examined
his gait and his concern about his treatment of MI can
caused the problem I reassured him and we didn't start
for him the management until the problem effected his
life they asked did you find tremor I said little bit
Really it was not obvious also he asked about gait I said
shuffling he said ok go for next case
I found young lady in the bed with hijab I started to
take history I found she has history of SLE after that I
target my questions about PE and also the pain
increase when she bending forward I put precarditis I
complete my examination and I finished early he asked
do u want to do anything more ,during examination I
found the pt has tight skin now I look for her again and
she didn't have other sign of scleroderma I ask her to
put her hand inside her mouth in that time the British
examiner laughing they ask me about investigation I
feel relax after I finish because st5 hear is only best st
in the exam and for sure its best than pervious one
they give me 26 and 27
In the end my advice don't occupied your mind by any
believes ,because we are Muslim it hard to said PACES
is gambling game but it's tawfig from Allah
Dr.Telal Eltyb

In Liverpool , Broad green hospital . I started with


:station 2
A pt with long standing dm presented with swelling of
. both leg and fatigue with +++ proteinuria
:Station3
Cvs: pt has midsternotomy scar with pacemaker scar
.with esm at aortic area.no srphanous harvesting graft
.CNS: both sensory and motor neuropathy
:Station 4
A female patient has admitted with pneumonia and
developed click difficile infection. One of medical staff
.not maintain hand hygiene. Angry patient
:Station 5
BCC1: Fatigue ,; Acromegaly with OSA
BCC2: A patient has c/o not seeing well after 4pm.
.Retinitis pigmentosa
Station 1 : Abd: Rt renal transplant with Rt AV fistula
.and pd scar with hand tremor Abdul gum hypertrophy
Res: a young female with khyphosis and contracture of
finger link and thin mid thoracotomy scar _ cystic
.fibrosis with lung transplant
.Dr Mohammad Sazzad Haider
.Rustaq hospital.Oman
Keep praying for me

For those who are interested in following (Difficult) UK


Exams
" This is a small collection of " UK Exams
Glasgow PACES -1
:Station 4
Delayed diagnosis of pheochromocytoma
Mr, jones 35 years male
Had High BP for last 5 years
Seen by psych for panic attacks
Tried many Med for HTN
But
His BP has been difficult to control
On his insistence , his GP has referred him to
hypertension clinic 2 weeks before
Results of tests now show
Urine : high metanrphrines
CT adrenal : 5 cm mass in right adrenal
Ur task is to explain the diagnosis
U don't need to know the details of further tests and
further management
Patient was concerned
Is it serious
Is it cancer
Is there a cure
Will I require future surgery
What future tests will be done
Was the delay justified
What medicine u will give me
Examiner : repeated similar questions
Overall not too harsh patient
Satisfied at the end
Agreed follow up GP Consultsnt website address alpha
blocker beta blocker
16/16
History station at same centre
Opening : 11 points
Discussion : as under
Young female 28
Blood Diarrhoea after Cyprus visit
Started 1 day before coming back
Mixed with stool
Similar episodes for last 2 years
Took amoxicillin in Cyprus
Diarrhoea aggregated
Now last 10 days
Frequent blood a salime in still
Painless
C/ o small joints pain
No backache
No other extra intestinal symptoms
No oral ulcers
No skin changes
No jaundice
Cousin IBD UC
Father CA colon
No blood thinners
No steroid
No warfarin
No bleeding disorder
No weight loss
Concern : cause
? Is it cancer
? What next tests
? What Med
? Need admission or not
DD: xIBD ( UC)
Infective Diarrhoea
Antibiotic associated Diarrhoea ( as patient said
Diarrhoea aggravated by amoxicillin ( but I told least
) chances
Explained to patient in detail and agreed a plan
: Closing
summary
Labs / Leaflets / NHS choices website
Agree
Examiner : just repeated all above
And
Asked
In
; History how will u rule out infective cause
Fever
Vomiting
But
He told
U will ask about symptoms to others accompanying
him
Marks : 4/20
History & Communication : zero
Concerns : zero
DD : zero
Judgement : zero
In feedback : written
: History and communication
PC
HOPI
Past Hx
Personal
Family
Drug
Allergy
Treatment
: Social
Occupational
Travel
Association of IBD
Used jargon : IBD ( during explanation of DD to patient
)
Oral ulcer ( mouth ulcer should be used )
Didn't get more details of past episodes
Result : zero marks
DD: 1st diagnosis was IBD
But actually it was infective Diarrhoea
Result : zero marks
Concerns : though addressed adequately but remarks
are he left patient worried about the diagnosis (
serious diagnosis as IBD)
Result : zero marks
: Clinical judgement
Question 1
: How u investigate
CBC to look For
CPR ESR
Electrolytes
Renal functions
LFT
Stool microscopy & culture for infective Diarrhoea
Sigmoidoscopy / colonoscopy if required
Question 2
Treatment: as per diagnosis if it's UC
Then steroids and mesalazine
If infections : antibiotics
Remarks : want to give steroids though preferred
diagnosis is infective Diarrhoea
Candidates remarks : This happens in real life
Though I was expecting 100% 20/20
But
. Actual 4/20
###############################
Castle Hill Hospital-2
Station 2
years old .DM.asthma presented 25
With recurrent chest infections for 6 months 6 times
I put DD bronchiactesis .TB
No Hx of fever .wt loss or travel or contact with pt with
chroinc cough.he has greenisg sputum..constipation..I
did not understand his accent clearly
He continue mentioning constipation and trying to
have a baby and I totaly ignore it..his concern why I
have this recurrent infection
His diabetes and asthma are not well controled I asked
about HIV risk which up set the examiner
I forget sinusitis and examiner was angery and
.heampotesis as well
I told him we are going to do bronchoscopy..also upset
the examiner
He asked me about d ...my dd was bronchiactesis and
TB
He asked about one blood test for specific for
bronchiactesis
I told I do not remember
.. He said serum antibodies for pathogenes
I was about to say immunoglins but bell rang
.CNS: Upper limb exam .3
He has hemiparesis
I did not finish sensation
Not examin e nech
He had truma with scar in head which I did not notice
.even when examiner point it
He ask me if you notice any facial asymetry I said
no..which acutaly was present
:CVS .3
A tall women I wasted time looking for alchol gel for
scruping and washing hands with water
Marfan syndrom with 2 sacrs on medisternotomy scar
with metalic clikc and aother an rt subcalvicukar..no
muremur but 2nd sound was loud and palpable..first
was soft
My d..aortic valve replacemtn
He asked about causes of chest pain in marfan
I told ACS
And pneumothatx he asked what else which I can not
answer
She had high arch palate and archenodactyly..I think by
other cause of chest pain he wants rupture anyuresm..I
just remember it now
:Communication skills :4
Staion 4 ...80 years old patinent..Alzehimer d...was on
NG feeding and she was agreesive and agitated all the
time and use to pull it out..her doughter facing
problem with feeding and want PEG tune insertion
..speak to her doughter and explaine ill_terminal care
...and palliative care for her
I do not now mentioning DNR waa suitable or not but I
..have mention it
Examiner asked about how are you going to feed her if
.. sh will not take oraly no NG no PEG tube
:5
Station 5 was diffecult
years with skin lesion over her forhead and scalp 60
Looks like morphea
Some candidate mentioned SLE
Apart from that she did not have any manifestation of
scl
? eroderma ..her concern is it a infecious
?Is it cancer
I reassure her ..but examiner asked what could cause
morphea
Second case 62 years old ..with blurring of vision
.exssive fatiguabilty..and more blurred by the end of
the day..deffintly she had exopthalmous and
opthalmobligia..diplopia on both lateral
gazes..thyrodyectomy scar and left firm thyroid
nodules
Dry hard skin..fundus normal..no other manestation of
..thyroid ..no proximal myopath
I told dd
Graves opthalmopathy and
Mysthenia graves
:Station 1
chest bilatral basal fibrosis and skin rash..I do not
now what is it...some candiadte examiners told them it
is dermatomyosistis..it was not typical she had hard
.skin..finger tips ulcer as well
Abdomen...abdominal pain
I could apprecaite 2 masses in rt side and one mass in
left side not liver not spleen...it was transplanted
kidney ..examiner asked why she is going to have
? abdomian pain
?What about immune supression side effect
...Examiner questions was more tough than the exam
But it was nice experiance
..Keep praying for me
#########################################
:UK PACES experience-3
I want to share my experience in Western General
...Hospital, Edinburgh 25 Feb 2016
..I started my exam by station 3
..Cardiovascular; 50 year old man complains of SOB )3
I did the exam, I appreciated a murmur in apex.. I could
.. not time it
for unknown resean I said it is diastolic murmur
considering I do believe that diastolic murmur can not
..be brought in PACES
The examiner ask me if that was diastolic murmur
what will be your differential.. at meet the patient
after the exam at hospital gate and he told me he has
) 20/8 ( AS and MR !!! I scored
CNS; lower limb exam.. patient was not cooperative )3
and misleading
he kept moving his lower limb during tone assessment
and giving contradicting information during sensory
..exam.. I could not formulate DD
) 20/7 ( I scored
Communication: 40 year old lady has IDDM her )4
HbA1c 9 referred for albuminurea
I was disappointed from previous station and forget to
ask her if she does attent all foloow up appointment ,
??does she check her glucose
)20/4( I scored
BCC1: psoriatic arthritis has joint pain.. has skin )5
)rash over elbows and hair line.. I scored ( 28/28
BCC2: 70 year old lady history of loss of )5
consciousness and abnormal movement, had murmur
during adulthood for which she does not require follow
..up
My DD : epilepsy and stroke
.. I could not appreciated any abnormality in exam
I instructed her not to drive for 1 year and to inform
..DVLA
they ask me if I appreciate any murmur.. I answered
!!No
)28/24 ( I scored
Abd: kidney and pancreas transplant , has gum )1
hyperatrophy and poor vision.. I said the cause is Type
.1 DM as patient has vitiligo
) 20/20 ( discussion about complication of transplant
Chest: Rt upper lobe lobectomy with deviated )1
trachea discussion about indication of lobectomy and
) types of lung cancer ( 20/20
History: 55 year old male with symptomatic )2
.. anemia and melena on ibuprofen for knees pain
??His concern: Is it colon cancer
..I told him I ll request upper and lower GI scope
) 20/19( I scored
)172/130( The End Result is PASS
.. It was My first trial
I have never been to UK before .. I had course in Ealing
..Hopsital, London for 2 day ( it is excellent )
Despite the bad beginning .. Still AlHamdullah I
..passed
..My Advice .. do not be relactant in applying to UK
My English language and accent is not perfect however
!! they consider that
################################
UK Colchester Hospital University-4
STATION 5
A.28 yrs old male admitted with diarrhoea and fever
.37.8.bloody no wt loss no other symptoms
Differential
Investigations
Management
.B 56 yrs old male complains of dryness in his eyes
.Apparent ptosis and miosis
.Lt Horner syndrome
.Left neck scar
What is a cause
How to investigate
.How to treat
?Concern is it reversible
Station 1
Abdomen
Failed kidney transplant
With AV fistula
.Questions straightward for transplant
Chest:left side pleural effusion and with skin lesions
.mycosis fungoides
What is the diagnosis
?How to investigate
?How to manage
Station 2
yrs old male with gait difficulty.has frequent falls 69
.and difficulty in getting upstairs
Its very difficult case.till I came to the drug history.was
prescribed prochlorperazine for dizziness by his Gp the
gait difficulty came after the medicine.then I went back
.asking about parkinsonism symptoms
All questions
?What are causes of parkinsonism
?Are you going to stop the medicine
?His concern could it be brain tumour
?How do you treat parkinsonism
?Drugs and side effects
Station3
CVS: ms with SBE
?Straight forward question
CNS: examine the lower limbs in the patient with gait
.difficulty
All finding consistent with lower motor
neuron.periphral neuropathy with features of mytonic
.dystrophy
Station4
lady admitted with SLE over night all labs normal 24
.except proteinuria
.Explain diagnosis and obtain consent for renal biopsy
?Concern she is worried about renal biopsy
She refused to do it.except in last 2 minutes when
checking understanding.I reinforce that is very
important because treatments are different to the
.stage.then agreed
?Questions whyneed to do biopsy
?Who will do it
?Complications
?Ethical isaues
If refused? I toldI will speak toher again after a while. if
?refused again
I will check competency if competent I have to respect
.decision
?.How to assess competency
######################################
Edinburgh -5
:station 1
andomen: hepatomegaly with? PD catheter - how are
?they related
respiratory: thoracotomy scar plus and chest tube
:station 2
middle age man with recurrent fits in pt with esrf? no
hx stroke. pt concern unable to take care of himself if
he has epilepsy
:station 3
:cardiovascular
multiple murmurs ?aortic regurgitation with metallic
1st heart sound. also got thoracotomy scar
neuro: PICA syndrome?/ brainstem syndrome (not
really sure about this one)
:station 4
discuss with pts father regarding bone marrow
transplant. pt (capable of making own decision) refuses
but father still insists
:station 5
optic atrophy. No INO/ RAPD ? -1
oral (i think with oesophageal) candidiasis in RVD -2
refused HAART
My fren sat the exam 1st of march 2016
Good luck 4 ALL

Royal infirmary, Glasgow


June / 2016
History
young female with frequent dizzy spell and blackout,
dx was hocm
Family h/o sudden cardiac death at 30
Communication
a man was intubated following anaphylaxis after eating
salad, Now ready to discharge - talk to him
Explaining about anaphylaxis and prevention
Respiratory
bibasal crepitation, d/d ;The man looked cushingoid, so
I told most probably fibrosis
But the creps was slightly coarse, Examiner said most
!probably bronchiectasis, he was not sure either
Cardio
was very difficult, there was mr, but not sure about as,
I think I messed it up, bad station
Neuro
cranial nerve examination of a young boy with slurred
speech
I got bilateral palatal pulsy, discussion was about dd,
not bad
Abdo
isolated splenomegaly - I think I have done well here,
examiner were happy
station 5
Bcc1- psoriatic arthropathy, but he had short 4th and
5th metatarsal, I don't know was it important or not,
he had back pain, I said may be spondylitis or
secondary arthropathy
Bcc2 - a woman with previous h/o pituitary surgery
came with headache, on fundoscopy there was
papilloedema
Exam haywood hospital
2016/6/9
St 5 ist case
osteogenesis imperfecta command was pt has
recurrent
fractures, viva about genetics types etc
nd case2
elderly with arm weakness on exam no weakness only
mild rigidity at wrist diagnosis was parkinson disease (i
saw that on examiner mark sheet otherwise hard as no
other features tremors etc) qs about how to diagnose
etc
station 2
young lady joint pains she gives full history about
rheumatoid arthritis at the end when asked about
concerns she says yes i get sun burns and blue hands
she has sle most candidates fail to diagnose her as she
.did not volunter this info
Abdomen
.renal transplant with pd scars and gum hypertrophy
CHEST
lady has scleroderma multiple telengectasias scar at
back one side fine crackles other side normal diag was
lung transplant with ild due to sclerosis examiner were
more intersted in telengectasias and ild
CARDIO
MR and AS examiner wants diffrence between sclerosis
and stenosis and full figures on echo about severity
Neuro
a man with upper motor neuron signs in upper limb on
right side with strange contractures i said ms or stroke
and qs were all regarding ms right from macdonald
.criteria till managment

Yangon center previous Q 2016/1st Diet


Day 1 R 1&2
St 2 - 40 yr old female lung cancer with poor mobility
and back pain
St 3 - Pleural effusion; Parkinson' d/s
St 4 - talk to son, mom Alzeimer, confusion, UTI, no IV
line, concern about confusion
St 5.1 - myotonic dystrophy
St 5.2 - joint pain, thalassemia, splenectomy scar,
polyuria
St 1 - COL & RIF scar; CVS - ASD
Day 1 Round 2 Yangon
Station 1 - Chronic liver disease; Dullness at left lung
base
Station 2 - known case of Ca lung, previously treated
with radiotherapy last 18 month, C/O back pain
Station 3 - Parkinson's disease
Station 4 - 82 yr lady with Alzheimer's and knee OA,
admitted with confusion and UTI, can't give antibiotic
because of dislodge cannula, talked with angry son
Station 5 - myotonic dystrophy, thalassemia with
Haemochromatosis
Day 1, Round 3
History - Three episodes of collapse within 8 months in
binge drinker
Communication - Fits occur after giving chlarithromycin
in asthma patient who takes aminophylline for a
long time
Station 1 COL; COPD
Station 3 - MS with AF; Third nerve palsy
Station 5 - Common paroneal nerve palsy; Headache in
Takayasu
st day 3rd round1
CVS - MR AR
CNS - Ataxia with dissociated sensory loss
Resp - Collapse
Abd - HS with jaundice
H/O - collapse ?ALcohol withdrawal fit
Commu - theophylline toxicity
BCC1- Tarkayasu
BCC2 - Leprosy (Common paroneal nerve palsy )
Day 2, Round 2
Station 4
year old ex. manger with headache for 3 months, 45
blurred vision 2 weeks, with fits 2 days ago. CT scan
head revealed high graded glioma at frontal lobe. His
wife worked at aboard and will come back the next
day. Breaking the bad news
Concern: Why he suffer fits? How long will he live?
How to tell his wife as he planned vacation with his
?wife
Station 5
BCC 1. Right Hemiplegia with visual problem - Right
Homonymous Hemianopia
BCC 2. Hand Pain with Acromegaly - Carpel Tunnel
Syndrome
Station 1
Resp. Rt upper lobe collapse. (Axilla lymph node biopsy
scar noted)
Abd. Renal Transplant with Hirustism
Station 2
year old lady with bloody diarrhoea and abnormal 42
LFT. History of travel last 6 months ago to Australia. Wt
.loss 5 kg
Concern: Is it cancer? Is it managable? I am not
.complete in concern
Station 3
CVS. AS AR
CNS. Facial Palsy with cerebellar and CP angle Tumor.
(Operated)
Day 2 R 1&2
Station 5 - Case 1. homonimus hemianopia, AF. Case 2
CT$ in ACROMEGALY
.Station 4 - Breaking bad news. Brain tumor
Station 2 - Bloody diarrhoea
Resp - ex. pleural effusion
CVS - AR
Neuro - 7th palsy due to CP angle tumor
Abd - hepatosplenomegaly, COL
Day 2 R 3
Pleural effusion
Renal abdomen
MVR
CN 3, 4 palsy
History - post partum thyroiditis
Comm - Medical error. Codeine given to a patient who
has allergy. No features of allergy
BCC Gout; Psoriasis
nd day 3rd round another pair2
St1. Hepatosplenomegaly probably thalassaemia
??RUZ collapse
History - the same
St3. Cranial nerve palsy due to MG
MVR, previous MS and pulmonary hypertension
Comm - the same
St5. 5.1. Psoirasis worsen by propranolol for
palpitation
Gout worsen by antihypertensive therapy .5.2
nd day 3rd round2
History
postpartum thyroiditis, H/O of palpitation in previous
pregnancy.Now 4 mth after delivery of 2nd baby.
palpitation 2 mth. H/O asthma. coffee 3 cups/day. H/O
thyroid disease in sister
Communication
yr old lady with pneumonia, CURB 3, 84
hyponatraemia, hypoxia, h/o adverse effect on
codeine. Daughter told that allergy to codeine but
night MO gave 3 dose of cocodamol. Now confuse. Talk
.to daughter
Concern Why happened? I previously told about this.
Antitode? Why my mom is confused? Can I see the
.chart for reason whether you note down it or not
Day 3 round 1
BCC - Systemic sclerosis
OSA -
Resp - COPD with basal crepts
Abd - COL with bilateral mastectomy scars with RIF scar
Neuro - dysarthria & examine UL - cerebellar sign(+)-
MS
CVS - AS AR TR MR? Examiners ask to measure BP
St 2 - Tiredness with ED, with U/L DM & HT
St 4 - Noncardiac chest pain ?Musculoskeletal ?
Functional
Day 3 R 1
Lung basal crepts - interstitial lung disease .1
Renal transplant bi fistula
Lethargy & loss libido, DM, HTN (+), gap shaving .2
interval DDx hypopituitarism; autonomic neuropathy
Metalic click I told DVR (but friend said that it is .3
MVR)
--- .4
same as Hsu May Oo post --- .5
Day 3 R 3
History - TIA+young HTN (?pheochromocytoma) -
TIA+headache+palpation+stress
Comm - UC for oral steroid counselling
BCC 1. SLE + TB; 2. DM with ?Laser scar on fundus
Resp - 1. consolidation?/mass?, 2. pleural effusion
Cvs - MS+Pul HT+CCF+TR
Abd - 1. hepatosplenomegaly, 2. renal transplant
Neuro - 1. Myasthenia, 2.?MND
Day 3 last round
Station 1 - Right sided pleural effusion; Thalassaemia
Station 2 - TIA
Station 3 - MG; Mitral stenosis
Station 4 - Known UC, afraid to take oral steriod bcoz
of side effects, explain management plans of UC
Station 5 - Laser scar; SLE with TB
th day 2nd round4
Stat 1 - pleural effusion, Thalassaemia
Stat 2 - breathlessness in RA pt taking Methotrexate
Stat 3 - Parkinsonism , MS
Stat 4 - oseophageal perforation d/t pneumatic
dilatation
Stat 5 - 1. Neurofibromatosis with H/T; 2. Vitilogo with
Goiter
D 4 3rd round
History - H/T with protein & RBC on urine
Comm - Anaemia with underlying IHD with taking
aspirin and clop - task further Ix
BCC 1. Thyroid eye; 2. Ankylosing Spondylitis
CVS - MS
CNS - Spastic parapresis
Abd - Hepatosplenomegaly (Thalassaemia )
Day 5 R 1&2
Station4 - delay diagnosis of pheochromocytoma
Station2 - chronic headache with menorrhagia
BCC1 - RA with carpal tunnel syndrome BCC2 -hypopit,
c/o fatigue with increase weight
???CVS - double valve replacement with AF
???CNS - syringomyelia
Resp Pl effusion with or without collapse
Abdo - thalassaemia
Day 5 2nd round
History - headache for several months with menorrhgia
for treatment
in detail - tension type HA with medication induced HA.
?concern - cancer
Comm - delayed dx of pheochromocytoma explain -
scenario - missed for 5yr and confirm by urine and CT
concern - cancer? why delay? need to again mood dr
?and surgeon
CVS - AS AR with pul H/T
Resp - i dont know think Rt upper lobe collapse
Abd - HS with jaundice (Thalassaemia)
CNS - MND ( bilateral small muscle wasting )
BCC1- RA with CT $; BCC2- hypopit
Day 5 Last round
Station 1 - Resp - effusion & tumor?? Abdo -
??hepatosplenomegaly with CLD
Station 2 - IDA & wt loss, epigastric pain, taken
Ibuprofan and diclo for knee pain
Station 3 - Cerebellar; MS with Pul HT
Station 5 - 1. Cushings; 2. DM with CRVO
EGYPT ,,,,Cairo 8/2
St1 ,hepato+splenectomy,cha(thalasemia)
s.scl .lung fibrosis
St2; known ca prostate, PW:confusion
??St3,ms, ASD
Lt .hemiplegia
St4,young w ESRD for rrt
St5,1.hypothyroid,Cushing
Headache,optic atrophy .2

..Maadi exam 2/6/2016 1st carosel


..started with st 4
patient with Parkinson disease the scenario said she
was diagnosed 3 years ago and didn't take medication
but it was not clear so i thought that she was
uncompliant to ttt the she deteriorated and admitted
because of uti.. the surrogate was so nice and her
concern was about her mather who is taking care of
her father and what other treatment she can be given..
the examiner was not that nice!he asked me about
that the pt didn't start medication from the start and
why u told her she was uncompliant? i told him I'm
sorry thought she started and discontinued
medication.. then he asked about other lines of
treatmen i said surgery then he asked what else i said i
!can't recall.. i think it was not good not bad station
..Station 5
st cas was straight forward clear cushing s and 1
complain is uncontrolled BP.. the discussion about DD
and investigation and asked me why u didn't do visual
!field.. i said I'm sorry
nd case.. excessive urination for differential 2
..diagnosis
not DM.. had history of trauma for 3 years ago but
..surrogate didn't gave any details
i said cranial diabetes insipidus and discussion about
how to diagnose diabetes insipidus.. i wasn't good on
.discussion honestly
...Station 1..abdomen
hepatosplenomegaly on a patient with mid sternotomy
scar and heart failure.. i said congestive hepatomegaly
or other DD like cld as viral hepatitis.. it was a good
.station
chest.. obstructive lung disease he told me what other
dd and he wanted to hear why not bronchial asthma?..
i don't know my assessment for that case although i
.did good examination
Station 2 history of young patient with repeated chest
infections and diarrhea and on history there was
..hearing loss, weight loss and conjunctivitis
i thought it's a cystic fibrosis case but he wanted to
hear common variable immunu deficiency syndrome..
and i told him it's a possible dd but when i was going to
leave the room.. i did not bad in sloving patient
!concern but not good also in dd and invstigations
..Station 3
Cardiology case.. AVR with aortic stenosis.. valve needs
to be assesed.. discussion was about what would u do
?if the valve is restenosed
..Neurology case
left sided hemiparesis and 7th and 12th nerve palsies
on same side.. she had cerebellar signe on right side..
1st i said it's a clear left sided uncrossed hemiparesis
with 7th and 12th nerves palsies.. then she asked me u
?notice hyporeflexia on heplegic side how do explain it
i said it could be shock stage she said no.. i told her she
has cerebellar signs in the form of finger nose
dysmetria she told me show me i showed to her.. i told
here the patient has either MS or double stroke.. then
.investigation question was good
i think overall need Allah mercy to pass and all of
!people prayers
Egypt
Maadi 1-6 - 2016
:Station 1
COPD with fibrosis
SM with shrunken liver
:Station 3
Double aortic valve disease with questionable MS
MS
:Station 2
Hypercalcemia from bone metastasis
:Station 4
Withdrawal of hemodialysis from terminally ill patient
:Station 5
$ Sheehan
$ Ramsey hunt

EGYPT ,,, Maadi ,,,2-6-2016


:Station 1
HSM with LNs
COPD with bronchiectasis
:Station 2
Colieac
:Station 3
VSD with dextrocardia
Right sided hemiparesis
:Station 4
Chef had anaphylactic reaction and ventilated and u
should advise him to leave the job ( Indian examiner;
very tough)
:Station5
A case of headache~
Mostly subarachnoid hge versus ICH
Thyrotoxicosis~

Egypt
Maadi cairo 31 - 5 - 2016
CVS: AVR and MVR WITH NO abnormality
CNS: MS
Hist: Recurrent pneumonia in young lady who is single
and no travel history or drug abuse
Communication : Giulian bares
ST 5: diabetic and hypertensive retinopathy with very
.bad, heroic old scope
Dermatomyositis
Chest: Lt pneumonectomy with COPD in right side
Abdomen: thalassemia with splenectomy and
hepatomegaly

Egypt - Maadi
2016-5-31
:Neuro
Left sided hemiparesis with normal reflexes
:Cardio
AVR
:Chest
COPD with bronchiectasis
:Abdomen
HM
:Station 2
Painless haematuria mostly APCKD
:Station 4
Refusal of inhaled steroid for asthma
:Station 5
with bleeding per rectum#Acromegaly
lesion#skin
Vague case may be psoriasis

.Paces today may 30 , 2016


.University brunei Darussalam
.St 4
Elderly lady with pneumonia , complicated with
c.difficile. Son angry, as junior doctor didnt follow
.protocol. And what treatment, why isolate
St 5
Blurry vision both eye. Visual acuity until waving
.finger. Underlying dm. Had eye operation before
. Funduscopy
.I see black scar at vessels, and pale optic disc
.My mx all dm retinopathy and eye specialist
.My fren said it was Retinitis Pigmentosa
St 5
.Scleroderma with fibrosis, obvious reynauld
St 1 respi
.Lobectomy with joint deformity
St 1 abdomen
.Transplanted right kidney with non functioning fistula
?Why he is abd pain
.I said maybe rejection, he ask what else, I said IBD
St 2
.Breathlessness, went to thailand
.Is said copd, tb, hiv, cancer
St 3 cardio
.Dual valve metalic. With AF
St 3 neuro
.Unilateral spastic paraparesis. With cerebellar
.I said stroke, alcohol, thyroid, phrnytoin
Examiner ask about rehab . Luckily they didnt ask
where is the stroke. I just mention cerebellar and post
.stroke
..I want to go home and relax. Paces so stressfull

Brunei exam 31/5/2016


ILD:1
chronic renal disease -recent mode of replacement is
haemodialysis
.headache ..cluster headache:2
double valve replacement:3
Rt side hemiparesis
Examine the upper limb
provoked seizure (hyponatraemia) concern can it :4
? come again ? And can I drive
psoriatic arthropathy:5
Vetiligo....present with tirednes; pernicious anaemia
Adrenal insufficiency

2016/5/30
nd carousel2
Started from station 1
Splenomegaly with normal liver
There is LNs but couldnot complete and discusiion
..about lymphoproliferative
Chest
COPD i couldnot hear bronchectic change said
secretion
Discussion about copd asthma and why not asthma
how to differentiate
Ttt of copd
St2
Female with type 1 dm with loss of wt fatigue dizzy
spells fh of hypothyroidism
I did all aspect well and asked about dd inv
St3
Cardio
Double mitral doumble aourtic with mild tricuspid
..regurge
.. Asked why he has angina i answered
He didnit want to ask any questions
Neuro
Spastic paraparesis
Diagnosis was primary lateral sclerosis asked me what
inv to do i told him it is clinical diagnosis
..What to investigae
S4
Idiopathic dilated cardiomyopathy with
..polypharmacy
Not very well
St5
SOB
Pulmonary Embolism
..DD pneumonia
Wtloss
Thyroid
On carbimazole 80mg and propranolo
Invs
..Can we raise dose i said no
Options surgical
?Signs of activity

Cairo Exam 29/5/2016


First carousel
Station 1
Chest : acute case with ascitis ...pleural effusion
...COPD ? Fibrosis ..didn't finish examination
Abdomen : young man with splenomegaly ...felt LN
? others didn't
Station 2: 45 yr old man with 2 weeks headache and
short memory loss abs concentration also he has right
heminopia..he HTN and hypercolestromia ...heavy
smoker and father has a history of stoke ....DD (
remember while walking back ) lung cancer metastasis
/ stroke
Station 3
Cvs : young lady with shortness of breath ....MVR with
TR
Neuro: young lady with spastic para paresis with no
sensory affection ...examine lower limb only
Station 4
Relative refuses the discharge of her 84 yr old mother
who was admitted for pneumonia 5 days received
antibiotics and feels better and has capacity and can
take care of herself but tired Explain to the relative
Station 5
yr old lady with menorrhgia since menarche ... 20 .1
Normal platelets ....bruises and history of gum
bleeding ... Drug history : tenaximic acid and vit C. .... I
?said most likely Von willbrand
yr old lady with old recurrent rash ...1 week on 35 .2
the soles and palms ... History : HTN on Beta blockers
and takes lithium for a psychological problem ... None
smoker ...mild pain in fingers not bothering her.... Has
lesions on the shins ... Mostly likely psoriasis
exacerbated ( medication )
.....
EGYPT
Today exam 29/5 Cairo
rd cycle3
Station 1
ABDOMEN: hepatomegaly +splenectomy
No ascites , pallor . Jaundice
Chest Lt pneumonectomy
Station 2
DM complaining of frequent hypoglycemia +diarrhea
Station 3
Neuro spastic paraparesis without sensory level
Cardiology mitral valve replacement
Station 4
HCOM DISCUSS SCREENING
STATION 5
Icthiosis
Benign intracranial hypertension (headache + oral
contraceptive pills)
Cairo Exam 29/5/2016
)Another experience for the same previous cases(
Started with history taking patient 45 with headache
confusion homonymous hemianopia and short term
memory loss. he is htn and hyper lipidemix and smoker
all complains there for 2 weeks. i put dd of space
occupng lesion abscess and stroke
Cvs i cant finish exam but patient having pSM gng to
axilla and parasternal heave i put MR plus PHTN
CNS spastic paraparesis no sensory level
coominucatn 50 y old lady want to go home decided by
medical team she can go home. but her daughter
saying she is weak and she should stay in hospital
station 5 psoriasis staright forward it was palmoplantar
variant of psoriasis patient had arthralgia also
station 5 15 old lady with menorrhagia woth bruises
platelets normal disaster for me i missed normal
platelets given in scenario exmanier not happy as i put
ITP but it was i think von willebrand
GIT splenomegaly pallor for DD
RESP == it was acute patient with abdominal ascites
there was dull and decreased fremitus at bases it put
DD of pleural effusin Examiner satisfd looking

Exam of LATER dates ,,, 11/3/2016


Myanmar - yangon
March 5tg day 3rd round 11
Respi - upperlobe collapse consolidation .1
Abd - hepato splenomegly +sign of chronic liver .2
insuficiency incl gynecomasia, bilateral parotid
yr old girl HMA, low iron 35 .2
Positive histroy - knee pain, takjnv NSAIDs, abd pain,
wt loss, no family histroy
CVS - MSMR,MS dominant ,AF, pul hypertension .3
b. CNS - young boy - examine gait n proceed - wide 3
base gait, bilateral cerebellee, increase knee jerk, no
sensory, CN - intact
Patient wafrin for AF, stroke again, INR -1.2, missed .4
last INR clinic appoinment. Talk to grandson
a.young lady, hypertension, blood sugar high - 5
Cushing
b. Known DM, impair vision - CRVO5
EGYPT ,,, Cairo Exam 29/5/2016
First carousel
Station 1
Chest : acute case with ascitis ...pleural effusion
...COPD ? Fibrosis ..didn't finish examination
Abdomen : young man with splenomegaly ...felt LN
? others didn't
Station 2: 45 yr old man with 2 weeks headache and
short memory loss abs concentration also he has right
heminopia..he HTN and hypercolestromia ...heavy
smoker and father has a history of stoke ....DD (
remember while walking back ) lung cancer metastasis
/ stroke
Station 3
Cvs : young lady with shortness of breath ....MVR with
TR
Neuro: young lady with spastic para paresis with no
sensory affection ...examine lower limb only
Station 4
Relative refuses the discharge of her 84 yr old mother
who was admitted for pneumonia 5 days received
antibiotics and feels better and has capacity and can
take care of herself but tired Explain to the relative
Station 5
yr old lady with menorrhgia since menarche ... 20 .1
Normal platelets ....bruises and history of gum
bleeding ... Drug history : tenaximic acid and vit C. .... I
?said most likely Von willbrand
yr old lady with old recurrent rash ...1 week on 35 .2
the soles and palms ... History : HTN on Beta blockers
and takes lithium for a psychological problem ... None
smoker ...mild pain in fingers not bothering her.... Has
lesions on the shins ... Mostly likely psoriasis
exacerbated ( medication )

Egypt,,,,Kasr Alainy
. .. Paces exam today 28 may. 2016
Station 1_ respiratory
c.o.p.d with rt basal fibrosis
Abdomen
..Chronic liver disease. ..decompensated
Station 2...female pt 55 yrs with history of loose
motion and abdominal swelling and bloating for 2yrs.
...p.H of ca breast with mastectomy 5y ago
Station 3..c.vs: ?? mixed mitral valve disease
C.n.s...peripheral neuropathy
Station 4....I.B.S diagnosed by consultant with normal
investigations even the sigmoidoscopy..pt concerns.
.he needs further test and he is afraid of cancer
Station 5 /acromegaly with obstructive sleep apnea
second case pemphigus vulgaris

Dear all
As this website helped me a lot in dealing with a lot of
stress during my examination period, especially with
the experience of many candidates, I feel I should
share some of my own as well. To begin with I passed
my MRCP Paces. And I am very happy about it because
.so many things were at stake with this exam
Let's begin. Is it my first attempt? No, it's my third one.
First one was like a bad dream. I don't know why I even
attempted because I was least prepared for it. Then,
second attempt: I tried my best. Due to some personal
reasons I couldn't practise with my frens at hospital
and I imagined cases at home and met all sorts of
MRCP cases in my lil room in the form of pillow.
Fortunately, I met a wonderful fren to practise with
over the skype. We practised a lot and felt ready. I
even attended a course, given a good feed back. So, I
went for it in the UK. Well, although, I missed a
diagnosis of only one neuro case which was Right sided
hemisensory loss with Carotid endarterectomy, I
thought I would pass but no. I had to have another 6-8
.months of stress
So, this time I started in my hospital with exam in
mind. I examined most of the cases just like in the
exam, everyday. So, my examination technique
improved significantly. For instance, I could examine
thyroid and extrathyroid manifestations withing 1-2
mins. I tried to communicate just like in the exam
although in reality our traditional practice differed in
many ways. As in my hospital there was none
appearing for this test, I did my best with my eyes on
the prize. Before 4-5 months, I again started practising
with my old fren who unfortunately couldn't pass like
me. But everything happens for a reason. The practise
has made me more confident and more clinical
oriented. So, I appeared for the third time in Kolkata. I
took a course there, and I failed badly in the mock
exam in the course. Got a very bad feedback and felt
very disheartened. That was the last thing I needed
before the exam. But my colleagues thought I was
.good enough, so that kept my lil flame alive
The exam day was the most stressful. I couldn't sleep
the whole night. Though I have tried to handle myself
as a cool guy throughout my life, I felt like a fool that
night. I asked for a taxi to drive me to the hospital and
we got lost. There were four hospitals with the same
name, and he didn't know neither did I. He called many
people over the phone and finally we reached there.
So, I thanked him for allowing me to appear for the
exam. He charged me double but I was in no mood to
.argue with this silly man
So, finally my exam started. I was taken to neuro case
which was stroke. Finished my examination before 1
min like in other stations. I was asked to examine the
limbs. Surrogate was not only annoying but
misinterpretating. Clearly the patient was in pain but
surrogate said no. I caused pain to the patient. So, you
can imagine what must have gone through my mind.
Question and answers were easy, which I had practised
hundred times and seen many such cases. So, easy
diagnosis but I know they are not looking only at
.diagnosis. Felt sad but got 20/20
I was taken to cardio station where I was happy to see
Midline sternotomy scar. So, I got the diagnosis and
answered as MVR, but the examiner was asking me
questions like what other treatment the patient is on
beside anticoagulation. I didn't know. He also asked
me causes of displaced apex beat, and I forgot to
mention about heart failure or cardiomyopathy. So,
.got screwed. got 13/20, not bad
My weakest skill is communication. Had tried a lot but
strangely failed a lot. Confidence, I lack a lot. It was a
simple TB case where I had to assure her not to travel
abroad becoz she had active TB. I missed many points
like HIV, contact tracing, and so on. The examiner
punched me with difficult MDR TB questions and I
almost fainted. Thank God, I survived. To my biggest
.surprise, I got 16/16
Station 5 was easy. Psoriatic arthropathy and stroke in
young. These cases have already been mentioned in
this site, so I don't want to talk about cases but my
experience. For the first time during the exam, I felt
good because I was able to diagnose both cases and
answer properly to the examiners, hence I got 24 and
.26. Pretty good
As I mentioned previously I was quick with my
examination, I finished before time in both respi and
abdo, and gave some differentials for RA induced ILD,
and hepatomegaly with funny scar(or scare, never seen
such in my life). Did badly with the examiners in abdo,
.but got 20 and 19 respectively
Finally, with little energy I was left with, I went to
history station. Some people outside were laughing.
That was probably the second time in my life when I
hated people who were smiling because I found it hard
to focus on the task at hand. Anyway, with fake smile
and pseudo confidence I entered the room. But there
was no surrogate. I had wait another two minutes. By
this time my energy had drained and I think I looked
like a Parkinson's patient with mask like facies. Took
history for 15 minutes regarding diarrhoea which I had
practised for at least 20 times with my fren. So, it was
easy but again with the examiners I was poor. Got
.13/20
I thought I would fail after the exam. I told my family
and frens that I might not make it again. When I saw
151/172, I was extremely happy and called everyone I
.knew
My advice: Never ever give up. Keep on practising, and
a time will come, as my fren told me - "You will pass
".even if you appear the exam in a drunk state
.Thank you all for taking time to read my experience
.God bless you

Exam of LATER dates


EGYPT,,, Exam Experience
Almaadi military hospital 9/2/2016
: Station 3
:Neuro
:Findings
Spastic paraparesis+PN with stocking distribution (Rt
.leg)+ sensory level on left side for DD
:Questions
DD: MS, spinocerebellar degeneration, SCD
.Investigations: spinal & brain MRI, CSF findings
Treatment:acute, chronic, pharmacologic &
nonpharmacologic
:Cardio
:Findings
.AVR & MVR
:Questions
findings, functioning valve or not, HF, IE
Investigations:routine, ECG, echo, INR
.indications for AVR in AS
?what symptomatic AS means
.Treatment: nonpharmacologic and pharmacologic
:Station 4
Elderly lady admitted to the hospital with confusion
and UTI, cant give her the AB as she keeps pulling the
IV cannula out, comorbidities are Alzheimers, knee
OA, frequent admission to the hospital in the last few
.month
Task: talk to her daughter (angry), who is asking about
.an update
:I think this scenario is looking for the following
dealing with an angry relative.who also was tearing ,
): offered her tissue that was on the table
Explain the need for a PICC line,draw what you are
.going to do and consent
Sort out the other comorbidities and any risks at home
(stove, shower, lost her way before, driving, the need
for an occupational and social workers and visiting
nurse after discharge)
the daughter kept saying that she wants to continue
taking care of her mom,and no way she will send her to
.a nursing home. you have to appreciate that
The daughter wants a brain CT done, because she is
confused(it must be her brain, doctor. Why you didnt
perform at CT, you are not giving her the appropriate
care)
Be patient and try to explain that it is a problem with
the chemistry of the blood not an actual brain
.problems
:Concern
Why my mom is not improving after few days from
?admission
?When she wil go home
:Questions
Ethical issues in the scenario (Beneficence and
.nonmaleficence), dealing with an angry relative
Why you didnt do a CT as her daughter wanted (there
is no focal neurological deficits that warrant doing a CT,
also there is an explanation for her confusion), not sure
if this is the good answer, I Would like to hear your
.comments
.Long term prognosis of the patient
.what do you want to offer her
What about sedation (I said it may worsen her
condition, but I heard after the exam about chemical
.and physical restrains), I leave that for the experts
:Station 5
(Ramsay Hunt $) Facial nerve LMNL (very clear) with -1
.history of ear rash few days before
Questions about DD: all causes of LMNL facial lesions
(CP angle tumours, parotid or face surgery, auditory
canal (cholesteatoma, abscess),also UMNL facial (he
)didn't like it, ,wanted the LMNL
Treatment(steroids, acyclovir, stomach and eye
protection, physiotherapy)
?Concern: could this be cured, how long it takes
Hypothyroidism (difficult case) -2
Presentation (fatigue , weight gain, menorrhagia, no
skin,voice or hair changes, on a treatment, she doesnt
know the name, which turned out to be thyroxin,
started a year after a surgery in the neck
(thyroidectomy))
Examination: fine tremors, no eye signs ,
.thyroidectomy scar
Questions about investigations, what is the single test
you want to do (TSH)
what is the most probable cause of her thyroid
problem (Gravess?, I am not sure if that is right, there
is no eye, hand or leg signs)
.Concern : what is the cause of the fatigue
Station 1(terrible examiners and difficult patients)
Chest: Rt upper lobectomy + obstructive lung disease+
)deviated trachea to the Rt side & left basal fibrosis
Questions: findings, he asked if the fibrosis is diffuse, I
said I couldnt appreciate that, investigations
(HRCT,sputum C&S,PFT findings)
Treatment (nonpharmacologic and pharmacologic)
Abdomen: Very obese patient with HSM, pallor,
pigmented striae, no LNS
Questions:findings, one diagnosis only (didnt want to
hear DD), I said Lymphoproliferative, asked about
blood film findings and other investigations, and
(: )treatment of lymphoma
:Station 2
Patient with macular rash over chest , neck back and
sometimes arms, started as vesicles that rupture after
that,no change with
sun exposure, on doxycycline for acne,no other
autoimmune disease) for DD
examiner asked about DD(they wanted
photosensitivity in the first place, he said if you
pressed on him more he will say it it increases with sun
exposure, but I asked about that clearly and about
travel history to Hurghada and after spending
sometime on the beach, he denied any change in the
rash)
Other questions about investigations and ttt
....Concern: will it leave a scar, I said yes

-: Dubia ,,,, May 2016, first day , third cycle


Neuro LMNL , not sure
.MND , all sensations are normal
CVS: systolic murmur
S1 normal, S2 accentuated so VSD vs Tricup Regurg &
P.htn
Chest: pleural effusion
With midsterontomy scar , harvesting scar rt leg & LL
.edema
Abdomin Kidney transplant, functioning well & AV
.fistula functioning & no recent puncture
Bcc1 : DVT with h/o travel to India , Indian female , FH
.of leg clot both mother and sister & also on OCP
Bcc2: Indian lady middle age with h/o MI 3 weeks ago
& present now with chest pain increase with deep
breathing, Dresslor syndrome, diff pericarditis,
. myocarditis, pleurisy
Medication post MI ACE inhibitors, statin , clopidogrel ,
.aspn& b blocker
:History
middle age male , with HTN & recent hematuria
POlycystKD( same case in Dr Zain ) adopted ) typical
.case
:Communications
Lady middle age with h/o asthma & steroid phobia
becz she used inh steriod and her voice changed & jobe
.is singer

Dubai 17/5
Cardio
Prosthetic mitral valve it was clear case
Neuro spastic paraparesis without sendory level
I told DD MS .parasagital meningioma.sarcoidosis he
got very angry when i told sarcoidosis any how i
continued for investigation and managment on the
right way
History taking
Patient has henoptysis .nasal block .ear block.joint pain
.hematuria and night fever and sweats .he lost 3 kg in 6
weeks i told DD vasculitis wegner granulomatosis .r/o
TB she asked about radiological finding in wegner and
managment it was not bad
Communication case was the worst
The patient is known case of rheumatoid on
methotrexate he recently has UTI for which the Gp
prescribed trimethoprim then he developed nasal
bleeding
Your role to discuss with the patient the plan to stop
methotrexate to control pancytopnia from erroronous
use of trimethoprim with methotrexate
He asked silly question
What is percentage of pancytopnia if used
trimethoprim with methotrexate
Is it absolute contraindication
He did not ask many about the ethics but he seems not
happy with my answers
I expect 4/16 in this case
Chest case was clear COPD WITH LOCALISED FIBROSIS
Abdomen jaundice anemia heoatospleenomegally ------
- Thalassemia
Then she asked if not hemolytic anemia what it could
be
The spleen was hugly enlarged so i told malaria
.leishmania .lymphoproliferative .i think i did well in
this case
Station 5 35 years old with typical chest pain lady
Smoker
Dyslipidemic with strong family h/o IHD
Brother and father on 50 age
I told admission as acute coronary syndrome
He asked if normal ecg and labs repeated over 24 hour
what u will do
I told send for stress echo or treadmell
Case 2 59 years lady with back pain since 3 days
After trauma
?????She is known case of artheritis
On prednisolone .methotrexate
For ladt 15 years
I examined the hand there was nodule on distal
interphalangeal joint .wasted hand muscles some
deformities i did not recognize then i examined the
back
He asked about hand signs and underlying disease i
told psoriatic arthropathy but it was z defirmity of
rheumatoid
However DD was right osteoporosis .r/o fracture
I wishb good luck for you all

Exam experience in Dubai 16/5/2016


station 5
yrs old man with dryness of his eye he has 27
exopthalmous with neck swelling diagnosed as hyper
thyroid on ttt with carbimazole and propranolol his
concern was what is the cause of his bulging eye as
people are commenting on this
yrs old woman with pain and swelling of both 40
hands family history of RA she is a secretary and on
examination she has swelling of DIP with no signs of
active synovitis and fair functional status she has also
knee pain and no skin rash her concern what is the
cause of this pain and swelling could it be RA and any
treatment she can't cope with her work
Then respiratory
middle age Lady with audible wheeze and on
examination vesicular breathing with prolonged
expiration most -COPD
Abdomen
hepatomegaly with palmar erythema two scars on the
abdomen one on the middle longitudinal and the other
is transverse on rt lumbar region when icommented
about the scar the examinar told leave it
History 50 yrs lady with cough and breathing difficulty
diagnosed 6months back as having breast cancer now
on tamoxifen on history no wight loss or night sweets
she has dry cough with dyspnea and paroxysmal
nocturnal dyspnea ll oedema no travel history not on
ocps husband died in accident and on son will be
graduated next year she is apart time teacher not
smoker not alcoholic and was concerned about that
she may has cancer and financial issues
Neuro
claw hand wasting of hypothenar and weakness in
ulnar distribution no scar at wrist or elbow no time for
testing sensation
Cardio
mitral and aortic valve replacement with pan systolic
murmur radiating to axilla in ayoung lady examinar
asked about target for INR and what other
investigation and what about if she is planning for
pregnancy
Communication 50 yrs old male dealing with pigeon in
his business with braethlessness and cough CT chest
high resolution with allergy test to avian ptn highly
positive so diagnosis of interstitial lung disease was
done and the task was to tell him about that and to tell
that he should avoid contact with pigeon and to start
corticosteroid for ttt

Myanmar ,,,Yangon
thday 2nd round4( 16/.3/.10
)
Stat 1 - pleural effusion, Thalassaemia
Stat 2 - breathlessness in RA pt taking Methotrexate
Stat 3 - Parkinsonism , MS
Stat 4 - oseophageal perforation d/t pneumatic
dilatation
Stat 5 - Neurofibromatosis with H/T
Vitiligo with Goiter
My experiences in old Yangon General Hospital, Day 4,
10.3.2016
diet1/2016
Station1
Lt Collapse Consolidation / fibrocavitory lesion
Etio TB Malignancy
Forgot to examine axillary LN
Luckily 20/20
Renal transplant AVF
DDx mass in RIF
20/20
Station2
Middle age female wheezing SOB increase in early
morning , night time cough, episodic
no sputum no blood, no palpitation,no leg oedema,no
syncope
H/o RA took methotrexate 7.5mg for 6 yrs
Salbutamol inhaler, steroid inhaler, rosedronate, folic
acid
DDx bronchiolitis obliterans
Lung fibrosis d/t methotrexate, RA
Pulmonary nodule
Bronchial asthma
Patient concern is it associated with drugs
Examiner asked about severity assessment, monitoring
and management of Bronchial Asthma
20/20
Station3
Middle age gentleman with difficulty in holding objects
O/E resting tremor
Bradykinesia
Rigidity ?
Dx Parkinson d/s
ddx ET
20/14
Middle age female
MS AF Pulmonary HT
20/15
Station4
yr achalasia doing oesophageal dilatation resulting 75
perforation
Previous 2 times ok
risk signed in document%5
Talk with anxious daughter
Is it Serious
Can discharge now
Want to discharge and transfer to other hospital
Further mgt
Why this happen
How to feed him
How long need to stay in hospital
She didn't want to tell him about perforation
Ethical principle
16 /11
Station5
BCC2
Skin rash with goiter in middle age female
Vitiligo+goiter m/b euthyroid
Sugar normal
No postural drop
Is it curable
Can her daughter get this
28/27
BCC1
Skin lesion with painful in gentleman
Neurofibromatosis
Is it cancer
Can his son get this
How treatment
28/16
Fortunately I have passed
This was my 2nd time
Thank you all my parents, teachers, and study partners
Fighting and best of luck! all the candidates in the
coming diets for PACES

Oman Med J on 13/10


I started with station 5
Case 1 paget disease
Patient with high alkaline phosphate and hearing
.difficulty
Paient on examination large skull typical paget disease
face have hearing aid. His main cincern was decrease
hearing . Legs were normal in examination and he did
good walk . I asked examiner that i want tunning fork
to examine rennie & webers test then one examiner
give me fork from his picket :) he deliberately hiding it
. from candidate to see that he asked for it or not
I got 28/28
Case 2 TIA with high BP RIGHT SIDED WEAKNESS AND
.DYSARTHRIA AFTER 30 MIN
BP 169/100. FEELS IMPROVED
This was simple case his ABCD2 score was high i
explained him his condition and needs of admission
28/28
Staion 1
Resp . Bronchiactasis young male . Examiner askwd me
causes of bronchiactasis in yiung patient . Then later he
start viva on immitile cilia . He asked about cilia
structure thanks God that i recalled my MBBS final tear
knowledge . I told him that on transverse section cilia is
look like wheel spoke and main defect is in demin arm
where LACK of ATPase enzymes which is nessesary fir
cilia to mobile . Then he asked me how many spokes
came out from cillia which i dont know and he told me
9 i got 18/20
Abdomen . Thelesemia with tender hepatomegaly and
has splenectomy . Sec hemachromatosis as pigmented
skin
Examiner asked me causes if hemachromatosis . I did
nt do well in this station missed hyperpigmentation as
pt dark skin . He had multiple abd. Scar i thought it was
chron's but then examiner give me some clue which
lead me to go with correct diagnosis 10/20
Station 2
year old female return from kenyia after spending 30
holidays started bloody diarrhea , abdomunal pain and
wt loss 3 kg 1 week history . Family hx of crohn dis (
uncle )
Conern was what happen to me
20/18
Station 3 cvs MVR
Examiner asked me indication of mitral valve rep. It
was simple case i did well 20/20
Cns muscular dystrophy
Vs motor neuropathy
This was difficult case i asked pat to walk but examiner
nit allowed me to do this on examination hypireflexia
hyoitonia generalized reduced power almost 2-3/5
sensory all intact . I told my finding and put my DD .
17/20
Station 4 young female admitted with rt. Sided
weakness all investigation normal seen by neurologist
and he say its functional . Patient known via nurse that
. doc. Saying its like this so pt. angry now . Talk to her
I explained her problem and apologuzed for that
incidence . Alsi discussed her lufe which was full of
problem she is unmarried only earning person at home
unable to pay house bills as earning nit enough .
Brother addicted nit doing any thing . She has problem
at work as nurse her duty 12 hours and she tired to do
these duties . Lit of issues . I tried my best to arranged
and solve her all issues. Concern what happen to me .
What action you will take against that doctor who told
that i am lying
16/11
Alhamdolilah i passed 149/172

Oman 11 April
I started with communication skills 29 yrs university
engeneer with ulcerative colotis on mesalazine with no
improvement 6 motions per day anemia with high ESR
to be started on steroids he is refusing bc of SE as he
read on
internet
Station 5 1st case 30 yrs acromegaly with bitemporal
hemianopia
nd pt with headache and blurring of vision diagnosis 2
from hx myathenia gravis
St1 chest bronchectasis
Abdomen renal tp with palpabe liver asked for single
diagnosis she has cushingoid feathers
St2 hx of patient with headache stress at work friend
diagnosed with brain tumor
St 3 cvs double valve replacement quite difficult the
metallic noise is not heard without the stethoscope I
am not sure about
Neurology as well hypotonia hyporeflxia nd depressed
sensation up to the umblicus they discussed Causes of
LMNL paraparese also I am not sure about
Plz pray for me and thank you all

Uk ,,,, Colchester Hospital


STATION 5
A.28 yrs old male admitted with diarrhoea and fever
.37.8.bloody no wt loss no other symptoms
Differential
Investigations
Management
.B 56 yrs old male complains of dryness in his eyes
.Apparent ptosis and miosis
.Lt Horner syndrome
.Left neck scar
What is a cause
How to investigate
.How to treat
?Concern is it reversible
Station 1
Abdomen
Failed kidney transplant
With AV fistula
.Questions straightward for transplant
Chest:left side pleural effusion and with skin lesions
.mycosis fungoides
What is the diagnosis
?How to investigate
?How to manage
Station 2
yrs old male with gait difficulty.has frequent falls 69
.and difficulty in getting upstairs
Its very difficult case.till I came to the drug history.was
prescribed prochlorperazine for dizziness by his Gp the
gait difficulty came after the medicine.then I went back
.asking about parkinsonism symptoms
All questions
?What are causes of parkinsonism
?Are you going to stop the medicine
?His concern could it be brain tumour
?How do you treat parkinsonism
?Drugs and side effects
Station3
CVS: ms with SBE
?Straight forward question
CNS: examine the lower limbs in the patient with gait
.difficulty
All finding consistent with lower motor
neuron.periphral neuropathy with features of mytonic
.dystrophy
Station4
lady admitted with SLE over night all labs normal 24
.except proteinuria
.Explain diagnosis and obtain consent for renal biopsy
?Concern she is worried about renal biopsy
She refused to do it.except in last 2 minutes when
checking understanding.I reinforce that is very
important because treatments are different to the
.stage.then agreed
?Questions whyneed to do biopsy
?Who will do it
?Complications
?Ethical isaues
If refused? I toldI will speak toher again after a while. if
?refused again
I will check competency if competent I have to respect
.decision
?.How to assess competency

Copied
PACES experience: was in the last day last cycle
.4/4/2016 in Khartoum center
:Communication Skills
I started with station 4 the scenario about a patient
who have achalasia and underwent a pneumatic
dilatation for the 3rd time but in this one he developed
.eosophageal perforation
It was mentioned that this complication can happen in
.5% of pts and the patient was consented
You will meet his son to explain for him what has
happened and the need for admission for 14 days and
.any issue raised by him
I started by the usual introduction and then checked
what he knows about his father condition then i
.explained for him what happened in BBN pattern
He asked why this happen to his father this time he has
done this procedure twice before.I explained for him
that any procedure has a possibility of bad effects and
it happens in a few patients; in every 100 it happens in
5 patients and no one can predict which one will be
.affected
He said do my father know this? i said any procedure
will not be done unless we explain for pt the benefit
and risk of it and let him to decide which is called
.consent and your father was informed
I told him that we need to keep your father in hospital
for 2 weeks but he refused. I asked why but his answer
was not clear for me but i proceed and explained to
him that this cut or perforation of his gullet will cause
leak of food and fluid to his chest and lungs and this
will cause damage and inflammation so that we need
to give fluid by his veins and medicines called
antibiotics and we need to involve our colleagues in
.surgery
Also i told him if he went home he may develop
complications and deteriorate more and i am sorry to
tell you that he may die . After this he agreed to admit
his father but he wants me not to tell his father i
replied to him this the right of your father to know
.about his condition
Then he kept silent and i asked him do you have any
other concern? he said no and still there is a time and i
wonder how to fill this time but fortunately while i am
thinking the examiner told 2 minutes left i summarized
for him and checked his understanding and thanked
.him
?Ex: what are ethical issues
Me: BBN,dealing with angry relative(realy he wasn't),
.doing no harm and autonomy
Ex: the son don't want his father to know what do you
?think about this
Me: i think this the right of his father to know to
.ensure ethical issue of autonomy
?Ex: any other principle
Me: i think we have to be honest and tell exactly what
.has happened
?Ex: how are you going to manage him
Me: monitoring
NPO
IV fluids
IV antibiotics
surgical consultation
?Ex: why you need to keep him NPO
Me: so no more food or fluid to get to mediastinum
.causing mediastinitis and allow time for healing
?Ex: what do you think the surgeon will do
Me: the management may be conservative or surgical
.but i am not sure of indication of surgery
.then the bell rang
Station 5
BBC 1:
young lady with deterioration of her vision in last 8
.months her vitals were ok
I started by open question then i analysed the visual
loss which was mainly at night and there was no eye
pain or headache and the course was progressive and
not episodic then immediately i asked about family
history which was positive her elder brother is blind
I proceed immediately to fundal examination to
confirm my diagnosis and i found scattered dark
pigmentation which was clear in the rt eye also i
couldn't appreciate the disc clearly after i finished
fundal examination i remembered that i didn't assess
her visual acuity i did it & was normal for finger
.counting
i returned back to the history and i asked about
associations of retinitis pigmentosa and other routine
parts of history
and i asked about driving
.which she is not
then i examined again for hearing aids and weakness
only bcz i thougt other associations were excluded by
history
then i asked about her concern? is she going to be
? blind? and what about job
.she was a teacher
i told un fortuanately this is a progressive disease and
till
now there no curative treatment but research are
ongoing and for her job she can continue as far as her
.vision can allow we can give some visual aids
the examiner asked about my diagnosis and the
.associations of RP
also what other areas you want to examine i said
cerebellar and peripheral neuropathy he said do u
want to examine her fingers i said yes for
polydactyly.then do you need to examine her visual
field i kept silent he said what do you expect to find i
.said tunnel vision
.then he took me the next pt
Station 5:
BBC 2:
A 28 years old male with skin lesions for several years
which are non-pruritic not painful and I expected it to
.be vitiligo
i started by asking its onset duration progression
distribution any starting lesion any aggravating or
relieving factors and involvement of mucous
membranes which were all negative then i request to
have a look. The lesions were raised small yellow
nodules on flexural part of the elbows
there was also another large one on his lateral
epicondyles and also in his back & eyelids and when i
.came closer to his eye i saw corneal arcus
it was clear this pt has xanthomas secondary to
hyperlipidemia then i asked about his FH which was
positive for sudden young death i asked specificly
about cholestrol problem he said no. then i asked
about macrovascular complications and the secondary
causes of hyperlipidemia ( DM,Renal
diseases,hypothyroidism,alcohol and primary biliary
cirrhosis) i asked about smoking and job
.then i examined his CVS which revealed AS
Lastly i asked about his concern which was is he going
?to die suddenly like his family members
I told him that these skin lesions are manifestation of
high cholesrol in his blood and this something run in
family and this high cholestrol harm your blood vessels
and this can cause heart attack and sudden death. We
need to do more blood tests and we will give you
.medicine to lower your cholestrol
Examiner asked me about my diagnosis what tests you
.need to do and how to manage him
Station 1
Chest:
The pt was comfortable, peripheral examination was
normal, Trachea was deviated to the rt and rt side was
depressed and moving less the percussion wad
heterogenous(dull+resonant) auscultation there was
fine end inspiratory crackles bilaterally but more on
.the right in upper zones
I presented my findings and said pt has bilateral apical
fibrosis mainly on the rt. Ex asked me about causes i
said most likely TB +other causes of bilateral apical
.fibrosis
.What investigations and management
Abdomen
A female patient with finger clubbing and functioning
.fistula + gingival hypertrophy
.At this point i was expecting a renal transplant
On abdominal examination there was no renal
transplant scar there was huge hepatomegally about
.14 cm bcm and splenomegally 4 cm
.Others normal
I presented my findings and i said the has
.hepatosplenomegaly and ESRD
?Ex: how to correlate them
Me : hepatitis on top of renal failure
?Ex(not convinced):what else
Me: amyloidosis
Ex: what else
Then i remebered the gingival hypertrophy and said
leukaemia by infiltrating the kidneys although it is rare
then the examimer seems to be convinced and asked
.me about investigations and management
Station 2 History
This was a difficult scenario of a young female feeling
fatigue for 2 months she went to her GP who found
high BP and have done some tests which revealed
.proteinuria and haematuria and normal RFT
I put differential of
CKD(stage 2 normal rft)
GN due to wegner's or goodpasture or post
streptococcal or IgA nephropathy
Polycystic kidney disease
Lupus nephritis
When i entered i analysed her fatigue and high BP then
i started by renal system then enquired about cvs/resp
including haemoptysis then i asked about URT features
sore throat nasal congestion epistaxis hearing
lmpairment then i asked about musculoskeletal skin
rash joint pain
All the above was negative
then i started to complete the other systems GIT and
neurology and i found that pt has loss of appetite and
non-specific headache then i asked about
.constitutional symptoms which revealed loss of wt
All
other parts of history were negative except she was
.taking OCP for menorrhagia for several years
When the ex told me 2 minutes left i have no idea what
?is diagnosis
I asked the pt about her concern she said could OCP be
the cause and i am planning to start a family does your
?treatment affect my future pregnancy
I told her i need to do more test to determine the
cause of her condion and it is unlikely for OCP to cause
high BP and the treament for your condition will
depend on the cause and there are different treatment
some of them may affect your pregnancy
.and others will not
?Then examiner asked me what is your DD
I told him about the one above
He asked me why you asked about nasal blockage i said
bcz i think of wegner's he said to me you mean
vasculitis as general i said yes but there is no skin rash
or joint pain he asked me could the high Bp be the
cause of her headache i said yes if it is malignant
he asked me how to know
i said i need to do fundal examination he said if it is
normal i said it is unlikely to be the cause of headache
he asked me what is the commonest presantation of
HTN i replied asymptomatic
?Ex: investigations
Me:CBC
Ex:what specific in CBC
Me: eosinophilia
Ex:why
Me:churg-strauss syndrome
Ex:how to manage vasculitis
Me:methylprednisolone and cyclophosphamide
Ex:is this may affect her pregnancy
Me: yes the cyclophosphamide
Ex:other anti-htn can affect pregnancy
Me: ACEI
Ex: other causes of htn
Me: endorinological like pheochromocytoma,
.....hyperparathyroidism
the bell rang
Station 3
CVS:
The pt was young all peripheral examination was
normal. Precordial examination was normal except in
auscultation there was systolic murmur allover and
radiating to carotids also there was early diastolic
murmur on lt sternal edge so my diagnosis was mixed
aortic valve disease and no one is dominant
.the examiner asked inv and management
Neurology:
The instruction was to examine the lower limbs it was
apparent that the pt has spastic paraparesis with
sensory level just below umbilicus the time finished
before i examine the back i told him i want to examine
the back
The asked about clinical diagnosis DD inv and
.management
Castle Hill Hospital..March 19
Station 2..chronic cough for 6 months in 25 years old
male + DM type 2+Hx of uncontroled
asthms+infertilty+constipation
Station 3 cvs..marafan with 2 scars and AVR discussion
about causes of chest pain in Marfan
Cns hemiplegia due to truma
PEGTube insertion in agressive agitated Alzehimer ..4
pt used to pull NG tube
doughter want PEG tube insertion tell her it is not ..
suitable + discuess palliative care in terminal ill
pt....how are you going to feed patient
morphea ..what is the causes of morphea.?.is it ??..5
cancer
..Lady with fatiguabilty and blurring of vision
Thyroid nodule +thyroidectomy scar +opthalmplegia
and exopthalmous
Station 1 .bilatral basal fibrosis +skin rash
??dermatomyositis
Abdomen..plycystic kidney +transplanted kidney +
abdominal pain
Discussion about causes of abdominal pain and
immune suppresion side effects
I am happy to say that I have passed. I took the examin
.in Castle Hill Hospital Cottingham 20th March 2016
Station 1: Respiratory; Middle aged, obese woman
with fine inspiratory crepitations more at the lung
bases. I reported that they were all over the chest as I
thought so. I was asked about differential diagnosis,
investigations and what I expect to see on HRCT and
treatment. I got 11/20. Abdomen; Young man with
right hypochondrial tenderness only. Differentials
included hepatitis, cholecystitis etc. I was asked if I
would discharge him if transaminases were mildly
.elevated, I said no. I got 19/20
Station 2: A 55 year old woman with a 4-week history
of weight loss, night sweats and joint pains. If you ask
only you will get a history of a tooth extraction 2 weeks
before onset of symptoms (History which I did not get).
I said Rheumatoid arthritis, lymphoma and vasculitis.
Diagnosis was Subacute Bacterial Endocarditis. I got
.10/20
Station 3: CVS; middle aged woman with
kyphoscoliosis, high arched palate and pes cavus. Had
AVR, no murmurs. I reported as AVR in Marfan's. No
murmurs. I got 20/20. Neurology: LL exam. Also a
middle aged woman with wide-based, high-steppage
gait. Had champagne bottle sign, pes cavus, distal
muscle weakness and stocking distribution of loss of
pin prick sensation. I picked an upgoing plantar on the
right, and for some obscure reason her joint position
was intact. Differentials were CMT and other
.peripheral neuropathies. I got 20/20
Station 4: My worst station and I really messed it up. I
was worried about my neuro case that I thought did
not make any sense, I thought it was a total disaster so
I did not concentrate and fully comprehend the
message I was supposed to give the patient's relative.
It was about a young man with metastatic colonic
cancer, who had massive UGB from duodenal cancer.
The team has planned arterial embolization for him but
his brother (whom I was to talk to) thought I should
just let him die. Meanwhile the patient himself wanted
surgery and lifesaving treatment. Up until now, I am
not clear about what I was supposed to say to him. I
.got 4/16
Station 5: BCC1; Ankylosing spondylitis. Staightforward
question mark sign, fletcher's sign etc. Asked about
investigation and treatment. I got 26/28. BCC2; Known
diabetic with blurring of vision. Fundoscopy showed
cottonwood spots and laser scars. I talked of a non-
urgent ophthalmology consult and tightening blood
sugar control to the patient. I was asked about
screening for nephropathy and neuropathy. I was
.asked if I saw hemorrhages and I said no. I got 26/28
Total 136/172
My exam experience
2016/4/2
Mater di hospital
Abdomen : splenomegaly with ascites for diffrential
diagnosis
Questions : DD, IX
Chest : left thoracotomy scar , aggressive shift of
trachea to the left side, air entry is diminished only left
. basal
I told the examiner : this shifting of trachea is going
with pneumonectomy but the air entry is diminished
. only in left basal which might be lower lobectomy
Questions : indication of pneumenectomy , he asked in
this patient what do u think the cause ? I said may be
cancer or suppurative lung disease because of clubbing
.. He said tell me only one possibility and why ? I said
cancer as the patient was cachectic and elderly , he
asked about PFT in this case : I told him mixed as the
patient might have compensatory hyperinflation also ..
I got 19
Cardio: double mitral and aortic regurge
Questions : indication of operation , echo findings
Neuron: examine upper limb
Short stature man with right upper limb deformity ,
examination of upper limb revealed some weakness in
right side , I examined lower limb showed spastic legs
more in the right side ...at that time I had a mental
block ..I considered that the patient has hemiparesis
and I told him the diffrential diagnosis of that including
stroke , he asked me about the treatment which I
answered ( stroke treatment )
I got full mark
History ; good case
younge male with family history of cardiac diseases
.presented with palpitation
Inside : father and mother died in their 70th due to
heart attack , history revealed only work stress and
excessive caffeine intake , other possibilities I ruled out
.
So my impression : was HOCM is less likely , mostly it is
stress related tachycardia
Communication
female , family history of cancer colon , presented 40
with cont diarrhea for the last 2 months , she is
worried about cancer colon , task is to address her
concerns
Initially I took a quick history which showed no any
alarming signs of cancer , father and brother and uncle
have cancer colon , the diarrhea mostly was due to
irritable bowel syndrome , so I assured her and I asked
her about colonoscopy before , she told me she had
normal colonoscopy 8 months ago , so my message to
her : cancer colon is less likely but in presence of
continues diarrhea and strong family history repeating
the clonoscopy after consulting a MDT will be advisable
...which seemed to be wrong as the examiner was
unhappy and asked me do you think that polyps will be
formed over 8 months only ?? I told him may be , he
told ..no it needs at least one year .. So no need to
another clonoscopy , then he asked me why you did
not inform her about the screening programme of
cancer colon ?9/16
Station 5
joint pain in younge female )1
Inside psoriatic arthropathy ( asymmetrical in the most
of the joints with psoriasis rash in the elbow)
I requested to examine the lungs for possible fibrosis
but he told no need , I requested to examine the eye
he told ok ..she has red eyes bilaterally ( ? Uveitis) th
examiner was very happy about that
Questions : patterns of psoriatic arthropathy
Treatment
repeated chest infection in old man )2
Inside : old man , with cough, sputum , clubbing , chest
infection recurrent since childhood , I auscultation the
back
D.D : bronchiactasis , he said what else could be ? I said
cystic fibrosis , he said what is the first possibility I said
bronchiactasis again , questions : causes
CT findings , treatment
Alhamdlellah I passed
My advise : extensive clinical practice , do not waste
your time in big books , cases of paces is enough for
clinical stations , you should have your approach for
. any medical complain
I would like to thank Dr Ahmed Ahmed Maher Eliwa for
his great efforts with me in history and communication
before the exam ...really I appreciated that unlimited
support from Dr. Ahmed
Thanks
Wish you all of the best
Wolverhampton,, UK,, new cross hospital,, 12
February 2016
history, collapse,, patient on thiazide & started -1
candisartan two weeks back / Cardio, instruction pt is
asymptomatic but referred by his GP,, I heard ejection
systolic murmur,, discussion about aortic stenosis &
sclerosis / neuro examine cranial nerves,, only
abnormality is diplobia on looking outward and
upward on both sides // communication,, pt with
essential tremor, carpenter diagnosed 3 years by
consultant, now concerns about Parkinsonism referred
by GP for deep Brian stimulation
station 5,,, fever in 27 year lady,, by history she had
lymphoma before,,, second case diarrhea,, I noticed
deformed nose,, finally its wegners plus diarrhea after
augmentin course for sinusitis abdomin,, HSM,, NO
stigmata,, plethoric. Copied
My Experience in Mater Dei Hospital Malta on 2/4/16
first carousel
I started with station 1
Chest : young patient with spares head hair( I Said
possibly 2 to chemo later on upon discussion and
actually I picked it up as I used to see this finding a lot
in my practice in oncology) ,RT side of the chest is
depressed and moving less, RT thoracotomy scar and
decreased chest expansion, impaired percussion and
dec breath sounds
Diagnosis: RT pneumonectomy
DD of etiology was bronchiactssis, fibrosis,Abcess and
malignancy
Discussion was about cancer causes in young patient
(germ cell, and Sarcoma )and workup also asked if he
developed SOB what might be the cause , I mentioned
infection and thrombosis
?How to investigate him
I got 20
Abd : middle aged male with features of CLD ( D
contracture, P erythema,thenar wasting and Tinge of
jaundice) and splenomegaly I said no asites
DD and work up
Honesty I felt that I missed hepatomegaly
I got 16
Hx: 50 years female , married , work as hospice nurse,
travelled to Kenya with her husband and came back
with nausea,vomiting, fever and upper and pain
radiating to back
Heavy alcohol intake
Had 3 miscarriages at Gestational ages of 26,28,28 no
personal or Fx history of VTE
Gp letter mentioned high bilirbin 70 and high all Liver
enzymes
? Concerned is it cancer
DD : I mentioned Alcoholic hepatitis, viral hepatitis(A)
and dengue,autoimmune hep, and malignancy
discussion was about working her up , and how to
manage, I mentioned that she needs admission, clinical
assessment and rehydration if dehydrated,pain control
and fever ttt with NSAID and avoidance of
acetaminophen and teat etiology
I emphasize on alcohol cessation referral
I got 20
CVS : old male has peripheral features of AR
apex displaced
Systolic murmur all over radiates to carotid
I said AS and AR although I didn't hear the diastolic
murmur , I was not comfortable to the auscultatory
findings and I felt may be something is missing, anyway
, they discussed with me what might be the causes of
systolic murmer in this age and how to differentiate
between AS and sclerosis, investigations to do
I got 20
CNS : middle aged patient
Instruction was : this patient has problem lefting
objects
I examined his upper limbs , he was sitting on a chair ,
he is non English speaker however examiners helped
with instructions and I passed few instructions in
Maltese my self( most of them sounds as in Arabic)
Findings are pure proximal atrophy and weakness at
shoulder girdle and scapular muscles with defined
supraclavicular and scapular margins, no facial
involvement
DD : proximal myopathy likely congenital causes as
patient has an atrophy
And I suggested scapulohumeral variant I enlisted few
other causes as well
Investigations including EMG,NCS, and muscles biopsy
He asked me about mode of inheritance I answered
that I can't recall
Management is supportive and I motioned that few
Novel therapies is under study
I got 20
Communication: speak with angry son of 70+ female
admitted initially in orthopedic ward with # femur and
underwent arthroplasty 2 weeks ago , 1 week after she
felt while doing rehabilitation, since this last fall she is
on and off confused, orthopedist assure son that this
because of UTI and she is receiving ttt for that , then
patient transferred to medical ward as her confusion
continues, CT scan arranged , showed intracerbral
bleed with midline shift, neurosurgery advised to hold
enoxparin ( which was started as prophylaxis) and her
usual aspirin and stop her oral feeding until the see her
Role : speak with son about CT findings and
subsequent plan and discuss the clinical judgment
when outweighing benefits and risk of LMWH
Son was angry but I listened to hem empathetically and
reassured that I'm here to help, I broke the CT findings
and explain the role of Neurosurgery opinion, his
concerns : what is the cause of her bleed, why giving
anther blood thinner while she is on ASA , could the fall
?be avoidable, why he has been told that she has UTI
Actually examiner's discussion revolved around
whether LMWH has caused her bleeding or not and
wether there is a way to know that I said unlikely it
was the direct cause however above therapeutic level
of anti factor Xa might give a clue that helps to reveal
.the uncertainty of her bleeding cause
I got 16
BCC 1 : old male , c/o slurred speech for 30 minutes,
three previous episodes of near fainting , during
. episodes he feels "fluttering" sensation of his heart
PMHx : HTN on amlodipine 5 mg , AF on pacemaker
and warfarin 3 mg and regular check, ranitidine for
gastritis
Exam : AF with rate of 80
Discussion was about DD
I mentioned TIA , orthostatic hypotension
How to investigate, he ask me will you change his anti
?hypertensive or not
?How do you know if pacemaker is non functioning
I got 28
BCC 2 young lady, pregnant in 18 weeks gestation with
SOB for 2/52 and cough with occasional whitish
phlegm and occurs at late night and early morning,no
any other symptoms upon discussion
KCO bronchial asthma was controlled before
pregnancy on INH SABA & INH steroids but she
stopped them both after got pregnant as She thought
they're harmful
Examination: all clear , LL clear
I explain for her the role of inhaled Mx in controlling
her asthma and that why she got these sympx ,
reassure about safety in pregnancy, adviced PFM diary
and FU with GP
Discussion: DD chest infection and less likely PE
Examiner asked what've s against infection, also asked
? if PE need to be rolled out what to do
Actually I peaked my marking sheet within the
examiner hands while pill was ringing and I'm about to
leave the room with all marks in satisfactory area , I
felt it was a comfort message from Allah at the end of
the exam
I got 28
Over all I scored 168
My conclusion that PACES is a MOSIAC experience, it
concludes different roles and various methods and the
probability of passing lies in practising as many as one
.. can do of these roles and methods

:My exam in Khartoum center day 2 cycle 2


Station 3
:CVS
A female SOB. Her pulse is small volume & regullar.
Loud S1 & S2. Pan systolic murmur heard all over not
.radiated to axilla
?Ex asked: what r your finding and D
I said l have DD pulmonary HTN functional TR, or VSD
.. P HTN
.Ex: is the murmur go to axilla? I said no
Ex: what about MR? I said it is one of dd but the
murmur not radiating to the axilla, l added i also want
to assess for MS because of loud S1. Ex asked is there
any murmur of MS? I replied no then asked how to
invx and tt and what are the causes of primary
.. pulmnary HTN
)I got 18(
:Neuro
A male e difficulty in walking pt had wasting proximally
e pus cavus, flaccid weakness, abscent reflexs and
normal sensations. l examined the upper limbs which
also shown proximal weakness e reduced tone & no
:reflexs bell ran :bell::bell
The examimer asked about +ve finding then he asked
what about coordination :grin: l told him that it is
difficult to assess in LL & in upper L time not allowed.
Then asked about each DD what with and what against
..then invx & tt when I mention muscle biopsy he asked
what do you find in the biopsy? I dont know he smiled
..and said this is pathologist level
)got 19(
Sation 4
The senario of delayed diagnosis of
pheochromocytoma. We have done this senario e dr
Imtithal
.so I was happy by getting this senario
I started as dr zein tough us e greeting pt, agreeing the
.agenda and if any one e him and ICE
The surrogates's main concern is negligence and
delayed diagnosis for 5 yrs. I said first I'm sorry for the
delay
for his suffering during those yrs and the good thing &
now we know the cause and try to help him
regarding negligence l said I doesn't know the situation
at that time but I am going to check and give you feed
back ..he asked when you will give me the feedback? I
told him I am not the one who do this, I will inform my
consultant and special office in the hospital and they
will check the records and sit e your GP to know the
situation, then they give you feedback ...he agreed
then I talked about his disease and the possibility of
malignancy ..he didn't respond
I thought he did not hear me so I repeated it again, he
said ok ..then we discussed the manegment plan and
he asked about the effects of high BP during those 5
yrs. l told him l can not tell now & I need to assess him
for bad effect of HTN and do some tests then i
summarized and checked his understanding ..the nice
surrogate left the room
Ex asked about ethical issue , is there any negligance ,i
heard you telling him it is difficult to diagnosed why , l
heard you telling him you wont to assess for effects of
HTN what did you wont to do , i heard you telling him
the surgery is high risk why then he asked about tt ..I
felt they are satisfied
)I got 16(
Station 5
BCC1: A male patient with h/o blackout all vital signs
.are normal
I explored pre during and after the attack. So it is
epilepsy. The surrogate said also he worried about a
...rash in his face ..+ve FH of epilepsy
O/E tge pt has adenoma sebaceum. I checked for focal
neurological deficit.. None. Then i checked for
subungual Fibroma +VE ex the mouth for high arch
palate and bifid uvula _ve. Abd for Palpable kidney -ve.
Then I wanted to do chest & CVS examinar said normal.
.. I examined the fundus & it is normal
He was concerned about the cause & whether it will
.affect his kids
After I answered the concerns the Ex said you still have
one min, so l council him about the driving issue and
tell the plan again ..ex ask about D & assosiations ask
about what I wont to check again ..yes i said shagreen
batches and ashleaf :ok_ I got 28
BCC2
A 55 female e Progressive lt hand weakness. In history I
asked about duration ,progression other hand
,sensations and the LL ...i found it bilateral carpal
tunnel :blush: PH of hypothyroidism on tt I examined
.the hands then the thyroid and thyroid status
Her concerns are what is the cause & whether gonna
be paralysed ? ..again I finished 1min earlier.. so l went
.back and explained more
:Examiner's questions
.. causes of carpal tunnel, inves & ttt
)I got 28(
Station 1
Chest.. by inspection there is a scar so i hurried to
reach the back the scar is strange i got confused. Later
when i presented my findings i said the trachea is
central the ex asked me to check it again then i said yes
it is to the lt he said do u mean to say this from the
.start, i said yes
Then the DD is pneumonectomy , fibrosis , collapse
Then the rest of discussion is about fibrosis , he was
...not happy & I thought I failed this station
)I got 20(
Abdomen. The pt has abnormal movements in her
.hands
So during the general ex I kept looking for the
instructions. I presented my finding CLD there is lymph
node e scar of biopsy ex asked me to show him the
node then he didn't comment & I ignored it. Examiner
asked what about the movements? l said it could be
..flapping tremer he said if not? I replied chorea
.DD of CLD
I mentioned Wilson disease, so he said if i told you this
pt has Wilson, how are you going to investigate. Also
.. he asked about the tt
)I got 18(
Finally I ended e history station
A 36 yrs female e fatigue. GP found iron dificiancy
anemia other tests are normal. Examination is normal
......
no history of blood loss in M&S she has knee joint pain
D as osteoarthritis. I asked about how she diagnosed
she said by her docter and was diagnosed .. MRI and
she was on NSAID for 2 yrs wt loss 5 kg and epegastric
pain. The fatigue afectted her job & life alot and she is
concerned about fatigue. I told her there are many
causes from simple PU to serious Ca so we need to
consult the gut doctor & do upper GI endoscopy also
need to stop the NSAID and review her joint problem e
.joint doctor
I summarized then they told me 2 minutes are left.. so I
.checked understanding and summ again
The ex asked about DD what is the most likely one..
. plan of tt & invs
during Iam Ansures the other examiner told me I heard
you telling her about other pain killer tell me about
them.. he also asked about H pylori, the best diagnostic
test & why ,ttt & follow up, the OGD finding and when
we took a biopsy. Then he asked the other ex if he
wont to ask me any Q he said I think nothing more we
.finished the Qs
)I got 20(

Exam on 2/4/2016 day 2 cycle 1 in soba university


hospital
I started by station 2 history
Scenario of (35 years old lady has fatigue for 6 months
her gp did a blood test and confirmed to be iron
deficiency anaemia )
I introduce my self, explain the role ask about her job
... she is a teacher and agree the agenda
As she has fatigue I start by analysis of her fatigue and
then general symptoms and she give hx of wt loss of 5
kg when I ask about joint pain as apart of general
symptoms she tell she has joints pain for 2 years and
she had been diagnose to have osteoarthritis by
orthopaedic consultant
Then I asked if she use any medicine for that she tell
she use 2 medicine ibuprofen and other NSAID the
medicine was given without PPI cover. She has hx of
localised epigastric pain made worse by eating ass with
nausea but no vomiting.... some times heart burn
.there was no melena or haematemesis
No mouth ulcer
No change in her bowel habits
.No bleeding through her back passage
On bloating and no tummy pain with specific type of
food ( wheat products )
Normal menstrual cycle
She take balanced diet and she give me example for
her diet
.Then review of her systems was negative
Her past hx and family hx is negative
I take the drug hx as part of HPI
In social hx she is affected greatly by her fatigue and
also she can't do her hobbies as she use to run and go
to gym
She concerned about the cause of her fatigue and how
.can I help her to do her hobbies
I explained to her the likely cause of her fatigue related
to medicine which she is taking and that we need to do
cammera test and we need to stop her medicine after
discussing this with her orthopedic surgeon and if we
need to continue on it we will give PPI and we are
going to give her iron replacement and that I will reply
back to her gp
Then I check the understanding
And thanks her
First examiner question
Did you ask about smoking I said no sorry
do you think it is important I sa
I said yes as pt most likely has gastritis or peptic ulcer
disease so smoking impaired the healing of the ulcer
: Then did you ask about alcohol
Again I forget
??So do you think it is important
Yes as it may cause gastritis
Then he ask why you ask about numbness and
???unsteadiness
Because if malabsorption is cause then B12 may cause
subacute combined degeneration of the cord
Then ask about DD
I put gastritis
Gastric and duodenal ulcer
Malignancy as other has wt loss
Then coeliac disease and IBD
he asked whether NSAID CAUSE small bowel ulcers
apart from duodenum? ?? I said it is not common but if
multiple then we need to think of zollinger elisson
syndrome
???How can endscopy help
Macroscopic we can see the ulcer and we can take
biopsy
???What to test in biopsy
The presence of malignant cell and also H.Pylori
Any relation between NSAID and H.Pylori? ??
:sweat::sweat::sweat: I said I don't know
Then how NSAID cause peptic ulcer? ?? After explain
then again he ask any relation between NSAID and
?? ?H.Pylori
I feel that he need me to say yes so I tell yes there may
be a relation
Then how you will treat H.Pylori? ?? I said triple before
give the name of triple he tell if you stop NSAID what
other medicine you will give to the pt I said
paracetamol
I think they will mark me negatively as I forget ****
important part of social hx but surprisingly I got 20
Then station 3
..... CVS
young female with small volume pulse and she is pale
She has chest deformity ... active pericardium with
visible pulsation the apex is not displaced with
palpable 2nd heart sound and positive lt parasternal
heave and on thrill
There pansystolic murmur in lt para sternal border
with maximum intensity in the apex but no radiating to
axilla wit loud 2nd heart sound
I present my case as MR with pulmonary htn then the
examiner asked whether th murmur radiate to axilla or
not??? I said no and the the murmur is in lt parasternal
border so it is differential of TR ,MR,VSD with
pulmonary htn
Then about the causes of pulmonary htn and
investigations
I got 20
.... CNS
young male with stick beside the bed
There is pes cavus and wasting of both leg with
hypotonia and weaknesses of LMNL but proximally
more than disatlly
Abscend reflexs and equivocal planter
Coordination difficult as power grade 0
Intact sensation
I said the gait he tell no need
Then I examine the upper limbs with same finding
?? ?Ex what is you positive finding
??? What is your diagnosis
I said LMNL weakness either muscle problem or pure
motor PN but with pattern of weakness proximal I will
go with muscle disorder then asked about how can gait
help you and DD of pure motor neuropathy then
.investigation and management of proximal myopthy
I got 20
Station 4 communication
Scenario of delaying a diagnosis of pheochromoctoma
in young male suffering for 5 years seeing by many
doctor including psychiatrist for panic attacks and been
prescribed diazepam and also has htn that difficult to
control and on HIS INSISTENCE the go refer him to you
clinic and you are the doctor in hypertension clinic...
the tests done for him show mass of 5 cm in his RT
adrenal and urine test also positive
Your task to explain for him the diagnosis and to
...answer his concerns
I stard by
Introducing my self explain the role ask about his job
and any one he would like to invite to attend the
meeting and then agree the agenda & ask him to tell
me more he was attacking in nature... that he is
suffering for 5 years and seen by many doctors and
prescribed sleeping pill but without any improvement.
I showed embathy regarding his suffering for 5 years
the explain to him that the result with me now and
that unfortunately it is not as we hope then telling that
it show pheochromoctoma and whether he hear about
it he say no the if he would like to explain more .... the
I explin pheochromoctoma and telling that the good
news that we find cause for your suffering and it is
curable condition in majority... then explain the cause
behind is a growth... he is not care about the growth
whether it us cancer or not but I explain to him the
possibility of cancer of 10% when I tell him it is curable
he tell how I tell surgery then he is habby an tell OK
just removed now (verbal cue) I think then I tell it is
not easy surgery we need to control you blood
pressure first as I am a doctor in hypertension clinic
then I am going to involve MDT he tell what MDT I tell
sorry a team of expert people including the gland
doctor, the surgeon and anaesthetist they will make
meeting and they will decide
?? ?He ask when the will decide
I tell as soon as possible
Then his main concern where there is negligence or not
I tell I need to go back to your record to see what
exactly done for you. Then he tell doctor there is
negligence and I will complain against my gp....I tell it is
...your right to make a complain
The he concern where there is a damage happen to
him from HTN? ?? I tell l need to examine you and to
.do some test test see the effect off htn
...I asked any other concern
He tell no then make summary ,,, check the
understanding and telling I will reply back to your gp
.then offer help and leaflets and if he can drive alone
The British examiner
What did you think about this case???then I give
summary of the case and what I did
Why you did not tell him about risk of surgery??? I tell I
just brake bad news for him and I dose want to give
him all bad news and he will have metting with
surgerical team who the will discuss with the risk of
surgery( then he smile)
???How you will treat his htn
Alpha blocker and then beta blocker
???Do you think there is negligence in this case
I give him the same answer for the surrogate then he
ask is there is any damage from his htn???I tell the
same that I need to examine him and to do fundus and
.investigation to see if he has damage
Then he ask again any negligence??? I notice that both
the surrogate and examiner concerning about damage
from htn then I tell you pt suffering for 5 years and not
diagnosed this is not usual and if there is damage
happen from his htn OF COURSE there is a negligence (I
Don't know why I said of course )
then he ask do you think the pt is habby and he can
drive alone??? I keep silent for while then tell yes he is
habby
The bell rang
The British examiner tell well done
I got 15
Station 5
BCC1
years male with blackout with normal vital signs 50
From hx blackout mainly in the morning and during
sleep also I get confuse how during sleep the a very
nice surrogate he tell he bite
His tongue and wet him self
I asked any shakes he tell no
I review CNS which is negative then asked about
trauma which is negative the about general symptoms
...including skin rash
He tell yes he has skin rash in his face for 20 years
which difficult to fade away... I look to the face and
...then feel relaxe as I catch the dignosis
Past hx of htn and on amlodipine 5 mg with no change
in the dose
Family hx of abdominal surgery in 2 of his sister and
skin rash in his brother
He is a teacher!!!! and I asked about his school
performance he tell good and he drive a private car
.and dose not drink alcohol
I examined the face for the rash and the doing pronater
drift for any weakness
Check the trunk for ash leaf spot and examine the back
for shagreen batch
Offer abdominal examination he tell normal offer chest
exam he tell normal offer to check BP tell 130/70 then
offer fundus for phakomas he smile and tell Do it
I check quickly as the pupil is not dilated I know that
.nothing well be there
The concern about what is going on ... I explain
tuberous sclerosis
...Another concern about his kids
Explain that each has 50% chance to get the disease
No other concern
The examiner tell still you have 1 minute
Then I explained the eplipsy and driving but still there
is time then I explain the screening of family and gentic
.counselling
Examiner question
?? ?What is your diagnosis
Investigations and management
I got 28
BCC2
years female with lt hand weakness 55
Before shaking pt hand it seem she is on pain so I tell
sorry I will not give you shake as you are in pain
Then surrogate give hx of both hands pain in disruption
of median nerve without any thing in the system
... review point to the cause
However in oast hx she diagnosed to have
hypothyroidism and not on follow up for tow years but
using her thyroxin 100 mcg
No other significant hx
O/E
clear signs of carpal tunnel syndrome bilaterally and
more in the lt
Then I checked for thyroid status
Concern about the cause of the problem
Other concern whether her lt hand will become
paralysed
I admit the 2 concern and explain the need to check
her thyroid status and regular follow up and we may
need to do surgery
The British examiner still you have 1 minute
Then again I tell the importance of regular follow up
The examiner smile still you have 30 second but no
.... problem you can regulate you thouhts
Then ask the dignosis the DD
The investigation and management
I got 28
My last station is station 1
I start by abdomen
Young female looks ill and very pale with cannula in
her RT arm with ting of jaundice and no stigmata of
CLD with hepatosplenomagly
I examine only for one group of axillary LN as time did
not allow me
Then ask about DD
the discussion about myeloproliferative disorders
I got 19
The chest
Amiddle aged male with obvious depress lt side and
moving less with very strange scar on the lt side only
the tip of scar is seen anteriorly so I try to go fast to
examine the back from anterior the trachea deviated
to lt with impaired percussion on lt and decreased air
entery on lt and vocal resonance however there is
increase vocal resonance in lt upper part and bronchial
breathing in same lt upper part
Posteriorly same finding and that scar stii confusing me
it look like long thoracotmy scar but surprisingly there
area about 2 cm of normal skin
So I present my case as lt pneumenoctomy
Then he ask did hear any thing abnormal in lt side I tell
increase vocal resonance I am afraid from inventing
sign so I did not tell bronchial breathing but he is very
helpful examiner and ask me what type of breathing in
lt upper zone then confidently I tell bronchial breathing
so he tell what could be the cause again I tell this could
from stump .... he smile he tell stump can not cause
this... he ask what could cause increase vocal
resonance and bronchial breathing I tell cavity ..... he
tell yes now what could be the cause I tell
fibrocavitatory lesion ...he becomes habby and asked
about the common cause and how to investigate TB
I GOT 20
Station3 Cardiology
Young lady with heart murmur ..advised to examine
.with limited exposure
All through examination were normal. I did all possible
manuevre,but could not find any particular
abnormality. So i said i could not find any significant
CVS abnormalities. Examiner did not challenge me.
Asked me suppose this lady has a very faint systolic
?murmur. What will be d/d
I replied physiological murmur. Then asked causes of it.
I gave a long lists of causes. Then any congenital heart
disaease? I replied ASD can cause flow murmur in pul.
Area, vsd can cause pan.sys.mur , but it is usually loud.
Then asked any other condition young female can go
with long time without any problem? I replied mitral
valve prolapse. Now examiner got the answer what he
was wating for. Then he asked me signs of right heart
failure, signs of pulmonary hypertension, why TR
occurs in pulmonary hypertension. Then time over. At
the end of examination, i found all of us said normal
cvs
started with station 3
CVS- MVR- midline scar , metallic sound, MD Murmur
examiner discussed about infective endo- how present,
wht investigation
20/17
Neuro- diffic. to walk- examine neurologically-- Rt
hemiplegia with facial palsy- where lesion? invest?
.acute presenta-- Alteplase
20/20
station-4
Open TB- sputum positive- wants to visit abroad
mother got stroke- adv to start treatment then after 2
wks recheck- if negative then allow-- contact trace- did
not tell side effect of anti-TB -- 10/16

station 5
BCC1- headache with visual loss-- surrogate told bump
-Rt side
examination-- Rt Homo Hemianopia. previous
unconsco history- d/d-- ICSOL, MS, Stroke,, inves,
26/28
BCC2- Hands- Small joint pain- stiffness > 1hr, h/o
Psoriasis 3 yr back, nail changes present-- Exam-- no
active inflammation, only nail change-- Examiner- nail
change- d/d- psoriasis or fungal, d/d- Psoaria or RA-
Investgation of Psoriatic. 28/28
Station 01
Abdo- anemia, jaundice, Hepatosplenomegaly-- 45
years age- CLD with Portal HTN- D/D- lymphoproli,
Malaria, Thalasse(age not supportive)- cause CLD,
Invest.-- 16/20
Respir- Rheumat hand with Fine creps- ILD-- D/D- MTX
induced ILD, investi, Rx.--- 20/20
station 02
female 32
bloody diarr 4 wks, visited Cyprus. low back pain with
years stiffness-no fever , no wt loss- grandfather 57
colon cancer-- D/D- Inflammatory(IBD) or Infective- but
i do colonoscopy, examiner asked to exclude cancer
Invest of Infection, IBD- Rx- do it on OPD basis. 20/20
Total 157/172
Thanks to all

Experience of our collegue Nagwa Mahmoud


St 1. Chest copd.clubbing Basal fibrosis . Chushing
features ask about finding. Cause of fibrosis .Inv. and
management.got 16 And. HSM. QUICK. Cause inv. Ask
about upper GI. Endoscopy in this case and ttt.if the
cause is HCV. Asked if it works? Bill rang.got 20
History bloody diarrhea and arthritis .history of travel
to miraco
History of long term diarrhea and recurrent and pain
relieved with defecation. FH. Cancer colon in grand
father aged 75 no other family members has cancer
colon . Cancern is it cancer like my grandfather.
Questions DD. Infective diarrhoea. IBD. .Asked what his
risk to develop cancer colon told like others asked if he
need admetion I told I after examining and doing basic
inv. U&E we will decide asked about inv.and
management of Infective diarrhoea . got 20. St 1 neuro.
Ms. Pyramidal weakness bilateral more in line. Side
loss of sensation in rt. Side till face cerebellar signs.
Time finished befor doing deep sensations actually
when he told one minute remaining I did cerebellar.
.... Told I wand to do
Discussion about DD inv. TTT. got 20. Finally
cardiovascular . Double aortic with AR. Predominant.
.Q. Finding . Causes. Inv and management got 20
I hope my exam experience help you all . My advice is
to concentrate at least 3 months befor exam and to
have studying partner for history and communicatios
also to make study group. In yr work to see patients
.and discuss
Good luck for all and thanks a lot for our colleagues
.who shared their experiences befor
+
.St4
A female pt about 70 known bronchial asthma that's
difficult to be control till recently. She was admitted
with congestive heart failure and was controlled on
diuretics and ACEI. Today one of the junior doctors
prescribed bisoprolol as he thought it is of benefit for
her heart Failure & a nurse gave her the ttt. Since 30
minuts no harm happened till now but the pharmacy
told it is harmful to her to be given bisoprolol as she is
asthmatic. So the nurse was worried and pt. feel that
. something went wrong
You are asked to speak to her and explain the
..condition ...very long scenario
After introduction I checked understanding &
explained what happened, apologised & explained
what will be done incident report, department meeting
..and put under observation for any SE. And follow up
by cardiovascular and chest team . Her concern why
happened what will happen to me what will u. do to
prevent this happening to others . Ist she was angry
but after explanation and apologies and stress on her
care and postpone discharge for one to two dayes till
. we are sure she is ok. she is satisfied
Examiners ask legal issues I told negligence but is not.
It is mistake he told any thing else told autonomy - I
should tell not to do harm also he asked why bisoprolol
is harmful in BA. We are giving small dose- he told- I
told as it is non selective b blockers asked what can we
give i told carvidilol . Asked how u do incident report I
.explained . I got 13
. St 5 #
Sudden loss of vesion pt. Hypertensive By history -1
last less than one hour plus hemiplegia. She is on
insulin and bisoprolol only . Ex. Pulse AF. Carotids
asked to examined precordium examiners refuse .
Examined visual aquety simply asked for fundus they
refuse asked to examine her neurologically told ok. I
checked power. Was normal . Her concern. Is it
dangerous. Why happened? Questions about positive
findings. DD. Management .got 26
St 5. 2nd case Active RA . Discussion about inv. ttt.
Components of multidisciplinary team for this case.
Got 28
.St 1#
Chest copd.clubbing Basal fibrosis . Cushingnoid *
features asked about finding. Cause of fibrosis .Inv. and
management.got 16
Abdomen. HSM. Discussion about Cause inv. Ask *
about upper GI. Endoscopy in this case and ttt.if the
cause is HCV. Asked if it works? Bill rang.got 20
History bloody diarrhea and arthritis .history of #
travel to Morocco
History of long term diarrhea and recurrent and pain
relieved with defecation. FH. Cancer colon in grand
father aged 75 no other family members has cancer
. colon
.Concern is it cancer like my grandfather
Questions DD. Infective diarrhoea. IBD. .Asked what his
risk to develop cancer colon told like others. asked if he
need admission? I told I after examining and doing
basic inv. U&E we will decide asked about inv.and
.management of Infective diarrhoea . got 20
Neuro. Ms. Pyramidal weakness bilateral more in #
right Side loss of sensation in rt. Side till face cerebellar
signs. Time finished befor doing deep sensations
actually when he told one minute remaining I did
.... cerebellar. Told I wand to do
Discussion about DD inv. TTT. got 20
Finally cardiovascular . Double aortic with AR. #
.Predominant
Q. Finding . Causes. Inv and management got 20
.I hope my exam experience help you
My advice is to concentrate at least 3 months befor
exam and to have studying partner for history and
communicatios also to make study group. In yr work to
.see patients and discuss
Good luck for all and thanks a lot for our colleagues
who shared their experiences before

:My exam in Kuwait 23/4/2016


: Station 4
Young lady with DM since 30 yrs on insulin, she has hx
of retinopathy before and on regular visit to her GP she
found to has proteinuria and GP started her on ACEI.
her HbA1c is 20.. your task is to discuss the result of
..this recent urine test and the management plan
I started my discussion with raport and i asked her to
tell me how much she know about her condition, and
she start to speak for more than 4 minutes until the
point of initiation of ACEI by the GP which she didn't
take it coz she don't know why given to her ( she said
the GP not explained to her for what this drug). So
then i told her iam here today to explain for you why
this drug for.. i asked her about DM in details when
diagnosed and if she is taking her ttt regularly and
complications. I discovered that she is taking her ttt
regularly BUT the problem is the follow-up.. she is not
going to clinic in a regular basis . Now is the main issue
which is the SOCIAL Problems.. i asked her is there any
thing prevent you to come to clinic? I asked her
specifically is there problem at work or at home that
makes you busy? She was separated from her husband
recently and also she is taking care of her old bed
bound mother and her kids that why she is not
following.. i showed her some empathy and sympathy
and i said it is good to take care of others but also not
forget your self.. i told her the importance of
controlling DM and consequences which may happen if
not controlled and here i explained why GP start ACEI..
her concerns are : heart attack, stroke and if she will
. .end with renal failure and dialysis
I said to her iam not here to make you afraid but all
these can occur that why is importance of taking ACEI
and to follow to prevent all these complications. I told
her i am here for help and i will involve the social
worker to help you.. then i summarized the meeting
and i checked her understanding and she is happy
..about the plan
Then the examiner asked me firstly what do you know
about autonomy ? And how to know she followed your
..advice? I told by follow-up if she came
16/16
Station 2
yrs female with Episodic weakness in Rt upper limb 35
..and last one yesterday which stayed for 10 minutes
After starting with raport ask her to tell me more.. she
has 4 episodes all for less than 10 minutes.. otherwise
all hx negative including systems review.. she has past
hx of HTN during her last pregnancy and she didn't
follow after and now her BP in the referal letter is
170/100. also she is smoker.. l asked about CVS and
rheumatological hx and family hx all negative.. no
features of MS (i put this in my mind coz of episodic
?weakness ). Her concern is it brain tumor
. Then i discussed the management plan
She asked me what about driving and i forgot the
duration exactly so i said the phrase of my friend who
is faced the same question ( iam not sure but i will go
now to the book of DVLA in my office and i will tell you
..) but i said it is better to inform the DVLA
Examiner : investigations and management.. Also
..about ABCD2 score
.20/20
Abd: CLD pt with jaundice, ascites , splenomegally,
.palmar erythema
.Examiner : investigations and management
.20/20
Neuro : paraplegic with sensory level. 16/20
Chest : bronchectesis
20/13
CVS : AR and MS pt also has AF
20/17
Station 5 Acromegally
yrs with headache and HTN 30
..He has typical features
.28/27
nd is ankylosis2
..yrs with back pain 25
.He has morning stiffness and uvitis in the hx neck pain
..O/E.. only mild restriction of spine flexion
28/26

Malaysia
--) 2016-4-17 (
res -Marfanoid guy with bronchiec, abdo renal
...transpant
...hx was IBD with joint pain
cvs i also donno wat....cns peripheral sensory
..neuropathy
...bcc was takayasu and PDR
comm phaeo late diagnosis

My exam was in Sabah hospital


I started with ST 5 BCC 1 was this pt presented with
...joint pain
He has pain in both wrist elbows some times his right
knee &swelling stiffness for half an hour..no skin
changes no photosenstivity no mouth ulcer or eye
changes no muscle pain or weakness ..his wt is some
times increasing &decreasing..systemic review is
negative
Not diagnosed before but for several years he is taking
azathioprin & steroid..no FH ..Menimal impact on the
..daily living
OE Indian surrogate with normal exam..iDid all hand
exam including hand function carpal tunnel
rheumatoid nodule proximal myopathy &I offer to
.. examine knee
Examiner question what is the diagnosis..Rh.artheritis
not active as no synovitis now stiffness less than
hour..how to investigate ..what is sign in x Ray for
Rh.arthritis..how would u manage this pt..I answered
all these question however I said according to test if
disease is active to add methotrexate examiner was
not happy about this I think may be because the pt has
history of hepatitis which I missed..also he told me you
didn't screen about side effects of steroids..I told him I
asked about wt and muscle problem & I examined for
proximal myopathy but he told me u should ask about
the others..I got 20/28
BCC 2
yrs old lady with rt side weaknesses..in history 50
started suddenly continued for 2 hrs then
resolved..first episode..no speach or visual problem no
alter sensation..no headache ,fits or Loc..no history of
trauma..no cvs symptoms esp palpitations no joint pain
..or skin rash
Known hypertensive no other vascular risk factors..no
history of AF..negative FH..social history including
impact is unremarkable..she is on OCP..anti
.hypertensive medication
OE iexamine for pulse,listen 4 carotid bruit examine
conjugate eye movements ,power in upper limb ..offer
..to examine cvs & fundus
Examiner questions what is problem with this lady
..what are risk factors for Tia in this lady ?isaid only
hyper tension he ask me if OCP is risk factor..isaid only
if pt has thrombophilia or connective tissue disease or
history of migraine with aura..how to investigate..how
to manage..of course based on her ABCD score..he
asked me about management of stroke..I got 28/28
It is really difficult to start with station 5 esp if this is ur
first attempt..but ithink we should try to control our
nerves as this is most imp thing& alhamdullah I gained
my confidence by second case
Station 1 Abdomen
Young man who is pale & jaundice in absence of
stigmata of CLD & prominent zygomatic bone..he has
venous ulcer in rt leg.. With scar in his back has
hepatospleenomegally no ascitis ..my diagnosis was
haemolytic anemia mostly thalassemia. he asked me
about what other differential how to investigate
including HB electrophoresis & bone marrow &
managment..I got 20/20
Station 1 chest.. I think itis same case as dr Mazin
amale who is excessively coughing with finger clubbing
has all signs of hyper inflation of chest including
decreased cricosternal notch hypersonant all over the
chest except in rt base vesicular with prolong exp &
coarse crackles mainly in rt base with little change by
cough as pt is excessively coughing..my diagnosis was
Copd & bronchiectsis.how to investigate..what r signs
of Copd in X-ray what r signs of bronchiectsis in xRay
&ct scan..what is expected PFT & management of both
conditions..if he came acutely with sob how to
manage..you said control oxygen why.. I got 20/20
Station 2
..Young lady with palpitation
On taking the history exertional mainly ..with no
adverse symptoms ..started n ended suddenly ..no
other cvs symptoms her wt is decreasing with normal
appetite...she feel hot but no other symptoms of
hyperthyroidism.. She just deliver 4 month ago with
amaenorrea since that time.. No skin changes or eye
changes..known asthmatic using salbutamol
inhaler..she used it once in month as her symptoms r
not so frequent ..FH of premature heart disease her
mom age 50 her brother in his 40th.. her dad also..her
sister has hypothyroidism..so here I emphasized with
her n I asked her if she checked her cholesterol as Iam
concerned as u have strong FH of heart disease so she
said no so I offered appointment for this..SH she said
she is not having any stresses in her life n taking
caffeine n alcohol moderately she is only in
salbutamol...so I asked her if she had idea about the
cause she said Iam afraid itis a heart attack I told her is
this ur concern she said I have lots of concern ..is it a
heart attack (I clearly said itis unlikely as she has no
chest pain no sob..Is it from my salbutamol inh (it is
unlikely as u r not using it so frequent..)..what ami
having... What r u going to do 4 me..I answered her n
..reply back to GP
Examiner asked about diagnosis post partum
thyroiditis.. Other deferential is post partum
excarbatation of graves which is unlikely as no neck
swelling no eye or skin changes...other causes of
palpitations is excluded from history..examiner said I
like the way u took history ...asked about
investigation & treatment.. I got 19/20
Station 3 cvs it was obese lady who refused full
exposure..her pulse is large not collapsing.. APex to be
palpated..systolic murmur all over pericardium also
radiated to root of neck...I knew at that time pt has AS
as murmur radiated to root of neck..but pulse is large
so she has regurg valve is it double aortic but no
diastolic murmur or MR..so at that time I said iwill
mention only AS I will say may be there coexistant
AR..n murmur in apex is from gallaverdine
phenomenon...examiner was angry ..he said if I heard
other murmur I said no so he ask me about cause
investigation n treatment ..I knew I did bad in this
station...after exam I met the pt n I asked her about
diagnosis she told me itis MR..she has AS but not
sever...I got 10/20
Cns instructions:examine this pt who has difficulty in
walking...isaid to my self if is tarted with gait I will
waist my time as he cannot walk beside bed as itis to
small he will take about 2 minutes so I started with
general inspection n started with lower limb he has a
reflexia even with reinforcement..down going
planter..loss of superficial sensation in stocking
distribution..difficulty in coordination while pt closing
his eyes which may indicate sensory ataxia..I didn't
finish examinations time finish when Istart to do
vibration....so I told I want to finish my examination
by examining gait...as if he was not listening examiner
asked me what is instructions why u didn't start with
gait...I apologize 4 that...what is diagnosis I told
peripheral sensory motor neuropathy..what causes
imentioned commonest...how to investigate...how to
treat...then British examiner said if itold u this pt is
having positive Romberg will it change ur diagnosis
...imentioned in that case it will be PN& dorsal column
affection..so most likely SACD of cord asked about
treatment ...is it reversible or not... I remember the
answer from dr Ramadan...surprisingly I got 18/20
Station 4
It took me more than 2 min to control my nerves after
which I thought bad performance ....this is 50 yrs old
man diabetic for more than 20 yrs admitted before
with MI under went catheter n was put on all anti
ischemic including plavix n aspirin during hospital
admission his HB was I think 9 before that it was 11 ...it
was mention clearly no action was done for this n he
wasn't given Aplan or referral regarding u were not
apart of treating team now referred by his GP with HB
7 gm...long scenario...after introduction pt immediate
shout was angry he told me he found out that his HB
since admissions is decreasing no one informing
him...he was discharged home without
appointment....no plan what to do...so I apologize n I
admit negligence...n i told him endoscopy usually will
not be told only after6 wks from mi ( which is as
iremembered mention by one college in the group)...so
he calm down n asked me about cause ...itold him
avarity of causes ranging from less serious one like
soreness in tummy to might be celiac disease as u have
DM for long time to more serious causes like cancer
which cannot be excluded now...so itis better to do
endoscopy upper & lower... Imentioned the details of
procedure & benefit & risk which might exclude hage
esp he is on dual antipaltelet..so we need to stop it
around a week before the test... He told me that he is
having sob now he is really tired...it was verbal cue
that he is symptomatic so I offered admission for blood
transfusion esp he is recovering from recent MI we
don't want to put a load in to his heart with this
anemia ... He was afraid he might catch some infection
so I assure him regarding that.. I summarized then I
asked him if he agrees about endoscopy n blood
... transfusion...he said yes
Examiner questions were what is cause of Anaemia in
this pt...is there is angligance n why... I told him usually
we r not doing endoscopy after MI only after 6 wks
however the pt was discharged without been informed
about result not given Aplan for
investigation...examiner agreed...asked about ethical
issue .... When to stop aspirin n plavix before
endoscopy...asked about chances of having infection
after blood transfusion.i got 16/16

Oman 13/04
St 5
Constipation in young man, father died with cancer
colon, by history polyuria , flank pains , hypoglycemic
episode , most likely MEN1
Second case gynecomastia , by examination
acromegaly vs kleinfelter
Chest bronchiectasis
CVS mv replacement
Neuro flacid quadreplegia , no sensory affection, not
sure abt the diagnosis
Abdomen renal tx with audible graft bruit with
functioning avf
History back pain and bowel incontinence in pt with
h/o lung cancer
Communication delayed diagnosis of
pheochromocytoma
Oman
Royal Hospital
2016/4/12
COPDand CLD -1
yrs old female has h/o Diarrhoea wt:loss smoker 40-2
,no family history malignancy
young male AS & Transverse Mylitis -3
COPDpt admitted with pneumonia and he got one fit -4
and theophylline level was high and pt was on
clarithromycin
Pt asked I will complain and Su it dr y not before level
done at admission time
Tuberous Sclerosis and Gynecomastia -5

Oman PACES, 14.4.2016


FROM ARMED FORCES HOSPITAL MUSCAT
Stn 4. Astrocytoma high grade to inform his wife he #
is confused decision taken for pallative menegement
.Stn 5 #
Case no 1 blurring of vision with exesive lacrimation *
discovered from history thyrotoxicosis on treatment he
looks local cause
Case no 2 uncontrolled HTN recently started on *
?? ?valsartan ? RENAL ARTERY STENOSIS
St4 1 #
COPD +lung fibrosis *
hepatosplenomegaly+ascites *
Stn 2. TIA #
Stn 3 #
Parkinsonism (disease) *
Mixed aortic valve *

Oman,round2, Thu14/4/2016
St2: tiredness in uncontrolled DM
St3: Cvs: AS+/-MR
Cns: mixed picture of LL weakness- MS
St4: father underwent pneumatic dilatation with
.perforation.Talk to the sun
St5: scleroderma
Gynaecomastia
St1: chest:Old+bronchiectasis + Lt thoracotomy
scar...very bad case
Abdomen: hepatosplenomegally +shifting dullness+ Rt
iliac fossa mass

-- Hospital kuala lumpur, malaysia 1st cycle


Station 1: respi : Sob , marfanoid with left lobectomy
.causes of lobectomy and treatment
Abd : hepatoslplenomegaly with no CLD features
Thalassemia / CML /ALL/infection
Management of thalaseemia
Station 2: young lady with headache and transient
weakness of upper limb ..d/d hemiplegia migraine /
TIA/vasculitis of brain
Station3: CVS : loud murmur at mitral systolic and
diastolic ..apex can't feel at all as too fat lady ..There is
systolic murmur over the aortic radiates to carotid
..initlly I said mixed mitral , Dato chandran very nice let
me listen again ..is mixed aortic valve ...asked causes ,
.. manamgenrt of AS
CNS : young lady with spastic hemipareiss no sensory
sign with cerbellar florid bilateral
Diff/MS , SCA , FA
ix and management of MS
Station 4: talking to patient son as patient post hip
replacement and given anti coagulation ..fall at ward
and confused , CT scab show bleed in brain ..address
concern
:Station 5
Old lady with vitiligo and presbet with dementia
Causes hypothyroidism secondary to non compliance
of Thyroxine reduced reflexes
Differential : b12 level deficient
Management : take blood and examine..help the
patient with Meds as alarm and also discuss with
family members regarding taking care of her as she
lives alone
gentleman presbet with frequent fall ...DM with )2
sensory ataxia and also DM retionpathy which
examiner expect us to do fundosocpy
Having Charcot joint
Optimized sugar and refer occupational and podiatrist
..foot ware
Refer Opthal
What is gold standard testing for sensory ataxia :
monofilament 10g and NCS
.. Noted tractional fibrosis AT eye
Cases Oman 11 April
communication skills 29 yrs university engeneer with
ulcerative colotis on mesalazine with no improvement
6 motions per day anemia with high ESR to be started
on steroids he is refusing bc of SE as he read on
internet
Station 5 1st case 30 yrs acromegaly with bitemporal
hemianopia
nd pt with headache and blurring of vision diagnosis 2
from hx myathenia gravis
St1 chest bronchectasis
Abdomen renal tp with palpabe liver asked for single
diagnosis she has cushingoid feathers
St2 hx of patient with headache stress at work friend
diagnosed with brain tumor
St 3 cvs double valve replacement quite difficult the
metallic noise is not heard without the stethoscope I
am not sure about
Neurology as well hypotonia hyporeflxia nd depressed
sensation up to the umblicus they discussed Causes of
LMNL paraparese also I am not sure about
Plz pray for me and thank you all
---Calcutta 3rd cycle
Sta.1 - hepatospleenomegaly with anemia
.rheumatoid lung ds .
Sta.2 -conversion disorder
Sta.3- ms or ar
examinations of cranial nerve with rteye laterus .
rectus palsy with vertical diplopia and resting
nystagmus in lft eye
Sta.4-- theoohyline toxicity with clarytromycin
Sta.5-- dm with lft lateral cutaneus nerve palsy
another case lft side stroke with MVR on warferin & .
.palpitation

-- Calcutta today 3rd cycle


sta.1 - abd - CLD with ascites
resp - ILD in rhematoid hand
sta. 2 - confusion in old man with prostatic carcinoma ,
talk to son
sta. 3 - cvs - MVR
neuro - spastic paraplegia
sta. 4 -rupture esophagus after pneumatic dilation for
achalesia cardia ,talk to son
sta. 5 - a ) fibrosis of lung
b) DVT

Kolkata April -2016


rd cycle3
Station 5 - MS - rt hemianopia with optic atrophy with
h/o rt sided weakness
Station 5 - ? Ankylosing with ILD/COPD - wheezes as
well as crepts - little odd
STation 1 - Chronic MR with ?? AR - you really had to
strain for the AR murmur, no peripheral s/o AR - seem
like a clear cut MR
Abdomen - hepatosplenomegaly - likely
hemoglobinopathy
Station 2 - Wegner's /PAN/Autoimmune with
Glomerulonephritis
Station 3 - Rheumatoid Lung - fixed hand deformities
with effusion vs collapse
Neuro - Rt LR palsy with rotatory nystagmus - likely
brain stem lesion ? vascular vs others
Station 4 - convincing patient regarding oral steroids
for ulcerative colitis

Oman
April 11, 2016
st 4
Communication skills
A 29 yrs university engineer with ulcerative colotis on
mesalazine with no improvement 6 motions per day
anemia with high ESR to be started on steroids he is
refusing bc of SE as he read in the internet
Station 5
st case 30 yrs acromegaly with bitemporal 1 -
hemianopia
nd pt with headache and blurring of vision 2 -
diagnosis from hx myathenia gravis
Stn 1
Chest bronchectasis -
Abdomen renal tp with palpabe liver asked for single -
diagnosis she has cushingoid features
St2
Hx of patient with headache stress at work friend
diagnosed with brain tumor
St 3
CVS double valve replacement quite difficult the -
metallic noise is not heard without the stethoscope I
am not sure about
Neurology as well hypotonia hyporeflxia nd -
depressed sensation up to the umblicus they discussed
Causes of LMNL paraparese also I am not sure about

My exam cases in Kilmarnock, Glasgow college


Abd: multiple scars including liver transplant and renal
transplant plus polycystic kidney plus widespread
melanoma like lesionsplus b/l dupytrens contracture. V
difficult to palpate as pt wz markedly tender
Score 14/20
Resp: COAD with bronchiectasis yellow thick nails
Score 20/20
Cvs: mixed aortic valve disease predominant AS
Viva went on diagnosis, severity classification(new
AHA) bad prognostic markers, ind n timing n types of
surg options
Score 20/20
Cns: rt homonymous hemianopia with rt sided
weakness( command was to examine the vision)
Finished in 3 mins, offered fundo, did a quick cvs exam
including pulse n carotid n surg scar on scalp
Stil had 1 min examiner stopped me(said no more
reqd)
Viva: causes, emergency management including invest,
protocol for thrombolysis according to guidelines, long
.term management
Aftrr this examiner said my Qs r finished but u still hav
7sec left n i turned to the second examiner for further
viva but he just smiled n waited for bell
Score 20/20
In all these cases the most imp thing is our confidence
in examining, picking right signs n giving ur 1 diagnosis
without being shaky
History: 50 yr male with blackouts
Diagnosis: vasovagal syncopy
?Viva: wht makes u think of this diagnosis
I said the imp causes of syncopy in this gentleman r
vasovagal considering his presyncopal symptoms of
flushing heart racing n immediate recovery, however in
background he does hav Afib which is asymptomatic
well controlled so i wil investigate that as well
Pt also had 1 seizure like movement as well
The next Q wz on dvla
How wil u manage: addressed all social issues involved
with the risk of injury to himself in his job and at home
as well
Examiner's remark while smiling: "excellent, plz can i
have ur notes" n bell rang
Score 20/20
St 4: amiodarone induced lung injury
Went v smoothly
Viva: ethics involved
Management of underlying arrhythmia? I mentioned
the AHA guidelines on stepwise approach
With few recent updates at which examiner again
smiled
St 5
Bcc1: abd pain with htn
Diagnosis neurofibromatosis with pheochromocytoma
Viva: r u not surprised there is no family history, i said
it can b a new mutation but now his next generation is
at risk
Next Q: u were v keen to know abt his hearing n
?balance u confirmed it twice y
Ans: i wanted to rule out central tumors in particular
cp angle schwanoma acoustic neurona meningioma
Next Q: u mentioned to him these r non cancerous
then y did he hav 3of them removed (scars were
present)
Ans: they can always increase in size, for cosmetic
reasons, impingement of underlying nerves n v v
unlikely but may b sarcomatous change
?Next Q: u offered him some scan of tummy y
Ans: for 2 reasons: NF is assoc with renal artery
stenosis and pheo which is the likely cause here
So wil get doppler mibi scan n urinary levels
He wz asking me some other Q regarding vision but bell
rang i just said the causes of eye involvement in NF in
2-3 points
Score 28/28
Bcc2: diarrhea n racing of heart
No other signs of hyperthyroid found
Complex external opthalmoplegia on exam
Past his of thyroid prob
No cause found
No AI diases present
Not driving not smoking
Concern : eye symptoms of grittiness redness
Explained him it could b ur thyroid again but ur eye
prob is persisting from ur previous thyroid issue which
we can help in several ways
?Viva: wht r ur findings? Eye findings
Apart from thyroid wht other causes can u think of
?exophthalmos
Ans: u/l causes
B/l causes
?How wil u manage
Ans: Thyroid
Eye
Social support
?Q) Wht can b the causes
Wht other causes u had in mind while u were asking
facial flushing n then htn
Ans: carcinoid, pheochromo anxiety but none were
found n he has obvious exopthalmos With
opthalmoplaegia
Bell rang at this
Score 28/28
:Overall summary
Pass AH
Score 166/172
This is the exams of ur clinical skills on background of
sound medical knowledge, the ability to keep ur nerves
well controlled, all of this comes with repeated
practice on ur patients and do alot of talking practice! I
mean ALOT! So that in actual exam u keep talking to
pts and examiner in a fluent yet unhasty speed. This is
a true rapid fire exam. Give it ur best shot with
adequate practice especially of ur weak points. The
exam is not difficult but tricky, mark urself on the real
marking sheets while u practice with ur friends to
!master ur skills
Best wishes for all
Dr Qurat ul Ain Amjad
MRCP(UK)

Calcutta yesterday first cycle


.Had TB for communication
Station 5 had facial nerve palsy and man with small
.joint problems
Station 1 had bilateral bronchi ecstasies with rt upper
lobe consolidation mostly TB and
man hepatomegaly with splenectomy, mostly
.thalassemia
Station 2 had bloody diarrhoea with recent trip to
.Cyprus, mostly infective or Ibd
.Station three had an AS and Potts paraplegia

:Khartoum PACES, Day 3 last cycle


Station1
:Chest
.Lt Apical lung fibrosis
:Abdomen
..Polycystic kidney
:History
HTN in a 25 years old female..on two occasions. .RFT
..normal. .protein and blood in urine
Station 3
:CVS
Mixed Mitral valve with P.HTN
Station 5
Retinitis pigmentosa 1
Familial hyperlipedemia 2

Khartoum PACES, 3rd April 2016


cycle 3
:Station 1
Abd: HSM+Lympadenopathy
Chest: Left u lobe fibrosis + pleural effusion
Station 2
.. Chronic diarrhoea + abnormal LFT ALP high (UC+ PSC)
Station 3
CVS: AVR
CNS: LL Examination pt. with hypotonia mute planter
& hyper reflexia
Station 4
BBN & Councelling female 37yrs with ESRD (the one in
the course)
Station 5
yrs male with Rt arm weakness501
Neurofibromatosis compresses the ulner nerve. The pt.
underwent surgury for removal of fibroma recentely
A 53yrs male with long standing arthritis present 1
..with dysphagia
have gritten eyes (sjogren) and have bibasal lung
fibrosis & is on methotrexate

Experience of my Friend,,get his Exam in Malta


2016/4/2
Cardio double mitral
Neuro : I did bad, examine upper limb. Right side
hemiplegic posture with mild weakness of no specific
pattern, left upper limb is normal, he told me to
examine lower limbs which was spastic in right side
with extensive planter , he told me that no need to
examine sensation as all are normal
For diffrential diagnosis
Communication : also was bad, 40 female with
diarrhea 2 months ago, father and brother with cancer
. colon
Inside: no reason for diarrhea except ? Irritable bowel,
she had normal colonoscopy 9 months ago
The examiner was tough , I said she has to do
colonoscopy again because the it was 9 months back
and the diarrhea 2 month ago but he was unhappy ,
also he asked me why u did not mention cancer colon
.screening programme
Station v
joint pain ..inside psoriatic arthropathy )1
recurrent chest infection : inside bronchiactasis )2
Abdomen : splenomegaly with ascites for diffrential
Chest : thoracotomy scar with deviated trachea ? But
there was some breath sound on upper and no
bronchial breathing of the stumb.. ???
Pneumenectomy
History .. Palpitation in young male with family he of
... cardic disease
Inside : the father and mother died on 70ths, no hocm,
mostly stress , caffeine related
Over all impression : I missed a lot due to my own
mistakes but the centre and most of the examiner are
.helpful

Experience in Malta Centre


second day , carousel 1 16/4/2
: ABD
splenomegally with CLD
: Chest
.Pneumonectomy in a young patient with alopecia
Hx: fever, upper abdominal pain , nausea and high
LFT in a returner from Kenya
CVS: AS /AR
Neuro : proximal myopathy (?congenital)
:Comm
A lady post Hip # and arthroplasty & on prophylactic
LMWH, fall down during physiotherapy and developed
.intacerbral bleedind
:Task
She is confusion , speak with her son (angrey)
:BCC 1
AF, pacemaker possibly non functioning presented
.with slurred speech for 30 minutes
:BBC 2
..A pregnant lady with SOB for 2 weeks
:Hx
A 50 years female , married , works as hospice nurse,
travelled to Kenya with her husband and came back
with nausea,vomiting, fever and upper and pain
radiating to back
Heavy alcohol intake
Had 3 miscarriages at Gestational ages of 26,28,28 no
personal or Fx history of VTE
Gp letter mentioned high bilirubin 70 and elevation of
..all Liver enzymes
? Concerned is it cancer
DD : I mentioned Alcoholic hepatitis, viral hepatitis(A) ,
.autoimmune hep, and malignancy
Discussion was about working her up , and how to
manage, I mentioned that she needs admission, clinical
assessment and rehydration if dehydrated, pain control
and fever ttt with NSAID and avoidance of
acetaminophen and to treat etiology
.I emphasize on alcohol cessation
:Communication
Task: To speak to an angry son of a 70+ female
admitted initially in orthopedic ward with # femur and
.underwent arthroplasty 2 weeks ago
One week after she has fallen down while doing
rehabilitation. Since this fall she is on and off confused,
orthopedist assure her son that this confusion is
.because of UTI and she is receiving ttt for that
Then, the patient is transferred to the medical ward as
her confusion continues, CT scan arranged & showed
intracerbral bleed with midline shift. The neurosurgeon
advised to hold enoxparin ( which was started as
prophylaxis) and her usual aspirin and to stop her oral
.feeding until the you see her
Role : To inform the son about CT findings and the
subsequent management plan and to discuss the
clinical judgment when outweighing benefits and risk
.of LMWH
Son was angry but I listened to him empathetically and
reassured that I'm here to help, I broke the BN (CT
findings) and explained the Neurosurgeon's opinion
:His concerns are
what is the cause of her bleed *
why giving another blood thinner while she is on ASA *

could the fall be avoidable *


?why he have been told that she has UTI *
BCC 1 : old male , c/o slurred speech for 30 minutes,
three previous episodes of near fainting , during
. episodes he feels "fluttering" sensation of his heart
PMHx : HTN on amlodipine 5 mg , AF on pacemaker
and warfarin 3 mg and regular check, ranitidine for
gastritis
Exam : AF with rate of 80
Discussion was about DD
I mentioned TIA , orthostatic hypotension
How to investigate, he ask me will you change his anti
?hypertensive or not
BCC 2
A young lady, pregnant in 18 weeks gestation with SOB
for 2/52 and cough with occasional whitish phlegm and
occurs at late night and early morning, no any other
symptoms upon discussion
K/C bronchial asthma & is controlled before pregnancy
on inhaled SABA & Inhaled steroids but she stopped
them both after got pregnant as She thought they're
harmful
Examination: all clear , LL clear
I explain for her the role of inhaled Mx in controlling
her asthma and that why she got these sympx ,
reassure about safety in pregnancy, adviced Peak flow
diary and FU with GP
Discussion: DD chest infection and less likely PE
Examiner asked what is against infection, also asked if
? PE need to be ruled out & what to do

Castle Hill Hospital


Station 2
years old .DM.asthma presented 25
With recurrent chest infections for 6 months 6 times
I put DD bronchiactesis .TB
No Hx of fever .wt loss or travel or contact with pt with
chroinc cough.he has greenisg sputum..constipation..I
did not understand his accent clearly
He continue mentioning constipation and trying to
have a baby and I totaly ignore it..his concern why I
have this recurrent infection
His diabetes and asthma are not well controled I asked
about HIV risk which up set the examiner
I forget sinusitis and examiner was angery and
.heampotesis as well
I told him we are going to do bronchoscopy..also upset
the examiner
He asked me about d ...my dd was bronchiactesis and
TB
He asked about one blood test for specific for
bronchiactesis
I told I do not remember
.. He said serum antibodies for pathogenes
I was about to say immunoglins but bell rang
.CNS: Upper limb exam .3
He has hemiparesis
I did not finish sensation
Not examin e nech
He had truma with scar in head which I did not notice
.even when examiner point it
He ask me if you notice any facial asymetry I said
no..which acutaly was present
:CVS .3
A tall women I wasted time looking for alchol gel for
scruping and washing hands with water
Marfan syndrom with 2 sacrs on medisternotomy scar
with metalic clikc and aother an rt subcalvicukar..no
muremur but 2nd sound was loud and palpable..first
was soft
My d..aortic valve replacemtn
He asked about causes of chest pain in marfan
I told ACS
And pneumothatx he asked what else which I can not
answer
She had high arch palate and archenodactyly..I think by
other cause of chest pain he wants rupture anyuresm..I
just remember it now
:Communication skills :4
Staion 4 ...80 years old patinent..Alzehimer d...was on
NG feeding and she was agreesive and agitated all the
time and use to pull it out..her doughter facing
problem with feeding and want PEG tune insertion
..speak to her doughter and explaine ill_terminal care
...and palliative care for her
I do not now mentioning DNR waa suitable or not but I
..have mention it
Examiner asked about how are you going to feed her if
.. sh will not take oraly no NG no PEG tube
:5
Station 5 was diffecult
years with skin lesion over her forhead and scalp 60
Looks like morphea
Some candidate mentioned SLE
Apart from that she did not have any manifestation of
scl
? eroderma ..her concern is it a infecious
?Is it cancer
I reassure her ..but examiner asked what could cause
morphea
Second case 62 years old ..with blurring of vision
.exssive fatiguabilty..and more blurred by the end of
the day..deffintly she had exopthalmous and
opthalmobligia..diplopia on both lateral
gazes..thyrodyectomy scar and left firm thyroid
nodules
Dry hard skin..fundus normal..no other manestation of
..thyroid ..no proximal myopath
I told dd
Graves opthalmopathy and
Mysthenia graves
:Station 1
chest bilatral basal fibrosis and skin rash..I do not
now what is it...some candiadte examiners told them it
is dermatomyosistis..it was not typical she had hard
.skin..finger tips ulcer as well
Abdomen...abdominal pain
I could apprecaite 2 masses in rt side and one mass in
left side not liver not spleen...it was transplanted
kidney ..examiner asked why she is going to have
? abdomian pain
?What about immune supression side effect
...Examiner questions was more tough than the exam
But it was nice experiance
..Keep praying for me

weird st 4 experience uk center:( friend of mine)


task: talk to this lady who is 40 years old,she had
referred by the GP to cardiologist for complain of chest
pain.her cardiac markers,ecg,cxr,stress echo everything
came out normal.your cardiologist is not present in
hospital and you as medical registrar been asked to
explain the findings to patient and tell her the pain is
.non cardiac
Hello, is this Mrs.Walker? hi Mrs.Walker ,my name is
Dr.Vincent and i am the medical registrar.i have been
.asked to speak with you regarding your test results
?first of all please tell me how are you feeling today
is anyone accompanying you or do you want anybody
to be here while we discuss? she is like no doc,i am just
!wondering what is the problem with me
i can understand that you are bit anxious about the
present situation, please tell me what do you know so
far?has anyone explained you about your present
?problem
no doc i just went to my GP few days back with this
horrible chest pain and he immediately sent me to the
heart doctor and he did bunch of tests and something
called echo and i don't know why i am having chest
!pain? i think its heart attack,what do you think doc
well,i am very sorry about that( making sad face and
concerned look ,as if i am going to get an oscar award
for this performance),let me assure you i am here to
help you and i am going to explain you what has been
happening ,okay! you have told me that you think its
heart attack,do you have any specific reason to believe
!this,can you share with me that thought
oh,you know doc ( sobs!........i am silent,offer her a
tissue,sobs....i am just waiting,suddenly crying spell is
.....)over( i feel sigh
you know doc,my mum and sis both had heart attack
and died coz of that and i am damn sure i am also
!having same issue
oh Mrs.Walker ,its sad to hear that and i am very sorry
that your mum and sis had to go through this
turnmoil,do you mind telling me a bit about their
health in general ,i mean did they smoke,drank alcohol
excessively or had history of high blood pressure
oh yes,my mum was heavy smoker and my sis was
alcoholic
and at what age you mum had heart attack ---at age of
70
and your sis-55
.again i am sorry about that,Mrs.Walker
Mrs.walker i understand that the pain you have been
having in left side of your chest is been there from 3-4
.days ,am i correct? oh yes dr
.and its always there? oh yes
and do you smoke and drink alcohol,no doc.i dont do
any of those and i exercise regualrly
i know we shouldnt take history but i was just (
confirming and trying to reinforce the thought before i
.tell her its non cardiac
.thank you mrs.walker
well we have done extensive blood tests as well
imaging of your heart and all tests appear to be normal
!and thats excellent news
isnt it? what do you mean doc? my tests are normal?
?there must be some mistake
Mrs. walker i understand your worry and concern
,however the tests are thoroughly verified and it
appears that the pain you are experiencing is not
!because of heart attack,i am quite sure about that
hmmmm doc! so you think there is anything else ,any
?other test can be done to be hundred % sure
Mrs. walker ,as a doctor i can reassure that we have
done every possible and relevant test in your case,and
everything leads to this outcome that the pain is most
.likely due to muscle injury in chest
!rest assured,its not your heart
!!!!!!!silence
also mrs walker you have been leading a very healthy
life style,you do regular exercise,stay away from all
sorts of toxins, and these factors do protect your heart
from developing any heart ilness, are you following
?me
yes doc ,so thats why my GP told me to not exercise for
(2 weeks
bam!!!!! i wonder ,why the hell i didnt ask her this
before but any way she has spit this info to me ,now i
am sure its costochondritis,but i cant take detailed
)history here
!yes Mrs.walker i believe so
would you mind telling me hows things at home lately
?! specially since the pain started
!well i have been feeling low! my appetite become low
hmmm! do you have nay hobby? yea i used to do
gardening but i am not doing any more from last one
.month! i also get less sleep in night
have you had any bad thoughts lately? what do u mena
.doc? i mean any thoughts of self harm........ hmm no
okay,these experiences may also be culprit in causing
you the pain you are having? what do mean doc ,am i
?going nuts
No Mrs.Walker,its nothing like that,you see our body is
very complex and sometimes our brain does creates
symptoms inspite of no obvious reasons,however in
your case your pain is real
but just to rule out any psychological reason ,i would
like you to have a chat with our psychologist today,if
!you are willing
!okay,we can do that doc
so let me ask you ,can you pls tell me what we have
?discussed so far
.she summarise it briefly and appears to be okay now
okay ,at this point do you have any question for
?me,Mrs.walker
?oh yes,doc shall i resume exercise after 2 weeks
well Mrs walker i will re-asses you after a week and see
how you have progressed and then we can take it on
?further. does that sound like a plan
oh yes doc ,thanks a lot
examiners were real jerk( see i am South asian but the
south asian examiners are worst,specially the one in
England)
?first question 1,how can you say its non cardiac
i feel puzzled!! i guess thats what my task is,to (
convey the message to patient in proper way that its
)..... non cardiac pain,i felt like
any way i answered him," sir ,as the all test were
normal as well she didnt have any significant risk and
both her mum and sis developed heart attack after the
age of 50 which is not risk factor,this patient has
healthy life style as well the pain is persisting from 3
days,its most likely costochondritis or functional
pain,to rule out one need detailed history however
here i am bound to stick with communication and
)ethical concern only
.nd qs2
?what are the ethical principles involved
Autonomy
beneficence
non-maleficence
justice
rd3
why did you tell her that you will get a psych consult?
arent you able to deal by yourself,you are also a
!Doctor
i said,yes the reason why i need a psych consult is
,initial depression screening does suggestion some
element of occult depression and its wise to get
professional help,as my current role is cardiology
.registrar
Now what he does is completely insane,he tells to
other examiner ," i think you should also ask some
"questions
i am like what the ****,ideally only one examiner (
)leads
anyway that guy asks me ,do you think patient was
?convinced?did you resolve her all issues
yes sir,i believe she did. as she agreed upon further
followup as well she got convinced that its basically
muscle pull in chest which causing the pain not heart
.attack
!okay,thank you,you may go
I felt ,atleast i will get 12-14 out of 16 in this case
????>when result came out,it was a shocker
16/3
they simply screwd me,i had 127 score overall,passed
!in all skills but st4 and communication
!story of my life

Whipps hospital in London


I scored 150 and passed all the station and scored 9
. in patients concern.So Failed the exam
I asked about the concerns in all the stations and I
.don't know why
:My stations were
:Clinical Stations
:CNS
Scleroderma and proximal myopathy
CVS:Mixed AR and AS
Abd:liver and renal transplant (PCKD)
: Respiratory
Apical fibrosis (Asian. Man -could be TB /and
discussion around asthma )
:Communication
I think we had it in the course
The old lady after hip fracture who was on aspirin and
clexane .She had a fall in the rehab ward and had a
stroke .Discuss with daughter who was angry and does
. not know why mother had scans
:History
A 55ys old pt with anemia and malena
On Ibuprofen for knee pain
:Station 5
Diabetic pt with visual problem
Uncontrolled hypertension in a young man ,has
hepatomegaly
DD Pheocromocytoma /PCKD
;This candidate is very unfortunate
It is unbelievable, to score 150 and pass All the stations
& the Skills with high mark and to fail the exam
..because of One mark in One skill
My Advice to this (& similar candidates) is to go to the
..next exam as it is unlikely to be unlucky twice
Good luck
:Glasgow PACES today
:Station 4
Delayed diagnosis of pheochromocytoma
Mr, jones 35 years male
Had High BP for last 5 years
Seen by psych for panic attacks
Tried many Med for HTN
But
His BP has been difficult to control
On his insistence , his GP has referred him to
hypertension clinic 2 weeks before
Results of tests now show
Urine : high metanrphrines
CT adrenal : 5 cm mass in right adrenal
Ur task is to explain the diagnosis
U don't need to know the details of further tests and
further management
Patient was concerned
Is it serious
Is it cancer
Is there a cure
Will I require future surgery
What future tests will be done
Was the delay justified
What medicine u will give me
Examiner : repeated similar questions
Overall not too harsh patient
Satisfied at the end
Agreed follow up GP Consultsnt website address alpha
blocker beta blocker
History station
Young female 28
Blood Diarrhoea after Cyprus visit
Started 1 day before coming back
Mixed with stool
Similar episodes for last 2 years
Took amoxicillin in Cyprus
Diarrhoea aggregated
Now last 10 days
Frequent blood a salime in still
Painless
C/ o small joints pain
No backache
No other extra intestinal symptoms
No oral ulcers
No skin changes
No jaundice
Cousin IBD UC
Father CA colon
No blood thinners
No steroid
No warfarin
No bleeding disorder
No weight loss
Concern : cause
? Is it cancer
? What next tests
? What Med
? Need admission or not
:DD
IBD ( UC)
Infective Diarrhoea
Antibiotic associated Diarrhoea
Examiner : just repeated all above
And
Asked
In
; History how will u rule out infective cause
Fever
Vomiting
But
He told
U will ask about symptoms to others accompanying
him
On your behalf, I thank this colleague for his
detailed n comprehensive feedback And I wish him all
.the best and of course success

Chennai
nd day2
Station 2 / palpitations for 1 month. Delivered 4
month. back Postpartum thyroiditis. Post partum
... .cardeomyopathy
cns charcot Mary Toth / 3
CVS systolic murmur all over the precordium. .. VSD
/MR not sure
/1
abd ADPK
Respiratory. Fibrosis +_ cavity .old TB
non cardiac chest pain. Seeking more investigation /4
SLE c/o pluritic chest pain /5
Distal phalanx arthritis. Known case of hypertension on
thiazide presented with lt wrist joint pain D/D gout
.arthritis

:Kuwait exam on 23/3


CVS:AVR
CNS:UL wkness for DD
COMMUNICATION:talk to pt's daughter>>her father
.CVA admitted to non_stroke unit> develop MRSA
BCC1:P.N for DD
BCC2:Bloody diarrhoea for DD
.CHEST:Rt pleural effusion
.Abd:HSM&Ascites
.H.T:SOB&WHEEZES
.Best luck for all
;Malta PACES
..The experience of another colleague
:Station 5
BCC1: scleroderma + lung fibrosis
BCC2: Retinitis pigmentosa
:CHEST
left thoracoplasty + lobar lung collapse , right upper
lobe crackles S/p Pulmonary TB
Abdomen: ESRD w RRT in a form of left sided
transplant & AV fistula ( functional & in use currently),
multiple scars for previous tunneled catheter,
peritoneal dialysis & RIF scar! I justified the active
fistula with transplant failure as patient was uremic &
hypervolemic, but couldn't justify why the transplant
!was left while it's usually done in right side
The other point I said right sided scar most likely not
related to the case & could be s/p appendectomy, later
I found the patient & he said the transplant was first in
!right but failed & redone in left side
:History
Deliberate self harm, paracetamol & alcohol toxicity,
..very annoying & arrogant lady
:cardio
midline sternotomy & left lateral thoracotomy S/p
Mitral valvotomy & later MVR, was in decompensation
with thrusting displaced apex, raised JVP, loud P2 ,
PSM.. Metallic click wasn't very clear, other candidates
.got confused with this case
:Neuro
Mixed Motor &Sensory neuropathy, Charcot joints &
left foot drop, absent reflexes & all sensory modalities
..distally, bedside orthosis
:Communication
Discuss brain death & organ donation with girlfriend ,
:very complicated case with many legal & ethical issues
She was complaining that my consultant already -
discussed the case with her boyfriend's mother
without taking her permission & she is No 1 in relative
.ranking by law
The mother agreed for organ donation & she is the -
nominated proxy w valid Lasting power of attorney
accredited by a solicitor , but the Girlfriend refused
organ donation & was challenging the power of
!attorney
GF requested to explain how she can officially -
!complaint against NHS
Malta Centre
second day , carousel 1 16/4/2
: ABD
splenomegally with CLD
: Chest
.Pneumonectomy in a young patient with alopecia
Hx: fever, upper abdominal pain , nausea and high
LFT in a returner from Kenya
CVS: AS /AR
Neuro : proximal myopathy (?congenital)
:Comm
A lady post Hip # and arthroplasty & on prophylactic
LMWH, fall down during physiotherapy and developed
.intacerbral bleedind
:Task
She is confusion , speak with her son (angrey)
:BCC 1
AF, pacemaker possibly non functioning presented
.with slurred speech for 30 minutes
:BBC 2
..A pregnant lady with SOB for 2 weeks

Experience of my Friend,,get his Exam in Malta


2016 /4/ 3
COMMUNICATION
Explain Diagnosis PHEOCHROMOCYTOMA
Dr Ahmed Maher Eliwa discuss thi scenario with me 4
dayes before the exam

Hisrory
Abnormal liver enzymes(Transaminase increase 20
folde)in young man taking Methotrexate for 8 mounth
ve sympt&signs+
epigastric pain
dark urine
pale stool
glasses of wine per night2
NEURO
Spastic paraparisi for DD
But i examiner stops me with each step ASKING can y
interprt what y r doing
eg:: hypertonia
hyperreflexia
The WHOLE time of i exam turned to ADISSCUSSION
about DDof Spastic paraparisi
eg:: in a young man what is the causes of Spastic
?? paraparisi
??? in old lady&
ask about ttt of MS
Again Expectation of Dr Ahmed Maher Eliwa especially
4 me
)(
CARDIO
MR
ask about signs of Infective Endocarditis
signs of severity of MR
investig&ttt
ABDOMEN
Pallor+PALPABLE liver
span 4 fingers below i costal margin
first i told him hepatomegally,,he asked how many
fingers,, I answered 4 fingers
???he asked do think it is enlarged
I answered ok,, it is palpable NOT enlarged
ask about DD
INVESTIG
TTT
CHEST
The first=The worst=The difficult one
chest deformity ...= ...Pectus excavatum-1
Left lung = area of bronchiectasis-2
Right lung=Boncheal breath+dullness+high vocal -3
resonance
Under built-4
ask about the right one
DD
INVESTIG
TTT
STATION 5
Loose motions for eight mounth&anaemia -1
10.5=serrogate
Coeliac
Irritable bowel syndrome
Intractable dry cough for 3 mounth=serrogate-2
she has childhood asthma
has gastritis
I ask every one read this ,,, only pray 4 me
if y please
Thanks&good luck 4 all

:Khartoum PACES, second day


Day 2 Cycle 1
St 1
.Bronchiectasis
Felty's syndrome
St 2
Iron deficiency anemia
St 3
.Proxymal myopathy
.Mitral regurgitation pulmonary Htn
.St 4
Delayed diagnosis of pheochromocytoma
.St 5
.Tuberous Sclerosis
Bilateral carpal tunnel in hypothyroid a 55 yrs old
.lady

#########################################
###################

PART 1
Today with us A very exciting and inspiring experience
She's a friend of mine
Tested the in Muscat, Oman April 2014
On the personal level I have benefited a lot from it

Abdomen was thalassemia it was clear ..scar in


abdomen and hepatomegaly ... ....and the question
......about hemolytic anemia
....... Chest was copd with bronchectasis
Also questions were about copd ABG and long time
oxygen therapy and ventilation
Cardiovascular I don't know what was the diagnosis. ...I
did so bad ...it was scar ....prosthetic valve ???
Ms.....and collapsing pulse and pulsating carotid and
.... murmure??? AR
.... Anyway I don't know what was the diagnosis
Neuro young pt ...left sid hemiplegia and cerebellar
..... .syndrome
..... Also I missed the case
Station 5....The first was neurofibroma. ...70 years with
.... .recently deaf
It was clear but the examiner asked me alot of
questions about why you didn't do weber and rinne
......even I didnt realize that the fork in the table and
...why ....I thought for neuro
He asked me alot of questions about brain tumor I
...... can't even remember the name
The second one was young like 15 years history of
double vision
On history was recurrent mouth and genitals and this
double vision and taking steroid for that but family
doesn't know why.....on examination was so tight the
place and he is not talking English not following you
....and for fundoscopy I asked the examiner for the
light in room he said didn't switch off .....Anyway it was
optic atrophy in right eye was clear in left I'm not sure
? ?......The question was how to confirm optic atrophy
And about behcet disease and he was not happy
....because asked me you ll not reply gp
Alot of small mistake ....but really because of short
...... time I missed so silly things
History was 52 history of migrain and high blood
....pressure and 3 weeks sob
... .years surgery on his legs for artery 10
.... .sister dead 40 years heart disease
Discussion was about left heart failure .....The time
finish befor finishing the management plan
... Comunication was long scenario .....My last station
The angry daughter want to see you because of her
father who was admitted 3 weeks because of cva ....He
developed pressure ulcers and swap show Mrca but
.. .clinical no signs of infection
The team in stroke unit refuse to admit him because no
.... place
In side the daughter was fighting the nurse not taking
care of him anytime we ask her for help she is busy
......
Then what this infection and why and what to treat
.. .and so many questions about mrca
Then about stroke unit why didn't admit him there
.... ......then at home no one to help him
.... All the world s problem was in her mind
For me I missed alot of things even her name and if
father or mother the one in ward and also even I didnt
red in the sinario that he supposed to go to stroke unit
.....Anyway the examiner was asking about mrca
.... This was my fantastic exam
.... Alhamdllelah
I wish the best for all of you
#########################################
##############

PART 2 = Feedback
Today with us A very exciting and inspiring experience
She's a friend of mine
Tested the in Muscat, Oman April 2014
On the personal level I have benefited a lot from it

..... .Good morning everyone


I'll send my feedback as I received because I want to
make things a little bit clear that to fail in one or two
.....station or three even still you can pass
In the exam you don't know what is going on in
examiner mind.....Anyway I'll comment on every
..... station
First in cardiovascular and neurology I did soo bad as I
wrote to you after exam and I didn't expect more than
...this
I missed the clinical findings so I didn't reach the DD
... and judgments
In chest and abdomen I did well but also I missed few
things especially in chest but in general I expect to get
18 or 20
In history I despite case was easy and because was first
station I was so confused and really after exam I was
depressed because the case was easy and I was soo
.clumsy
For comunication ...I did alot of mistakes because the
scenario was soo long ... like forget who's the sick
...The pt name...even it written in scenario that the pt
didn't admit to icu because no bed and daughter asking
me why and I'm just looking at her without answer
....Also I finish befor one minute and was just sitting
without talking
I thought I did soo bad ..but still got full mark.....
In short station also the tow scenario which I took I red
the first and the second also I forgot who send the pt
.... .and again pt name and if pt in clinic or hospital
Again in behcet disease he wanted foundscopy so i was
so stressed when i saw the ophthamoscope in table
because I'm not familiar with that .....Anyway the optic
atrophy was soo clear and asked me two questions like
how to diagnose behcet and how to confirm the optic
atrophy i answered wrong. ..and asked me you want to
reply anyone ...i was looking to the scenario ...without
... answer
But still i got 25 more than i expect
In neurofibroma ......it was spot diagnosis but in
examination I examined just the lesions without
looking for frickling or other criteria for
neurofibroma...I rememberd the criteria after the
...exam
And the most important thing was he kept fork on the
fir hearing examination but I didn't realize that but he
asked me to look at the table. ...I was shocked because
I forgot even the name for the test..and asked me then
how to treat also I didn't answer well
For welfare. ....I forget to wash my hand .....Every
stations the examiner was asking me please dr wash
:your hand ......:persevere
This is my feedback ......I know my score not high but I
sent it to everyone just to be optimistic and do your
.... maximum and leave the things for our God
See that in judgments i got 18
And my score just 133
But what I want to say that I studied tooo much ....and
I was trying to go to teaching hospital after finishing
... my 12 hr duty
And many times the nurses not allowed to me to
examine pt because I'm from other hospital
Many times I back crying .... .........The only space for
studying was in work ....despite last tow months I'm
doing 27 duty per months because two of our
...... colleagues on leave
.. But still i was trying ....my best
And every day I was praying and ask God to be with me
...... in the exam
#########################################
#######################

--UK on 25th Cumbria


started with station 5
Lady with MS coming with frequent UTI andurinary .1
.incontinence
On history also told she had uterine and rectal
.prolapse
Was confused what to examine and then the
examiners told me to examine the abdomen
only.discussed possibility of worsening MS and
prolapse related problems
Got 28/28
.2
.Lady with pain in hand joints
.She had systemi sclerosis
Forgot to take swallowing history and didn't listen to
lung bases in order to ask her concerns bu5 in
discussion explained thqt I will take swallowing
.history
.They asked my diagnosis
Said SSc. Asked what physical examination will u do if
.allowed to have extra time
I forgot to tell respiratory exan and remembered qfter
.coming out
.Got 27/28
Station 1
.Respiratory was COPD
I said fibrosis but they directed me to COPD and gave a
.good discussion how we differentiate them
.20/18
Abdo
.I thought Chronic liver disease and ascites
.Got 8/20
!!.So I was wrong
Station 3
Cardio
Was pan systolic murmur at apex radiating to axilla
.amd midline sternotomy scar without any other scar
.Said MR and MVp as lady was in 30s
.They said what else
I said VSD and then we discussed endocarditis
management
20/18
Neuro
.Left sided Hemiplegia
.Couldn't complete examination
.Discussion messed up
.20/12
Station 4
.Lady with diabetis coming with proteinuria
.Explain
Went in she was very angry.calm her down and
explained relationship between diabetes and
.proteinuria
.examiner gave me 8 other gave me 0 1
16/8
sta-2 - tremor , was simple got 20/20
result -- pass
last round Chennai 18.3.2016
Station 5 loose motion for 3 months
Bilateral knee pain
Station 1 Respiration COPD Bronchiectasis
Abdomen. APKD
Station 2. Headache with menorrhagia
Station 3 CVS MVR
CNS Facial palsy
Station 4 Type 1 DM with proteinuria
Poor drug compliance

Kuwait 24/3
Station 1: Copd.... renal dyalisis pt with left A-V fistula
Station 2: headache
Station3: MR .... GB
Station 4: breaking bad news for a lady whos husband
had meningiococal sepsis
Station5 : DM macular edema .... hypopituitarism
This feedback from a colleague who appeared in last
..PACES. . Whipps hospital in London
I scored 150 and passed all the station and scored 9
. in patients concern. So Failed the exam
I asked about the concerns in all the stations and I
.don't know why
:My stations were
:Clinical Stations
:CNS
Scleroderma and proximal myopathy
CVS: Mixed AR and AS
Abd: liver and renal transplant (PCKD)
: Respiratory
Apical fibrosis (Asian. Man -could be TB /and
discussion around asthma )
:Communication
I think we had it in the course
The old lady after hip fracture who was on aspirin and
clexane .She had a fall in the rehab ward and had a
stroke .Discuss with daughter who was angry and does
. not know why mother had scans
:History
A 55ys old pt with anemia and melena
On Ibuprofen for knee pain
:Station 5
Diabetic pt with visual problem
Uncontrolled hypertension in a young man ,has
hepatomegaly
DD Pheocromocytoma /PCKD
;This candidate is very unfortunate
It is unbelievable, to score 150 and pass All the stations
& the Skills with high mark and to fail the exam
..because of One mark in One skill
My Advice to this (& similar candidates) is to go to the
..next exam as it is unlikely to be unlucky twice
Good luck

-- exam in Kuwait yesterday


Neuro. GB
Card. Mr
Coum. PT with uncertain diagnosis for discussion with
.relative
St5 diarrhea 4 year. With iron deficiency Celiac
.St 5 leg swelling
.Chest - lung fibrosis

Yangon centre day3 round 1


BCC- systemic sclerosis
OSA -
Respi-COPD with basal crepts
Abd-COL with bilateral mastectomy scars with RIF scar
Neuro-dysarthria & examine UL- cerebellar sign(+)- MS
?CVS - AS AR TR MR
Examiners ask to measure BP
St 2- Tiredness with ED
with U/L DM & HT
St 4- noncardiac chest pain
Musculoskeletal ? Functional?

-- exam today in Kuwait yesterday


Station 5
Acromegally
Ankylosis spond
Hx TIA
.Communication.. uncontrolled DM with proteinuria
Abd: CLD
Chest: bronchectesis
CVS:Ms and AR
.Neuro : paraplegic with sensory level

.exam 20-3-2016,Castle Hill hospital cottingham


Started with station 5 back pain increasing , my Dx
Ankylosing Spondylitis ,next progressive visual
..... deterioration,diabetic ,,pdr
Station 1, RS persistent cough,couldn't diagnose put as
ILD ...abdo acut pain abdo with tatoo tender right
..hypochondrium....discussions about it
tation 2 malaise,loss of appetite fever night sweat wt $
loss.....discussion put DD as cancer ,TB,asked anything
else I forgot sexual Hx 55 yr old women....told may b
HIV,, connective tissue diseases
Station 3 CVS sternotomy scar no murmur look like
valve replacement aortic later examiner diverted to
causes of AR I told a few Marfan's ....later saw patient
has kyphoscoliosis missed while presenting
CNS HMSN
Communication: patient had disseminated bowel
cancer presented with bleeding ulcer talk to brother
who says his brother is dying any how why we are
doing any procedure ....brother has consent patient
has capacity I told his choice to get treatment or refuse
no one can force him

Chennai 18/3./2016
last round
Station 5 loose motion for 3 months
Bilateral knee pain
Station 1 Respiration COPD Bronchiectasis
Abdomen. APKD
Station 2. Headache with menorrhagia
Station 3 CVS MVR
CNS Facial palsy
Station 4 Type 1 DM with proteinuria
Poor drug compliance
Examinations of LATER dates
I HAD MY PACES IN ONE OF THE OVERSEAS CENTERS
AND HERE IS MY EXPERIENCE
STARTED IN STATION 3
NEURO
REQUEST WAS TO EXAMINE MOTOR SYSTEM
THE PATEINT HAS GLOBAL APHASIA(I DON'T KNOW
HOW DID THEY CONSENT HIM FOR THE EXAM) WITH
RIGHT SIDED WEAKNESS THE PT WAS NOT
RESPONDING TO MY COMMANDS AN HIS UPPER LIMB
WAS PAINFUL TO TOUCH, I DID TONE AND REFLEXES
AND I STRUGGLED A LOT TECHNICALLY THE
DISCUSSION WAS ABOUT CVA AND MANAGEMENT
CARDIO
THE TIME WAS VERY SHORT AND I AUSCULTATED ONLY
FOR ONE MINUTE THE CASE WAS DIFFICULT
(COMBINED MITRAL VALVE DISEASE AND PULM HTN)
MY PRESENTATION WAS BAD I WENT THROUGH MANY
VALVE LESIONS BEFORE I SAID COMBINED MVD THERE
WAS NO TIME FOR DICUSSION
STATION 4
COMMUNICATIONS
A GIRL WHO HAD HER FATHER DIAGNOSED WITH SUP
VENA CAVA OBST AND ADVANCED LUNG CANCER AND
SHE WAS CRYING AND CRYING TO DEAL WITH HER
STATION 5
SKIN : NEUROFIBROMATOSIS 1
MSK: OSTEOARTHRITIS OF THE HANDS
ENDO: GOITER WITH OPHTHALMOPATHY DEFINE
THYROID STATUS
OPHTHALMOLOGY:(NO IDEA) I SO OPTIC ATROPHY AS
A DOMINANT SIGN AND THERE WAS SOME HARD
EXUDATE?? DM+ISCHEMIC OPTIC ATROPHY
STATION 1
ABD
CHRONIC LIVER DISEASE WITH ASCITIS
THE EXAMINER DIDNT GIVE ME THE CHANCE TO
COMPLETE MY PRESENTATION AND HE TOOK THE ROLE
CHEST
COPD AND BRONCHIECTASIS( LOCALIZED)
STATION 2
A GUY WITH HEMOPTYSIS AND NIGHT SWEATS
OVERALL THE CASES WAS NOT SO DIFFICULT
THE BRITISH EXAMINER WILL LET YOU TALK AND
EXPRESS YOURSELF WITHOUT INTERUPTION
WHILE THE OVERSEAS' THEY ARE UNBELEIVABLY RUDE
AND THEY KEEP ON INTERRUPTING AS IF THEY ARE
EXAMINING A MEDICAL STUDENT
THE TIME WILL PASS VERY QUICKLY
THE SIX MINUTE IS VERY SHORT FOR THE HEART
STATION
THE NEUROLOGY CASE WAS UNFAIR AND THE GUY
WAS SICK AND NOT A CASE FOR THE EXAM
I DONT KNOW WHAT WILL HAPPEN BUT I KNOW ONE
THING, IF I SHOULD DO A SECOND ATTEPT I WILL
DEFINETLY DO IT IN THE UK
BEST OF LUCK EVERY BODY

Examinations of LATER dates


ABDOMEN WAS YOUNG MAN WITH JUST PALPABLEEN
,PALE WITH FINGER CLUBBING ALL FINGERS AND TOES
CHEST CASE THE SENARIO WAS PT WITH PRODUCTIVE
COUGH AND HEMOPTYSIS MALL AREA OF DULLNESS IN
RT LOWER LOBE IN THE AXILLA AND FEW SCATTERED
CRACKLES(NO COMMENT)
STATION 2
YEARS OLD WITH ABDOMINAL SWELLING -ASCITES 61
AND SMALL PLEURAL EFFUSION ON RT LUNG --TALK TO
HIM
STATION3
YOUNG LADY WTH TENDON XANTHOMAS
XANTHELASM EYE LIDS ARCUS SENILIS MEDISN
STERNOTOMY SCAR---MICOMPETENCE AORTIC
STENOSIS NO MECHANICAL VALVE SOUNDS
CNS
YOUNG MASPASTIC PARAPLEGIA KNEE REFLEXES
EXAGERATES ANKLES LOST NO SENSORY LOSS AT ALL
ALSO PES CAVUS
STATION 4
ANGN OF 71YEARS MAN WHO WAS INVESTIGATED 6
MONTHS AGO BEFORE CABG --HE WAS ANEMIC
GIVEOOD TRANSFUSION AND PERFORMED CABG AND
NOIS ADMITTED FOR INVESTIGATION OF SUSPECTED GI
MALIGNANCY--HIS SON GRY FOR THE DELAY OF
DIAGNOSIS OF HIS FATHER MALIGNANCY ALL THIS
TIME
STATION5
ENDOCRINE SIMPLE GOITRE WITH NO HYPO OR
HYPERACTIVITY
LOCOMR
RA
EYE
YOUNG LADY WITH DILATED RT EYE PUPIL NOT
REACTIVE TO LIGHT BLIND CAN NOT MOVE LATERALLY
UPWARD OR DOWNWARDS LT EYE OK---NIGHTMARE
CASE
SKIN
PSORIASIS
THE PT WAS YOUNG WITH CLUBBING OF ALL FINGERS
OF BOTH HANDS AND FEET WAS PALE JUST PALPABLE
SPLEEN WHAT ARE YOUR FINDINGS? I SAID THOSE--
WHAT IS YOUR DD?MYEL-
LYMPHOPROLIFERATIVES.LIVER SCHIRROSIS WITH
PH.CONGENITAL HEMOLYTIC ANEMIA---WHAT DO YOU
THINK THE CAUSE OF FINGER CLUBBING IN
ABDOMINAL CASE?LIVER CICHROSIS-PBC-IBD
ARE THESE DISEASES IN YOUR CASE? NO WHAT ELSE
CAN CAUSE CLUBBING IN THIS CASE? HEREDITARY ----
WHAT QUESTIONS COULD YOU ASK THE PT?---DO YOU
HAVE THIS CLUBBING SINCE BIRTH? ANY MEMBER OF
?YOUR FAMILY HAS THE SAME CHANGES LIKE YUORS
WHAT OTHER CAUSES OF CLUBBING?HOW DID YOU
?KNOW THAT THIS WAS SPLEEN
THE YOUNG ASIAN LADY IN HER 20S HAS ---
GENERALIZED TENDON XANTHOMATA
XANTHELASMATA ON HER EYE LIDS ARCUS ON
CORNEA WITH MEDLINE STERNOTOMY SCAR ---APEX
WAS NOT PALBABLE -NO RAISING OF JVP --MR
MURMUR AND AORTIC STENOSIS MURMER ----WHAT
ARE THE FINDINGS?I SAID ALL OF THAT ---WHAT IS
YOUR DD FOR AS---I SAID AORTIC SCLEROSIS AS THE
MURMER WAS NOT WELL PROPAGATED TO CAROTIDS-
---WHAT IS THE CAUSE OF ALL THESE FINDINGS?I SAID -
HEREDITARY COMBINED HYPERLIPIDEMIA --WHAT IS
THE MODE OF INHERITANCE?AUTOSOMAL DOMINANT
WHAT IS THE TTT-STATINS AND FIBRATES---FROM
WHAT YOU FEAR FRO THIINATIONS OF DRUGS?
MYOPATHY

Hi
I took PACES in LONDON
S1
RS: COPD -Chronic Bronchirtis(I couldn't finish the back
examination so I did just auscultation) asked me how
to confirm my diagnosis I said PFT FEV1 <70 and ration
<80 and the reverse is correct(FEV1<80 and ratio <70)
.The time realy went veey quickly
:Abdo
the patient elderly and was cold!!! so I exposed his
abdo just till mid chest
and chachectic with huge asites ,duptryn's contrcture
,Jaundice
I present it ok but I mention the most likely diagnsis is
Malignancy but I didn't find LAP and he asked what
else may be the cause I said Cardiac failure but(I seaid )
he is lying down on bed without SOB ,what else? said
TB pertonitis.asked can cause cachexia? I siad Yes.then
ask me about the IXs I said bl, U/E, US ,then tap for
exudate ,transudate,.... he daid what do u concern
about this pt. I said SBP and asked how to diagnose
this?I said Tap if more than 250 cell then postive .he
said thank u
S2
about 34 y man present with syncopal attack
He had had 1 episode of syncop??as he said 14 y ago
and he didn't loss his consciousness but his wife shout
to him but he didn't able to reply this for 1 min ,but
last 8 months it happens 3 times the last one i lost my
consciousness .father died from ICH and mother RTA
he didn't went to see dr at 1st one because he thought
it was trivial accident ,he is driver
and he concers about his work and as u know(he
said)now adays the financial crisis and it's very unlikely
.to get work rapidly
I asked about the all format like PMH,FH,drug H,
Personal smoking , alcohol, recreational drugs , ROS off
course about the nature of the coma and witness and
.....
finally I didn't summarize or get his expectations about
.the illness becuase the time was very short as well
they ask me about the problem list I said either
Idiopathis epileps or secondary epilepsy , he described
abcense siezure (actuly I think it was focal epilepsy
then trun to secondary generalize epilepsy . He asked
me how to Ix I said EEG ECG Blood , U/E, he then asked
me what u have to told to him : I siad I am sorry to tell
u that but u are banned from driving and u need to
contact DVLA and your Insurance company.The
examiner surprizly says But he is driver his whole life
depends on it? I said I'll tell him I am sorry but this is
.the law
) Realy I don't know if this is good or not(
S3
NS
y man LL examination 30
I saw faciculation (he wears jens rised it just bove the
knees Itried to roll up it but just small part of the lower
part of the thigh was appear) then there was wasting ,
and hypertonia (spasticilty bilateral) so spastic paraesis
pop up to my mind and asked him to move his legs he
couldn't almost power in both legs was 0-1 and went
directly to light touch (iI siad to him I'll gonna to touch
your leg by this swap of cotton plz if you feel it as same
as this(and try to touch it to the sternum)say yes he
said actually i didn't feel it dr.can do it on my ckeeks
!!what great offer!I said ohh yes then he felt it when I
begun to test it he was talking i didn't pay attention to
him then he opened his eyes
!OOOOOOOOOOOOOOffff I again said to plz If u feel it
as same as u felt it on your cheek plz say yes
and then change it to the lf leg he lost his sense till
T5?????????? and again whith vibration he lost till the
Knee without sensation so I told the examiners I need
to put it on ASIS but it was covered by his Jense so I
used the lower edge of the rebs???he felt the
vibration(I don't know if this is correct or not)any way
the time is over without seeing the back
I presented my findings and said this is combined UML
and LML so my most likely dx is MND????? I haven't to
say that but it just pop up .and asked me what goes
with LML? then how to IX then I mentioned one of the
test is EMG he asked me and what u 'll find in EMG?I
!!said I don't know
then Asked me :u said MND does it fit with the
senseory level u found? I pursed my lips and said no it
doesn't .Thank me and
CVS
He typically was Mrafan but I didn't find the Apex
beat??? I thought it Dextrocardia but it wasn't then
went through all examination but without lean him
forward I found early diastolic murmur in apex area
and the time over!!!!!!!!!!!!!!!!!!!!!! I gave them my
findings suggest most likely this AR due to Marfan and
the murmur high intensity in apex area!! (I don't know
if this will make me fail this station or not because it
doesn't fit with AR but I am sure it was AR) they asked
me about IX then causes I mentioned all causes but I
also said IHD he asked me and how this can cause AR ?I
said may by degenration of the valves!!(I didn't know
the relation)
S4
IBS the pt wants to see consultant (not me!!) and about
-ve and +ve of Ixs because she wants more IXs to find
out why her symptoms contnue inspite of using
.medications 3 months .and then seek second opinion
I don't know I was ok but u can't know that till u
. receive the result
They asked me what u will do for here I said may
change the medication .then asked :who will change
it?I said the consultant asked:and how the consultant
will know ?I said I'll explained to him and see her notes
and contact her GP asked me if this will not be useful
.what else u 'll do? refer her to Psychatrist
S5
psoriatic arthropathy .(do u think she has synovitis
before I said yes ?how do u know? because her left
index finger was deformed what else? he accompanied
!!me back to the pt but i didn't see anything else
Thyroid status with neck scar
Neurofibromatosis
Fudo: The left eye was abnoraml may be old choroiditis
?I don't know
And the test over
The examiners were very nice and the cases all
predicted all in Ryder no time for theory no time for
.perfect examination
Thanks
Hope all pass

Chennai 3rd day 1st round


Copied
????cvs... MR/VSD/TR
CNS....HEMIPLEGIA
Abd... Acities alone
Resp.... underlying copd with fibrocavitatory or fibrosis
History.... confusion with underlying CA prostate
Comm... amiodarone taking patient got lung fibrosis
BCC.... OSA
Exam experience Chennai
nd day2
Station 2 / palpitations for 1 month. Delivered 4
month. back Postpartum thyroiditis. Post partum
... .cardeomyopathy
cns charcot Mary Toth / 3
CVS systolic murmur all over the precordium. .. VSD
/MR not sure
/1
abd ADPK
Respiratory. Fibrosis +_ cavity .old TB
non cardiac chest pain. Seeking more investigation /4
SLE c/o pluritic chest pain /5
Distal phalanx arthritis. Known case of hypertension on
thiazide presented with lt wrist joint pain D/D gout
.arthritis

EXCITING Experience from your colleague Mohamed


Kawari
I want to share my experience in Western General
...Hospital, Edinburgh 25 Feb 2016
..I started my exam by station 3
..Cardiovascular; 50 year old man complains of SOB )3
I did the exam, I appreciated a murmur in apex.. I could
.. not time it
for unknown resean I said it is diastolic murmur
considering I do believe that diastolic murmur can not
..be brought in PACES
The examiner ask me if that was diastolic murmur
what will be your differential.. at meet the patient
after the exam at hospital gate and he told me he has
) 20/8 ( AS and MR !!! I scored
CNS; lower limb exam.. patient was not cooperative )3
and misleading
he kept moving his lower limb during tone assessment
and giving contradicting information during sensory
..exam.. I could not formulate DD
) 20/7 ( I scored
Communication: 40 year old lady has IDDM her )4
HbA1c 9 referred for albuminurea
I was disappointed from previous station and forget to
ask her if she does attent all foloow up appointment ,
??does she check her glucose
)20/4( I scored
BCC1: psoriatic arthritis has joint pain.. has skin )5
)rash over elbows and hair line.. I scored ( 28/28
BCC2: 70 year old lady history of loss of )5
consciousness and abnormal movement, had murmur
during adulthood for which she does not require follow
..up
My DD : epilepsy and stroke
.. I could not appreciated any abnormality in exam
I instructed her not to drive for 1 year and to inform
..DVLA
they ask me if I appreciate any murmur.. I answered
!!No
)28/24 ( I scored
Abd: kidney and pancreas transplant , has gum )1
hyperatrophy and poor vision.. I said the cause is Type
.1 DM as patient has vitiligo
) 20/20 ( discussion about complication of transplant
Chest: Rt upper lobe lobectomy with deviated )1
trachea discussion about indication of lobectomy and
) types of lung cancer ( 20/20
History: 55 year old male with symptomatic anemia )2
.. and melena on ibuprofen for knees pain
??His concern: Is it colon cancer
..I told him I ll request upper and lower GI scope
) 20/19( I scored
)172/130( The End Result is PASS
.. It was My first trial
I have never been to UK before .. I had course in Ealing
..Hopsital, London for 2 day ( it is excellent )
Despite the bad beginning .. Still AlHamdullah I
..passed
..My Advice .. do not be relactant in applying to UK
My English language and accent is not perfect however
!! they consider that
..Good Luck for All

Station respiratry pulmonary fibrosis secondary to drug


induced, Cvs AVR, Abdomen renal transplant Sec to
APKD,CNS brown Seward syndrome
Station 2 young teacher with two children complain of
palpitation n two year baby
Station 4 dealing with angry pt n explaining
management
And elderly tiredness and headache and previous
surgery of tumor in head. Panhypopituatrism
One of my frnd exam in uk
Myanmar
Day 1 Round 2 Yangon
Station 1 - Chronic liver disease, Dullness at Lt lung
base
Station 2 - known case of Ca lung, previously treated
with radiotherapy last 18 month, complaint of back
pain
Station 3 - Parkinson's disease
Station 4 - 82 yr lady with Alzheimer's and knee OA,
admitted with confusion and UTI, can't give antibiotic
because of dislodge cannula, talked with angry son
Station 5 - myotonic dystrophy, thalassemia with
Haemochromatosis

Chennai 1st day 3rd cycle


BCC... 1. persistent htn with knee pain. 2. Frequent
.headache within 2-3mths and impact on job
History.... 25yr old lady with hypertension and URE
.shows RBC and protein
Communication.... 25 yr old lady come to yesterday ED
with haemoptysis and fever and done CXR show
bilateral apical fibrocavitatory lession and sputum
show lots of AFB positive bacilli. Pt discharge from
hospital without the result. ED ph her to come to
hospital for result and pt is reluctant to come to hosp
.but today come to hosp
Task... explain the risk to the pt herself and others and
.advice to protect of spred of infection to others
CVS... restenosis MS with AF. Complaint... SOB
CNS... Hemiplegia, only examine LL. Complaint...
.difficulty in walking
RESP.... complaint... SOB. lt upper lobe fibrocavitatory
.lession and lt lower lobe pleural effusion
Abd.... complaint... abd discomfort. Lt arm AV fistula
functioning and recent puncture mark present with
.hepatomegaly
.That's all. Good luck to all
Examinations of LATER dates
STATION 1
RESP COPD FINDING,LOBECTOMY,EXAMINER WANTED
TO KNOW MOST LIKELY CAUSE WHICH WAS NOT
OBVIOUS
GIT
POLYCYSTIC KIDNEYS,RT TRANSPLANT ,NO FISTULA
SOME GUM HYPERTROPHY
STATION 2
HISTORY....40 F ARTHRITIS,TAKE HISTORY
STATION 3CNS DEREBELLAR SIGNS BOTH LIMBS BUT
SOME HYPERTONIA N SOME SENSORY LOSS LT
LEG,PROBABLLY SPINOCEREBELLER BUT COULD NOY
GET SPINAL HALF,EXAMINER WERE OK ON CEREBELLAR
SIGNS
STATION 4
WORSE ONE FOR ME,ANGRY MAN FATHER CAME TO
DAYWARD FOR PROCEDURE ,ESOPHAGEAL
DILATION,ENDED UP IN ESOPHAGEAL RUPTURE,HE
WANTS HIM TO TAKE HIM HOME,BLAMING
.CONSULTANT,,I COULD NOT CONTRL HIM
CAN ANY ONE SAY CLEAR FAIL IN THIS STATIO MEANS
?FAIL AS A WHOLE
STATIO 5
NO.1
SCLERODERMA WITH ONLY SOME SKIN TIGHTNESS
AND FEW TALENCIECTASI ON FACE,EVERY ONE TRIED
THEIR BEST TO MAKE IT SCLERODERMA INCLUDING
EXAMINER
NO.2
ANKYLOSING SPONDYLITIS WITH MILD PSORIASIS

Examinations of LATER dates


CASE 1
ABD MULTIPLE SCARS,TRANSPLANT
RESP COPD
CASE 2
HISTORY TAKING POSSIBLE RA
CASE 3
CVS AORTIC REPLACEMENT
CNS PERIPHERAL NEUROPATHY
ETHICS
RELATIVE NOT HAPPY WITH TREATMENT,BLAMING
NURSES
I THINK I FAILD THIS BADLY
STATION5
PSORIASIS
DIABETIC EYE,I COULD NOT MAKE SENSE OF
IT?LASER?CHORITIS
OA
RASH OD UNKNOWN ORIGION?DRUG INDUCED

Myanmar 9/3 /2016 last round


--Station1
rt sided pleural eff
thalassemia
Station2--TIA
--Station3
Myath.Grav
Mitr.Stenos
--Station4
known case UC, afraid to take oral steriod bcoz of side
effects, explain management plans of UC
--Station5
laser scar
SLE with TB

Myanmar 11/3/2016 last round


station1
??respi- effusion n tumor
??Abdo- hepato splenomegaly with CLD
Ststion 2
IDA n wt loss , epigastric pain , take Ibuprofan and diclo
for knee pain
Station 3- cerebellum, MS e pul H/T
Station 5 - Cushing . DM with CRVO

Experience of my colleague
I have finished my exam today
Myanmar 11 /3 /2016
nd round2
H/o - headache for several months with menorrhgia for
treatment
in detail - tension type HA with medication induced HA
? concern- cancer
commu - delayed dx of pheochromocytoma explain
scenrio - missed for 5yr and confirm by urine and CT
concern - cancer ? why delay ? need to again mood dr
? and surgeon
CVS - AS AR with pul H/T
Resp - i dont know think Rt upper lobe collapse
Abd -HS with jaudice (Thal)
CNS - MND ( bilatral small muscle wasting )
$ BCC1- RA with CT
BCC2- hypopit

.Sharjah center 10/02/2016


Cardio- double valve replacement
Resp- right sided pulmonary fibrosis
Abdomen- polycystic kidney disease+ functioning left
av fistula+ascitis+ heptomegaly= Dialysis related
amyloidosis
Neuro- Examine upper limb 16 years old boy has
proximal myopathy+ cerebellar signs+ UMN signs,
!sensation intact=friedricks ataxia as per examiner
History= 56 years old male long standing diabetes
coming with episodic vomiting and diarrhea, has also
postural hypotension = Autonomic dysfunction,
discussion about investigation and management
Communication skills= 35 years old female with history
of chest pain, has family history of ischemic heart
disease at a young age among her father and
brother.all cardiac investigations are normal.
Cardiologist asked you to tell her that this is most likely
functional and further investigations are not waranted
shes concerned about her symptoms, got pissed when
told its functional and shes asking for Angiography, all
u need to do is REASSURE REASSURE REASSURE and
.address her fears
Station 5
Case 1- young lady coming with red eye for
differentials- nothing on physical exam as shes a
normal actor and not a patient. Diagnosis is thyroid eye
disease
Case 2- old man coming with facial swelling. ESRD has
brachiocephalic fistula that was recently changed
because it was clotted. Diagnosis superior vena cava
obstruction
The first case was a young lady arround the age of 35
complains of both red eye for few months duration
There is no pain, discharges or headahche
No h/o of trauma or bleeding and systemic review is
otherwise normal
Pmh is insignficant
I did complete eye examination including fundoscopy
and it was normal
Examiner asked me wuts ur differential
I said keratitis episcleritis keratoconjuctivities
Chemosis
He said if its chemosis wut else would u look for on
?physical exam
I said exophthalmos opthalmoplegia and i will do
cimplete thyroid exam
I got 25/28
The 2nd case
Old man end stage renal disease
Has functioning left brachiocephalic fistula
Got obstructed 2 weeks ago
It wad removed a new one inserted
Then later on pstient presented with facial swelling
Sob
And neck engorgment
Which gets worse when raising arm and hands above
head
Is the case clear to pickup
On examination there is no obvious finding to pick up
Examiner asked wuts differential
I said superior vena cava obstruction
Investigation Ct Chest
Treatment anticoagulation
Goodluck to all

.Sharjah center 10/02/2016


Cardio- double valve replacement
Resp- right sided pulmonary fibrosis
Abdomen- polycystic kidney disease+ functioning left
av fistula+ascitis+ heptomegaly= Dialysis related
amyloidosis
Neuro- Examine upper limb 16 years old boy has
proximal myopathy+ cerebellar signs+ UMN signs,
!sensation intact=friedricks ataxia as per examiner
History= 56 years old male long standing diabetes
coming with episodic vomiting and diarrhea, has also
postural hypotension = Autonomic dysfunction,
discussion about investigation and management
Communication skills= 35 years old female with history
of chest pain, has family history of ischemic heart
disease at a young age among her father and
brother.all cardiac investigations are normal.
Cardiologist asked you to tell her that this is most likely
functional and further investigations are not waranted
shes concerned about her symptoms, got pissed when
told its functional and shes asking for Angiography, all
u need to do is REASSURE REASSURE REASSURE and
.address her fears
Station 5
Case 1- young lady coming with red eye for
differentials- nothing on physical exam as shes a
normal actor and not a patient. Diagnosis is thyroid eye
disease
Case 2- old man coming with facial swelling. ESRD has
brachiocephalic fistula that was recently changed
because it was clotted. Diagnosis superior vena cava
obstruction
.Goodluck to all
MYANMAR
ygn d3 r1
lung basal crepts iasked itisitial lg ds.1
renal transplt bi fistula
lethargy loss libido dm nht +gap shaving interval . .2
igive hypopit n auto neuropathy
metalic click igive dvr but friend said that it is mvr .3
--- .4
same as Hsu May Oo post --- .5
good luck

cases exams edinburgh


:station 1
andomen: hepatomegaly with? PD catheter - how are
?they related
respiratory: thoracotomy scar plus and chest tube
:station 2
middle age man with recurrent fits in pt with esrf? no
hx stroke. pt concern unable to take care of himself if
he has epilepsy
:station 3
:cardiovascular
multiple murmurs ?aortic regurgitation with metallic
1st heart sound. also got thoracotomy scar
neuro: PICA syndrome?/ brainstem syndrome (not
really sure about this one)
:station 4
discuss with pts father regarding bone marrow
transplant. pt (capable of making own decision) refuses
but father still insists
:station 5
optic atrophy. No INO/ RAPD ? -1
oral (i think with oesophageal) candidiasis in RVD -2
refused HAART
My fren sat the exam 1st of march 2016

Myanmar centre, Day 1, Round 3


History - Three episodes of collapse within 8 months in
binge drinker
Communication - Fits occur after giving clarithromycin
in asthma patient who takes aminophylline for a long
time

Myanmar 8-3-2016
Yangon 2nd day 3rd round
st 2
post partum thyroiditis H/o of palpitation in previous
preg .Now 4 mth after delivery of 2nd baby. palpitation
2 mth. Ho asthma. coffee 3 cups/day H/o thyroid ds in
sister
Communication
yr old lady e pnia, CURB 3, hyponatraemia,hypoxia, 84
h/o adverse eff on codein. Daughter tell that allergy to
coedin but nigt mo gave 3 dose of cocodamol. Now
.confuse. Talk to daughter
Concern Why happened?I previously told about this.
Antitode?Why my mon is confused? Can i see the chart
.for reason whether you note down it or not
Myanmar, Yangon Center
New Yangon General Hospital
Day 2, Round 2
:Station 4
year old ex.manger with headache for 3 months, 45
blurred vision 2 weeks, with fits 2 days ago. CT scan
head revealed high graded glioma at frontal lobe. His
wife worked at aboard and will come back the next
.day. Breaking the bad news
?Concern.. Why he suffer fits
?How long will he live
How to tell his wife as he planned vacation with his
.wife
.Station 5
BCC 1. Rt Hemiplegia with visual problem. Rt
Homonymous Hemianopia
BCC 2. Hand Pain with Acromegaly. Carpel Tunnel
.Syndrome
.Station 1
Resp. Rt upper lobe collapse. (Axilla lymph node biopsy
scar noted)
Abd. Renal Transplant with Hirustism
.Station 2
year old lady with bloody diarrhoea and abnormal 42
LFT. History of travel last 6 months ago to Australia. Wt
.loss..5 kg
?Concern. Is it cancer
?Is it managable
.I am not complete in concern
.Station 3
CVS. AS AR
CNS. Facial Palsy with cerebellar and CP Angle Tumor.
(Operated)
.Pray for me please
.Wish you all best of luck

Myanmar centre, Day 1, Round 3


History - Three episodes of collapse within 8 months in
binge drinker
Communication - Fits occur after giving clarithromycin
in asthma patient who takes aminophylline for a long
time
Cairo 2/ 2016
.St4
A female pt about 70 known bronchial asthma that's
difficult to be control till recently. She was admitted
with congestive heart failure and was controlled on
diuretics and ACEI. Today one of the junior doctors
prescribed bisoprolol as he thought it is of benefit for
her heart Failure & a nurse gave her the ttt. Since 30
minuts no harm happened till now but the pharmacy
told it is harmful to her to be given bisoprolol as she is
asthmatic. So the nurse was worried and pt. feel that
. something went wrong
You are asked to speak to her and explain the
..condition ...very long scenario
After introduction I checked understanding &
explained what happened, apologised & explained
what will be done incident report, department meeting
..and put under observation for any SE. And follow up
by cardiovascular and chest team . Her concern why
happened what will happen to me what will u. do to
prevent this happening to others . Ist she was angry
but after explanation and apologies and stress on her
care and postpone discharge for one to two dayes till
. we are sure she is ok. she is satisfied
Examiners ask legal issues I told negligence but is not.
It is mistake he told any thing else told autonomy - I
should tell not to do harm also he asked why bisoprolol
is harmful in BA. We are giving small dose- he told- I
told as it is non selective b blockers asked what can we
give i told carvidilol . Asked how u do incident report I
.explained . I got 13
. St 5 #
Sudden loss of vesion pt. Hypertensive By history -1
last less than one hour plus hemiplegia. She is on
insulin and bisoprolol only . Ex. Pulse AF. Carotids
asked to examined precordium examiners refuse .
Examined visual aquety simply asked for fundus they
refuse asked to examine her neurologically told ok. I
checked power. Was normal . Her concern. Is it
dangerous. Why happened? Questions about positive
findings. DD. Management .got 26
St 5. 2nd case Active RA . Discussion about inv. ttt.
Components of multidisciplinary team for this case.
Got 28
.1 #St
Chest copd.clubbing Basal fibrosis . Cushingnoid *
features asked about finding. Cause of fibrosis .Inv. and
management.got 16
Abdomen. HSM. Discussion about Cause inv. Ask *
about upper GI. Endoscopy in this case and ttt.if the
cause is HCV. Asked if it works? Bill rang.got 20
History bloody diarrhea and arthritis .history of #
travel to Morocco
History of long term diarrhea and recurrent and pain
relieved with defecation. FH. Cancer colon in grand
father aged 75 no other family members has cancer
. colon
.Concern is it cancer like my grandfather
Questions DD. Infective diarrhoea. IBD. .Asked what his
risk to develop cancer colon told like others. asked if he
need admission? I told I after examining and doing
basic inv. U&E we will decide asked about inv.and
.management of Infective diarrhoea . got 20
Neuro. Ms. Pyramidal weakness bilateral more in #
right Side loss of sensation in rt. Side till face cerebellar
signs. Time finished befor doing deep sensations
actually when he told one minute remaining I did
.... cerebellar. Told I wand to do
Discussion about DD inv. TTT. got 20
Finally cardiovascular . Double aortic with AR. #
.Predominant
Q. Finding . Causes. Inv and management got 20
.I hope my exam experience help you
My advice is to concentrate at least 3 months befor
exam and to have studying partner for history and
communicatios also to make study group. In yr work to
.see patients and discuss
Good luck for all and thanks a lot for our colleagues
who shared their experiences before
Exam in Glasgow 2016
: New scanerio
:Station 4
Delayed diagnosis of pheochromocytoma
Mr, jones 35 years male
Had High BP for last 5 years
Seen by psych for panic attacks
Tried many Med for HTN
But
His BP has been difficult to control
On his insistence , his GP has referred him to
hypertension clinic 2 weeks before
Results of tests now show
Urine : high metanrphrines
CT adrenal : 5 cm mass in right adrenal
Ur task is to explain the diagnosis
U don't need to know the details of further tests and
further management
Patient was concerned
Is it serious
Is it cancer
Is there a cure
Will I require future surgery
What future tests will be done
Was the delay justified
What medicine u will give me
Examiner : repeated similar questions
Overall not too harsh patient
Satisfied at the end
Agreed follow up GP Consultsnt website address alpha
blocker beta blocker
History station
Young female 28
Blood Diarrhoea after Cyprus visit
Started 1 day before coming back
Mixed with stool
Similar episodes for last 2 years
Took amoxicillin in Cyprus
Diarrhoea aggregated
Now last 10 days
Frequent blood a salime in still
Painless
C/ o small joints pain
No backache
No other extra intestinal symptoms
No oral ulcers
No skin changes
No jaundice
Cousin IBD UC
Father CA colon
No blood thinners
No steroid
No warfarin
No bleeding disorder
No weight loss
Concern : cause
? Is it cancer
? What next tests
? What Med
? Need admission or not
:DD
IBD ( UC)
Infective Diarrhoea
Antibiotic associated Diarrhoea
Examiner : just repeated all above
And
Asked
In
; History how will u rule out infective cause
Fever
Vomit
But
He told
U will ask about symptoms to others accompanying
him

Birmingham City hospital ....9/11


I started with station 3 : neurology was examining
upper limb ..... it was proximal muscle weakness with
no sensory affection or incordination ...... the
discussion was about possible DD diagnosis and
.investigations
Station 4 was breaking the diagnosis of MS in Young
female which an episode of unsteadiness ..... MRI
confirms the disease ...... discuss with her ttt options
and her main concern was her wedding was after few
weeks ..... to tell her husband or not ..... she a teacher
......what about her job ? ...... concerns about driving
Station 5 a was a young male with rash 2 hrs. after
eating a food ...... the rash subsides now ..... he has
shortness of breath little
b was a female with collapse ..... after history taking 5
was postural hypotension
St 1 abdomen was APCKD ..... chest lung fibrosis with
corpulmonale
st.2-It was afemale 68 years with shoulder stiffness and
hand stiffness ...... normocytic normochromic anemia
.... tiredness .....weight loss.... but she gave a history of
cough on last years which still present ....... the GP is
afraid of starting corticosteroids because she is
osteoporotic on vit D .... Ca .... bisphosphonates
.Polymyalgia rheumatica is the diagnosis i think but I
rash into the cough as may be lung cancer and this is
. Para neoplastic

Station ( 5 ) October 2015


Dubai
Station 5
DVT /1
Amurosis fugax /2
Station 5
Pt has SOB: Then found to have Wt loss, diarrhoea, /1
.thyroid nodule & neck scar
ALSO SOB: progressive then found to have RA on /2
.MTX
Station 5
hypertension newly diagnosed with fatigue and /1
dizziness on telmesartan,o/e had postural hypotension
Young female with depression one day Hx of /2
epigastric pain and vomiting?? she took 16 Tabs
panadol intermittently
Sharjah
Station 5
year old young man presented with excessive 27 /1
thirst
year old abdominal pain ..flatulence ...lethargic 22 /2
..family history of thyroid ..coeliac disease
Station 5
A young lady with 4 attacks of hypoglycaemia not /1
diabetic her 2 siblings have DM on insulin
yrs old lady with gradual loss of vision with 60 /2
symptoms of increased intracranial pressure
Cairo
Station 5
A 30 yrs old male with tiredness & feeling hot vital /1
.signs all normal except temp 37.8
Inside the exam: fever e sweating not profuse & there
is Wt loss, no difference bw day & night. No Hx of
cough joint problem GI CVS CNS MSK or GUS symptoms
& no risk factors for HIV. NO PH or DH of note but FH
.father had TB & there is contact with him
OE No pallor there is generalized LNpathy involving Cx
axillary & epitrochlear time not allowed for inguinal or
hepatosplenomegaly. Alcohol cause pain in these
glands. Concern is it TB like father contagious can be
transmitted to his kids?. DD Lymphoma TB others also
.but less likely
Adult female e HCV infection just recently started /2
.IFN & now came e skin rash
Inside rash typically of plaque psoriasis but only
started after IFN. PH DH FH SH & SR were negative.
Concern to stop IFN is it the cause for rash? Answered
by that she has psoriasis that can be exacerbated by
IFN but HCV if not ttt could be serious so to arrange e
both liver & skin doctor regarding what is best for her:
changing to other HCV ttt or continue IFN under
supervision of both of them
Station 5
Gravis disease /1
Diabetic maculopathy /2
Station 5
Pt e visual distunance /1
Had optic atrophy and spastic paraparesis ... MS
old lady e features of hypothyroidism and tender /2
thyroid ... Thyroiditis
Station 5
.yr male c/o headache, blurring of vision 35 /1
Normal bp and labs
Case was acromegaly, with no field defects, had
numbness as well with no carpal tunnel sign on exam.
Discussion about acromegaly management
yrs male with HTN, SKIN RASH 45 /2
IT INVOLVED THE ORAL CAVITY
CASE WAS PEMPHIGUS vulgaris, discussion about
differential, management and relation to anti
htn(amlodepine, acei)
Station 5
Systemic Sclrosis. Lady with SOB and skin problem.. /1
yr old male with sudden left side chest pain 28.. /2
with SO2: 88 and BP: 110/60.PE
Station 5
osteoarthritis /1
Small joints of the hand and knee with heberden's
nodes
fundus /2
DM and Hypertension
Blurring of vision
Station 5
Chest pain ( ischemia ) /1
Unstable angina or mi
Pain in both hands RA /2
Oman
Station 5
Man with history of lithotripsy for renal stones /1
comes with excessive hunger and hypoglycemia
Normal blood pressure no findings in the neck
?MEN
Typical history of breast discharge and mentrrual /2
problems and came with head ache and blurring of
vision
On history bitemporal hemianopia
On examination TUNNEL VISION : surrogate killed me
I checked three times
Did fundus examination normal
Asked dd
I said intracranial tumor
Then said prolactinoma
Said wud like to do MRI and refer to ophthalmologist
Messed this up
Station 5
young lady c/o irregular cycle ,headache & /1
abnormal vision -----> Prolactinoma
male with h/o renal stone ..presented with /2
hypoglycaemic attack -----> MEN1
Station 5
.young female e polyuria polydipsia and lethargy /1
.Pt in history e DM uncontrolled
.In exam faint hyperpigmenation
.Pt e hemochromatosis
. Abd exam with many scars
.So thalassemia is the case most likely
.Young e headache and facial pain /2
.Acromegally but unless you look to pt many times
Station 5
male 60 years old did routine blood check up , all /1
normal , high alkaline phosphate. . C/o decrease
hearing over years ... Clinically has sensory impairment
...? Paget disease
male of 50 years presented with weaknesses for Rt /2
side for 30 min .. he has DM on metformin ,HTN on
lisinopril not well compliance for his medications , BP
150/100 mmHg ..thought due that weakness due to
hypoglycaemia inspite no symptoms suggested to that
, he used juce , RBS on arrival 9 mmol/L , CT BRAIN
NORMAL ... Clinically : NAD . ... CVA .. Discussion about
CVA , diagnosis and management
Station 5
Obese man with headache last 2 month /1
From history. Pt known htn on lisinopril. No other past
medical history
Last 2 month gain weight
Snoring
Day time semulance
Taxy driver
RTA
Obstructive sleep apnoea ???
Young female /2
Complaints of headache
All history going with migraine
Pt concern. Need ct scan because feer of ca brain
Station 5
acromegally /1
.Angioedema complicating ACEI /2
Station 5
paget disease /1
Patient with high alkaline phosphate and hearing
difficulty
TIA with high BP RIGHT SIDED WEAKNESS AND /2
DYSARTHRIA AFTER 30 MIN FEELS IMPROVED

-- exam in uk ...14/2/16
Station 1 : pt coming with SOb and has Rt thoracotomy
scar , trachea deviated to Rt and decrease air entry on
bases with dull percusion ..other side some coarse
?crepts ..what is differential
Abdomen : no signs of chronic liver disease , abdomen
:hepatosplenomegaly and cervical lymphadenopathy,
after I finished they said u still have 1 min , in which I
?picked axillary LN..what is diagnosis
Station 2 :pt referred from GP with bloody diarrhea
and deranged LFTs (if anyone wanna be a candidate
?!!)
Station 3: cardio : pt coming with SOB ,I thought he
was in SR , one other candidate said so and 2 others
said AF ..pansystolic murmur at the apex radiates to
?axilla , differential and how are u gonna manage
Neuro: examine LL limb
Normal gait , didn't finish examination , loss of
sensation up to mid shin and loss of joint position, I
thought there is loss of ankle reflexes as well and
planters down going ..ppwer 5/5
Diagnosis ? What could be the cause in this gentleman
? And what rarer causes? These are the questions
asked by examiner
Station 4 : Mrs X coming with rash, GCS 8 and you
suspecting meningiococcal septicemia ...explain to
husband and address his concerns
Station 5
Lady had Stent inserted 3 weeks ago on dual therapy
coming with GI bleed ...hypotensive and tachycardic
....INR 5 proceed
yrs old girl presenting with headache , obs stable 32
...proceed
An Experience by our colleague
Tomadir Tag Eldin
My exsm was 11/2 Sharjah center
statio2 Hx
Problem, difficult mobility.. diagnosed with lung cancer
10 month back, recieved radiotherapy
on quetsioning, problem started with back pain 10
days ago, lower limb weakness today, loss of sphincter
control, diagnosis acute cord compression due to mets,
examiner asked about management of acute cord
compression including pain management options, this
was my first station, I was so tense didn't notice time,
but overall I did well in this station
Station 3
cardio case, young guy, systolic murmur all over
pericordium radiating even to axilla and root of the
neck, not audible in carotids, I presented my findings
well but diagnosis I said MR! Don't know why I said so!
Examiner was not happy, he said do you want to
change your mind at this point, I said yes! It is AS smile
discussion was about AS causes in young pt,
indications for valve repplacement
I scored 14/20
Neuro, peripheral motor neuropathy, sensations intact,
scars over the ankle and knee, diagnosis
HSMN, discussion about causes of
isolated peripheral motor
neuropathy and how do you
manage this patient
I scored 20/20
Station 4, I did very bad, scenario was very common I
knew it before, did it with Dr. Ahmed Maher Eliwa
but for some reason I did bad
Problem: pt eith stroke admitted to general ward
becos no bed available in stroke unit, developed bed
sores in hospital( which I didn't see at all, I thought ot
happened at home! ) and MRSA, ur task is to talk to
daughter and explain the situation and plans of
management , I kept explaining the MRSA and the
management, precautions, when 2 min were remaining
I asked about concern, she said am concerned about
my father sitation, he developed bed sores in your
hospital and I didn' t hear even apology from your side!
!! I got shocked really! How did I miss this! ! I said am
really sorry for this and I opologize on behalf of the
whole team, and we already issued incident report and
we started investigations, she wanted to complain
Examiner asked me do you think you adressed this
patient concern? Why you didn't admit the possibility
!!of negligence frankly
It was terrible Station , scored 12/16 , i thought I had
!failed
Station 5a, female 16 yrs with recurrent attacks of
stiffness and shaking movements on Rt. Side of the
body, there was rashes over face, diagnosis tuberous
sclerosis, discussion, what is this type of abnormal
movements, what investigations you will do, CT brain
?findings, what brain lesions
I got 28/28
Station 5b
Young man with polydipsia, and polyuria, otherwise
nothing in Hx, examination totally normal
Discussion about causes of polyuria and polydipsia,
what investigations
Station 1
chest, patient with midsternotomy scar, venous
harvest scar, 3 AV fistula, one is functioning and newly
dressed! !! This was my chest patient! Chest
examination I couldn't pick any abnormality apart from
!mild crepitation basally
my diagnosis was pulmonary fibrosis Vs pulmonary
edema, but I think it is well controlled COPD
I got 10/20 only
!Abdomen was another disaster
Female with cushinoid features, fistula again
functioning well and newly dressed! Abdomen, big RIF
scar but no palple kidney under it! ! Big hepatomegaly,
I said there is spleen also but there was no spleen,
examiner catched me on this spleen, he kept asking
what could be the cause of hepatosplenomegaly in this
patient, I kept saying this is CKD most probably due to
APKD, I couldn't explain the spleen, I was sure there is
no spleen, but I said it, and he cann't forget it! !! I got
15/20
This was my worst station
Over all I passed thanks to Allah
-----------------------------------------------
The lesson I got from this experience, exam is not easy
yet not impossible to pass very easily, what is needed
is to organise yourself, you need to but your own
approach to each station, what you will say and what
you shouldn't say! Think very well before you talk or
present your findings, remain calm, remain calm,
remain CALM,no matter what happens, don't argue
with the examiner at all, prepare your self by good
course, study, but exam is not about knowledge only,
study moderately, last week before exam stop reading
books and organise yourself and but schemes and ur
! approach for the possible cases
Lastly again remain calm before the exam, during the
exam and between stations forget the previous station,
stress will not help you, I was so much stressed and
.this really affected my performance and thinking
Lastly you don't know what is going on inside the
examiners mind, so don't be affected by their attitude
towards you and remain calm, the one who are smiling
to you could be giving you very bad marks and you are
!totally off point
------------------------------------------------------
Best of luck to everyone

Dont know if exam in uk is gonna be useful for u guys


...14/2/16
Station 1 : pt coming with SOb and has Rt thoracotomy
scar , trachea deviated to Rt and decrease air entry on
bases with dull percusion ..other side some coarse
?crepts ..what is differential
Abdomen : no signos of chronic liver disease , abdomen
:hepatosplenomegaly and cervical lymphadenopathy,
after I finished they said u still have 1 min , in which I
?picked axillary LN..what is diagnosis
Station 2 :pt referred from GP with bloody diarrhea
and deranged LFTs (if anyone wanna be a candidate
?!!)
Station 3: cardio : pt coming with SOB ,I thought he
was in SR , one other candidate said so and 2 others
said AF ..pansystolic murmur at the apex radiates to
?axilla , differential and how are u gonna manage
Neuro: examine LL limb
Normal gait , didn't finish examination , loss of
sensation up to mid shin and loss of joint position, I
thought there is loss of ankle reflexes as well and
planters down going ..ppwer 5/5
Diagnosis ? What could be the cause in this gentleman
? And what rarer causes? These are the questions
asked by examiner
Station 4 : Mrs X coming with rash, GCS 8 and you
suspecting meningiococcal septicemia ...explain to
husband and address his concerns
Station 5
Lady had Stent inserted 3 weeks ago on dual therapy
coming with GI bleed ...hypotensive and tachycardic
....INR 5 proceed
yrs old girl presenting with headache , obs stable 32
...proceed
Experience of our collegue Mohamed Alama
My exam experience
Almaadi military hospital 9/2/2016
: Station 3
:Neuro
:Findings
Spastic paraparesis+PN with stocking distribution (Rt
.leg)+ sensory level on left side for DD
:Questions
DD: MS, spinocerebellar degeneration, SCD
.Investigations: spinal & brain MRI, CSF findings
Treatment:acute, chronic, pharmacologic &
nonpharmacologic
:Cardio
:Findings
.AVR & MVR
:Questions
findings, functioning valve or not, HF, IE
Investigations:routine, ECG, echo, INR
.indications for AVR in AS
?what symptomatic AS means
.Treatment: nonpharmacologic and pharmacologic
:Station 4
Elderly lady admitted to the hospital with confusion
and UTI, cant give her the AB as she keeps pulling the
IV cannula out, comorbidities are Alzheimers, knee
OA, frequent admission to the hospital in the last few
.month
Task: talk to her daughter (angry), who is asking about
.an update
:I think this scenario is looking for the following
dealing with an angry relative.who also was tearing ,
offered her tissue that was on the table smile
Explain the need for a PICC line,draw what you are
.going to do and consent
Sort out the other comorbidities and any risks at home
(stove, shower, lost her way before, driving, the need
for an occupational and social workers and visiting
nurse after discharge)
the daughter kept saying that she wants to continue
taking care of her mom,and no way she will send her to
.a nursing home. you have to appreciate that
The daughter wants a brain CT done, because she is
confused(it must be her brain, doctor. Why you didnt
perform at CT, you are not giving her the appropriate
care)
Be patient and try to explain that it is a problem with
the chemistry of the blood not an actual brain
.problems
:Concern
Why my mom is not improving after few days from
?admission
?When she wil go home
:Questions
Ethical issues in the scenario (Beneficence and
.nonmaleficence), dealing with an angry relative
Why you didnt do a CT as her daughter wanted (there
is no focal neurological deficits that warrant doing a CT,
also there is an explanation for her confusion), not sure
if this is the good answer, I Would like to hear your
.comments
.Long term prognosis of the patient
.what do you want to offer her
What about sedation (I said it may worsen her
condition, but I heard after the exam about chemical
.and physical restrains), I leave that for the experts
:Station 5
(Ramsay Hunt $) Facial nerve LMNL (very clear) with -1
.history of ear rash few days before
Questions about DD: all causes of LMNL facial lesions
(CP angle tumours, parotid or face surgery, auditory
canal (cholesteatoma, abscess),also UMNL facial (he
)didn't like it, ,wanted the LMNL
Treatment(steroids, acyclovir, stomach and eye
protection, physiotherapy)
?Concern: could this be cured, how long it takes
Hypothyroidism (difficult case) -2
Presentation (fatigue , weight gain, menorrhagia, no
skin,voice or hair changes, on a treatment, she doesnt
know the name, which turned out to be thyroxin,
started a year after a surgery in the neck
(thyroidectomy))
Examination: fine tremors, no eye signs ,
.thyroidectomy scar
Questions about investigations, what is the single test
you want to do (TSH)
what is the most probable cause of her thyroid
problem (Gravess?, I am not sure if that is right, there
is no eye, hand or leg signs)
.Concern : what is the cause of the fatigue
Station 1(terrible examiners and difficult patients)
Chest: Rt upper lobectomy + obstructive lung disease+
)deviated trachea to the Rt side & left basal fibrosis
Questions: findings, he asked if the fibrosis is diffuse, I
said I couldnt appreciate that, investigations
(HRCT,sputum C&S,PFT findings)
Treatment (nonpharmacologic and pharmacologic)
Abdomen: Very obese patient with HSM, pallor,
pigmented striae, no LNS
Questions:findings, one diagnosis only (didnt want to
hear DD), I said Lymphoproliferative, asked about
blood film findings and other investigations, and
)treatment of lymphoma
:Station 2
Patient with macular rash over chest , neck back and
sometimes arms, started as vesicles that rupture after
that,no change with sun exposure, on doxycycline for
acne,no other autoimmune disease) for DD
examiner asked about DD(they wanted
photosensitivity in the first place, he said if you
pressed on him more he will say it it increases with sun
exposure, but I asked about that clearly and about
travel history to Hurghada and after spending
sometime on the beach, he denied any change in the
rash)
Other questions about investigations and ttt
.Concern: will it leave a scar, I said yes
Examinations of later Dates
Brunei Exam 11/6/2014
) my friend exam (:
Hx: fatige and microcytic anemia
.
Communication: BBN methiselioma
.
Neuro: weakness and hypotonia upper limb and
normal power and upgoing planter lower limb
Cardio aortic stenosis-+ sclerosis and-+ MR
Abdomen anemia +jaundice+hepatomegaly,no spleen
Chest basal fibrosis
.
St 5 young female with transient weakness
Middle age male diabetic with chronic diarrhea

.Sharjah center 10/02/2016


Cardio- double valve replacement
Resp- right sided pulmonary fibrosis
Abdomen- polycystic kidney disease+ functioning left
av fistula+ascitis+ heptomegaly= Dialysis related
amyloidosis
Neuro- Examine upper limb 16 years old boy has
proximal myopathy+ cerebellar signs+ UMN signs,
!sensation intact=friedricks ataxia as per examiner
History= 56 years old male long standing diabetes
coming with episodic vomiting and diarrhea, has also
postural hypotension = Autonomic dysfunction,
discussion about investigation and management
Communication skills= 35 years old female with history
of chest pain, has family history of ischemic heart
disease at a young age among her father and
brother.all cardiac investigations are normal.
Cardiologist asked you to tell her that this is most likely
functional and further investigations are not waranted
shes concerned about her symptoms, got pissed when
told its functional and shes asking for Angiography, all
u need to do is REASSURE REASSURE REASSURE and
.address her fears
Station 5
Case 1- young lady coming with red eye for
differentials- nothing on physical exam as shes a
normal actor and not a patient. Diagnosis is thyroid eye
disease
Case 2- old man coming with facial swelling. ESRD has
brachiocephalic fistula that was recently changed
because it was clotted. Diagnosis superior vena cava
obstruction

Cochin ( Kochi ) Kerala, India, 26 Feb 2016


station 5
pt 35 year old lady fever high grade 10 days with skin
.rash
Second patient 65 year lady with palpitations had right
.sided multinodular goitre
Station 1
.Resp case
.examine the chest. Left oblique scar posteriorly
Abdomen
young man. Left forearm av fistula. Two scars in both
iliac fossae with a palpable transplanted kidney. No
.signs of any immunosuppression
Station 2
Palpitaions of one month in a 35 year old lady
Station 3
Cardio
.HOCM vs Aortic stenosis
Neuro
lower limb examination. Peripheral neuropathy
Station 4
You are Dr in out patient clinic. Lady 27 year old came
to ER yesterday with hemoptysis, h/o low grade fever.
Sputum AFB full of bacilli. CXR bilateral apical fibro
cavitary lesions. TB. She left ED yesterday. Today she
was called to the clinic for admission but she refuses.
She was reluctant on phone even to come to the clinic.
.Talk to her
.They kept two more cases. Abd -spleenomegaly
.CNS - Bells palsy
.Alternating between candidates

..Cochin India ..16/2/25


station 1
..Abdomen..CLD
..respiratory.. ?bronchiectasis
Station 2.. 29 yr old gentleman with Episodic Skin
rashes on and off from 3 months and recent onset
blisters on forearm
Station3 ..
CVS 2 cases were kept.. Few guys faced HOCM and ...
other MVR.. CNS: i faced peripheral neuropathy and for
..few others myopathy
Station 4: talk to the son of the patient who underwent
pneumatic dilatation of esophagus complicated wid
..perforation
station 5: 30yr old lady with skin lesions and blisters for
..6months
And one more case 76 yr old gentleman with recent
..onset of weight gain

My experience for PACES in Tameside hospital near


Manchester 6 Feb 2016
Station 1
:Abdomen
Young male with fine tremors on outstretched hand
and skin warts. On abdominal examination: there was
a left iliac fossa scar with a mass under it, also there
were multiple abdominal scars. My diagnosis was left
renal transplant with previous peritoneal dialysis
history, Patient is mostly on tacrolimus or cyclosporine.
Discussion was about causes I said commonly it's
diabetes but for the patient's age can be ADPCK, GN,
obstructive or reflux uropathy. I scored 20/20
:Respiratory
About 50 years old male, on general inspection there's
peripheral cyanosis, also plethoric face with multiple
telangiectasia. On chest ex: bilateral fine inspiratory
creps that didn't change with cough. My diagnosis was
Interstitial lung disease; patient may be on steroids.
discussion was about causes. I scored 20/20
====
Station 2: History
Young female presented with 3 episodes of rash on sun
exposed areas, last one associated with blisters on
both arms. On history taking: Patient mentioned going
to the beach before last episode, no symptoms for
rheumatological, connective tissue disease or
abdominal complaints. Past history of acne for which
she is taking doxycycline prescribed by GP. My
diagnosis was drug eruption caused photosensitivity
rash accentuated by sun exposure, my differential was
porphyria cutana tarda, dermatitis herptiforum and
pemphigoid. Discussion was about the management:
investigations ANA, TTG, eosinophilia, stopping the
offending drug. I scored 19/20
====
Station 3
:CVS
More than 60 years old female, peripheral examination
was with in normal, heart revealed systolic murmur on
the aortic area radiating all over the pericardium, also
soft S1. My dx was Aortic stenosis, the presence of soft
S1 make associated Mitral regurgitation is possible,
Discussion was about the difference between Sclerosis
and stenosis. Plan of action and indications of surgery. I
scored 20/20
:Neuro
Young male with spastic posture on general inspection,
on Examination: Hypertonia and hyperreflexia with no
clonus, there is some cerebellar involvement, no
sensory involvement, I finished examination early so I
had time so preceded to examine the eye which
revealed bilateral nystagmus, isolated left 6th cranial
palsy. My diagnosis was MS. Discussion was about my
differential if no eye examination was performed.
Management plan for MS including investigations, ttt
of acute, progressive ds and symptomatic ttt. Score
was 19/20
====
:Station 4: Communication skills
Patient admitted with stroke, there was no places in
stroke ward so admitted in surgical ward. There was no
improvement in his condition, he developed bed sores,
isolated swab revealed MRSA. I had to speak with his
.son
I divided the case into 3 sections: 1st one regarding the
admission, then explained bed sores and care about it,
then I explained regarding MRSA infection. Initially the
relative was angry but after calming him down and
explaining the situation and what we will do he was
cooperative. I scored 16/16
====
Station 5 was quite unexpected to me as there was no
physical findings at all
:BCC1
years old female presented with melena on and off 60
over 3 years duration. On history taking there was no
positive data toward a specific diagnosis. No history of
epigastric pain, liver disease, no NSAIDs, no alcohol or
steroid use. On examination also nothing was
appearing, I examined the abdomen, aortic area for AS,
looked for signs of pallor, mentioned the need for BP
and digital rectal examination. Explained the need for
further blood investigations, OGD as outpatient and
may be colonoscopy according to the results.
Discussion was about the causes, management plan. I
didn't feel good at this case. I scored 22/28
:BCC2
years old female presented with history of Loss of 23
consciousness and jerky hands and leg movements. No
tongue biting, no incontinence, no frothing. CT brain
done is normal, electrolytes and initial investigations
with in normal. Had mild chest discomfort before it. No
attacks previously. History of occasional palpitation
attacks. I examined the heart, pulse, carotids for bruit,
upper limbs power (just grip and shoulder abduction),
planters and did fundus examination. All came normal.
I explained that it mostly a seizure or cardiac
arrhythmia. Plan: Holter, EEG, stop driving. Discussion
was about driving restrictions and further
management. I also didn't feel much comfortable with
.what I've done. but I scored 24/28
======
Overall Pass: 160/172
==========================
Advice for the exam preparation as requested on my
inbox
)this wasn't my 1st trial(
:Books for study
OST for clinical examination (Book 1)
Ryder for station 5 (only the section with the real case
simulation)
Ryder for History and communication (extreme value
especially if your going for UK)
There's also notes done by some one named Fady (last
minute revision for PACES) of very good value
for revision can use Cases for PACES book
:Duration of study
years after 2nd Part 2-1
:Duration needed for clinical experience
years or more will be perfect (seen people passed 2
without hospital work)
:Courses
Preparatory course: any course in Egypt will be enough
for intial preparation
For UK: I went for PACESahead course which was really
good, it was more than 100 cases full of findings, but
you have to be very well prepared before attending it
as it's more of an exam simulation for each case, you
have only 7 minutes to examine the case and few
minutes for discussion. It will be a perfect exam
simulation especially if done few days before the
exam. My advice is not to attend it too early before the
actual exam as you lose the main advantage of it which
.is making you be at the exam mood
Other courses in UK with good reputation: PassPaces
but it only accepts UK trainee, PasTest, ealingPaces (I
really can't give an opinion about them)
:Tips
You need to see many cases, and examine them like -
.you are in the exam
Practice with other exam candidated for CS, HT, -
Station 5
Work on your presentation to be focused, concise -
.and smart
When practicing how to present your case, you -
should be able to mention the following in less than 2
minutes (Positive signs, releavent Labs, Imaging, TTT:
General measures, specific measures, disposal for
Station 2, 5). I know it's difficult but believe me it will
make you score all the station marks in less than 2
.minutes of discussion

Examinations of later Dates


By Dr.Mohamed Gohar
:Brunei Test, Monday 8 June, third cycle
:Station 1
RESP: Man with depressed right lung base and small
scar on right side and deviated trachea to right,
expansion limited on right base and dull percussion. air
entry diminished in right base. Breathing vesicular with
area of bronchial breathing in right upper and middle
zones "over shifted trachea" with increase VR and WP
in these areas" for DD
ABD: Man with jaundice, median sternotomy scar and
massive HSM. Most probably infiltrative dieses
:Station 2
years old man with two weeks history of headache, 40
confusion, impaired short term memory and exam
showed right homonymous hemianopia, known HTN,
and heavy smoker. DD mainly of space occupying
lesion either primary or secondary from lung cancer,
how ever pt denies cough or chest symptoms
:Station 3
Neuro: woman with difficulty in walking, lower limb
shows increase tone bilat more on right and
exaggerated knee reflex but no ankle reflex and
equivocal planter and pyramidal weakness around hip
and knee. intact coordination and intact superficial and
deep sensation. I said DD MND or spastic paraparesis
""no back scars
CARDIO: Man with median sternotomy scar and
audible click, and lower limb edema, metallic first
heart sound and irregular pulse, bibasal lung crackles
and sacral edema and jaundice, no murmurs. There
might have been increased JVP and Loud P2 but i
..missed that
:Station 4
Young Man work as a chef and known asthmatic for
long time and also some episode of allergic reaction to
nuts. came to ER with anaphylactic shock after eating
mixed salad and had to be ventilated for a while now
he is fine. He works as a chef. to explain condition and
seriousness and impact on work
:Station 5
lady c/o diarrahea for 1 week, bloody watery :1
diarrhea. pt known pf psoriasis on adalimumaba and
methotrexate and has also vitiigio, and only other
finding is right hypochondrial pain and tenderness. I
said DD either IBD or Infection mainly TB given her
immunosuppresion
young lady known of DM since age of 7 on insulin :2
and Graves disease since age of 11 on carbimazole, c/o
of attacks of hypoglycemia, also has vitiligo and
postural hypotension. I said DD addison's disease
Good luck to all still taking the exam and pray for us
who took it already

Wolverhampton,, UK,, new cross hospital,, 12 February


2016
history, collapse,, patient on thiazide & started -1
candisartan two weeks back / Cardio, instruction pt is
asymptomatic but referred by his GP,, I heard ejection
systolic murmur,, discussion about aortic stenosis &
sclerosis / neuro examine cranial nerves,, only
abnormality is diplobia on looking outward and
upward on both sides // communication,, pt with
essential tremor, carpenter diagnosed 3 years by
consultant, now concerns about Parkinsonism referred
by GP for deep Brian stimulation
station 5,,, fever in 27 year lady,, by history she had
lymphoma before,,, second case diarrhea,, I noticed
deformed nose,, finally its wegners plus diarrhea after
augmentin course for sinusitis abdomin,, HSM,, NO
stigmata,, plethoric

Thursday 11 feb 2016 maadi military cairo


Station 5
Carpal tunnel with background of acromegaly
Asked about investigations for complaint and causes
and ttt of acromegaly
Joint pains bith hands and knees with morning stiffness
15 mins
...Osteoarthritis
Asked why not RA
ttt lines of osteoarthritis
Station 1
Obstructive airway disease
With bibasal crackles...was not sure bronchiectasis or
fibrosis
Asked about investigations and ttt
HSM was huge spleen
Asked about infectious causes of large spleen and
asked about malaria investigations and ttt and
..bilharziases investigations.....was tough station
Station 2
Story of syncopal attack for 1 min
Vasovagal vs drug induced postural hypotension due to
recently added candesartan..asked about
investigstions and ttt and necessary admission or not
Station 3
Bilateral UMNL with bilateral cerebellar and normal
sensation
???Asked most likely i told MS...WAS NOT HAPPY
asked about investigations and ttt asked about new
?drug.i told fingolimod
AVR mechanical with malfunction due to radiated
murmur over neck
Asked about investigation ..ttt...other ways to replace
AV
STATION 4
IHD prescribed ASA and plavix
After 6 weeks HB drop to 7
Task to eplain causes of anemia and management
Concern
Blood transfusion
??Is it cancer doctor
I told unlikely but needs to be ruled out as he denied
any ALARM features
Seems he didnt like my concern answer

EGYPT==CAIRO,,,last cycle
St1
Abdomen HSM with Lymphnodes
Chest. Lobectomy with lung fibrosis
St 2
Occupational Asthma
St 3
Cardio
PMV PAV
Neuro
Hemiparesis
St 4
She had obstructive jaundice and probably has cancer
speak with the daughter
St5
Gravies ophthalmopathy
After getting UTI she developed confusion

An Important EXPERIENCE from my dear friend (M.A)


who examined in Brunei last diet = December 2015
N.B. He is my GUIDE in paces
Station 1
Chest pul fibrosis in sleep patient actually mixed CTD
HE ASK ABOUT IVESTIGATION management ipf
:Abd
Huge splenomegaly in young lady with anaemia in feed
back I missed also jaundice
He ask about DD investigation. Management
Station 2 iron Def anaemia in middle age lady with
mech MVR and hypothyroidism not respond to oral
iron no bleeding from body orifices I missed to ask
about menses.no alcohol intake or malnutrition.woried
about colonoscopy as she was planned to have before
but developed presyncopy .ask about DD I mention
over coagulation or under coagulation. Haemolysis
.from valve.coeliac disease .also about management
:Station 3
.Cvs MR &AR with displaced apex
Neuro: spastic paraparesis with planters down on one
leg and equivocal on the other with loss only of pain
and touch up to mid calf on Rt leg only .joint sense
normal. In feed back transverse myelitis. I don't know
how? It was difficult case.they ask about DD
..management
:Station 4
Lady admitted with CHB .PPM inserted but lead
discovered dislodged during programming next
day.needs lead repositioning. She is angry and want to
be discharged .want to change her cardiologist. I
apologise. Ask about consent she signed before what
she was told about complications? Explain this
complications can happen but rare.canot be discharged
till repositioning of the lead other wise threatening her
.life.she agree for repositioning at last
Examiner ask about consent informed or written?reply
written as this invasive procedure. She can change her
physician through him.if she insist to go DAMA? Reply I
will do my best to convince her ,consultant
appointment, still insist ?she is competent and has the
right to refuse ttt even if if life threatening but after
.explanation of all the risk
:Station 5
A-35 lady with 2NDRY amenorrhoea. Only with hand
sweating and headache no more symp.it was
acromegaly. There was spade hands .hirsutism.
macroglossia and skin tags.i present the case as
pituitary tumour and I need to R/of acromegaly but
.they were Un satisfied
B-young lady with MCT with chest pain concerned
about heart attack.hx of ll swelling but no DVT SIGNS
..spot 98% HR 88
They ask about DD .mention myocarditis, pericarditis.
Vasculitis.could be due to pul htn (she had accentuated
2nd PUL SOUND) .less likely PE IN feed back they down
.grade me bacause not Menston PE
.I hope will be beneficial for preparation
Good luck

Sharjah
2016/2/10
Station 2 : diabetic autonomic neuropathy
Station 3 : double valve replacement , spinothalamic
degeneration
Station 4 : chest pain , young women , all cardiac
workup normal , reassure her no more tests required
Station 5 : requrent red painful eye ( most likely
thyroid case)
nd case : svc obstruction ( senario facial swelling )2
Station 1 : obstructive lung disease
Polycystic kidney with massive ascites and
tenderhepatomegaly with functiong fistula

EGYPT 10-2-2016
Almaadi
St 1 . Lung consolidation with fibrosis. Abd : hsm
St 2: collapse due to postural hpot caused by acei
St 3 : neuro ms, Freidrech ataxia cardio : aortic VR with
AS
St4: pt with aneamia after taking asp and clopidogrel
for his IHD concern is it cancer
St 5: osteo arthriris . Acromegally with carbal tunnel
syndrom
: Dubai paces
Station 1 : lobectomy - HSM with inginal LNs
: Station 2
Post streptococcal GN
: Station 3
Mitral regurgitation
Combined ulner and median nerve palsy
:Station 4
BBN : meningitis comatosed pt
: Station 5
Cushing
Scleroderma present with reynauds

2014 10 20
University malaya ..malaysia
St 5 ..my first
Bcc1 thyroid cardiomyopathy with icd...big mistake not
exposed fully
With overwarfarinisation
Bcc2
Diabetic retinopathy post laser
Respi unsure
Stem..c.o sob
Coad..might b right upper lobe fibrosis as tracheal to ?
right
I missed the fine crepts may b
Cardio
No murmur
Clubbing with polycythaemic
Asd with esseimenger
Abdo
Renal transplant
Cns
Fascioscapula humeral
St 2 radiation proctitis
St 4 addision poor adherence to steroid

Maadi hospital 10/2/2016


) Egypt (
Cardio ms +mixed aorta
Neuro charot
Abdomen hepatomegaly hemolytic anemia
Respiratory cobd fibrosis
Comuication diabetic patient have all complications
refuse to take insulin
station 2- Post partumthyroiditis
Station 5 antiphospholid syndrome
Optic atrophy 2nd case

Cairo 10/2
Hepato splenomegally
Copd+ bilat basal fibrosis
History : bloody diarrhea
Double aortic + double mitral
??? Ms+ stroke
Communi: medical error
Sudden painless transient loss of visin
Hand pain in rheumatoid carbal tunnel
Cairo 9/2/2016
rd carousel3
Abd
Thalassemia
Chest
COPD with fibrosis
Neuro
Cervical myelopathy
Cardio
Double Aortic
Communication
Medical error
Hx
Iron deficiency anemia in 40ys old lady with OA &
weight loss
Station 5
??? Mallory weis syndrome
Short stature
Exam Experience of Dr. Noha Attia
EGYPT 6/2l2016
my exam today kasr al3eny cairo first cycle
st 1 abd young pt wz large spleenomegally
chest :- lt lung fibrosis wz OLD
q how to investigate what treatment of fibrosis
st 4 motor neurons dis
concern what if symptoms aspiration and weakness is
m living alone
recure no one to help me iforget z beg tail
st3 neuro pt wz LMNL & down going planter asymm
weakness loss of sensation ididnt finish examination
idid very bady dont know what was diagnosis
st 2 diarrhea 3month
difficult to flush and smelly 3times per day more wz
fatty food wt loss 5kg in 3 month good appetite history
of pneumonia received amoxicillin 4 months ago for
1wk concern is it cancer
st 5 knee pain in acromegally pt ididnt do visual field
was asked by examiner
case 2 tirdeness
history anaemia melena epistaxis loss of wt not know
how much examin pallor
they didn't allow abd examination and red spots on z
tongue concern is it
serious
overall im not happy wish
u all good luck

Egypt Cairo 7/2/2016. 2nd cycle


Copird from ** Paces uk study group** On Telegram
St2 @
.Mr. Youssef 40yrs M -
.On call doctor in medical admission unit -
C/O poor mobility -
He has been diagnosed of lung ca 18 months ago, Has a
.back pain, received radiotherapy 10 days ago
?Concern::: is it related to ca
?Am I going to walk again
St4 @
.A doctor in cardiology clinic
Mrs Noran 40yrs Female
Problem : non cardiac chest pain
Pt waiting for the result. Which showed normal lipid
.profile, normal ECG& normal stress ECHO
The COSULTANT decision that the pain non cardiac and
.no need for further test
.Manage the pt concern, reassure her accordingly
the pt keep asking for further test. And asked do u ::::
?think doctor I have to be seen by chest physician
.she has Fhx of heart attacks ( father55, bro 55) ::::
why I need to be seen by psychiatrist ? Do u think ::::
?I'm mad
St1 @
.Abd :: HSM + multiple Abd scars & masses. For DD -
.Chest :: pt was distressed COPD + lt basal fibrosis -
St3 @
Cardio :: severe AS -
Neuro :: pt c/o difficulty in walking. O/E -
.asymmetrical UMNL + PC + CEREBELLAR ::::: MS
St5 @
yr M hx of loss of wt 3 month, loss of appetite , 50 -
.diarrhea , lumps
O/E cachexic , pale , Generalized lymphadenopathy :::::
.DD
.yrs M with recurrent oral ulcer55 -
Hx of knee pain , sore eyes, painful skin rash over the
shin. ::: b
.Behet D + DD

Examinations of later Dates


th april 2015 oman paces9
Station 5a 55yrs old lady with mechanical type back
ache post menupausal
b. 30yrs old lady with skin lesions neurofibromatosis 5
neurofibromas and cafe aulait spots
Station 1 abdomen ascites with positive shifting
dullness discussion around cld
Resp: 60 yrs old man with copd
Station 2: 35yrs old gentleman 6yrs history of RA on
methotrexate having breathlessness night cough and
wheezes. Discussion went around br asthma
Station 3 neuro: guillain barre synd ardflexia and
inability to walk
Cardio: MR with AS discussion around MR
management
Station 4 breaking bad news to wife whose husband
admitted with seizures controlled with diazepam CT
showed frontal tumor Biopsy confirmed grade 4
.astrocytoma. Experts advised palliative treatment

EGYPT 6/2/2016
St 3
Motor neurone disease
Wasting fasciculations,Extensor planter
DISCUSSED IN THE COURCE OF Dr.Ahmed Maher (
)Eliwa
Cardiology
!! Mitral stenosis
St 4
years old lady 75
While she was on physiotherapy due to fracture neck
femur she fallen down Developed confusion but no
neurological deficit
Ct showed minimal cerebral haemorrahge
Speak with her son
DISCUSSED IN THE COURCE OF Dr.Ahmed Maher (
)Eliwa
St5
Skin disease with s o b
It was scleroderma with lung fibrosis and
pulm.hypertension
DISCUSSED IN THE COURCE OF Dr.Ahmed Maher (
)Eliwa
At5
Blurring of vision in a diabetic patient
Fundus uncooperative patient
St 1
Abdomen
Hepatomegly with no signs of CLD
Chest
Obstructive airway disease
Pulmonary fibrosis
St 2
Fever rash loss of weight
X ray lung cavitation consolidation+GN & Nasal
blockage
==Wegenar granuloma==
DISCUSSED IN THE COURCE OF Dr.Ahmed Maher (
)Eliwa
I start my exam with station 3
Cvs:it was case of shortness of breath diagnodis
wasMVR with pulmonary HTN In AF question was
about AF managment , B blocker contraindication ,
target INR for mitral valve replacement
Score 19/20
CNN case of difficulty in walking in young patient
finding was pallor, jaundice with hemiparesis lt side q
was about causes of hemiparesis how to investigate
and how to ttt
Score 18/20
Station 4
yrs old lady on renal dialysis with past history of 80
stroke after which she become blind she experience
wish to stop dialysis if her condition become worse
and the renal team decide it is time to stop dialysis she
is drowsy with shortness of breath and expected to die
after 3 day if dialysis is stopped speak to her son about
his mother condition
It was tough and I don't know how I will manage I
Remember the consequence of Dr. Zain I start with
same manner after greating and permission of note
and if any relative wont to attend , how much he know
about his mother condition he know little about it I
clarify her condition and the need to stop dialysis to
her and I ask if he know that his mother she has any
wish and he know about the wish of his mother he ask
to take mother home since dialysis is stop I counsel
him about the need for her to stay in hospital for her
best interest his concern was about his mother
condition and if he is able to take her home and after
how many day she will die I tell it will shorten her life
then summarize and check his understanding the till
me still u have time I don't know what I will tell more I
didn't discuss about DNR
Examiner q was about issue
And why u will keep patient in the hospital and what
about the wish if her son
Itwas v.bad station for me
Score 10/16

Manchester pacess exam


S1
Abd renal transplant on tacrolimus
Chest IPF
S2 drug photosensitivity (doxycycline)
S3
Cardio AS
Neuro spastic paraplegia with cranial nerve (MS)
S4 stroke pt admitted in surgical ward developed bed
sore infected with MRSA
to speak with his daughter
S5
A
st time seizure no obvious cause1
B
Black stool for 3yrs no obvious cause

Dubai exam 8/2/2016


Station 1
Respiratory.. Pulmonary fibrosis
Abdomen.. Transplanted kidney
Station 2. Bloody diarrhea with skin rash and history of
joint pain and mouth ulcer
Station 3
Cvs.. Young female with mid sternotomy scar and MR
...and xanthelasma and tuberous xanthoma
Cns.. Spastic paraparesis with normal sensation
Station 4 pt with post hip surgery and on heparin plus
aspirin felt during session of physiotherapy and
develop intracerbral hemorrhage... Discuss the
situation with Her son
Station 5
st case... Pt with proximal mypoathy and morning 1
stiffness... Dd. Polymyalgia rheumatica.. Polymylitis
nd... Neck swelling with dysphagia.. Solitary thyroid 2
nodule. No lymphadnopathy.no symptoms of
hyperthyroidism. Wt loss

EGYPT 8/2/2018
Station 1
..splenomegally( HC)
station 2
young girl with HTN and protein and RBCS in urine my
diagnosis was Igm neph. Which was appreciated by
. .examiner
station 3
.. ms with PHT and opening snap..+ spastic paparesis
stat. 4 stroke for telling the relative
. stat. 5 hyperthyroidism +pemphigus v

EGYPT TODAY 7/2/2016


Station 4
Pt with haemorrhage and inoperable shewanoma bbn
deal with wife concern (from the source <<<BUT
Dr.Ahmed Ahmed Maher Eliwa CORRECT that by :: The
case of communication was astrocytoma not
shewanoma)
Station 5 first short stature
Second psoriasis exacerbated by b blockers
Station 1 jau and huge acities with vitiligo
Chest cold with fibrosis
Station 2
headache with menorrhagia
Station 3
Neuro : lower stroke in yong pt
Cardio : arortic stenosis with aortic regair
EGYPT-kasr alaany -- cairo 6-2-2016 first cycle
st 1 abd young pt wz large splenomegaly
chest :- lt lung fibrosis wz OLD
q how to investigate what treatment of fibrosis
st 4 motor neurons dis
concern what if symptoms aspiration and weakness , m
living alone
recure no one to help me
st3 neuro pt wz LMNL & down going planter a symm
weakness loss of sensation ididnt finish examination
st 2 diarrhea 3month
difficult to flush and smelly 3times per day more wz
fatty food wt loss 5kg in 3 month good appetite history
of pneumonia received amoxicillin 4 months ago for
1wk concern is it cancer
st 5 knee pain in acromegaly pt i didn't do visual field
was asked by examiner
case 2 tiredness
history anemia melena epistaxis loss of wt not know
how much examine pallor
they didn't allow abd examination and red spots on
tongue concern is it
serious
also there was young pt wz malar flush no one heard
murmur but she had pulmonary hypertension

Glasgow PACES March 3, 2015

:Station 4

Delayed diagnosis of pheochromocytoma

Mr, jones 35 years male

Had High BP for last 5 years


Seen by psych for panic attacks
Tried many Med for HTN
But
His BP has been difficult to control

On his insistence , his GP has referred him to


hypertension clinic 2 weeks before

Results of tests now show


Urine : high metanrphrines
CT adrenal : 5 cm mass in right adrenal

Ur task is to explain the diagnosis


U don't need to know the details of further tests and
further management

Patient was concerned


Is it serious
Is it cancer
Is there a cure
Will I require future surgery
What future tests will be done
Was the delay justified
What medicine u will give me

Examiner : repeated similar questions

Overall not too harsh patient


Satisfied at the end
Agreed follow up GP Consultsnt website address alpha
blocker beta blocker
16/16

History station at same centre

Opening : 11 points

Discussion : as under

Young female 28

Blood Diarrhoea after Cyprus visit


Started 1 day before coming back
Mixed with stool
Similar episodes for last 2 years
Took amoxicillin in Cyprus
Diarrhoea aggregated
Now last 10 days
Frequent blood a salime in still
Painless
C/ o small joints pain
No backache
No other extra intestinal symptoms
No oral ulcers
No skin changes
No jaundice
Cousin IBD UC
Father CA colon
No blood thinners
No steroid
No warfarin
No bleeding disorder
No weight loss

Concern : cause
? Is it cancer
? What next tests
? What Med
? Need admission or not

DD: xIBD ( UC)


Infective Diarrhoea
Antibiotic associated Diarrhoea ( as patient said
Diarrhoea aggravated by amoxicillin ( but I told least
) chances
Explained to patient in detail and agreed a plan

: Closing
summary

Labs / Leaflets / NHS choices website


Agree

Examiner : just repeated all above

And
Asked
In
; History how will u rule out infective cause
Fever
Vomiting

But
He told
U will ask about symptoms to others accompanying
him

Marks : 4/20

History & Communication : zero


Concerns : zero
DD : zero
Judgement : zero

In feedback : written

: History and communication


PC
HOPI
Past Hx
Personal
Family
Drug
Allergy
Treatment
: Social
Occupational
Travel
Association of IBD

Used jargon : IBD ( during explanation of DD to patient


)
Oral ulcer ( mouth ulcer should be used )

Didn't get more details of past episodes

Result : zero marks

DD: 1st diagnosis was IBD


But actually it was infective Diarrhoea

Result : zero marks

Concerns : though addressed adequately but remarks


are he left patient worried about the diagnosis (
serious diagnosis as IBD)

Result : zero marks


: Clinical judgement

Question 1
: How u investigate
CBC to look For
CPR ESR
Electrolytes
Renal functions
LFT
Stool microscopy & culture for infective Diarrhoea
Sigmoidoscopy / colonoscopy if required

Question 2
Treatment: as per diagnosis if it's UC
Then steroids and mesalazine

If infections : antibiotics

Remarks : want to give steroids though preferred


diagnosis is infective Diarrhoea
Candidates remarks : This happens in real life
Though I was expecting 100% 20/20
But
. Actual 4/20

:UK PACES experience

I want to share my experience in Western General


...Hospital, Edinburgh 25 Feb 2016

..I started my exam by station 3


..Cardiovascular; 50 year old man complains of SOB )3
I did the exam, I appreciated a murmur in apex.. I could
.. not time it
for unknown resean I said it is diastolic murmur
considering I do believe that diastolic murmur can not
..be brought in PACES
The examiner ask me if that was diastolic murmur
what will be your differential.. at meet the patient
after the exam at hospital gate and he told me he has
) 20/8 ( AS and MR !!! I scored
CNS; lower limb exam.. patient was not cooperative )3
and misleading
he kept moving his lower limb during tone assessment
and giving contradicting information during sensory
..exam.. I could not formulate DD
) 20/7 ( I scored

Communication: 40 year old lady has IDDM her )4


HbA1c 9 referred for albuminurea
I was disappointed from previous station and forget to
ask her if she does attent all foloow up appointment ,
??does she check her glucose
)20/4( I scored

BCC1: psoriatic arthritis has joint pain.. has skin )5


)rash over elbows and hair line.. I scored ( 28/28

BCC2: 70 year old lady history of loss of )5


consciousness and abnormal movement, had murmur
during adulthood for which she does not require follow
..up
My DD : epilepsy and stroke
.. I could not appreciated any abnormality in exam
I instructed her not to drive for 1 year and to inform
..DVLA
they ask me if I appreciate any murmur.. I answered
!!No
)28/24 ( I scored

Abd: kidney and pancreas transplant , has gum )1


hyperatrophy and poor vision.. I said the cause is Type
.1 DM as patient has vitiligo
) 20/20 ( discussion about complication of transplant

Chest: Rt upper lobe lobectomy with deviated )1


trachea discussion about indication of lobectomy and
) types of lung cancer ( 20/20

History: 55 year old male with symptomatic )2


.. anemia and melena on ibuprofen for knees pain
??His concern: Is it colon cancer
..I told him I ll request upper and lower GI scope
) 20/19( I scored

)172/130( The End Result is PASS

.. It was My first trial


I have never been to UK before .. I had course in Ealing
..Hopsital, London for 2 day ( it is excellent )
Despite the bad beginning .. Still AlHamdullah I
..passed

..My Advice .. do not be relactant in applying to UK


My English language and accent is not perfect however
!! they consider that

:UK experience of one colleague

Castle Hill Hospital


.. tough
...

Station 2
years old .DM.asthma presented 25
With recurrent chest infections for 6 months 6 times
I put DD bronchiactesis .TB
No Hx of fever .wt loss or travel or contact with pt with
chroinc cough.he has greenisg sputum..constipation..I
did not understand his accent clearly
He continue mentioning constipation and trying to
have a baby and I totaly ignore it..his concern why I
have this recurrent infection
His diabetes and asthma are not well controled I asked
about HIV risk which up set the examiner
I forget sinusitis and examiner was angery and
.heampotesis as well
I told him we are going to do bronchoscopy..also upset
the examiner
He asked me about d ...my dd was bronchiactesis and
TB
He asked about one blood test for specific for
bronchiactesis
I told I do not remember
.. He said serum antibodies for pathogenes
I was about to say immunoglins but bell rang

.CNS: Upper limb exam .3


He has hemiparesis
I did not finish sensation
Not examin e nech
He had truma with scar in head which I did not notice
.even when examiner point it
He ask me if you notice any facial asymetry I said
no..which acutaly was present

:CVS .3
A tall women I wasted time looking for alchol gel for
scruping and washing hands with water

Marfan syndrom with 2 sacrs on medisternotomy scar


with metalic clikc and aother an rt subcalvicukar..no
muremur but 2nd sound was loud and palpable..first
was soft
My d..aortic valve replacemtn
He asked about causes of chest pain in marfan
I told ACS
And pneumothatx he asked what else which I can not
answer

She had high arch palate and archenodactyly..I think by


other cause of chest pain he wants rupture anyuresm..I
just remember it now

:Communication skills :4
Staion 4 ...80 years old patinent..Alzehimer d...was on
NG feeding and she was agreesive and agitated all the
time and use to pull it out..her doughter facing
problem with feeding and want PEG tune insertion
..speak to her doughter and explaine ill_terminal care
...and palliative care for her
I do not now mentioning DNR waa suitable or not but I
..have mention it
Examiner asked about how are you going to feed her if
.. sh will not take oraly no NG no PEG tube

:5
Station 5 was diffecult
years with skin lesion over her forhead and scalp 60
Looks like morphea
Some candidate mentioned SLE
Apart from that she did not have any manifestation of
scl
? eroderma ..her concern is it a infecious
?Is it cancer
I reassure her ..but examiner asked what could cause
morphea
Second case 62 years old ..with blurring of vision
.exssive fatiguabilty..and more blurred by the end of
the day..deffintly she had exopthalmous and
opthalmobligia..diplopia on both lateral
gazes..thyrodyectomy scar and left firm thyroid
nodules
Dry hard skin..fundus normal..no other manestation of
..thyroid ..no proximal myopath
I told dd
Graves opthalmopathy and
Mysthenia graves

:Station 1
chest bilatral basal fibrosis and skin rash..I do not
now what is it...some candiadte examiners told them it
is dermatomyosistis..it was not typical she had hard
.skin..finger tips ulcer as well

Abdomen...abdominal pain
I could apprecaite 2 masses in rt side and one mass in
left side not liver not spleen...it was transplanted
kidney ..examiner asked why she is going to have
? abdomian pain

?What about immune supression side effect


...Examiner questions was more tough than the exam
But it was nice experiance
..Keep praying for me

Examined in Egypt last diet

My experience in paces exam in cairo 3rd day 2nd


carosl
score 154/172
Dr.faisal hemeda 2016
I started with station 3 neuro
first was a case of spastic paraparsis without sensory
level
I finish my examination before the time by one and
half minute :D
and the examiner told me if you wish we can start the
discussion now i told him NOOOO :D i not finish yet i
would like to examine the gate to waste the remaing
.. time
DISCUSSION
? what ur DD
MND.1
Hereditary spastic paraparsis.2
i told him i want to take travel H to exclude tropical .3
spastic paraparsis
MS (he asked me what against MS i told the age and .4
the symmetry of the symptoms )
? what ur inv
.... Basic
= specific
EMG and NCS
MRI to exclude MS
? what ur ttt
according to the cause
he asked me about the planter response i told it was
not upgoing he asked what we call it ? i answered
eqivocal
the examiner was stasfied
score 20/20
nd caseCARDIO was a disaster for me :D2
my college in the same carosel who know the pt told
that he has douple mitral and mixed aortic and P HTN
I SAID ONLY MR WITH P HTN
discussion was not satisfactory for me rather than the
examiner
any way I score 19/20
station 4 cmmunication was ctastrophic
explain to this gentlman about dailated cardiomypathy
)\ i did not know any single useful info about this dis(
i follow the instruction of dr.@Ahmed Maher Eliwa i
start by def and cl/p and causes and compl and
mangement
I keep telling the surrogate you have to take ur RX
he asked me about the surgical ttt for his condition i
told him we have protocol to start with the medical ttt
first and according to ur response and the MDT team
!! decision cuz i didnot know the answer
the surrogate was a young man 25 y but in the scenario
he is 55 years and have 2 child so i got a little shocked
when he told me that he has 2 kids one them is 15
!! years
i made summary and checked the understanding
DISCUSSION
examiner have a strong english accent i did not get him
in every qs
he asked me 1.how you dare to tell the pt that he have
to take RX ?did not you know about AUTONMY and the
pt has the rt to refuse any ttt ..SHOKED ..SWEATY ..
PALPITATION ... I FELT
!! he told me i give u chance to repeat ur phrase again
then he asked me about the surgical ttt for
cardiomyopathy
drug ttt for it and the role of every medication in the
mangemet
score 10/16
sation 5
case was young pt with chest pain start this moring .1
stabbing with history of leg swelling last weak
score 26/28
neck swelling.2
the discussion was good but i did 2 stupid things
first one i pretended to be calculting the pulse while i
was not and i told him there is tachy he told me
howmuch ? i told did not calculate
i told that there is bruit but here was not and that .2
was my last word before bell
rest of discussion was Ok
score 24/28
Station 1
abdomen
splenomegly bout to cross the medline
? DD
? signs of portal HTN
what other dd ? CML
no ascitis no LN pt under built
discussion was good
score 20/20
chest
i found the pt have COPD Basasl fibrosis and area of
!!!! brochiectasis on the rt side ... i was sure
the examiner shoked told me 3 finding !! are u sure
i told him yes that what i found and discussion about
inv and ttt
when i mentioned postural drainage he told me if
!! there is brochiectasis
i was sure but it seem that there was not any way i
insist
Score 17/20
station 2
history
] patient with DM 1 with wt loss
actually my dd before entering the room was
addison.1
malaborsption like.2
coiliac.3
inflammtory bowel UC.4
autonmic diarrhea.4
panhypopitiurism.5
the discussion was going was postive dizzy spell but no
abd pain or hyperpigmention to support addison and
no PN or Diarrhea or frothy urine to support autonomic
!! any way i answered the concern as addison
examiner asked me why not TB i told him there is no
fever and no chest symptoms and the concern of the pt
? was why not cnacer
score 20/20

I had my exam in Brunei on the last day in second


schedule. Exam was tough with some atypical cases,
but ALHAMDULILLAH (All praise to Allah), I passed it. It
was my first attempt. My sincere thanks to PACES
EXAM CASES and all it's contributors, esp. Bebo bebo
and Mahiuddin. I had been a silent observer here. Dr
Mahiuddin gave a lot of useful tips here which really
helped me. I also thank to my all teachers esp Dr
Abdulfattah, who taught me the basics of this exam in
.a very simple way. I would like to share my cases here
Respiration: Young short lady, with SOB. Patient .1
could not lie down, so all examination in sitting
position. No clubbing, central trachea, B/L basal
crackles not fine but doesn't change with cough as
well. My diagnosis Pulmonary fibrosis, Other DD
Brochiectasis. Examiner asked about diagnosis and
different causes. British lady examiner was very
cooperative and she sensed my nervousness as it was
my first ever PACES station, that also respiratory (time
.taking) and plus young lady
.I got full marks
Abdomen: Obese man, round face, and abdominal .2
striae; with active fistula at left wrist. Few scars in the
neck, left subcostal scar with few scars beside it. No
hepatosplenomegaly. I felt some fluid hitting my hand
when patient turned his body. It was a very difficult
palpation. I got shifting dullness as well (??). My
diagnosis- Patient with end stage renal disease on
haemodialysis, most probably on steroids, cause could
be due to Glomerulonephritis. Examiner asked me why
he had ascites. I said due to volume overload
(uraemic). Then why not pedal edema? I told may be
partially treated. He asked for any other reason for this
ascites in renal patient. I told he might have peritoneal
dialysis, which could be reason for fluid. He asked me
for any proof? I showed him the scars on abdomen. He
said it could be due to surgical drainage. I said it could
be. Then he repeated the question, any other reason
for ascites in renal patient. I was very nervous and
.couldn't answer further and the bell rang
History: Middle aged man with SOB and leg swelling .3
and past history of recurrent chest infection. I finished
before time. Examiner asked me about diagnosis. My
diagnosis Bronchiectasis with cor pulmonale (right
heart failure). He asked me of any other possibility. I
could not get it. He asked me about complications of
bronchiectasis, I said local and systemic. He asked
further about systemic. When I told amyloidosis, he
asked, "could it affect kidney" . I told yes, it can cause
Nephrotic syndrome and that is one of the possibility
in this case. He was very happy to hear this from me
.and he gave me thumbs up
Nervous system: Middle aged lady lying down with .4
her right hand near body and wrist looks dropped. I
asked her to put her hands in front and turn the hands
up. Initially the right wrist was dropped but slowly she
raised it. That added to my confusion. I immediately
started typical upper limb examination. Power 4/5 in
the right upper limb. Tone - normal, reflexes - absent
bilaterally with negative Hoffmann. Sensations - I
checked pain and vibration only, due to shortage of
time. And both were reduced on the right side. There
was no obvious facial deviation. I was fully confused. I
went for common thing first and said it could be stroke
in spinal shock. British examiner asked me the proof to
support my diagnosis. I told it is difficult to say without
examining the lower limbs and cranial nerves. But the
typical pyramidal pattern of weakness with unilateral
sensation loss of all modalities could be the clue. She
asked what did it mean by pyramidal weakness, I said
"even though it is more typical in lower limb here I can
see that abductors of shoulder and extensors of elbow
".and wrist are weaker, giving the typical posture
I got full marks ( I can't believe, I am still not sure about
.diagnosis)

CVS : Middle aged man, with midline sternotomy .5


scar. Dual valve replacement with MR, AR and AS, with
chest congestion but no pedal edema. I forgot to check
thrills. British examiner did not agree with my apex
finding, which I immediately accepted. He asked me
about diagnosis and complication. It was a typical
.station
Communication skills: Young man from military was .6
referred by GP for further check up as his brother died
of HOCM last year. His ECG done by GP was normal. He
had appointment for Echo after 2 weeks but still
couldn't get appointment for genetic studies. He was
not eager for further tests and had concern that his life
would be disturbed and he might lose job if it came out
to be positive. He started aggressively, Alhamdulillah, I
tamed him and convinced him. My MRCGP skill helped
me. Examiner asked some typical questions and also
what would I do if he didn't turn up for further
investigation. I told I would take the help of GP or
employer to trace him back. Chief examination
.coordinator was present during this consultation
.I got full marks
BCC1: The coordinator confused me with other .7
case. I lost some time in confirmation. Young lady with
decreased vision of sudden onset in both eyes for 2
days. Diabetic for 6 months, not following up, not
controlled. Father had glaucoma. Past history of
gestational DM. She could only read the top line of
chart. Field normal. Before I started fundoscopy,
examiner informed that two minutes were left. I
looked in the right eye, there were black pigments
suggesting retinitis pigmentosa. I had no time to look
at optic disc or macula. I told I would like to refer her
urgently to Ophthalmologist and also check her blood
sugar. Examiner asked me about diagnosis. I said it
could be due to osmotic changes in the eye due her
uncontrolled sugar. She asked me about anterior
chamber. I said I could not examine due to shortage of
time. As there is no pain the chances of glaucoma is
less. As it is acute and bilateral, Retinitis pigmentosa
can't explain this. She asked me about complications of
DM, I answered everything except Retinopathy (funny?
I felt very depressed that how I forgot this... Exam
.tension). I am still not sure about diagnosis
BCC2: Young lady with hand deformity. She had .8
pain in hand joints and backache. Fingers were
deformed just like rheumatoid arthritis. Nails were
normal. On asking I got to know she had rashes over
elbows which were well hidden with clothes.
Alhamdulillah I got it. I examined her properly. I
managed the time very well here. Examiner asked me
about diagnosis I said Psoriatic arthritis. Then he asked
about type of deformities, signs of activity of disease,
.chest findings and management
.I got full marks
Alhamdulillah, I passed the examination comfortably.
.All praise to Allah

My experience for PACES in Tameside hospital near


Manchester 6 Feb 2016
Station 1
:Abdomen
Young male with fine tremors on outstretched hand
and skin warts. On abdominal examination: there was
a left iliac fossa scar with a mass under it, also there
were multiple abdominal scars. My diagnosis was left
renal transplant with previous peritoneal dialysis
history, Patient is mostly on tacrolimus or cyclosporine.
Discussion was about causes I said normally it's
diabetes but for the patient' age can be ADPCK, GN,
obstructive or reflux uropathy. I scored 20/20
:Respiratory
About 50 years old male, on general inspection there's
peripheral cyanosis, also plethoric face with multiple
telangiectasia. On chest ex: bilateral fine inspiratory
creps that didn't change with cough. My diagnosis was
Interstitial lung disease; patient may be on steroids.
discussion was about causes. I scored 20/20
====
Station 2: History
Young female presented with 3 episodes of rash on sun
exposed areas, last one associated with blisters on
both arms. On history taking: Patient mentioned going
to the beach before last episode, no symptoms for
rheumatological, connective tissue disease or
abdominal complaints. Past history of acne for which
she is taking doxycycline prescribed by GP. My
diagnosis was drug eruption caused photosensitivity
rash accentuated by sun exposure, my differential was
porphyria cutana tarda, dermatitis herptiforum and
pemphigoid. Discussion was about the management:
investigations ANA, TTG, eosinophilia, stopping the
offending drug. I scored 19/20
====
Station 3
:CVS
More than 60 years old female, peripheral examination
was with in normal, heart revealed systolic murmur on
the aortic area radiating all over the pericardium, also
soft S1. My dx was Aortic stenosis, the presence of soft
S1 make associated Mitral regurgitation is possible,
Discussion was about the difference between Sclerosis
and stenosis. Plan of action and indications of surgery. I
scored 20/20
:Neuro
Young male with spastic posture on general inspection,
on Examination: Hypertonia and hyperreflexia with no
clonus, there is some cerebellar involvement, no
sensory involvement, I finished examination early so I
had time so preceded to examine the eye which
revealed bilateral nystagmus, isolated left 6th cranial
palsy. My diagnosis was MS. Discussion was about my
differential if no eye examination was performed.
Management plan for MS including investigations, ttt
of acute, progressive ds and symptomatic ttt. Score
was 19/20
====
:Station 4: Communication skills
Patient admitted with stroke, there was no places in
stroke ward so admitted in surgical ward. There was no
improvement in his condition, he developed bed sores,
isolated swab revealed MRSA. I had to speak with his
.son
I divided the case into 3 sections: 1st one regarding the
admission, then explained bed sores and care about it,
then I explained regarding MRSA infection. Initially the
relative was angry but after calming him down and
explaining the situation and we will do he calmed
down. I scored 16/16
====
Station 5 was quite unexpected to me as there was no
physical findings at all
:BCC1
years old female presented with melena on and off 60
over 3 years duration. On history taking there was no
positive data toward a specific diagnosis. No history of
epigastric pain, liver disease, no NSAIDs, no alcohol or
use. On examination also nothing was steroid
appearing, I examined the abdomen, aortic area for AS,
looked for signs of pallor, mentioned the need for BP
and digital rectal examination. Explained the need for
further blood investigations, OGD as outpatient and
may be colonoscopy according to the results.
Discussion was about the causes, management plan. I
didn't feel good at this case. I scored 22/28
:BCC2
years old female presented with history of Loss of 23
consciousness and jerky hands and leg movements. No
tongue biting, no incontinence, no frothing. CT brain
done is normal, electrolytes and initial investigations
with in normal. Had mild chest discomfort before it. No
attacks previously. Hi

General western hospital edinbrough 6/7/2016


History station fever for dd (mostly lymphoma)
Neuro parkinsonism
Cvs: i couldnt get a clue intially but the examiner asked
me to feel the pericardium and it was pacemaker/ ICD
Abdomen: bilateral kidney transplant
Chest: copd
Communication: missed FNAC result and breaking bad
news of uncertain diagnosis of cancer
Stat 5 : pulmonary embolism
Ank spond. With new diagnosis of parkinsonism

) passed PACES IN UK (
Here is one of New PACES scanerio

Glasgow college
2016/2
July 3 , 2016
Manchester

A 37 years old patient accountant has been found to be


hypertensive , his ABPM confirmed BP 160-170/110-
100
He lives an active life style
With daily exercise and plays tennis three times a week
.
His renal functions , CBC and liver functions are normal
His total cholesterol is high 6.0 mmol

Your consultant wants to rule out secondary


hypertension and planned for CT scan of abdomen and
.24 hours urine collection test and Doppler USG

Kindly discuss with patient about the diagnosis , and


. further management and address his concerns

Good morning
Introduce
Relax patient
Agenda
Rapport
Anyone with u
Anyone to attend the session
Notes taking

? What patient knows

!Then explained diagnosis


Of high blood pressure
Life style
Diet ( salt / fats / vegetables / fruits / meat)
Walk
Exercise
Smoking
Alcohol
Drugs
Job
Avoid tension / anxiety
Home blood pressure monitoring
Smoker 5 cigarette per day
Family h/o premature Heart disease
Father died with MI 49
Mother healthy
Brother had Angina at age <45
Alcohol
Tension at job / busy schedule but still manage to go
walk / exercise and play tennis
Married
kids 2
All healthy
Discussed smoking cessation 1 word
Dietician referral 1word only

? Explained the need for further testing


? High risk for cardiovascular disease
So
Need to stop smoking

Blood pressure medication ( amlodipine )


? Patient was asking Side effects
? What Diet
!
? Any further tests
? How u will perform these test
? Do I need to be admitted

Explained further tests ( by making a picture of kidneys


/ adrenal / kidney vessel )
hour collection of urine for urine metanrphrines 24
Doppler to see blood vessels
CT scan to see adrenal and for possible Pheo / conns

That's it

Any other concerns


Satisfied patient

Check understandings
Closure
Leaflets
NHS choices websites
Wrote spellings for Hypertension / pheochromocytoma
and told patient to read on website before next
appointment Sothat if any questions
We can discuss

Thank you

Examiner asked each point

That did u discuss this


Did u discuss this
What tests
? Y u do these tests
? Y Doppler
? Y urine test
? Y CT scan
Did u told patient about side effects
? Of amlodipine
? Y u didn't give ACE inhibitors
? Y not beta blockers
? Can we give thiazides in this patient
Result : 16/16

History was also new for me ( will send later to u)

I have passed my PACES exam in Mandalay center


recently.Thanks to Dr Bebo Bebo and other friends in
this group for sharing invaluable experiences.I w'd like
.to share my experiences
Abdomen.heptosplenomegaly w .1
anemia.Q.finding,dx,ddx,mx.14/20
Resp.moderate pleural effusion.w tracheal shift-
Q.finding,dx,ddx,mx.18/20
.History.2
unilateral Headache.in female 30 yr.not relieved by
.simple analgesics,pizotifen and sumatryptan
Pt have used OC pill for 6 mth then GP asked to
stop.not related to OC pills and not improved by
stopping it.no features of migraine.cluster.increased
.ICP.stress present at work and related to HA
.I said tension HA and migraine as DDx
Q.how to invest.to differentiate.I said clinically and by
.response to drugs
Q.how to manage.I said I want to do full neuro.exam
and trial of other analgesics like ibuprofen,diclo. and
.reduce stress and follow up for new symptoms
.Q.how to reduce stress .l said biofeedback and CBT
It is not fit to typical history of any paticular HA and I
think examiners want discussions about possible
ddx.18/20
CVS.MS with valvotomy scar with AF .3
Q.finding,dx,ddx,mx.simple case 15/20
CNS.examine the lower limbs neurologically
. flaccid paraparesis with indwelling catheter
I examined tone.power.reflexes,planter.pinprick and
joint position sense in time.Forget and do not have
.time to examine the spine
. DDx.cauda equina and peripheral neuropathies
I said cauda equina and ddx are peripheral
.neuropathies like lead poisoning,porphyria,DM
Examiner asked about pattern of neurological deficit in
each d.dx,then mx.I said CT or MRI spine,bowel and
bladder care..treatment of underlying cause.20/20
.Station4-medical error
pt with psoriatic arthropathy taking methotrexate was
.given trimethoprim for a UTI
.pt was admitted for nosebleed with pancytopenia
I apologize very early after taking rapport and checking
pt's prior knowledge about her condition,I said we
shouldd't have given that combination as it have led to
serious damage to you.Surrogate show only little anger
and with repeated apology ,she accepted.Ask if she can
conplain,I said yes and explain I will help her to write
.conplaint to PALS
Concern.if she can get recovery and when will she can
restart methotrexate or not.I said it depends on
recovery of her blood cells and I will ask my consultant
and if necessary will get opinion of joint
specialists.when can she go home.? It depends on her
codition and I will let her know after checking her
recovery.Then I summerize and check pt's
understanding and say thank you.finished early and we
have to sit in silence for 5 mins!Examiner warned me
to say something to pt but we have not much to say at
that time.What ethical issues,?I said truth telling about
our mistake,.non.maleficience, beneficience 14/16
My BCC cases are interesting and I got dx only in last 2
!minutes somewhat luckily
BCC 1.a 25 yr old man with repeated blood
transfusions since 5 yrs of age ,presented with
fever.,high colour urine ,tiredness
.Examination show moderate splenomegaly and pallor
Pt's concern.what is his problem?I said thalassemia
?intermedia.Why he has fever
l said UTI or malaria or other sort of infection and I will
do blood tests.How can you help me to reduce
?transfusion interval
I said you have a big spleen ..that is why it destruct
your blood cells and U need blood transfusion.You
need operation to remove spleen to reduce transfusion
.interval
Examiner ask finding .dx.Why he has fever.?I said UTI
or other infection.not satisfied.Why fever in this pt
?.with splenomegaly
I thought long way and said he may have
hemochromatosis leading to diabetes leading to
immune suppression and infection.Any other
pissibility.? I said hypersplenism leading to
pancytopenia leading to infection.Examiner was very
happy to hear it.How to mx,,,? I said neutropenic
regime.not satisfied.What is definitive mx,?I said
!splenectomy.Examiner.happy
?.what will u do before splenectomy
I said vaccination.For what? for encapsulated
.bacteria.Time was up
28/24

BCC.2.50yr old smoker present with cough for 2 weeks


not responding to 2 courses of antibiotics.pt said cough
worse on lying down but no other symptoms.I ask
other chest and CVS symptoms and did chest
examination and found no abnormality.Pt asked what
!is his problem and I didn't know dx
I replied it will be chest infection or heart problem and
I can't tell exactly at this stage and I will do some blood
tests and imaging of chest .Is it serious?is it cancer? I
said he has no sign of cancer at this stage although it is
still a possibility as he smoked heavily.I will do tests to
make sure that everything is OK.Then.I thought that
this pt must have some signs to be in exam and it
appeared in my mind that he had a hyperresonant
percussion and reduced BS. I quickly said to the pt that
he has a condition called COPD and I will give him
inhalers and some tablets.pt quickly asked is it related
to smoking and I said yes and advised to quit
.smoking.Time left only 2 min for discussion
Examiner asked my findings and accepted.Any other
sign that show other specific dx?I said no.He
accepted.As my dx is COPD any other ddx?I said
asthma but no wheezing and rhonchi.Any other
ddx?HF but no other CVS symptoms.accepted.Any
?other dx for cough worsen by lying down
I said GERD and examiner was very happy to
hear.What advice will U give to pt?I said high pillows
and to avoid food at bed time.Time was up. 23/28
There are 2 types of candidates.The first one is very
bright ,smart ,lucky and they can easily passed exam
after studying 2 to 3 months.The second type is
majority of candidates and they have to work very
hard and take a year or more of studying time to pass.I
am the second type and have to study a long time
waiting to get a seat in Myanmar for about 3 years.This
.is my first attempt
.Exam luck is also an important factor
?Then,can we improve our exam luck
As for me,yes.I shared my knowledge to others and
shared some books and mp3 podcasts in this group by
my another account.I had also helped other candidates
with their study and practice so that my exam luck can
!be good.I have met with good natured examiners
In the exam,some candidates said they have time only
.to discuss ddx.They will lose marks for judgements
As for me,I have my own note of common
causes,inv,mx and I memorized them so I can discuss
fluently in 2 to 4 minutes of discussion time and I
reached to management in every station and passed
.every station
!Best of luck to all future candidates
Here is the feedback of Dr. Munzir Al Gadi, who passed
.his PACES in the first run of Malta Centre
Thank you Dr. Munzir for the detailed feedback and
.congratulations again on the well deserved success

My Experience in Mater Dei Hospital Malta on 2/4/16


first carousel

Station 1
: Chest
A young patient with spares head hair( I Said possibly
2 to chemo later on upon discussion and actually I
picked it up as I used to see this finding a lot in my
practice in oncology).. RT side of the chest is depressed
and moving less, RT thoracotomy scar and decreased
chest expansion, impaired percussion and dec breath
sounds
Diagnosis: RT pneumonectomy
DD of etiology was bronchiactssis, fibrosis, Abcess and
malignancy
Discussion was about cancer causes in young patient
(germ cell, and Satcoma ) and workup also asked if he
developed SOB what might be the cause , I mentioned
infection and thrombosis PE
?How to investigate him
)I got 20(

: Abdomen
A middle aged male with features of CLD (D
contracture, P erythema, thenar wasting and Tinge of
jaundice) and splenomegaly I said no ascites
DD and work up
Honesty I felt that I missed hepatomegaly
)I got 16(

:History
A 50 years female , married , works as hospice nurse,
travelled to Kenya with her husband and came back
with nausea,vomiting, fever and upper and pain
radiating to back
Heavy alcohol intake
Had 3 miscarriages at Gestational ages of 26,28,28 no
personal or Fx history of VTE
Gp letter mentioned high T bilirubin 70 and high all
Liver enzymes
? Concerned is it cancer
DD : I mentioned Alcoholic hepatitis, viral hepatitis(A)
and dengue, autoimmune hep, and malignancy
discussion was about working her up , and how to
manage, I mentioned that she needs admission, clinical
assessment and rehydration if dehydrated, pain control
and fever ttt with NSAID and avoidance of
acetaminophen and teat etiology
I emphasize on alcohol cessation referral
)I got 20(

:Station 3
CVS: old male has peripheral features of AR
apex displaced
Systolic murmur all over radiates to carotid
I said AS and AR although I didn't hear the diastolic
murmur , I was not comfortable to the auscultatory
findings and I felt may be something is missing, anyway
, they discussed with me what might be the causes of
systolic murmer in this age and how to differentiate
between AS and sclerosis, investigations to do
)I got 20(

: CNS
A middle aged patient
Instruction was : this patient has problem lifting
objects
I examined his upper limbs , he was sitting on a chair ,
he is non English speaker however examiners helped
with instructions and I passed few instructions in
Maltese my self( most of them sounds as in Arabic)
Findings are pure proximal atrophy and weakness at
shoulder girdle and scapular muscles with defined
supraclavicular and scapular margins, no facial
involvement

DD : proximal myopathy likely congenital causes as


patient has an atrophy
And I suggested scapulohumeral variant I enlisted few
other causes as well
Investigations including EMG,NCS, and muscles biopsy
He asked me about mode of inheritance I answered
that I can't recall
Management is supportive and I motioned that few
Novel therapies is under study
)I got 20(

:Communication
Speak to an angry son of 70+ female admitted initially
in orthopedic ward with # femur and underwent
arthroplasty 2 weeks ago , 1 week after she felt while
doing rehabilitation, since this last fall she is on and off
confused, orthopedist assure son that this because of
UTI and she is receiving ttt for that , then patient
transferred to medical ward as her confusion
continues, CT scan arranged , showed intracerbral
bleed with midline shift, neurosurgery advised to hold
enoxparin ( which was started as prophylaxis) and her
.usual aspirin and stop her oral feeding until he see her
Role : speak with son about CT findings and
subsequent plan and discuss the clinical judgment
.when outweighing benefits and risk of LMWH

Son was angry but I listened to him empathetically and


reassured that I'm here to help, I broke the news of the
CT findings and explain the role of Neurosurgery
opinion, his concerns : what is the cause of her bleed,
why giving anther blood thinner while she is on ASA ,
could the fall be avoidable, why he has been told that
?she has UTI
Actually examiner's discussion revolved around
whether LMWH has caused her bleeding or not and
wether there is a way to know that I said unlikely it
was the direct cause however above therapeutic level
of anti factor Xa might give a clue that helps to reveal
.the uncertainty of her bleeding cause
)I got 16(

Station 5
: BCC1
An old male , c/o slurred speech for 30 minutes, three
previous episodes of near fainting , during episodes he
. feels "fluttering" sensation of his heart
PMHx : HTN on amlodipine 5 mg , AF on pacemaker
and warfarin 3 mg and regular check, ranitidine for
gastritis
Exam : AF with rate of 80

Discussion was about DD


I mentioned TIA , orthostatic hypotension
How to investigate, he ask me will you change his anti
?hypertensive or not
?How do you know if the pacemaker is non functioning
)I got 28(

BCC 2
A young lady, pregnant in 18 weeks gestation with SOB
for 2/52 and cough with occasional whitish phlegm and
occurs at late night and early morning,no any other
symptoms upon discussion
KCO bronchial asthma was controlled before
pregnancy on INH SABA & INH steroids but she
stopped them both after got pregnant as she thought
they're harmful
Examination: all clear , LL clear
I explain for her the role of inhaled Mx in controlling
her asthma and that why she got these sympx ,
reassure about safety in pregnancy, adviced PFM diary
and FU with GP
Discussion: DD chest infection and less likely PE
Examiner asked what've s against infection, also asked
? if PE need to be rolled out what to do

Actually I peaked my marking sheet within the


examiner hands while pill was ringing and I'm about to
leave the room with all marks in satisfactory area , I
felt it was a comfort message from Allah at the end of
the exam
)I got 28(

Over all I scored 168/172

My conclusion that PACES is a MOSIAC experience,


it concludes different roles and various methods and
the probability of passing lies in practising as many as
.. one can do of these roles and methods
Inhance your best qualities and fill your defects and as
Prof Zein says eliminate your chance of failure by
.avoiding the failing practice
Thanks Dr Zain again and again for your support and
effort and may Allah grace you with health and
. serenity

Thanks all members of the group for the endless


effort that helped me and others, may Allah bless you
.all

Dr Munzir Algadi

exam experience gasr al3eni hospital first day 3rd cycle


CVS -1
prosthetic valve mitral with AF
Discussion was so long I finished my examination early
he asked me about indication for replacement ,
treatment, and cause of chest pain in such case, target
INR
Score 19/20
Abdomen
Pale pt with hepatosplenomegally
DD start with hematological cause and still CLD on my
list then he asked me about common cause of CLD in
egypt then how to approach pt and treatment
20/18
Chest
Female with rt apical fibrosis and pleural effusion
Discussion was about causes and treatment but I
scored bad because I didn't exposed pt completely she
asked me not to do she was young and I respect that
but examiner didn't like it
20 /8
Neuro
Peripheral neuropathy gulliam barri and discussion was
about DD and treatment when to admit pt
The funny thing in this station
That before i start i asked her if she has pain any where
and if she felt and to tell me then while am doing tone
she scream of pain I stopped immediately i told
examiner i
Can't continue examination she is on pain he told me
proceed I thought i lost it but al7amdole ALLAH
Score 20/20
History
Pheochromocytoma men
Young pt recently diagnosed with HTN and he had
panic attack he was started on diazepam
Discussion
DD add hyperthyroidism he asked me how u will
explain wt loss in Pheochromocytoma i told him 10%
can be malignant
?Why men
ve family hx+
Symptoms of hypercalcemia
20/16
Communication
I scored bad and I didn't read scenario good
Middle age pt newly started on thiazide for HTN he
was walking on hot weather he drink water then he
had fit
Na was 114
Explain to wife about conditions and prognosis
What i did i explained why he had fit and the idea of
dilution hyponatremia and the effect of thiazide and i
told the wife its provoked seizure but still we need
image to role out other causes
But this part upset examiner he said no need for
further image no need to discuss job and driving
16/8
Station 5
proximal muscle weakness wt gain -1
History everything was negative the only positive that
he is on thyroxine i asked surrogate why he is on
thyroxine because i asked about previous medical
illness he said nothing he told me I don't know
My DD at this point cushing hypothyroidsm
I examine to role in or out one of them it was
hypothyroidism diffuse goiter
Discussion was about investigation treatment
28
middle age pt with lower limb weakness with oral -2
ulcer
Hx was suggest to behecet disease i examined lower
limb neuro and for erythema nodosum,And oral ulcer
Discussion was about cause of weakness how to
diagnose and treatment
28
I attended dr zain course for history and
communication
And for clinical stations dr ahmad maher mock course

This is Dr. Abubakr who passed PACES at Khartoum
..centre (Soba university hospital) with a score of 170
My exam on 2/4/2016 day 2 cycle 1 in soba university
hospital
Station 2 history
Scenario of (35 years old lady has fatigue for 6 months
her gp did a blood test and confirmed to be iron
deficiency anaemia )
I introduced my self, explained my role asked about
her job (she is a teacher) and agreed the agenda
As she has fatigue I started by analysing her fatigue
and then general symptoms and she gave hx of wt loss
of 5 kg.. When I asked about joint pain as apart of
general symptoms she answered that she had joints
pain for 2 years and she had been diagnosed to have
.osteoarthritis by orthopaedic consultant
Then I asked if she use any medicines for that she said
she uses 2 medicines ibuprofen and another NSAID.
These medicines were given without prophylactic PPI.
She has hx of localised epigastric pain made worse by
eating ass with nausea but no vomiting.... some times
heart burn
.there was no melena or haematemesis
No mouth ulcer
No change in her bowel habits
.No bleeding through her back passage
There is no bloating and no tummy pain with specific
type of food (wheat products)
Normal menstrual cycle
She takes balanced diet and she gave me example of
.her usual diet
.Then review of her systems was negative
.Her past hx and family hx are negative
I take the drug hx as part of HPI
In social hx she is affected greatly by her fatigue and
also she can't do her hobbies as she used previously to
.run and go to gym
She is concerned about the cause of her fatigue and
.how can I help her to continue her hobbies
I explained to her the likely cause of her fatigue is
related to the joint medicines and that we need to do a
cammera test and we need to stop her medicines after
discussing this with her orthopedic surgeon and if we
need to continue on it we will give PPI and we are
going to give her iron replacement and after that I will
.reply back to her gp
.Then I check her understanding and thanked her
First examiner question
.Did you ask about smoking I said no sorry
Do you think it is important? I said yes as pt most likely
has gastritis or peptic ulcer disease so smoking impairs
.the healing of the ulcer
? Then did you ask about alcohol
Again I forget
??So do you think it is important
.Yes as it may cause gastritis
Then he ask why you ask about numbness and
???unsteadiness
Because if malabsorption is the cause then B12 may
cause subacute combined degeneration of the cord
Then ask about DD
I put gastritis
Gastric and duodenal ulcer
Malignancy because of the wt loss
Then coeliac disease and IBD
He asked whether NSAID can CAUSE small bowel ulcers
apart from duodenum? ?? I said it is not common but if
multiple then we need to think of Zollinger Elisson
syndrome
???How can endscopy help
Macroscopic we can see if there is the ulcer and we can
take biopsy
???What to test in biopsy
The presence of malignant cell and also H.Pylori
Any relation between NSAID and H.Pylori? ?? I
said I don't know
Then how NSAID cause peptic ulcer? ?? After explain
then again he ask any relation between NSAID and
?? ?H.Pylori
I feel that he need me to say yes so I tell yes there may
be a relation
Then how you will treat H.Pylori? ?? I said triple before
give the name of triple he tell if you stop NSAID what
other medicine you will give to the pt I said
paracetamol
I think they will mark me negatively as I forget ****
important part of social hx but surprisingly I got 20
Then station 3
..... CVS
A young female with small volume pulse and she is
pale
She has chest deformity ... active pericardium with
visible pulsations. The apex is not displaced with
palpable 2nd heart sound and positive lt parasternal
.heave and on thrill
There is a pansystolic murmur in lt parasternal border
with maximum intensity in the apex but no radiating to
axilla with loud 2nd heart sound
I present my case as MR with pulmonary htn then the
examiner asked whether the murmur radiate to axilla
or not??? I said no and the murmur is in Lt sternal
border so its differential is TR , MR, VSD with
.pulmonary htn
Then asked about the causes of pulmonary htn and
investigations
)I got 20(
.... CNS
A young male with stick beside the bed.. Examine the
.lower limbs
There is pes cavus and wasting of both leg with
hypotonia and weaknesses of LMNL but proximally
more than disatlly
.Absent reflexs and equivocal planter
Coordination difficult as power grade 0
.Intact sensations
I wanted to examine the gait but he tell no need
Then I examine the upper limbs with same finding
?? ?Ex what is you positive finding
??? What is your diagnosis
I said LMNL weakness either muscle problem or pure
motor PN but with pattern of weakness proximal I will
go with muscle disorder then asked about how can gait
help you and DD of pure motor neuropathy then
.investigation and management of proximal myopthy
)I got 20(
Station 4 communication
Scenario of delayed diagnosis of pheochromoctoma in
a young male suffering for 5 years. He was seen by
many doctors including a psychiatrist for panic attacks
and have been prescribed diazepam and also has htn
that was difficult to control and on HIS INSISTENCE the
gp referred him to your clinic. You are the doctor in
hypertension clinic... the tests done for him show a
mass of 5 cm in his RT adrenal and urine test also
.positive
Your task is to explain for him the diagnosis and to
...answer his concerns
;I stard as Dr zein taught us
Introduced my self, explained my role, asked about his
job and if there is any one he would like to invite to
attend the meeting and then agreed the agenda &
asked him to tell me more. He was attacking in
nature... feels that he is suffering for 5 years and seen
by many doctors and prescribed sleeping pills but
.without any improvement
I showed empathy regarding his suffering for 5 years
the explained to him that the results are with me now
and that unfortunately it is not as we hope then telling
that it show pheochromoctoma and whether he heard
about it he say no then i asked if he would like me to
explain more .... the I explin pheochromoctoma and
that the good news are that we found the cause of his
suffering and it is curable condition in the majority...
then i explained that it is a growth... he asked whether
it is cancer or not. I explained to him the possibility of
cancer is 10% when I told him it is curable he asked
how? I told him surgery then he is habby and said OK
just removed it now (verbal cue).. I told him it is not
easy surgery, we need to control your blood pressure
first as I am a doctor in hypertension clinic then I am
going to involve MDT. He asked what MDT? I
apologised and then explained it is a team of expert
people including the gland doctor, the surgeon and
anaesthetist and they will make a meeting to decide
.the way to manage him
?? ?He asked when they will decide
.I replied as soon as possible
Then his main concern whether there is negligence or
not
I answered I need to go back to your records to see
what exactly done for you. Then he said doctor there is
negligence and I will complain against my gp....I told
...him it is your right to make a complaint
The he brought another concern about any damage
?? ?happen to him from HTN
I told him l need to examine him and to do some tests
.to see the effects of htn
...I asked any other concern
..He replied no
Then i made a summary ,,, checked the understanding
and told I will reply back to the gp, and offered help
.and leaflets and if he can drive alone
The British examiner
What do you think about this case???then I give
.summary of the case and my plan
???Why you did not tell him about risks of surgery
I answered I just broke the bad news for him and I
don't want to give him all bad news at one time since
he will have a metting with the surgical team who is
going to discuss the risks of surgery( then he smiled)
???How you will treat his htn
Alpha blocker and then beta blocker

Do you think there is negligence in this case???


I give him the same answer for the surrogate then he
???asked is there any damage from his htn
I gave the same answer given to the patient, that I
need to examine him and to do fundoscopy and
.investigations to see if he has damage
???Then he asked again if there is any negligence
I noticed that both the surrogate and examiner are
concerned about damage from htn, then I told him the
pt is suffering for 5 years and not diagnosed this is not
usual and if there is damage happen from his htn OF
COURSE there is a negligence (I Don't know why I said
of course )
Then he asked me do you think the pt is happy and he
???can drive alone
.I kept silent for a while then said yes he is happy
The bell rang
.The British examiner saidwell done
)I got 15(
Station 5
BCC 1
..A 55 years old female with Lt hand weakness
Before shaking the pt hand it seem she is in pain so I
.apologised for not shaking hands bcs of the pain
Then surrogate gave a hx of both hands pain in
disruption of median nerve without any thing in the
... systemic review giving clue to the cause
However in the PMH she was diagnosed to have
hypothyroidism and not on follow up for two years but
.using her thyroxin 100 mcg
No other significant hx
O/E
Clear signs of carpal tunnel syndrome bilaterally and
.more in the lt
.Then I checked for thyroid status
Concerned about the cause of her problem
Other concern whether her lt hand will become
paralysed
I responded to the 2 concerns and explained the need
to check her thyroid status and regular follow up and
.we may need to do surgery
The British examiner said still you have 1 minute
Then again I explained the importance of regular
.follow up
The examiner smiled & said still you have 30 second
.... but no problem you can review your thouhts
Then ask the diagnosis and the DD
The investigation and management
)I got 28(
BCC 2
A 50 years old male with blackout and normal vital
.signs
From hx blackout mainly in the morning and during
!!sleep also I got confused how during sleep
He is a very nice surrogate he told me he bits his
.tongue and wets himself
.I asked any shakes he replied no
I reviewed the CNS and it is negative. then asked about
trauma which is negative, then about general
...symptoms including skin rash
He said yes he have skin rash in his face for 20 years
which difficult to fade away... I looked to the face and
...then felt relaxed as I catch the dignosis
Past hx of htn and on amlodipine 5 mg with no change
.in the dose
Family hx of abdominal surgery in 2 of his sister and
.skin rash in his brother
He is a teacher!!!! and I asked about his school
performance he said it is good and he drive a private
.car and do not drink alcohol
I examined the face for the rash and the the pronater
.drift for any weakness
Checked the trunk for ash leaf spot and examine the
.back for shagreen batch
Offer abdominal examination but examiner said
normal. Offered chest exam again he said normal.
Offered to check BP and he said130/70. Offered fundus
for phakomas he smiled and said Do it
I check quickly as the pupil is not dilated I know that
.nothing will be there
... The patient is concerned about what is going on
I explained tuberous sclerosis
...Another concern about his kids
.Explain that each has 50% chance to get the disease
No other concern
.The examiner: still you have 1 minute
Then I explained the epilepsy and driving but still there
is time then I explained the screening of family and
.gentic counselling
Examiner question
?? ?What is your diagnosis
Investigations and management
)I got 28(
Station 1
Abdomen
Young female looks ill and very pale with cannula in
her RT arm with ting of jaundice, hepatosplenomagly
.and absent stigmata of CLD
I examined only for one group of axillary LN because
.time didn't allow more than that
.Then asked about DD
The discussion about myeloproliferative disorders
)I got 19(
Chest
A middle aged male with obviously depressed lt side of
the chest which was moving less, and a very strange
scar on the lt side only the tip of scar is seen anteriorly
so I tried to go fast to examine the back from anterior.
The trachea deviated to lt with impaired percussion on
lt and decreased air entery on lt and vocal resonance.
However there is increased vocal resonance in lt upper
.part and bronchial breathing in same lt upper zone
Posteriorly same finding and that scar (still confusing
me it look like long thoracotmy scar but surprisingly
there area about 2 cm of normal skin)
.So I presented my case as Lt pneumenoctomy
Then the examiner asked did hear any thing abnormal
in lt side I replied increase vocal resonance. I was afraid
of inventing signs, so I did not mentioned bronchial
breathing but he is very helpful examiner and asked
me what type of breathing in lt upper zone then
confidently I tell bronchial breathing. Then he asked
what could be the cause again I said this could be from
stump .... he smiled & said stump can not cause this?
he asked what could cause increase vocal resonance
and bronchial breathing I said cavity ..... he said yes
now what could be the cause? I mentioned
fibrocavitatory lesion ...he looked satisfied and asked
about the commonest cause and how to
investigate..TB
)I GOT 20(

Dr. Mohammed A Mutalab (who scored 171 in


:Khartoum) PACES experience
My exam was in the last day last cycle 4/4/2016 in
.Soba centre, Khartoum
:Communication Skills
I started with station 4 the scenario about a patient
who have achalasia and underwent a pneumatic
dilatation for the 3rd time but in this one he developed
.eosophageal perforation
It was mentioned that this complication can happen in
.5% of pts and the patient was consented
You will meet his son to explain for him what has
happened and the need for admission for 14 days and
.any issue raised by him
I started by the usual introduction and then checked
what he knows about his father condition then i
.explained for him what happened in BBN pattern
He asked why this happen to his father this time he has
done this procedure twice before.I explained for him
that any procedure has a possibility of bad effects and
it happens in a few patients; in every 100 it happens in
5 patients and no one can predict which one will be
.affected
He said do my father know this? i said any procedure
will not be done unless we explain for pt the benefit
and risk of it and let him to decide which is called
.consent and your father was informed
I told him that we need to keep your father in hospital
for 2 weeks but he refused. I asked why but his answer
was not clear for me but i proceed and explained to
him that this cut or perforation of his gullet will cause
leak of food and fluid to his chest and lungs and this
will cause damage and inflammation so that we need
to give fluid by his veins and medicines called
antibiotics and we need to involve our colleagues in
.surgery
Also i told him if he went home he may develop
complications and deteriorate more and i am sorry to
tell you that he may die . After this he agreed to admit
his father but he wants me not to tell his father i
replied to him this the right of your father to know
.about his condition
Then he kept silent and i asked him do you have any
other concern? he said no and still there is a time and i
wonder how to fill this time but fortunately while i am
thinking the examiner told 2 minutes left i summarized
for him and checked his understanding and thanked
.him
?Ex: what are ethical issues
Me: BBN,dealing with angry relative(realy he wasn't),
.doing no harm and autonomy
Ex: the son don't want his father to know what do you
?think about this
Me: i think this the right of his father to know to
.ensure ethical issue of autonomy
?Ex: any other principle
Me: i think we have to be honest and tell exactly what
.has happened
?Ex: how are you going to manage him
Me: monitoring
NPO
IV fluids
IV antibiotics
surgical consultation
?Ex: why you need to keep him NPO
Me: so no more food or fluid to get to mediastinum
.causing mediastinitis and allow time for healing
?Ex: what do you think the surgeon will do
Me: the management may be conservative or surgical
.but i am not sure of indication of surgery
.then the bell rang
Station 5
BBC 1
young lady with deterioration of her vision in last 8
.months her vitals were ok
I started by open question then i analysed the visual
loss which was mainly at night and there was no eye
pain or headache and the course was progressive and
not episodic then immediately i asked about family
history which was positive her elder brother is blind
I proceed immediately to fundal examination to
confirm my diagnosis and i found scattered dark
pigmentation which was clear in the rt eye also i
couldn't appreciate the disc clearly after i finished
fundal examination i remembered that i didn't assess
her visual acuity i did it & was normal for finger
.counting
i returned back to the history and i asked about
associations of retinitis pigmentosa and other routine
parts of history
and i asked about driving
.which she is not
then i examined again for hearing aids and weakness
only bcz i thougt other associations were excluded by
history
then i asked about her concern? is she going to be
? blind? and what about job
.she was a teacher

i told unfortuanately this is a progressive disease and


till now there no curative treatment but research are
ongoing and for her job she can continue as far as her
.vision can allow we can give some visual aids
the examiner asked about my diagnosis and the
.associations of RP
also what other areas you want to examine i said
cerebellar and peripheral neuropathy he said do u
want to examine her fingers i said yes for
polydactyly.then do you need to examine her visual
field i kept silent he said what do you expect to find i
.said tunnel vision
.then he took me the next pt
BBC 2
A 28 years old male with skin lesions for several years
which are non-pruritic not painful and I expected it to
.be vitiligo
i started by asking its onset duration progression
distribution any starting lesion any aggravating or
relieving factors and involvement of mucous
membranes which were all negative then i request to
have a look. The lesions were raised small yellow
nodules on flexural part of the elbows
there was also another large one on his lateral
epicondyles and also in his back & eyelids and when i
.came closer to his eye i saw corneal arcus
it was clear this pt has xanthomas secondary to
hyperlipidemia then i asked about his FH which was
positive for sudden young death i asked specificly
about cholestrol problem he said no. then i asked
about macrovascular complications and the secondary
causes of hyperlipidemia ( DM,Renal
diseases,hypothyroidism,alcohol and primary biliary
cirrhosis) i asked about smoking and job
.then i examined his CVS which revealed AS
Lastly i asked about his concern which was is he going
?to die suddenly like his family members
I told him that these skin lesions are manifestation of
high cholesrol in his blood and this something run in
family and this high cholestrol harm your blood vessels
and this can cause heart attack and sudden death. We
need to do more blood tests and we will give you
.medicine to lower your cholestrol
Examiner asked me about my diagnosis what tests you
.need to do and how to manage him
Station 1
Chest
The pt was comfortable, peripheral examination was
normal, Trachea was deviated to the rt and rt side was
depressed and moving less the percussion wad
heterogenous(dull+resonant) auscultation there was
fine end inspiratory crackles bilaterally but more on
.the right in upper zones
I presented my findings and said pt has bilateral apical
fibrosis mainly on the rt. Ex asked me about causes i
said most likely TB +other causes of bilateral apical
.fibrosis
.What investigations and management
Abdomen
A female patient with finger clubbing and functioning
.fistula + gingival hypertrophy
.At this point i was expecting a renal transplant
On abdominal examination there was no renal
transplant scar there was huge hepatomegally about
.14 cm bcm and splenomegally 4 cm
.Others normal
I presented my findings and i said the has
.hepatosplenomegaly and ESRD
?Ex: how to correlate them
Me : hepatitis on top of renal failure
?Ex(not convinced):what else
Me: amyloidosis
Ex: what else
Then i remebered the gingival hypertrophy and said
leukaemia by infiltrating the kidneys although it is rare
then the examimer seems to be convinced and asked
.me about investigations and management
Station 2 History
This was a difficult scenario of a young female feeling
fatigue for 2 months she went to her GP who found
high BP and have done some tests which revealed
.proteinuria and haematuria and normal RFT
I put differential of
CKD(stage 2 normal rft)
GN due to wegner's or goodpasture or post
streptococcal or IgA nephropathy
Polycystic kidney disease
Lupus nephritis
When i entered i analysed her fatigue and high BP then
i started by renal system then enquired about cvs/resp
including haemoptysis then i asked about URT features
sore throat nasal congestion epistaxis hearing
lmpairment then i asked about musculoskeletal skin
rash joint pain
All the above was negative

then i started to complete the other systems GIT and


neurology and i found that pt has loss of appetite and
non-specific headache then i asked about
.constitutional symptoms which revealed loss of wt
All other parts of history were negative except she was
.taking OCP for menorrhagia for several years
When the ex told me 2 minutes left i have no idea what
?is diagnosis
I asked the pt about her concern she said could OCP be
the cause and i am planning to start a family does your
?treatment affect my future pregnancy
I told her i need to do more test to determine the
cause of her condion and it is unlikely for OCP to cause
high BP and the treament for your condition will
depend on the cause and there are different treatment
some of them may affect your pregnancy
.and others will not
?Then examiner asked me what is your DD
I told him about the one above
He asked me why you asked about nasal blockage i said
bcz i think of wegner's he said to me you mean
vasculitis as general i said yes but there is no skin rash
or joint pain he asked me could the high Bp be the
cause of her headache i said yes if it is malignant
he asked me how to know
i said i need to do fundal examination he said if it is
normal i said it is unlikely to be the cause of headache
he asked me what is the commonest presantation of
HTN i replied asymptomatic
?Ex: investigations
Me:CBC
Ex:what specific in CBC
Me: eosinophilia
Ex:why
Me:churg-strauss syndrome
Ex:how to manage vasculitis
Me:methylprednisolone and cyclophosphamide
Ex:is this may affect her pregnancy
Me: yes the cyclophosphamide
Ex:other anti-htn can affect pregnancy
Me: ACEI
Ex: other causes of htn
Me: endorinological like pheochromocytoma,
.....hyperparathyroidism
the bell rang
Station 3
CVS
The pt was young all peripheral examination was
normal. Precordial examination was normal except in
auscultation there was systolic murmur allover and
radiating to carotids also there was early diastolic
murmur on lt sternal edge so my diagnosis was mixed
aortic valve disease and no one is dominant
.the examiner asked inv and management
Neurology
The instruction was to examine the lower limbs it was
apparent that the pt has spastic paraparesis with
sensory level just below umbilicus the time finished
before i examine the back i told him i want to examine
the back
The asked about clinical diagnosis DD inv and
.management

This is my paces exam experience in eygpt,it was a


.tough one ,but al7mdolellah kathiran i passed
I started with station 1
Chest: Copd+bilateral lung fibrosis+ some brochiectetic
.changes on z right side
I took 5 mins examining the pt generally and the ant
chest ,the examiner told me that i have just 1 min left
,so i examined z pt back and lymph nodes and sacral
.oedema
?the examiners ,asked me for z positive findings
i told her there are obst changes with end insp crackles
bilaterally ,and medium sized crackels littly changed by
cough ,so there r brocheictatic changes
She asked me what type of crackels again,what invs
? you want to do for him
when i told her lung f test and it will be obst changes
. she asked me just that, i said mixed
why he has these changes i told her pcoz he may have
repeated infections on top of copd
like the usual bacterial inf ,she asked me what other inf
i told TB
what management ,i told pharmaclogical and non
. pharmaclogical ,and i told her all till steriod
I got 19/20
abdominal station
D: decompensated chronic liver disease +huge
splenomegaly +ascites
They asked me what the cause,then what other
infections cause huge spleen i told kalzar and
shcistosoma, what invs i told all till i came to ascitic
tapping ,she said for even small ascites i told her
.according to US
when i said check serum albumin she asked why you
? want to do it
What mangement ?accordingly to dd,complications
?she asked when you want to give antibiotics
Why is he decompensated i said j+ ascites,she asked is
? he j
20/20
Station 2
History: it was advanced breast ca + hypercalcemia
The scenario was tough they just told us she has breast
ca and she was treated with chemo and radio ,she feels
.unwell pls asses her
So i couldnt figured what is happening and i thought
.that i am going to talk to z daughter
So when i entered i shaked hands i was blank,i greet
r u z daughter of ms.maha, she said no i am ms her
maha,so i surpreised and said sorry,then i told her
would you just tell me about your condition,she told
me the story ,she feels drowsy and unwell recently ,i
said may be brain matestsis ,so i asked about all cns
system,then i didnt get anything i said may be
dermatomyositis ,but nothing ,then i asked her about
the treatment ,what she was given and for how long,i
thought it may be tamoxifen induced cardiomyopathy
,but no hf .just sob on moving to bath ,or may be
adrenal metastasis,causing addison disease,but not
typical
Till i came to z water system ,she has ploy uria at night
.
and she is so depressed ,i was lost,then i told her i
want to reherse what i get from her ,i said you have
increase water frequency +depression+ constipation (i
think it may be from morpheine)+ abd pain+back pain
(metastasis)
The examiner told me u have just 2 mins left ,so iasked
about smoking,alchohol,impact & drug history ,then
concern,i told her i want to admitt you and do some
imaging n blood tests ,may be you have some
metastasis,and i want to ask the phsycatry to asses you
and give you some nuritional support and fluides then
time finish
examiner asked me what do you think,and why you
?want to admitt her
i told him i want to give her nutritional support + iv
.fluides+ do imaging
Asked why you want to give her iv fluides i said pcoz
she is not eating,and dehydrated,i want to asess her
.first
?then what else
she is dehyderated
And has polyuria and polydepsia,so it may be
hyponitremia ,then the examiner told me so z pt has
polyuria,depression,abd pain ,what do you
think
i siad hypercalcaemia
What is z management?Rehydration + calcitonin ,he
asked what else ?i forgot z besphosphonate totally
.so replied i couldnt remember
Then asked me what other speciality you want to
.consult,apart from the psychatrist ,i said z oncologist
?Finally what z dd o her sob on moving
I said PE, or metastasis or pleural effusion ,n i will do
.imaging ,but i think he was looking for anaemia
What is z cause of her abd pain ? I said could obst or
metatsis,he said could z hypercalcamia
I said yes ,lastly he asked what abour her social issues

I said i am so sorry i couldnt ask her with whome she


lives ,he asked is it important i said sure because if i
want to admitt her ,she may have some issues to be
.solved.(lives with her daughter who travelling now
I got 10 /20
:Station 3
Cvs :As+Ar with dominant AR
?They asked me what is d?what you want to do for him
What is cause? In this young pt bicusbed aortic valve or
.rhuamtic heart disease
What about his pulse rate? large volume collapsing and
.regular
What you want to see in echo? What r signs o severity
?on echo
then what else?what about complications ?IE,but he is
.not febrile and has no signs
What management? Accordingly,duretics if he present
in Hf,asked me is he in hf ? No,i couldnt appreciate any
.crackles or ll oedema
then ACEI ,examiner even with this AS,i said according
.to ECO if is it significant or not
Then surgical,aortic valve replacement most probabely
.metalic pcoz he is young
20/20
Neuro: Rt hemiparesis((upper motor neuron
lesion+cerbellar signs))
DD:(Ms or multiple strocks or spino cerbellar
degeneration)
.The instructions was examine z motor system
I started by the LL,then UL finally the face i examine for
horzintal nystagmus, facial nerve and hypoglossal
.nerve
pt has rt hemiparesis,has cerbellar signs in form of
dysdyadokinsia ,rebound phenomenon,finger nose
test,all evident on the rt upper limb plus horzintal
.nystagmus
In addition he has UMN signs in form of upgoing
planter in the Rt side , the refelexes r normal in the LL
.but increased in UL on the rt side
want to examine his gait and speech (what type of
?speech
? examiner asked
. what about the Lt side i said it was normal
.what about z tone ? hyptonia
.asked why ?due to cerbellar lesion
?What diagnosis?DD
What investigations? MRi brain looking for plaques of
? ms,Ncs (he asked what do you see
Lumbar puncture(looking for what ? Oligoclonal band
? (what is it
? What management
Pharmaclogical and non pharmaclogical
Staion 4: the senario was about an elderly lady which
had multiple strocks and recent brain
heamorrhage,known DM and ESRD on regular
heamodialysis ,now she is deteriorating ,and her
wishes was to stop the dialysis if she is getting
deteriorating,and the treating team decided to follow
.her wishes
My task was to inform her son about her wishes and
.the team decision
I started by asking the son ,is he z next of ken,does he
want anyone to attend this meeting with us,did he see
his mother recently and what does he know about her
?condition
Then i told him unfortantely her condition is
deteriorating as he told me ,and about her wishes,and
. that our team decided to respect her wishes
?Surrogate: if you stop dialysis what will happen
?S: is she going to die?and when
S: ok if so ,let me to take her home ,i will bring a nurse
?to stay with her
Me : i apprecite your feeling ,i know yr keen about your
beloved mother,but it is difficult to be managed at
home,pcoz there is substance called k ,it is going to be
we need to monitor her closely. to give her the high
....proper management,etc
?S: what about her Dm and other things
Me: i assure you ,we are going to treat her respectfuly
and with diginty, taking care about all her needs and
manage her blood sugar.only the dialysis was stopped
?S : i am afriad she is feeling pain
Me : she is not aware about her surroundings most
.probabley
?Do u want me to call any one for you
What about you? Who was taking care of your mother
?at home ? And with whome she was living
S :i am a business man,was so busy recently ,i couldnt
stay with her,i hired a nurse for her, i have no siblings
.or other family member
Me: i can understand how is difficult for you,and
. appreciat yr feelings
? Do you want us to offer any social support for you
?What is concerning u more about her
S : ok thanks dr, i jusr want to be sure that she is not
.feeling pain,and to stay with her for now
Me :your more than welcome ,if u want i can arranged
a meeting with my consultant ,and the kidney
consultant to discuss with them.and your welcome to
.visit her at any time
. Only 2 min was left

Me : did your mother has any advance directive or did


she tell you about her wishes ? Or anyone told you
?about that
S : no she didnt
.Then i summarize for him and he agreed
?Examiner asked me? What z issues in this senario
.Bbn, empathy,autonomy of z pt ,advance care of ill pt
? What z issues of her son
?How do you konw this is rt decision
Me : i trusted z senario & my team so most probabely
.they r sure that z pt was competent when she decided
?E : how do u know z pt is competent
Me : that she can understand z information ,recall
E : no there r 4 componenets of it
Me : recall and weight benifits and risks and no one
.inforce her
E :not recalling it is retaining
Me :yes that what i mean
? E :what ethical issues in it
Me : autonomy ,empathy
E : empathy isnt an ethical issues
? Me : benfecience (what is it
Malefecience (what is it )
. Finally finished
16/16
: Station 5
:1
Female 40 ,came with headache
I was totally exhuasted and it was my last station,when
.i read i suppose it was a male and i put different dd
So when i entered the room,examiner told let us start
with female pt ,i was shocked
she has headache for 2 months,no signs of ICP ,no
fever or symptoms of manangism,no trauma,no cns
.symptons,no aura,i felt i was lost .no drug history
Till asked about her period ,she have just gave birth to
.her baby 2 month ago,period stopped from that time
asked did she bled a lot,she said yes
? what happen,asked about lactation
.she couldnt lactate her baby since that time
).it is shehan syndrome(
asked about symptoms of panhypopitutrism.she is
depressed,feeling hot ,fatigue,etc
:examination
Started by hands ,checking PR,rough skin,i asked to do
bp standing and sitting.examiner told me it is written
behinde you in z wall
I asked to examine her neck,gave her water to drink for
thyroid examination
to check her for breast atrophy and examine the axilae(
for hair distribution). Examine abdomen (for straie)
. ,back for interscapular fat, examiner told me no need
.to do fundoscopy ,examiner told no need
. I forgot to do visual field
Then i answered concern, the need for urgent
admission,give iv fluides.do some imaging and blood
.tests
?Examiner asked what is d
pitutary apoplexy due to post partum haemmorrahage
.causing panhypopiturism
iv fluides ,iv What management? Urgent Admission
.steriods,thyroxin
What investigations?MRI brain for pitutary and blood
tests,etc
?What dd
.I said migrane but there was no a typical aura
.Infection but no fever
I think they r were looking for bengin ICP,and pitutary
.tumor i forget to say
28/26
:Bcc2
yrs male with facial weakness,vitals r normal 54
He has rt facial weakness for 1 week ,no other cns
symptoms ,when came to hearing problem,surrogate
told me he has rt ear vesicles 1 week before with ear
.pain
.Also he is a heavy smoker
I examined facial nerve,rough examination for hearing
,asked for torch to examine the mouth for 9th
.crn.examiner told no need
.Examine the ear for rash
.Examine arms for pronator driift
Asked to examine for upgoing planter,speech and
walking. and to do chest examination ,examiner told all
.r normal
Concern was what z d? Is it strock?does he need
.admittion
I said it facial nerve affection most probably due to
recent viral infection, no need for anti viral pcoz it is
not active now .it is unlikely to be strock because he
has no signs of cva or weakness ,but we need to refer
him to nerve dr ,do MRI brian as out pt ,to be sure
.there is no lesion in z brain,esp he is a heavy smoker
.I adviced regarding to stop smoking
We will gave him drugs called steriods,he should cover
.his eye and eat gums to move his mouth
We will give him refreshing eye drops and refer him to
.physiotherapist
?Then examiner asked: what z d
Is it strock? I said it is unlikely pcoz most probably it
.will be in the brain stem,has weakness and more ill
He said but it could be strock .i said may be
?Asked what invst
Brain MRi to be sure there is cerebellopontine lesion
.esp he is a heavy smoker
.Then basic invt
.I replied the same managment i said to surrogate

What complications ? Eye keratitis


What speciallity dr you need to ask him to see the pt a
?part of the nerologist ,ENt,physiotherapist
Opthalmologist
?What abou his speech
28/28

Maadi Military Hospital 2/6/3016


Well organized very good atmosphere
: I started my exam with cardiology
MVR with A.FIB ,valve functioning well
Q1:causes,RHD then immediately
They ask me about management as he SOB;diuretics
and anticoagulation
Then Q2 if patient had fever what he could have
??infective endocarditis and what is the Target INR2.5-
3.5
I score 20/20
Then neurology station :while am examing young man i
can hear the click he has pyramidal weakness on left
side with clonus ,and they ask d/d left sided
hemiparesis stroke in young ;then i said as i could hear
click cardiac cause A.FIB and then ask how you will
decide about Anticoagulation i said CHADS2 score
other D/D demyelination then ask how you investigate
for MS I said MRI VEP and LP
20/20
Then communication station i felt i did Bad
elderly with UTI and Parkinsonism which was
diagnosed 3 years but not on treatment ,now she is
admitted with UTI ,her parkinsonism become evident
and started on treatment carpidopa ,role to D/W her
daughter msmagement
My D/about that she is elderly fragile with uti her
symptoms appear
Then daughter ask about side effect of Parkinson drugs
Then i asked social history she told my father bed
bound with stroke and my mum is only care giver ,then
i discuss other modalities of treatment like deep brain
stimulation
Then i told we will involve social worker if no solution
then might need to think about nursing home for your
parents
Examiner ask me about treatment of Parkinsonism
Feeding i said PEG tube then i need family ,they told
me to why u need family to Discuss ;i said she might
have LPA or advance directive As she is incompetent
I felt am of point as i didn't talk much about UTI
But i score 16/16
; Then i moved to station 5
Cushing i asked what is your concern but I didn't
answer the concern cuz no time ,examiner ask me
what is your D/D cushing ,hypothyroidism ,then he ask
me about how to investigate and treat cushing and
what is the difference between cushing disease and
.syndrom
28/24
:Second st5
years old presented with polyurea 29
History of RTA three years concer could it be cancer
? Examiner ask what is your diagnosis
Diabetis insipidus also ask about investigation i told
water deprivation,desmopressin then examiner said
more simple one i said urin and serum osmolality and
.treatment
I forgot to refer both patients to speciality,may be that
is why i score less
28/25
Then Abdomen:hepatosplenomegaly with heart failure
and also she had auidble click .gum bleeding,echemotic
patch in her hands
D/D Decompensated CLD i told hepatitis C ,then he ask
management i told referral to hepatologist and ask
about latest treatment for Hep C i told bocepravir
20/18
Chest:COPD ,with basal crepirations i told with fibrosis
examiner didn't like it he want COPD only then he ask
me about non pharmacological therapy and then
.pulmonary rehabilitation
20/17
History station:35 years old with hearing difficiency
from recurrent infections with meningitis at age 17
,chest infection ;UTIs and came with dirrhoea and
weight loss ,i told examiner
hypogammaglibulibaemia,he told me what else then
HIV ,then he helped me cuz I forgot Cystic Fibrosis,till i
said cystic fibrosis so am not sure what he wanted or
how he will judge me cuz i gave only
hypogammaglibulibaemia and CF after his help
20/19

My PACES experience in Golden Jubilee Hospital,


Glasgow, UK in June 2016

:Station 1
Respi: A elderly man with obvious pectus excavatum.
However, the chest signs were subtle. I got left LZ
crepitations with reduced breath sounds, giving the
diagnosis of pectus excavatum with left LZ
bronchiectasis. Another candidate got right LZ
crepitations, the 3rd candidate got bilateral LZ
crepitations. Turned out the answer was right LZ
bronchiectasis. Lost all marks in physical signs
)20/12( .component

Abdomen: Another station with subtle clinical findings.


Stem: this man has abdominal pain; please examine
and find out why. This middle aged man has very
subtle hepatomegaly. Discussion on causes and
management. Another candidate reported
hepatosplenomegaly, and the 3rd candidate reported
normal findings. The answer was hepatomegaly, but I
missed the gynecomastia, so identifying physical signs
marks were deducted. Gave the correct DDX of
)20/18( .alcoholic liver disease

:Station 2
A middle aged lady with prolonged fever, symptoms
persisted despite admission and treatment for UTI.
Further hx revealed prolonged fever with weight loss.
She will also mention a lump in the inguinal area. DDX
given was lymphoma, occult malignancy, CTD, TB, IE.
Concern: What is causing my symptoms? Spent a lot of
time explaining diagnosis, the need for biopsy,
admissions, further tests. Need to explore how the
fever has affected her daily life and offer
)20/19(.solutions

:Station 3
Neurology: Stem: this lady complained of double
vision. Please examine her. A case of Myasthenia gravis
with thymectomy. The only sign was double vision with
fatiguability and thymectomy scar. Questions were
)20/20( .standard
CVS: An elderly man with central sternotomy scar, vein
harvest scar, and MR. Got panicked and gave the
)20/10( .wrong diagnosis of AS. Did badly overall

:Station 4
A elderly man was admitted for pneumonia with
confusion. Given amoxicillin in ward and developed
anapylaxis. He recovered but still remained confused.
Talk to the daughter and address her concern. Need to
elicit the fact that the daughter mentioned to a doctor
regarding patient's allergy to penicillin. Thus, this is a
case of error of drug administration. Need to apologize
profusely. Lodge critical incident reporting. Need to
address her concern and reassure her in every way this
will not happen again, and provide her the example
how you intend to avoid this from happening again.
She will have a lot of concerns and anger and you need
to apologize, reassure, offer solutions and answers to
her concern. I didnt mention about PALS as she never
mentioned lodging a complaint but if she did, offer her
)16/16(.ways to lodge a complain

:Station 5
BCC1: A elderly lady with dark pigmentations over her
shins. Further hx: long standing DM on OHA, long
standing pigmentation for years, not causing
symptoms apart from itchiness. It is a case of
necrobiosis lipoidica diabeticorum (most likely healed
lesions). Given differentials of chronic venous
.insufficiency with stasis eczema, diabetic dermopathy
)28/28(

BCC2: A case of a young man with headache. A


challenging station as there is a lot to get from history
and to examine, and all need to be done within 8
minutes. Further history revealed symptoms of
headache worse in morning and with sneezing,
vomiting and blurring of vision. Examinations were
normal. Didn't perform fundoscopy but did mention it.
Concern: Is it brain tumor? My mom had brain tumor
at age of 40. DDX: headache due to raised ICP, e.g. IIH,
.less likely SOL, migraine. Mx: Offer urgent CT brain
)28/25(

Overall: 148/172 (PASS)

:Personal opinion

Exam case in UK are generally fair. It has tendency to


put up cases with subtle clinical findings esp. BCC.
Normal surrogates are frequently used in BCC, with
scenarios like headache, syncope, fever etc being not
.uncommon
The examiners were rather strict and particular about
identifying correct physical signs. This is the
component that scared me the most. This applies to
PACES everywhere

My PACES experience in Golden Jubilee Hospital,


Glasgow, UK in June 2016

:Station 1
Respi: A elderly man with obvious pectus excavatum.
However, the chest signs were subtle. I got left LZ
crepitations with reduced breath sounds, giving the
diagnosis of pectus excavatum with left LZ
bronchiectasis. Another candidate got right LZ
crepitations, the 3rd candidate got bilateral LZ
crepitations. Turned out the answer was right LZ
bronchiectasis. Lost all marks in physical signs
)20/12( .component
Abdomen: Another station with subtle clinical findings.
Stem: this man has abdominal pain; please examine
and find out why. This middle aged man has very
subtle hepatomegaly. Discussion on causes and
management. Another candidate reported
hepatosplenomegaly, and the 3rd candidate reported
normal findings. The answer was hepatomegaly, but I
missed the gynecomastia, so identifying physical signs
marks were deducted. Gave the correct DDX of
)20/18( .alcoholic liver disease

:Station 2
A middle aged lady with prolonged fever, symptoms
persisted despite admission and treatment for UTI.
Further hx revealed prolonged fever with weight loss.
She will also mention a lump in the inguinal area. DDX
given was lymphoma, occult malignancy, CTD, TB, IE.
Concern: What is causing my symptoms? Spent a lot of
time explaining diagnosis, the need for biopsy,
admissions, further tests. Need to explore how the
fever has affected her daily life and offer
)20/19(.solutions

:Station 3
Neurology: Stem: this lady complained of double
vision. Please examine her. A case of Myasthenia gravis
with thymectomy. The only sign was double vision with
fatiguability and thymectomy scar. Questions were
)20/20( .standard

CVS: An elderly man with central sternotomy scar, vein


harvest scar, and MR. Got panicked and gave the
)20/10( .wrong diagnosis of AS. Did badly overall

:Station 4
A elderly man was admitted for pneumonia with
confusion. Given amoxicillin in ward and developed
anapylaxis. He recovered but still remained confused.
Talk to the daughter and address her concern. Need to
elicit the fact that the daughter mentioned to a doctor
regarding patient's allergy to penicillin. Thus, this is a
case of error of drug administration. Need to apologize
profusely. Lodge critical incident reporting. Need to
address her concern and reassure her in every way this
will not happen again, and provide her the example
how you intend to avoid this from happening again.
She will have a lot of concerns and anger and you need
to apologize, reassure, offer solutions and answers to
her concern. I didnt mention about PALS as she never
mentioned lodging a complaint but if she did, offer her
)16/16(.ways to lodge a complain

:Station 5
BCC1: A elderly lady with dark pigmentations over her
shins. Further hx: long standing DM on OHA, long
standing pigmentation for years, not causing
symptoms apart from itchiness. It is a case of
necrobiosis lipoidica diabeticorum (most likely healed
lesions). Given differentials of chronic venous
.insufficiency with stasis eczema, diabetic dermopathy
)28/28(

BCC2: A case of a young man with headache. A


challenging station as there is a lot to get from history
and to examine, and all need to be done within 8
minutes. Further history revealed symptoms of
headache worse in morning and with sneezing,
vomiting and blurring of vision. Examinations were
normal. Didn't perform fundoscopy but did mention it.
Concern: Is it brain tumor? My mom had brain tumor
at age of 40. DDX: headache due to raised ICP, e.g. IIH,
.less likely SOL, migraine. Mx: Offer urgent CT brain
)28/25(

Overall: 148/172 (PASS)

:Personal opinion

Exam case in UK are generally fair. It has tendency to


put up cases with subtle clinical findings esp. BCC.
Normal surrogates are frequently used in BCC, with
scenarios like headache, syncope, fever etc being not
.uncommon

The examiners were rather strict and particular about


identifying correct physical signs. This is the
component that scared me the most. This applies to
PACES everywhere and a lot of practice is required to
be able to pick up subt

.le signs. Never create signs as this is really fatal

Station 4 is very unpredictable. Cases can be easy or


complex with multiple agendas. Suggest to review all
the cases posted up here previously and practise them.
Need to have some knowledge regarding DVLA, Mx of
meningococcal ds and prophylaxis etc... Need to really
elicit the concerns, and offer solutions/answer as much
.as you can

.Good luck and all the best


Station 1
Abdomen: Lady around 50y.o with cushingoid features,
Perma cath, scar on the Right iliac fossa ( failed renal
transplant) and multiple scars around the umbilicus (
previous Peritoneal dialysis)
The examiner asked about the complications ( esp.
bone complications and he asked about dietary
restriction {Shappati} as the pt and examiners are
Indian)

Respiratory: Male pt around 55y.o well- nourished with


right thoracotomy scar on the back+ end-insp crackles.
No clubbing, no cyanosis, no signs of pulmonary HTN
Dx ILD, the scar is for lung biopsy ( I said to the
examiner it's for lobectomy but he asked me what else
it could be for, I said for lung biopsy then he agree with
me)

Station 2
Lady aged 55y.o heavy smoker with 3months h/o SOB,
coughing blood and loss of weight. She sought medical
advice recently and given antibiotic ( she doesn't know
the name of it) by GP who diagnosed her as acute
bronchitis, but no improvement. One week ago she
developed dysphagia for solid food. No h/o fever, no
.vasculitis symptoms, no other GI symptoms
Station3
Cardio: young lady with mid-sternotomy scar and
palmar erythema. No signs of pericarditis. S1 is metalic.
No murmurs or additional heart sounds. No signs of
pulm HTN or pulm cngestion
Dx Mitral valve replacement ( metalic)

Neuro: instruction: examine lower limbs

old man with walking aids beside him, indwelling


Foley's cath. Perioheral neuropathy for DD. I
mensioned them specifically paraneoplastic syndrome
( ? Prostatic cancer)

Station 4
Middle age lady diagnosed to have bird fancier lung
disease. She presented today to know the result ( BBN)
and to discuss with her the need for corticosteroid
treatment and to avoid exposure to pigeon ( she's
breeding pigeon and she's famous in her region )
She resisted first to take the steroid but when I
explained to her its benefits and risks ( including
osteoporosis) and the prophylaxis for the side effects
she accepted. Also she got angry when I suggested to
her to avoid exposure to pigeon.. I appreciated her
upset and I explained that she will not get better
unless she avoids exposure. I suggested to wear mask
in case she has to see her pigeon or to train somebody
to feed them. She said her son may help her in taking
.care of the pigeon finally agreed

Station5
Case 1
y.o. Lady presents with fever (39.5) and diarrhea. 25
She admitted eating from restaurant. When I asked
about travel she said she came from Thailand. I asked
about insect bite including mosquitos she said yes.
Then I asked about malaria prophylaxis before during
and after travel she said yes. I also asked about HIV
.risks
O/E : no signs (surrogate)

Case 2
y.o male with headache, high blood pressure 30
(180/100) and urine dipstick showing proteinuria and
microscopic hematuria. He had h/o childhood chest
.infection and family h/o SLE
O/E no signs
There is ophthalmoscope on the table. I noticed it late.
"): I said " I would like to do fundoscopy but no time
Dx AkI ( Glomerulonephritis needed kidney biopsy and
Autoimmune profile+ Renal US)

UK EXPERIENCE

I started with station 5


young female recurrent abdominal pain. Refered .1
.from surgerical team.scan normal
On history she had rash,mild headache and some joints
.pain
.I made differential of vasculitis and porphyria
.Examiner ask about investigation of porphyria
young gentle man with collapse. History goes with . 2
seizures . grossly no neurological deficit . I explained
about diagnosis, ask about driving

.My experience at whipps cross hospital 31/8/2016


Started with station 5
young female referred from surgical department due .1
.to recurrent abdominal pain
.History was negative, no diarrhoea, no loss of weight
.No relieving or aggravating factors
Systemic review showed rash at forearm, mild
headache and some joints pain. No weight loss
Periods normal
Examination; no jaundice, abdomen soft nontender
and no viscromagely
;Concerns
what is the cause.1
.why ultrasound normal.2
.I explained likely vasculitis or porphyria
. Needs other blood and urine test to confirm
Examiner asked about differential i said as above and
the next question was investigation of porphyria
years old university student with collapse. I 2.25
started what happened he told he passed out while
.watching movie
I ask if happen before, Pt told 3 weeks ago while he
was working on computer in library. I started with
prodormal symptoms, they were none.i ask any friends
.observed jerky movements, Pt told yes
Than history goes on with incontinence and fatigue
.after recovery
I ask about any thing unusual a night before (lack of
sleep ),Pt told no . then asked about driving, drugs, and
hobbies (keen swimmer).grossly examine tone power
in both limbs,gait and ask for fundus. (Examiner
.refuse)
Concerns 1.what is my problem
.what you will do (scan +eeg).2
Consouil about driving and any attendant while he
.swims
Examiner ask! What will be finding in ct? I told him
likely to be normal as there is no neurological deficit
but would like to have com

.Complete neurological examination


Is it possible to have any cardiac problem to this
.patient
I explained possible but less likely as both events occur
while Pt was sitting, however tacyarrthmias can be
.possible
Would you start treatment. I said refereed to seizure
clinic and neurologists will decide

Abdomen# young female with central larotomy


scar,subclavian

Dialysis catheter and right palpable kidney. Not sure


about larotomy scar (which was the main question by
examiner),other question was about causes of fatigue
in this patient I told him uremia, possibility of
underlying hypothyroidism, anemia and infection.
Overall not very good
Respiratory # young female, no rheumatological
manifestations, wheezing from bedside. Minimal basal
.crepetations
Indian examiner started with respiratory rate
(forget)
Next question was jvp findings (),followed by did this
.patient had loud P2 ()
I said sorry for above 3 questions
.Than he ask differential i told him copd /fibrosis
He ask which will be your priority diagnosis, I told copd
due to prominent wheezing than investigation of copd
with xray findings and pulmonary function test. Overall
it was tough

History ##50 years old women complaints of


.abdominal discomfort and bloating
I started with usual pattern of pain,location, bowel
changes, all none. Nonspecific pain not related to any
thing . half stone weight loss. Than I asked any tummy
distension, she said yes her trouser are tighter and she
is using large size from before. I switch to orthopnea,
pnd, negative. No lower leg swelling no periorbital
swelling no problems with water. No signs of liver
disease. Clueless I proceed to past history which was
significant for mastectomy secondary to malignancy.
Family history positive for ca breast in sister . mild low
feeling due to recent mother died because of ca
breast.post menopausal (no dysparunia/break through
.bleeding)
Concern 1. What is cause of tummy distension. I
explained likely that some tumour cell spread .2. Is it
too late as I have symptoms since 3 months. I told her
we have to investigate and don't worry we will do your
test on priority
.Examiner ask# diagnosis i told him metastasis
He ask if Pt don't have distension than what do you
think. I told I consider irritable bowel as recent death
.of her mother and only half stone of weight loss
What other possibilty I told ca ovaries. Then tumour
markers of ca ovary. What do you do? Scan ct . any
investigation would you like to offer while she was in
.opd. I don't have any answer. He told chest xray
What measures you told to other sisters and
daughters. I told repeated manual breast examination
and after 40 years of age mammogram. Got full marks

Cvs # 75 years old male with sob . murmer of AR. I


.checked collapsing pulse
.Routine questions about causes
Causes of acute AR (dissection of aorta, endocarditis
and ruptured sinus of valsulva)
Type of valve

Cns# 50 years old gentle man with difficulty in walking


please examine upper limbs

It was parkinsonism. I mentioned to check sitting and


standing BP, micographia and gaze palsy

.Examiner ask about causes


Treatment
New treatment, mention deep brain stimulation and
.dopamine containg implants
Who will be involve in management of this patient
.#MDT
She asks what occupational therapist will do

I told occupational therapist will visit the home and


arrange some rails and support to prevent patient from
.falling
Alhamdillah went well. Got 19 in both
Communication # spoke to wife, husband in icu. Keen
cycle rider and went for long marathon and take extra
fluids to prevent dehydration. At home he also drink
water continously till he was found to seize in garden
and brought by neighbours. CT and all other labs
.normal. Sodium 114
Better but still confused with gcs 15 . two weeks ago
started on bendrafluthiazide for htn (Pt age 45)

I started with wife with sympathy, what she Knows so


.far regarding husband
Gave good news that scan is normal. Likely seizure due
.to low salt in body
She asks why salt become low. I explained . she asks
why still confused I told her take time to correct
sodium slowly . she asks about discharge.,explains it
.will take coupleof days
She asked they are moving to dubai, so he can do cycle
ride there. Its will happen again like little puzzle
with this question but told her that chances are low
but instead of taking plane water if he took carbonated
.!!water it contains some salt
She asked about BP medication attributing. I told
possible. She asks continue bendrafluthiazide. I told we
.ask cardiology colleges
Came back to driving and profession .Pt was enginer
.but not exposed to heavy machine. I told dvla
She asks follow up for how long as they are moving to
dubai. I told we don't need long term follow up as
prognosis is good and we're will gave detail medical
.report to be shown to doctors in dubai

Last concern where he will ride cycle in dubai as it is


very hot there

I just mention i am not sure But in dubai you may find


indoor cycling track as most of the activities there are
indoor even ski

Examiner ask why Pt confused I told still sodium is not


correct. He ask other reason I told him possible
.cerebral edema due to seizure and low sodium
He ask at what rate you will correct sodium. I told 5 -8
meq/day . then he ask what happens with rapid
correction. I answered. He ask at what sodium level
you are happy to discharge. I told him 135 -140. He ask
.what about cycle riding rules after seizures in uk
I told him I have no idea, but advisable not to do in
early few months . last question is bendrafluthiazide
was a good choice of anti hypertension for this patient.
I told no as patient ids less than 55 an ACEI should be
considered. Alhamdillah Got full marks

Overall experience of exam in uk was good . there is no


problem of understanding of English with surrogate in
.station 2 and 4
..Thanks for this group
Best of luck and good wishes to all my friends in this
group

S-ar putea să vă placă și